SOLUTIONS MANUAL
SOLUTIONS MANUAL
FUNDAMENTALS OF CORPORATE FINANCE 4TH EDITION BY ROBERT PARRINO, HUE HWA AU YONG, NIGEL MORKEL-KINGSBUSY, JENIFFER JAMES, JAMES MURRAY ( CHAPTER 1_21) SOLUTIONS MANUAL Chapter 1 The Financial Manager and the Firm
Before You Go On Questions and Answers Section 1.1 1. What are the three most basic types of financial decisions managers must make? The three most basic decisions each business must make are the capital budgeting decision, the financing decision, and the working capital management decision. These decisions determine which productive assets to buy, how to pay for or finance these purchases, and how to manage the day-to-day financial matters so the company can pay its bills.
2. Explain why you would make an investment if the value of the expected cash flows exceeds the cost of the project. You would accept an investment project whose cash flows exceed the cost of the project because such projects will increase the value of the firm, making the owners wealthier. Most people start a business to increase their wealth.
3.
Why are capital budgeting decisions among the most important decisions in the life of a firm? The capital budgeting decisions are considered the most important in the life of the firm because these decisions determine which productive assets the firm purchases and these assets generate most of the firm’s cash flows. Furthermore, capital decisions are long-term decisions and if you make a mistake in selecting a productive asset, you are stuck with the decision for a long time.
Section 1.2 SM 19-1
1. Why are many businesses operated as sole proprietorships or partnerships?
SM 19-2
Many businesses elect to operate as sole proprietorships or partnerships because of the small operating scale and capital base of their firms. Both of these forms of business organization are fairly easy to start and impose few regulations on the owners.
2. What are some advantages and disadvantages of operating as a public corporation? The main advantages of operating as a public corporation are the access to the public securities markets, which makes it easier to raise large amounts of capital, and the ease of ownership transfer. All the shareholders have to do is to call their broker to buy or sell shares of stock. And because a public corporation usually has many shares outstanding, large blocks of securities can be purchased or sold without an appreciable impact on the price of the stock. The major disadvantage of corporations is the tax situation. Not only must the corporation pay taxes on its income, but the owners of the corporation get taxed again when dividends are paid to them. This is referred to as double taxation. 3. Explain why professional partnerships such as physicians’ groups organize as limited liability partnerships. Professional partnerships such as physicians’ groups desire to organize as limited liability partnerships (LLPs) to take advantage of the tax arrangements of partnerships combined with the advantages of the limited liability of a corporation. By operating as an LLP, the partnership is able to avoid a potential financial disaster resulting from the misconduct of one partner.
Section 1.3 1. What are the major responsibilities of the CFO? The major responsibilities of a CFO are recommendation and financial analysis of financial decisions. Although all top managers in a firm participate in these decisions, the final report and analysis is ultimately the responsibility of the CFO.
2. Identify three financial officers who typically report to the CFO and describe their duties.
SM 19-3
The financial officers discussed in the chapter who report to the CFO are the controller, the treasurer, and the internal auditor. The controller is the firm’s chief accounting officer, and thus prepares the financial statements and taxes. This position also requires close cooperation with the external auditors. The treasurer’s responsibility is the collection and disbursement of cash, investing excess cash, raising new capital, handling foreign exchange, and overseeing the company’s pension fund management. He also assists the CFO in handling important Wall Street relationships. Finally, the internal auditor is responsible for conducting risk assessment and for performing audits of high-risk areas.
3. Why does the internal auditor report to both the CFO and the board of directors? The internal auditor reports to the CFO on a day-to-day basis but is ultimately accountable for reporting any accounting irregularities to the board of directors. The dual reporting system serves as a check to ensure that there are no discrepancies in the company’s financial statements.
Section 1.4 1. Why is profit maximization an unsatisfactory goal for managing a firm? Profit maximization is not a satisfactory goal when managing a firm because it is rather difficult to define profits since accountants can apply and interpret the same accounting principles differently. Also, profit maximization does not define the size, the uncertainty, and the timing of cash flows; it ignores the time value of money concept. 2. Explain why maximizing the current market price of a firm’s stock is an appropriate goal for the firm’s management. Maximizing the current market price of a firm’s stock is an appropriate goal for the firm’s management because it is an unambiguous objective and it is easy to measure. One can simply look at the value of the company’s stock on any given day to determine whether it went up or down. 3. What is the fundamental determinant of an asset’s value?
SM 19-4
The fundamental determinant of an asset’s value is the future cash flow the asset is expected to generate. Other factors that may help determine the price of an asset are internal decisions, such as the company’s expansion strategy, as well as external stimulants, such as the state of the economy.
Section 1.5 1. What are agency conflicts? An agency conflict occurs when the goals of the principals are not aligned with the goals of the agents. Management is often more concerned with pursuing its own self-interest, and so the maximization of shareholder value is pushed to the side.
2. What are corporate raiders? Corporate raiders can make the economy more efficient by keeping the top managers on their toes. Top managers know that if the company’s performance declines and its stock slips, it makes itself vulnerable to takeovers by corporate raiders who are just waiting to temporarily acquire a company, turn it around, and sell it for profit. Therefore, the role of the corporate raiders in the economy is twofold: first, the fear of takeovers pushes managers to do a better job, and second, if the managers are not performing up to expectations, the company can be rescued and restructured into a contributor again.
3. List the three main objectives of the Sarbanes-Oxley Act. The three main goals of the Sarbanes-Oxley Act are to reduce agency costs in corporations, to restore ethical conduct in the business sector, and to improve the integrity of accounting reporting systems within firms.
Section 1.6 1. What is a conflict of interest in a business setting? Conflict of interest in the business setting refers to a conflict between a person’s personal or institutional gain and the obligation to serve the interest of another party. For example, the chapter discussed the problem that arises when the real estate agent helping you buy a house is also the listing agent. SM 19-5
2. How would you define an ethical business culture? An ethical business culture means that people have a set of principles, or moral values, that helps them identify moral issues and then make ethical judgments without being told what to do.
Self-Study Problems and Solutions 1.1
Give an example of a capital budgeting decision and a financing decision.
Solution:
Capital budgeting involves deciding which productive assets the firm invests in, such as buying a new plant or investing in the renovation of an existing facility. Financing decisions determine how a firm will raise capital. Examples of financing decisions include the decision to borrow from a bank or issue debt in the public capital markets.
LO: 1 Level: Basic
1.2
What is the appropriate decision criterion for financial managers to use when selecting a capital project?
Solution:
Financial managers should select a capital project only if the value of the project’s expected future cash flows exceeds the cost of the project. In other words, managers should only make investments that will increase firm value, and thus increase the stockholders’ wealth.
LO: 1 Level: Basic
1.3
What are some of the things that managers do to manage a firm’s working capital?
Solution:
Working capital management is the day-to-day management of a firm’s shortterm assets and liabilities. Working capital can be managed by maintaining the optimal level of inventory, managing receivables and payables, deciding to whom SM 19-6
the firm should extend credit, and making appropriate investments with excess cash. LO: 1 Level: Basic
1.4
Which one of the following characteristics does not pertain to corporations? a.
Can enter into contracts
b.
Can borrow money
c.
Are the easiest type of business to form
d.
Can be sued
e.
Can own stock in other companies
Solution:
The answer that does not pertain to corporations is: c. Are the easiest type of business to form.
LO: 2 Level: Basic
1.5
What are typically the main components of an executive compensation package?
Solution:
The three main components of a typical executive compensation package are: base salary, bonus based on accounting performance, and compensation tied to the firm’s stock price.
LO: 5 Level: Basic
Discussion Questions and Answers 1.1
Describe the cash flows between a firm and its stakeholders. Cash flows are generated by a firm’s productive assets that were purchased through either issuing debt or raising equity. These assets generate revenues through the sale of goods SM 19-7
and services. A portion of this revenue is then used to pay wages and salaries to employees, pay suppliers, pay taxes, and pay interest on the borrowed money. The leftover money, residual cash, is then either reinvested back in the business or is paid out to stockholders in the form of dividends. LO: 1 Level: Basic Bloomcode: Comprehension AACSB: Analytic IMA: Corporate Finance AICPA: Industry/Sector Perspective
1.2
What are the three fundamental decisions the financial manager is concerned with, and how do they affect the firm’s balance sheet? The primary financial management decisions every company faces are capital budgeting decisions, financing decisions, and working capital management decisions. Capital budgeting addresses the question of which productive assets to buy; thus, it affects the asset side of the balance sheet. Financing decisions focus on raising the money the firm needs to buy productive assets. This is typically accomplished by selling long-term debt and equity. Finally, working capital decisions involve how firms manage their current assets and liabilities. The focus here is seeing that a firm has enough money to pay its bills and that any spare money is invested to earn a return.
LO: 1 Level: Basic Bloomcode: Comprehension AACSB: Analytic IMA: Decision Analysis AICPA: Decision Modeling
1.3
What is the difference between stockholders and stakeholders? Stockholders, also referred to as shareholders, are the owners of the company. A stakeholder, on the other hand, is anyone with a claim on the assets of the firm, including, SM 19-8
but not limited to, shareholders. Stakeholders include the firm’s employees, suppliers, creditors, and the government. LO: 1 Level: Basic Bloomcode: Analysis AACSB: Analytic IMA: Corporate Finance AICPA: Industry/Sector Perspective
1.4
Suppose that a group of accountants wants to start an accounting business. What organizational form would they most likely choose, and why? Most lawyers, accountants, and doctors form what are known as limited liability partnerships. This formation combines the tax advantages of partnerships with the limited liability of corporations.
LO: 2 Level: Basic Bloomcode: Application AACSB: Analysis IMA: Business Economics AICPA: Industry/Sector Perspective
1.5
Why would the owners of a business choose to form a corporation even though they will face double taxation? Because the benefits, such as limited liability and access to large amounts of capital at relatively low cost in the public markets, outweigh the cost of double taxation (as well as the higher costs associated with forming a corporation).
LO: 2 Level: Basic Bloomcode: Comprehension AACSB: Analytic SM 19-9
IMA: Business Economics AICPA: Industry/Sector Perspective
1.6
Explain why profit maximization is not the best goal for a company. What is a better goal? Although profit maximization appears to be the logical goal for any company, it has many drawbacks. First, profit can be defined in a number of different ways, and variations in net income for similar firms can vary widely. Second, accounting profits do not exactly equal cash flows. Third, profit maximization does not account for timing and ignores risk associated with cash flows. An appropriate goal for financial managers who do not have these objections is to maximize the value of the firm’s current stock price. In order to achieve this goal, management must make financial decisions so that the total value of cash inflows exceeds the total value of cash outflows.
LO: 4 Level: Intermediate Bloomcode: Comprehension AACSB: Analytic IMA: Performance Measurement AICPA: Strategic/Critical Thinking
1.7
What are some of the major external and internal factors that affect a firm’s stock price? What is the difference between the two general types of factors? External factors that affect the firm’s stock price are: (1) economic shocks, such as natural disasters or wars, (2) the state of the economy, such as the level of interest rates, and (3) the business environment, such as taxes or regulations. On one hand, external factors are variables over which the management has no control. On the other hand, internal factors that affect the stock price can be controlled by management to some degree, because they are firm specific, such as financial management decisions, product quality and cost, and the line of business, the management has selected to enter. Finally, perhaps the most important internal variable that determines the stock price is the expected cash flow stream: its magnitude, timing, and riskiness. SM 19-10
LO: 4 Level: Basic Bloomcode: Comprehension AACSB: Analytic IMA: Strategic Marketing AICPA: Measurement
1.8
Identify the sources of agency costs. What are some ways these costs can be controlled in a company? Agency costs are the costs that result from conflicts of interest between the agent and the principal. They can either be direct, such as lavish dinners or trips, or indirect, which are usually missed investment opportunities. A company can control these costs by tying management compensation to company’s performance and by establishing an independent board of directors. Outside factors that contribute to the minimization of agency costs are the threat of corporate raiders that can take over a company not performing up to expectations and the competitive nature of the managerial labor market.
LO: 5 Level: Basic Bloomcode: Comprehension AACSB: Analytic IMA: Business Economics AICPA: Industry/Sector Perspective
1.9
What is the Sarbanes-Oxley Act, and what does it focus on? Why does it focus in these areas? The Sarbanes-Oxley Act is an act of Congress that was passed in 2002. This act was passed in the aftermath of several corporate scandals that occurred at the turn of the century. The act focuses on (1) reducing agency costs in corporations, (2) restoring ethical conduct within the business sector, and (3) improving the integrity of accounting reporting system within firms. Failures in these areas led to the corporate scandals that preceded passage of Sarbanes-Oxley.
LO: 5 SM 19-11
Level: Basic Bloomcode: Knowledge AACSB: Analytic IMA: FSA AICPA: Legal/Regulatory Perspective
1.10
Give an example of a conflict of interest in a business setting, other than the one involving the real estate agent discussed in the chapter text. For example, imagine a situation in which you are a financial officer at a growing software company and your firm has decided to hire outside consultants to formulate a global expansion strategy. Coincidentally, your wife works for one of the major consulting firms that your company is considering hiring. In this scenario, you have a conflict of interest, because instinctively, you might be inclined to give the business to your wife’s firm, because it will benefit your family’s financial situation if she lands the contract, regardless of whether or not it makes the best sense for your firm.
LO: 6 Level: Intermediate Bloomcode: Application AACSB: Analytic IMA: Business Economics AICPA: Industry/Sector Perspective
Questions and Problems and their Solutions BASIC 1.1
Capital: What are the two basic sources of funds for all businesses?
Solution: The two basic sources of funds for all businesses are debt and equity. LO:
1.1 SM 19-12
Bloomcode: Knowledge AACSB: Analytic IMA: Investment Decisions; Corporate Finance AICPA: Resource Management
1.2
Management role: What is net working capital?
Solution: Net working capital is the difference between a firm’s total current assets and its total current liabilities. LO:
1.1
Bloomcode: Knowledge AACSB: Analytic IMA: Corporate Finance AICPA: Resource Management
1.3
Cash flows: Explain the difference between profitable and unprofitable firms.
Solution: A profitable firm is able to generate enough cash flows from productive assets to cover its operating expenses, taxes, and payments to creditors. Unprofitable firms fail to do this, and therefore they may be forced to declare bankruptcy. LO:
1.1
Bloomcode: Analysis AACSB: Analytic IMA: Cost Management AICPA: Measurement
1.4
Management role: What three major decisions are of most concern to financial managers?
Solution: Financial managers are most concerned with the capital budgeting decisions, the financing decisions, and the working capital management decisions. LO:
1.1 SM 19-13
Bloomcode: Comprehension AACSB: Analytic IMA: FSA AICPA: Resource Management
1.5
Cash flows: What is the appropriate decision rule for a firm considering undertaking a capital project? Give a real-life example.
Solution: A firm should undertake a capital project only if the value of its future cash flows exceeds the cost of the project. For example, a financial manager would not invest $10,000,000 in a new production line if the future cash flows from that line are expected to produce only $9,000,000 in future cash flows. That would be like throwing $1,000,000 away. LO:
1.1
Bloomcode: Application AACSB: Analytic IMA: Decision Analysis AICPA: Problem Solving and Decision Making
1.6
Management role: What is a firm’s capital structure, and why is it important?
Solution: Capital structure shows how a company is financed; it is the mix of debt and equity on the liability side of the balance sheet. It is important as it affects the risk and the value of the company. In general, companies with higher debt-to-equity proportions are riskier because debt comes with legal obligations to pay periodic payments to creditors and to repay the principal at the end. LO:
1.1
Bloomcode: Comprehension AACSB: Analytic IMA: Corporate Finance AICPA: Industry/Sector Perspective 1.7
Management role: What are some of the working capital decisions that a financial manager faces?
SM 19-14
Solution: The financial manager must make working capital decisions regarding the level of inventory to hold, the terms of granting credit (account receivables), and the firm’s policy on paying accounts payable. LO:
1.1
Bloomcode: Comprehension AACSB: Analytic IMA: Decision Analysis AICPA: Strategic/Critical Thinking
1.8
Organizational form: What are the common forms of business organization discussed in this chapter?
Solution: The common forms of business organization discussed are sole proprietorship, partnership, corporation, and limited liability company and partnerships. LO:
1.2
Bloomcode: Knowledge AACSB: Reflective Thinking IMA: Business Economics AICPA: Industry/Sector Perspective
1.9
Organizational form: What are the advantages and disadvantages of a sole proprietorship?
Solution: Advantages:
It is the easiest business organization to start.
It is the least regulated.
Owners keep all the profits and do not have to share the decision-making authority with anyone.
All income is taxed as personal income, which is usually in a lower tax bracket than corporate income.
Disadvantages:
The proprietor has an unlimited liability for all business debt and financial obligations of the firm. SM 19-15
The amount of capital that can be invested in the firm is limited by the proprietor’s wealth.
LO:
It is difficult to transfer ownership (requires sale of the business).
1.2
Bloomcode: Comprehension AACSB: Analytic IMA: Business Economics AICPA: Industry/Sector Perspective
1.10
Organizational form: What is a partnership, and what is the biggest disadvantage of this form of business organization? How can this disadvantage be avoided?
Solution: A partnership consists of two or more owners legally joined together to manage a business. The major disadvantage to partnerships is that all partners have unlimited liability for the organization’s debts and legal obligations no matter what stake they have in the business. One way to avoid this is to form a limited partnership in which only general partners have unlimited liability and limited partners are only responsible for business obligations up to the amount of capital they have invested to the partnership. LO:
1.2
Bloomcode: Comprehension AACSB: Analytic IMA: Business Economics AICPA: Industry/Sector Perspective
1.11
Organizational form: Who are the owners in a corporation, and how is their ownership represented?
Solution: The owners of a corporation are its stockholders or shareholders, and the evidence of their ownership is represented by shares of common stock. Other types of ownership do exist and include preferred stock. LO:
1.2
Bloomcode: Comprehension AACSB: Analytic SM 19-16
IMA: Business Economics AICPA: Industry/Sector Perspective 1.12
Organizational form: Explain what is meant by stockholders’ limited liability.
Solution: Limited Liability for a stockholder means that the stockholder’s legal liability extends only to the capital contributed or the amount invested. LO:
1.2
Bloomcode: Comprehension AACSB: Analytic IMA: Business Economics AICPA: Strategic/Critical Thinking
1.13
Organizational form: What is double taxation?
Solution: Double taxation occurs when earnings are taxed twice. The owners of a C-corporation are subject to double taxation—first at the corporate level when the firm’s earnings are taxed and then again at a personal level when they receive the dividends. LO:
1.2
Bloomcode: Comprehension AACSB: Analytic IMA: Business Economics AICPA: Industry/Sector Perspective
1.14
Organizational form: What is the form of business organization taken by most large companies and why?
Solution: Most large companies prefer to operate as public corporations because large amounts of capital can be raised in public markets at a relatively low cost. LO:
1.2
Bloomcode: Comprehension AACSB: Analytic IMA: Business Economics AICPA: Industry/Sector Perspective
SM 19-17
1.15
Finance function: What is the primary responsibility of the board of directors in a corporation?
Solution: The board of directors of a corporation is responsible for serving the interests of stockholders in managing the corporation. It is possible that the interest of managers may deviate from those of the stockholders. The board’s objective is to monitor and correct any management decisions that might not be in the best interest of the stockholders. For example, board duties include hiring and firing the CEO, setting CEO pay, and monitoring the investment decisions of managers. LO:
1.3
Bloomcode: Comprehension AACSB: Analytic IMA: Business Economics AICPA: Resource Management
1.16
Finance function: All public companies must hire a certified public accounting firm to perform an independent audit of their financial statements. What exactly does the term audit mean?
Solution: An independent CPA firm that performs an audit of a firm ensures that the financial numbers are reasonably accurate, that accounting principles have been adhered to year after year and not in a manner that distorts the firm’s performance, and that the accounting principles used are in accordance with generally accepted accounting principles (GAAP). LO:
1.3
Bloomcode: Comprehension AACSB: Analytic IMA: FSA; Reporting AICPA: Risk Analysis; Reporting 1.17 Firm’s goal: What are some of the drawbacks to setting profit maximization as the main goal of a company? Solution:
It is difficult to determine what is meant by profits.
SM 19-18
It does not address the size and timing of cash flows—it does not account for the time value of money.
LO:
It ignores the uncertainty or risk of cash flows.
1.4
Bloomcode: Comprehension AACSB: Analytic AICPA: Strategic/Critical Thinking IMA: Corporate Finance
1.18
Firm’s goal: What is the appropriate goal of financial managers? How do managers’ decisions affect how successful the firm is in achieving this goal?
Solution: The appropriate goal of financial managers should be to maximize the current value of the firm’s stock price. Managers’ decisions affect the stock price in many ways as the value of the stock is determined by the future cash flows the firm can generate. Managers can affect the cash flows by, for example, selecting what products or services to produce, what type of assets to purchase, or what advertising campaign to undertake. LO:
1.4
Bloomcode: Comprehension AACSB: Analytic AICPA: Strategic/Critical Thinking IMA: Decision Analysis
1.19
Firm’s goal: What are the major factors that affect a firm’s stock price?
Solution: The factors affecting the stock price include: the characteristics of the firm, the economy, economic shocks, the business environment, expected cash flows, and current market conditions. LO:
1.4
Bloomcode: Comprehension AACSB: Analytic AICPA: Strategic/Critical Thinking IMA: Corporate Finance SM 19-19
1.20
Agency conflicts: What is an agency relationship, and what is an agency conflict? How can agency conflicts be reduced in a corporation?
Solution: Agency relationships develop when a principal hires an agent to perform some service or represent the firm. An agency conflict arises when the agent’s interests and behaviors are at odds with those of the principal. Agency conflicts can be reduced through the following three mechanisms: management compensation, control of the firm, and the board of directors. LO:
1.5
Bloomcode: Comprehension AACSB: Analytic IMA: Business Economics AICPA: Industry/Sector Perspective
1.21
Firm’s goal: What can happen if a firm is poorly managed and its stock price falls substantially below its maximum potential price?
Solution: If the stock price falls below its maximum potential price, it attracts corporate raiders, who look for fundamentally sound but poorly managed companies they can buy, turn around, and sell for a handsome profit. LO:
1.4
Bloomcode: Comprehension AACSB: Analytic IMA: Corporate Finance AICPA: Strategic/Critical Thinking
1.22
Agency conflicts: What are some of the regulations that pertain to boards of directors that were put in place to reduce agency conflicts?
Solution: Some of the regulations include:
The majority of board members must be outsiders. SM 19-20
LO:
A separation of the CEO and chairman of the board positions is recommended.
Firm is required to have a code of ethics approved by the board.
1.5
Bloomcode: Comprehension AACSB: Analytic IMA: FSA AICPA: Legal/Regulatory Perspective 1.23
Business ethics: How can a lack of business ethics negatively affect the performance of an economy? Give an example.
Solution: A lack of business ethics can lead to corruption, which, in turn, creates inefficiencies in an economy, inhibits the growth of capital markets, and slows the rate of overall economic growth. For example, the Russian economy has had a relatively difficult time attracting foreign investment since the fall of the Soviet Union due, in part, weak ethics in the business community and corruption in the business community and local and national governments. Lower foreign investment has led to slower overall economic growth than the country might otherwise have enjoyed. LO:
1.6
Bloomcode: Comprehension AACSB: Analytic IMA: Business Economics AICPA: Industry/Global Perspective
1.24
Agency conflicts: What are some ways to resolve a conflict of interest?
Solution: One way to resolve a conflict of interest is by complete disclosure. As long as both parties are aware of the fact that, for example, both parties in a lawsuit are represented by the same firm, disclosure is sufficient. Another way to avoid a conflict of interest is for the company to remove itself from serving the interest of one of the parties. This is, for example, the case with accounting firms not being allowed to serve as consultants to companies for whom they perform audits. LO:
1.5
Bloomcode: Application AACSB: Analytic SM 19-21
IMA: FSA AICPA: Legal/Regulatory Perspective
1.25
Information asymmetry: Describe what an information asymmetry is in a business transaction. Explain how the inequity associated with an information asymmetry might be, at least partially, solved through the market for goods or services.
Solution: An information asymmetry exists when one party to a business transaction possesses information that is not available to the other parties in the transaction. If the parties with less information understand their relative disadvantage, they are likely to pay lower prices for the goods and services they purchase, or charge higher prices for the goods and services they sell. LO:
1.6
Bloomcode: Comprehension AACSB: Analytic IMA: Strategic Marketing AICPA: Marketing/Client Focus
1.26
Business ethics: What ethical conflict does insider trading present?
Solution: Insider trading is an example of information asymmetry. The main idea is that investment decisions should be made on an even playing field. Insider trading is morally wrong and has also been made illegal. LO: 1.6 Bloomcode: Comprehension AACSB: Ethics IMA: Business Applications AICPA: Professional Demeanor
Sample Test Problems SM 19-22
1.1
Identify three fundamental types of decisions that financial managers make and identify which part of the balance sheet each of these decisions affects.
Solution: 1. Capital Budgeting Decisions – identifying the productive assets the firm should buy. These decisions affect long-term assets on the balance sheet. 2. Financing Decisions – determining how the firm should finance or pay for assets. These decisions affect long-term debt and stockholders’ equity on the balance sheet. 3. Working Capital Management Decisions – determining how day-to-day financial matters should be managed. These decisions affect current assets and current liabilities on the balance sheet. LO:
1
Level: Intermediate Bloomcode: Comprehension AACSB: Analytic IMA: Decision Analysis AICPA: Decision Modeling
1.2 Which of the following is/are advantages of the corporate form of organization? a. Reduced start-up costs b. Greater access to capital markets c. Unlimited liability d. Single taxation Solution: b. (Shares in a corporation can be sold to raise capital from investors who are not involved in the business. This greatly increases the amount of capital that can be raised to fund the business.) LO:
2
Level: Intermediate Bloomcode: Comprehension AACSB: Analytic IMA: Business Economics SM 19-23
AICPA: Industry/Sector Perspective
1.3
Why is stock value maximization superior to profit maximization as a goal for management?
Solution: While profit maximization appears to be a logical goal at first glance, it has some serious drawbacks. First, since accounting profit is the difference between revenues and expenses, it can be distorted by accounting decisions. Second, accounting profits are quite different from cash flows. Since cash flows are the focus of investors, they should also be the focus of managers. Third, profit maximization does not account for when cash flows actually occur. Finally, profit maximization as a goal ignores the risk involved in generating the cash flows. Stock value maximization is superior to profit maximization because it overcomes all of the listed shortcomings of profit maximization. This is because the value of a firm’s stock is determined by the cash flows that the firm is expected to produce. It accounts for (1) the size of the expected cash flows, (2) the timing of the expected cash flows, and (3) the riskiness of the expected cash flows. LO:
4
Level: Intermediate Bloomcode: Comprehension AACSB: Analytic IMA: Strategic/Critical thinking AICPA: Corporate Finance
1.4
What are agency costs? Explain.
Solution: Agency costs are costs that result from an agency relationship in which there is a conflict of interest between a principal and an agent. An agency relationship exists in a business when a firm’s managers (agents) are not also its owners (principals). Agency costs are incurred when managers act in ways that harm owners’ interests.
SM 19-24
The cost of mechanisms, such as audits, that help control agency conflicts are also agency costs. LO:
5
Level: Intermediate Bloomcode: Comprehension AACSB: Analytic AICPA: Industry/Sector Perpsective IMA: Business Economics
1.5
Identify seven mechanisms that can help better align the goals of managers with those of stockholders.
Solution: Mechanisms that can help align the goals of managers with those of stockholders are as follows: (1) the board of directors, (2) management compensation, (3) the managerial labor market, (4) competition among managers, (5) large stockholders, (6) the takeover market, and (7) the legal and regulatory environment. Boards of directors that are independent from managers can help limit the extent to which managers are able to act solely in their own interest. Firms design compensation plans that are tied to firm performance in order to provide managers with incentives to make decisions consistent with the goal of stockholder value maximization. A third mechanism that helps align the goals of managers is the managerial labor market. It is difficult for poorly performing managers to find a good job elsewhere, and it is difficult for a poorly performing firm to hire good managers, which a firm’s current managers typically want to do. Competition among managers within a firm is a fourth mechanism that helps align the goals of managers with those of stockholders. This is because managers who act in the interest of stockholders are more likely to advance within the firm. Because large stockholders have a lot to gain from aligning the goals of managers with their own, they are likely to expend resources to encourage managers to maximize stock value. Sixth, the threat of a takeover, which leads to firing of poor managers, can provide current managers with
SM 19-25
incentives to perform well. Finally, laws and regulations limit the ability of managers to make decisions that harm stockholders. LO:
5
Level: Intermediate Bloomcode: Comprehension AACSB: Analytic AICPA: Strategic/Critical Thinking IMA: Performance Measurement
SM 19-26
Chapter 2 The Financial System and the Level of Interest Rates
Before You Go On Questions and Answers Section 2.1 1.
What critical economic role does the financial system play in the economy? The financial system is in place to gather money from people and businesses and then channel these funds to those who need it. An efficient financial system is essential for a healthy economy. The major players in the U.S. financial system are big institutions such as the New York Stock Exchange, Citigroup, or State Farm Insurance.
2.
What are the two basic ways in which funds flow through the financial system from lender–savers to borrower–spenders? There are two basic mechanisms by which funds flow through the financial system: 1) Funds can flow directly through financial markets, and (2) funds can flow indirectly through financial institutions.
Section 2.2 1.
Why is it difficult for individuals to participate in the direct financial markets? The financial markets where direct transactions take place are wholesale markets with a typical minimum transaction size of $1 million. Major buyers and sellers of securities in direct financial markets include commercial banks, large corporations, the federal government, hedge funds, and some wealthy individuals.
2.
Why might a firm prefer to have a security issue underwritten by an investment banking firm? SM 19-27
In the most common type of underwriting arrangement, called firm-commitment underwriting, the investment banker assumes the risk of buying the new securities from the issuing company and reselling them to investors. The investment banker guarantees to buy the entire security issue from the company at a fixed price.
Section 2.3 1.
What is the difference between primary and secondary markets? Primary markets are markets where new securities are sold for the first time. Secondary markets are where the owners of outstanding securities can sell them to other investors. They provide the means for investors to sell their securities and get cash.
2.
How and why do large business firms use money markets? Large businesses use money markets to adjust their liquidity positions. If a firm has idle cash sitting around, it can invest it in negotiable CDs, Treasury bills, or other money market instruments. On the other hand, if a company has a temporary cash shortage, it can borrow in the money markets by selling commercial paper at lower interest rates than it could borrow through a commercial bank.
3.
What are capital markets, and why are they important to corporations? Capital markets refer to the segment of the marketplace where capital goods are financed with long-term debt or equities. The most important capital market instruments are common stocks and corporate bonds. Capital markets are important to corporations because they allow them to obtain necessary financing.
Section 2.4 1.
How does information about a firm’s prospects get reflected in its share price? Investors act upon the expectations of a firm’s prospects through trading of the securities. The buying and selling then causes the price of the security to reflect their assessment of its value.
SM 19-28
2.
What is strong-form market efficiency? semistrong-form market efficiency? weak-form market efficiency? Strong-form market efficiency is a market in which all information, private and public, is reflected in the price of the security. The semistrong-form of market efficiency suggests that only public information is reflected in a security’s price, while the weak-form market efficiency holds that both public and private information is reflected in the current price of a security, but also both public and private information has not been taken into account.
Section 2.5 1.
What is financial intermediation, and why is it important? Financial intermediation is the process of converting financial securities with one set of characteristics into securities with another set of characteristics. For example, commercial banks use consumer CD deposits to make loans to small businesses.
2.
What are some services that commercial banks provide to businesses? Commercial banks are the largest financial intermediaries in the economy and offer the widest range of financial services to businesses. Nearly every business has a significant relationship with a commercial bank – usually a checking or transaction account and some type of credit or loan arrangement. In addition, banks do a significant amount of equipment lease financing.
3.
What is an IPO, and what role does an investment banker play in the process? Investment bankers specialize in helping firms to sell their new debt or equity issues in financial markets. In an initial public offering (IPO), the investment banker prepares the new issue for sale and then underwrites the deal. Other functions of the investment
SM 19-29
banker in an IPO process include preparing the prospectus, registering the documentation with the SEC, and providing general financial advice to the issuer.
Section 2.6 1. Explain how the real rate of interest is determined. The real rate of interest depends on interaction between the rate of return that businesses can expect to earn on investments in capital goods and savers’ time preference for consumption today versus willingness to save. Therefore, the real rate of interest is determined when the desired saving level equals the desired level of investment.
2. How are inflationary expectations accounted for in the nominal rate of interest? The nominal interest rate is the rate that is actually observed in the financial markets, and it is equal to the real interest rate plus the expected annualized changes in commodity prices, or inflation premium. This is commonly referred to as the Fisher effect.
3. Explain why interest rates follow the business cycle. Interest rates tend to follow the business cycle to rise during economic expansion and decline during recession. On the one hand, during an expansion, there is upward pressure on interest rates as businesses begin to grow and borrow more money. On the other hand, during a recession, the demand for goods and services is lower, businesses borrow less, and as a result the economy slows down and the interest rates decline. Typically, the Fed also loosens credit to stimulate the economy, which puts further downward pressure on the interest rates.
Self-Study Problems and Solutions 2.1
Economic units that need to borrow money are said to be: a.
Lender–savers
b.
Borrower–spenders SM 19-30
c.
Balanced budget keepers.
d.
None of the above.
Solution: LO:
Such units are said to be (b) Borrower-spenders.
1
Level: Basic
2.2
Explain what the marketability of a security is and how it is determined.
Solution:
Marketability refers to the ease with which a security can be sold and converted into cash. The level of marketability depends on the cost of trading the security and the cost of searching for information. The lower these costs are, the greater the security’s marketability.
LO:
3
Level: Basic
2.3
What are over-the-counter markets (OTCs), and how do they differ from organized exchanges?
Solution:
Securities that are not listed on an organized exchange are sold OTC. An OTC market differs from an organized exchange in that there is no central trading location. OTC security transactions are made via phone or computer as opposed to on the floor of an exchange.
LO:
3
Level: Basic
2.4
What effect does an increase in the demand for business goods and services have on the real interest rate? What other factors can affect the real interest rate?
Solution:
An increase in the demand for business goods and services will cause the borrowing schedule in Exhibit 2.4 to shift to the right, thus increasing the real rate of interest. Other factors that can affect the real interest rate include increases in SM 19-31
productivity, changes in technology, or changes in the corporate tax rate. Demographic factors, such as growth or age of the population, and cultural differences can also affect the real rate of interest. LO:
6
Level: Intermediate
2.5
How does the business cycle affect the nominal interest rate and inflation rate?
Solution:
Both the nominal interest and inflation rates tend to follow the business cycle; that is, they rise with economic expansion and fall during a recession.
LO:
6
Level: Basic
Discussion Questions 2.1
Explain why total financial assets in the economy must equal total financial liabilities. Every financial asset must be financed with some type of a claim or liability. Since all of an economy’s financial assets are just a collection of the individual financial assets, then they should also sum to the collective claims on those assets in the economy.
LO:
1
Level: Basic Bloomcode: Comprehension AACSB: Analytic IMA: Corporate Finance; Business Economics AICPA: Resource Management
2.2
Why don’t small businesses make greater use of the direct credit markets since these markets enable firms to finance their activities at a very low cost? Direct credit markets are geared toward big, established companies since they are wholesale in nature and the minimum transaction size is far beyond the needs of a small SM 19-32
business. Small businesses are better off borrowing money from financial intermediaries, such as commercial banks. LO:
5
Level: Basic Bloomcode: Comprehension AACSB: Analytic IMA: Corporate Finance AICPA: Resource Management
2.3
Explain the economic role of brokers and dealers. How does each make a profit? Brokers and dealers play a similar economic role in that they both bring buyers and sellers of a commodity together in a market. However, brokers only facilitate a transaction by helping the two parties make a transaction and brokers are therefore only compensated for taking on that role. They bear no risk of ownership of securities during the transaction. Dealers on the other hand, take risk in that they will purchase (sell) a commodity from a seller (buyer) without another buyer (seller) necessarily being available. In other words, a dealer will take the risk of purchasing (selling) a commodity and will therefore be compensated for taking that risk.
LO:
3
Level: Basic Bloomcode: Comprehension AACSB: Analytic IMA: Business Economics AICPA: Resource Management
2.4
Why were commercial banks prohibited from engaging in investment banking activities until 1999? Banks had been barred from investment banking following the Great Depression because it was believed that these activities were too risky for banks. At the time, it was believed that excessive risk taking by banks had resulted in a large number of bank failures, which precipitated the Great Depression. Recent research has exonerated the banking system of this charge.
LO:
2 SM 19-33
Level: Basic Bloomcode: Knowledge AACSB: Analytic IMA: FSA AICPA: Legal/Regulatory Perspective
2.5
What are two basic services that investment banks provide in the economy? Investment banks specialize in helping companies sell new debt or equity as well as provide other services such as broker and dealer services.
LO:
2
Level: Basic Bloomcode: Comprehension AACSB: Analytic IMA: Business Economics AICPA: Industry/Sector Perspective
2.6
How do large corporations adjust their liquidity in the money markets? Large corporations can take advantage of money markets to adjust for their liquidity by selling or buying short-term financial instruments such as commercial paper, CDs, or Treasury bills. Large corporations with cash surplus can invest in short-term securities, while corporations with cash shortfall can sell securities or borrow funds on a short-term basis. Money market instruments have a maturity anywhere between one day and one year and therefore are very liquid and less risky than long-term debt.
LO:
3
Level: Basic Bloomcode: Comprehension AACSB: Analytic IMA: Corporate Finance AICPA: Resource Management
2.7
The CFO of a certain company always wears his green suit on a day that the firm is about to release positive information about his company. You believe that you can profit from this information by buying the firm’s shares at the beginning of every day that the
SM 19-34
CFO shows up wearing this green suit. Describe which form of market efficiency is consistent with your belief. You believe that the CFO’s decision to wear a green suit indicates that positive information will be announced and that the company’s stock price will increase following that announcement. If you are correct, knowing what the CFO is wearing before any announcement is valuable private information which should enable you to earn abnormally high returns. Therefore, your belief is consistent with the semistrong form of market efficiency – according to which it is possible to earn abnormally high returns by trading on private information. LO:
4
Level: Intermediate Bloomcode: Application AACSB: Analytic IMA: Corporate Finance AICPA: Strategic/Critical Thinking
2.8
Shouldn’t the nominal rate of interest (Equation 2.1) be determined by the actual rate of inflation (∆Pa), which can be easily measured, rather than by the expected rate of inflation (∆Pe)? The nominal rate of interest is a forward-looking measure, and therefore it makes sense that it is using the expected rate of inflation as opposed to the actual rate of inflation. The expected rate of inflation is the market’s best estimate of what the inflation rate will be in the future.
LO:
6
Level: Basic Bloomcode: Comprehension AACSB: Analytic IMA: Business Economics AICPA: Industry/Sector Perspective
2.9
How does Exhibit 2.5 help explain why interest rates were so high during the early 1980s as compared to the relatively low interest rates in the early 1960s?
SM 19-35
The nominal rate of interest is determined by the real rate of interest plus the expected rate of inflation, and during the 1980s, the U.S. economy experienced a very high rate of inflation and, thus, high interest rates. Looking at Exhibit 2.5 we can see that the inflation increased from less than 2 percent in the 1960s to almost 13 percent in the 1980s. This was a result of the monetary policy instituted by the U.S. government during this period of time. LO:
6
Level: Basic Bloomcode: Analysis AACSB: Analytic IMA: Business Economics AICPA: Resource Management
2.10
When determining the real interest rate, what happens to businesses that find themselves with unfunded capital projects whose rate of return exceeds the cost of capital? The real rate of interest reflects a complex set of forces that control the desired level of lending and borrowing in the economy. In this example, businesses are not investing in projects where the rate of return exceeds the cost of capital. This means that there is lessened a demand for investment funds at the current real interest rate. This will remain so until either the real interest rate changes or until something changes for the firm such as introducing a new technology that will increase the rate of return on projects for the firm.
LO:
6
Level: Basic Bloomcode: Comprehension AACSB: Analytic IMA: Corporate Finance AICPA: Resource Management
Questions and Problems
SM 19-36
BASIC
2.1
Financial System: What is the role of the financial system, and what are the two major components of the financial system?
Solution: The role of the financial system is to gather money from businesses and individuals who have surplus funds and channel funds to those who need them. The financial system consists of financial markets and financial institutions. LO: 2.1 Bloomcode: Knowledge AACSB: Analytic IMA: Business Economics AICPA: Industry/Sector Perspective
2.2
Financial System: What does a competitive financial system imply about interest rates?
Solution: If the financial system is competitive, one will receive the highest possible rate for money invested with a bank and the lowest possible interest rate when borrowing money. Also, only firms with good credit ratings and projects with high rates of return will be financed. LO: 2.1 Bloomcode: Comprehension AACSB: Analytic IMA: Corporate Finance AICPA: Industry/Sector Perspective
SM 19-37
2.3
Financial System: What is the difference between saver–lenders and borrower–spenders, and who are the major representatives of each group?
Solution: Saver–lenders are those who have more money than they need right now. The principal saver–lenders in the economy are households. Borrower–spenders are those who need the money saver–lenders are offering. The main borrower–spenders in the economy are businesses followed by the federal government, although households are important mortgage borrowers. LO: 2.1 Bloomcode: Analysis AACSB: Reflective Thinking IMA: Business Economics AICPA: Industry/Sector Perspective
2.4
Financial Markets: List the two ways in which a transfer of funds takes place in an economy. What is the main difference between these two?
Solution: Funds can flow directly through financial markets or indirectly through intermediation markets where funds flow through financial institutions first. LO: 2.2 Bloomcode: Knowledge AACSB: Analytic IMA: Business Economics AICPA: Resource Management
2.5
Financial Markets: Suppose you own a security that you know can be easily sold in the secondary market, but the security will sell at a lower price than you paid for it. What does this imply for the security’s marketability and liquidity?
Solution: As the price of the security is lower than that you paid for it, it has a lower degree of liquidity to you, the owner. That is because the security cannot now be sold without a loss in SM 19-38
value to the owner. Marketability refers to the ease with which a security can be sold or converted to cash. The information in the problem mentions that the security could be easily sold in secondary market, which implies it has high degree of marketability to you. LO: 2.3 Bloomcode: Application AACSB: Analytic IMA: Corporate Finance AICPA: Resource Management
2.6
Financial Markets: Why are direct financial markets also called wholesale markets?
Solution: The financial markets are also called wholesale markets because the minimum transaction or security denomination is $1 million or more. LO: 2.2 Bloomcode: Comprehension AACSB: Analytic IMA: Business Economics AICPA: Industry/Sector Perspective
2.7
Financial Markets: Trader Inc.is a $300 million company, as measured by asset value, and Horst Corp. is a $35 million company. Both are privately held corporations. Explain which firm more likely to go public and register with the SEC, and why.
Solution: Trader Inc. is more likely to go public because of its larger size. Though the cost of SEC registration and compliance is very high, larger firms can offset these costs by the lower funding cost in public markets. Smaller companies find the cost prohibitive for the dollar amount of securities they sell. LO: 2.3 Bloomcode: Application AACSB: Analytic IMA: Corporate Finance AICPA: Industry/Sector Perspective
SM 19-39
2.8
Primary Markets: What is a primary market? What does IPO stand for?
Solution: A primary market is where new securities are sold for the first time. IPO stands for Initial Public Offering. LO: 2.3 Bloomcode: Knowledge AACSB: Analytic IMA: Business Economics AICPA: Industry/Sector Perspective
2.9
Primary Markets: Identify whether the following transactions are primary market or secondary market transactions. a. Jim Hendry bought 300 shares of IBM through his brokerage account. b. Peggy Jones bought $5,000 of General Motors bonds from another investor. c. Hathaway Insurance Company bought 500,000 shares of Trigen Corp. when the company issued stock.
Solution: a. Secondary market transactions. b. Secondary market transactions. c. Primary market transactions. LO: 2.3 Bloomcode: Application AACSB: Analytic IMA: Corporate Finance AICPA: Resource Management
2.10
Investment Banking: What does it mean to ―underwrite‖ a new security issue? What compensation does an investment banker get from underwriting a security issue?
Solution: To underwrite a new security issue means that the investment banker buys the entire issue from the firm at a guaranteed price and then resells the security to individual investors or other institutions at a higher price. The difference between the banker’s purchase price and the total resale price is called the underwriting spread, and it is the banker’s compensation. In SM 19-40
addition to underwriting new securities, investment banks also provide other services, such as preparing the prospectus, preparing legal documents to be filed with the SEC, and providing general financial advice to the issuer. LO: 2.2 Bloomcode: Comprehension AACSB: Analytic IMA: Corporate Finance AICPA: Industry/Sector Perspective
2.11
Investment Banking: Cranjet Inc. is issuing 10,000 bonds, and its investment banker has guaranteed a price of $985 per bond. If the investment banker sells the entire issue to investors for $10,150,000. a. What is the underwriting spread for this issue? b. What is the percentage underwriting cost? c. How much did Cranjet raise?
Solution: a. $300,000 ($10,150,000 – $985 x 10,000) b. 3.05 percent ($30/$985) c. $9,850,000 ($985 x 10,000) LO: 2.2 Bloomcode: Application AACSB: Analytic IMA: Corporate Finance AICPA: Resource Management
2.12
Financial Institutions: What are some of the ways in which a financial institution or intermediary can raise money?
Solution: A financial intermediary can raise money through the sale of financial products that individuals or businesses will purchase, such as checking and savings accounts, life insurance policies, pension or retirement funds. LO: 2.1 SM 19-41
Bloomcode: Application AACSB: Analytic IMA: Corporate Finance AICPA: Resource Management
2.13
Financial Institutions: How do financial institutions act as intermediaries to provide services to small businesses?
Solution: Financial intermediaries allow smaller companies to access the financial markets. They do this by converting securities with one set of characteristics into securities with another set of characteristics that meets the needs of smaller companies. By repackaging securities, they are able to meet the needs of different clients. LO: 2.5 Bloomcode: Comprehension AACSB: Analytic IMA: Business Economics AICPA: Industry/Sector Perspective
2.14
Financial Institutions: Which financial institution is usually the most important to businesses?
Solution: The primary financial intermediaries are commercial banks, life insurance companies, casualty insurance companies, pension funds, investment funds, and business finance companies. Commercial banks are the largest and most prominent financial intermediaries in the economy and offer the widest range of financial services to businesses. LO: 2.5 Bloomcode: Comprehension AACSB: Analytic IMA: Business Economics AICPA: Industry/Sector Perspective
2.15
Financial Markets: What is the main difference between money markets and capital markets? SM 19-42
Solution: Money markets are markets in which short-term debt instruments with maturities of less than one year are bought and sold. Capital markets are markets in which equity securities and debt instruments with maturities of more than one year are bought and sold. LO: 2.3 Bloomcode: Analysis AACSB: Reflective Thinking IMA: Corporate Finance AICPA: Industry/Sector Perspective
2.16
Money Markets: What is the primary role of money markets? Explain how the money markets work.
Solution: Money markets provide an option for large corporations to adjust their liquidity positions. Since only seldom are cash receipts and cash expenditures perfectly synchronized, money markets allow companies to temporarily invest idle cash in Treasury bills or negotiable CDs. If a company is short on cash, it can borrow the money from money markets by selling commercial paper at lower interest rates than through commercial banks. LO: 2.3 Bloomcode: Comprehension AACSB: Analytic IMA: Corporate Finance AICPA: Industry/Sector Perspective
2.17
Money Markets: What are the main types of securities in the money markets?
Solution: Treasury bills, bank negotiable CDs, and commercial paper. LO: 2.3 Bloomcode: Knowledge AACSB: Analytic IMA: Corporate Finance AICPA: Industry/Sector Perspective
SM 19-43
2.18
Capital Markets: How do capital market instruments differ from money market instruments?
Solution: Capital market instruments are less liquid or marketable, they have longer maturities, usually between 1 and 30 years, and they carry more financial risk. LO: 2.3 Bloomcode: Analysis AACSB: Analytic IMA: Corporate Finance AICPA: Resource Management
2.19
Market Efficiency: Describe the informational differences that distinguish the three forms of market efficiency.
Solution: The strong form of market efficiency states that all information is reflected in the security prices. In other words, there is no private or inside information, that if released would potentially change the price. The semistrong-form holds that all public information available to investors is reflected in the security’s price. Therefore, insiders with access to private information could potentially profit from trading on this knowledge before it becomes public. Finally, the weak form of market efficiency holds that there is both public and private information that is not reflected in the security’s price and having access to it can lead to abnormal profits. LO: 2.4 Bloomcode: Comprehension AACSB: Analytic IMA: Corporate Finance AICPA: Industry/Sector Perspective
2.20
Market Efficiency: Zippy Computers announced strong fourth quarter results. Sales and earnings were both above analysts’ expectations. You notice in the newspaper that Zippy’s stock price went up sharply on the day of the announcement. If no other information about Zippy became public on the day of the announcement and the overall market was down, is this evidence of market efficiency? SM 19-44
Solution: Yes, if no other information became public and the market was down, the increase in Zippy’s price most likely reflects the effects of investors trading on the good news. Investors, believing that Zippy is now more valuable than they had thought, are willing to pay a higher price for the shares. LO: 2.4 Bloomcode: Application AACSB: Analytic IMA: Corporate Finance AICPA: Industry/Sector Perspective
2.21
Market Efficiency: In problem 2.20, if the market is efficient, would it have been possible for Zippy’s stock price to go down in the day that the firm announced the strong fourth quarter results?
Solution: Yes. The last sentence in the statement of problem 2.20 suggests why this might happen. If, on the same day of the announcement, some very bad news about the future prospects for Zippy became public or if the market went down substantially, Zippy’s stock price might also have gone down despite the positive sales and earnings announcement. LO: 2.4 Bloomcode: Application AACSB: Analytic IMA: Corporate Finance AICPA: Industry/Sector Perspective
2.22
Market Efficiency: If the market is strong-form efficient, then trading on tips you hear from Jim Cramer (the host of Mad Money on CNBC) will generate no excess returns (i.e., returns in excess of fair compensation for the risk you are bearing). True or false?
Solution: True. If the market is strong-form efficient then all new information gets reflected in stock prices very quickly. In such a market there is nothing you will hear from Jim Cramer on his TV show that will enable you to consistently earn excess returns. The information in his tips will already be reflected in stock prices by the time you can trade on them. LO: 2.4 SM 19-45
Bloomcode: Application AACSB: Analytic IMA: Corporate Finance AICPA: Resource Management
2.23
Financial Markets: What are the major differences between public and private markets?
Solution: Public markets are organized financial markets (also referred as Exchange) where the public buys and sells securities through their stock brokers. The SEC regulates public securities markets in the United States. In contrast, private markets involve direct transactions between two parties. These transactions lack SEC regulation. LO: 2.3 Bloomcode: Analysis AACSB: Analytic IMA: Business Economics AICPA: Industry/Sector Perspective
2.24
Financial Instruments: What are the two risk-hedging instruments discussed in the chapter?
Solution: The two risk-hedging instruments discussed are futures and options contracts. LO: 2.3 Bloomcode: Comprehension AACSB: Analytic IMA: Corporate Finance AICPA: Risk Analysis
2.25
Interest Rates: What is the real rate of interest, and how is it determined?
Solution: The real rate of interest measures the return earned on savings, and it represents the cost of borrowing to finance capital goods. It is the interest rate determined in the absence of inflation. The real rate of interest is determined by the interaction between firms that invest in capital projects and the rate of return businesses can expect to earn on investments in capital
SM 19-46
goods, and individuals’ time preference for consumption. Graphically, it is that point when the desired saving level equals the desired level of investment in the economy. LO: 2.6 Bloomcode: Comprehension AACSB: Analytic IMA: Corporate Finance AICPA: Resource Management
2.26
Interest Rates: How does the nominal rate of interest vary over time?
Solution: The nominal rate is the rate that we observe in the marketplace. It is determined by both the real rate as well expected inflation. Therefore, the nominal rate will fluctuate according the changes in the real rate as well as changes in expected inflation. LO: 2.6 Bloomcode: Comprehension AACSB: Analytic IMA: Corporate Finance AICPA: Resource Management
2.27
Interest Rates: What is the Fisher equation, and how is it used?
Solution: The Fisher equation is the expected, not the reported or actual, annualized change in commodity prices (∆Pe). It is used to protect the buying power from changes in inflation, and it is incorporated into a loan contract by adding it to the real interest rate that would exist in the absence of inflation. LO: 2.6 Bloomcode: Knowledge AACSB: Analytic IMA: Corporate Finance AICPA: Resource Management
2.28
Interest Rates: Imagine you borrow $500 from your roommate, agreeing to pay her back $500 plus 7 percent nominal interest in one year. Assume inflation over the life of the SM 19-47
contract is expected to be 4.25 percent. What is the total dollar amount you will have to pay her back in a year? What percentage of the interest payment is the result of the real rate of interest? Solution: You will pay her back $535 ($500 × 1.07) in one year. Given an inflation of 4.25 percent, the real rate of interest is 2.75 percent. This means that $13.75 will be a result of the real interest rate ($500 × 1.0275). LO: 2.6 Bloomcode: Analysis AACSB: Reflective Thinking IMA: Corporate Finance AICPA: Resource Management
2.29
Interest Rates: Your parents have given you $1,000 a year before your graduation so that you can take a trip when you graduate. You wisely decide to invest the money in a bank CD that pays 6.75 percent interest. You know that the trip costs $1,025 right now and that the inflation for the year is predicted to be 4 percent. Will you have enough money in a year to purchase the trip?
Solution: Yes. The CD will be worth $1,067.50 at the end of the year ($1,000 x 6.75% + $1,000), and the price of the trip will be $1,066 ($1,025 x 4% + $1,025). The CD will be able to cover the trip. LO: 2.6 Bloomcode: Analysis AACSB: Reflective Thinking IMA: Corporate Finance AICPA: Resource Management
2.30
Interest Rates: When are the nominal and real interest rates equal?
Solution: The only time the nominal and real interest rates are equal is when the expected rate of inflation over the contract period is zero. LO: 2.6 Bloomcode: Comprehension SM 19-48
AACSB: Analytic IMA: Corporate Finance AICPA: Resource Management
SM 19-49
Sample Test Problems 2.1
What are the two basic mechanisms through which funds flow through the financial
system, and how do they differ? Solution: The two basic mechanisms are the direct financing mechanism and the indirect financing mechanism. In the direct financing mechanism, issuers of securities (borrowerspenders) sell the securities directly to investors (lender-savers). In the indirect financing mechanism, financial institutions aggregate money from lender-savers and make this capital available through loans to borrower-spenders. LO: 1 Level: Intermediate Bloomcode: Comprehension AACSB: Analytic IMA: Corporate Finance AICPA: Resource Management
2.2
You just purchased a share of IBM stock on the New York Stock Exchange. What kind of
transaction was this? a.
Primary market transaction.
b.
Secondary market transaction.
c.
Futures market transaction.
d.
Private placement.
Solution: b (The primary market is the one in which owners of outstanding securities sell their securities to other investors.) LO: 3 Level: Intermediate Bloomcode: Application AACSB: Analytic IMA: Corporate Finance AICPA: Industry/Sector Perspective
SM 19-50
2.3
How are brokers different from dealers?
Solution: Brokers bring buyers and sellers together. They execute a transaction for their client and are compensated with a commission fee. Brokers never own the securities being sold and therefore do not bear any risk of ownership. In contrast, dealers purchase securities and sell them from an inventory that they own. Dealers profit if they are able to sell securities for than they paid for them. Because they own the securities, dealers face the risk that the prices of the securities in their inventory will fall below what they paid for those securities. LO: 3 Level: Intermediate Bloomcode: Analysis AACSB: Reflective Thinking IMA: Business Economics AICPA: Industry/Sector Perspective
2.4
List the three forms of the efficient market hypothesis, and describe what information is
assumed to be reflected in security prices under each of these hypotheses. Solution: 1) Strong-Form Efficiency – all information is reflected in the security prices 2) Semistrong-Form Efficiency – all public information is reflected in the security prices. 3) Weak-Form Efficiency – all information contained in past prices is reflected in security prices. LO: 4 Level: Intermediate Bloomcode: Comprehension AACSB: Analytic IMA: Business Economics AICPA: Industry/Sector Perspective
2.5 If the nominal rate of interest is 4.25 percent and the expected rate of inflation is 1.75 percent, what is the real rate of interest? SM 19-51
Solution: Using the Fisher equation: i = r + ∆Pe + r ∆Pe where i = 0.0425 and ∆Pe = 0.0175 Solving for r, we get r = 0.02457, or 2.457% LO: 6 Level: Intermediate Bloomcode: Application AACSB: Analytic IMA: Corporate Finance AICPA: Resource Management
2.6
What is the relation between business cycles and the general level of interest rates?
Solution: As Exhibit 2.5 shows, interest rates tend to follow the business cycle. The level of interest rates tends to rise during periods of economic expansion and decline during periods of economic contraction. LO: 6 Level: Intermediate Bloomcode: Comprehension AACSB: Analytic IMA: Business Economics AICPA: Industry/Sector Perspective
Chapter 3 Financial Statements, Cash Flows, and Taxes
Before You Go On Questions and Answers Section 3.1 1.
What types of information does a firm’s annual report contain?
SM 19-52
A firm’s annual report is typically divided into three sections: financial tables with an accompanying verbal explanation of the firm’s performance over the past year; a corporate public relations section discussing the firm’s operations, and the audited financial statements (balance sheet, income statement, statement of cash flows, and statement of retained earnings).
2.
What is the realization principle, and why may it lead to a difference in the timing of when revenues are recognized on the books and cash is collected? According to the realization principle, revenue should only be recognized when the earning process is completed and the exchange of goods or services can be determined by an arm’s length transaction. Although this principle works in theory, it still does not specify whether this is the point when the goods are ordered, when they are shipped, or when the payment is actually received from the customer. Also, not many purchases are paid for in cash any more. Therefore, even if the transaction is recognized at the point at which the customer receives the goods, the actual cash flow might not occur until days later (depending what the terms are).
Section 3.2 1.
What is net working capital? Why might a low value for this number be considered undesirable? Net working capital is the difference between total current assets and total current liabilities. A low value for this number is undesirable, for it indicates that the company may not have enough cash on hand to cover its immediate expenses.
2.
Explain the accounting concept behind depreciation. Depreciation in accounting is a noncash expense that helps to allocate the cost of an item over its expected life. It reflects the estimated decrease in the value of an asset due to wear and tear and obsolescence.
3.
What is treasury stock?
SM 19-53
Treasury stock is the stock that the company purchased back from its investors. These shares do not pay dividends, have no voting rights, and should not be included in sharesoutstanding calculations.
Section 3.3 1.
What is the difference between book value and market value? Book value is the price you paid for a particular asset. This price does not change as long as you own the asset. On the other hand, market value is the price at which you can sell an asset today, as it takes into account how much it can earn in the future.
2.
What are some objections to the preparation of marked-to-market balance sheets? Marked-to-market balance sheets list the firm’s assets and liabilities at their current market prices. Even though a balance sheet constructed with actual market values might paint a more accurate picture of the company’s financial situation, current values are difficult to estimate, and a lot of the complicated models are potentially open to abuse. Therefore, as of the present, the norm is to use book values.
Section 3.4 1.
How is net income computed? Net income is calculated as revenues minus expenses. It is the most comprehensive accounting measure of a company’s performance because it reflects the firm’s accomplishments (revenues) relative to its efforts (expenses) during a time period.
2.
What is EBITDA, and what does it measure? EBITDA stands for earnings before interest, taxes, depreciation, and amortization. EBIT is defined as earnings before taxes and interest. The main difference between these two figures is that EBITDA shows the income earned purely from operations and reflects how efficiently a firm can manufacture and sell its products without taking into account the cost of the productive asset base.
3.
What accounting events trigger changes to the retained earnings account? SM 19-54
Two events will trigger changes to the retained earnings account: (1) a firm’s report of a net income or loss and (2) the board of directors’ declaration of a cash dividend.
Section 3.5 1.
How do increases in fixed assets from one period to the next affect cash holdings for the firm? An increase in fixed assets from one period to the next is a use of cash. If a company purchases fixed assets during the year, it decreases cash because it must use cash to pay for the purchase.
2.
Name two working capital accounts that represent sources of cash for the firm. An increase in current liabilities is a source of cash. Two common current liabilities are accounts and notes payable. An increase in either of these accounts from one period to the next will represent a source of cash for the firm.
3.
Explain the difference between cash flows from financing and investing activities. Cash flows from financing activities occur when cash is obtained from or repaid to creditors or owners (stockholders). Cash flows from investing activities relate to the buying and selling of long-term assets.
Section 3.6 1.
Explain how the four financial statements are related. The four financial statements are linked together as follows: the ending cash balance from the statement of cash flows is used as the cash balance on the balance sheet, and the net income reported in the income statement is transferred to retained earnings on the balance sheet. So as you can see, the balance sheet is the one financial statement that ties all four statements together.
Section 3.7 SM 19-55
1.
How does the calculation of net income differ from the calculation of cash flow to investors from operating activity? Net income is calculated as the difference between revenues and expenses from the income statement. Cash flow from operating activity is equal to the firm’s earnings before interest and taxes (EBIT) minus taxes paid in cash plus the firm’s noncash expenses including depreciation and amortization.
2.
All else being equal, if a firm increases its accounts payable, what effect will this have on cash flow to investors? An increase in a current liability such as accounts payable from one period to the next will reduce the cash flow from net working capital and will, therefore, increase cash flow to investors.
3.
What does it mean when a firm’s cash flow to investors is negative? When a firm’s investment of cash flow from net working capital and long-term assets exceed the firm’s cash flow from operating activity, the cash flow to investors will be negative. A negative cash flow to investors means that the firm must raise money from new issues of debt or equity.
Section 3.8 1.
Why is it important to consider the consequences of taxes when financing a new project? When financing a new project, it is important to consider the consequences of taxes because ultimately they have a significant impact on the company’s income.
2.
Which type of tax rate, marginal or average, should be used in analyzing the expansion of a product line, and why? When analyzing the expansion of a product line, the marginal tax rate should be the type to consider because it is the amount paid on an additional dollar of income earned. Since expansion of a product line is expected to generate new cash flows, the company will be taxed based on the additional earnings. Average tax rate is not as relevant when making financing decisions because it is simply the total taxes paid divided by taxable income. SM 19-56
3.
What are the tax implications of a decision to finance a project using debt rather than new equity? The difference between debt financing and financing through new equity is in the tax treatment of interest and dividends. While interest payments on debt are tax-deductible business expenses, dividends paid to common or preferred stockholders are not deductible.
Self-Study Problems and Solutions 3.1
The going concern assumption of GAAP implies that the firm: a.
Is going under and needs to be liquidated at historical cost.
b.
Will continue to operate and its assets should be recorded at historical cost.
c.
Will continue to operate and that all assets should be recorded at their cost rather than at their liquidation value.
d. Solution:
Is going under and needs to be liquidated at liquidation value. One of the key assumptions under GAAP is the going concern assumption, which states that the firm: (c) will continue to operate and that all assets should be recorded at their cost rather than at their liquidation value.
LO: 1 Level: Basic
3.2
The Ellicott City Ice Cream Company management has just completed an assessment of the company’s assets and liabilities and has obtained the following information. The firm has total current assets worth $625,000 at book value and $519,000 at market value. In addition, its long-term assets include plant and equipment valued at market for $695,000, while their book value is $940,000. The company’s total current liabilities are valued at market for $543,000, while their book value is $495,000. Both the book value and the SM 19-57
market value of long-term debt is $350,000. If the company’s total assets are equal to a market value of $1,214,000 (book value of $1,565,000), what are the book value and market value of its stockholders’ equity? Solution:
The book value and market value of stockholders’ equity are shown below (in thousands of dollars):
Assets
Liabilities and Equity
Total current
Book
Market
$ 625
$ 519
940
695
Book
Market
$ 495
$ 543
Long-term debt
350
350
Stockholders’ equity
720
321
$1,565
$1,214
Total current liabilities
assets Fixed assets
Total liabilities and Total assets
$1,565
$1,214
stockholders’ equity
LO: 3 Level: Basic
3.3
Depreciation and amortization expenses are: a.
Part of current assets on the balance sheet.
b.
After-tax expenses that reduce a firm’s cash flows.
c.
Long-term liabilities that reduce a firm’s net worth.
d.
Noncash expenses that cause a firm’s after-tax cash flows to exceed its net income.
Solution:
Depreciation and amortization expenses are: (d) noncash expenses that cause a firm’s after-tax cash flows to exceed its net income.
LO: 4 Level: Basic
SM 19-58
3.4
You are given the following information about Clarkesville Plumbing Company. Revenues in 2014 totaled $896, depreciation expenses $75, costs of goods sold $365, and interest expenses $54. At the end of the year, current assets were $121 and current liabilities were $107. The company has an average tax rate of 34 percent. Calculate its net income by setting up an income statement.
Solution:
Clarkesville’s income statement and net income are as follows: Clarkesville Plumbing Company Income Statement for the Fiscal Year Ending December 31, 2014 Amount Revenues
$896.00
COGS
365.00
EBITDA
$531.00
Depreciation
75.00
EBIT
$456.00
Interest
54.00
EBT
$402.00
Taxes (34%)
136.68
Net income
$265.32
LO: 4 Level: Basic
3.5
The Huntington Rain Gear Company had $633,125 in taxable income in the year ending September 30, 2013. Calculate the company’s tax using the tax schedule in Exhibit 3.6.
Solution:
Huntington’s tax bill is calculated as follows: Tax rate
Income
Tax
15%
$50,000
$ 7,500
25
(75,000–50,000)
6,250
34
(100,000–75,000)
8,500
SM 19-59
39
(335,000–100,000)
91,650
34
(633,125–335,000)
101,363
Total taxes payable
$215,263
LO: 8 Level: Basic
Discussion Questions 3.1
What is a major reason for the accounting scandals in the early 2000s? How do firms sometimes attempt to meet Wall Street analysts’ earnings projections? Most people believe that managers’ short-term focus is driven by Wall Street’s demand that companies meet or beat the earnings forecasted by stock analysts. Rather than report the actual earnings of the firm, managers try to meet the market’s expectations by starting with the bottom-line number and backing into a sales figure. Thus, the sales may be consistent with the reported earnings figure, but do not represent the true revenue generated by the firm.
LO: 1 Level: Basic Bloomcode: Comprehension AACSB: Communication IMA: Reporting AICPA: Reporting
3.2
Why are taxes and the tax code important for managerial decision making? Understanding the tax code is critical to finance managers, since most decisions are made on an after-tax basis. Furthermore, taxes affect any valuation analysis and also determine the bottom-line figure that is of concern to shareholders and managers.
LO: 8 Level: Basic
SM 19-60
Bloomcode: Comprehension AACSB: Analytic IMA: Budget Preparation AICPA: Decision Modeling
3.3
Identify the five fundamental principles of GAAP, and explain briefly their importance. The assumption of arm’slength transaction assumes that all business transactions between two parties are made rationally from an economic perspective and both parties will make the deal that provides them the best value. The cost principle calls for the recognition of all accounting transactions at historic cost, or the amount paid or received when the transaction was concluded at arm’s length. The realization principle implies that revenue should be recognized only at the time of the sale. The matching principle dictates that revenue is first recognized and then is matched with the costs incurred in producing the revenue. Finally, the going concern assumption implies that the firm will continue to operate and that all assets should be recorded at their cost rather than at their liquidation value.
LO: 1 Level: Basic Bloomcode: Comprehension AACSB: Communication IMA: Reporting AICPA: Reporting
3.4
Explain why firms prefer to use accelerated depreciation methods over the straight-line method for tax purposes. When a firm uses accelerated depreciation, the depreciation expense is higher than with the straight-line method. This reduces the taxable income and the amount of tax paid by the firm. As a result of this higher noncash expense, the firm’s cash flow is higher.
LO: 2 Level: Basic SM 19-61
Bloomcode: Comprehension AACSB: Communication IMA: Budget Preparation AICPA: Reporting
3.5
What is treasury stock? Why do firms have treasury stock? Any shares repurchased by the company in the open market are recorded as treasury stock in the shareholders’ equity account in the balance sheet. The most common reason for firms doing this is to reduce the number of shares outstanding in the market when the management believes that its firm’s stock is undervalued. This reduction in the number of shares outstanding is expected to boost the share price.
LO: 2 Level: Basic Bloomcode: Comprehension AACSB: Analytic IMA: Coorporate Finance AICPA: Resource Management
3.6
Define book-value accounting and market-value accounting. Book-value accounting implies that all assets and liabilities are recorded and reported at the historical cost when they were acquired. Market-value accounting requires that all assets and liabilities are reported at their current market value.
LO: 3 Level: Basic Bloomcode: Knowledge AACSB: Analytic IMA: Reporting AICPA: Reporting
SM 19-62
3.7
Compare and contrast depreciation expense and amortization expense. Depreciation expense is the amount by which a firm’s fixed assets are written down in a period during which the assets are utilized for generating cash flows. Amortization is the amount by which intangible assets like goodwill, patents, license, copyrights, and trademarks are written down in any period that they are utilized by the firm to generate benefits. Both depreciation and amortization are noncash expenses that will serve to boost the firm’s after-tax cash flows, and generate actual value of the asset at the end of a period.
LO: 4 Level: Basic Bloomcode: Analysis AACSB: Analytic IMA: Corporate Finance AICPA: Measurement
3.8
Why are retained earnings not considered an asset of the firm? Retained earnings are part of shareholders’ equity that has already been utilized by the company. It is a liability of the company and corresponds to a claim by the firm’s shareholders. The retained earnings reported on the balance sheet have already been allocated by the company among various assets and hence are not available for current or future uses. New retained earnings have to be generated to provide for new uses.
LO: 2 Level: Basic Bloomcode: Comprehension AACSB: Communication IMA: Reporting AICPA: Reporting
SM 19-63
3.9
How does a firm’s cash flow to investors from operating activity differ from net income, and why? Net income is an accounting figure that includes both cash and noncash expenses. In addition, revenues are recognized before they are collected and expenses are recognized before they are paid. Cash flow from operating activity recognizes that certain revenues and expenses, while recognized for accounting purposes, do not actually involve a cash flow and ignores the effects of these non-cash items on the income statement.
LO: 5 Level: Basic Bloomcode: Comprehension AACSB: Communication IMA: Reporting AICPA: Reporting
3.10
What is the statement of cash flows, and what is its role? This financial statement records both the cash inflows and cash outflows for a period of time. Thus, it reports the changes in the cash position of a firm between successive accounting periods.
LO: 5 Level: Basic Bloomcode: Knowledge AACSB: Communication IMA: Reporting AICPA: Reporting
SM 19-64
Questions and Problems BASIC 3.1
Balance sheet: Given the following information about Elkridge Sporting Goods, Inc., construct a balance sheet for June 30, 2014. On that date the firm had cash and marketable securities of $25,135, accounts receivables of $43,758, inventory of $167,112, net fixed assets of $325,422, and other assets of $13,125. It had accounts payables of $67,855, notes payables of $36,454, long-term debt of $223,125, and common stock of $150,000. How much retained earnings did the firm have?
Solution: Assets
Book Value
Liabilities and
Book Value
Stockholders’ Equity Cash and marketable
$ 25,135
Accounts payables
$ 67,855
securities Accounts receivable
43,758
Notes payables
36,454
Inventories
167,112
Total current assets
$236,005
Total current liabilities
$104,309
Net fixed assets
325,422
Long-term debt
223,125
Other assets
13,125
Common stock
150,000
Retained earnings
97,118
Total liabilities and Total assets
$574,552
stockholders’ equity
$574,552
LO: 3.2 Bloomcode: Analysis AACSB: Communication IMA: Reporting AICPA: Reporting
SM 19-65
3.2
Inventory accounting: Differentiate between FIFO and LIFO accounting.
Solution: FIFO (first in, first out) refers to the practice of firms, when making sales, assuming that the inventory that came in first (at a lower price) is being sold first. During the period of rising prices, LIFO (last in, last out) implies that a firm is selling the higher cost, newer inventory first, leaving the lower cost, older inventory on the balance sheet. LO: 3.2 Bloomcode: Comprehension AACSB: Communication IMA: Budget Preparation AICPA: Reporting
3.3
Inventory accounting: Explain how the choice of FIFO versus LIFO can affect a firm’s balance sheet and income statement.
Solution: FIFO makes sense during times of rising prices because it allows the firm to eliminate the lower priced inventory first, resulting in higher profit margin. This allows the firm to leave higher valued inventory on the balance sheet. During inflationary times, a firm using LIFO would see a lower profit margin and lower values of inventory on the balance sheet. It is important that anyone who is analyzing firms using different accounting methods on inventory recognize the impact on the bottom line (profit margin and net income) and on current assets. LO: 3.2 Bloomcode: Comprehension AACSB: Communication IMA: Budget Preparation AICPA: Reporting
3.4
Market-value accounting: How does the use of market-value accounting help managers? SM 19-66
Solution: Market-value accounting of both assets and liabilities allows managers to have a truer picture of their company’s financial condition and to do a better job of estimating cash flows that the assets would generate. However, marking-to-market is not as easy as it sounds because of the difficulties involved in coming up with the correct market value of current assets and liabilities. LO: 3.3 Bloomcode: Comprehension AACSB: Communication IMA: Reporting AICPA: Reporting
3.5
Working capital: Laurel Electronics reported the following information at its annual meetings: The company had cash and marketable securities worth $1,235,455, accounts payables worth $4,159,357, inventory of $7,121,599, accounts receivables of $3,488,121, short-term notes payable worth $1,151,663, and other current assets of $121,455. What is the company’s net working capital?
Solution: Total current assets
= $1,235,455 + $3,488,121 + $7,121, 599 + 121,455 = $11,966,630
Total current liabilities = $4,159,357 + $1,151,663 = $5,311,020 Net working capital
= $11,966,630 - $5,311,020 = $6,655,610
LO: 3.2 Bloomcode: Application AACSB: Analytic IMA: Reporting AICPA: Reporting
SM 19-67
3.6
Working capital: The financial information for Laurel Electronics referred to in Problem 3.5 is all at book value. Suppose marking to market reveals that the market value of the firm’s inventory is 20 percent below its book value, its receivables are 25 percent below their book value, and the market value of its current liabilities is identical to the book value. What is the firm’s net working capital using market values? What is the percentage change in net working capital?
Solution: Market value of inventory = $7,121,599 × 0.80 = $5,697,279 Market value of receivables = $3,488,121 × 0.75 = $2,616,091 Total current assets
=
$1,235,455 + $2,616,091 + $5,697,279 + 121,455
= $9,670,280 Total current liabilities = $4,159,357 + $1,151,663 = $5,311,020 Net working capital = $9,670,280 - $5,311,020 = $4,359,260 Percent Change =
$4,359,260 − $6,655,610 $6,655,610
= −34. 5% LO: 3.2 Bloomcode: Analysis AACSB: Communication IMA: Reporting AICPA: Reporting
3.7
Income statement: The Oakland Mills Company has disclosed the following financial information in its annual reports for the period ending March 31, 2014: sales of $1.45 million, cost of goods sold of $812,500, depreciation expenses of $175,000, and interest expenses of $89,575. Assume that the firm has an average tax rate of 35 percent. What is the company’s net income? Set up an income statement to answer the question.
Solution: Oakland Mills Company Income Statement
SM 19-68
For the period ended on March 31, 2014 Amount Revenues
$1,450,000.00
COGS
812,500.00
EBITDA
$ 637,500.00
Depreciation
175,000.00
EBIT
$ 462,500.00
Interest
89,575.00
EBT
$ 372,925.00
Taxes (35%)
130,523.75
Net income
$ 242,401.25
LO: 3.4 Bloomcode: Analysis AACSB: Communication IMA: Reporting AICPA: Reporting
3.8
Cash flow: Describe the organization of the statement of cash flows.
Solution: The statement of cash flows identifies the cash inflows and cash outflows of the firms for a specified period. This allows one to estimate the net cash flows from operations. This financial statement is organized to report the cash flows resulting from the three basic activities in any firm—operating, investing, and financing. The cash flows from operations are the results of netting all revenues and expenses that result from the operating activities of the firm. Buying and selling a firm’s assets lead to cash flows from investing activities. Cash flows from financing activities arise from the firm borrowing from its investors and/or making payments to its lenders and shareholders. LO: 3.5 Bloomcode: Comprehension AACSB: Communication IMA: Reporting AICPA: Reporting SM 19-69
3.9
Cash flows: During 2014, Towson Recording Company increased its investment in marketable securities by $36,845, funded fixed-assets acquisitions of $109,455, and had marketable securities of $14,215 mature. What is the net cash used in investing activities?
Solution: Long-Term Investing Activities Net property, equipment, and other assets
$(109,455.00)
Net acquisitions and dispositions
0.00
Investments in marketable securities
(22,630.00)
Net cash used in investing activities
$(132,085.00)
LO: 3.5 Bloomcode: Comprehension AACSB: Communication IMA: Reporting AICPA: Reporting
3.10
Cash flows: Caustic Chemicals management identified the following cash flows as significant in its year-end meeting with analysts. During the year Caustic had repaid existing debt of $312,080 and raised additional debt capital of $650,000. It also repurchased stock in the open market for a total of $45,250. What is the net cash provided by financing activities?
Solution: Financing Activities Loan repayment
$(312,080)
Increase in long-term debt
650,000
Purchase of treasury stock
(45,250)
Net cash provided by financing activities
$ 292,670
LO: 3.5 SM 19-70
Bloomcode: Application AACSB: Communication IMA: Reporting AICPA: Reporting
3.11
Cash flows: Identify and describe the noncash expenses that a firm may incur.
Solution: A firm may have several items on its income statement that did not result in any cash outflow to the firm. The two largest are depreciation expenses and amortization expenses. Other non-cash expenses include deferred taxes, deferred wages, and depletion charges, which is similar to depreciation and used for natural resource assets. Prepaid expenses also fit into this category as they represent expenses to the firm that were paid in previous period, but yet to incur. LO: 3.5 Bloomcode: Comprehension AACSB: Communication IMA: Reporting AICPA: Reporting
3.12 Cash flows: Given the data for Oakland Mills Company in problem 3.7 above, compute the cash flows to investors from operating activity. Solution: CFOA = EBIT – current taxes + Non-cash expenses = $462,500.00 - $130,523.75 + $175,000.00 = $506,976.25 LO: 3.7 Bloomcode: Application AACSB: Communication IMA: Reporting AICPA: Reporting
SM 19-71
3.13 Cash flows: Hillman Corporation reported current assets of $3,495,055 for the year ending December 31, 2014 and current assets of $3,103,839 on December 31, 2013. Current liabilities for the firm were $2,867,225 and $2,760,124 at the end of 2014 and 2013 respectively. Compute the cash flow invested in net working capital at Hillman Corporation during 2014. Solution: CFNWC = ($3,495,055 - $2,867,225) – ($3,103,839 - $2,760,124) = $284,115 LO: 3.7 Bloomcode: Application AACSB: Communication IMA: Reporting AICPA: Reporting
3.14
Cash flows: Del Bridge Construction had long- term assets before depreciation of $990,560 on December 31, 2013 and $1,211,105 on December 31, 2014. How much cash flow was invested in long-term assets for Del Bridge during 2014?
Solution: CFLTA = $1,211,105 - $990,560 = $220,545. LO: 3.7 Bloomcode: Application AACSB: Communication IMA: Reporting AICPA: Reporting
3.15
Tax: Define average tax rate and marginal tax rate.
Solution: The average tax rate is defined as the total taxes paid divided by taxable income. The marginal tax rate, meanwhile, represents the tax rate that is paid on the last dollar of income earned, or the rate that will be paid on the next dollar earned. LO: 3.8 Bloomcode: Knowledge AACSB: Communication IMA: Reporting SM 19-72
AICPA: Reporting
3.16
Tax: What is the relevant tax rate to use when making financial decisions? Explain why.
Solution: Managers need to use the marginal tax rate for making financial decisions. This is because any additional cash flows that result from a firm’s new projects will be taxed at the marginal tax rate. Thus, this is the appropriate rate to use. LO: 3.8 Bloomcode: Comprehension AACSB: Communication IMA: Investment Decisions AICPA: Decision Modeling
3.17
Tax: Manz Property Management Company announced that in the year ended June 30, 2014, its earnings before taxes amounted to $1,478,936. Calculate its taxes using Exhibit 3.6.
Solution: Earnings before tax = $1,478,936 Tax rate
Income
Tax
15%
$0 to $50,000
$ 7,500.00
25
50,001 75,000
6,250.00
34
75,001 100,000
8,500.00
39
100,001 335,000
91,650.00
34
335,001 10,000,000
388,938.24
35
10,000,001 15,000,000
38
15,000,001 18,333,333
35
More than $18,333,333 Total taxes payable
$502,838.24 SM 19-73
LO: 3.8 Bloomcode: Application AACSB: Communication IMA: Reporting AICPA: Measurement
INTERMEDIATE 3.18
Balance sheet: Tim Dye, the CFO of Blackwell Automotive, Inc., is putting together this year’s financial statements. He has gathered the following balance sheet information. The firm had a cash balance of $23,015, accounts payable of $163,257, common stock of $313,299, retained earnings of $512,159, inventory of $212,444, goodwill and other assets equal to $78,656, net plant and equipment of $711,256, and short-term notes payable of $21,115. It also has accounts receivables of $141,258 and other current assets of $11,223. How much long-term debt does Blackwell Automotive have?
Solution: Blackwell Automotive, Inc. Balance Sheet as of December 31 Liabilities and Stockholders’ Equity
Assets Cash
$
23,015
Accounts receivable
141,258
Inventories
212,444
Other current assets
11,223
Total current assets
$ 387,940
Accounts payable and
$
accruals
163,257
Notes payable
21,115
Total current liabilities
$ 184,372
Net plant and
711,256
Long-term debt
168,022
SM 19-74
equipment Goodwill and other
78,656
Total liabilities
assets
$ 352,394
Common stock
313,299
Retained earnings
512,159
Stockholders’ Equity
$ 825,458
Total liabilities and Total assets
$1,177,85
stockholders’ equity
2
$1,177,85 2
LO: 3.2 Bloomcode: Application AACSB: Analytic IMA: Reporting AICPA: Measurement
3.19
Working capital: Mukhopadhya Network Associates has a current ratio of 1.60, where the current ratio is defined as follows: current ratio = current assets/current liabilities. The firm’s current assets are equal to $1,233,265, its accounts payables are $419,357, and its notes payables are $351,663. Its inventory is currently at $721,599. The company plans to raise funds in the short-term debt market and invest the entire amount in additional inventory. How much can notes payable increase without the current ratio falling below 1.50?
Solution: Let x be the amount raised through notes payables. New current liabilities = $419,357+ $351,663 + x New current assets = $1,233,265 + x
SM 19-75
Current assets $1, 233, 265 x 1.5 Current liabilities $771, 020 x 1.5 ($771, 020 x) $1, 233, 265 x 1.5x x $1, 233, 265 $1,156,530 0.5x $76, 735 x $76, 735 0.5 $153, 470 Thus the firm can add up to $153,470 in inventory by raising money through notes payable without changing the ratio of current assets to current liabilities to more than 1.50. (This ratio of current assets to current liabilities is known as the current ratio and will be discussed in the next chapter.) LO: 3.2 Bloomcode: Analysis AACSB: Analytic IMA: Corporate Finance AICPA: Measurement
3.20
Market value: Reservoir Bottling Co. reported the following information at the end of the year. Total current assets are worth $237,513 at book value and $219,344 at market value. In addition, plant and equipment have a market value of $343,222, and a book value of $362,145. The company’s total current liabilities are valued at market for $134,889 and have a book value of $129,175. Both the book value and the market value of long-term debt is $144,000. If the company’s total assets have a market value of $562,566 and a book value of $599,658, what is the difference between the book value and market value of its stockholders’ equity?
Solution:
Assets
Book
Market
Value
Value
Liabilities and
Book
Market
Value
Value
SM 19-76
stockholders’ equity Total current
$237,51
assets
$219,344
3
Net fixed assets
Total current
$129,175
$134,889
liabilities
362,145
343,222
Long-term debt
144,000
144,000
Other assets
0
0
Common stock
326,483
283,677
Total assets
$599,65
$562,566
Total liabilities
$599,658
$562,566
and stockholders’
8
equity Change in value of stockholders’ equity = $283,677 – $326,483 = $(42,806) LO: 3.3 Bloomcode: Analysis AACSB: Analytic IMA: Corporate finance AICPA: Measurement
3.21
Income statement: Nimitz Rental Company provided the following information to its auditors. For the year ended March 31, 2014, the company had revenues of $878,412, general and administrative expenses of $352,666, depreciation expenses of $131,455, leasing expenses of $108,195, and interest expenses equal to $78,122. If the company’s average tax rate is 34 percent, what is its net income after taxes?
Solution: Nimitz Rental Company Income Statement as of March 31, 2014 Amount Revenues
$878,412
General and administrative expenses
352,666
Leasing expenses
108,195
EBITDA
$417,551
Depreciation expenses
131,455 SM 19-77
EBIT Interest expenses EBT
$286,096 78,122 $207,974
Taxes (34%)
70,711
Net income
$137,263
LO: 3.4 Bloomcode: Analysis AACSB: Analytic IMA: Reporting AICPA: Measurement
3.22
Income statement: Sosa Corporation recently reported an EBITDA of $31.3 million and net income of $9.7 million. The company had $6.8 million in interest expense, and its average corporate tax rate was 35 percent. What was its depreciation and amortization expense?
Solution: Amount EBITDA Less: Depreciation and amortization
$31,300,000.00 9,576,923.08
EBIT
$21,723,076.92
Interest
6,800,000.00
EBT
$14,923,076.92
Taxes (35%)
5,223,076.92
Net income
$ 9,700,000.00
LO: 3.4 Bloomcode: Application AACSB: Analytic IMA: Reporting AICPA: Measurement
SM 19-78
3.23
Income statement: Fraser Corporation has announced that its net income for the year ended June 30, 2014, was $1,353,412. The company had EBITDA of $ 4,967,855, and its depreciation and amortization expense was equal to $1,112,685. The company’s average tax rate is 34 percent. What was its interest expense?
Solution: Amount EBITDA
$4,967,855.00
Depreciation and Amortization
1,112,685.00
EBIT
$3,855,170.00
Interest
1,804,545.76
EBT
$2,050,624.24
Taxes (34%)
697,212.24
Net income
$1,353,412.00
LO: 3.4 Bloomcode: Application AACSB: Analytic IMA: Reporting AICPA: Measurement
3.24
Income Statement: For its most recent fiscal year, Carmichael Hobby Shop recorded EBITDA of $512,725.20, EBIT of $362,450.20, zero interest expense, and cash flow to investors from operating activity of $348,461.25. Assuming there are no non-cash revenues recorded on the income statement, what is the firm’s net income after taxes?
Solution: We know that: Cash flow from operating activity CFOA EBIT Current taxes Noncash expenses Since interest expense = $0, we know that:
EBIT = EBT Therefore, Net income = EBIT - Current taxes SM 19-79
and so: Current taxes = EBIT – Net income Substituting into the equation for CFOA above (Equation 3.4) yields: CFOA = EBIT – (EBIT – Net Income) + Noncash expenses CFOA = Net income + Noncash expenses Since: Depreciation and amortization Noncash expenses EBITDA EBIT $512, 275.20 $362, 450.20 $150, 275 We can calculate net income as: Net income CFOA Depreciation & amortization $348, 461.25 - $150, 275 $198,186.25 LO: 3.4, 3.7 Bloomcode: Analysis AACSB: Analytic IMA: Reporting AICPA: Measurement
3.25
Retained earnings: Columbia Construction Company earned $451,888 during the year ended June 30, 2014. After paying out $225,794 in dividends, the balance went into retained earnings. If the firm’s total retained earnings were $846,972 at the end of fiscal year 2014, what were the retained earnings on its balance sheet on July 1, 2013?
Solution: Columbia Construction Company Retained Earnings for 2014 Balance of retained earnings, July 1, 2013
$ 620,878.00
Add: Net income for the year
451,888.00
Less: Dividends to common stockholders
(225,794.00)
SM 19-80
Balance of retained earnings, June 30, 2014
$ 846,972.00
LO: 3.4 Bloomcode: Application AACSB: Analytic IMA: Reporting AICPA: Measurement
3.26
Cash flow: Refer to the information given in Problem 3.21. What is the cash flow from operations for Nimitz Rental?
Solution: Cash flow from operations
=
Net income + Depreciation
=
$137,263 + $131,455
=
$268,718
LO: 3.5 Bloomcode: Application AACSB: Analytic IMA: Reporting AICPA: Measurement
3.27
Tax: Mount Hebron Electrical Company’s financial statements indicated that the company had earnings before interest and taxes of $718,323. The interest rate on its $850,000 debt was 8.95 percent. Calculate the taxes the company is likely to owe. What are the marginal and the average tax rates for this company?
Solution: EBIT
$718,323.00
Interest
76,075.00
EBT
$642,248.00
SM 19-81
Tax rate
Tax
Income
15%
$0 to $50,000
$ 7,500.00
25
50,001 75,000
6,250.00
34
75,001 100,000
8,500.00
39
100,001 335,000
91,650.00
34
335,001 10,000,000
104,464.32
35
10,000,001 15,000,000
38
15,000,001 18,333,333
35
More than $18,333,333 Total taxes payable
$218,364.32
Marginal tax rate
=
34%
Average tax rate
=
Total taxes payable / Taxable income
=
$218,364.22 / $642,248
=
34%
LO: 3.8 Bloomcode: Analysis AACSB: Analytic IMA: Budget Preparation AICPA: Measurement
ADVANCED 3.28
The Centennial Chemical Corporation announced that, for the period ending March 31, 2014, it had earned income after taxes of $2,768,028.25 on revenues of $13,144,680. The company’s costs (excluding depreciation and amortization) amounted to 61 percent of sales and it had interest expenses of $392,168. What is the firm’s depreciation and amortization expense if its average tax rate is 34 percent?
Solution: SM 19-82
Centennial Chemical Corporation Income statement For the period ending March 31, 2014 Amount Revenues
$13,144,680.00
Costs
$8,018,254.80
EBITDA
$5,126,425.20
Depreciation and amortization
$540,275.00
EBIT
$4,586,150.20
Interest
$392,168.00
EBT
$4,193,982.20
Taxes (34%)
$1,425,953.95
Net income
$2,768,028.25
LO: 3.4 Bloomcode: Analysis AACSB: Analytic IMA: Reporting AICPA: Measurement
3.29
Eau Claire Paper Mill, Inc., had, at the beginning of the current fiscal year, April 1, 2013, retained earnings of $323,325. During the year ended March 31, 2014, the company produced net income after taxes of $713,445 and paid out 45 percent of its net income as dividends. Construct a statement of retained earnings and compute the year-end balance of retained earnings.
Solution: Eau Claire Paper Mill, Inc. Retained Earnings for 2014 Balance of retained earnings, April 1, 2013
$ 323,325.00 SM 19-83
Add: Net income, 2014
713,445.00
Less: Dividends to common stockholders
(321,050.25)
Balance of retained earnings, March 31, 2014
$ 715,719.75
LO: 3.4 Bloomcode: Analysis AACSB: Analytic IMA: Reporting AICPA: Measurement
3.30
Menomonie Casino Company earned $23,458,933 before interest and taxes for the fiscal year ending March 31, 2014. If the casino had interest expenses of $1,645,123, calculate its tax obligation using Exhibit 3.6. What are the marginal and the average tax rates for this company?
Solution: EBIT
$23,458,933.00
Interest
1,645,123.00
EBT
$21,813,810.00
Tax rate
Tax
Income
15%
$0 to $50,000
$
25
50,001 75,000
6,250.00
34
75,001 100,000
8,500.00
39
100,001 335,000
91,650.00
34
335,001 10,000,000
3,286,100.00
35
10,000,001 15,000,000
1,750,000.00
38
15,000,001 18,333,333
1,266,666.54
35
More than $18,333,333
1,218,166.95
Total taxes payable
$7,634,833.49
Marginal tax rate
=
35%
Average tax rate
=
Total taxes payable / Taxable income
7,500.00
SM 19-84
=
$7,634,833.49 / $21,813,810
=
35%
LO: 3.8 Bloomcode: Analysis AACSB: Analytic IMA: Reporting AICPA: Measurement
3.31
Vanderheiden Hog Products Corp. provided the following financial information for the quarter ending June 30, 2014: Net income: $189,425 Depreciation and amortization: $63,114 Increase in receivables: $ 62,154 Increase in inventory: $57,338 Increase in accounts payables: $37,655 Decrease in other current assets: $27,450 What is this firm’s cash flow from operating activities during this quarter?
Solution: Vanderheiden Hog Products Corp. Statement of Cash Flows (Partial) For the quarter ended on June 30, 2014 Operating Activities Net income
$189,425
Additions (sources of cash) Depreciation and amortization
63,114
Increase in accounts payable
37,655
Decrease in other current assets
27,450
Increase in accrued income taxes
0
Subtractions (uses of cash) Increase in receivable
(62,154)
Increase in inventories
(57,338) SM 19-85
Net cash provided by operating activities
$198,152
LO: 3.5 Bloomcode: Analysis AACSB: Analytic IMA: Reporting AICPA: Measurement
3.32
Cash flows: Analysts following the Tomkovick Golf Company were given the following balance sheet information for the years ended June 30, 2014 and June 30 2013: 2014
2013
Assets Cash and marketable securities
$
33,411
$
16,566
Accounts receivable
260,205
318,768
Inventory
423,819
352,740
Other current assets
41,251
29,912
Total current assets
$ 758,686
$ 717,986
Plant and equipment
1,931,719
1,609,898
Less: Accumulated depreciation
(419,044)
(206,678)
Net plant and equipment
$1,512,675
$1,403,220
Goodwill and other assets
382,145
412,565
Total assets
$2,653,506
$2,533,771
Liabilities and Equity
2014
2013
Accounts payable and accruals
$ 378,236
$ 332,004
Notes payable
14,487
7,862
Accrued income taxes
21,125
16,815
$ 413,848
$ 356,681
Long-term debt
679,981
793,515
Total liabilities
$1,093,829
$1,150,196
10,000
10,000
Total current liabilities
Preferred stock Common stock (10,000 shares)
SM 19-86
Additional paid-in capital
975,465
975,465
Retained earnings
587,546
398,110
Less: Treasury stock
(13,334)
Total common equity
$1,559,677
$1,383,575
Total liabilities and equity
$2,653,506
$2,533,771
In addition, it was reported that the company had a net income of $ 3,155,848 and that depreciation expenses were equal to $212,366 during 2014. Assume amortization expense was $0 in 2014. a. Construct a 2014 cash flow statement for this firm. b. Calculate the net cash provided by operating activities for the statement of cash flows. c. What is the net cash used in investing activities? d. Compute the net cash provided by financing activities. Solution: a. Tomkovick Golf Company Statement of Cash FlowsYear Ended June 30, 2014 Operating Activities Net income
$ 3,155,848.00
Additions (sources of cash) Depreciation
212,366.00
Increase in accounts payable
46,232.00
Decrease in accounts receivable
58,563.00
Increase in accrued income taxes
4,310.00
Subtractions (uses of cash) Increase in other current assets
(11,339.00)
Increase in inventories
(71,079.00)
Net cash provided by operating activities
$ 3,394,901.00
Long-Term Investing Activities Increase in property equipment Decrease in goodwill and other assets Net cash used in investing activities
$ (321,821.00) 30,420.00 $ (291,401.00) SM 19-87
Financing Activities Increase in notes payable
$
6,625.00
Decrease in long-term debt
(113,534.00)
Payment of cash dividends
(2,966,412.00)
Purchase of treasury stock
(13,334.00)
Net cash provided by financing activities
Effect of exchange rates on cash
$(3,086.655.00)
$ 0.00
Net increase in cash and marketable
16,845.00
securities Cash and marketable securities at
16,566.00
beginning of year Cash and marketable securities at end of year
$ 33,411.00
b. $ 3,394,901.00 c. $ (291,401.00) d. $(3,086.655.00) LO: 3.5 Bloomcode: Analysis AACSB: Analytic IMA: Reporting AICPA: Measurement
3.33
Cash flows: Based on the financial statements for Tomkovick Golf Company in Problem 3.32, compute the cash flow invested in net working capital and the cash flow invested in long-term assets that you would use in a calculation of the cash flows to investors for 2014.
Solution: CFNWC = ($758,686 – $413,848) – ($717,986 – $356,681) = -$16,467 CFLTA = ($1,931,719 – $1,609,898) + ($382,145 - $412,565) = $291,401 SM 19-88
LO: 3.7 Bloomcode: Analysis AACSB: Analytic IMA: Reporting AICPA: Measurement
Sample Test Problems 3.1
What is the matching principle, and how can it cause accounting expenses to differ from actual cash outflows?
Solution: The matching principle is an accounting principle under which accountants match revenues with the costs incurred to generate those revenues when they prepare an income statement. Because the actual cash flows associated with expenses might not be incurred in the same period that those expenses are recognized in the income statement, the accounting expenses can differ from actual cash outflows. LO: 1 Level: Intermediate Bloomcode: Knowledge AACSB: Analytic IMA: Budget Preparation AICPA: Measurement
3.2
Wolf Pack Enterprises has total current assets of $346,002 and fixed assets of $476,306. The company also has long-term debt of $276,400, $100,000 in its common stock account, and retained earnings of $187,567. What is the value of its total current liabilities?
Solution: Assets Total current assets
Liabilities and Equity $346,002
Total current liabilities
$???????
SM 19-89
Fixed assets
$476,306
Long-term debt
$276,400
Common stock
$100,000
Retained earnings
$187,567
Liabilities and Total assets
$???????
stockholders’ equity
$???????
First, calculate total assets: $346,002 + $476,306 = $822,308 From Equation 3.1: $822,308 - $276,400 – 100,000 – 187,567 = $258,341 (total current liabilities) LO: 2 Level: Intermediate Bloomcode: Analysis AACSB: Analytic IMA: Reporting AICPA: Measurement
3.3
What is the difference between a book-value balance sheet and a market-value balance sheet? Which provides better information to investors and management?
Solution: A book-value balance sheet is an accounting balance sheet in which asset values are listed at their historical cost. A market-value balance sheet reflects current asset values (what the assets are worth today). Market values provide better economic information for investors and managers than book values. Therefore, a market-value balance sheet provides better decision-making information. LO: 3 Level: Intermediate Bloomcode: Analysis AACSB: Analytic IMA: Reporting AICPA: Reporting
SM 19-90
3.4
ACME Corporation had revenues of $867,030 in 2014. It also had expenses (excluding depreciation) of $356,240, depreciation of $103,456, and interest expense of $52,423. What was the company’s net income after taxes if its average tax rate was 40 percent?
Solution: Amount Revenues
$867,030.00
Expenses
356,240.00
EBITDA
$510,790.00
Depreciation
103,456.00
EBIT
$407,334.00
Interest
52,423.00
EBT
$354,911.00
Taxes (40%)
141,964.40
Net income
$212,946.60
LO: 4 Level: Intermediate Bloomcode: Analysis AACSB: Analytic IMA: Reporting AICPA: Measurement
3.5
True Blue Company increased its investments in marketable securities by $323,370 and paid $1,220,231 for new fixed assets during 2014. The company also repaid $779,200 of existing long-term debt while raising $913,455 of new debt capital. In addition, True Blue had a net cash inflow of $345,002 from the sale of fixed assets, and repurchased stock in the open markets for a total of $56,001. What is the net cash used in long term investing activities by True Blue? What is the net cash provided by the company’s financing activities?
Solution: Investing Activities SM 19-91
Net property, equipment and other assets
$(1,220,231)
Net acquisitions and dispositions
345,002
Investments in marketable securities Net cash used in investing activities
$(875,229)
Financing Activities Long-term debt repayment
$(779,200)
Increase in long-term debt
913,455
Purchase of treasury stock
(56,001)
Net cash provided by financing activities
$
78,254
LO: 5 Level: Intermediate Bloomcode: Analysis AACSB: Analytic IMA: Reporting AICPA: Measurement
3.6
Sun Devil Corporation reported EBITDA of $7,300,125 and net income of $3,328,950 for the fiscal year ended December 31, 2014. During the same period, the company had $1,155,378 in interest expense, $1,023,285 in depreciation and amortization expense, and an average corporate tax rate of 35 percent. What was the cash flow to investors from operating activity during 2014?
Solution: CFOA = EBIT – Current taxes + Noncash expenses CFOA = EBITDA – Current taxes CFOA = $7,300,125 – [0.35 × ($7,300,125 - $1,023,285 - $1,155,378)] = $5,507,613 LO: 7 Level: Intermediate Bloomcode: Analysis AACSB: Analytic IMA: Reporting SM 19-92
AICPA: Measurement
Chapter 4 Analyzing Financial Statements Before You Go On Questions and Answers Section 4.1 1.
Why is it important to look at a firm’s historical financial statements? When one does a ratio analysis on a firm, it is important to do the analysis over a three- to five-year period. Looking at the historical financial statements allows one to see the trends on the various ratios—whether they are improving, worsening, or staying the same.
2.
What is the primary concern of a firm’s creditors? The creditors of a firm will want to know if the firm is generating enough cash flows to meet all of its required obligations, like employee wages and trade credit, and still has enough to meet the principal and interest payments due periodically. Their ultimate concern is being paid on time and being repaid in full.
Section 4.2 1.
Why does it make sense to standardize financial statements? Standardized, or common-size, financial statements allow you to make meaningful comparisons of financial statements between two firms that are dramatically different in size. They also make interpretation of data much easier. Standardized or common-size financial statements are those that are prepared with numbers that have been compiled using a common measure of the firm’s size.
2.
What are common-size, or standardized, financial statements, and how are they prepared? Typically, balance sheets are standardized by dividing each asset and liability by total assets (or total deposits in the case of banks) and expressing it as a percentage of total assets. For income statements the scaling factor is sales or revenue, thus expressing each item as a percentage of sales.
Section 4.3 1.
What are the efficiency ratios, and what do they measure? Why, for some firms, is the total asset turnover more important than the fixed asset turnover? SM 19-93
Efficiency ratios (also called turnover ratios) measure how efficiently a firm uses its assets to generate sales. The following ratios were discussed: Inventory turnover/days’ sales in inventory—calculated as cost of goods sold divided by inventory; it shows how many times the inventory of a firm is sold and replaced over a specific period. Accounts receivable turnover/days’ sales outstanding—computed as net sales divided by accounts receivable; it is a measure used to quantify a firm’s effectiveness in extending credit and success in collection of debts. Total asset turnover—calculated as net sales divided by total assets; it gives us the amount of sales generated for every dollar’s worth of assets. Fixed asset turnover—calculated by dividing sales by net fixed assets; it shows the amount of sales generated with each dollar of fixed assets. The fixed asset turnover is not very useful for service-oriented firms with little plant or equipment. Instead, the total asset turnover is more relevant. 2.
List the leverage ratios discussed in this section, and explain how they are related. Leverage ratios measure the extent to which a firm uses debt rather than equity financing and show the firm’s ability to meet its long-term financial obligations. The three leverage ratios discussed in this book are total debt ratio (plus its variations of debt-equity ratio and the equity multiplier), times interest earned, and cash coverage ratio. This is how all three ratios are linked together: the total debt ratio indicates what proportion of assets are financed from other sources than shareholders, and times interest earned measures how effectively the firm can meet the interest obligations on this financing based on operating earnings (EBIT). Cash coverage takes the analysis a step further, determining whether the firm has enough cash on hand to cover the interest payments.
3.
List the profitability ratios discussed in this section, and explain how they differ from each other. The most commonly used profitability ratios are profit margin (gross profit margin, operating profit margin, and net profit margin), return on assets (ROA), and return on equity (ROE). The profit margin ratios examine the company’s profitability as a percentage of sales, whereas ROE and ROA help us analyze profitability as percentage of an investment, either in assets or in the firm’s equity. In other words, ROA tells us what earnings were generated from invested capital, and ROE indicates how well management is employing the investors’ capital invested in the company.
Section 4.4 1.
What is the purpose of the DuPont system of analysis? The purpose of the DuPont system of analysis is to determine a company’s financial health. It breaks down ROE into three parts: profit margin, total asset turnover, and SM 19-94
financial leverage (equity multiplier). If ROE is unsatisfactory, the DuPont identity helps locate which part of the business is underperforming. 2.
What is the equation for ROA in the DuPont system, and how do the factors in that equation influence the ratio? In the previous section you learned that ROA is calculated as net income divided by total assets. Modifying the equation by dividing both the numerator and denominator by net sales and rearranging the terms, we can rewrite the equation as: ROA= Net income/Net sales x Net sales/Total assets or ROA = Net profit margin x Total asset turnover Therefore, based on the above equation, we can conclude that ROA is a combination of the firm’s ability to efficiently manage its operating expenses (operating efficiency) and efficient asset management (asset use efficiency). In order for the management to increase ROA, it can increase either the profit margin or asset turnover, or both. The strategies that companies choose to pursue to increase their ROA depend on their business type and long-term goals.
3.
What are the three major shortcomings of ROE? The three major shortcomings of ROE are that it does not consider the size of the initial investment or the size of future cash payments, it ignores risk, and it does not directly consider the cash flow involved in the business decision-making process. However, despite the ratio’s drawbacks, it is a handy tool for measuring a company’s profitability and identifying industry leaders, as long as you know where the numbers are coming from.
Section 4.5 1.
In what three ways can a financial manager choose a benchmark? A financial manager can choose a benchmark based on either of the following methods: trend analysis, industry average analysis, or peer group analysis. Trend analysis uses history as its standard by evaluating the firm’s performance against itself over time. Industry average analysis compares the firm’s performance to the performance of sameindustry companies, and peer group analysis benchmarks the company’s performance to that of its competitors. Most of the time, a combination of all three methods is used when analyzing the company.
2.
Explain what the SIC codes are, and discuss the pros and cons of using them in financial analysis. Standard Industrial Classification (SIC) codes are four-digit numbers that are used to categorize business activities. The first two digits determine the broad type of business, SM 19-95
such as mining of metals, and then each additional digit narrows down the industry in greater detail. The advantage of using SIC codes in financial analysis is that the financial ratio data can be segmented into six size categories by asset size or by sales, which allows for more meaningful comparisons of like firms. The main drawback to using SIC codes is that companies with similar business lines, but not necessarily similar target markets, often have the same SIC codes (for example, Neiman Marcus and Wal-Mart). Therefore, using SIC codes as a basis for financial analysis requires more thought than just relying on the classification numbers.
Section 4.6 1.
Explain how the DuPont identity allows us to evaluate a firm’s performance. The DuPont identity, which leads to the measurement of a firm’s return on equity (ROE), allows one to break down the measure into three areas of performance. First, a firm’s operational efficiency is measured by the net profit margin; second, the firm’s efficiency in managing its assets is reflected in its asset turnover ratio; and finally, the impact of financial leverage is shown by the equity multiplier. One is then able to pinpoint areas of strengths and weaknesses by breaking down these measures, which are tied to either the balance sheet or the income statement.
2.
What are the limitations on traditional financial statement analysis? The main problems with financial statement analysis are that since it is based on book values (historical data), it may not reflect a firm’s true economic condition, and there is a lack of economic underpinning for much of the decision rules. It is difficult to say a current ratio of 2.2 is good or that one ratio is more important than another.
3.
List some of the problems that financial analysts confront when analyzing financial statements. Some of the problems that financial analysts run into when analyzing financial statements are: Multiple business lines Inflation adjustments Globalization Seasonality Different accounting principles Therefore, a good analyst does not simply look at the numbers provided in the financial statements, but instead adjusts these numbers for all of the above factors to provide a meaningful and value-added analysis.
SM 19-96
Self Study Problems 4.1
The Abercrombie Supply Company reported the following information for 2014. Prepare a common-size income statement for the year ended June 30, 2014. Abercrombie Supply Company Income Statement for the Fiscal Year Ended June 30, 2014 ($ thousands) Net sales Cost of goods sold Selling and administrative expenses Nonrecurring expenses
$2,110,965 1,459,455 312,044 27,215
Earnings before interest, taxes, depreciation, and amortization (EBITDA)
$ 312,251
Depreciation Earnings before interest and taxes (EBIT) Interest expense Earnings before taxes (EBT) Taxes (35%) Net income
112,178 $ 200,073 117,587 $ 82,486 28,870 $ 53,616
Solution: The common-size income statement for Abercrombie Supply Company should look like the following one: Abercrombie Supply Company Income Statement for the Fiscal Year Ended June 30, 2014 ($ thousands)
Net sales Cost of goods sold Selling and administrative expenses Nonrecurring expenses
$2,110,965 1,459,455 312,044 27,215
% of Net Sales 100.0 69.1 14.8 1.3
Earnings before interest, taxes, depreciation and amortization (EBITDA) Depreciation Earnings before interest and taxes (EBIT) Interest expense Earnings before taxes (EBT) Taxes (35%) Net income
$ 312,251 112,178 $ 200,073 117,587 $ 82,486 28,870 $ 53,616
14.8 5.3 9.5 5.6 3.9 1.4 2.5
SM 19-97
LO: 2 Level: Basic
4.2
Prepare a common-size balance sheet from the following information for Abercrombie Supply Company.
Assets: Cash and marketable securities Accounts receivable Inventory Other current assets Total current assets Net plant and equipment
Total assets
Abercrombie Supply Company Balance Sheet as of June 30, 2014 ($ thousands) Liabilities and Equity: $ 396,494 Accounts payable 708,275 Notes payable 1,152,398 Accrued income taxes 42,115 $2,299,282 Total current liabilities 1,978,455 Long-term debt Total liabilities Common stock Retained earnings Total common equity $4,277,737 Total liabilities and equity
$ 817,845 101,229 41,322 $ 960,396 1,149,520 $2,109,916 1,312,137 855,684 $2,167,821 $4,277,737
Solution: Abercrombie Supply Company Balance Sheet as of June 30, 2014 ($ thousands) Assets Cash and marketable sec. Accounts receivable Inventory Other current assets Total current assets Net plant and equipment
$ 396,494 708,275 1,152,398 42,115 $2,299,282 1,978,455
% of Assets 9.3 16.5 26.9 1.0 53.7 46.3
Total assets
$4,277,737
100.0
Liabilities and Equity: Accounts payable and accruals Notes payable Accrued income taxes
$ 817,845 101,229 41,322
% of Assets 19.1 2.4 1.0
Total current liabilities Long-term debt Total liabilities Common stock Retained earnings Total common equity Total liabilities and equity
$ 960,396 1,149,520 $2,109,916 1,312,137 855,684 $2,167,821 $4,277,737
22.5 26.9 49.3 30.7 20.0 50.7 100.0
LO: 2 Level: Basic
4.3
Using the 2014 data above for Abercrombie Supply Company calculate the following liquidity ratios: a. Current ratio b. Quick ratio
SM 19-98
Solution: Current ratio
$2,299,282
2.39 $960,396 $2,299,282 - $1,152,398 Quick ratio 1.19 $960,396
a. b. LO: 3 Level: Basic
4.4
Refer above to the balance sheet and income statement for Abercrombie Supply Company for the year ended June 30, 2014. Calculate the following ratios: a. Inventory turnover b. Days’ sales outstanding c. Total asset turnover d. Fixed asset turnover e. Total debt ratio f. Debt-to-equity ratio g. Times interest earned h. Cash coverage
Solution: Ratio Inventory turnover Days’ sales outstanding Total asset turnover Fixed asset turnover Total debt ratio Debt-to-equity ratio Times-interest-earned Cash coverage LO: 3 Level: Basic
4.5
Calculation $1,459,455 / $1,152,398 $708,275 / ($2,110,965/365) $2,110,965 / $4,277,737 $2,110,965 / $1,978,455 $2,109,916 / $4,277,737 $2,109,916 / $2,167,821 $200,073 / $117,587 $312,251 / $117,587
Value 1.27 122.5 days 0.49 1.07 0.493 0.974 1.7 2.66
Refer to the balance sheet and income statement for Abercrombie Supply Company for the fiscal year ended June 30, 2014. Use the DuPont equation to calculate the return on equity (ROE). In the process, calculate the following ratios: profit margin, EBIT return on assets, equity multiplier, total asset turnover.
Solution: Following are the calculations for the ROE and associated ratios: SM 19-99
Net Profit margin = Net income / Net Sales = $ 53,616 / $2,110,965 = 0.0254, or 2.54% EBIT ROA = EBIT / Total assets = $200,073 / $4,277,737 = 0.0468, or 4.68% Return on assets = Net income / Total assets = $ 53,616 / $4,277,737= 0.0125, or 1.25% Equity multiplier = Total assets / Total Equity = $4,277,737 / $2,167,821 = 1.97 Total asset turnover = Net Sales / Total assets = $2,110,965 / $4,277,737 = 0.49 DuPont identity: ROE = ROA × EM = Net profit margin × total assets turnover × equity multiplier Net income Net sales Total assets = × × Net sales Total assets Total equity = 0.0254 × 0.49 × 1.97 = 0. 0245, or 2. 45% LO: 3 Level: Basic
Discussion Questions 4.1
What does it mean when a company’s return on assets (ROA) is equal to its return on equity (ROE)? When ROA equals ROE, it means that the firm does not use any leverage (debt financing). For firms that do use leverage, ROE will be higher than ROA.
LO: 3 Level: Basic Bloomcode: Comprehension AASCB: Analytic IMA: FSA AICPA: Measurement
4.2
Why is too much liquidity not a good thing? Too much liquidity could mean that a firm is not putting its money to work as the shareholders would want it to. It could mean that the firm’s managers are being too conservative and investing in low-yield assets, or it could mean that the firm does not have enough investment opportunities and is therefore hanging onto its cash. Recently, several firms including Microsoft had several billions of dollars in cash on their books, and, ultimately, Microsoft paid a special dividend to its shareholders. Too much liquidity can also make it a takeover target for firms looking to utilize the debt capacity of the liquid firm.
LO: 3 SM 19-100
Level: Intermediate Bloomcode: Comprehension AASCB: Analytic IMA: FSA AICPA: Measurement 4.3
Inventory is excluded when the quick ratio or acid-test ratio is calculated because inventory is the most difficult current asset to convert to cash without loss of value. What types of inventory are likely to be most easily converted to cash without loss of value? For the quick ratio, one uses only the most liquid of all assets—that is, all current assets less inventory, which is not very liquid relative to cash or receivables. While the current ratio assumes that inventory could be sold at book value, the quick ratio assumes that inventory has no value. Hence, this gives a more conservative estimate of a firm’s liquidity than the current ratio, and gives a better estimate of the firm’s ability to meet its short-term obligations. Inventories of a raw material commodity, such as gold or crude oil, is more likely to be sold with little loss in value.
LO: 3 Level: Basic Bloomcode: Comprehension AASCB: Analytic IMA: FSA AICPA: Measurement 4.4
What does a very high inventory turnover ratio signify? This could mean a number of things, including that the firm is using up its inventory too fast and is unable to meet the demand for its products, or it has priced its products too low relative to its competitors, or worse, the firm is selling defective products that would eventually be returned.
LO: 3 Level: Basic
4.5
How would one explain a low receivables turnover ratio? A low receivables turnover implies a high DSO. This could mean that the firm’s customers are not paying on time, either because of an inefficient collection system or because of a slowdown in their customers’ business or even in the entire economy.
LO: 3 Level: Basic
SM 19-101
Bloomcode: Comprehension AASCB: Analytic IMA: FSA AICPA: Measurement 4.6
What additional information does the fixed asset turnover ratio provide over the total asset turnover ratio? For which industries does it carry greater significance? The total asset turnover ratio measures the level of sales per dollar invested in total assets. The higher the number, the more efficiently the management is using the firm’s assets. Too high a number relative to its peers could imply that the firm is reaching its full capacity and may require an additional investment in plant and equipment to generate additional sales. The fixed asset turnover ratio can be utilized to break down the performance of individual manufacturing facilities or a division. This ratio provides significant information for manufacturing firms that are capital-intensive, while it will be of much less significance for the service industry, where there is less reliance on plant and equipment.
LO: 3 Level: Intermediate Bloomcode: Analysis AASCB: Analytic IMA: FSA AICPA: Measurement
4.7
How does financial leverage help stockholders? Financial leverage implies the use of debt capital in addition to the owners’ capital to finance the firm. With the addition of debt, the owners’ capital can go a long way in acquiring assets for the firm. Given that creditors only get the fixed-interest payments and do not get any share of the gains from the company, the stockholders gain from the usage of debt. This is called the leverage multiplier effect. As the company’s revenues grow, stockholders get all the gain and the debt holders merely receive their interest payments.
LO: 3 Level: Basic Bloomcode: Comprehension AASCB: Analytic IMA: FSA AICPA: Measurement
4.8
Why is ROE generally much higher than ROA for banks relative to other industries?
SM 19-102
Banks have a very small equity base relative to firms in most other industries. Thus, they are highly leveraged with borrowed funds. Since their equity base is small, this magnifies the return on equity, but the return on assets is relatively small for the large asset base. LO: 3 Level: Intermediate Bloomcode: Analysis AASCB: Analytic IMA: FSA AICPA: Measurement
4.9
Why is the ROE a more appropriate proxy (for stockholder value maximization) for some firms than for other firms? The basis on which any business or investment decisions are evaluated must include the size, timing, and uncertainty in the future cash flows. ROE considers neither the risk of the cash flows nor the size of the initial investment or future cash flows from that investment. While the ROE and shareholder wealth are correlated, this is still a problem in large, well-diversified companies with resources from multiple sources. Smaller firms have fewer resources and sources and can better correlate their ROE to shareholder wealth.
LO: 4 Level: Basic Bloomcode: Comprehension AASCB: Analytic IMA: FSA AICPA: Measurement
4.10
Why is it not enough for an analyst to look at just the short-term and long-term debt on a firm’s balance sheet when assessing the firm’s fixed obligations? The amount of liabilities shown on a firm’s balance sheet is not the total obligation of a firm in any given period. To get a true picture, one needs to look at the financial footnotes that follow the financial statements. This is where you will be able to find the amount of debt repayment that the firm is responsible for in the coming years. In addition, off-balance sheet items could reflect certain future liabilities of the firm that do not have to be reported on the balance sheet. One also should look for lease obligations of the firm that are reported off the balance sheet but nevertheless remain a fixed obligation that the firm has to meet with its cash flows. Thus, it is important for an analyst to look beyond the short-term and long-term debt on the balance sheet to get a true measure of the firm’s true financial commitments in any given period. LO: 3, 5 SM 19-103
Level: Intermediate Bloomcode: Analysis AASCB: Analytic IMA: FSA AICPA: Measurement
Questions and Problems BASIC 4.1
Liquidity ratios: Explain why the quick ratio or acid-test ratio is a better measure of a firm’s liquidity than the current ratio. Solution: The quick ratio is a better or more conservative measure of liquidity than the current ratio. The difference in the measurement of the two is that for the quick ratio we exclude the inventory in accounting of the short- term assets. Thus the quick ratio is measured as: Current assets Inventory Quick ratio Current liabilities This measure includes only the most liquid of the current assets and hence gives a better measure of liquidity. LO: 4.3 Bloomcode: Analysis AASCB: Analytic IMA: FSA AICPA: Measurement
4.2
Liquidity ratios: Flying Penguins Corp. has total current assets of $11,845,175, current liabilities of $5,311,020, and a quick ratio of 0.89. How much inventory does it have? Solution: Current assets = $11,845,175 Current liabilities = $5,311,020 Quick ratio = 0.89 Current assets − Inventory Current liabilities $11,845,175 − Inventory 0.89 = $5,311,020 0.89 × $5,311,020 = $11,845,175 − Inventory Quick ratio =
SM 19-104
Inventory = $11,845,175 − (0.89 × $5,311,020) = $7, 118, 367 LO: 4.3 Bloomcode: Application AASCB: Analytic IMA: FSA AICPA: Measurement
4.3
Efficiency ratio: If Newton Manufacturers has an accounts receivable turnover of 4.8 times and net sales of $7,812,379, what would its receivables be? Solution: Accounts receivable turnover = 4.8 times Net sales = $7,812,379 Net sales Accounts receivable turnover = Accounts receivable 7,812,379 4.8 = Accounts receivable 7,812,379 Accounts receivable = = $1, 627, 579 4.8 LO: 4.3 Bloomcode: Application AASCB: Analytic IMA: FSA AICPA: Measurement
4.4
Efficiency ratio: Bummel and Strand Corp. has a gross profit margin of 33.7 percent, sales of $47,112,365, and inventory of $14,595,435. What is its inventory turnover ratio? Solution: Gross profit margin = 33.7% Sales = $ 47,112,365 Inventory = $14,595,435 Net sales-Cost of goods sold Gross Profit Margin Net Sales 0.337
$47,112, 365 COGS 47,112, 365
Cost of goods sold $47,112, 365 (0.337 47,112, 365) 31, 235, 498 SM 19-105
Inventory Turnover Ratio = Inventory Turnover Ratio =
Cost of goods sold Inventory
31,235,498 14,595,435
= 2. 14 times
LO: 4.3 Bloomcode: Application AASCB: Analytic IMA: FSA AICPA: Measurement
4.5
Efficiency ratio: Sorenson Inc. has sales of $3,112,489, a gross profit margin of 23.1 percent, and inventory of $833,145. What are the company’s inventory turnover ratio and days’ sales in inventory? Solution: Sales = $3,112,489 Gross profit margin = 23.1% Inventory = $833,145 Sales-Cost of goods sold Gross profit margin Sales 0.231
3,112, 489 Cost of Goods Sold 3,112,489
Cost of goods sold 3,112, 489 (0.231 3,112, 489) $2, 393,504
Inventory turnover ratio
Cost of goods sold $2, 393,504 Inventory $833,145
2.873 times
Day's sales in inventory
365 365 Inventory turnover ratio 2.873
127.1 days LO: 4.3
SM 19-106
Bloomcode: Application AASCB: Analytic IMA: FSA AICPA: Measurement
4.6
Leverage ratios: Breckenridge Ski Company has total assets of $422,235,811 and a debt ratio of 29.5 percent. Calculate the company’s debt-to-equity ratio and equity multiplier. Solution: Total assets = $422,235,811 Debt ratio = 29.5% Total debt = $422,235,811 × 0.295 = $124,559,564 Total equity = Total assets − Total debt = $422,235,811 − $124,559,564 = $297,676,247 $124,559,564 = 0. 42 $297,676,247 $422,235,811 Total assets = Equity multiplier = = 1. 42 Total equity $297,676,247 LO: 4.3 Bloomcode: Application AASCB: Analytic IMA: FSA AICPA: Measurement Debt to equity ratio =
4.7
Leverage ratios: Norton Company has a debt-to-equity ratio of 1.65, ROA of 11.3 percent, and total equity of $1,322,796. What are the company’s equity multiplier, debt ratio, and ROE? Solution: Debt-equity ratio = 1.65 ROA = 11.3% Total equity = $1,322,796
SM 19-107
Equity multiplier 1 Debt to equity ratio 1 1.65 2.65 Total assets Equity multiplier Total equity Total assets Equity multiplier Total equity 2.65 $1,322,796 $3,505,409 Net income Total assets Net income ROA Total assets 0.113 3,505,409 ROA
$396,111 Net income $396,111 29.9% $1,322,796 Equity 1 Debt ratio 1 - Equity 1 Total assets Equity multiplier 1 1 0.623 2.65 LO: 4.3 Bloomcode: Application AASCB: Analytic IMA: FSA AICPA: Measurement ROE
4.8
DuPont Equation: The Rangoon Timber Company has the following ratios: Net sales/Total assets = 2.23; ROA = 9.69%; ROE = 16.4% What are Rangoon’s profit margin and debt ratios? Solution: Total assets turnover = 2.23 ROA = 9.69% ROE = 16.4% ROA = Profit margin × Total assets turnover 0.0969 ROA 0.04345 Profit margin Total assets turnover 2.23 4.35% ROE = ROA x Equity multiplier
SM 19-108
ROE 0.164 1.69 ROA 0.0969 1 Debt ratio 1 Equity 1 Total assets Equity multiplier 1 1 0.41 1.69 LO: 4.4 Bloomcode: Application AASCB: Analytic IMA: FSA AICPA: Measurement Equity multiplier
4.9
DuPont Equation: Lemmon Enterprises has a total asset turnover of 2.1 and a net profit margin of 7.5%. If its equity multiplier is 1.90, what is the ROE for Lemmon Enterprises? Solution: Total Asset Turnover = 2.1 Net Profit Margin = 7.5% Equity Multiplier = 1.90 ROE = ROA × Equity multiplier ROA = Net profit margin × Total asset turnover = 7.5% × 2.1 = 15.75% ROE = 15.75% × 1.9 = 29.93% LO: 4.4 Bloomcode: Application AASCB: Analytic IMA: FSA AICPA: Measurement
Benchmark analysis: List the ways a company’s financial manager can benchmark the company’s own performance. Solution: Financial managers can benchmark their firm’s performance by collecting data in three ways: through trend analysis, industry average analysis, and peer group analysis. LO: 4.5 4.10
SM 19-109
Bloomcode: Knowledge AASCB: Analytic IMA: FSA AICPA: Measurement
Benchmark analysis: Trademark Corp.’s financial manager collected the following information for its peer group to compare its performance against that of its peers. Ratios Trademark Peer Group DSO 33.5 days 27.9 days Total asset turnover 2.3 3.7 Inventory turnover 1.8 2.8 Quick ratio 0.6 1.3 a. Explain how Trademark is performing relative to its peers. b. How do the industry ratios help Trademark’s management? Solution: a. One can see that Trademark is not doing as well as its competitors. The asset turnover ratio indicates that the firm either needs to increase its sales relative to its level of assets or reduce its assets relative to its level of sales. In addition, the lower quick ratio indicates a lower level of liquidity, perhaps because of too much inventory. A reduction in the inventory would help Trademark improve its performance to a level that is comparable to its peer group. In addition, collection of receivables is longer, which could result in cash flow problems and nonpayments. b. Average industry ratios serve as benchmarks for the firm’s management to measure its own performance. While no two firms are identical in any industry, the average ratios across the industry are good target ratios for a firm. LO: 4.5 Bloomcode: Analysis AASCB: Analytic IMA: FSA AICPA: Measurement 4.11
4.12
Market-value ratios: Rockwell Jewelers management announced that the company had net earnings of $6,481,778 for this year. The company has 2,543,800 shares outstanding, and the year-end stock price is $54.21. What are Rockwell’s earnings per share and P/E ratio? Solution: Net earnings = $6,481,778 SM 19-110
Shares outstanding = 2,543,800 Year-end stock price = $54.21 Net earnings $6,481,778 Earnings per share EPS 2,543,800 Shares outstanding $2.55 Price per share $54.21 Price - earnings ratio Earnings per share $2.55 21.26times LO: 4.3 Bloomcode: Application AASCB: Analytic IMA: FSA AICPA: Measurement
4.13
Market-value ratios: Chisel Corporation has 3 million shares outstanding at a price per share of $3.25. If the debt-to-equity ratio is 1.7 and total book value of debt equals $12,400,000, what is the market-to-book ratio for Chisel Corporation? Solution: Shares outstanding = 3,000,000 Market Price per share = $3.25 Debt-to-Equity ratio = 1.7 Total book value of debts = $12,400,000 Market value of equity = 3,000,000 × $3.25 = $9,750,000 $12,400,000 Book value of equity 1.7 $7,294,117.65 $9,750,000 Market - to - book ratio $7,294,117 .65 1.34 LO: 4.3 Bloomcode: Application AASCB: Analytic IMA: FSA AICPA: Measurement
SM 19-111
INTERMEDIATE 4.14
Liquidity ratios: Laurel Electronics has a quick ratio of 1.15, current liabilities of $5,311,020, and inventories of $7,121,599. What is the firm’s current ratio? Solution: Quick ratio = 1.15 Current liabilities = $5,311,020 Inventory = $7,121,599 Current asset - Inventory Current assets - 7,121,599 Quick ratio Current liabilities $5,311,020 1.15
Current asset - $7,121,599 $5,311,020
Current assets (1.15 5, 311, 020) 7,121,599 Current assets $13,229,272
Current ratio =
Current assets Current liabilities
=
$13,229,272 = 2. 49 $5,311,020
LO: 4.3 Bloomcode: Application AASCB: Analytic IMA: FSA AICPA: Measurement
4.15
Efficiency Ratio: Lambda Corporation has current liabilities of $450,000, a quick ratio of 1.8, inventory turnover of 5.0, and a current ratio of 3.5. What is the cost of goods sold for Lambda Corporation? Solution: Current Liabilities = $450,000 Quick Ratio = 1.8 Inventory Turnover ratio = 5.0 Current ratio = 3.5 Current assets = 3.5 × $450,000 = $1,575,000
SM 19-112
Quick ratio 1.8
Current assets - Inventory Current liabilities $1,575, 000 Inventory $450, 000
Inventory $765, 000 Inventory Turnover
Cost of goods sold Inventory
Cost of goods sold 5.0 $765, 000 $3,825, 000 LO: 4.3 Bloomcode: Application AASCB: Analytic IMA: FSA AICPA: Measurement
4.16
Efficiency ratio: Norwood Corp. currently has accounts receivable of $1,223,675 on net sales of $6,216,900. What are its accounts receivable turnover and days’ sales outstanding (DSO)? Solution: Accounts receivable = $1,223,675 Net sales = $6,216,900 Accounts receivables turnover =
Net sales Accounts receivables
=
$6,216,900 $1,223,675 = 5. 08 times
365 365 DSO 71.8days Accounts receivable turnover 5.08 LO: 4.3 Bloomcode: Application AASCB: Analytic IMA: FSA AICPA: Measurement
SM 19-113
Efficiency ratio: If Norwood Corp.’s management wants to reduce the DSO from that calculated in Problem 4.16 to an industry average of 56.3 days and its net sales are expected to decline by about 12 percent, what would be the new level of receivables? Solution: Target DSO = 56.3 days New level of sales = $6,216,900 x 0.88 = $5,470,872 365 DSO 56.3 Days Net sales/Accounts receivable 4.17
Accounts receivable DSO Net sales 365
56.3 5, 470,872 365
$843, 863 LO: 4.3 Bloomcode: Application AASCB: Analytic IMA: FSA AICPA: Measurement
4.18
Coverage ratios: Nimitz Rental Company had depreciation expenses of $108,905, interest expenses of $78,112, and an EBIT of $1,254,338 for the year ended June 30, 2014. What are the times-interest-earned and cash coverage ratios for this company? Solution: Depreciation = $108,905 Interest expenses = $78,112 EDIT = $1,254,338 EBIT $1,254,338 Times interest earned Interest expense $78,112 16.1times EBITDA EBIT Depreciation Cash coverage ratio Interest expense Interest expense $1,363,243 $78,112 17.5times
LO: 4.3
SM 19-114
Bloomcode: Application AASCB: Analytic IMA: FSA AICPA: Measurement
Leverage ratios: Conseco, Inc., has a debt ratio of 0.56. What are the company’s debt-toequity ratio and equity multiplier? Solution: Debt ratio = 0.56 4.19
Equity multiplier =
=
Total assets Total equity
1 1−Debt/Total assets
=
=
1 Total equity/Total Assets
1 1−0.56
=2.27 Equity multiplier = 1 + (Debt to equity ratio) Debt-to-equity ratio = Equity multiplier-1 =2.27-1 = 1.27 LO: 4.3 Bloomcode: Application AASCB: Analytic IMA: FSA AICPA: Measurement
4.20
Profitability ratios: Cisco Systems has total assets of $35.594 billion, total debt of $9.678 billion, and net sales of $22.045 billion. Its net profit margin for the year is 20 percent, while the operating profit margin is 30 percent. What are Cisco’s net income, EBIT ROA, ROA, and ROE? Solution: Total assets = $35.594 billion Total debt = 9.678 billion Net sales = $22.045 billion Net profit margin = 20% SM 19-115
Operating profit margin = 30% Net income Net profit margin Sales Net income 0.2 $22.045 $4.409 billion Operating profit margin =
EBIT SALES
EBIT = 0.3 × $22.045 =$6.614 billion EBIT $6.613 18.6% Total assets $35.594 Net income $4.409 ROA 12.4% Total assets $35.594 Total equity = Total assets – Total debt = $35.594 - $9.678 = $25.916 billion Equity multiplier Total assets $35.594 Total equity $25.916 1.37 ROE ROA EM 0.124 1.37 17% LO: 4.3 Bloomcode: Application AASCB: Analytic IMA: FSA AICPA: Measurement EBIT ROA
4.21
Profitability ratios: Procter & Gamble reported the following information for its fiscal year end: On net sales of $51.407 billion, the company earned net income after taxes of $6.481 billion. It had cost of goods sold of $25.076 billion and EBIT of $9.827 billion. What are the company’s gross profit margin, operating profit margin, and net profit margin? Solution: Net sales = $51.407 billion Net income = $6.481 billion SM 19-116
Cost of goods sold = $25.076 billion EBIT = $9.827 billion Net sales - Cost of goodssold Grossprofit margin Net sales $51.407 $25.076 51.2% $51.407 EBIT $9.827 Operating profit margin 19.1% Net sales $51.407 Net income $6.481 Net profit margin 12.6% $51.407 Net sales LO: 4.3 Bloomcode: Application AASCB: Analytic IMA: FSA AICPA: Measurement
4.22
Profitability ratios: Wal-Mart, Inc., has net income of $9,054,000 on net sales of $256,329,812. The company has total assets of $104,912,112 and stockholders’ equity of $43,623,445. Use the extended DuPont identity to find the return on assets and return on equity for the firm. Solution: Net income = $9,054,000 Net sales = $256,329,812 Total assets = $104,912,112 Stockholders equity = $43,623,445 Net income Profit margin $9,054,000 3.53% $256,329,812 Net sales Total assets turnover Net sales $256,329,812 2.44 times Total assets $104,912,112
SM 19-117
Return on assets Profit margin Total assets turnover 0.0353 2.44 8.61% ROE ROA EM 0.0861
Total assets Total equity
0.0861
$104, 912,112 $43, 623, 445
20.71% LO: 4.3 Bloomcode: Application AASCB: Analytic IMA: FSA AICPA: Measurement
4.23
Profitability ratios: Xtreme Sports Innovations has disclosed the following information: EBIT = $25,664,300 Net income = $13,054,000 Net sales = $83,125,336 Total debt = $20,885,753 Total assets = $71,244,863 Compute the following ratios for this firm using the DuPont Identity: debt-to-equity ratio, EBIT ROA, ROA, and ROE. Solution: EBIT = $25,664,300 Net income = $13,054,000 Net sales = $83,125,336 Total debt = $20,885,753 Total assets = $71, 244,863 Debt Debt = Equity (Total assets − Total debt $20,885,753 = = 0. 41 ($71,244,863 − $20,885,753 Equity multiplier = 1 + Debt to equity ratio = 1 + 0.41 = 1. 41 Debt to equity ratio =
EBIT $25,664,300 Total assets $71,244,863 36.02%
EBIT ROA
SM 19-118
Net income $13,054,000 Total assets $71,244,863 18.32% ROE ROA EM
ROA
0.1832 1.41 25.83% LO: 4.3 Bloomcode: Application AASCB: Analytic IMA: FSA AICPA: Measurement
4.24
Market-value ratios: Cisco Systems had net income of $4.401 billion and at year end 6.735 billion shares outstanding. Calculate the earnings per share for the company.
Solution: Net income = $4.401 billion Shares outstanding = 6.735 billion $4.401 Net income Earnings per share = = = $0. 65 per share Shares outstanding 6.735 LO: 4.3 Bloomcode: Application AASCB: Analytic IMA: FSA AICPA: Measurement
4.25
Market-value ratios: Use the information for Cisco Systems in Problem 4.24. In addition, the company’s EBITDA was $6.834 billion and its share price was $22.36. Compute the firm’s price-earnings ratio and the price-EBITDA ratio. Solution: EBITDA = $6.834 billion Share price = $22.36 $22.36 Price - earnings ratio 34.4times $0.65 $6.834 EBITA per share = = $1.015 per share 6.735 SM 19-119
Price EBITDA ratio
$22.36
22.04times
$1.015 LO: 4.3 Bloomcode: Application AASCB: Analytic IMA: FSA AICPA: Measurement
4.26
DuPont equation: Carter, Inc., a manufacturer of electrical supplies, has an ROE of 23.1 percent, a profit margin of 4.9 percent, and a total asset turnover ratio of 2.6 times. Its peer group also has an ROE of 23.1 percent, but has outperformed Carter with a net profit margin of 5.3 percent and a total asset turnover ratio of 3.0 times. Explain how Carter managed to achieve the same level of profitability as reflected by the ROE. Solution: Carter Inc: ROE = 23.1%, PM = 4.9%, TATO = 2.6 times ROA PM TATO 0.049 2.6 0.1274 ROE 0.231 ROA EM 0.231 EM 1.81 times 0.1274 Peer Group: ROE = 23.1%, PM = 5.3%, TATO = 3 times ROE 0.231 PM TATO EM 0.231 EM 1.45 times 0.053 3 Carter matched its peer group’s ROE by using a higher degree of financial leverage as indicated by its higher equity multiplier. LO: 4.4 Bloomcode: Application AASCB: Analytic IMA: FSA AICPA: Measurement
4.27
DuPont equation: Grossman Enterprises has an equity multiplier of 2.6 times, total assets of $2,312,000, an ROE of 14.8 percent, and a total asset turnover ratio of 2.8 times. Calculate the firm’s sales and ROA. SM 19-120
Solution: EM = 2.6 times, TA = $2,312,000, ROE = 14.8%, TATO = 2.8 times
Sales Total assets Sales 2.8 $2,312,000 Sales 2.8 $2,312,000 $6,473,600 ROE = ROA × EM ROE ROA = EM 0.148 = = 0.0569 = 5. 69% 2.6 LO: 4.4 Bloomcode: Application AASCB: Analytic IMA: FSA AICPA: Measurement Total asset turnover
ADVANCED 4.28
Complete the balance sheet of Flying Roos Corporation. Flying Roos Corporation Balance Sheet as of December 31, 2014
Assets: Cash and marketable securities Accounts receivable Inventory Total current assets
Liabilities and Equity: Accounts payable and accruals Notes payable Total current liabilities
Net plant and equipment Total assets
$8,000,000
Long-term debt Common stock Retained earnings Total liabilities and equity
$ 300,000
$2,000,000 $1,250,000
You have the following information: Debt ratio = 40% Current ratio = 1.5 Net Sales = $2.25 million DSO = 39 days Inventory turnover ratio = 3.375 Cost of goods sold = $1.6875 million
SM 19-121
Solution: Total debt 0.4 Total assets 0.4 $8,000,000 $3,200,000 Accounts payable Total debt - (Long term debt Notes payable) $3,200,000 - (2,000,000 300,000) $900,000 Cost of goodssold Inventory turnover ratio Inventory Cost of goodssold $1,687,500 Inventory ITO 3.375 $500,000 Accounts receivables DSO Sales/365 Sales 2, 250, 000 39 Accounts receivables DSO 365 365
$240, 411 Current asset Current ratio Current liabilities 1.5 $1,200,000 $1,800,000 Net plant and equipment (Fixed assets) = Total assets – Total Current assets = $8,000,000 - $1,800,000 = $6,200,000 Cash = Current assets – (Accounts receivables + Inventory) = $1,800,000 – ($240,411 + $500,000) = $1,059,589 Common stock = Total liabilities and Equity – (Total liabilities + Retained earnings) = $8,000,000 – ($3,200,000 + $1,250,000) = $3,550,000 Flying Roos Corporation Balance Sheet as of December 31, 2014 Assets Cash and marketable securities Accounts receivable Inventories Total current assets
$1,059,589 240,411 500,000 $1,800,000
Net plant and equipment
6,200,000
Total assets
$8,000,000
Liabilities and Equity Accounts payable Notes payable
$ 900,000 300,000
Total current liabilities Long-term debt Common stock Retained earnings Total liabilities and equity
$1,200,000 2,000,000 3,550,000 1,250,000 $8,000,000
LO: 4.3 SM 19-122
Bloomcode: Application AASCB: Analytic IMA: FSA AICPA: Measurement
4.29
For the year ended June 30, 2014, Northern Clothing Company has total assets of $87,631,181, ROA of 11.67 percent, ROE of 21.19 percent, and a net profit margin of 11.59 percent. What are the company’s net income and net sales? Calculate the firm’s debt-to-equity ratio. Solution: Total assets = $87,631,181 ROA = 11.67%, ROE = 21.19%, PM = 11.59% NI ROA TA 0.1167 87, 631,181 $10, 226, 559 Net income $10,226,559 $88,236,057 Net sales Profit margin 0.1159 Total equity NI/ROE $10,226,559 / 0.2119 $48,261,251 Total debt Total assets - Total equity $87,631,181 - $48,261,251 $39,369,931 Debt $39,369,931 Equity $48,261,251 81.6%
Debt to equity ratio
LO: 4.3, 4.4 Bloomcode: Application AASCB: Analytic IMA: FSA AICPA: Measurement
4.30
Blackwell Automotive’s balance sheet at the end of its most recent fiscal year shows the following information: Blackwell Automotive Balance Sheet as of March 31, 2014 SM 19-123
Assets: Cash and marketable sec. Accounts receivable Inventory Total current assets
$
23,015 141,258 212,444 $ 376,717
Liabilities and Equity: Accounts payable and accruals Notes payable Total current liabilities
$ 163,257 21,115 $ 184,372
Long-term debt 168,022 Net plant and equipment 711,256 Total liabilities $ 352,394 Goodwill and other assets 89,879 Common stock 313,299 Retained earnings 512,159 Total assets $1,177,852 Total liabilities and equity $1,177,852 In addition, it was reported that the firm had a net income of $156,042 on net sales of $4,063,589. a. What are the firm’s current ratio and quick ratio? b. Calculate the firm’s days’ sales outstanding, total asset turnover ratio, and fixed asset turnover ratio.
Solution:
Current ratio a.
Current assets $376, 717 Current liabilities $184, 372
2.04 times Current asset Inventory $376, 717 $212, 444 Quick ratio Current liabilities $184, 372
0.89 times b. DSO
365 365 Accounts receivables turnover Net sales/Accounts receivables
365 Accounts receivables 365 $141,258 Net sales $4,063,589 12.69days
Net sales $4,063,589 Total assets $1,177,852 3.45times Net sales $4,063,589 Fixed asset turnover Net fixed assets $711,256 5.71times LO: 4.3 Total asset turnover
SM 19-124
Bloomcode: Application AASCB: Analytic IMA: FSA AICPA: Measurement
4.31
The following are the financial statements for Nederland Consumer Products Company for the fiscal year ended September 30, 2014. Nederland Consumer Products Company Income Statement for the Fiscal Year Ended September 30, 2014 Net sales Cost of products sold Gross profit Marketing, research, administrative exp. Depreciation Operating income (loss) Interest expense Earnings (loss) before income taxes Income taxes Net earnings (loss)
$51,407 25,076 $26,331 15,746 758 $ 9,827 477 $ 9,350 2,869 $ 6,481
Nederland Consumer Products Company Balance Sheet as of September 30, 2014 Assets: Cash and marketable securities Investment securities Accounts receivable Inventory Deferred income taxes Prepaid expenses & other receivables Total current assets Property, plant, and equip., at cost Less: Accumulated depreciation Net property, plant, and equipment Net goodwill and other intangible assets Other noncurrent assets Total assets
$ 5,469 423 4,062 4,400 958 1,803 $17,115 25,304 11,196 $14,108 23,900 1,925 $57,048
Liabilities and Equity: Accounts payable Accrued and other liabilities Taxes payable Debt due within one year Total current liabilities Long-term debt Deferred income taxes Other noncurrent liabilities Total liabilities Convertible Class A preferred stock Common stock
$ 3,617 7,689 2,554 8,287 $22,147 12,554 2,261 2,808 $39,770 1,526 2,141
Retained earnings Total stockholders’ equity Total liabilities and equity
13,611 $17,278 $57,048
Calculate all the ratios for which industry figures are available below for Nederland and compare the firm’s ratios with the industry ratios. Ratio Industry Average SM 19-125
Current ratio Quick ratio Gross margin Net profit margin Debt ratio Long-term debt to equity Interest coverage ROA ROE
2.05 0.78 23.9% 12.3% 0.23 0.98 5.62 5.3% 18.8%
Solution: Ratio Current ratio Quick ratio Gross margin Net Profit margin Debt ratio Long-term debt to equity Interest coverage ROA ROE
Industry Average 2.05 0.78 23.9% 12.3% 0.23 0.98 5.62 5.3% 18.8%
Nederland 0.77 0.57 51.2% 12.6% 0.70 0.73 20.6 11.4% 37.5%
LO: 4.3
4.32
Refer to the preceding information for Nederland Consumer Products Company. Compute the firm’s ratios for the following categories and briefly evaluate the company’s performance using these numbers. a. Efficiency ratios b. Asset turnover ratios c. Leverage ratios d. Coverage ratios Solution: Efficiency Ratios Inventory turnover ratio Days sales in inventory Accounts receivables turnover DSO Asset Turnover Ratios Total asset turnover Fixed assets turnover Leverage Ratios Total debt ratio
5.70 64.05 days 12.66 times 28.84 days
COGS/Inventory 365/ITO Sales/AR 365/AR turnover
0.90 times 3.64 times
Sales/TA Sales/FA
0.70
TD/TA SM 19-126
Debt to equity ratio Equity multiplier Coverage Ratios Times interest earned Cash coverage
2.30 3.30
D/E TA/E
20.60 22.19
EBIT/Interest (EBIT + Depreciation)/Interest
LO: 4.3 Bloomcode: Application AASCB: Analytic IMA: FSA AICPA: Measurement
4.33
Refer to the earlier information for Nederland Consumer Products Company. Using the DuPont identity, calculate the return on equity for Nederland, after calculating the ratios that make up the DuPont identity. Solution: Net Profit margin Total assets turnover ratio Equity multiplier Return on assets Return on equity
12.61% 0.90 times 3.30 11.4% 37.5%
NI/Sales Sales/TA TA/E NI/TA PM×TATO×EM
LO: 4.4 Bloomcode: Application AASCB: Analytic IMA: FSA AICPA: Measurement
4.34
Nugent, Inc., has a gross profit margin of 31.7 percent on net sales of $9,865,214 and total assets of $7,125,852. The company has a current ratio of 2.7 times, accounts receivable of $1,715,363, cash and marketable securities of $315,488, and current liabilities of $870,938. a. What is Nugent’s total current assets? b. How much inventory does the firm have? What is the inventory turnover ratio? c. What is Nugent’s days’ sales outstanding? d. If management sets a target DSO of 30 days, what should Nugent’s accounts receivable be? Solution: Cash & marketable securities = $ 315,488; Accounts receivables = $1,715,363 Current liabilities = $870,938; Total assets = $7,125,852 SM 19-127
Sales = $9,865,214; Current ratio = 2.7 a.
Gross profit margin = 31.7%
Current ratio 2.7
b.
Current assets Current liabilities Current assets $870,938
Current assets 2.7 $870,938 $2, 351, 533 Current assets = Cash + Accounts receivables + Inventory $2,351,533 = $315,488 + $1,715,363 + Inventory Inventory = $2,351,533– ($315,488 + $1,715,363) = $320,682 Cost of goods sold = Sales x (1 – Gross PM) = $9,865,214 × (1 – 0.317) = $6,737,941 Cost of goods sold Inventory turnover ratio Inventory $6,737,941 21.01 times $320,682
c. DSO =
365 × Accounts receivables 365 = Net sales/Accounts receivables Net sales =
d.
365 × $1,715,363 = 63. 5 days $9,865,214
Target DSO = 30 days DSO
30= Accounts receivables
365 365 Accounts receivables Net sales/Accounts receivables Net sales
365×Accounts receivables $9,865,214 30 $9,865,214
$810, 840 365 The firm has to limit its accounts receivables to $810,840 at its current sales level to achieve its target DSO of 30 days. LO: 4.3
SM 19-128
Bloomcode: Application AASCB: Analytic IMA: FSA AICPA: Measurement
4.35
Recreational Supplies Co. has net sales of $11,655,000, an ROE of 17.64 percent, and a total asset turnover of 2.89 times. If the firm has a debt-to-equity ratio of 1.43, what is the company’s net income? Solution: EM 1 D / E ratio 1 1.43 2.43 ROE PM TATO EM ROE 0.1764 PM TATO EM 2.89 2.43 2.51% NI Sales PM $11, 655, 000 0.0251 $292, 540.50 LO: 4.4 Bloomcode: Application AASCB: Analytic IMA: FSA AICPA: Measurement
4.36
Nutmeg Houseware, Inc., has an operating profit margin of 10.3 percent on revenues of $24,547,125 and total assets of $8,652,352. a. Calculate the company’s total asset turnover ratio and its operating profit (EBIT). b. The company’s management has set a target for the total asset turnover ratio of 3.25 for next year. If there is no change in the total assets of the company, what will the new sales level have to be next year? Calculate the dollar change in sales necessary and the percentage change in sales necessary. c. If the operating profit margin declines to 10 percent, what will be the EBIT at the new level of net sales? Solution: Operating PM = 10.3% Sales = $24,547,125 Total assets = $8,652,352
SM 19-129
a.
Sales $24,547,125 Total assets $8,652,352 2.84 times Operating profit EBIT Op. PM Sales TATO
0.103 $24,547,125
b.
$2, 528, 354 New TATO = 3.25 times = New sales/TA New sales TATO TA 3.25 $8,652,352 $28,120,144 $ Change in Sales $28,120,144 - $24,547,125
c.
$3,573,019 % Change in sales $3,573,019/ $24,547,125 14.6% Operating PM 10% Sales $28,120,144 New EBIT Profit margin Sales 0.10 $28,120,144 $2,812,014
LO: 4.3 Bloomcode: Application AASCB: Analytic IMA: FSA AICPA: Measurement
4.37
Modern Appliances Corporation has reported its financial results for the year ended December 31, 2014. Modern Appliances Corporation Income Statement for the Fiscal Year Ended December 31, 2014 Net Sales Cost of goods sold Gross profit Selling, general, and admin. expenses Depreciation Operating income Interest expense
$5,398,412,000 3,432,925,255 $1,965,486,745 1,036,311,231 299,928,155 $ 629,247,359 35,826,000 SM 19-130
EBT Income taxes Net earnings
Assets: Cash and cash equivalents Accounts receivable Inventory Other current assets Total current assets Net fixed assets Goodwill Other assets Total assets
$ 593,421,359 163,104,554 $ 430,316,805 Modern Appliances Corporation Balance Sheet as of December 31, 2014 Liabilities and Equity: $ 514,412,159 Short-term borrowings 1,046,612,233 Trade accounts payable 981,870,990 Other current liabilities 313,621,610 $2,856,516,992 Total current liabilities 754,660,275 Long-term debt 118,407,710 Common stock 665,058,761 Retained earnings $4,394,643,738 Total liabilities and equity
$ 117,109,865 466,937,985 994,289,383 $1,578,337,233 1,200,691,565 397,407,352 1,218,207,588 $4,394,643,738
Using the information from the financial statements, complete a comprehensive ratio analysis for Modern Appliances Corporation. a. Calculate these liquidity ratios: current and quick ratios. b. Calculate these efficiency ratios: inventory turnover, accounts receivable turnover, DSO. c. Calculate these asset turnover ratios: total asset turnover, fixed asset turnover. d. Calculate these leverage ratios: total debt ratio, debt-to-equity ratio, equity multiplier. e. Calculate these coverage ratios: times interest earned, cash coverage. f. Calculate these profitability ratios: gross profit margin, net profit margin, ROA, ROE. g. Use the DuPont identity after calculating the component ratios to compute ROE. Solution: a.
b.
c.
d.
Liquidity Ratios Current ratio Quick ratio Efficiency Ratios Inventory turnover ratio Accounts receivables turnover DSO Asset Turnover Ratios Total asset turnover Fixed assets turnover Leverage Ratios Total debt ratio
1.81 Current assets/Current liabilities 1.19 (Current assets-Inventory)/Current liabilities 3.50 times 5.16 times 70.76 days
COGS/Inventory Net Sales/AR 365/AR turnover
1.23 times 7.15 times
Net Sales/Total assets Net Sales/Fixed assets
0.63
Total debt/Total assets
SM 19-131
e.
f.
g.
Debt to equity ratio Equity multiplier Coverage Ratios Times interest earned Cash coverage Profitability Ratios Gross profit margin Net profit margin ROA ROE
1.72 2.72
Debt/Equity Total assets/Equity
17.56 25.94
EBIT/Interest (EBIT + Depreciation)/Interest
0.36 0.08 0.10 0.27
Gross profit/Net Sales Net income/Net Sales Net income/Total assets Net income/Equity
DuPont identity. ROE
Net income Net Sales Total assets Net Sales Total assets Common equity
0.08 1.23 2.72 0.27 LO: 4.3, 4.4 Bloomcode: Application AASCB: Analytic IMA: FSA AICPA: Measurement
CFA PROBLEMS 4.38
Common-size analysis is used in financial analysis to: a. evaluate changes in a company’s operating cycle over time. b. predict changes in a company’s capital structure using regression analysis. c. compare companies of different sizes or compare a company with itself over time. d. restate each element in a company’s financial statement as a proportion of the similar account for another company in the same industry.
Solution: c is correct. The most significant benefit of using common-size statements is scaling, whether for a given company or over time. Common-size analysis allows us to make comparisons of investments, financing, and profitability between companies of different sizes and over time for a single company. LO: 4.2 Bloomcode: Application AASCB: Analytic IMA: FSA AICPA: Measurement
SM 19-132
4.39
The TBI Company has a number of days of inventory of 50. Therefore, the TBI Company’s inventory turnover is closest to: a. 4.8 times. b. 7.3 times. c. 8.4 times. d. 9.6 times. Solution: b is correct. We perform the calculations using the following relationship: Cost of goods sold Inventory = 365 × Inventory Cost of goods sold / 365 Inventory turnover Number of days of inventory = 365 Inserting the given information, we have Inventory turnover × 50 = 365, and solving for Inventory turnover provides a turnover of 7.3 times. LO: 4.3 Bloomcode: Application AASCB: Analytic IMA: FSA AICPA: Measurement
4.40
DuPont analysis involves breaking return-on-assets ratios into their a. profit components. b. marginal and average components. c. operating and financing components. d. profit margin and turnover components. Solution: d is correct. This is the DuPont ―triangle,‖ in which profit margins and
turnovers are used to explain returns. LO: 4.4 Bloomcode: Comprehension AASCB: Analytic IMA: FSA AICPA: Measurement
4.41
If a company’s net profit margin is –5 percent, its total asset turnover is 1.5 times, and its equity multiplier is 1.2 times, its return on equity is closest to SM 19-133
a. –9.0 percent. b. –7.5 percent. c. –3.2 percent. d. 1.8 percent. Solution: a is correct. Return on equity =
Net income Average total equity
=
Net income Net sales
×
Net sales Average total assets
×
Average total assets Average total equity
Return on equity = –5% × 1.5 × 1.2 = –9.0% LO: 4.4 Bloomcode: Application AASCB: Analytic IMA: FSA AICPA: Measurement
Sample Test Problems Greenfern Corporation recently filed the following financial statements with the SEC. Greenfern Corporation Income Statement for the Fiscal Year Ended July 31, 2014 Net sales $73,236 Cost of products sold 52,092 Gross profit $21,144 Selling, general, and administrative expenses 9,333 Depreciation 1,060 Operating income (loss) $ 10,751 Interest expense 649 Earnings (loss) before income taxes 10,102 Income taxes 3,536 Net earnings (loss) $ 6,566 Greenfern Corporation Balance Sheet as of July 31, 2014 Assets Liabilities and Stockholders’ Equity Cash and marketable securities $ 9,118 Accounts payable Accounts receivable 7,844 Accrued and other liabilities Inventory 8,900 Taxes payable
$ 6,379 5,663 4,821
SM 19-134
Deferred income taxes Prepaid expenses & other receivables Total current assets Property, plant, and equipment Less: Accumulated depreciation Net property plant and equip.
878 2,803 $29,543 62,467 22,196 $40,271
Goodwill and other assets
16,270
Total assets
$86,084
Debt due within one year Total current liabilities Long-term debt Deferred income taxes Other non-current liabilities Total liabilities Common stock Retained earnings Total stockholders’ equity (deficit) Total liabilities and stockholders’ equity
10,778 $27,641 24,280 6,903 5,608 $64,432 3,667 17,985 $21,652 $86,084
4.1
Refer to the preceding balance sheet and income statement for Greenfern Corporation for the fiscal year ended July 31, 2014. What are the company’s current ratio and quick ratio? What do these ratios tell us about Greenfern? Solution: Current ratio = Current assets = $29,543 =1.07 Current liabilities $27,641
Quick ratio =
Current assets Inventory $29,543$8,900 = = 0.75 Current liabilities $27,641
The current ratio of just over 1 tells us that Greenfern’s cash and marketable securities, and other current assets, such as accounts receivable and inventory, if liquidated, will just provide enough proceeds to cover short-term liabilities. The quick ratio of Greenfern indicates that if we exclude inventory, which might be considerably less liquid than other current assets, the remaining current assets can cover just 75 percent of the short-term liabilities. LO: 3 Level: Intermediate Bloomcode: Analysis AASCB: Analytic IMA: FSA AICPA: Measurement
4.2
Refer to the preceding balance sheet and income statement for Greenfern Corporation for the fiscal year ended July 31, 2014. Calculate the following ratios: SM 19-135
a. Inventory turnover b. Days’ sales outstanding. c. Total asset turnover. d. Fixed asset turnover. e. Total debt ratio. f. Debt-to-equity ratio. g. Times interest earned ratio. h. Cash coverage ratio. Solution: Ratio a. Inventory turnover b. Days’ sales outstanding c. Total asset turnover d. Fixed asset turnover e. Total debt ratio f. Debt-to-equity ratio g. Times interest earned ratio h. Cash coverage ratio LO: 3 Level: Intermediate Bloomcode: Application AASCB: Analytic IMA: FSA AICPA: Measurement
Calculation $52,092/$8,900 $7,844/($73,236/365) $73,236/$86,084 $73,236/$40,271 $64,432/$86,084 $64,432/$21,652 $10,751/$649 ($10,751 + $1,060)/$649
Value 5.85 39.1 days 0.85 1.82 0.75 2.98 16.57 18.20
4.3
Refer to the preceding balance sheet and income statement for Greenfern Corporation for the fiscal year ended July 31, 2014. Use the DuPont identity to calculate the return on equity (ROE). In the process, calculate the following ratios: net profit margin, total asset turnover, equity multiplier, EBIT return on assets (EROA), and return on assets. Solution: Net profit margin = Net income/Net sales = $ 6,566 / $73,236 = 0.0897 or 8.97% Total asset turnover = Net sales/Total assets = $73,236 / $86,084 = 0.85 Equity multiplier = Total assets/Total equity = $86,084 / $21,652 = 3.98 EROA = EBIT/Total assets = $10,751/$86,084 = 0.1249 or 12.49% Return on assets = Net income/Total assets = $ 6,566 / $86,084 = 0.0763 or 7.63% DuPont identity: ROE = ROA × Equity multiplier = Net profit margin × Total asset turnover × Equity multiplier = 0.0897 × 0.85 × 3.98 SM 19-136
= 0.3033 = 30.35% LO: 3, 4 Level: Intermediate Bloomcode: Application AASCB: Analytic IMA: FSA AICPA: Measurement
4.4
Last year Pontiff Enterprises reported net sales of $13,144,680, a gross profit $4,127,429, EBIT of $2,586,150, and net income of $867,555. Compute Pontiff’s cost of goods sold, gross profit margin, operating profit margin, and net profit margin? Solution: Cost of goodssold = Net sales Gross profit = $13,144,680 - $4,127,429 = $9,017,251 Gross profit margin = Grossprofit/Net sales = $4,127,429/$13,144,680 = 0.3140, or 31.40% Operating profit margin = EBIT/Net sales = $2,586,150/$13,144,680 = 0.1967, or19.67% Net profit margin = Net income/Net sales = $867,555/$13,144,680 = 0.0660, or 6.60% LO: 3 Level: Intermediate Bloomcode: Application AASCB: Analytic IMA: FSA AICPA: Measurement
4.5
National City Bank has 646,749,650 shares of common stock outstanding that are currently selling for $37.55 per share on the New York Stock Exchange. If National City’s net income was $2,780,955,000 in the year that just ended, what was its earnings per share and what is its current price-earnings ratio? If the typical price-earnings ratio for a bank is currently 15, what does the price-earnings ratio for National City tell you about its prospects? SM 19-137
Solution: Earnings per share(EPS) = = Price-earnings ratio =
Net income Shares outstanding $2,780,955,000 =$4.30 646,749,650
Share price EPS
$37.55 = 8.73 $4.30 The fact that the price-earnings ratio for National City of 8.73 is well below the typical value for a bank tells us that the market places a smaller value on each current dollar of earnings at National City than on each current dollar of earnings at the typical bank. This suggests that the market is less optimistic about National City’s prospects than it is about the prospects of the typical bank. =
LO: 3 Level: Intermediate Bloomcode: Analysis AASCB: Analytic IMA: FSA AICPA: Measurement
Chapter 5 The Time Value of Money Before You Go On Questions and Answers Section 5.1 1.
Why is a dollar today worth more than a dollar one year from now? A dollar is worth more today than one year from now, due to its potential earning capacity. If you have the money in your hand today, you have the opportunity to invest it and earn interest or you can purchase goods and services for your immediate consumption. Given that people have a positive preference for consumption, time value of money holds true. SM 19-138
2.
What is a time line, and why is it important in financial analysis? A time line is a horizontal line that starts at time zero (today) and shows cash flows as they occur over time. It is an important tool used to analyze cash flows over certain time periods, as timing of each cash flow has a big impact on the final figure, and therefore on the resulting investment decision.
Section 5.2 1.
What is compounding, and how does it affect the future value of an investment? Compounding is the process that refers to converting the initial (principal) amount into a future value. In order to obtain the future value of the principal amount, you calculate what the value at the end of the time period will be assuming the initial investment will earn interest, which is reinvested and will earn additional interest in the future periods.
2.
What is the difference between simple interest and compound interest? The difference is the interest earned on interest.
3.
How does changing the compounding period affect the amount of interest earned on an investment? The more frequent the compounding schedule, the higher the interest earned. For example, $100 invested for one year at 10 percent compounded annually will earn you $10 of interest at the end of the year, but if your bank compounded interest quarterly, your earnings from interest would increase to $10.38.
Section 5.3 1.
What is the present value, and when is it used? Present value is the amount a future sum is worth today given a certain return rate. The present value concept should be used when calculating how much money you need today in order to reach your financial goal sometime in the future.
2.
What is the discount rate? How does the discount rate differ from the interest rate in the future value equation? The discount rate is the compound interest rate used to determine the present value of future cash flows. Both discount and interest rates essentially represent the same concept. The only difference is the context in which they are used.
3.
What is the relation between the present value factor and the future value factor?
SM 19-139
The present value factor is the reverse of the future value factor. To obtain the present value factor, you divide 1 by the future value factor (1 + i). 4.
Explain why you would expect the discount factor to become smaller when based on the longer the time to payment. The discount factor will become smaller the longer the time to payment due to time value of money. The longer you have to wait to obtain the money, the less value it will have to you. Mathematically, the discount factor is calculated as 1/(1 + i)n. The longer the time to payment, the larger n gets, which will make the discount factor smaller.
Section 5.4 1.
What is the difference between the interest rate (i) and the growth rate (g) in the future value equation? The interest rate and the growth rate in the future value equation essentially represent the same concept. The growth rate is used when we deal with numerical values such as sales or change over time. When referring to money being invested, we use the term interest rate.
Self-Study Problems and Solutions 5.1
Amit Patel is planning to invest $10,000 in a bank certificate of deposit (CD) for five years. The CD will pay interest of 9 percent. What is the future value of Amit’s investment? Solution: Present value of the investment = PV = $10,000 Interest rate = i = 9% Number of years = n = 5. 0 1 2 3 4 5 Years ├───┼───┼───┼────┼───┤ -$10,000 FV5=? FVn PV(1 i)n FV5 $10, 000(1 0.09)5 $15, 386.24
SM 19-140
5.2
Megan Gaumer expects to need $50,000 for a down payment on a house in six years. How much does she need to invest today in an account paying 7.25 percent in order to have $50,000 in six years? Solution: Amount Megan will need in six years = FV6 = $50,000 Number of years = n = 6 Interest rate = i = 7.25% Amount needed to be invested now = PV = ? 0 1 2 3 4 5 6 Year ├───┼───┼───┼────┼───┼───┤ PV = ? FV6 = $50,000
PV = =
FV𝑛 (1+i)𝑛 $50,000
(1+0.0725)6
= $32,853.84 5.3
Kelly Martin has $10,000 that she can deposit into a savings account for five years. Bank A pays compounds interest annually, Bank B twice a year, and Bank C quarterly. Each bank has a stated interest rate of 4 percent. What amount would Kelly have at the end of the fifth year if she left all the interest paid on the deposit in each bank? Solution: Present value of Kelly’s deposit = PV = $10,000 Number of years = n = 5 Interest rate = i = 4% Compound period m: A=1 B=2 C=4 Amount at the end of 5 years = FV5 = ? 0 4% 1 2 3 4 5 Year ├───┼───┼───┼────┼───┤ -$10,000 FV5 = ? SM 19-141
A:
FVn FV5
= PV × (1 + i/m)m x n = 10,000 × (1 + 0.04/1)1x5 = $12,166.53
B:
FV5
= 10,000 × (1 + 0.04/2)2x5 = $12,189.94
C:
FV5
= 10,000 × (1 + 0.04/4)4x5 = $12,201.90
5.4
You have an opportunity to invest $2,500 today and receive $3,000 in three years. What would be the return on your investment if you accepted this opportunity? Solution: Your investment today = PV = $2,500 Amount to be received = FV3= $3,000 Time of investment = n = 3 Return on the investment = i = ? 0i=? 1 2 3 Year ├───┼────┼───┤ -$2,500 $3,000 FV𝑛 = PV × (1 + i)𝑛 $3,000 = $2,500 × (1 + i)3 $3,000 $2,500
= (1 + i)3
i = 6.27%
5.5
Emily Smith deposits $1,200 in her bank today. If the bank pays 4 percent simple interest, how much money will she have at the end of five years? What if the bank pays compound interest? How much of the earnings will be interest on interest? Solution: Emily’s deposit today= PV = $1,200 Interest rate = i = 4% Number of years = n = 5 Amount to be received = FV5 = ? SM 19-142
0 4% 1 2 3 4 5 Year ├───┼───┼───┼────┼───┤ -$1.200 FV5 = ? a. Future value with simple interest Simple interest per year = $1,200 × 0.04 = $48 Simple interest for 5 years = $48 × 5 = $240 FV5 = $1,200 + $240 = $1,440 b. Future value with compound interest FV5 = $1,200 × (1 + 0.04)5 = $1,459.98 Simple interest = ($1,440 – $1,200) = $240 Interest on interest = $1,459.98 – $1,200 – $240 = $19.98
SM 19-143
Discussion Questions 5.1
Explain the phrase ―a dollar today is worth more than a dollar tomorrow.‖ The implication is that if one was to receive a dollar today instead of in the future, the dollar could be invested and will be worth more than a dollar tomorrow because of the interest earned during that one day. This makes it more valuable than receiving a dollar tomorrow.
LO: 1 Level: Basic Bloomcode: Knowledge AASCB: Analytic IMA: Quantitative Methods AICPA: Measurement 5.2
Explain the importance of a time line. Time lines are important tools used to analyze investments that involve cash flow streams over a period of time. They are horizontal lines that start at time zero (today) and show cash flows as they occur over time. Because of time value of money, it is crucial to keep track of not only the size, but also the timing of the cash flows.
LO: 1 Level: Basic Bloomcode: Comprehension AASCB: Analytic IMA: Quantitative Methods AICPA: Measurement 5.3
What are the two factors to be considered in time value of money? The factors that are critical in time value of money are the size of the cash flows and the timing of the cash flows.
LO: 1 Level: Basic Bloomcode: Knowledge AASCB: Analytic IMA: Quantitative Methods AICPA: Measurement 5.4
Explain the difference between future value and present value. Future value measures what one or more cash flows are worth at the end of a specified period, while present value measures what one or more cash flows that are to be received in the future will be worth today (at t = 0). LO: 2, 3 Level: Basic SM 19-144
Bloomcode: Analysis AASCB: Analytic IMA: Quantitative Methods AICPA: Measurement 5.5
Explain the difference between compounding and discounting. The process of converting an amount given at the present time into a future value is called compounding. It is the process of earning interest over time. Discounting is the process of converting future cash flows to what its present value is. In other words, present value is the current value of the future cash flows that are discounted at an appropriate interest rate. LO: 2, 3 Level: Basic Bloomcode: Analysis AASCB: Analytic IMA: Quantitative Methods AICPA: Measurement 5.6
Explain how compound interest differs from simple interest. Suppose I invest $100 for three years at a rate of 10 percent. Simple interest would imply that I will earn $10 for each of the three years for a total of $30 interest. At the end of three years I would have $130. Compound interest recognizes that the interest earned in years 1 and 2 will also earn interest over the remaining period. Thus, the $10 earned in the first year would earn interest at 10 percent for the next two years, and the $10 earned in the second year would earn interest for the third year. Thus the total amount that I would have at the end of three years would be: $100(1.10)3 $133.10 . By compounding, I have earned an additional interest of $3.10. The total interest or compound interest is the $33.10 earned on the $100 invested, while the simple interest earned is equal to $30.
LO: 2 Level: Intermediate Bloomcode: Analysis AASCB: Analytic IMA: Quantitative Methods AICPA: Measurement 5.7
If you were given a choice between investing in a savings account that paid quarterly interest and one that paid monthly interest, which one should you choose if they both offered the same stated interest rate and why? The impact of compounding really dictates that one should pick the account that pays interest more frequently (as long as the interest rates are the same). This allows for the interest earned in the earlier periods to earn interest and the investment to grow more. SM 19-145
LO: 4 Level: Basic Bloomcode: Analysis AASCB: Analytic IMA: Quantitative Methods AICPA: Measurement 5.8
Compound growth is exponential over time. Explain. Growth rates, as well as interest rates, are not linear, but rather exponential over time. In other words, the growth rate of the invested funds is accelerated by the compounding of interest. Over time, the principal amount you receive interest on will get larger with compounding, thus generating higher interest payments.
LO: 4 Level: Basic Bloomcode: Comprehension AASCB: Analytic IMA: Quantitative Methods AICPA: Measurement 5.9
What is the Rule of 72? This is a rule of thumb to determine how fast an investment can double. It is a rule that allows you to closely approximate the time that it would take to double your money. It works well with interest rates between 5 and 20 percent, but varies more with higher rates. The Rule of 72 says that the time to double your money (TDM) approximately equals 72/i, where i is the rate of return expressed as a percentage.
LO: 4 Level: Basic Bloomcode: Knowledge AASCB: Analytic IMA: Quantitative Methods AICPA: Measurement 5.10
You are planning to take a spring break trip to Cancun your senior year. The trip is exactly two years away, but you want to be prepared and have enough money when the time comes. Explain how you would determine the amount of money you will have to save in order to pay for the trip. First, determine how much money you will need for the trip. Second, check how much you already have and how it translates into future value cash—how much it will be worth in two years. Next, determine how much you will have to deposit today, given the bank’s offered interest rate, to ensure that you will have saved up the difference when the time for your senior spring break comes.
LO: 4 SM 19-146
Level: Intermediate Bloomcode: Application AASCB: Analytic IMA: Quantitative Methods AICPA: Measurement
SM 19-147
Questions and Problems BASIC 5.1
Future value: Chuck Tomkovick is planning to invest $25,000 today in a mutual fund that will provide a return of 8 percent each year. What will be the value of the investment in 10 years?
Solution: 0 10 years ├────────────────────┤ PV = -$25,000 FV10 = ? Amount invested today = PV = $25,000 Return expected from investment = i = 8% Duration of investment = n = 10 years Value of investment after 10 years = FV10 FV10 PV (1 i)n $25,000 (1.08)10 $53,973.12
LO 2 Bloomcode: Application AASCB: Analytic IMA: Quantitative Methods AICPA: Measurement
5.2
Future value: Ted Rogers is investing $7,500 in a bank CD that pays a 6 percent annual interest. How much will the CD be worth at the end of five years? Solution: 0 5 years ├────────────────────┤ PV = $7,500 FV5 = ? Amount invested today = PV = $7,500 Return expected from investment = i = 6% Duration of investment = n = 5 years Value of investment after 5 years = FV5 FV5 PV (1 i)n $7,500 (1.06)5 $10,036.69 SM 19-148
LO 2 Bloomcode: Application AASCB: Analytic IMA: Quantitative Methods AICPA: Measurement
5.3
Future value: Your aunt is planning to invest in a bank CD that will pay 7.5 percent interest compounded semiannually. If she has $5,000 to invest, how much will she have at the end of four years? Solution: 0 4 years ├────────────────────┤ PV = $5,000 FV4 = ? Amount invested today = PV = $5,000 Return expected from investment = i = 7.5% Duration of investment = n = 4 years Frequency of compounding = m = 2 Value of investment after 4 years = FV4 24 mn 0.075 i FV4 PV 1 $5, 000 1 m 2 $5, 000 (1.0375)8
$6, 712.35
LO 2 Bloomcode: Application AASCB: Analytic IMA: Quantitative Methods AICPA: Measurement
5.4
Future value: Kate Eden received a graduation present of $2,000 that she is planning on investing in a mutual fund that earns 8.5 percent each year. How much money will she have in three years? Solution: 0 3 years ├────────────────────┤ PV = $2,000 FV3 = ? Amount Kate invested today = PV = $2,000
SM 19-149
Return expected from investment = i = 8.5% Duration of investment = n = 3 years Value of investment after 3 years = FV3 FV3 PV (1 i)n $2,000 (1.085)3 $2,554.58
LO 2 Bloomcode: Application AASCB: Analytic IMA: Quantitative Methods AICPA: Measurement
5.5
Future value: Your bank pays 5 percent interest semiannually on your savings account. You don’t expect to add to the current balance of $2,700 over the next four years. How much money can you expect to have at the end of this period? Solution: 0 4 years ├────────────────────┤ PV = -$2,700 FV4 = ? Amount invested today = PV = $2,700 Return expected from investment = i = 5% Duration of investment = n = 4 years Frequency of compounding = m = 2 Value of investment after 4 years = FV4 24 mn 0.05 i FV4 PV 1 $2,700 1 m 2 8 $2,700 (1.025)
$3,289.69
LO 2 Bloomcode: Application AASCB: Analytic IMA: Quantitative Methods AICPA: Measurement
5.6
Future value: Your birthday is coming up and instead of other presents, your parents promised to give you $1,000 in cash. Since you have a part time job and thus don’t need SM 19-150
the cash immediately, you decide to invest the money in a bank CD that pays 5.2 percent, compounded quarterly, for the next two years. How much money can you expect to earn in this period of time? Solution: 0 2 years ├────────────────────┤ PV = -$1,000 FV2 = ? Amount invested today = PV = $1,000 Return expected from investment = i = 5.2% Duration of investment = n = 2 years Frequency of compounding = m = 4 Value of investment after 2 years = FV2 42 mn 0.052 i FV2 PV 1 $1,000 1 4 m $1,000 (1.013)8 $1,108.86
LO 2 Bloomcode: Application AASCB: Analytic IMA: Quantitative Methods AICPA: Measurement
5.7
Multiple compounding periods: Find the future value of a five-year $100,000 investment that pays 8.75 percent and that has the following compounding periods: a. Quarterly. b. Monthly. c. Daily. d. Continuous. Solution: 0 5 years ├────────────────────┤ PV = -$100,000 FV5 = ?
Amount invested today = PV = $100,000 Return expected from investment = i = 8.75% Duration of investment = n = 5 years
SM 19-151
a.
Frequency of compounding = m = 4 Value of investment after 5 years = FV5
45
mn 0.0875 FV5 PV 1 i $100,000 1 m 4 20 $100,000 (1.021875) $154,154.24
b.
Frequency of compounding = m = 12 Value of investment after 5 years = FV5
125
mn 0.0875 FV5 PV 1 i $100,000 1 12 m 60 $100,000 (1.00729) $154,637.37
c.
Frequency of compounding = m = 365 Value of investment after 5 years = FV5
3655
mn 0.0875 FV5 PV 1 i $100,000 1 365 m 1825 $100,000 (1.00024) $154,874.91
d.
Frequency of compounding = m = Continuous Value of investment after 5 years = FV5 FV5 PV ein $100,000 e0.08755 $100,000 1.5488303 $154,883.03
LO 2 Bloomcode: Application AASCB: Analytic IMA: Quantitative Methods AICPA: Measurement
5.8
Growth rates: Joe Mauer, a catcher for the Minnesota Twins, is expected to hit 15 home runs in 2014. If his home-run-hitting ability is expected to grow by 12 percent every year for the following five years, how many home runs is he expected to hit in 2019? Solution: 0 5 years ├────────────────────┤ PV = -15 FV5 = ? SM 19-152
Number of home runs hit in 2014 = PV = 15 Expected annual increase in home runs hit = i = 12% Growth period = n = 5 years Expected home runs in 2019 = FV5 FV5 PV (1 g)n 15 (1.12)5 26.4 26 home runs
LO 4 Bloomcode: Application AASCB: Analytic IMA: Quantitative Methods AICPA: Measurement
5.9
Present value: Roy Gross is considering an investment that pays 7.6 percent, compounded annually. How much will he have to invest today so that the investment will be worth $25,000 in six years? Solution: 0 6 years ├────────────────────┤ PV = ? FV6 = $25,000 Value of investment after 6 years = FV6 = $25,000 Return expected from investment = i = 7.6% Duration of investment = n = 6 years Amount to be invested today = PV FVn $25,000 PV n (1 i) (1.076)6 $16,108.92
LO 3 Bloomcode: Application AASCB: Analytic IMA: Quantitative Methods AICPA: Measurement
SM 19-153
5.10
Present value: Maria Addai has been offered a future payment of $750 two years from now. If she can earn 6.5 percent, compounded annually, on her investment, what should she pay for this investment today? Solution: 0 2 years ├────────────────────┤ PV = ? FV2 = $750 Value of investment after 2 years = FV2 = $750 Return expected from investment = i = 6.5% Duration of investment = n = 2 years Amount to be invested today = PV $750 FVn PV n 1 i (1.065)2 $661.24
LO 3 Bloomcode: Application AASCB: Analytic IMA: Quantitative Methods AICPA: Measurement
5.11
Present value: Your brother has asked you for a loan and has promised to pay you $7,750 at the end of three years. If you normally invest to earn 6 percent per year, how much will you be willing to lend to your brother if you view this purely as a financial transaction (i.e. you don’t give your brother a special deal)? Solution: 0 3 years ├────────────────────┤ PV = ? FV3 = $7,750 Loan repayment amount after 3 years = FV3 = $7,750 Return expected from investment = i = 6% Duration of investment = n = 3 years Amount to be invested today = PV $7,750 FVn PV n 1 i (1.06)3 $6,507.05 SM 19-154
LO 3 Bloomcode: Application AASCB: Analytic IMA: Quantitative Methods AICPA: Measurement
5.12
Present value: Tracy Chapman is saving to buy a house in five years. She plans to put 20 percent down at that time, and she believes that she will need $35,000 for the down payment. If Tracy can invest in a fund that pays 9.25 percent annually, how much will she need to invest today? Solution: 0 5 years ├────────────────────┤ PV = ? FV5 = $35,000 Amount needed for down payment after 5 years = FV5 = $35,000 Return expected from investment = i = 9.25% Duration of investment = n = 5 years Amount to be invested today = PV $35,000 FVn PV n 1 i (1.0925)5 $22,488.52
LO 3 Bloomcode: Application AASCB: Analytic IMA: Quantitative Methods AICPA: Measurement
5.13
Present value: You want to buy some bonds that will have a value of $1,000 at the end of seven years. The bonds pay 4.5 percent interest annually. How much should you pay for them today? Solution: 0 7 years ├────────────────────┤ SM 19-155
PV = ?
FV7 = $1,000
Face value of bond at maturity = FV7 = $1,000 Appropriate discount rate = i = 4.5% Number of years to maturity = n = 7 years. Present value of bond = PV $1,000 FVn PV n 1 i (1.045)7 $734.83
LO 3 Bloomcode: Application AASCB: Analytic IMA: Quantitative Methods AICPA: Measurement
5.14
Present value: Elizabeth Sweeney wants to accumulate $12,000 by the end of 12 years. If the annual interest rate is 7 percent, how much will she have to invest today to achieve her goal? Solution: 0 12 years ├────────────────────┤ PV = ? FV12 = $12,000 Amount Ms. Sweeney wants at end of 12 years = FV12 = $12,000 Interest rate on investment = i = 7% Duration of investment = n = 12 years. Present value of investment = PV $12,000 FVn PV n 1 i (1.07)12 $5,328.14
LO 3 Bloomcode: Application AASCB: Analytic IMA: Quantitative Methods AICPA: Measurement
SM 19-156
5.15
Interest rate: You are in desperate need of cash and turn to your uncle, who has offered to lend you some money. You decide to borrow $1,300 and agree to pay back $1,500 in two years. Alternatively, you could borrow from your bank that is charging 6.5 percent interest annually. Should you go with your uncle or the bank? Solution: 0 2 years ├────────────────────┤ PV = -$1,300 FV2 = $1,500 Amount to be borrowed = PV = $1,300 Amount to be paid back after 2 years = FV2 = $1,500 Interest rate on investment = i = ? Duration of investment = n = 2 years. Present value of investment = PV FVn PV 1 i n $1,500 $1,300 (1 i) 2 $1,500 1.1538 (1 i)2 $1,300 i 1.1538 1 i 7.42% You should borrow from the bank.
LO 2, 3 Bloomcode: Application AASCB: Analytic IMA: Quantitative Methods AICPA: Measurement
5.16
Number of periods: You invest $150 in a mutual fund today that pays 9 percent interest annually. How long will it take to double your money? Solution: 0 n years ├────────────────────┤ PV = -$150 FVn = $300 SM 19-157
Value of investment today = PV = $150 Interest on investment = n = 9% Future value of investment = FV = $300 Number of years to double investment = n FVn PV (1 i) n $300 $150 (1.09)n (1.09)n $300 150 2.00 n ln(1.09) ln(2.00) n
ln(2.00)
8 years
ln(1.09)
LO 2, 3 Bloomcode: Application AASCB: Analytic IMA: Quantitative Methods AICPA: Measurement
INTERMEDIATE 5.17
Future value: Your finance textbook sold 53,250 copies in its first year. The publishing company expects the sales to grow at a rate of 20 percent each year for the next three years and by 10 percent in the fourth year. Calculate the total number of copies that the publisher expects to sell in years 3 and 4. Draw a time line to show the sales level for each of the next four years. Solution: Number of copies sold in its first year = PV = 53,250 Expected annual growth in the next 3 years = g = 20% Number of copies sold after 3 years = FV3 = FVn = PV (1 g)n 53, 250 (1.20)3 92, 016 copies Number of copies sold in the fourth year = FV4 FVn PV (1 g)n 92, 016 (1.10) 101, 218 copies SM 19-158
0 3 4 years ├───────────┼────────┤ PV = -53,250 92,016 101,218 copies
LO 4 Bloomcode: Application AASCB: Analytic IMA: Quantitative Methods AICPA: Measurement
5.18
Future Value: CelebNav, Inc., had sales last year of $700,000, and the analysts are predicting a good year for the start-up, with sales growing 20 percent a year for the next three years. After that, the sales should grow 11 percent per year for two years, at which time the owners are planning to sell the company. What are the projected sales for the last year before the sale? Solution: 0 1 2 3 4 5 years ├───────┼────────┼───────┼────────┼───────┤ g1 = 20% g2 = 11% PV = -$700,000 FV5=? Sales of CelebNav last year = PV = $700,000 Expected annual growth in the next 3 years = g1 = 20% Expected annual growth in years 4 and 5 = g2= 11% Sales in year 5 = FV5 FV PV (1 g )3 (1 g )2 $700, 000 (1.20)3 (1.11)2 5
1
2
$1, 490, 348.16
LO 4 Bloomcode: Application AASCB: Analytic IMA: Quantitative Methods AICPA: Measurement
5.19
Future value: You decide to take advantage of the current online dating craze and start your own Web site. You know that you have 450 people who will sign up immediately and, through a careful marketing research and analysis, determine that membership can SM 19-159
grow by 27 percent in the first two years, 22 percent in year 3, and 18 percent in year 4. How many members do you expect to have at the end of four years? Solution: 0 1 2 3 4 years ├───────┼────────┼───────┼────────┤ g1-2=27% g3=22% g4=18% PV = -450 FV4 = ? Number of Web site memberships at t = 0 = PV = 450 Expected annual growth in the next 2 years = g1-2 = 27% Expected annual growth in years 3 = g3= 22% Expected annual growth in years 4 = g4= 18% Number of members in year 4 = FV4 FV PV(1 g )2 (1 g ) (1 g ) 450 (1.27)2 (1.22) (1.18) 4
1
3
4
1, 045 members
LO 4 Bloomcode: Application AASCB: Analytic IMA: Quantitative Methods AICPA: Measurement
5.20
Multiple compounding periods: Find the future value of an investment of $2,500 made today for the following rates and periods: a. 6.25 percent compounded semiannually for 12 years b. 7.63 percent compounded quarterly for 6 years c. 8.9 percent compounded monthly for 10 years d. 10 percent compounded daily for 3 years e. 8 percent compounded continuously for 2 years Solution: 212 0.0625 FV12 PV 1 $2,500 (2.0928) a. 2 $5,232.09 b.
FV6 = PV × (1 +
0.0763 4×6 4
)
= $2,500 × (1.5738)
= $3,934.48
SM 19-160
0.089 12×10
c.
d.
e.
FV10 = PV × (1 + 12 ) = $6,067.86
= $2,500 × (2.4271)
0.010 365×3 = $2,500 × (1.3498) FV3 = PV × (1 + 365 ) = $3,374.51
FV2 = PV × ein = $3,000 × e0.08 X 2 = $2,500 × 1.1735 = $2,933.78
LO 2 Bloomcode: Application AASCB: Analytic IMA: Quantitative Methods AICPA: Measurement
5.21
Multiple compounding periods: Find the present value of $3,500 under each of the following rates and periods. a. 8.9% compounded monthly for five years. b. 6.6% compounded quarterly for eight years. c. 4.3% compounded daily for four years. d. 5.7% compounded continuously for three years. Solution: 0 n years ├────────────────────┤ PV = ? FVn = $3,500 a. Return expected from investment = i = 8.9% Duration of investment = n = 5 years Frequency of compounding = m = 12 Present value of amount = PV FV5 $3,500 125 PV mn i 0.089 12 1 1 m $3,500 $2,246.57 1.5579 b. Return expected from investment = i = 6.6% Duration of investment = n = 8 years Frequency of compounding = m = 4 SM 19-161
Present Value of amount = PV
c.
d.
FV8 $3,500 48 PV mn i 0.066 1 1 m 4 $3,500 $2,073.16 1.6882 Return expected from investment = i = 4.3% Duration of investment = n = 4 years Frequency of compounding = m = 365 Present Value of amount = PV FV4 $3,500 3654 PV i mn 0.043 365 1 1 m $3,500 $2,946.96 1.1877 Return expected from investment = i = 5.7% Duration of investment = n = 3 years Frequency of compounding = m = Continuous Present value of amount = PV FV $3,500 PV in 3 0.0573 e e
$3,500
$2, 949.88
1.1865
LO 3 Bloomcode: Application AASCB: Analytic IMA: Quantitative Methods AICPA: Measurement
5.22
Multiple compounding periods: Samantha plans to invest some money so that she has $5,500 at the end of three years. How much should she invest today given the following choices: a. 4.2 percent compounded daily b. 4.9 percent compounded monthly c. 5.2 percent compounded quarterly d. 5.4 percent compounded annually Solution:
SM 19-162
a.
b.
0 3 years ├────────────────────┤ PV = ? FV3 = $5,500 Return expected from investment = i = 4.2% Duration of investment = n = 3 years Frequency of compounding = m = 365 Present value of amount = PV FV3 $5,500 3653 PV i mn 0.042 365 1 1 m $5,500 $4,848.92 1.1343 Samantha should invest $4,848.92 today to reach her target of $5,500 in three years. Return expected from investment = i = 4.9% Duration of investment = n = 3 years Frequency of compounding = m = 12 Present value of amount = PV FV3
PV = (1 + i )mn = m
=
c.
d.
$5,500
0.049 12 ×3
(1 + 12 )
$5,500 1.1580
= $4,749.54
Samantha should invest $4,749.54 today to reach her target of $5,500 in three years. Return expected from investment = i = 5.2% Duration of investment = n = 3 years Frequency of compounding = m = 4 Present Value of amount = PV FV3 $5,500 43 PV mn i 0.052 1 1 m 4 $5,500 $4,710.31 1.1677 Samantha should invest $4,710.31 today to reach her target of $5,500 in three years. Return expected from investment = i = 5.4% Duration of investment = n = 3 years Frequency of compounding = m = 1 Present value of amount = PV
SM 19-163
FV3 $5,500 $4,697.22 3 (1 i) (1.054)3 Samantha should invest $4,697.22 today to reach her target of $5,500 in three years. Samantha should invest in choice D. PV
LO 3 Bloomcode: Application AASCB: Analytic IMA: Quantitative Methods AICPA: Measurement
Time to grow: Zephyr Sales Company has sales of $1.125 million. If the company’s management expects sales to grow 6.5 percent annually, how long will it be before sales doubles? Use a financial calculator to solve this problem. Solution: Enter 6.5% -$1.125 $2.250 N g% PMT PV FV Answer: 11 years 5.23
LO 4 Bloomcode: Analysis AASCB: Analytic IMA: Quantitative Methods AICPA: Measurement
5.24
Time to grow: You are able to deposit $850 in a bank CD today, and you will withdraw the money only once the balance is $1,000. If the bank pays 5 percent interest, how long will it take for the balance to reach $1,000? Solution: Amount invested today = PV = $850 Expected amount in the future = FV = $1,000 Interest rate on CD = i = 5% To calculate the time needed to reach the target FV, we set up the future value equation.
SM 19-164
FVn PV(1 i) n $1,000 $850 (1.05)n $1,000 (1.05)n 1.1764 $850 n ln(1.05) ln(1.1764) ln(1.1764) n 3.3 years ln(1.05)
LO 2, 3 Bloomcode: Analysis AASCB: Analytic IMA: Quantitative Methods AICPA: Measurement
5.25
Time to grow: Neon Lights Company is a private company with sales of $1.3 million a year. Management wants to take the company public but has to wait until the sales reach $2 million. If the sales are expected to grow 12 percent annually, when is the earliest that Neon Lights will go public? Solution: Current level of sales = PV = $1,300,000 Target sales level in the future = FV = $2,000,000 Projected growth rate = g = 12% To calculate the time needed to reach the target FV, we set up the future value equation. FV3 = PV × (1 + 𝑔)𝑛 $2,000,000 = $1,300,000 × (1.12)𝑛 $2,000,000 (1.12)𝑛 = = 1.5385 $1,300,000 n × ln(1.12) = ln(1.5385) n=
ln(1.5385) ln(1.12)
= 3.8 years
LO 4 Bloomcode: Analysis AASCB: Analytic IMA: Quantitative Methods AICPA: Measurement SM 19-165
5.26
Time to grow: You have just inherited $550,000. You plan to save this money and continue to live off the money that you are earning in your current job. If you can invest the money in a bond that pays 4.6 percent interest annually, how long will it be before your inheritance is worth $1 million? Solution: FV𝑛 = PV × (1 + i)𝑛 $1,000,000 = $550,000 × (1.046)𝑛 $1,000,000 $550,000
= (1.046)𝑛
$1,000,000 ln ( $550,000 ) = n × ln(1.046) ln(
n=
n=
$1,000,000 ) $550,000
ln(1.046) $0.59784 $0.04497
n = 13.29 years
LO 2, 3 Bloomcode: Analysis AASCB: Analytic IMA: Quantitative Methods AICPA: Measurement
5.27
Growth rates: Xenix Corp had sales of $353,866 in 2014. If management expects its sales to be $476,450 in three years, what is the rate at which the company’s sales are expected to grow? Solution: Sales in 2014 = PV = $353,866 SM 19-166
Expected sales three years from now = $476,450 To calculate the expected sales growth rate, we set up the future value equation. FV3 PV (1 g) 3 $476, 450 $353,866 (1 g)3 (1 g)3
$476, 450
1.3464
$353,866 1
g (1.3464) 3 1 10.42%
LO 4 Bloomcode: Application AASCB: Analytic IMA: Quantitative Methods AICPA: Measurement
5.28
Growth rate: Infosys Technologies, Inc., an Indian technology company, reported net income of $419 million this year. Analysts expect the company’s earnings to be $1.468 billion in five years. What is the expected growth rate in the company’s earnings? Solution: Earnings in current year = PV = $419,000,000 Expected earnings five years from now = $1,468,000,000 To calculate the expected earnings growth rate, we set up the future value equation. FV5 PV (1 g) 5 $1, 468, 000, 000 $419, 000, 000 (1 g)5 (1 g)5
$1, 468, 000, 000 3.5036 $419, 000, 000 1
g (3.5036) 5 1 28.5%
LO 4 Bloomcode: Application AASCB: Analytic IMA: Quantitative Methods AICPA: Measurement
SM 19-167
5.29
Present value: Caroline Weslin needs to decide whether to accept a bonus of $1,820 today or wait two years and receive $2,100 then. She can invest at 6 percent. What should she do? Solution: 0 2 years ├────────────────────┤ PV = -$1,820 FV2 = ? Amount to be received in 2 years = FV2 = $2,100 Return expected from investment = i = 6% Duration of investment = n = 2 years Present value of amount today PV = FV2 $2,100 PV n (1.06)2 (1 i) $1,868.99 Since $1,869 is greater than $1,820, Caroline should wait two years unless she needs the money sooner.
LO 3 Bloomcode: Analysis AASCB: Analytic IMA: Quantitative Methods AICPA: Measurement
5.30. Present value: Congress and the President have decided to increase the federal tax rate in an effort to reduce the budget deficit. Suppose that Caroline Weslin, from problem 5.29, will pay 35 percent of her bonus to the federal government for taxes if she accepts the bonus today and 40 percent if she receives her bonus in two years. Will the increase in tax rates affect her decision? Solution: Yes. It will affect her decision. If Caroline accepts the bonus today, after paying the taxes, she will have $1,820 × (1 - 0.35) = $1,183.00 left over. If she waits two years and pays the high tax rate, the present value of what she will have left over is only $1,869 × (1 - 0.40) = $1,121.40.
LO 3 Bloomcode: Analysis AASCB: Analytic IMA: Quantitative Methods AICPA: Measurement SM 19-168
ADVANCED You have $2,500 that you want to invest in your classmate’s start-up business. You believe the business idea to be great and hope to get $3,700 back at the end of three years. If all goes according to the plan, what will be the return on your investment? Solution: 0 3 years ├────────────────────┤ PV = -$2,500 FV3 = $3,700 Amount invested in project = PV = $2,500 Expected return three years from now = FV =$3,700 To calculate the expected rate of return, we set up the future value equation. FV3 PV (1 i) 3 5.31
$3,700 $2,500(1 i)3 $3,700 (1 i)3 1.4800 $2,500 1
i (1.4800) 3 1 0.1396 13.96%
LO 2, 3 Bloomcode: Application AASCB: Analytic IMA: Quantitative Methods AICPA: Measurement
5.32
Patrick Seeley has $2,400 to invest. His brother approached him with an investment opportunity that could double his money in four years. What interest rate would the investment have to yield in order for Patrick’s brother to deliver on his promise?
Solution: 0 4 years ├────────────────────┤ PV = -$2,400 FV4 = $4,800 SM 19-169
Amount invested in project = PV = $2,400 Expected return three years from now = FV =$4,800 Investment period = n = 4 years To calculate the expected rate of return, we set up the future value equation. FV4 = PV × (1 + i)4 $4,800 = $2,400 × (1 + i)4 (1 + i)4 =
$4,800
= 2.0000
$2,400
i = (2.000)1/4 – 1 = 0.1892 = 18.92%
LO 2, 3 Bloomcode: Application AASCB: Analytic IMA: Quantitative Methods AICPA: Measurement
5.33
You have $12,000 in cash. You can deposit it today in a mutual fund earning 8.2 percent compounded semiannually, or you can wait, enjoy some of it, and invest $11,000 in your brother’s business in two years. Your brother is promising you a return of at least 10 percent on your investment. Whichever alternative you choose, you will need to cash in at the end of 10 years. Assume your brother is trustworthy and that both investments carry the same risk. Which one would yield the largest amount in 10 years? Solution: Option 1: Invest in account paying 8.2 percent semiannually for 10 years. 0 10 years ├────────────────────┤ PV = -$12,000 FV10 = ? Amount invested in project = PV = $12,000 Investment period = n = 10 years Interest earned on investment = i = 8.2% Frequency of compounding = m = 2 Value of investment after 10 years = FV10 210 0.082 FV10 PV 1 $12,000 (2.23365) 2 $26,803.77 Option 2: Invest in brother’s business to earn 10 percent for eight years. 0 8 years SM 19-170
├────────────────────┤ PV = -$11,000 FV8 = ? Amount invested in project = PV = $11,000 Investment period = n = 8 years Interest earned on investment = i = 10% Frequency of compounding = m = 1 Value of investment after 8 years = FV8
FV8 PV 1 0.10 8 $11,000 (2.14359)
$23,579.48 You are better off investing today in the mutual fund and earn 8.2 percent semiannually for 10 years.
LO 2 Bloomcode: Application AASCB: Analytic IMA: Quantitative Methods AICPA: Measurement
5.34
When you were born your parents set up a bank account in your name with an initial investment of $5,000. You are turning 21 in a few days and will have access to all your funds. The account was earning 7.3 percent for the first seven years, but then the rates went down to 5.5 percent for six years. The economy was doing well from 2007 to 2009, and your account earned 8.2 percent three years in a row. Unfortunately, the next two years you earned only 4.6 percent. Finally, as the economy recovered, the return jumped to 7.6 percent for the last three years. a. How much money was in your account at the end of Year 16? b. How much money is in your account now, at the end of year 21? c. What would be the balance now if your parents made another deposit of $1,200 at the end of year 7? Solution: 0 1 7 13 14 15 16 21 years ├───┼∙∙∙∙∙∙∙∙∙∙┼∙∙∙∙∙∙∙∙∙∙∙∙────┼────┼───┼───∙∙∙∙∙∙∙∙∙∙∙∙∙∙∙──┤ PV = -$5,000 FV21 = ? i1 = 7.3% i2 = 5.5% i3 = 8.2% i4 = 4.6% i5 = 7.6% a. Initial investment = PV = $5,000 Interest rate for first 7 years = i1 = 7.3% Interest rate for next 6 years = i2 = 5.5% SM 19-171
Interest rate for next 3 years = i3 = 8.2% Investment value at age 16 years = FV16 FV16 PV (1 i1 )7 (1 i 2 )6 (1 i 3 )3 $5,000 1 0.073 7 (1.055) 6 (1.082) 3 $5,000 (1.6376) (1.3788) (1.2667) b.
$14,300.55 Interest rate for from age 17 to 18 = i4 = 4.6% Interest rate for next 3 years = i5 = 7.6% Investment at start of 16th year = PV = $14,300.95 Investment value at age 21 years = FV21 FV21 FV16 (1 i4 )2 (1 i5 )3
c.
$14,300.55 1 0.046 2 (1.076) 3 $14,300.55 (1.0941) (1.2458)) $19,492.38 Additional investment at start of 8th year = $1,200 Total investment for next 6 years = $8,187.82 + $1,200 = $9,387.82 Interest rate for next 6 years = i2 = 5.5% Interest rate for years 13 to 16 = i3 = 8.2% Interest rate for from age 17 to 18 = i4 = 4.6% Interest rate for next 3 years = i5 = 7.6% Investment value at age 21 = FV21 FV FV (1 i )6 (1 i )3 (1 i )2 (1 i )3 21
7
2
3
4
5
$9, 387.82 (1.055)6 (1.082) 3 1.046 (1.076) 3 2
$9387.82 (1.3788) (1.2667) (1.0941) (1.2458) $22, 349.16 LO 2 Bloomcode: Analysis AASCB: Analytic IMA: Quantitative Methods AICPA: Measurement
5.35
Eric Fisher, a number 1 draft pick of the Kansas City Chiefs, and his agent are evaluating three contract options. Each option offers a signing bonus and a series of payments over
SM 19-172
the life of the contract. Fisher uses a 10.25 percent rate of return to evaluate the contracts. Given the cash flows for each option, which one should he choose? Year Cash Flow Type Option A Option B Option C 0 Signing Bonus $3,100,000 $4,000,000 $4,250,000 1 Annual Salary $ 650,000 $ 825,000 $ 550,000 2 Annual Salary $ 715,000 $ 850,000 $ 625,000 3 Annual Salary $ 822,250 $ 925,000 $ 800,000 4 Annual Salary $ 975,000 $1,250,000 $ 900,000 5 Annual Salary $1,100,000 $1,000,000 6 Annual Salary $1,250,000 Solution: To decide on the best contract from Eric Fisher’s viewpoint, we need to find the present value of each option. The contract with the highest present value should be the one chosen. Option A: Discount rate to be used = i= 10.25% Present value of contract = PVA $650,000 $715,000 $822,250 $975,000 $1,100,000 $1,250,000 PVA $3,100,000 (1.1025)1 (1.1025)2 (1.1025)3 (1.1025)4 (1.1025)5 (1.1025)6 $3,100,000 $589,569 $588,232 $613,576 $659,918 $675,305 $696,047 $6,922,647 Option B: Discount rate to be used = i= 10.25% Present value of contract = PVB $825,000 $850,000 $925,000 $1,250,000 PVB $4,000,000 (1.1025)1 (1.1025)2 (1.1025)3 (1.1025)4 $4,000,000 $748,299 $699,297 $690,249 $846,049 $6,983,894 Option C: Discount rate to be used = i= 10.25% Present value of contract = PVC $550,000 $625,000 $800,000 $900,000 $1,000,000 PVC $4,250,000 (1.1025)1 (1.1025)2 (1.1025)3 (1.1025)4 (1.1025)5 $4,250,000 $498,866 $514,189 $596,972 $609,155 $613,913 $7,083,096 Option C is the best choice for Eric Fisher.
LO 3
SM 19-173
Bloomcode: Application AASCB: Analytic IMA: Quantitative Methods AICPA: Measurement
Surmec, Inc., reported sales of $2.1 million last year. The company’s primary business is the manufacture of nuts and bolts. Since this is a mature industry, analysts are confident that sales will grow at a steady rate of 7 percent per year The company’s net income equals 23 percent of sales. Management would like to buy a new fleet of trucks but can only do so once the net income reaches $620,000 a year. At the end of what year will Surmec be able to buy the trucks? What will sales and net income be in that year? Solution: Current level of sales for Surmec = PV = $2,100,000 Profit margin = 23% Net Income for the year = 0.23 x $2,100,000 = $483,000 Target profit level in the future = FV = $620,000 Projected growth rate of sales = g = 7% To calculate the time needed to reach the target FV, we set up the future value equation. 5.36
FV𝑛 = PV × (1 + g)𝑛 $620,000 = $483,000 × (1.07)𝑛 $620,000 (1.07)𝑛 = = 1.2836 $483,000 n × ln(1.07) = ln(1.2836) ln(1.2836) n= = 3.7 years ln(1.07) The company achieves its profit target during the fourth year. Sales level at end of year 4 = FV4 FVn PV (1 g)n $2,100,000 (1.07)4 $2,752,671.62
Profit for the year = $2,752,671.62 x 0.23 = $633,114.47
LO 4 Bloomcode: Application AASCB: Analytic IMA: Quantitative Methods AICPA: Measurement SM 19-174
5.37
You will be graduating in two years and are thinking about your future. You know that you will want to buy a house five years after you graduate and that you will want to put down $60,000. As of right now, you have $8,000 in your savings account. You are also fairly certain that once you graduate, you can work in the family business and earn $32,000 a year, with a 5 percent raise every year. You plan to live with your parents for the first two years after graduation, which will enable you to minimize your expenses and put away $10,000 each year. The next three years, you will have to live on your own, as your younger sister will be graduating from college and has already announced her plan to move back into the family house. Thus, you will be able to save only 13 percent of your annual salary. Assume that you will be able to earn 7.2 percent on the savings from your salary. At what interest rate will you need to invest the current savings account balance in order to achieve your goal? Hint: Draw a time line that shows all the cash flows for years 0 through 7. Remember, you want to buy a house seven years from now and your first salary will be in year 3. Solution: 0 1 2 3 4 5 6 7 ├─────┼──────┼─────┼─────┼──────┼─────┼──────┤ $10,000 $10,000 $4,586.40 $4,815,72 $5,056.48 Starting salary in year 3 = $32,000 Annual pay increase = 5% Savings in first 2 years = $10,000 Savings rate for years 3 to 7 = 13% Year 1 2 3 4 5 6 7 Salary $0 $0 $32,000 $33,600 $35,280 $37,044 $38,896 Savings $0 $0 $10,000 $10,000 $4,586.40 $4,815.72 $5,056.48 Investment rate = i = 7.2% Future value of savings from salary = FV7 FV7 = $0 + $0 + $10,000 × (1.072)4 + $10,000 × (1.072)3 + $4,586.40 x (1.072)2 = $4,815.72 X (1.072)1 + $5,056.48 x (1.072)0 = $13,206.24 + $12,319.25 + $5,270.62 + $5,162.45 + $5,056.48 = $41,015.07 Target down payment = $60,000 Amount needed to reach target = $60,000 - $41,015.07= FV = $18,984.93 Current savings balance = PV $8,000 Time to achieve target = n = 7 years.
SM 19-175
To solve for the investment rate needed to achieve target, we need to set up the future value equation: FV = PV × (1 + i)7 $18, 984.93 = $8,000 × (1 + i)7 (1 + i)7 + $18,984.98 = 2.3731 $8,000
1
i = (2.3731)7 − 1 = 1.1314 − 1 = 13.14%
LO 2 Bloomcode: Analysis AASCB: Analytic IMA: Quantitative Methods AICPA: Measurement
Sample Test Problems 5.1
Holding all else constant, what will happen to the present value of a future amount if you increase the discount rate? What if you increase the number of years? Solution: Holding all else constant, both increasing the discount rate and increasing the number of years, will decrease the present value.
LO 3 Bloomcode: Comprehension AASCB: Analytic IMA: Quantitative Methods AICPA: Measurement
5.2
Juliette Bronson anticipates needing $500,000 to start a business. If she can earn 4.5 percent compounded annually on her investments, how much money would Juliette have to invest today to have $500,000 in three years?
Solution: SM 19-176
0 3 Year 1 2 4 .5 % ├────────────────────┤ PV = ? FV3 = $500,000 Amount required at end on investment period = FV3 = $500,000 Return expected from investment = i = 4.5% Duration of investment = n = 3 years Amount to be invested today = PV $500,000 FVn PV n 1 i (1.045)3 $438,148.30
LO 3 Bloomcode: Analysis AASCB: Analytic IMA: Quantitative Methods AICPA: Measurement
5.3
Christopher Thompkins must decide how to invest $10,000 that he just inherited. What would be the future value of his investment after five years under each of the following three investment opportunities? a. 6.28% compounded quarterly. b. 6.20% compounded monthly. c. 6.12% compounded continuously.
Solution: a.
b.
c.
Interest rate on investment = i = 6.28% Frequency of compounding = m = 4 Value of investment after 5 years = FV5 i mn 0.0628 4x5 FV5 PV 1 $10,000 1 4 m 20 $10,000 (1.01570) $13,655.55 Interest rate on investment = i = 6.20% Frequency of compounding = m = 12 Value of investment after 5 years =mnFV5 125 i 0.0620 FV5 PV 1 $10, 0001 m 12 60 $10, 000 (1.00517) $13, 623.37 Interest rate on investment = i = 6.12% SM 19-177
Frequency of compounding = m = Continuous Value of investment after 5 years = FV5 FV5 PV ein $10, 000 e0.06125 $10, 000 1.358 $13, 579.82
LO 2 Bloomcode: Analysis AASCB: Analytic IMA: Quantitative Methods AICPA: Measurement
5.4.
Tina DeLeon deposited $2,500 today in an account paying 6 percent interest annually. What would be the simple interest earned on this investment in five years? With annual compounding, how much interest-on-interest would Tina earn in five years?
Solution: 0 5 years 1 2 3 4 ├────────────────────┤ PV = -$2,500 FV5 = ? Amount invested today = PV = $2,500 Return expected from investment = i = 6% Duration of investment = n = 5 years Frequency of compounding = m = 1 Value of investment after 5 years = FV5 FV5 PV (1 i) n $2,500(1.06)5 $3,345.56 Simple interest on investment = $2,500 × 0.06 × 5 = $750 Interest-on-interest = FV5 – PV – simple interest on investment Interest-on-interest = $3,345.56 – $2,500 – $750 = $95.56
LO 2 Bloomcode: Application AASCB: Analytic IMA: Quantitative Methods AICPA: Measurement
SM 19-178
5.5
The state of Texas had 42,725 active patient care physicians in 2008 and by 2012 this number had grown to 47,663. What was the compound annual growth rate (CAGR) in the number of active care physicians during this period?
Solution: 0 4 years 1 2 3 ├────────────────────┤ PV = -42,725 FV4 = 47,663 Number of physicians in 2008 = PV = 42,725 Number of physicians in 2012 = FV4 = 47,663 Years to maturity = n = 4 years. growth rate = i FVn PV n 1 i 47, 663 42, 725 (1 i)4 $47, 663 (1 i)4 1.1156 $42, 725 i (1.1156)14 1 i = 0.0277, or 2.77% The CAGR was 2.77 percent
LO 4 Bloomcode: Application AASCB: Analytic IMA: Quantitative Methods AICPA: Measurement
Chapter 6 Discounted Cash Flows and Valuation Before You Go On Questions and Answers Section 6.1 1.
Explain how to calculate the future value of a stream of cash flows. It would helpful to first construct a time line so that we can identify the timing of each cash flow. Then you would calculate the future vale of each individual cash flow. Finally, SM 19-179
you would add up the future values of all the individual cash flows to determine the future value of the cash flow stream. 2.
Explain how to calculate the present value of a stream of cash flows. To calculate the present value of a stream of cash flows, you should first draw a time line so that you can see that each cash flow is placed in its correct time period. Then you simply calculate the present value of each cash flow for its time period, and finally you add up all the present values.
3.
Why is it important to adjust all cash flows to a common date? When making economic decisions, we need to compare ―apples to apples.‖ This is possible only when we bring all the cash flows to a common date, which can either be a present time or some future date. The reason is the time value of money: a dollar today is worth more than a dollar in the future. Thus, when cash flows are converted to the same time period, the time value of money concept holds true, and we can concentrate on the economic aspects of the decision.
Section 6.2 1.
How do an ordinary annuity, an annuity due, and a perpetuity differ? Ordinary annuity assumes that the cash flows occur at the end of a period. Most types of loans are ordinary annuities. On the other hand, annuity due is an annuity whose payment is to be made immediately (or at the beginning of a period) instead of at the end of the period. For example, in many leases the first payment is due immediately, and each successive payment must be made at the beginning of the month. Perpetuity is a special case of annuity, and it refers to a constant stream of identical cash flows with no end.
2.
Give two examples of perpetuities. The text gives the example of British government bonds called consols that have no maturity and have been traded in the markets since the end of the Napoleonic wars. Another example could be a preferred stock of a company that has no maturity and will pay a constant dividend forever.
3.
What is the annuity transformation method? The annuity transformation method refers to the conversion of an ordinary annuity to annuity due. In this process, you first plot all the cash flows on a time line as if the cash flows were an ordinary annuity. Then you calculate the present or future value factor as you would with an ordinary annuity, and finally, you multiple your answer by (1 + i). Conveniently, this relationship works for both present and future value calculations.
Section 6.3 SM 19-180
1.
What is the difference between a growing annuity and a growing perpetuity? A stream of cash flows that is growing at a constant rate over time can be called a growing annuity or growing perpetuity. If the cash flows extend over a finite length of time, then we call it a growing annuity and can use Equation6.5 to compute the present value. If the growth will continue for a very long time period and perhaps, forever, we refer to it as the growing perpetuity. We would then use Equation 6.6 to estimate the present value of this cash flow stream.
Section 6.4 1.
What is the APR, and why are lending institutions required to disclose this rate? APR, or the annual percentage rate, is the annualized interest rate using simple interest. It is defined as the simple interest charged per period multiplied by the number of compounding periods per year. Lending institutions are mandated by Federal Truth-inLending Act regulations to disclose this rate to essentially make it easier for consumers to be exposed to the same kind of rate by all businesses.
2.
What is the correct way to annualize an interest rate in financial decision making? The correct way to annualize interest rates is by computing the effective annual interest rate (EAR). This is the annual growth rate that allows for compounding, which means you earn interest on interest. To calculate the EAR, take the quoted rate and divide it by the number of compounding periods (quoted rate/m). Then take the resulting interest rate, add 1 to it, and raise it to the power equal to m. Finally, subtract 1 and the result is EAR.
3.
Distinguish between quoted interest rate, interest rate per period, and effective annual interest rate. Quoted interest rate, such as APR, is the interest rate that has been annualized by multiplying the rate per period by the number of compounding periods. Interest rate per period is the quoted rate per period. It can be stated in the form of an APR—in that case, just divide it by the number of compounding periods to obtain the interest rate per period. Finally, EAR is the annual rate of interest that accounts for the effects of compounding.
Self-Study Problems 6.1
Kronka, Inc., is expecting cash inflows of $13,000, $11,500, $12,750, and $9,635 over the next four years. What is the present value of these cash flows if the appropriate discount rate is 8 percent? Solution: The time line for the cash flows and their present value is as follows: 0 8% 1 2 3 4 Year SM 19-181
├─────────┼─────────┼─────────┼─────────┤ $13,000 $11,500 $12,750 $9,635 $13,000 $11,500 $12,750 $9,635 PV4 (1.08) (1.08)2 (1.08)3 (1.08)4 $12,037.03 $9,859.40 $10,121.36 $7,082.01 $39,099.80 LO: 4 Level: Basic
6.2
Your grandfather has agreed to deposit a certain amount of money each year into an account paying 7.25 percent annually to help you go to graduate school. Starting next year, and for the following four years, he plans to deposit $2,250, $8,150, $7,675, $6,125, and $12,345 into the account. How much will you have at the end of the five years? Solution: The time line for the cash flows and their future value is as follows: 0 7.25% 1 2 3 4 5 ├────────┼────────┼────────┼─────────┼────────┤ -$2,250 -$8,150 -$7,675 -$6,125 -$12,345
Year
FV5 = [$2,250 x (1.0725)4] + [$8,150 x (1.0725)3] + [$7,675 x (1.0725)2] + [$6,125 x 1.0725] + $12,345 = $2,976.95 + $10,054.25 + $8,828.22 + $6,569.06 + $12,345 = $40,773.48 LO: 2 Level: Basic
6.3
Mike White is planning to save up for a trip to Europe in three years. He will need $7,500 when he is ready to make the trip. He plans to invest the same amount at the end of each of the next three years in an account paying 6 percent. What is the amount he will have to save every year to reach his goal of $7,500 in three years? Solution: Amount Mike White will need in three years = FVA3 = $7,500 Number of years = n = 3 Interest rate on investment = i = 6.0% Amount needed to be invested every year = PMT = ? 0 6% 1 2 3 Year SM 19-182
├─────────┼──────┼───────┤ CF=? CF=? CF=? FVA3 = $7,500
FVAn CF
(1 i)n 1 i
$7,500 CF
(1 0.06)3 1 0.06
CF 3.1836 CF
$7,500 3.1836
$2, 355.82 Mike will have to invest $2,355.82 every year for the next three years. LO: 5 Level: Basic
6.4
Becky Scholes has $150,000 to invest. She wants to be able to withdraw $12,500 every year forever without using up any of her principal. What interest rate would her investment have to earn in order for her to be able to do so? Solution: Present value of Becky Scholes’s investment = $150,000 Amount needed annually = $12,500 This is a perpetuity! CF PVP = i CF $12,500 i= = PVP $150,000 i = 8. 33% LO: 6 Level: Basic
6.5
Dynamo Corp. is expecting annual payments of $34,225 for the next seven years from a customer. What is the present value of this annuity if the discount rate is 8.5 percent? Solution: The time line for Dynamo’s cash flows and their present value is as follows: 0 8.5% 1 2 3 4 5 6 7 Year
SM 19-183
PVA = ?
$34,225
$34,225
$34,225
$34,225 $34,225 $34,225
$34,225
1 1 (1 i)n PVA7 CF i 1 1 (1.085)7 $34, 225 0.085 $34, 225 5.1185 $175,181.14 LO: 2 Level: Basic
Discussion Questions 6.1
Identify the steps involved in computing the future value when you have multiple cash flows. First, prepare a time line to identify the size and timing of the cash flows. Second, calculate the present value of each individual cash flow using an appropriate discount rate. Finally, add up the present values of the individual cash flows to obtain the present value of a cash flow stream. This approach is especially useful in the real world where the cash flows for each period are not the same.
LO: 1 Level: Basic Bloomcode: Comprehension AASCB: Analytic IMA: Corporate Finance AICPA: Measurement 6.2
What is the key economic principle involved in calculating the present value or future value of multiple cash flows? Regardless of whether you are calculating the present value or the future value of a cash flow stream, the key idea is to discount or compound the cash flows to the same point in time.
LO: 1 Level: Basic
SM 19-184
Bloomcode: Knowledge AASCB: Analytic IMA: Corporate Finance AICPA: Measurement
6.3
What is the difference between a perpetuity and an annuity? A cash flow stream that consists of the same amount being received or paid on a periodic basis for a limited time period is called an annuity. If the same payments are made periodically forever, the contract is called a perpetuity.
LO: 2 Level: Basic Bloomcode: Analysis AASCB: Analytic IMA: Corporate Finance AICPA: Measurement
6.4
Define annuity due. Would an investment be worth more if it were an ordinary annuity or an annuity due? Explain. When annuity cash flows occur at the beginning of each period, it is called an annuity due. Annuity due will result in a bigger investment than an ordinary annuity because each cash flow will accrue an extra interest payment.
LO: 2 Level: Basic Bloomcode: Knowledge AASCB: Analytic IMA: Corporate Finance AICPA: Measurement
6.5
Raymond Bartz is trying to choose between two equally risky annuities, each paying $5,000 per year for five years. One is an ordinary annuity, the other is an annuity due. Which of the following statements is most correct? a. The present value of the ordinary annuity must exceed the present value of the annuity due, but the future value of an ordinary annuity may be less than the future value of the annuity due. b. The present value of the annuity due exceeds the present value of the ordinary annuity, while the future value of the annuity due is less than the future value of the ordinary annuity.
SM 19-185
c. The present value of the annuity due exceeds the present value of the ordinary annuity, and the future value of the annuity due also exceeds the future value of the ordinary annuity. d. If interest rates increase, the difference between the present value of the ordinary annuity and the present value of the annuity due remains the same. The answer is c. The present value of the annuity due exceeds the present value of the ordinary annuity, and the future value of the annuity due also exceeds the future value of the ordinary annuity. LO: 2 Level: Basic Bloomcode: Application AASCB: Analytic IMA: Corporate Finance AICPA: Measurement
6.6
Which of the following investments will have the highest future value at the end of three years? Assume that the effective annual rate for all investments is the same. a. You earn $3,000 at the end of three years (a total of one payment). b. You earn $1,000 at the end of every year for the next three years (a total of three payments). c. You earn $1,000 at the beginning of every year for the next three years (a total of three payments). The answer is c. Earning $1,000 at the beginning of each year for the next three years will have the highest future value as it is an annuity due.
LO: 2 Level: Intermediate Bloomcode: Application AASCB: Analytic IMA: Corporate Finance AICPA: Measurement
6.7
Explain whether or not each of the following statements is correct. a. A 15-year mortgage will have larger monthly payments than a 30-year mortgage of the same amount and same interest rate. b. If an investment pays 10 percent interest compounded annually, its effective rate will also be 10 percent. Statement A is a true statement. The 15-year mortgage will have higher monthly payments since more of the principal will have to be paid each month than in the case of a 30-year mortgage. SM 19-186
Statement B is true since the frequency of compounding is annual and hence the rate for a single period is the same as the rate for a year. LO: 2, 4 Level: Intermediate Bloomcode: Comprehension AASCB: Analytic IMA: Corporate Finance AICPA: Measurement 6.8
When will the annual percentage rate (APR) be the same as the effective annual rate (EAR)? The annual percentage rate (APR) will be the same as the effective annual rate only if the compounding period is annual, not otherwise.
LO: 4 Level: Basic Bloomcode: Comprehension AASCB: Analytic IMA: Corporate Finance AICPA: Measurement
6.9
Why is the effective annual rate (EAR) superior to the annual percentage rate (APR) in measuring the true economic cost or return? Unlike the APR, which reflects annual compounding, the EAR takes into account the actual number of compounding periods. For example, suppose there are two investment alternatives that both pay an APR of 10 percent. Assume that the first pays interest annually and that the second pays interest quarterly. It would be a mistake to assume that both investments will provide the same return. The real return on the first one is 10 percent, but the second investment actually provides a return of 10.38 percent because of the quarterly compounding. Thus, this is the superior investment.
LO: 4 Level: Basic Bloomcode: Comprehension AASCB: Analytic IMA: Corporate Finance AICPA: Measurement
6.10
Suppose three investments have equal lives and multiple cash flows. A high discount rate tends to favor: a. The investment with large cash flows early. b. The investment with large cash flows late. SM 19-187
c. The investment with even cash flows. d. None of the investments since they have equal lives. The correct answer is a. The investment with large cash flows early will be worth more compared to the one with the large cash flows late. The cash flows that come in later will have a heavier penalty when using a higher discount rate. Thus the investment with large cash flows early will be favored. LO: 4 Level: Intermediate Bloomcode: Comprehension AASCB: Analytic IMA: Corporate Finance AICPA: Measurement
Questions and Problems BASIC 6.1
Future value with multiple cash flows: Konerko, Inc., management expects the company to earn cash flows of $13,227, $15,611, $18,970, and $19,114 over the next four years. If the company uses an 8 percent discount rate, what is the future value of these cash flows at the end of year 4? Solution: 0 8% 1 2 3 4 ├───────┼────────┼───────┼────────┤ $13,227 $15,611 $18,970 $19,114
FV4 = [$13,227 × (1.08)3 ] + [$15,611 × (1.08)2 ] + [$18,970 × (1.08)1 ] + $19,114 = $16,662.21 + $18,208.67 + $20,487.60 + $19,114 = $74,502.48
LO 1 Bloomcode: Application AASCB: Analytic IMA: Corporate Finance AICPA: Measurement
SM 19-188
6.2
Future value with multiple cash flows: Ben Woolmer has an investment that will pay him the following cash flows over the next five years: $2,350, $2,725, $3,128, $3,366, and $3,695. If his investments typically earn 7.65 percent, what is the future value of the investment’s cash flows at the end of five years? Solution: 0 7.65% 1 2 3 4 5 Year ├───────┼────────┼───────┼────────┼───────┤ $2,350 $2,725 $3,128 $3,366 $3,695 FV $2, 350 (1.0765)4 $2, 725 (1.0765)3 $3,128 (1.0765)2 $3, 366 (1.0765)1 $3, 695 5 $3,155.91 $3, 399.45 $3, 624.89 $3, 623.50 $3, 695 $17, 498.75 LO 1 Bloomcode: Application AASCB: Analytic IMA: Corporate Finance AICPA: Measurement
6.3
Future value with multiple cash flows: You are a freshman in college and are planning a trip to Europe when you graduate from college at the end of four years. You plan to save the following amounts annually, starting today: $625, $700, $700, and $750. If you can earn 5.75 percent annually, how much will you have at the end of four years? Solution: 0 5.75% 1 2 3 4 Year ├───────┼────────┼───────┼────────┤ $625 $700 $700 $750 ? FV4 $625(1.0575)4 $700(1.0575)3 $700(1.0575)2 $750(1.0575) $781.63 $827.83 $782.81 793.13 $3,185.40 LO 1 Bloomcode: Application AASCB: Analytic IMA: Corporate Finance AICPA: Measurement
SM 19-189
6.4
Present value with multiple cash flows: Saul Cervantes has just purchased some equipment for his landscaping business. For this equipment he must pay the following amounts at the end of each of the next five years: $10,450, $8,500, $9,675, $12,500, and $11,635. If the appropriate , what is the cost in today’s dollars of the equipment Saul purchased today? Solution: 0 10.875% 1 2 3 4 5 Year ├───────┼────────┼───────┼────────┼───────┤ $10,450 $8,500 $9,675 $12,500 $11,635 $10,450 $8,500 $9,675 $12,500 $11,635 2 3 4 (1.10875) (1.10875) (1.10875)5 (1.10875) (1.10875) $9,425.03 $6,914.35 $7,098.23 8,271.33 $6,943.82
PV
$38,652.76 LO 1 Bloomcode: Application AASCB: Analytic IMA: Corporate Finance AICPA: Measurement 6.5
Present value with multiple cash flows: Jeremy Fenloch borrowed some money from his friend and promised to repay him $1,225, $1,350, $1,500, $1,600, and $1,600 over the next five years. If the friend normally discounts investments at 8 percent annually, how much did Jeremy borrow? Solution: 0 8% 1 2 3 4 5 Year ├───────┼────────┼───────┼────────┼───────┤ $1,225 $1,350 $1,500 $1,600 $1,600 $1,225 $1,350 $1,500 $1,600 $1,600 (1.08) (1.08)2 (1.08)3 (1.08)4 (1.08)5 $1,134.26 $1,157.41 $1,190.75 $1,176.05 $1,088.93
PV
$5,747.40 LO 1 Bloomcode: Application AASCB: Analytic IMA: Corporate Finance AICPA: Measurement
SM 19-190
6.6
Present value with multiple cash flows: Biogenesis Inc. management expects the following cash flow stream over the next five years. They discount all cash flows using a 23 percent discount rate. What is the present value of this cash flow stream?
1 2 3 4 5 Year ┼────────┼───────┼────────┼───────┤ -$1,133,676 -$978,452 $275,455 $878,326 $1,835,444 Solution: 0 23% 1 2 3 4 5 ├───────┼────────┼───────┼────────┼───────┤ -$1,133,676 -$978,452 $275,455 $878,326 $1,835,444
Year
$1,133,676 $978,452 $275,455 $878,326 $1,835,444 (1.23) (1.23)2 (1.23)3 (1.23)4 (1.23)5 $921,687.80 $646,739.37 $148,025.09 $383,738.43. $651,951.94
PV
$384,711.72 LO 1 Bloomcode: Application AASCB: Analytic IMA: Corporate Finance AICPA: Measurement
6.7
Present value of an ordinary annuity: An investment opportunity requires a payment of $750 for 12 years, starting a year from today. If your required rate of return is 8 percent, what is the value of the investment to you today? Solution: 0 8% 1 2 3 11 12 Year ├───────┼────────┼───────┼………………┼───────┤ $750 $750 $750 $750 $750 Annual payment = PMT = $750 No. of payments = n = 12 Required rate of return = 8% Present value of investment = PVA12
SM 19-191
1 n PVAn PMT (1 i) i 1 1 (1.08)12 $750 $750 7.5361 0.08 1
$5, 652.06 LO 2 Bloomcode: Application AASCB: Analytic IMA: Corporate Finance AICPA: Measurement
6.8
Present value of an ordinary annuity: Dynamics Telecommunications Corp. has made an investment in another company that will guarantee it a cash flow of $22,500 each year for the next five years. If the company uses a discount rate of 15 percent on its investments, what is the present value of this investment? Solution: 0 15% 1 2 3 4 5 ├───────┼────────┼───────┼────────┼───────┤ $22,500 $22,500 $22,500 $22,500 $22,500 Annual payment = PMT = $22,500 No. of payments = n = 5 Required rate of return = 15% Present value of investment = PVA5 1 1 (1 i)n PVAn PMT i 1 1 (1.15)5 $22,500 $22,500 3.3522 0.15 $75, 423.49 LO 2 Bloomcode: Application AASCB: Analytic IMA: Corporate Finance AICPA: Measurement
SM 19-192
6.9
Future value of an ordinary annuity: Robert Hobbes plans to invest $25,000 a year at the end of each year for the next seven years in an investment that will pay him a rate of return of 11.4 percent. How much money will Robert have at the end of seven years? Solution: 0 11.4% 1 2 3 6 7 Year ├───────┼────────┼───────┼………………┼───────┤ $25,000 $25,000 $25,000 $25,000 $25,000 Annual investment = PMT = $25,000 No. of payments = n = 7 Investment rate of return = 11.4% Future value of investment = FVA7 (1 i)n 1 FVAn PMT i (1.114)7 1 $25, 000 $25, 000 9.9044 0.114
$247, 609.95 LO 2 Bloomcode: Application AASCB: Analytic IMA: Corporate Finance AICPA: Measurement
6.10
Future value of an ordinary annuity: Cecelia Thomas is a sales executive at a Baltimore firm. She is 25 years old and plans to invest $3,000 every year in an IRA account, beginning at the end of this year until she reaches the age of 65. If the IRA investment will earn 9.75 percent annually, how much will she have in 40 years, when she turns 65? Solution: 0 9.75% 1 2 3 39 40 Year ├───────┼────────┼───────┼………………┼───────┤ $3,000 $3,000 $3,000 $3,000 $3,000 Annual investment = PMT = $3,000 No. of payments = n = 40 Investment rate of return = 9.75% Future value of investment = FVA40 SM 19-193
(1 i)n 1 FVAn PMT i (1.0975)40 1 $3, 000 $3, 000 413.5588 0.0975
$1, 240, 676.41 LO 2 Bloomcode: Application AASCB: Analytic IMA: Corporate Finance AICPA: Measurement
6.11
Future value of an annuity due: Refer to Problem 6.10. If Cecelia Thomas invests at the beginning of each year, how much will she have at age 65? Solution: 0 9.75% 1 2 3 39 40 Year ├───────┼────────┼───────┼………………┼───────┤ $3,000 $3,000 $3,000 $3,000 $3,000 Annual investment = PMT = $3,000 No. of payments = n = 40 Type of annuity = Annuity due Investment rate of return = 9.75% Future value of investment = FVA40 (1 i)n 1 FVAn PMT (1 i) i $3, 000
(1.0975)40 1 0.0975
(1.0975) $3, 000 413.5588 1.0975
$1, 361, 642.36 LO 2 Bloomcode: Application AASCB: Analytic IMA: Corporate Finance AICPA: Measurement
6.12
Computing annuity payment: Kevin Winthrop is saving for an Australian vacation in three years. He estimates that he will need $5,000 to cover his airfare and all other expenses for a week-long holiday in Australia. If he can invest his money in an S&P 500 SM 19-194
equity index fund that is expected to earn an average annual return of 10.3 percent over the next three years, how much will he have to save every year if he starts saving at the end of this year? Solution: 0 10.3% 1 2 3 Year ├───────┼────────┼───────┤ PMT PMT PMT FVAn = $5,000 Future value of annuity = FVA = $5,000 Return on investment = i = 10.3% Payment required to meet target = PMT Using the FVA equation: (1 i)n 1 FVAn PMT i
$5, 000 PMT
(1.103)3 1 0.103
PMT $5, 000 $5, 000 (1.103)3 1 3.3196 0.103 $1, 506.20 Kevin has to save $1,506.20 every year for the next three years to reach his target of $5,000. LO 2 Bloomcode: Application AASCB: Analytic IMA: Corporate Finance AICPA: Measurement 6.13
Computing annuity payment: The Elkridge Bar & Grill has a seven-year loan of $23,500 with Bank of America. It plans to repay the loan in seven equal installments starting today. If the rate of interest is 8.4 percent, how much will each payment be? Solution: 0 1 2 3 6 7 Year ├───────┼────────┼───────┼………………┼───────┤ PMT PMT PMT PMT PMT PMT PVAn = $23,500 n = 7; i = 8.4% Present value of annuity = PVA = $23,500 SM 19-195
Return on investment = i = 8.4% Payment required to meet target = PMT Type of annuity = Annuity due Using the PVA equation: 1 1 (1 i)n PVAn PMT (1 i) i PMT
$23,500 $23,500 1 5.1359 1.084 1 (1.084)7 (1.084) 0.084
$4, 221.07 Each payment made by Elkridge Bar & Grill will be $4,221.07, starting today. LO 2 Bloomcode: Application AASCB: Analytic IMA: Corporate Finance AICPA: Measurement 6.14
Perpetuity: Your grandfather is retiring at the end of next year. He would like to ensure that his heirs receive payments of $10,000 a year forever, starting when he retires. If he can earn 6.5 percent annually, how much does your grandfather need to invest to produce the desired cash flow? Solution: Annual payment needed = PMT = $10,000 Investment rate of return = i = 6.5% Term of payment = Perpetuity Present value of investment needed = PV PMT $10,000 PV of Perpetuity i 0.065 $153,846.15 LO 3 Bloomcode: Application AASCB: Analytic IMA: Corporate Finance AICPA: Measurement 6.15
Perpetuity: Calculate the annual cash flows for each of the following investments: a. $250,000 invested at 6% b. $50,000 invested at 12% SM 19-196
c. $100,000 invested at 10% Solution: a. Annual payment = PMT Investment rate of return = i = 6% Term of payment = Perpetuity Present value of investment needed = PV = $250,000 PMT PV of Perpetuity i PMT PV i $250,000 0.06 b.
c.
$15,000 Annual payment = PMT Investment rate of return = i = 12% Term of payment = Perpetuity Present value of investment needed = PV = $50,000 PMT PV of Perpetuity i PMT PV i $50,000 0.12 $6,000 Annual payment = PMT Investment rate of return = i = 10% Term of payment = Perpetuity Present value of investment needed = PV = $100,000 PMT PV of Perpetuity i PMT PV i $100,000 0.10 $10,000
LO 3 Bloomcode: Application AASCB: Analytic IMA: Corporate Finance AICPA: Measurement 6.16. Effective annual interest rate: Marshall Chavez bought a Honda Civic for $17,345. He put down $6,000 and financed the rest through the dealer at an APR of 4.9 percent for four years. What is the effective annual interest rate (EAR) if the loan payments are made monthly? Solution: Loan amount = PV = $11,345 Interest rate on loan = i = 4.9% Frequency of compounding = m = 12 SM 19-197
Effective annual rate = EAR 12 m1 0.049 i EAR 1 1 1 1 m 12 1.05 1 5%
LO 5 Bloomcode: Application AASCB: Analytic IMA: Corporate Finance AICPA: Measurement 6.17
Effective annual interest rate: Cyclone Rentals borrowed $15,550 from a bank for three years. If the quoted rate (APR) is 6.75 percent, and the compounding is daily, what is the effective annual interest rate (EAR)? Solution: Loan amount = PV = $15,550 Interest rate on loan = i = 6.75% Frequency of compounding = m = 365 Effective annual rate = EAR 365 m1 0.0675 1 i EAR 1 1 1 365 m
1.0698 1 7% LO 5 Bloomcode: Application AASCB: Analytic IMA: Corporate Finance AICPA: Measurement
6.18
Growing perpetuity: You are evaluating a growing perpetuity investment from a large financial services firm. The investment promises an initial payment of $20,000 at the end of this year and subsequent payments that will grow at a rate of 3.4 percent annually. If you use a 9 percent discount rate for investments like this, what is the present value of this growing perpetuity? Solution: Cash flow at t = 1 = CF1 = $20,000 Annual growth rate = g = 3.4% Discount rate = i = 9% Present value of growing perpetuity = PVA∞
SM 19-198
CF1 $20,000 (i g) (0.09 0.034) $357,142.86
PVA
LO 4 Bloomcode: Application AASCB: Analytic IMA: Corporate Finance AICPA: Measurement
INTERMEDIATE 6.19
Future value with multiple cash flows: Trigen Corp. management will invest cash flows of $331,000, $616,450, $212,775, $818,400, $1,239,644, and $1,617,848 in research and development over the next six years. If the appropriate interest rate is 6.75 percent, what is the future value of these investment cash flows six years from today? Solution: 0 6.75% 1 2 3 4 5 6 ├───────┼────────┼───────┼────────┼───────┼────────┤ $331,000 $616,450 $212,775 $818,400 $1,239,644 $1,617,848 FV $331, 000 (1.0675)5 $616, 450 (1.0675)4 $212, 775 (1.0675)3 $818, 400 (1.0675)2 6 $1, 239, 644 (1.0765)1 $1, 617,848 $458,846.49 $800,514.85 $258,835.74 $932, 612.84 $1, 323, 319.97 $1, 617,848 $5, 391, 977.89 LO 1 Bloomcode: Application AASCB: Analytic IMA: Corporate Finance AICPA: Measurement 6.20
Future value with multiple cash flows: Stephanie Watson plans to make the following investments beginning next year. She will invest $3,125 in each of the next three years and will then make investments of $3,650, $3,725, $3,875, and $4,000 over the following four years. If the investments are expected to earn 11.5 percent annually, how much will Stephanie have at the end of the seven years? Solution: Expected rate of return = i = 11.5% Investment period = n = 7 years SM 19-199
Future value of investment = FV FV $3,125 (1.115)6 $3,125 (1.115)5 $616, 450 (1.115)4 $3, 650 (1.115)3 7 $3, 725 (1.115)2 $3,875 (1.115)1 $4, 000 $6, 004.81 $5, 385.48 $4,830.03 $5, 059.61 $4, 631.01 $4, 320.63 $4, 000 $34, 231.57 LO 1 Bloomcode: Application AASCB: Analytic IMA: Corporate Finance AICPA: Measurement
6.21
Present value with multiple cash flows: Carol Jenkins, a lottery winner, will receive the following payments over the next seven years. If she can invest her cash flows in a fund that will earn 10.5 percent annually, what is the present value of her winnings? 0 1 2 3 4 5 6 7 Year ├────┼─────┼─────┼──────┼─────┼──────┼─────┤ $200,000 $250,000 $275,000 $300,000 $350,000 $400,000 $550,000 Solution: Expected rate of return = i = 10.5% Investment period = n = 7 years Future value of investment = FV $200,000 $250,000 $275,000 $300,000 $350,000 $400,000 $550,000 FV7 (1.105)1 (1.105)2 (1.105)3 (1.105)4 (1.105)5 (1.105)6 (1.105)7 $180,995.48 $204,746.01 $203,819.56 $201,220.46 $212,449.96 $219,728.47 $273,417.77 $1,496,377.71 LO 1 Bloomcode: Application AASCB: Analytic IMA: Corporate Finance AICPA: Measurement 6.22
Computing annuity payment: Gary Whitmore is a high school sophomore. He currently has $7,500 in a savings account that pays 5.65 percent annually. Gary plans to use his current savings plus what he can save over the next four years to buy a car. He estimates that the car will cost $12,000 in four years. How much money should Gary save each year if he wants to buy the car? SM 19-200
Solution: Cost of car in four years = $12,000 Amount invested in money market account now = PV = $7,500 Return earned by investment = i = 5.65% Value of current investment in 4 years = FV4 FV4 PV(1 i)4 $7,500(1.0565)4 $9,344.14 Balance of money needed to buy car = $12,000 – $9,344.14 = $2,655.86 = FVA Payment needed to reach target = PMT (1 i)n 1 FVA PMT i PMT
FVA $2, 655.86 $2, 655.86 n 4 1 (1 i) (1.0565) 1 4.351949 i 0.0565
$610.27 LO 1 Bloomcode: Analysis AASCB: Analytic IMA: Corporate Finance AICPA: Measurement
6.23
Growing annuity: Modern Energy Company owns several gas stations. Management is looking to open a new station in the western suburbs of Baltimore. One possibility that managers at the company are evaluating is to take over a station located at a site that has been leased from the county. The lease, originally for 99 years, currently has 73 years before expiration. The gas station generated a net cash flow of $92,500 last year, and the current owners expect an annual growth rate of 6.3 percent. If Modern Energy uses a discount rate of 14.5 percent to evaluate such businesses, what is the present value of this growing annuity? Solution: Time for lease to expire = n = 73 years Last year’s net cash flow = CF0 = $92,500 Expected annual growth rate = g = 6.3% Firm’s required rate of return = i = 14.5% Expected cash flow next year = CF1 = $92,500(1 + g) = $92,500(1.063) = $98,327.50 Present value of growing annuity = PVAn
SM 19-201
1 g n 1.063 73 $98,327.50 1 PVAn 1 (i g) 1 i (0.145 0.063) 1.145 $1,199,115.85 0.995593 CF1
$1,193,831.54 LO 4 Bloomcode: Application AASCB: Analytic IMA: Corporate Finance AICPA: Measurement
6.24
Future value of an annuity due: Jeremy Denham plans to save $5,000 every year for the next eight years, starting today. At the end of eight years, Jeremy will turn 30 years old and plans to use his savings toward the down payment on a house. If his investment in a mutual fund will earn him 10.3 percent annually, how much will he have saved in eight years when he buys his house? Solution: 0 10.3% 1 2 3 7 8 Year ├───────┼────────┼───────┼………………┼───────┤ $5,000 $5,000 $5,000 $5,000 $5,000
Annual investment = PMT = $5,000 No. of payments = n = 8 Type of annuity = Annuity due Investment rate of return = 10.3% Future value of investment = FVA8 (1 i)n 1 FVAn PMT (1 i) i (1.103)8 1 $5,000 (1.103) $5,000 11.5612 1.103 0.103 $63,760.19 LO 2 Bloomcode: Application AASCB: Analytic IMA: Corporate Finance AICPA: Measurement
SM 19-202
6.25
Present value of an annuity due: Grant Productions borrowed some money from the California Finance Company at a rate of 17.5 percent for a seven-year period. The loan calls for a payment of $1,540,862.19 each year beginning today. How much did Grant borrow? Solution: 0 17.5% 1 2 3 6 7 ├───────┼────────┼───────┼………………┼───────┤ PMT =$1,540,862.19 at the beginning of each year Annual payment = PMT = $1,540,862.19 Type of annuity = Annuity due No. of payments = n = 7 Required rate of return = 17.5% Present value of investment = PVA8 1 1 (1 i)n (1 i) PVAn PMT i 1 1 (1.175)7 $1,540,862.19 (1.175) $1,540,862.19 3.86631.175 0.175 $6, 999, 999.98 $7, 000, 000 LO 2 Bloomcode: Analysis AASCB: Analytic IMA: Corporate Finance AICPA: Measurement 6.26
Present value of an annuity due: Sharon Kabana has won a state lottery and will receive a payment of $89,729.45 every year, starting today, for the next 20 years. If she invests the proceeds at a rate of 7.25 percent, what is the present value of the cash flows that she will receive? Round to the nearest dollar. Solution: 0 7.25% 1 2 3 19 20 ├───────┼────────┼───────┼………………┼───────┤ PMT = $89,729.45 at the beginning of each year Annual payment = PMT = $89,729.45 Type of annuity = Annuity due No. of payments = n = 20 Required rate of return = 7.25% SM 19-203
Present value of investment = PVA20 1 1 (1 i)n (1 i) PVAn PMT i 1 $89, 729.45
1 (1.0725)20 0.0725
(1.0725) $89, 729.45 10.39121.0725
$999, 999.95 $1, 000, 000 LO 2 Bloomcode: Application AASCB: Analytic IMA: Corporate Finance AICPA: Measurement
6.27
Present value of an annuity due: You wrote a piece of software that does a better job of allowing computers to network than any other program designed for this purpose. A large networking company wants to incorporate your software into its systems and is offering to pay you $500,000 today, plus $500,000 at the begining of each of the following six years, for permission to do this. If the appropriate interest rate is 6 percent, what is the present value of the cash flow stream that the company is offering you? Solution: You are being offered a seven year annuity due. You can solve this problem several different ways. First, you can calculate the present value of each of the individual cash flows and then add the present values together. Second, you can use the annuity transformation method discussed in the chapter. Third, you can calculate the present value of an ordinary six-year annuity and then add $500,000 to that value. Of course you can also use your calculator or Excel to do the calculations for you. Below are the calculations for the annuity transformation method. PVA7 = CF × PV annuity factor = $500,000 × 5.5824 = $2,791,190.72 Annuity value due
= PVA7 × (1 + i) = $2,791,190.72 × 1.06 = $2,958,662.16
LO 2
SM 19-204
Bloomcode: Application AASCB: Analytic IMA: Corporate Finance AICPA: Measurement
6.28
Present value of an annuity: Suppose that the networking company in problem 6.27 will not start paying you until the first of new systems that uses your software is sold in two years. What is the present value of that annuity? Assume that the appropriate interest rate is still 6 percent. Solution: Since the first cash flow will not be received for two years, the present value will be less than in problem 6.27. All you have to do to solve this problem is to discount the answer from problem 6.27 for two years. Annuity value = value of annuity value due/(1 + i)2 = $2,958,662/(1.06)2 = $2,633,198.79 This is also equal to the value of the regular annuity in problem 6.27, PVA7, discounted one year at 6 percent. LO 2 Bloomcode: Application AASCB: Analytic IMA: Corporate Finance AICPA: Measurement
6.29
Perpetuity: Calculate the present value of the following perpetuities: a. $1,250 discounted to the present at 7percent b. $7,250 discounted to the present at 6.33percent c. $850 discounted to the present at 20 percent Solution: a. Annual payment = PMT =$1,250 Investment rate of return = i = 7% Term of payment = Perpetuity Present value of investment needed = PV PMT $1,250 PV of Perpetuity i 0.07 $17,857.14 b. Annual payment = PMT =$7,250 Investment rate of return = i = 6.33% Term of payment = Perpetuity. SM 19-205
c.
Present value of perpetuity = PV PMT $7,250 PV of Perpetuity i 0.0633 $114,533.97 Annual payment = PMT =$850 Investment rate of return = i = 20% Term of payment = Perpetuity. Present value of investment needed = PV PMT $850 PV of Perpetuity i 0.20 $4,250
LO 3 Bloomcode: Application AASCB: Analytic IMA: Corporate Finance AICPA: Measurement
6.30
Effective annual interest rate: Find the effective annual interest rate (EAR) on each of the following: a. 6 percent compounded quarterly. b. 4.99 percent compounded monthly. c. 7.25 percent compounded semiannually. d. 5.6 percent compounded daily. Solution: a. Interest rate = i = 6% Frequency of compounding = m = 4 Effective annual rate = EAR 4 m1 0.06 i 1 EAR 1 1 1 m 4 1.06136 1 6.14% b. Interest rate = i = 4.99% Frequency of compounding = m = 12 Effective annual rate = EAR 12 m1 0.0499 i EAR 1 1 1 1 m 12 1.0511 1 5.11% c. Interest rate = i = 7.25% Frequency of compounding = m = 2 Effective annual rate = EAR SM 19-206
2
d.
m1 0.0725 EAR 1 i 1 1 1 m 2 1.0738 1 7.38% Interest rate = i = 5.6% Frequency of compounding = m = 365 Effective annual rate = EAR 365 m1 0.056 1 i EAR 1 1 1 365 m
1.0576 1 5.76% LO 5 Bloomcode: Application AASCB: Analytic IMA: Corporate Finance AICPA: Measurement
6.31
Effective annual interest rate: Which of the following investments has the highest effective annual interest rate (EAR)? a. A bank CD that pays 8.25 percent compounded quarterly. b. A bank CD that pays 8.25 percent compounded monthly. c. A bank CD that pays 8.45 percent compounded annually. d. A bank CD that pays 8.25 percent compounded semiannually. e. A bank CD that pays 8 percent compounded daily (on a 365-day per year basis). Solution: a. Interest rate on CD = i = 8.25% Frequency of compounding = m = 4 Effective annual rate = EAR 4 m1 0.0825 i 1 EAR 1 1 1 m 4 1.08509 1 8.51% b. Interest rate on CD = i = 8.25% Frequency of compounding = m = 12 Effective annual rate = EAR 12 m1 0.0825 i EAR 1 1 1 1 m 12 1.0857 1 8.57% c. Interest rate on CD = i = 8.45% Frequency of compounding = m = 12 Effective annual rate = EAR
SM 19-207
1
m1 0.0845 EAR 1 i 1 1 1 m 1 1.0845 1 8.45% d. Interest rate on CD = i = 8.25% Frequency of compounding = m = 2 Effective annual rate = EAR 2 m1 0.0825 i EAR 1 1 1 1 m 2 1.0842 1 8.42% e. Interest rate on CD = i = 8% Frequency of compounding = m = 365 Effective annual rate = EAR 365 m1 0.08 i EAR 1 1 1 1 m 365 1.0833 1 8.33% Investment b. (8.25% monthly) has the highest EAR.
LO 5 Bloomcode: Analysis AASCB: Analytic IMA: Corporate Finance AICPA: Measurement
6.32
Effective annual interest rate: You are considering three alternative investments: (1) a three-year bank CD paying 7.5 percent compounded quarterly; (2) a three-year bank CD paying 7.3 percent compounded monthly; and (3) a three-year bank CD paying 7.75 percent compounded annually. Which investment has the highest effective annual interest rate (EAR)? Solution: (1) Interest rate on CD = i = 7.5% Frequency of compounding = m = 4 Effective annual rate = EAR 4 m1 0.075 i EAR 1 1 1 1 m 4 1.0771 1 7.71% (2) Interest rate on CD = i = 7.3% Frequency of compounding = m = 12 Effective annual rate = EAR
SM 19-208
12
m1 0.073 EAR 1 i 1 1 1 m 12 1.0755 1 7.55% (3) Interest rate on CD = i = 7.75% Frequency of compounding = m = 1 Effective annual rate = EAR 1 m1 0.0775 i EAR 1 1 1 1 m 1 1.0775 1 7.75% Investment (3) The three-year bank CD paying 7.75 percent interest compounded annually has the highest effective yield.
LO 5 Bloomcode: Analysis AASCB: Analytic IMA: Corporate Finance AICPA: Measurement
ADVANCED 6.33
You have been offered the opportunity to invest in a project which is expected to provide you with the following cash flows: $4,000 in one year, $12,000 in two years, and $8,000 in three years. If the appropriate interest rates are 6 percent for the first year, 8 percent for the second year, and 12 percent for the third year, what is the present value of these cash flows? Solution: The answer is $20,495.15. Calculated as follows: $4, 000 $12, 000 $8, 000 PV 1.06 1.061.08 1.061.081.12 $3, 773.58 $10, 482.18$6, 239.39 $20, 495.15 If the interest rate is different each year, then you must perform the calculation in this way. What you are effectively doing here is discounting each cash flow one year at a time using the appropriate rate. For example, you discount the year three cash flow of $8,000 for one year at 12 percent: $8,000/1.12 = $7,142.86 You then discount this result for one year at 8 percent: $7,142.86/1.08 = $6,613.76
SM 19-209
Finally, you get the present value of the $8,000 by discounting the above result by 6 percent: $6,613.76/1.06 = $6,239.40 The one cent difference from the number above is due to rounding. LO 1 Bloomcode: Application AASCB: Analytic IMA: Corporate Finance AICPA: Measurement
6.34
Tirade Owens, a professional athlete, currently has a contract that will pay him a large amount in the first year of his contract and smaller amounts thereafter. He and his agent have asked the team to restructure the contract. The team, though reluctant, obliged. Tirade and his agent came up with a counteroffer. What are the present values of each of the three alternatives below using a 14 percent discount rate? Which of the three has the highest present value? Year Current Contract Team’s Offer Counteroffer 1 $8,125,000 $4,000,000.00 $5,250,000.00 2 $3,650,000 $3,825,000.00 $7,550,000.00 3 $2,715,000 $3,850,000.00 $3,625,000.00 4 $1,822,250 $3,925,000.00 $2,800,000.00 Solution: Current Contract $8,125,000 $3,650,000 $2,715,000 $1,822,250 PV (1.14) (1.14)2 (1.14)3 (1.14)4 $7,127,192.98 $2,808,556.48 $1,832,547.67 $1,078,918.29 $12,847,215.41 Team’s Offer $4,000,000 $3,825,000 $3,850,000 $3,925,000 PV (1.14) (1.14)2 (1.14)3 (1.14)4 $3,508,771.93 $2,943,213.30 $2,598,640.34 $2,323,915.09 $11,374,540.65 Counteroffer $5,250,000 $7,550,000 $3,625,000 $2,800,000 PV (1.14) (1.14)2 (1.14)3 (1.14)4 $4,605,263.16 $5,809,479.84 $2,446,771.75 $1,657,824.78 $14,519,339.52 The counteroffer has the best value for the player. LO 1 SM 19-210
Bloomcode: Analysis AASCB: Analytic IMA: Corporate Finance AICPA: Measurement
6.35
Gary Kornig will be 30 years old next year and wants to retire when he is 65. So far he has saved (1) $6,950 in an IRA account in which his money is earning 8.3 percent annually and (2) $5,000 in a money market account in which he is earning 5.25 percent annually. Gary wants to have $1 million when he retires. Starting next year, he plans to invest a fixed amount every year until he retires in a mutual fund in which he expects to earn 9 percent annually. How much will Gary have to invest every year to achieve his savings goal? Solution: Investment (1) Balance in IRA investment = PV = $6,950 Return on IRA account = i = 8.3% Time to retirement = n = 35 years Value of IRA at age 65 = FVIRA FVIRA PV (1 i)n $6, 950 (1.083)35 $113, 235.03 Investment (2) Balance in money market investment = PV = $5,000 Return on money market account = i = 5.25% Time to retirement = n = 35 years Value of money market at age 65 = FVMMA FVMMA PV (1 i)n $5, 000 (1.0525)35 $29, 973.93 Target retirement balance = $1,000,000 Future value of current savings = $113,235.03 + $29,973.93 = $143,208.96 Amount needed to reach retirement target = FVA = $856,774.04 Annual payment needed to meet target = PMT Expected return from mutual fund = i = 9%
SM 19-211
(1 i)n 1 FVA PMT i
PMT
FVA $856, 791.04 $856, 791.04 (1 i)n 1 (1.09)35 1 215.711 0.09 i
$3, 971.94 LO 1, LO 2 Bloomcode: Application AASCB: Analytic IMA: Corporate Finance AICPA: Measurement
6.36 The top prize for the state lottery is $100,000,000. You have decided it is time for you to take a chance and purchase a ticket. Before you purchase the ticket, you must decide whether to choose the cash option or the annual payment option. If you choose the annual payment option and win, you will receive $100,000,000 in 25 equal payments of $4,000,000─one payment today and one payment at the end of each of the next 24 years. If you choose the cash payment, you will receive a one-time lump sum payment of $59,194,567.18. If you can invest the proceeds and earn 6 percent, which option should you choose? Solution: You should choose the lump sum option. To see this, all you have to do is calculate the present value of the cash flows from the annual payment option using the 6 percent discount rate. Since the annual payment option is like a 25- year annuity due, this value is: PVA25 = CF × PV annuity factor = $4,000,000 × 12.7834 = $51,133,424.63Annuity value due = PVA25 × (1 + i) = $51,133,424.63 × 1.06 = $54,201,430.11 Since this is less than the $59,194,567.18 lump sum, you are better off taking the lump sum. LO 2 Bloomcode: Anlysis AASCB: Analytic IMA: Corporate Finance AICPA: Measurement
SM 19-212
6.37
At what interest rate would you be indifferent between the cash and annual payment options in problem 6.36? Solution: You would be indifferent if you could only earn 5 percent from your investments. With this discount rate, the present value of the cash flows from the annual payment option equals the $59,194,567.18 lump sum. PVA25 = CF × PV annuity factor = $4,000,000 × 14.0949 = $56,375,778.26 Annuity value due
= PVA25 × (1 + i) = $56,376,000 × 1.05 = $59,194,567.18 If you use your calculator or Excel to calculate the present value of the cash flows from the annual payment option you will get exactly $59,194,567.18. LO 2 Bloomcode: Analysis AASCB: Analytic IMA: Corporate Finance AICPA: Measurement
Babu Baradwaj is saving for his son’s college tuition. His son is currently 11 years old and will begin college in seven years. Babu has an index fund investment of $7,500 that is earning 9.5 percent annually. Total expenses at the University of Maryland, where his son says he plans to go, currently total $15,000 per year, but are expected to grow at roughly 6 percent each year. Babu plans to invest in a mutual fund that will earn 11 percent annually to make up the difference between the college expenses and his current savings. In total, Babu will make seven equal investments with the first starting today and with the last being made a year before his son begins college. a. What will be the present value of the four years of college expenses at the time that Babu’s son starts college? Assume a discount rate of 5.5 percent. b. What will be the value of the index mutual fund when his son just starts college? c. What is the amount that Babu will have to have saved when his son turns 18 if Babu plans to cover all of his son’s college expenses? d. How much will Babu have to invest every year in order to have enough funds to cover all his son’s expenses? Solution: Annual cost of college tuition today (t = 0) = $15,000 Expected increase in annual tuition costs = g = 6% a. Four-year tuition costs (t = 7 to t = 10) 6.38
SM 19-213
Years from now Future value calculation 7 $15,000(1.06)7 8 $15,000(1.06)8 9 $15,000(1.06)9 10 $15,000(1.06)10 Discount rate = i = 5.5% Present value of tuition costs = PV $22,554.45 $23,907.72 $25,342.18 $26,862.72 PV (1.055) (1.055)2 (1.055)3 (1.055)4 $21,378.63 $21,479.95 $21,581.75 $21,684.03 b.
Tuition costs $22,554.45 $23,907.72 $25,342.18 $26,862.72
$86,124.36 Future value of the index mutual fund at t = 7 Present value of index fund investment = PV = $7,500 Return on fund = i = 9.5% Future value of investment = FV FV PV (1 i)n $7,500 (1.095)7
c.
d.
$14,156.64 Target savings needed at t = 7 PV of tuition costs – Future value of investment
= $86,124.36 – $14,156.64 = $71,967.72
Annual savings needed Return on fund = i = 11% Amount that needs to be saved = FVA = $71,967.72 Annuity payment needed = PMT (1 i)n 1 FVA PMT (1 i) i PMT
$71, 967.72 $71, 967.72 FVA 7 (1 i) 1 (1.11) 1 (1 i) (1.11) 9.78331.11 i 0.11 n
$6, 627.21 LO 1, LO 2 Bloomcode: Application AASCB: Analytic IMA: Corporate Finance AICPA: Measurement
SM 19-214
6.39
You are now 50 years old and plan to retire at age 65. You currently have a stock portfolio worth $150,000, a 401(k) retirement plan worth $250,000, and a money market account worth $50,000. Your stock portfolio is expected to provide annual returns of 12 percent, your 401(k) investment will earn 9.5 percent annually, and the money market account earns 5.25 percent, compounded monthly. a. If you do not save another penny, what will be the total value of your investments when you retire at age 65? b. Assume you plan to invest $12,000 every year in your 401(k) plan for the next 15 years (starting one year from now). How much will your investments be worth when you retire at age 65? c. Assume that you expect to live 25 years after you retire (until age 90). Today, at age 50, you take all of your investments and place them in an account that pays 8 percent (use the scenario from part b in which you continue saving). If you start withdrawing funds starting at age 66, how much can you withdraw every year (e.g., an ordinary annuity) and leave nothing in your account after a 25th and final withdrawal at age 90? d. You want your current investments, which are described in the problem statement, to support a perpetuity that starts a year from now. How much can you withdraw each year without touching your principal? Solution: a. Stock Portfolio Current value of stock portfolio = $150,000 Expected return on portfolio = i = 12% Time to retirement = n = 15 years Expected value of portfolio at age 65 = FVStock
FVStock PV(1 i)15 $150,000 (1.12)15 $821,034.86 410(k) Investment Current value of 410(k) portfolio = $250,000 Expected return on portfolio = i = 9.5% Time to retirement = n = 15 years Expected value of portfolio at age 65 = FV401k
FV401k PV(1 i)15 $250,000 (1.095)15 $975,330.48 Money market account Current value of savings = $50,000 Expected return on portfolio = i = 5.25% Time to retirement = n = 15 years Frequency of compounding = m = 12 Expected value of portfolio at age 65 = FVMMA
SM 19-215
1215
mn
i FVMMA PV 1 m
b.
c.
d.
0.0525 $50, 000 1 12
$50, 000 2.1941 $109, 706.14 Total value of all three investments = $821,034.86 + $975,330.48 + $109,706.14 = $1,906,071.48 Planned annual investment in 401k plan = $12,000 Future value of annuity = FVA (1 i)n 1 FVAn PMT i (1.095)15 1 $12,000 $12,000 30.5402 0.095 $366,482.77 Total investment amount at retirement = $1,906,071.48 + $366,482.77 = $2,272,554.25 Amount available at retirement = PVA = $2,272,554.25 Length of annuity = n = 25 Expected return on investment = i = 8% Annuity amount expected = PMT Using the PVA equation: 1 1 n (1 i) PVAn PMT i $2,272,554.25 $2,272,554.25 PMT 1 10.6748 1 (1.08)25 0.08 $212,889.63 Each payment received for the next 25 years will be $212,889.63. Type of payment = Perpetuity Present value of perpetuity = PVA = $2,272,554.25 Expected return on investment = i = 8%
SM 19-216
PV of Perpetuity $2,272,553.75
PMT i PMT
0.08 PMT $2,272,554.25 0.08 $181,804.34 You could receive an annual payment of $181,804.34 forever. LO 1, LO 2, LO 3 Bloomcode: Application AASCB: Analytic IMA: Corporate Finance AICPA: Measurement
6.40
Trevor Diaz wants to purchase a Mercedes Benz SL600 Roadster, which has an invoice price of $121,737 and a total cost of $129,482. Trevor plans to put down $20,000 and will pay the rest by taking on a 5.75 percent five-year bank loan. What is the monthly payment on this auto loan? Prepare an amortization table using Excel. Solution: Cost of new car = $129,482 Down payment = $20,000 Loan amount = $129,482 – $20,000 = $109,482 Interest rate on loan = i = 5.75% Term of loan = n = 5 years Frequency of payment = m = 12 Monthly payment on loan = PMT 1 1 (1 i)n PVAn PMT i PMT
$109, 482 $109, 482 1 52.0379 1 125 1 0575 12 0.0575 12
$2,103.89 LO 2
SM 19-217
Bloomcode: Application AASCB: Analytic IMA: Corporate Finance AICPA: Measurement
6.41
The Yan family buying a new 3,500-square-foot house in Muncie, Indiana, and will borrow $237,000 from Bank One at a rate of 6.375 percent for 15 years. What will be their monthly loan payment? Prepare an amortization schedule using Excel. Solution: Home loan amount = $237,000 Interest rate on loan = i = 6.375% Term of loan = n = 15 years Frequency of payment = m = 12 Monthly payment on loan = PMT 1 1 (1 i)n PVAn PMT i PMT
$237, 000 1 1215 1 0.06375 1 12 0.06375 12 $237, 000
$2, 048.27
115.7072 LO 2 Bloomcode: Application AASCB: Analytic IMA: Corporate Finance AICPA: Measurement
6.42
Assume you will start working as soon as you graduate from college. You plan to start saving for your retirement on your 25th birthday and retire on your 65th birthday. After retirement, you expect to live at least until you are 85. You wish to be able to withdraw $40,000 (in today’s dollars) every year from the time of your retirement until you are 85 years old (i.e., for 20 years). The average inflation rate is likely to be 5 percent.
SM 19-218
a. Calculate the lump sum you need to have accumulated at age 65 to be able to draw the desired income. Assume that your annual return on your investment is likely to be 10 percent. b. What is the dollar amount you need to invest every year, starting at age 26 and ending at age 65 (i.e., for 40 years, to reach the target lump sum at age 65? c. Now answer questions a. and b. assuming the rate of return to be 8 percent per year, then again at 15 percent per year. d. Now assume you start investing for your retirement when you turn 30 years old and analyze the situation under rate of return assumptions of (i) 8 percent, (ii) 10 percent, and (iii) 15 percent. e. Repeat the analysis by assuming that you start investing only when you are 35 years old. Solution: RETIREMENT ANALYSIS SUMMARY INVESTMENT AGE = 25 INVESTMENT AGE = 30 Rate of Return Inflation rate Retirement Income Level Lump sum needed at age 65 Annuity payment needed
8%
10%
15%
8%
10%
INVESTMENT AGE = 35
15%
8%
10%
15%
5% $40,000 $5,160,266
$4,353,087
$3,011,353
$5,160,266
$4,353,087
$3,011,353
$5,160,266
$4,353,087
$3,011,353
$19,919
$9,835
$1,693
$29,946
$16,062
$3,417
$45,552
$26,463
$6,927
LO 1, LO 2, LO 3 Bloomcode: Application AASCB: Analytic IMA: Corporate Finance AICPA: Measurement
Sample Test Problems 6.1
Freisinger, Inc., is expecting a new project to start paying off, beginning at the end of next year. It expects cash flows to be as follows: 0 7.8% 1 2 3 4 5 ├───────┼────────┼───────┼────────┼───────┤ $433,676 $478,452 $475,455 $478,326 $535,444
SM 19-219
If Freisinger can reinvest these cash flows to earn a return of 7.8 percent, what is the future value of this cash flow stream at the end of five years? What is its present value? Solution: 4 3 2 1
FV $433, 676 (1.078) $478, 452 (1.078) $475, 455 (1.078) $478, 326 (1.078) 5 $535, 444 $585, 653.08 $599, 369.52 $552,518.65 $515, 635.43 $535, 444 $2, 788, 620.68
PV $433, 676 / 1.078 $478, 452 / (1.078)2 $475, 455 / (1.078)3 $478, 326 / (1.078)4 [$535, 444 / (1.078)5 ] $402, 296.85 $411, 718.95 $379, 536.14 $354, 200.32 $367,807.22 $1, 915, 559.47
LO: 1 Level: Intermediate Bloomcode: Application AASCB: Analytic IMA: Corporate Finance AICPA: Measurement
6.2
Compare an annuity due with an ordinary annuity. The payments for both are made annually and are of the same dollar amounts. The two annuities also have the same duration in years and the same discount rate. Which of the following statements is /are correct? a. The present value of the ordinary annuity is greater. b. The present value of the annuity due is greater. c. The future value of the ordinary annuity is greater. d. The future value of the annuity due is greater. Solution: b and d are both correct The cash flows of the annuity due are received at the beginning of the period, resulting in less discounting and a higher PV and more compounding and a higher FV. LO: 2 Level: Intermediate Bloomcode: Analysis AASCB: Analytic IMA: Corporate Finance AICPA: Measurement SM 19-220
6.3
You plan to set up an endowment at your alma mater that will fund $200,000 of scholarships each year indefinitely. If the principal (the amount you donate) can be invested at 5.5 percent, compounded annually, how much do you need to donate to the university today, so that the first scholarships can be awarded beginning one year from now? Solution: Annual payment needed = PMT = $200,000 Investment rate of return = i = 5.5% Term of payment = Perpetuity PMT $200,000 PV of Perpetuity Present value of investment needed = PV i 0.055 $3,636,363.64 LO: 3 Level: Intermediate Bloomcode: Application AASCB: Analytic IMA: Corporate Finance AICPA: Measurement
6.4
Annalise Genric wants to open a restaurant in a historic building. The property can be leased for 20 years, but not purchased. She believes her restaurant can generate a net cash flow of $76,000 the first year and expects an annual growth rate of 4 percent thereafter. If a discount rate of 15 percent is used to evaluate this business, what is the present value of the cash flows that it will generate? Solution: Length of lease = n = 20 years Next year’s net cash flow = CF1 = $76,000 Expected annual growth rate = g = 4% Required rate of return = i = 15% Present value of growing annuity = PVAn 1 g n 1.04 20 $76,000 1 PVA 1 n (i g) 1 i (0.15 0.04) 1.15 CF1
$690,909 0.86612 $598,410.10 LO: 4 Level: Intermediate SM 19-221
Bloomcode: Application AASCB: Analytic IMA: Corporate Finance AICPA: Measurement
6.5
A credit card offers financing at an APR of 18 percent, with monthly compounding on outstanding charges. What is the effective annual rate (EAR)? Solution: Interest rate = i = 18% Frequency of compounding = m = 12 Effective annual rate = EAR 12
m1 0.18 EAR 1 i 1 1 1 m 12 1.1956 1 0.1956, or19.56 percent
LO: 5 Level: Intermediate Bloomcode: Application AASCB: Analytic IMA: Corporate Finance AICPA: Measurement
6.6
Thomas Nguyen currently has $10,000 in the bank earning interest of 6 percent per year, compounded monthly. If he needs $25,000 to purchase a car and can save an additional $100 a month stating at the end of this month, how long will it take him to accumulate the $25,000? Solution: months 0 5% 1 2 3 4 ? ├───────┼────────┼───────┼────────┤……………. $10,000 $100 $100 $100 $100 $100 $25,000 Monthly payment = CF = $100 Present Value = PV = 10,000 Future Value = FV = 25,000 Annual rate of return = i = 6% Payments per year = m = 12 Type of annuity = Ordinary Annuity Using the FVA equation and the equation for the FV of a single amount: SM 19-222
i nm 1 1 nm m + PV 1 i FV CF i m m 0.06 nm nm 0.06 1 1 + $10, 0001 $25,000 $100 12 0.06 12 12
nm
nm
0.06 0.06 $25, 000 $20, 0001 $20, 000 $10, 000 1 12 12 nm 0.06 $45, 000 $30, 0001 12 $30, $45,000 000 0.06 ln n mln 1 12 ln 1.5 n m 81.3months ln 1.005 Solving for n × m yields: n × m = 81.3 months It will take Shannon 81.3 months or 6.8 years to save the $25,000. LO: 2 Level: Intermediate Bloomcode: Application AASCB: Analytic IMA: Corporate Finance AICPA: Measurement
SM 19-223
Appendix: Deriving the Formula for the Present Value of an Ordinary Annuity In this chapter we showed that the formula for a perpetuity can be obtained from the formula for the present value of an ordinary annuity if n is set equal to ∞. It is also possible to go the other way. In other words, the present value of an ordinary annuity formula can be derived from the formula for a perpetuity. In fact, this is how the annuity formula was originally obtained. To see how this was done, assume that someone has offered to pay you $1 per year forever, beginning next year, but that, in return, you will have to pay that person $1 per year forever, beginning in year n + 1. Solution: In this problem, you will receive an annual payment that grows at a rate of g forever. In return, you will have to pay that person $1(1 + g)n each year forever, beginning in year n + 1. The cash flows that you will receive can be represented as in the following time line. 0 1 2 3 n-1 n n+1 n+2 ∞
Receive $1 $1(1+g) $1(1+g)2……$1(1+g)n-2n $1(1+g)n-1 $1(1+g)n Pay $0 $0 $0 $0 $0 $1(1+g) The first time line shows the cash flows for the perpetuity that you will receive. This perpetuity is worth: $1 CF PVRe ceive i g (i g) The second time line shows the cash flows for the perpetuity that you will pay. The present value of what you owe is: $1 (1 g)n CF (1 g)n (i - g) (i - g) PVOwe n (1 i)n 1 i Notice that if you subtract, year-by-year, the cash flows you would pay from the cash flows you would receive, you get the cash flows for an n-year annuity. 0 1 2 3 n–1 n n+1 n+2 ∞
Difference $1 $1(1 + g) $1(1 + g)2………$1(1 + g)n-2 $1(1 + g)n-1 $0
SM 19-224
Therefore, the value of the offer equals the value of an n-year growing annuity. Solving for the difference between PVOwe from PVRe ceive ,we see that this is the same as Equation 6.5.
CF(1 g)n n i n CF 1 1 g (i - g) 1 i PVReceive PVOwe (1 i) (i - g) CF
Problem 6A.1 In the chapter text, you saw that the formula for a growing perpetuity can be obtained from the formula for the present value of a growing annuity if n is set equal to ∞. It is also possible to go the other way. In other words, the present value of a growing annuity formula can be derived from the formula for a growing perpetuity. In fact, this is how Equation 6.5 was actually derived. Show how Equation 6.5 can be derived from Equation 6.6. Solution: LO: 4 Level:
Chapter 7 Risk and Return Before You Go On Questions and Answers Section 7.1 Note to instructor: There are no questions for this section.
Section 7.2 1.
What are the two components of a total holding period return? Capital appreciation and income. This can be seen in Equation 7.1.
2.
How is the expected return on an investment calculated? The expected return is calculated as a weighted-average of the possible returns on an investment (outcomes) where the weights are the probabilities that each of the possible returns will be realized.
Section 7.3 1.
What is the relation between the variance and the standard deviation? SM 19-225
The standard deviation is the square root of the variance. Alternatively, the variance equals the standard deviation squared. 2.
What relation do we generally observe between risk and return when we examine historical returns? Investments with higher risks tend to have higher returns. This is illustrated in Exhibit 6.3.
3.
How would we expect the standard deviation of the return on an individual stock to compare with the standard deviation of the return on a stock index? We would expect the standard deviation of the return on an individual stock to be greater than the standard deviation of the return on a stock index. For an illustration, see Exhibit 6.4.
Section 7.4 1.
What does the coefficient of variation tell us, and how is it related to the Sharpe Ratio? The coefficient of variation is a measure of risk per unit of return. It tells us the amount of risk, defined as the standard deviation of returns, associated with each 1 percent of expected return for an asset. A larger coefficient of variation indicates greater risk for each 1 percent of return. The Sharpe ratio differs from the coefficient of variation in two ways. First, instead of using the total return in the calculation, with the Sharpe ratio we use the difference between the total return and the risk free rate. Second, the Sharpe ratio is the inverse of the coefficient of variation ratio that would be calculated using this adjusted return. Computing the inverse of the coefficient of variation yields a measure return per unit of risk, which some investors find more intuitive than a measure of risk for each 1 percent of return.
2.
What are the two components of total risk? The two components of total risk are unique risk and systematic risk. Unique risk is risk that is unique to a particular asset. This is the risk that can be eliminated through diversification. Systematic risk is risk that is common to all assets and cannot be diversified away.
3.
Why does the total risk of a portfolio not approach zero as the number of assets in a portfolio becomes very large? Because the systematic risk associated with the individual assets cannot be diversified away. With a very large number of assets, the total risk of a portfolio will approach the
SM 19-226
weighted average (where the weights are the fractions of total portfolio value represented by each asset) of the systematic risks associated with each asset.
Section 7.5 1.
Why are returns on the stock market used as a benchmark in measuring systematic risk? Because the stock market portfolio is the most diversified portfolio for which good return data are available, it is the portfolio that both comes closest to eliminating all unique risk and has good return data. This makes the returns on the stock market a practical choice as a benchmark for measuring the systematic risk of individual assets.
2.
How is beta estimated? Beta is estimated using regression analysis in which returns on an individual asset are regressed against returns on the market. Beta is the slope of the regression line. This is illustrated in Exhibit 6.11.
3.
How would you interpret a beta of 1.5 for an asset? A beta of 0.75? A beta of 1.5 indicates that the asset has 1.5 times as much systematic risk as the market. A beta of 0.75 indicates that the asset has 75 percent as much systematic risk as the market.
Section 7.6 Note to instructor: There are no questions for this section.
Section 7.7 1.
How is the expected return on an asset related to its systematic risk? CAPM tells us that there is a linear relation between expected return and systematic risk. With zero systematic risk, the expected return equals the risk-free rate. For systematic risk greater than zero, the expected return on an asset increases as its systematic risk increases and this increase is linear. This relation is illustrated in Equation 7.9.
2.
What name is given to the relation between risk and expected return implied by the CAPM? This plot is called the Security Market Line, or SML, and is illustrated in Exhibit 7.11.
3.
If an asset’s expected return does not plot on the line in question 2 above, what does that imply about its price? If the expected return on an asset does not plot on the SML, then this indicates that the expected return on the asset is either too low or too high in view of its systematic risk. If
SM 19-227
the asset plots below the SML, the expected return is too low and the price is too high. If the asset plots above the SML, the expected return is too high and its price is too low.
SM 19-228
Self-Study Problems 7.1
Kaaran made a friendly wager with a colleague that involves the result from flipping a coin. If heads comes up, Kaaran must pay her colleague $15; otherwise, her colleague will pay Kaaran $15. What is Kaaran’s expected cash flow, and what is the variance of that cash flow if the coin has an equal probability of coming up heads or tails? Suppose Kaaran’s colleague is willing to handicap the bet by paying her $20 if the coin toss results in tails. If everything else remains the same, what are Kaaran’s expected cash flow and the variance of that cash flow? Solution: Part 1: E(cash flow) = (0.5 × –$15) + (0.5 × $15) = 0 σ2cash flow = [0.5 × (–$15 - $0)2] + [0.5 × ($15 – $0)2] = $225 Part 2: E(cash flow) = (0.5 × –$15) + (0.5 × $20) = $2.50 σ2cash flow = [0.5 × (–$15 – $2.50)2] + [0.5 × ($20 – $2.50)2] = $306.25
7.2
You know that the price of CFI, Inc., stock will be $12 exactly one year from today. Today the price of the stock is $11. Describe what must happen to the price of CFI, Inc., today in order for an investor to generate a 20 percent return over the next year. Assume that CFI does not pay dividends. Solution: The expected return for CFI based on today’s stock price is ($12 – $11)/$11 = 9.09 percent, which is lower than 20 percent. Since the stock price one year from today is fixed, then the only way that you will generate a 20 percent return is if the price of the stock drops today. Consequently, the price of the stock today must drop to $10. It is found by solving the following: 0.2 = ($12 – x)/ x, or x = $10.
7.3
The expected value of a normal distribution of prices for a stock is $50. If you are 90 percent sure that the price of the stock will be between $40 and $60, then what is the variance of the prices for the stock? Solution: Since you know that 1.645 standard deviations around the expected return captures 90 percent of the distribution, you can set up either of the following equations: $40 = $50 – 1.645σ or $60 = $50 + 1.645σ and solve for σ. Doing this with either equation yields σ = $6.079 and σ2 = 36.954
SM 19-229
7.4
You must choose between investing in stock A and stock B. You have already used CAPM to calculate the rate of return you should expect to receive for each stock given their systematic risk and decided that the expected return for both exceeds that predicted by CAPM by the same amount. In other words, both are equally attractive investments for a diversified investor. However, since you are still in school and do not have a lot of money, your investment portfolio is not diversified. You have decided to invest in the stock that has the highest expected return per unit of total risk. If the expected return and standard deviation of returns for stock A are 10 percent and 25 percent, respectively, and the expected return and standard deviation of returns for stock B are 15 percent and 40 percent, respectively, which should you choose? Assume that the risk free rate is 5 percent. Solution: A comparison of the Sharpe ratios for the two stocks will tell you which has the highest expected return per unit of total risk. E(RA ) - Rrf 0.10 0.05 SA = 0.20 σR 0.25 A
SB =
E(RB ) - Rrf 0.15 0.05 0.25 σR 0.40 B
Stock B has the highest expected return per unit of risk.
7.5
CSB, Inc., has a beta of 1.35. If the expected market return is 14.5 percent and the risk-free rate is 5.5 percent, what is the appropriate required return of CSB (using the CAPM)? Solution: E(RCSB) = Rrf - 0.055)] = 0.1765 or, CSB[E(RM) - Rrf 17.65%
Discussion Questions 7.1
Suppose that you know the risk and the expected return for two stocks. Discuss the process you might utilize to determine which of the two stocks is a better buy. You may assume that the two stocks will be the only assets held in your portfolio. You should be looking to maximize your expected return on an investment given the level of risk that such an investment requires the investor to bear. Therefore, you should compare the expected return and risk associated with each of the two stocks. If the stocks SM 19-230
have the same expected return, then choose the stock with the lower risk. If the stocks have the same risk, then choose the stock with the greatest expected return. If the expected return and risk of the two assets have no common level, perhaps you should compare the ratio of the risk/expected return to see which stock contains the least risk per unit of expected return. LO: 1 Level: Basic Bloomcode: Analysis AASCB: Analytic IMA: Corporate Finance AICPA: Measurement 7.2
What is the difference between the expected rate of return and the required rate of return? What does it mean if they are different for a particular asset at a particular point in time? The required rate of return is the rate of return that investors require to compensate them for the risk associated with an investment. The expected return will not necessarily equal the required rate of return. The expected return can be lower, in which case the return will not be sufficient to compensate the investor for the risk associated with the investment if the expected return is realized. It can also be higher, in which case the expected return will be greater than that necessary to compensate the investor for the riskiness of the asset.
LO: 6 Level: Basic Bloomcode: Analysis AASCB: Analytic IMA: Corporate Finance AICPA: Measurement 7.3
Suppose that the standard deviation of the returns on the shares of stock at two different companies is exactly the same. Does this mean that the required rate of return will be the same for these two stocks? How might the required rate of return on the stock of a third company be greater than the required rates of return on the stocks of the first two companies even if the standard deviation of the returns of the third company’s stock is lower? No. Because some risk can be diversified away, it is possible that two stocks with the same standard deviation of returns can have different required rates of return. One of these stocks can have a higher systematic risk than the other stock and, therefore, a higher required rate of return. The third stock can have a higher required rate of return if its systematic risk is greater than the systematic risk of the stock in the other two companies. LO: 4 SM 19-231
Level: Basic Bloomcode: Comprehension AASCB: Analytic IMA: Corporate Finance AICPA: Measurement
7.4
The correlation between stocks A and B is 0.50, while the correlation between stocks A and C is –0.5. You already own stock A and are thinking of buying either stock B or stock C. If you want your portfolio to have the lowest possible risk, would you buy stock B or C? Would you expect the stock you choose to affect the return that you earn on your portfolio? You would buy stock C because it would result in your portfolio having a lower beta. If you buy stock C, the required return for your portfolio would be lower than the required return would be if you bought stock B. If the expected returns on stocks C and B equal their required returns, then you would expect your portfolio to earn less with stock C.
LO: 5 Level: Basic Bloomcode: Analysis AASCB: Analytic IMA: Corporate Finance AICPA: Measurement
7.5
The idea that we can know the return on a security for each possible outcome is overly simplistic. However, even though we cannot possibly predict all possible outcomes, this fact has little bearing on the risk-free return. Explain why. The risk-free security delivers the same return in all states of the world. Even though we do not know all of the possible states of the world in future periods, we do know that the U.S. government will be able to repay its borrowing in every state of the world. Therefore, the shortcoming of the model does not affect the risk-free security’s return.
LO: 6 Level: Basic Bloomcode: Comprehension AASCB: Analytic IMA: Corporate Finance AICPA: Measurement 7.6
Which investment category included in Exhibit 7.3 has shown the greatest degree of risk in the United States since 1926? Explain why that makes sense in a world where SM 19-232
the value of an asset in this investment category is likely to be more sensitive to changes in market conditions than is the price of a corporate bond. Small stocks have generally been riskier than large stocks, long-term corporate bonds, long-term government bonds, intermediate government bonds, and short-term government bonds. The explanation for this can be best understood if we realize that small stocks will be affected to a greater extent than the list of investments above by either good or bad states of the world. These good and bad effects translate into a distribution with greater spread or greater risk than the other investments. LO: 4 Level: Intermediate Bloomcode: Comprehension AASCB: Analytic IMA: Investment Decisions AICPA: Measurement 7.7
You are concerned about one of the investments in your fully diversified portfolio. You just have an uneasy feeling about the CFO, Iam Shifty, of that particular firm. You do believe, however, that the firm makes a good product and that it is appropriately priced by the market. Should you be concerned about the effect on your portfolio if Shifty embezzles a portion of the firm’s cash? The risk of Shifty embezzling is a nonsystematic risk that will most likely be offset by a more fortunate event affecting another holding in your portfolio. Therefore, Shifty’s actions should not affect the risk that you bear by investing in your diversified portfolio (systematic risk). LO: 5, 6 Level: Basic Bloomcode: Application AASCB: Analytic IMA: Investment Decisions AICPA: Measurement 7.8
The CAPM is used to price the risk (estimate the expected return) for any asset. Our examples have focused on stocks, but we could also use CAPM to estimate the expected rate of return for bonds. Explain why. A firm’s ability to repay its debt obligations will be affected in a very similar, and yet lessened, way than the firm’s stock will be affected. That is, systematic and nonsystematic factors will also affect returns for debt securities. However, if held in a diversified portfolio, then only the systematic risk component will be borne and then compensated for bearing. Therefore, we can use the CAPM to price debt securities.
LO: 7 Level: Intermediate SM 19-233
Bloomcode: Comprehension AASCB: Analytic IMA: Corporate Finance AICPA: Measurement 7.9
In recent years, investors have agreed that the market portfolio consists of more than just a group of U.S. stocks and bonds. If you are an investor who invests in only U.S. stocks and bonds, describe the effects on the risk in your portfolio. If the market portfolio is composed of all assets, then the U.S.-only portfolio will probably have a small amount of nonsystematic risk that is not providing return compensation for that risk. Therefore, the portfolio is bearing too much risk given its expected returns. LO: 5, 6 Level: Basic Bloomcode: Application AASCB: Analytic IMA: Corporate Finance AICPA: Measurement 7.10
You may have heard the statement that you should not include your home as an asset in your investment portfolio. Assume that your house will comprise up to 75 percent of your assets in the early part of your investment life. Evaluate the implications of omitting it when calculating the risk of your overall investment portfolio. From a systematic risk measurement perspective, omitting the beta of your real estate investment, which does not have a beta equal to zero, could have a serious impact on your portfolio’s perceived systematic risk. In a volatile real estate market, you could be understating the risk in your portfolio, and in a flat real estate market, you could be overestimating the risk in your portfolio.
LO: 7 Level: Intermediate Bloomcode: Analysis AASCB: Analytic IMA: Investment Decisions AICPA: Measurement
SM 19-234
Questions and Problems BASIC 7.1
Returns: Describe the difference between a total holding period return and an expected return. LO 2, LO3 Bloomcode: Analysis AASCB: Analytic IMA: Investment Decision AICPA: Measurement Solution: A holding period return is the total return over some investment or ―holding‖ period. It consists of a capital appreciation component and an income component. A holding period return reflects past performance. An expected return is a return that is based on the probability-weighted average of the possible returns from an investment. It describes a possible return (or even a return that may not be possible) for a yet- to-occur investment period.
7.2
Expected returns: John is watching an old game show rerun on television called Let’s Make a Deal in which the contestant chooses a prize behind one of two curtains. Behind one of the curtains is a gag prize worth $150, and behind the other is a round-the-world trip worth $7,200. The producer of the game show has placed a subliminal message on the curtain containing the gag prize, which makes the probability of choosing the gag prize equal to 75 percent. What is the expected value of the selection, and what is the standard deviation of that selection?
LO 3 Bloomcode: Application AASCB: Analytic IMA: Quantitative Methods AICPA: Measurement Solution: E(prize) = (0 .75 × $150) + (0.25 × $7,200) = $1,912.50 σ2prize = (0.75 × ($150 – $1,912.50)2) + (0.25 × ($7,200 – $1,912.50)2 ) = $9,319,218.75 σprize = ($9,319,218.75)1/2 = $3,052.74
7.3
Expected returns: You have chosen biology as your college major because you would like to be a medical doctor. However, you find that the probability of being accepted to SM 19-235
medical school is about 10 percent. If you are accepted to medical school, then your starting salary when you graduate will be $300,000 per year. However, if you are not accepted, then you would choose to work in a zoo, where you will earn $40,000 per year. Without considering the additional years you would spend in school if you study medicine or the time value of money, what is your expected starting salary as well as the standard deviation of that starting salary? LO 3 Bloomcode: Application AASCB: Analytic IMA: Quantitative Methods AICPA: Measurement Solution: E(salary) = 0.9($40,000) + (0.1) ($300,000) = $66,000 σ2salary = 0.9($40,000 – $66,000)2 + (0.1) ($300,000 – $66,000)2 = $6,084,000,000 σsalary = ($6,084,000,000)1/2 = $78,000
7.4
Historical market: Describe the general relation between risk and return that we observe in the historical bond and stock market data. LO 3, LO4 Bloomcode: Comprehension AASCB: Analytic IMA: Corporate Finance AICPA: Measurement Solution: The general axiom that the greater the risk, the greater the return describes the historical returns of the bond and stock market. If we look at Exhibit 6.3 in the text, we see that small stocks have averaged the greatest returns but that they also have the greatest standard deviation for the returns. When compared to large stocks, the average return and standard deviation of the small stocks are greater. Large stock average returns and standard deviation numbers are larger than those of long-term government bonds, which are larger than those of intermediate-term government bonds, which in turn are larger than those of U.S. Treasury bills. The comparison shows that the riskier the investment category, the greater the average return as well as standard deviation of returns.
7.5
Single-asset portfolios: Stocks A, B, and C have expected returns of 15 percent, 15 percent, and 12 percent, respectively, while their standard deviations are 45 percent, 30 percent, and 30 percent, respectively. If you were considering the purchase of each of these stocks as the only holding in your portfolio and the risk free rate is 0 percent, which stock should you choose? SM 19-236
LO 4 Bloomcode: Analysis AASCB: Analytic IMA: Quantitative Methods AICPA: Measurement Solution: Since the holding will be made in a completely undiversified portfolio, then we can calculate the risk per unit of return for each stock, the coefficient of variation, and choose the stock with the lowest value. CV(RA) = 0.45/0.15 = 3.0 CV(RB) = 0.30/0.15 = 2.0===> Choose B CV(RC) = 0.30/0.12 = 2.5 Alternatively, we could have noted that the expected return for A and B was the same, with A having a greater degree of risk. B and C have the same degree of risk, but B has a greater expected return. This would lead you to the conclusion, just as our coefficient of variation calculations did, that Stock B is superior.
7.6
Diversification: Describe how investing in more than one asset can reduce risk through diversification.
LO 5 Bloomcode: Comprehension AASCB: Analytic IMA: Investment Decisions AICPA: Measurement Solution: An investor can reduce the risk of his or her investments by investing in two or more assets whose values do not always move in the same direction at the same time. This is because the movements in the values of the different investments will partially cancel each other out.
7.7 Systematic risk: Define systematic risk. LO 6 Bloomcode: Knowledge AASCB: Analytic IMA: Investment Decisions AICPA: Measurement Solution: Risk that cannot be diversified away is systematic risk. It is the only type of risk that exists in a diversified portfolio, and it is the only type of risk that is rewarded in asset markets. SM 19-237
7.8
Measuring systematic risk: Susan is expecting the returns on the market portfolio to be negative in the near term. Since she is managing a stock mutual fund, she must remain invested in a portfolio of stocks. However, she is allowed to adjust the beta of her portfolio. What kind of beta would you recommend for Susan’s portfolio?
LO 6 Bloomcode: Application AASCB: Analytic IMA: Corporate Finance AICPA: Measurement Solution: If we confine our analysis to portfolios with positive beta values, and since beta describes how much and what direction our portfolio is expected to vary with the market portfolio, then Susan should construct a very low beta portfolio. In that case, Susan’s portfolio is not expected to have losses quite as large as that of the market portfolio. A large beta portfolio would have larger losses than that of the market portfolio. If Susan could construct a negative beta portfolio, then she would like to construct as negative a portfolio beta as possible.
7.9
Measuring systematic risk: Describe and justify what the value of the beta of a U.S. Treasury bill should be.
LO 6 Bloomcode: Comprehension AASCB: Analytic IMA: Corporate Finance AICPA: Measurement Solution: Since the beta of any asset is the slope of the line of best fit for the plot of an asset against that of the market return, then we can use that logic to help us understand the beta of a Tbill. If we purchased a T-bill five years ago and held the same T-bill through each of the last 60 months, then the return for each of those 60 months would be exactly the same. Therefore, the vertical axis coordinates of each of the monthly returns would have the same value and the slope (beta) of the line of best fit would be zero. The meaning of a beta of zero means that our T-bill has no systematic risk. That is logically given that we know that a T-bill has no risk at all since it is a riskless asset.
SM 19-238
7.10
Measuring systematic risk: If the expected rate of return for the market is not much greater than the risk-free rate of return, what does this suggest about the general level of compensation for bearing systematic risk?
LO 6 Bloomcode: Comprehension AASCB: Analytic IMA: Corporate Finance AICPA: Measurement Solution: Such a situation suggests that return compensation for investing in an asset is determined more by the risk-free return than by the market’s compensation for bearing systematic risk. This means that the price for bearing systematic risk is very low. This may be caused by a very low perceived level of risk in the market or by an abundance of funds in the market seeking to be invested in risky assets.
7.11 CAPM: Describe the Capital Asset Pricing Model (CAPM) and what it tells us. LO 7 Bloomcode: Knkowledge AASCB: Analytic IMA: Corporate Finance AICPA: Measurement Solution: The CAPM describes the relation between systematic risk and the expected return. The model tells us that the expected return on an asset with no systematic risk equals the riskfree rate. As systematic risk increases, the expected return increases linearly with beta. The CAPM is written as E(Ri) = Rrf + β i(E(Rm) – Rrf) .
7.12
The Security market line: If the expected return on the market is 10 percent and the risk-free rate is 4 percent, what is the expected return for a stock with a beta equal to 1.5? What is the market risk premium?
LO 7 Bloomcode: Comprehension AASCB: Analytic IMA: Corporate Finance AICPA: Measurement Solution: Following the CAPM prediction: E(Ri) = Rrf + β (E(Rm) – Rrf) = 0.04 + 1.5 × (0.1 – 0.04) = 0.13 The market risk premium is (E(Rm) – Rrf) = 0.06
SM 19-239
SM 19-240
INTERMEDIATE 7.13
Expected returns: Jose is thinking about purchasing a soft drink machine and placing it in a business office. He knows that there is a 5 percent probability that someone who walks by the machine will make a purchase from the machine, and he knows that the profit on each soft drink sold is $0.10. If Jose expects a thousand people per day to pass by the machine and requires a complete return of his investment in one year, then what is the maximum price that he should be willing to pay for the soft drink machine? Assume 250 working days in a year, and ignore taxes and the time value of money.
LO 3 Bloomcode: Application AASCB: Analytic IMA: Investment Decisions AICPA: Measurement Solution: E(Revenue) = 1,000 × 0.05 x $.10 × 250 days = $1,250 Therefore, the most Jose should pay for the machine is $1,250.
7.14
Interpreting the variance and standard deviation: The distribution of grades in an introductory finance class is normally distributed, with an expected grade of 75. If the standard deviation of grades is 7, in what range would you expect 95 percent of the grades to fall?
LO 4 Bloomcode: Application AASCB: Analytic IMA: Quantitative Methods AICPA: Measurement Solution: 95% is 1.960 standard deviations from the mean. Thus, the range can be determined as: 75 ± (1.960 × (7)) So, the lower end is: 75 – (1.960 × (7)) = 61.28 And the upper end is: 75 + (1.960 × (7)) = 88.72 So, 95 percent of the grades should fall between 61.28 and 88.72.
SM 19-241
7.15
Calculating the variance and standard deviation: Kate recently invested in real estate with the intention of selling the property one year from today. She has modeled the returns on that investment based on three economic scenarios. She believes that if the economy stays healthy, then her investment will generate a 30 percent return. However, if the economy softens, as predicted, the return will be 10 percent, while the return will be -25 percent if the economy slips into a recession. If the probabilities of the healthy, soft, and recessionary states are 0.4, 0.5, and 0.1, respectively, then what are the expected return and the standard deviation of the return on Kate’s investment?
LO 4 Bloomcode: Application AASCB: Analytic IMA: Quantitative Methods AICPA: Measurement Solution: E(Ri) = (0.4)(0.3) + (0.5) (0.1) + (0.1) (–.25) = 0.145 σ2return = (0.4)(0.3 – 0.145)2 + (0.5) (0.1 – 0.145)2 + (0.1) (–0.25 – 0.145)2 = 0.02623 σreturn = (0.02623)1/2 = 0.16194 rounded to 0.162
7.16
Calculating the variance and standard deviation: Barbara is considering investing in a company’s stock and is aware that the return on that investment is particularly sensitive to how the economy is performing. Her analysis suggests that four states of the economy can affect the return on the investment. Using the table of returns and probabilities below, find the expected return and the standard deviation of the return on Barbara’s investment. Probability Return Boom 0.1 25.00% Good 0.4 15.00% Level 0.3 10.00% Slump 0.2 -5.00%
LO 4 Bloomcode: Application AASCB: Analytic IMA: Quantitative Methods AICPA: Measurement Solution: E(Ri) = 0.1 × (0.25) + (0.4) × (0.15) + (0.3) × (0.1) + (0.2) (–0.05) = 0.105 σ2return = 0.1 × (0.25 – 0.105)2 + (0.4) × (0.15 – 0.105)2 + (0.3) × (0.1 – 0.105)2 + (0.2) × (–0.05 – 0.105)2
= 0.00773 SM 19-242
σreturn = (0.00773)1/2 = 0.08789
7.17
Calculating the variance and standard deviation: Ben would like to invest in gold and is aware that the returns on such an investment can be quite volatile. Use the following table of states, probabilities, and returns to determine the expected return and the standard deviation of the return on Ben’s gold investment. Probability Return Boom 0.1 40.00% Good 0.2 30.00% OK 0.3 15.00% Level 0.2 2.00% Slump 0.2 -12.00%
LO 4 Bloomcode: Application AASCB: Analytic IMA: Quantitative Methods AICPA: Measurement Solution E(Ri) = 0.1 × (0.4) + (0.2) × (0.3) + (0.3) × (0.15) + (0.2) × (0.02) + (0.2) × (–0.12) = 0.125 σ2return = 0.1 × (0.4 – 0.125)2 + (0.2) × (0.3 – 0.125)2 + (0.3) × (0.15 – 0.125)2 + (0.2) × (0.02 – 0.125)2 + (0.2) × (–0.12 – 0.125)2 = 0.02809 σreturn = (0.02809)1/2 = 0.16759 rounded to 0.168
7.18
Single-asset portfolios: Using the information from Problems 7.15, 7.16, and 7.17, calculate the coefficient of variation for each of the investments in those problems. LO 3, LO 4 Bloomcode: Application AASCB: Analytic IMA: Quantitative Methods AICPA: Measurement Solution: Coefficient of variation = σReturn / E(Ri) Problem 15: 0.16194/0.145 = 1.11684 (using the exact values rather than the printed) Problem 16: 0.08789/0.105 = 0.83707 (using the exact values rather than the printed) Problem 17: 0.16759/0.125 = 1.34069 (using the exact values rather than the printed)
SM 19-243
7.19
Portfolios with more than one asset: Emmy is analyzing a two-stock portfolio that consists of a utility stock and a commodity stock. She knows that the return on the utility stock has a standard deviation of 40 percent and the return on the commodity stock has a standard deviation of 30 percent. However, she does not know the exact covariance in the returns of the two stocks. Emmy would like to plot the variance of the portfolio for each of three cases—covariance of 0.12, 0, and –0.12—in order to understand what the variance of the portfolio would be for a range of covariances. Do the calculation for all three cases (0.12, 0 and –0.12), assuming an equal proportion of each stock in the portfolio.
LO 5 Bloomcode: Analysis AASCB: Analytic IMA: Corporate Finance AICPA: Measurement Solution: ) x 2 2 x 2 2 2x x Var(R 2 asset port
1
1
2
2
1 2
1,2
Case 1, σ1,2 = 0.12: (0.5)2 × (0.4)2 + (0.5)2 × (0.3)2 + 2 × (0.5) × (0.5) × (0.12) = 0.1225 Case 2, σ1,2 = 0.0: (0.5)2 × (0.4)2 + (0.5)2 × (0.3)2 + 2 × (0.5) × (0.5) × (0.0) = 0.0625 Case 3, σ1,2 = -0.12: (0.5)2 × (0.4)2 + (0.5)2 × (0.3)2 + 2 × (0.5) × (0.5) × (-0.12) = 0.0025
7.20 Portfolios with more than one asset: Given the returns and probabilities for the three possible states listed below, calculate the covariance between the returns of Stock A and Stock B. For convenience, assume that the expected returns of Stock A and Stock B are 11.75 percent and 18 percent, respectively. Probability Return on A Return on B Good 0.35 0.30 0.50 OK 0.50 0.10 0.10 Poor 0.15 -0.25 -0.30 LO 5 Bloomcode: Application AASCB: Analytic IMA: Corporate Finance AICPA: Measurement Solution:
SM 19-244
Cov(RA , R B ) AB 0.35 (0.3 0.1175) (0.5 0.18) 0.5 (0.1 0.1175) (0.1 0.18) 0.15 (0.25 0.1175) (0.3 0.18) 0.0476 7.21
Compensation for bearing systematic risk: You have constructed a diversified portfolio of stocks such that there is no unsystematic risk. Explain why the expected return of that portfolio should be greater than the expected return of a risk-free security.
LO 6 Bloomcode: Comprehension AASCB: Analytic IMA: Quantitative Methods AICPA: Measurement Solution: Your portfolio contains no unsystematic risk but it does contain systematic risk. Therefore, the market should compensate the holder of this portfolio for the systematic risk that the investor bears. A risk-free security has no risk and therefore requires no compensation for risk bearing. The expected return of your portfolio should therefore be greater than the return of the risk-free security.
7.22
Compensation for bearing systematic risk: Write out the equation for the covariance in the returns of two assets, Asset 1 and Asset 2. Using that equation, explain the easiest way for the two asset returns to have a covariance of zero.
LO 6 Bloomcode: Application AASCB: Analytic IMA: Quantitative Methods AICPA: Measurement Solution: Cov(Return1 , Return 2 ) R12 n
pi (Return1,i E(Return1 ) (Return 2,i E(Return 2 ) i1
We know that all state probabilities must be greater than zero, and thus the source of a zero covariance cannot be from the state probabilities. The easiest way for the entire probability weighted sum to equal zero is for one of the assets, say Number 1(2), to have a value in all states j that is equal to the expected return of Number 1(2). Another way of saying that is for one of the assets to have a constant return in all states. If that occurs, then the second term in the equation will always be equal to zero, causing the sum, or covariance, to be zero.
SM 19-245
7.23
Compensation for bearing systematic risk: Evaluate the following statement: ―By fully diversifying a portfolio, such as by buying every asset in the market, we can completely eliminate all types of risk, thereby creating a synthetic Treasury bill.‖
LO 6 Bloomcode: Comprehension AASCB: Analytic IMA: Corporate Finance AICPA: Measurement Solution: The statement is false. A portfolio of all assets would only eliminate unsystematic risk. The systematic risk would remain. If you could eliminate both systematic and unsystematic risk, the expected rate of return on the market portfolio would be equal to the risk-free rate of return, and we know that this is not true.
7.24
CAPM: Damien knows that the beta of his portfolio is equal to 1, but he does not know the risk-free rate of return or the market risk premium. He also knows that the expected return on the market is 8 percent. What is the expected return on Damien’s portfolio?
LO 7 Bloomcode: Application AASCB: Analytic IMA: Corporate Finance AICPA: Measurement Solution: Following the CAPM prediction: E(Ri) = Rrf + β (E(Rm) – Rrf) = Rrf + E(Rm) – Rrf = E(Rm) = 0.08 = 8.0%
7.25
CAPM: In February 2014 the risk free rate was 4.75 percent, the market risk premium was 6 percent and the beta for Twitter stock was 1.31. What is the expected return that was consistent with the systematic risk associated with the returns on Twitter stock?
LO 7 Bloomcode: Application AASCB: Analytic IMA: Corporate Finance AICPA: Measurement Solution: From Equation 7.10: E(RTwitter stock) = Rrf + βTwitter stock[E(Rm) - Rrf] E(RTwitter stock) = 0.0475 + 1.31[0.06] E(RTwitter stock) = 0.0475 + 0.0786 = 0.1261, or 12.61% SM 19-246
7.26
CAPM: The market risk premium is 6 percent, and the risk-free rate is 5 percent. If the expected return on a bond is 6.5 percent, what is its beta?
LO 7 Bloomcode: Application AASCB: Analytic IMA: Corporate Finance AICPA: Measurement Solution: From Equation 7.10, we can calculate the beta of the bond as: E(Rbond) = Rrf + β bond × [E(Rm) - Rrf] 0.065 = 0.050 + β bond × 0.06 β bond = (0.065 – 0.050)/0.060 = 0.25
ADVANCED
7.27
David is going to purchase two stocks to form the initial holdings in his portfolio. Iron stock has an expected return of 15 percent, while Copper stock has an expected return of 20 percent. If David plans to invest 30 percent of his funds in Iron and the remainder in Copper, what will be the expected return from his portfolio? What if David invests 70 percent of his funds in Iron stock?
LO 3 Bloomcode: Application AASCB: Analytic IMA: Corporate Finance AICPA: Measurement Solution: Part 1: E(Rport) = (0.3) × (0.15) + (0.7) × (0.2) = 0.185 = 18.5% Part 2: E(Rport) = (0.7) × (0.15) + (0.3) × (0.2) = 0.165 = 16.5%
Peter knows that the covariance in the return on two assets is –0.0025. Without knowing the expected return of the two assets, explain what that covariance means. LO 3, LO 5 Bloomcode: Comprehension AASCB: Analytic IMA: Investment Decisions AICPA: Measurement 7.28
SM 19-247
Solution: The covariance measure is dependent on the expected return of the two assets but in question without knowingthe expected return of the two assets, it is difficult to characterize the scale of the covariance. However, since the covariance is negative, we can say that generally the two assets move in opposite directions, with respect to their own means, from each other in given states of nature.
7.29
In order to expect that it will fund her retirement, Glenda needs her portfolio to have an expected return of 12 percent per year over the next 30 years. She has decided to invest in Stocks 1, 2, and 3, with 25 percent in Stock 1, 50 percent in Stock 2, and 25 percent in Stock 3. If Stocks 1 and 2 have expected returns of 9 percent and 10 percent per year, respectively, then what is the minimum expected annual return for Stock 3 that is likely to enable Glenda to achieve her investment requirement? LO 3, LO 5 Bloomcode: Application AASCB: Analytic IMA: Corporate Finance AICPA: Measurement Solution: The formula for the expected return of a three-stock portfolio is: E(R3 asset port ) x1E(R1 ) x2 E(R2 ) x3 E(R3 ) Therefore, we can solve as in the following: 0.12 = 0.25 × (0.09) + 0.5 × (0.1) + 0.25 × E(R3) E(R3) = 0.19 or 19.0%
7.30
Tonalli is putting together a portfolio of 10 stocks in equal proportions. What is the relative importance of the variance for each stock versus the covariance for the pairs of stocks in her portfolio? For this exercise, ignore the actual values of the variance and covariance terms and explain their importance conceptually. LO 4, LO 5 Bloomcode: Application AASCB: Analytic IMA: Corporate Finance AICPA: Measurement Solution: The variance of the portfolio will be composed of 10 (n = 10) individual stock variance terms and 45 ((n2 –n)/2) covariance terms (really 90). Therefore, the vast majority of the portfolio variance calculation will be determined by the covariance terms of the portfolio in most cases. SM 19-248
7.31
Explain why investors who have diversified their portfolios will determine the price and, consequently, the expected return on an asset. LO 3, LO 5 Bloomcode: Comprehension AASCB: Analytic IMA: Investment Decisions AICPA: Measurement Solution: If all investors require returns that compensate them for the level of risk that they bear, then undiversified investors will require a greater return for a given investment than diversified investors will. In other words, diversified investors will be willing to pay a higher price for an asset than will undiversified investors. Therefore, a diversified investor is the marginal investor whose purchase will determine the price, and therefore the expected return for an asset.
7.32
Brad is about to purchase an additional asset for his well-diversified portfolio. He notices that when he plots the historical returns of the asset against those of the market portfolio, the line of best fit tends to have a large amount of prediction error for each data point (the scatter plot is not very tight around the line of best fit). Do you think that this will have a large or a small impact on the beta of the asset? Explain your opinion.
LO 5 Bloomcode: Application AASCB: Analytic IMA: Corporate Finance AICPA: Measurement Solution: It will have no effect on the beta of the asset. The beta measures only the systematic risk or variation in the returns of the asset. The prediction error reflects the nonsystematic risk inherent in the returns of the asset and will consequently not affect the beta of the asset.
7.33
Draw the Security Market Line (SML) for the case where the market risk premium is 5 percent and the risk-free rate is 7 percent. Now suppose an asset has a beta of –1.0 and an expected return of 4 percent. Plot it on your graph. Is the security properly priced? If not, explain what we might expect to happen to the price of this security in the market. Next, suppose another asset has a beta of 3.0 and an expected return of 20 percent. Plot it on the graph. Is this security properly priced? If not, explain what we might expect to happen to the price of this security in the market. SM 19-249
LO 7 Bloomcode: Application AASCB: Analytic IMA: Quantitative Methods AICPA: Measurement Solution: The Security Market Line (SML) shows the relationship between an asset’s expected return and its beta. We know the market has a beta of one, and we know the risk-free rate has a beta of zero. The risk-free rate of return is 7 percent, and the market is expected to return 5 percent more than this. Therefore, the expected rate of return for the market (a beta one asset) is 12 percent. To draw this SML, we need only connect the dots:
Expected Return
18% 15% 12% 9% 6% 3% 0% 0
1
2
Beta We can see from the following diagram that an asset with expected return of 4 percent and a beta of –1.0 is underpriced (its expected return is too high). As the market becomes aware of this underpricing, investors will purchase the asset, bidding up its price until its expected return falls on the SML. (Recall that as the initial purchase price of an asset increases, the expected return from purchasing the asset will decrease because you are paying a higher initial cost for the asset.)
SM 19-250
18%
Expected Return
15%
The investment will fall here in this plot
12%
9%
6%
3%
0% -1
0
1
2
Beta
As we can see from the following diagram, an asset with a beta of 3.0 should have an expected return of 7% + (3)(5%) = 22%. The asset only has an expected return of 20 percent. Therefore, this asset is overpriced. Demand for this asset will be low, driving down its market price, until the asset’s expected return falls on the SML. 23%
Expected Return
18%
14%
9%
The investment will fall here in this plot
5%
0% 0
1
2
3
Beta
7.34
If the CAPM describes the relation between systematic risk and expected returns, can both an individual asset and the market portfolio of all risky assets have negative expected real rates of return? Why or why not?
LO7 Bloomcode: Comprehension AASCB: Analytic IMA: Corporate Finance AICPA: Measurement Solution: A negative expected real rate of return implies that the expected total rate of return is less SM 19-251
than the risk-free rate. 1) It is possible for the beta of an individual stock to be negative. Therefore, from CAPM, E[Ri] = Rrf + βi × [E(Rm) - Rrf)], it is also possible for a share of stock to have a negative expected real rate of return. 2) It is not possible for the market portfolio to have a negative expected real rate of return since the beta for the market equals 1.
7.35
You have been provided the following data on the securities of three firms and the market: σRi Security E[Ri] βi Stock A 0.15 1.0 1.5 Stock B 0.15 0.18 0.5 Stock C 0.10 0.02 0.5 Market portfolio 0.10 0.04 Treasury bills 0.05 0 Assume the CAPM and SML are true and fill in the missing values in the table. Would you invest in the stock of any of the three firms? If so, which one(s) and why?
LO 7 Bloomcode: Application AASCB: Analytic IMA: Corporate Finance AICPA: Measurement Solution: 1) The missing values in the table are: A RA,M (RA ) RM 1.0(RA ) (0.04) RA 0.06 B RB,M (RB ) RM 0.5(0.18) (0.04) 2.25 C RC,M (RC ) RM RC,M 0.02 0.04 RC,M 1.00 R M ,M 1.00; M 1.00
R Tbill
0; Tbill 0 ,M
2) Using Equation 7.10 you can calculate the expected returns for the different shares: SM 19-252
E(RA) = 0.05 + [1.5 × (0.10 - 0.05)] = 0.125, or 12.5% E(RB) = 0.05 + [2.25 × (0.10 - 0.05)] = 0.1625, or 16.25% E(RC) = 0.05 + [0.5 × (0.10 - 0.05)] = 0.075, or 7.5% A comparison of the expected returns that are given in the table above,with the returns that CAPM predicts (which are calculated above), indicates that you should buy stocks A and C and avoid stock B.
Sample Test Problems INTERMEDIATE 7.1 Given the following information from Capstone Corporation, what price would CAPM predict that the company’s stock will trade for 1 year from today. Assume that the risk free rate is 3 percent and that the market risk premium is 8 percent? Beta: 0.65 Current stock price: $64.61 Annual dividend: $1.92 LO 2, LO 7 Bloomcode: Application AASCB: Analytic IMA: Corporate Finance AICPA: Measurement Solution: Using CAPM, we find: E(RCapstone) = Rrf + ßCapstone × (E(Rm) – Rrf) = 0.03 + (0.65 × 0.08) = 0.082 or 8.2 percent Using the total holding period return formula (Equation 7.1) we can solve for the stock price in one year: RT
7.2.
P1 P0 CF1 P0
0.082
P1 $64.611.92 $64.61
P1 $67.99
You are considering investing in a mutual fund. The fund is expected to earn a return of 15 percent in the next year. If its annual return is normally distributed with a standard
SM 19-253
deviation of 6.5 percent, what return can you expect the fund to beat 95 percent of the time? LO 4 Bloomcode: Application AASCB: Analytic IMA: Corporate Finance AICPA: Measurement Solution: Since the annual returns earned by the mutual fund are normally distributed, a 90 percent confidence level corresponds to 1.645 standard deviations from the mean. Therefore, 90 percent of the time you would expect the annual return for next year to be in the range: 0.15 + (1.645× 0.065). The lower end of this range is: 0.15 – (1.645 × 0.065) = 0.04308, or 4.31%, which is the return that you can expect the fund to beat 95 percent of the time.
7.3
You have just invested in a portfolio of three stocks. The amount of money that you invested in each stock and its beta are summarized below. Calculate the beta of the portfolio and use the capital asset pricing model (CAPM) to compute the expected rate of return for the portfolio. Assume that the expected rate of return on the market is 15 percent and that the risk-free rate is 7 percent. Stock Investment Beta A $ 200,000 1.50 B 300,000 0.65 C 500,000 1.25
LO 3, LO 7 Bloomcode: Application AASCB: Analytic IMA: Corporate Finance AICPA: Measurement Solution: N
Portfolio Beta = p wi i i 1
A B C Total
$200,000 $300,000 $500,000 $1,000,000
20% 30% 50% 100%
p (0.20 × 1.50) + (0.30 × 0.65) + (0.50 × 1.25) = 1.12 SM 19-254
E(RPortfolio) = Rrf + βPortfolio (E(RM) – Rrf) = 0.07 + [1.12 × (0.15 - 0.07)] = 0.1596, or 15.96%
7.4
What would you recommend to an investor who is considering making an investment in a stock which plots below the security market line (SML)? Explain.
LO 7 Bloomcode: Comprehension AASCB: Analytic IMA: Corporate Finance AICPA: Measurement Solution: You should recommend that the investor not make this investment. An investment that plots below the SML has an expected return that is too low to fully compensate the investor for its systematic risk. In other words, its price is so high that it is not expected to yield a sufficiently high return to compensate for its risk. If investors in the market as a whole realize this, they will sell the stock in question and its price will be driven down by such sales to the point where the expected return fully compensates them for the risk. If the investor you are advising buys the stock now and its price subsequently declines, he or she will bear the cost of that decline.
7.5 Why does an investor want a diversified portfolio? Can an investor eliminate all risk? LO 5, LO 6 Bloomcode: Application AASCB: Analytic IMA: Investment Decision AICPA: Measurement Solution: An investor of a diversified portfolio reduces total risk by investing in two or more assets whose values do not always move in the same direction at the same time. With enough diversification, the investor can eliminate unsystematic or unique risk, but, by definition, will not be able to eliminate systematic, or market risk
CHAPTER 8 Bond Valuation and the Structure of Interest Rates
SM 19-255
Before You Go On Questions and Answers Section 8.1 1.
What are the main differences between the bond markets and stock markets?
A corporate bond market is much larger than the stock market. The biggest investors in corporate bonds are mutual funds, life insurance companies, and pension funds, and given the size of these investors, the trades are conducted in much larger blocks than in the stock market. Also, while most stocks are traded in organized securities markets, most bond transactions take place through dealers in the OTC market.
2.
A bond has a 7 percent coupon rate, a face value of $1,000, and a maturity of four years. On a time line, lay out the cash flows for the bond.
The annual payments for the bond will be $70 ($1,000 x 7%); thus the time line for cash inflows would be as follows: 0
3.
1
_2
3
$70
$70
$70
4 $1,070 ($1,000 + $70)
Explain what a convertible bond is.
Convertible bonds are bonds that can be converted into shares of common stock at some predetermined ratio at the discretion of the bondholder. The convertible feature allows the bondholder to take advantage of the firm’s prosperity if the share prices rises above a certain value.
Section 8.2 1.
Explain conceptually how bonds are priced.
SM 19-256
The current price of a bond is equal to the present value of all the cash flows that will be received from the investment. There are two sets of cash flows from a bond investment. First, there are the coupon payments to be received either annually or semiannually throughout the life of the bond. Second, there is the principal or face value of $1,000 that will be received when the bond matures. In order to find the price of the bond, we must find the present value of the coupons and the present value of the face value. We do this by discounting the entire cash flow stream at the current market rate and adding them up. This gives us the current price of the bond. Recognize that the coupons represent an annuity and that we can use the equation for the present value of an annuity from Chapter 6 to calculate the present value of this cash flow stream.
2.
What is the compounding period for most bonds sold in the United States?
Most bonds sold in the United States pay interest semiannually, whereas European bonds typically only pay interest once a year.
3.
What are zero coupon bonds, and how are they priced?
Zero coupon bonds are debt instruments that do not pay coupon interest but promise a single payment (interest earned plus principal) paid at maturity. The price of a zero coupon bond can be calculated using the same equation as used for coupon bonds, but setting the coupon payments to zero. The resulting formula is as follows: PB = Fmn/(1 + i/m)mn Because zero coupon bonds offer the entire payment at maturity, for a given change in interest rates, their price fluctuates more than coupon bonds with a similar maturity.
Section 8.3 1.
Explain how bond yields are calculated. A bond’s yield can be defined as the interest rate that equates a bond’s price to the present value of its interest payments and principal amount. The calculation of a bond’s SM 19-257
yield, or its yield to maturity, takes into account the bond’s time to maturity, the coupon rate, and par.
Section 8.4 1.
What is interest rate risk?
Bond prices are negatively related to interest rate movements. As interest rates rise, bond prices fall, and vice versa. Interest rate risk simply recognizes the fact that bond prices fluctuate as interest rates change, and, if you sell a bond before maturity, you may sell the bond for a price other than what you paid for it. The greater the fluctuation in bond prices due to changes in interest rates, the greater the interest rate risk.
2.
Explain why long-term bonds with zero coupons are riskier than short-term bonds that pay coupon interest.
According to bond theorems number two and three, for a given change in interest rates, longer-term bonds with low coupon rates have greater price changes than shorter-term bonds with higher coupon rates. Thus, long-term zero coupon bonds have greater interest rate risk—greater price swings—than short-term bonds that pay coupon payments.
Section 8.5 1.
What are default risk premiums, and what do they measure?
Default risk premiums are the amount of return that investors must be paid to purchase a security that possesses default risk compared to a similar risk-free investment. Default risk premiums, at any point in time, represent compensation for the expected financial injury for owning a bond plus some additional premium for bearing risk.
2.
Describe the three most prominent bond rating systems.
SM 19-258
Default risk premiums tend to increase during periods of economic decline and to narrow during periods of economic expansion. This phenomenon is due to changes in investors’ willingness to own bonds with different credit ratings over the business cycle, the socalled flight to quality argument. Specifically, during periods of expansion when few defaults take place, investors are willing to invest in bonds with low credit quality to gain higher yields. In contrast, during tough economic times when many businesses fail, investors are concerned with safety. Accordingly, they adjust their portfolios to include more high-quality credits and sell off bonds with low credit ratings. The three most prominent credit rating agencies are Moody’s Investors Service (Moody’s), Standard & Poor’s (S&P) and Fitch. Exhibit 8.4 describes the corporate bond rating systems used by the three rating agencies.
3.
What are the key factors that most affect the level and shape of the yield curve? The key factors that most affect the shape of the yield curve are the real rate of interest, the expected rate of inflation, and interest rate risk. If the future real rate of interest is expected to rise, it will result in an upward slope of the real rate of interest and consequently in an upward bias to the market yield curve. Similarly, increasing the expected rate of inflation will result in an upward-sloping yield curve, because long-term interest rates will contain a larger inflation premium than short-term interest rates. If these two variables are expected to decline in the future, the result will be a downward bias to the yield curve. In contrast, the longer a bond’s maturity, the greater the bond’s interest rate risk. Thus, interest rate risk premium always adds an upward bias to the slope of the yield curve, since the longer the maturity of a security, the greater its interest rate risk.
SM 19-259
Self-Study Problems 8.1
Calculate the price of a five-year bond that has a coupon of 6.5 percent paid annually. The current market rate is 5.75 percent.
Solution: 0
5.75%
1
2
3
4
5 Year
├───────┼────────┼───────┼────────┼───────┤ $65
$65
$65
$65
$1,065
C C PB 1 5 1 i F5 (12 2 (1 i)5i) (1 i)3 3 4 4 (1 i) $65 ($65 $1, 000) $65 $65 $65 2 3 4 1 (1.0575)5 (1 0.0575) (1.0575) (1.0575) (1.0575) $61.47 $58.12 $54.96 $51.95 $805.28 $1, 031.81
8.2
Bigbie Corp. issued a five-year bond a year ago with a coupon of 8 percent. The bond pays interest semiannually. If the yield to maturity on this bond is 9 percent, what is the price of the bond?
Solution: 0 9% 1
2
3
4
5
6
7
8 Semiannual Period
├───┼───┼───┼────┼───┼───┼───┼────┤ PB =? $40
$40
$40
$40
$40
$40
$40
$1,040
SM 19-260
PB
C / m C / m C F C1 / m (1 2i / m)2 (1 3i / m)3 .......... 8 8 8 1 (1 i / m) (1 i / m)
($40 $1, 000) $80 / 2 $40 $40 ........ 2 3 1 (1.045)8 (1 0.09 / 2) (1.045) (1.045)
$38.28 $36.63 $35.05 $33.54 $32.10 $30.72 $29.39 $731.31 $967.02 Alternatively, we can use the present value annuity factor (in Equation 5.5) and the present value equation (Equation 5.4) from Chapter 5 to solve for the price of the bond: 1 PB C m
1 8 11 0.045 $1, 000 Fn 1 i m mn $40 mn 1.045 8 1 i m 0.045 i m 1
$263.84 $703.19 $967.03
8.3
Rockwell Industries has a three-year bond outstanding that pays a 7.25 percent coupon and is currently priced at $913.88. What is the yield to maturity of this bond? Assume annual coupon payments.
Solution: 0
1
2
3
├───────┼────────┼───────┤ PB = $913.88
$72.50
$72.50
$1,072.50
Use the trial-and error approach to solve for YTM. Since the bond is selling at a discount, we know that the yield to maturity is higher than the coupon rate. Try YTM = 10%.
SM 19-261
PB
C 2 C3 F3 C1 2 3 1 i 1 i 1 i $72.50 $72.50 $72.50 $1, 000 1.10 (1.10)2 (1.10)3
$65.91 $59.92 $805.79 $931.61
Try a higher rate, say YTM = 11%.
PB
C 2 C1 C3 F3 2 3 1 i 1 i 1 i $72.50 $72.50 $72.50 $1, 000 1.11 (1.11)2 (1.11)3
$65.32 $58.84 $784.20 $908.36
Since this is less than the price of the bond, we know that the YTM is between 10 and 11 percent and closer to 11 percent. Try YTM = 10.75%. C C F C PB 1 1 2i 2 3 33 1 i 1 i
$72.50 $72.50 $72.50 $1, 000 2 1.1075 (1.1075) (1.1075)3
$65.46 $59.11 $789.53 $914.09
Alternatively, we can use Equation 5.5 and the present value equation (Equation 5.4) from Chapter 5 to solve for the price of the bond:
SM 19-262
1 1 (1 i)n Fn P C B n i (1 i) 1 1 (1.1075)3 $1, 000 $913.88 $72.50 3 0.1075 (1.1075)
$177.94 $736.15 $914.09
Thus, the YTM is approximately 10.75 percent. Using a financial calculator provided an exact YTM of 10.7594 percent.
8.4
Hindenberg, Inc., has a 10-year bond that is priced at $1,100.00. It has a coupon of 8 percent paid semiannually. What is the yield to maturity on this bond?
Solution: 0
1
2
3
4
5
6
19
├───┼────┼───┼───┼───┼────┼── $40
$40
$40
$40
$40
20
─┼────┤
$40
$40
$40 $1,000
The easiest way to calculate the yield to maturity is with a financial calculator. The inputs are as follows: Enter
20 N
Answer
i
40
−1,100
1,000
PMT
PV
FV
3.31
The answer we get is 3.31 percent, which is the semiannual interest rate. To obtain an annualized yield to maturity, we multiply this by two: YTM = 3.31% 2 YTM = 6.62%
SM 19-263
8.5
Highland Corp., a U.S. company, has a five-year bond whose yield to maturity is 6.5 percent. The bond has no coupon payments. What is the price of this zero coupon bond?
Solution: You have the following information: YTM = 6.5% No coupon payments Most U.S. bonds pay interest semiannually. Thus m x n = 5 × 2 = 10 and i/2 = 0.065/2 = 0.0325. Using Equation 7.3, we obtain the following: PB
Fmn
1 i m
mn
$1, 000 (1.0325)10
$726.27
Discussion Questions 8.1
Because the conversion feature in a convertible bond is valuable to bondholders, convertible bond issues have lower coupon payments than otherwise similar bonds that are not convertible. Does this mean that a company can lower its cost of borrowing by selling convertible debt? Explain. No. The interest (coupon) payments can be reduced by the company but not the overall cost of borrowing by selling convertible debt. The reduction in the value of the interest payments is offset by the value of the conversion feature. If the company’s stock price goes above the price implied by the conversion ratio, the existing stockholders must share some of their gains with the bondholders. Investors are going to look for a required rate of return that compensates them for the risk that they are bearing. The only difference with a
SM 19-264
convertible bond is that some of that compensation comes in the form of the ability to benefit from appreciation in the company’s stock price. LO: 1 Level: Basic Bloomcode: Comprehension AASCB: Analytic IMA: Corporate Finance AICPA: Measurement
What economic conditions would prompt investors to take advantage of a bond’s
8.2
convertibility feature? A bond’s convertibility feature becomes attractive when the company’s stock price rises above the bond’s price. This usually happens in times of economic expansion when the stock market is booming and interest rates are increasing, hence lowering the bond’s price. LO: 1 Level: Basic Bloomcode: Comprehension AASCB: Analytic IMA: Corporate Finance AICPA: Measurement 8.3
We know that a vanilla bond with a coupon rate below the market rate of interest will sell for a discount and that a vanilla bond with a coupon rate above the market rate of interest will sell for a premium. What kind of bond or loan will sell at its par value regardless of what happens to the market rate of interest?
A bond that pays a variable coupon rate that moves up and down with the market rate of interest. While corporate bonds in the U.S. do not have variable coupon payments, bank loans often have variable rates which adjust frequently enough so that the value of the loans remains relatively constant as interest rates move up and down over time. LO: 1 SM 19-265
Level: Basic Bloomcode: Comprehension AASCB: Analytic IMA: Corporate Finance AICPA: Measurement
8.4
Define yield to maturity. Why is it important?
Yield to maturity (YTM) is the rate of return earned by investors if they buy a bond today at its market price and hold it to maturity. It is important because it represents the opportunity cost to the investor or the discount rate that makes the present value of the bond’s cash flows (i.e., its coupons and its principal payments) equal to the market price. So, YTM is also referred to as the going market rate or the appropriate discount rate for a bond’s cash flows. It is important to understand that any investor who buys a bond and holds it to maturity will have a realized gain equal to the yield to maturity. If the investor sells before the maturity date, then realized gain will not be equal to the YTM, but will only be based on cash flows earned to that point. Similarly, for callable bonds, investors are guaranteed a gain to the point in time when the bond is first called, but they cannot be assured of the yield to maturity because the issuer could call the bond before maturity. LO: 3 Level: Basic Bloomcode: Knowledge AASCB: Analytic IMA: Corporate Finance AICPA: Measurement
8.5
Define interest rate risk. How can the CFOs manage this risk?
The change in a bond's prices caused by changes in interest rates is called interest rate risk. In other words, we can measure the interest rate risk to a bond’s investor by measuring the
SM 19-266
percentage change in the bond’s price caused by a 1 percent change in the market interest rates. The key to managing interest rate risk is to understand the relationships between interest rates, bond prices, the coupon rate, and the bond’s term to maturity. Portfolio managers need to understand that as interest rates rise bond prices decline, and it declines more for low-coupon bonds and longer-term bonds than for the others. In such a scenario, bond portfolio managers can reduce the size and maturity of their portfolio to reduce the impact of interest rate increases. When interest rates decline, bond prices increase and rise more for longer-term bonds and higher coupon bonds. At such times, CFOs can increase the size and maturity of their portfolios to take advantage of the inverse relationship between interest rates and bond prices. LO: 4 Level: Basic Bloomcode: Comprehension AASCB: Analytic IMA: Corporate Finance AICPA: Risk Analysis
8.6
Explain why bond prices and interest rates are negatively related. What are the roles of the coupon rate and the term to maturity in this relation?
Bond prices and interest rates are negatively related because the market rate varies, while the coupon rate is constant over the life of the bond. Thus, as rates increase, demand and bond prices of existing bonds decline, while newer bonds with coupon rates at the current rate are in greater demand. o
For a given change in interest rates, longer-term bonds experience greater price changes (price volatility) than shorter-term bonds. Longer-term bonds have more of their cash flows farther in the future, and their present value of cash flows will be lower due to heavily discounted. In addition, the longer it takes for investors to receive the cash flows, the more uncertainty they have to deal with and hence the more price-volatile the bond will be.
SM 19-267
Lower coupon bonds are more price volatile than higher coupon bonds. The same argument used above also explains this relationship. The lower the coupon on a bond, the greater the proportion of cash flows that investors receive at maturity.LO: 4 Level: Basic Bloomcode: Comprehension AASCB: Analytic IMA: Corporate Finance AICPA: Measurement
8.7
If interest rates are expected to increase, should investors look to long-term bonds or short-term securities? Explain. As interest rates increase, bond prices decrease with longer-term bonds, experiencing a bigger decline than shorter-term securities. So, investors expecting an increase in interest rates should choose short-term securities over long-term securities and reduce their interest rate risk.
LO: 4 Level: Basic Bloomcode: Comprehension AASCB: Analytic IMA: Corporate Finance AICPA: Measurement
8.8
Explain what you would assume the yield curve would look like during economic expansion and why.
At the beginning of an economic expansion, the yield curve tends to be rather steep as the rates begin to rise once the demand for capital is beginning to pick up due to growing economic activity. The yield curve will retain its positive slope during the economic expansion, which reflects the investors’ expectations that the economy will grow in the future and that the inflation rates will also rise in the future.
LO: 5 SM 19-268
Level: Basic Bloomcode: Comprehension AASCB: Analytic IMA: Corporate Finance AICPA: Measurement
8.9
An investor holds a 10-year bond paying a coupon rate of 9 percent. The yield to maturity of the bond is 7.8 percent. Would you expect the investor to be holding a par-value, premium, or discount bond? What if the yield to maturity were 10.2 percent? Explain. Since the bond’s coupon of 9 percent is greater than the yield to maturity, the bond will be a premium bond. As market rates of interest drop below the coupon rate of the 9 percent bond, demand for the bond increases, driving up the price of the bond above face value. If the yield to maturity is at 10.2 percent, then the bond is paying a lower coupon than the going market rate and will be less attractive to investors. The demand for the 9 percent bond will decline, driving its price below the face value. This will be a discount bond.
LO: 2 Level: Basic Bloomcode: Application AASCB: Analytic IMA: Corporate Finance AICPA: Measurement
8.10 a. Investor A holds a 10-year bond, while investor B holds an 8-year bond. If interest rate increases by 1 percent, which investor has the higher interest rate risk? Explain.
SM 19-269
Since A holds the longer-term bond, he or she will face the higher interest rate risk. Longer-term bonds are more price volatile than shorter-term bonds.
b. Investor A holds a 10-year bond paying 8 percent a year, while investor B also has a 10-year bond that pays a 6 percent coupon. Which investor has the higher interest rate risk? Explain.
Investor B will have the higher interest rate risk since lower coupon bonds have a higher interest rate risk than higher coupon bonds of the same maturity. LO: 4 Level: Basic Bloomcode: Analysis AASCB: Analytic IMA: Corporate Finance AICPA: Measurement
Questions and Problems BASIC 8.1
Bond price: BA Corp is issuing a 10-year bond with a coupon rate of 8 percent. The interest rate for similar bonds is currently 6 percent. Assuming annual payments, what is the value of the bond?
Solution: Years to maturity = n = 10 Coupon rate = C = 8% Annual coupon = $1,000 × 0.08 = $80 Current market rate = i = 6% Present value of bond = PB 0 6% 1
2
3
4
5
6
10 Years SM 19-270
├───┼────┼───┼───┼───┼────┼── $80
$80
$80
$80
$80
─────┤
$80
$80 + $1,000
C1 C3 C F 10 C2 10 10 3 2 1 (1 i) (1 i) (1 i) (1 i) 1 1 1 1 F n n C (1 i) $80 (1.06)10 $1,000 n (1 i) (1.06)10 i 0.06 $588.81 $558.39 $1,147.20
PB
LO 2 Bloomcode: Application AASCB: Analytic IMA: Corporate Finance AICPA: Measurement
8.2
Bond price: Pierre Dupont just received a cash gift from his grandfather. He plans to invest in a five-year bond issued by Venice Corp. that pays an annual coupon rate of 5.5 percent. If the current market rate is 7.25 percent, what is the maximum amount Pierre should be willing to pay for this bond?
Solution: 0
7.25%
1
2
3
4
5 Years
├───────┼────────┼───────┼────────┼───────┤ $55
$55
$55
$55
$1,055
Coupon rate = C = 5.5% Annual coupon = $1,000 × 0.055 = $55 Current market rate = i = 7.25% Present value of bond = PB
SM 19-271
1 1 1 (1 i)n 1 (1.0725)5 $1,000 Fn $55 PB C n 0.0725 (1.0725)5 i (1 i) $224.01 $704.72 $928.72 LO 2 Bloomcode: Application AASCB: Analytic IMA: Corporate Finance AICPA: Measurement
8.3
Bond price: Knight, Inc., has issued a three-year bond that pays a coupon rate of 6.10 percent. Coupon payments are made semiannually. Given the market rate of interest of 5.80 percent, what is the market value of the bond?
Solution: Years to maturity = n = 3 Coupon rate = C = 6.1% Frequency of payment = m = 2 Semiannual coupon = $1,000 × (0.061/2) = $30.50 Current market rate = i = 5.8% Present value of bond = PB 0 5.8%
1
2
3
4
5
6
Years
├───┼────┼───┼───┼───┼────┤ $30.50 $30.50 $30.50 $30.50 $30.50 $30.50 + $1,000 1 1 6 i 1 1 2n 1 C F 2n P $30.50 (1.029) $1,000 2 B 6 2 i 0.029 (1.029) 1 i 2 2n 2 $165.77 $842.38 $1,008.15
LO 2 Bloomcode: Application SM 19-272
AASCB: Analytic IMA: Corporate Finance AICPA: Measurement
8.4
Bond price: Regatta, Inc., has seven-year bonds outstanding that pay a 12 percent coupon rate. Investors buying these bonds today can expect to earn a yield to maturity of 8.875 percent. What is the current value of these bonds? Assume annual coupon payments.
Solution: Years to maturity = n = 7 Coupon rate = C = 12% Annual coupon = $1,000 x 0.12 = $120 Current market rate = i = 8.875% Present value of bond = PB 08.875%1
2
3
4
5
6
7
Years
├───┼────┼───┼───┼───┼────┼───┤ $120
$120
$120
$120
$120
$120
$120+ $1,000
1 1 1 n 1 (1 i) (1.08875)7 $1,000 Fn PB C n $120 7 (1 i) 0.08875 i (1.08875) $606.50 $551.44 $1,157.94 LO 2 Bloomcode: Application AASCB: Analytic IMA: Corporate Finance AICPA: Measurement
SM 19-273
8.5
Bond price: You are interested in investing in a five-year bond that pays a 7.8 percent coupon rate with interest to be received semiannually. Your required rate of return is 8.4 percent. What is the most you would be willing to pay for this bond?
Solution: Years to maturity = n = 5 Coupon rate = C = 7.8% Frequency of payment = m = 2 Semiannual coupon = $1,000 × (0.078/2) = $39.00 Current market rate = i = 8.4% Present value of bond = PB
0 8.4% 1
2
3
4
5
6
10 Years
├───┼────┼───┼───┼───┼────┼── $39
$39
$39
$39
$39
─────┤
$39
$39+ $1,000
1 1 10 i 1 1 1 2n C P $39 (1.042) $1,000 2 F 2n B 10 2 i 0.042 (1.042) 1 i 2 2n 2 $313.20 $662.71 $975.91
LO 2 Bloomcode: Application AASCB: Analytic IMA: Corporate Finance AICPA: Measurement
8.6
Zero coupon bonds: Diane Carter is interested in buying a five-year zero coupon bond with a face value of $1,000. She understands that the market interest rate for similar
SM 19-274
investments is 9 percent. Assume annual coupon payments. What is the current value of this bond?
Solution: Years to maturity = n = 5 Coupon rate = C = 0% Current market rate = i = 9% 0 9% 1
2
3
4
5 Years
├───┼────┼───┼───┼───┤ $0
$0
$0
$0
$0+ $1,000
PB
Fmn $1,000 $649.93 5 mn 1.09 i 1 m
LO 1, LO 2 Bloomcode: Application AASCB: Analytic IMA: Corporate Finance AICPA: Measurement
8.7
Zero coupon bonds: Ten-year zero coupon bonds issued by the U.S. Treasury have a face value of $1,000 and interest is compounded semiannually. If similar bonds in the market yield 10.5 percent, what is the value of these bonds?
Solution: Years to maturity = n = 10 Frequency of payment = m = 2 Coupon rate = C = 0% Current market rate = i = 10.5% 010.5%
1
2
3
4
5
6
20
Years ├───┼────┼───┼───┼───┼────┼──
─────┤ SM 19-275
$0
$0
$0
$0
$0
$0
$0 + $1,000
PB
$1,000 Fmn $359.38 mn 1.0525 20 1 i m
LO 1, LO 2 Bloomcode: Application AASCB: Analytic IMA: Corporate Finance AICPA: Measurement
8.8
Zero coupon bonds: Northrop Real Estate Company management is planning to fund a development project by issuing 10-year zero coupon bonds with a face value of $1,000. Assuming semiannual compounding, what will be the price of these bonds if the appropriate discount rate is 14 percent?
Solution: Years to maturity = n = 10 Frequency of payment = m = 2 Coupon rate = C = 0% Current market rate = i = 14% 014% 1
2
3
4
5
6
├───┼────┼───┼───┼───┼────┼── $0
$0
$0
$0
$0
$0
20 Years ─────┤ $0 + $1,000
PB
$1,000 Fmn $258.42 mn 1.07 20 1 i m
LO 1, LO 2 Bloomcode: Application AASCB: Analytic IMA: Corporate Finance AICPA: Measurement SM 19-276
8.9
Yield to maturity: Ruth Hornsby is looking to invest in a three-year bond that makes semiannual coupon payments at a rate of 5.875 percent. If these bonds have a market price of $981.13, what yield to maturity and effective annual yield can she expect to earn?
Solution: Years to maturity = n = 3 Coupon rate = C = 5.875% Frequency of payment = m = 2 Semiannual coupon = $1,000 × (0.05875/2) = $29.375 Yield to maturity = i Present value of bond = PB = $981.13 Use the trial-and-error approach to solve for YTM. Since the bond is selling at a discount, we know that the yield to maturity is higher than the coupon rate. Try YTM = 6%.
1 P C 2 B
F2n 1 i 2 2n 2n i i 1 2 2 1
1 1 6 1 0.03 $1, 000 $981.13 $29.375 1.03 6 0.03
$159.13 $837.48 $996.61
Try a higher rate, say YTM = 6.6%.
SM 19-277
1 P C 2 B
F2n 2n 1 i 2 2n i i 1 2 2 1
1 1 6 1 0.033 $1, 000 $981.13 $29.375 1.033 6 0.033
$157.56 $823.00 $980.56
The YTM is approximately 6.6 percent. Using a financial calculator provided an exact YTM of 6.58 percent Enter
6 N
Answer
$29.375 -$981.13 $1,000 i%
PMT
PV
FV
6.58%
The effective annual yield can be computed as: EAY (1 Quoted rate m)m 1 1 0.06578 2 2 1 (1.03289)2 1 0.06686 6.69% LO 3 Bloomcode: Application AASCB: Analytic IMA: Corporate Finance AICPA: Measurement
8.10
Yield to maturity: Rudy Sandberg wants to invest in four-year bonds that are currently priced at $868.43. These bonds have a coupon rate of 6 percent and make semiannual coupon payments. What is the current market yield on this bond?
SM 19-278
Solution: Years to maturity = n = 4 Coupon rate = C = 6% Frequency of payment = m = 2 Semiannual coupon = $1,000 × (0.06/2) = $30 Yield to maturity = i Present value of bond = PB = $868.43 Use the trial-and-error approach to solve for YTM. Since the bond is selling at a discount, we know that the yield to maturity is higher than the coupon rate. Try YTM = 10%.
1 P C 2 B
F2n 2n 1 i 2 2n i i 1 2 2 1
1 1 8 1 0.05 $1, 000 $868.43 $30 1.05 8 0.05
$193.90 $676.84 $870.74
Try a higher rate, say YTM = 10.1%.
1 P C 2 B
F2n 2n 1 i 2 2n i i 1 2 2 1
1 1 8 $1, 000 1 0.0505 $868.43 $30 * 1.0505 8 0.0505
$193.51 $674.27 $867.78
SM 19-279
The YTM is approximately 10.1 percent. Using a financial calculator provided an exact YTM of 10.08 percent. Enter
8 N
Answer
i%
$30
-$868.43
$1,000
PMT
PV
FV
10.08%
LO 3 Bloomcode: Application AASCB: Analytic IMA: Corporate Finance AICPA: Measurement
8.11
Realized yield: Josh Kavern bought 10-year, 12 percent coupon bonds issued by the U.S. Treasury three years ago at $913.44. If he sells these bonds, for which he paid the face value of $1,000, at the current price of $804.59, what is his realized yield on the bonds? Assume similar coupon-paying bonds make annual coupon payments.
Solution: Purchase price of bond = $913.44 Years investment held = n = 3 Coupon rate = C = 12% Frequency of payment = m = 1 Annual coupon = $1,000 × (0.12) = $120 Realized yield = i Selling price of bond = PB = $804.59 To compute the realized return, either the trial-and-error approach or the financial calculator can be used. Since the price has declined, market rates must have increased. So, the realized return is going to be less than the bond’s coupon. Try rates lower than the coupon rate. Try i = 10%.
SM 19-280
1 1 (1 i)n Fn PB C n i (1 i) 1 1 $913.44 $120 (1.10)3 $804.59 3 0.10 (1.10) $298.42 $604.50 $902.92 Try a lower rate, i = 9.5%. 1 1 n (1 i) Fn PB C n i (1 i) 1 1 3 $913.44 $120 (1.095) $804.59 3 0.095 (1.095) $301.07 $612.82 $913.89 The realized rate of return is approximately 9.5 percent. Using a financial calculator provided an exact yield of 9.52 percent. Enter
3 N
Answer
i%
$120
-$913.44
$804.59
PMT
PV
FV
9.52%
LO 3 Bloomcode: Application AASCB: Analytic IMA: Corporate Finance AICPA: Measurement
8.12
Realized yield: Four years ago, Lisa Stills bought six-year, 5.5 percent coupon bonds issued by the Fairways Corp. for $947.68. If she sells these bonds at the current price of
SM 19-281
$894.52, what will be her realized yield on the bonds? Assume similar coupon-paying bonds make annual coupon payments.
Solution: Purchase price of bond = $947.68 Years investment held = n = 4 Coupon rate = C = 5.5% Frequency of payment = m = 1 Annual coupon = $1,000 × (0.055) = $55 Realized yield = i Selling price of bond = PB = $894.52 To compute the realized return, either the trial-and-error approach or the financial calculator can be used. Since the price has declined, market rates must have increased. So, the realized return is going to be less than the bond’s coupon. Try rates lower than the coupon rate. Try i = 5%. 1 1 (1 i)n Fn PB C n i (1 i) 1 1 4 $947.68 $55 (1.05) $894.52 4 0.05 (1.05) $195.03 $735.92 $930.95 Try a lower rate, i = 4.5%.
SM 19-282
1 1 (1 i)n Fn PB C n i (1 i) 1 1 (1.045)4 $894.52 $947.68 $55 4 0.045 (1.045) $197.31 $750.11 $947.42 The realized rate of return is approximately 4.5 percent. Using a financial calculator provided an exact yield of 4.49 percent. Enter
4 N
Answer
i%
$55
-$947.68
$894.52
PMT
PV
FV
4.49%
LO 3 Bloomcode: Application AASCB: Analytic IMA: Corporate Finance AICPA: Measurement
INTERMEDIATE 8.13
Bond price: The International Publishing Group is raising $10 million by issuing 15-year bonds with a coupon rate of 8.5 percent. Coupon payments will be made annually. Investors buying the bonds today will earn a yield to maturity of 8.5 percent. At what price will the bonds sell in the marketplace? Explain.
Solution: Years to maturity = n = 15 Coupon rate = C = 8.5% Annual coupon = $1,000 × 0.085 = $85 Current market rate = i = 8.5% SM 19-283
Present value of bond = PB 0 8.5% 1
2
3
4
15
├───┼────┼───┼───┼─── $85
$85
$85
Year
─────┤
$85
$85+ $1,000
n = 15;
C = 8.5%;
i = YTM = 8.5%
1 1 1 n 1 (1 i) (1.085)15 $1,000 Fn $85 PB C n 0.085 (1.085)15 i (1 i) $705.86 $294.14 $1,000.00 This answer should have been intuitive. Since the bond is paying a coupon equal to the going market rate of 8.5 percent, the bond should be selling at its par value of $1,000.
Enter
15
8.5%
$85
N
i%
PMT
Answer
$1,000 PV
FV
-$1,000
LO 2 Bloomcode: Application AASCB: Analytic IMA: Corporate Finance AICPA: Measurement
8.14
Bond price: Pullman Corp issued 10-year bonds four years ago with a coupon rate of 9.375 percent. At the time of issue, the bonds sold at par. Today bonds of similar risk and maturity must pay an annual coupon of 6.25 percent to sell at par value. Assuming semiannual coupon payments, what will be the current market price of the firm’s bonds?
Solution: Years to maturity = n = 6 Frequency of payment = m = 2 SM 19-284
Coupon rate = C = 9.375% Semiannual coupon = $1,000 × (0.09375/2) = $46.875 Current market rate = i = 6.25% Present value of bond = PB 0 6.25%
1
2
3
4
12
├───┼────┼───┼───┼───
─────┤
$46.875 $46.875………
$46.875+ $1,000
n = 6; m = 2; C = 9.375%; i = YTM = 6.25% 1 12 1 1 i12 2n 1 (1.03125) F $1, 000 12 (1.03125) 2n $46.875 0.03125 P C i 2 B 2 1 i 2
$463.13 $691.25 $1,154.38 Enter
12
3.125%
$46.875
N
i%
PMT
Answer
$1,000 PV
FV
- $1,154.38
LO 2 Bloomcode: Application AASCB: Analytic IMA: Corporate Finance AICPA: Measurement
8.15
Bond price: Marshall Company is issuing eight-year bonds with a coupon rate of 6.5 percent and semiannual coupon payments. If the current market rate for similar bonds is 8 percent, what will be the bond price? If company management wants to raise $1.25 million, how many bonds does the firm have to sell?
Solution: Years to maturity = n = 8 Frequency of payment = m = 2 SM 19-285
Coupon rate = C = 6.5% Semiannual coupon = $1,000 × (0.065/2) = $32.50 Current market rate = i = 8% Present value of bond = PB 0 8% 1
2
3
4
16
├───┼────┼───┼───┼───
$32.50
Year
─────┤
$32.50………..$32.50
$32.50+ $1,000
1 i 1 1 1 16 1 mn C F 2n P $32.50 (1.04) $1,000 2 B 16 2 i i 0.04 (1.04) 1 2 mn 2 $378.70 $533.91 $912.61
To raise $1.25 million, the firm would have to sell: Number of bonds = $1,250,000 / $912.61 = 1,370 bond contracts Enter
16 N
Answer
4% i%
$32.50 PMT
$1,000 PV
FV
-$912.61
LO 2 Bloomcode: Application AASCB: Analytic IMA: Corporate Finance AICPA: Measurement
8.16
Bond price: Rockne, Inc., has outstanding bonds that will mature in six years and pay an 8 percent coupon semiannually. If you paid $1036.65 today and your required rate of return was 6.6 percent, did you pay the right price for the bond?
SM 19-286
Solution: Years to maturity = n = 6 Frequency of payment = m = 2 Coupon rate = C = 8% Semiannual coupon = $1,000 × (0.08/2) = $40 Current market rate = i = 6.6% Present value of bond = PB 0 6.6%
1
2
3
12
├───────┼────────┼────────┼── $40
$40
─────────┤
$40
$40+ $1,000
1 i 1 1 1 12 1 mn C P $40 (1.033) $1,000 2 F 2n B 12 2 i i 0.033 (1.033) 1 2 mn 2 $391.12 $677.32 $1,068.45
You paid less than what the bond is worth. That was a good price. Enter
Answer
12
3.3%
$40
N
i%
PMT
$1,000 PV
FV
-$1,068.45
LO 2 Bloomcode: Application AASCB: Analytic IMA: Corporate Finance AICPA: Measurement
8.17
Bond price: Nanotech, Inc., has a bond issue maturing in seven years that is paying a coupon rate of 9.5 percent (semiannual payments). Management wants to retire a portion of the issue by buying the securities in the open market. If it can refinance at 8 percent, how much will Nanotech pay to buy back its current outstanding bonds? SM 19-287
Solution: Years to maturity = n = 7 Frequency of payment = m = 2Coupon rate = C = 9.5% Semiannual coupon = $1,000 × (0.095/2) = $47.50 Current market rate = i = 8%
Present value of bond = PB 0 8%
1
2
3
14
├───────┼────────┼────────┼── $47.50
$47.50
──────┤
$47.50
$47.50 + $1,000
1 1 1 14 1 1 i mn F 2n P C $47.50 (1.04) $1,000 2 14 B 2 i i mn 0.04 (1.04) 1 2 2 $501.75 $577.48 $1,079.22
The firm will be willing to pay no more than $1,079.22 for their bond.
Enter
14 N
Answer
4% i%
$47.50 PMT
$1,000 PV
FV
-$1,079.22
LO 2 Bloomcode: Application AASCB: Analytic IMA: Corporate Finance AICPA: Measurement
SM 19-288
8.18
Zero coupon bonds: Kintel, Inc., management wants to raise $1 million by issuing sixyear zero coupon bonds with a face value of $1,000. The company’s investment banker states that investors would use an 11.4 percent discount rate to value such bonds. At what price would these bonds sell in the marketplace? How many bonds would the firm have to issue to raise $1 million? Assume semiannual coupon payments.
Solution: Years to maturity = n = 6 Frequency of payment = m = 2 Coupon rate = C = 0% Current market rate = i = 11.4% Assume semiannual coupon payments. 0 11.4%
1
2
3
4
5
6
12
Year ├───┼────┼───┼───┼───┼────┼── $0
$0
$0
$0
$0
$0
─────┤ $0 $1,000
PB
$1,000 Fmn 12 $514.16 mn 1.057 i 1 m
At the price of $514.16, the firm needs to raise $1 million. To do so, the firm will have to issue: Number of bonds
= $1,000,000 / $514.16 = 1,945 bond contracts
LO 1, LO 2 Bloomcode: Application AASCB: Analytic IMA: Corporate Finance AICPA: Measurement
SM 19-289
8.19
Zero coupon bonds: Rockinghouse Corp. management plans to issue seven-year zero coupon bonds. It has learned that these bonds will sell today at a price of $439.76. Assuming annual coupon payments, what is the yield to maturity on these bonds?
Solution: Years to maturity = n = 7 Coupon rate = C = 0% YTM = i Present value of bond = PB = $439.76 0i=? 1
2
3
4
5
6
7
├───┼────┼───┼───┼───┼────┼───┤ $0
$0
$0
$0
$0
$0
$0+ $1,000
To solve for the YTM, a trial-and-error approach has to be used. Try YTM = 10%. PB
Fn
1 i
n
$1,000
1.10 7
$439.76 $513.16
Try a higher rate, YTM = 12%. $1,000 Fn n 1 i 1.127 $439.76 $452.35 PB
Try YTM=12.5%. $1,000 Fn n 1 i 1.125 7 $439.76 $438.46 PB
The YTM is approximately 12.5 percent. Enter
7 N
Answer
$0 i%
PMT
-$439.76 $1,000 PV
FV
12.453%
LO 1, LO 2 Bloomcode: Application SM 19-290
AASCB: Analytic IMA: Corporate Finance AICPA: Measurement
8.20
Yield to maturity: Electrolex, Inc., has four-year bonds outstanding that pay a coupon rate of 6.6 percent and make coupon payments semiannually. If these bonds are currently selling at $914.89, what is the yield to maturity that an investor can expect to earn on these bonds? What is the effective annual yield?
Solution: Years to maturity = n = 4 Frequency of payment = m = 2Coupon rate = C = 6.6% YTM = i Semiannual coupon payments = $1,000 × (0.066/2) = $33 Present value of bond = PB = $914.89 0i=?
1
2
3
├───────┼────────┼────────┼── $33
$33
$33
8 ─────────┤Yrs. $33+ $1,000
To solve for the YTM, a trial-and-error approach has to be used. Since this is a discount bond, the market rate should be higher than 6.6 percent. Try i = 8% or i/2 = 4%. 1 1 n (1 i) Fn P C B n i (1 i) 1 1 (1.04)8 $1, 000 $914.89 $33 0.04 (1.04)8 $222.18 $730.69 $952.87 SM 19-291
Try a higher rate, i = 9%, i/2 = 4.5%. 1 1 n (1 i) Fn P C B n i (1 i) 1 1 (1.045)8 $1, 000 $914.89 $33 0.045 (1.045)8 $217.66 $703.19 $920.85 Try a higher rate, i = 9.2%, i/2 = 4.6%. 1 1 n (1 i) Fn P C B n i (1 i) 1 1 (1.046)8 $1, 000 $914.89 $33 0.046 (1.046)8 $216.78 $697.82 $914.60 The yield to maturity is approximately 9.2 percent. The effective annual yield can be computed as: EAY (1 Quoted rate m)m 1 1.046 1 2
0.0941 9.41%
Enter
8
$33
N
i%
Answer
-$914.89
$1,000
PV
FV
PMT
4.5954%
The effective annual yield can be computed as: EAY (1 Quoted rate m)m 1 1.045954 1 2
0.09399 9.4% SM 19-292
LO 3 Bloomcode: Application AASCB: Analytic IMA: Corporate Finance AICPA: Measurement 8.21
Yield to maturity: Serengeti Corp. has five-year bonds outstanding that pay a coupon of 8.8 percent. If these bonds are priced at $1,064.86, what is the yield to maturity on these bonds? Assume semiannual coupon payments. What is the effective annual yield?
Solution: Years to maturity = n = 5 Frequency of payment = m = 2 Coupon rate = C = 8.8% Current market rate = i Semiannual coupon payments = $1,000 x (0.088/2) = $44 Present value of bond = PB = $1,064.86 0 i=?
1
2
3
├───────┼────────┼────────┼── $44
44
$44
10 ─────────┤Yrs. $44+ $1,000
To solve for the YTM, a trial-and-error approach has to be used. Since this is a premium bond, the market rate should be lower than 8.8 percent. Try i = 7% or i/2 = 3.5%. 1 1 n (1 i) FV PB C n i (1 i) 1 1 (1.035)10 $1,000 $1,064.86 $44 10 0.035 (1.035) $365.93 $708.92 $1,074.85 Try a higher rate, i = 7.2%, i/2 = 3.6%. SM 19-293
1 1 (1 i)n Fn PB C n i (1 i) 1 1 (1.036)10 $1,000 $1,068.86 $44 10 0.036 (1.036) $364.09 $702.11 $1,066.20 The YTM is approximately 7.2 percent. The effective annual yield can be computed as: EAY (1 Quoted rate m)m 1 1.036 1 2
0.0733 7.33%
Enter
10 N
i%
Answer
$44
-$1,064.86
$1,000
PMT
PV
FV
3.6156%
The effective annual yield can be computed as: EAY (1 Quotedrate m)m 1 1.036156 1 2
0.0736 7.36%
LO 3 Bloomcode: Application AASCB: Analytic IMA: Corporate Finance AICPA: Measurement
8.22
Yield to maturity: Adrienne Dawson is planning to buy 10-year zero coupon bonds issued by the U.S. Treasury. If these bonds have a face value of $1,000 and are currently selling at $404.59, what is the expected return on them? Assume that interest compounds semiannually on similar coupon paying bonds.
SM 19-294
Solution: Years to maturity = n = 10 Frequency of payment = m = 2Coupon rate = C = 0% Current market rate = i Assume annual coupon payments. Present value of bond = PB = $404.59 0 i=?
1
2
3
20 Yrs.
├───────┼────────┼────────┼── $0
$0
─────────┤
$0
$0+ $1,000
To solve for the YTM, a trial-and-error approach has to be used. Try YTM = 10%. $1,000 Fmn mn 1.0520 1 i m $404.59 $376.89 PB
Try a lower rate, YTM = 9%. $1,000 Fmn mn 1.04520 1 i m $404.59 $414.64 PB
Try YTM=9.25%. $1,000 Fmn mn 1.04625 20 1 i m $404.59 $404.85 PB
The YTM is approximately 9.25 percent. EAY (1 Quoted rate m)m 1 1.04625 1 2
0.09464 9.46%
The expected return from this investment is 9.46 percent. Enter
20 N
Answer
$0 i%
PMT
-$404.59
$1,000
PV
FV
4.63% SM 19-295
The effective annual yield can be computed as: EAY (1 Quoted rate m)m 1 1.046283 1 2
0.0947 9.47%
LO 3 Bloomcode: Application AASCB: Analytic IMA: Corporate Finance AICPA: Measurement
8.23
Realized yield: Brown & Co. issued seven-year bonds two years ago that can be called after two years. The bonds make semiannual coupon payments at a coupon rate of 7.875 percent. Each bond has a market value of $1,053.40, and the call price is $1,078.75. If an investor purchased the bonds at par value when they were originally issued and the bonds are called by the firm today, what is the investor’s realized yield?
Solution: Purchase price of bond = $1,000 Years investment held = n = 2 Coupon rate = C = 7.875% Frequency of payment = m = 2 Annual coupon = $1,000 × (0.07875/2) = $39.375 Realized yield = i Call price of bond = CP = $1,078.75 Current market value = $1,053.40
To compute the realized return, either the trial-and-error approach or the financial calculator can be used. Since the price has increased, market rates must have decreased. So, the realized return is going to be greater than the bond’s coupon. Try rates higher than the coupon rate.
SM 19-296
Try i = 10%, or i/2 = 5%. 1 1(1 i m n C 2 ) CP P B (1 i ) m n i 2 2 2 1 1 4 (1.05) $1, 078.75 $1, 000 $39.375 4 0.05 (1.05) $139.62 $887.49 $1, 027.11
Try a higher rate, i = 11.48% or i/2 = 5.74%. 1 1 (1 i 2 )mn CP P B i (1 i )mn 2 2 2 1 1 (1.0574)4 $1, 078.75 $1, 000 $39.375 4 0.0574 (1.0574) $137.25 $862.91 $1, 000.16 C
EAY = (1 + Quoted rate m)m -1 = 1.0574 -1 2
= 0.11809 = 11.81% The realized rate of return is approximately 11.81 percent. Using a financial calculator provided an exact yield of 11.49 percent.
Enter
4 N
Answer
$39.375 i%
PMT
-$1,000 $1,078.75 PV
FV
5.74%
The effective annual yield can be computed as: SM 19-297
EAY = (1+ Quoted rate m) m -1 = 1.0574 -1 2
= 0.1181 = 11.81% LO 3 Bloomcode: Application AASCB: Analytic IMA: Corporate Finance AICPA: Measurement
8.24
Realized yield: Trevor Price bought 10-year bonds issued by Harvest Foods five years ago for $936.05. The bonds make semiannual coupon payments at a rate of 8.4 percent. If the current price of the bonds is $1,048.77, what is the yield that Trevor would earn by selling the bonds today?
Solution: Purchase price of bond = $936.05 Years investment held = n = 5 Coupon rate = C = 8.4% Frequency of payment = m = 2 Annual coupon = $1,000 × (0.084/2) = $42 Realized yield = i Selling price of bond = PB = $1,048.77
To compute the realized return, either the trial-and-error approach or the financial calculator can be used. Since the price has increased, market rates must have decreased. So, the realized return is going to be greater than the bond’s coupon. Try rates higher than the coupon rate.
Try i = 11%, or i/2 = 5.5%.
SM 19-298
1 1(1 i mn C ) 2 F 2n P B (1 i )mn i 2 2 2 1 1 (1.055)10 $1,048.77 $936.05 $42 10 0.055 (1.055) $316.58 $613.98 $930.56
Try a lower rate, i = 10.8% or i/2 = 5.4%. 1 1(1 i mn C 2 ) FV P B (1 i )mn i 2 2 2 1 1 10 $936.05 $42 (1.054) $1,048.77 10 0.054 (1.054) $318.10 $619.83 $937.94
EAY (1 Quoted rate m)m 1 1.054 1 2
0.1109 11.09%
The realized rate of return is approximately 11.09 percent. Using a financial calculator provided an exact yield of 11.14 percent.
Enter
10 N
Answer
i%
$42
-$936.05
PMT
PV
$1,048.77 FV
5.425%
The effective annual yield can be computed as: SM 19-299
EAY (1 Quoted rate m)m 1 1.05425 1 2
0.1114 11.14%
LO 3 Bloomcode: Application AASCB: Analytic IMA: Corporate Finance AICPA: Measurement
8.25
Realized yield: You bought a six-year bond issued by Runaway Corp. four years ago. At that time, you paid $974.33 for the bond. The bond pays a coupon rate of 7.375 percent, and coupon payments are made semiannually. Currently, the bond is priced at $1,023.56. What yield can you expect to earn on this bond if you sell it today?
Solution: Purchase price of bond = $974.33 Years investment held = n = 4 Coupon rate = C = 7.375% Frequency of payment = m = 2 Annual coupon = $1,000 × (0.07375/2) = $36.875 Realized yield = i Selling price of bond = PB = $1,023.56 To compute the realized return, either the trial-and-error approach or the financial calculator can be used. Since the price has increased, market rates must have decreased. So, the realized return is going to be greater than the bond’s coupon. Try rates higher than the coupon rate. Try i = 9%, or i/2 = 4.5%.
SM 19-300
1 1 i mn (1 2) F 2n P B (1 i ) mn i 2 2 2 1 1 (1.045)8 $1,023.56 $974.33 $36.875 0.045 (1.045)8 $243.22 $719.75 $962.98 C
Try a lower rate, i = 8.6% or i/2 = 4.3%. P
C
1 1 (1 i mn 2)
(1 i ) mn 2 1 1 (1.043)8 $1,023.56 $974.33 $36.875 0.043 (1.043)8 $245.22 $730.87 $976.09 B
F 2n
2
i
2
EAY (1 Quoted rate m)m 1 1.043 1 2
0.08785 8.78%
The realized rate of return is approximately 8.78 percent. Using a financial calculator provided an exact yield of 8.84 percent.
Enter
8 N
Answer
i%
$36.875
-$974.33
$1,023.56
PMT
PV
FV
4.327%
The effective annual yield can be computed as: EAY (1 Quoted rate m)m 1 1.04327 1 2
0.0884 8.84%
LO 3 SM 19-301
Bloomcode: Application AASCB: Analytic IMA: Corporate Finance AICPA: Measurement
ADVANCED 8.26
Lopez Information Systems management is planning to issue 10-year bonds. The going market yield for such bonds is 8.125 percent. Assume that coupon payments will be made semiannually. Management is trying to decide between issuing an 8 percent coupon bond or a zero coupon bond. Lopez needs to raise $1 million. a.
What will be the price of an 8 percent coupon bond?
b.
How many 8 percent coupon bonds would have to be issued?
c.
What will be the price of a zero coupon bond?
d.
How many zero coupon bonds will have to be issued?
Solution: a.
Years to maturity = n = 10 Frequency of payment = m = 2 Coupon rate = C = 8.00% Semiannual coupon = $1,000 × (0.08/2) = $40 Current market rate = i = 8.125% Present value of bond = PB 0
8.125%
1
2
3
20
├───────┼────────┼────────┼── $40
$40
─────────┤
$40
$40+ $1,000
1 1 i 1 m n 1 1 20 F 2n (1.040625) $1,000 P C $40 2 B 20 2 i 0.040625 (1.040625) 1 i 2 m n 2 $540.62 $450.94 $991.56
SM 19-302
The firm can sell these bonds at $991.56.
Enter
20
4.0625%
$40
N
i%
PMT
$1,000 PV
Answer
b.
FV
-$991.55
Amount needed to be raised = $1,000,000 Number of bonds sold = $1,000,000 / $991.55 = 1,009 bond contracts
c.
Years to maturity = n = 10 Coupon rate = C = 0% Current market rate = i = 8.125% 0 8.125%
1
2
3
4
5
6
20
├───── ┼────┼───┼───┼───┼────┼── $0
$0
$0
$0
$0
─────┤
$0
$0+ $1,000
PB
Fmn
1 i
Enter
mn
m
1.040625 20
$450.94
20
4.0625%
N
i%
Answer
d.
$1,000
$0 PMT
$1,000 PV
FV
-$450.94
At the price of $450.94, the firm needs to raise $1 million. To do so, the firm will have to issue: Number of bonds sold = $1,000,000 / $450.94 = 2,218 bond contracts (Rounded)
LO 1, LO 2 Bloomcode: Analysis AASCB: Analytic SM 19-303
IMA: Corporate Finance AICPA: Measurement
8.27
Showbiz, Inc., has issued eight-year bonds with a coupon of 6.375 percent and semiannual coupon payments. The market’s required rate of return on such bonds is 7.65 percent. a.
What is the market price of these bonds?
b.
If the above bond is callable after five years at an 8.5 percent premium on the face value, what is the expected return on this bond?
Solution: a.
Years to maturity = n = 8 Frequency of payment = m = 2 Coupon rate = C = 6.375% Semiannual coupon = $1,000 × (0.06375/2) = $31.875 Current market rate = i = 7.65% Present value of bond = PB 0 7.65%
1
2
3
16 ├───────┼────────┼────────┼── $31.875
$31.875
─────────┤Yrs.
$31.875
$31.875+ $1,000
1 1 16 i mn 1 1 1 C P $31.875 (1.03825) $1,000 2 F 2n B 16 2 i 0.03825 (1.03825) 1 i 2 mn 2 $376.26 $548.49 $924.75
The firm can sell these bonds at $924.75. SM 19-304
b.
Purchase price of bond = $924.75 Years investment held = n = 5 Coupon rate = C = 6.375% Semiannual coupon = $1,000 × (0.06375/2) = $31.875 Frequency of payment = m = 2 Realized yield = i Call price of bond = CP = $1,000 × (1.085) = $1,085.00 To compute the expected return, either the trial-and-error approach or the financial calculator can be used. Try rates higher than the coupon rate. Try i = 8%, or i/2 = 4%. 1 1 (1 i 2)mn CP PB mn 2 i 2 (1 i 2) 1 1 (1.04)10 $1, 085 $924.75 $31.875 10 0.04 (1.04) $258.54 $732.99 $991.53 C
Try a higher rate, i = 9.67% or i/2 = 4.835%. 1 1 (1 i 2) mn CP PB mn 2 i 2 (1 i 2) 1 1 $1, 085 (1.04835)10 $924.75 $31.875 10 0.04835 (1.04835) $248.11 $676.65 $924.77 C
The realized rate of return is approximately 9.67% percent. Using a financial calculator provided an exact yield of 9.6705 percent.
SM 19-305
Enter
10
$31.875
N Answer
i%
PMT
-$924.75 $1,085 PV
FV
4.835%
The effective annual yield can be computed as: EAY (1 Quoted rate m)m 1 1.04835 1 2
0.0990 9.90%
LO 2, LO 3 Bloomcode: Application AASCB: Analytic IMA: Corporate Finance AICPA: Measurement
8.28
Peabody Corp. has seven-year bonds outstanding. The bonds pay a coupon of 8.375 percent semiannually and are currently worth $1,063.49. The bonds can be called in three years at price of $1,075. a.
What is the yield to maturity of these bonds?
b.
What is the effective annual yield?
c.
What is the realized yield on the bonds if they are called?
d.
If you plan to invest in one of these bonds today, what is the expected yield on the investment? Explain.
Solution: a.
Years to maturity = n = 7 Frequency of payment = m = 2 Coupon rate = C = 8.375% Current market rate = i Semiannual coupon payments = $1,000 × (0.08375/2) = $41.875 Present value of bond = PB = $1,063.49
SM 19-306
0i=?
1
2
3
├───────┼────────┼────────┼── $41.875
$41.875
$41.875
14 Yrs. ─────────┤ $41.875+ $1,000
To solve for the YTM, a trial-and-error approach has to be used. Since this is a premium bond, the market rate should be lower than 8.375 percent. Try i = 8% or i/2 = 4%. 1 1(1 i n 2) P C
F 2n
(1 i ) mn 2 1 1 $1,063.49 $41.875 (1.04)14 $1,000 14 0.04 (1.04) $442.33 $577.48 $1,019.81 B
i
2
Try a lower rate, i = 7.2%, or i/2 = 3.6%. 1 1 (1 i 2)n F 2n PB C i (1 i ) mn 2 2 1 1 14 $1,063.49 $41.875 (1.036) $1,000 14 0.036 (1.036) $454.24 $609.49 $1,063.73 The yield-to maturity is approximately 7.2 percent.
b.
The effective annual yield can be computed as: EAY (1 Quoted rate m)m 1 1.036 1 2
0.073296 7.3%
SM 19-307
Enter
14
$41.875
N
i%
Answer
PMT
-$1,063.49 $1,000 PV
FV
3.602%
The effective annual yield can be computed as: EAY (1 Quoted rate m)m 1 1.036021 1 2
0.073340 7.3%
c.
Purchase price of bond = $1,063.49 Years investment held = n = 3 Coupon rate = C = 8.375% Semiannual coupon payments = $1,000 x (0.08375/2) = $41.875 Frequency of payment = m = 2 Realized yield = i Selling price of bond = PB = $1,075
To compute the realized return, either the trial-and-error approach or the financial calculator can be used. Since the price has increased, market rates must have decreased. So, the realized return is going to be higher than the bond’s coupon. Try rates higher than the coupon rate. Try i = 9%, or i/2 = 4.5%. 1 F2n C 1 (1 i 2)mn P B mn 2 i 2 (1 i 2) 1 1 (1.045)6 $1, 075 $1, 063.49 $41.875 6 0.045 (1.045)
$215.99 $825.49 $1, 041.48
Try a lower rate, i = 8.2% or i/2 = 4.1%. SM 19-308
1 F2n C 1 (1 i 2)mn P B mn 2 i 2 (1 i 2) 1 1 (1.041)6 $1, 075 $1, 063.49 $41.875 6 0.041 (1.041)
$218.80 $844.70 $1, 063.50
EAY (1 Quoted rate m)m 1 1.041 1 2
0.08368 8.368%
The realized rate of return is approximately 8.37 percent. Using a financial calculator provided an exact yield of 8.2 percent.
Enter
6 N
i%
Answer
$41.875 -$1,063.49
$1075
PMT
FV
PV
4.1%
The effective annual yield can be computed as: EAY (1 Quoted rate m)m 1 1.04100 1 2
0.08368 8.368%
d.
Purchase price of bond = PB = $1,063.49 Years to maturity = n =7 Coupon rate = C = 8.375% Semiannual coupon payments = $1,000 × (0.08375/2) = $41.875 SM 19-309
Frequency of payment = m = 2 Maturity value = FV = $1,000 Use the trial-and-error approach to compute the yield to maturity. Since we have a premium bond, market rates are lower than the bond’s coupon. Try i = 7%, or i/2 = 3.5%. 1 C 1(1 i )mn F2n 2 PB (1 i )mn i 2 2 2 1 1 (1.035)14 $1, 000 $1, 063.49 $41.875 0.035 (1.035)14 $457.30 $617.78 $1, 075.08
Try a higher rate, i = 7.2%, or i/2 = 3.6%. 1 1 (1 i 2)n F 2n PB C i (1 i ) mn 2 2 1 1 14 $1,063.49 $41.875 (1.036) $1,000 14 0.036 (1.036) $454.24 $609.49 $1,063.73 The expected yield is approximately 7.2 percent which is the same as the yield to maturity obtained in (a). If the bond is not called and is held to maturity, then the expected yield is the yield to maturity. EAY (1 Quoted rate m)m 1 1.036 1 2
= 0.073296 = 7.33%
SM 19-310
The expected yield is approximately 7.33 percent. Using a financial calculator provided an exact yield 7.334 percent.
Enter
14
$41.875 -$1,063.49 $1,000
N
i%
Answer
PMT
PV
FV
3.602%
The effective annual yield can be computed as: EAY (1 Quoted rate m)m 1 1.03602 1 2
0.073337 7.33% LO 3 Bloomcode: Analysis AASCB: Analytic IMA: Corporate Finance AICPA: Measurement
8.29
The Maryland Department of Transportation has issued 25-year bonds that make semiannual coupon payments at a rate of 9.875 percent. The current market rate for similar securities is 11 percent. a.
What is the current market value of one of these bonds?
b.
What will be the bond’s price if rates in the market (i) decrease to 9 percent or (ii) increase to 12 percent?
c.
Refer to your answers in part b. How do the interest rate changes affect premium bonds and discount bonds?
d.
Suppose the bond were to mature in 12 years. How do the interest rate changes in part b affect the bond prices?
Solution: a.
Years to maturity = n = 25
SM 19-311
Frequency of payment = m = 2 Coupon rate = C = 9.875% Semiannual coupon = $1,000 × (0.09875/2) = $49.375 Current market rate = i = 11% Present value of bond = PB 0 11%
1
2
3
50 Yrs.
├───────┼────────┼────────┼── $49.375
$49.375
─────────┤
$49.375
$49.375+ $1,000
1 1 50 i 1 2n 1 1 F 2n $1,000 (1.055) P C $49.375 2 B 50 2 i 0.055 (1.055) 1 i 2 m n 2 $835.99 $68.77 $904.76
The Maryland bonds will sell at $904.76.
Enter
50
5.5% $49.375 N
i
PMT
Answer
b.
(i)
$1,000 PV
FV
-$904.76
Current market rate = i = 9%
1 1 50 i 1 2n 1 1 C F 2n $1,000 (1.045) P $49.375 2 B 50 2 i 0.045 (1.045) 1 i 2 m n 2 $975.75 $110.71 $1,086.46
The Maryland bonds will increase in price to sell at $1,086.46. Enter
50
4.5% $49.375 N
Answer
i
PMT
$1,000 PV
FV
-$1,086.46 SM 19-312
(ii)
Current market rate = i = 12%
1 1 1 i 2n 1 1 50 F 2n P C $49.375 (1.06) $1,000 2 B 50 2 i 0.06 (1.06) 1 i 2 m n 2 $778.24 $54.29 $832.53
The Maryland bonds will drop in price to $832.53. Enter
50
6% $49.375 N
i
$1,000
PMT
Answer
c.
PV
FV
-$832.53
Bonds, in general, decrease in price when interest rates go up. When interest rates decrease, bond prices increase.
d.
(i)
Current market rate = i = 9% Term to maturity = 12 years
1 1 24 i 1 1 1 2n C F 2n $1,000 (1.045) P $49.375 2 B 24 2 i 0.045 (1.045) 1 i 2 m n 2 $715.71 $347.70 $1,063.42
The Maryland bonds will increase in price to sell at $1,063.42. Enter
24
4.5%
N
i
$49.375
Answer
(ii)
PMT
$1,000 PV
FV
-$1,063.42
Current market rate = i = 12%
SM 19-313
1 i 1 2n 1 1 24 1 C P $49.375 (1.06) $1,000 2 F 2n B 24 2 i 0.06 (1.06) 1 i 2 m n 2 $619.67 $246.98 $866.65
The Maryland bonds will drop in price to $866.65. Enter
24 N
6% i
$49.375 PMT
Answer
$1,000 PV
FV
-$866.65
With shorter maturity, bond prices react the same way as in part b, but to a lesser extent. When interest rates increase, the bond’s price declines; but the decline in price is less than that for a longer-term bond. When interest rates decrease, bond prices increase with longer-term bonds, increasing more than shorter-term bonds. LO 2, LO 3, LO 4 Bloomcode: Application AASCB: Analytic IMA: Corporate Finance AICPA: Measurement
8.30
Rachette Corp. has 18-year bonds outstanding. These bonds, which pay interest semiannually, have a coupon rate of 9.735 percent and a yield to maturity of 7.95 percent. a. Compute the current price of these bonds.
b. If the bonds can be called in five years at a premium of 13.5 percent over par value, what is the investor’s realized yield? c. If you bought one of these bonds today, what is your expected rate of return? Explain.
Solution: a.
Years to maturity = n = 18 Frequency of payment = m = 2
SM 19-314
Coupon rate = C = 9.735% Semiannual coupon = $1,000 × (0.09735/2) = $48.675 Current market rate = i = 7.95% Present value of bond = PB 0
1
2
3
36
├───────┼────────┼────────┼── $48.675
$48.675
─────────┤Yrs.
$48.675
$48.675+ $1,000
1 1 1 1(1.03975)36 $1, 000 F i 1 2 2n PB C 2 2n 2n $48.675 36 i 0.03975 (1.03975) 1 i 2 2 $923.56 $245.79 $1,169.34
The bond’s current price is at $1,169.34.
Enter
36
3.975%
$48.675
N
i
PMT
Answer
b.
$1,000 PV
FV
-$1,169.34
Purchase price of bond = $1,169.34 (value today); Call price = $1,135
To compute the realized return, either the trial-and-error approach or the financial calculator can be used.
Try rates lower than the coupon rate.
Try i = 8%, or i/2 = 4%.
SM 19-315
1 C 1(1 i )mn F2n 2 PB (1 i )mn i 2 2 2 1 1 (1.04)10 $1,135 $1,169.34 $48.675 0.04 (1.04)10 $394.80 $766.77 $1,161.56
Try a lower rate, i = 7.8% or i/2 = 3.9%. 1 C 1(1 i )mn F2n 2 PB (1 i )mn i 2 2 2 1 1 (1.039)10 $1,135 $1,169.34 $48.675 0.039 (1.039)10 $396.77 $774.18 $1,170.95 EAY (1 Quoted rate m)m 1 1.039 1 2
0.07952 7.95% The realized rate of return is approximately 7.95 percent. Using a financial calculator provided an exact yield of 7.834 percent.
Enter
10 N
Answer
$48.675 -$1,169.34 $1,135.00 i
PMT
PV
FV
3.917%
The effective annual yield can be computed as:
SM 19-316
EAY (1 Quoted rate m)m 1 1.03917 1 2
0.07987 7.99%
c.
Purchase price of bond = PB = $1,169.34 Years to maturity = n = 18 Coupon rate = C = 9.735% Semiannual coupon = $1,000 × (0.09735/2) = $48.675 Frequency of payment = m = 2 Maturity value = FV = $1,000
Use the trial-and-error approach to compute the yield to maturity. Since we have a premium bond, market rates are lower than the bond’s coupon. Try i = 8%, or i/2 = 4.0%. 1 C 1(1 i )mn F2n 2 PB (1 i )mn i 2 2 2 1 1 (1.04)36 $1, 000 $1,169.34 $48.675 0.04 (1.04)36 $920.36 $243.67 $1,164.03
Try i = 7.9%, or i/2 = 3.95%.
SM 19-317
1 1 i mn (1 C ) F2n 2 PB (1 i )mn i 2 2 2 1 1 (1.0395)36 $1, 000 $1,169.34 $48.675 0.0395 (1.0395)36 $926.77 $247.92 $1,174.69
Thus the expected yield is between 7.9 percent and 8 percent. Using a financial calculator provided an exact yield of 7.95 percent.
Enter
36
$48.675
N
i
Answer
PMT
-$1,169.34 $1,000 PV
FV
3.975%
The effective annual yield can be computed as: EAY (1 Quoted rate m)m 1 1.03975 1 2
= 0.08108 = 8.11% LO 2, LO 3, LO 4 Bloomcode: Application AASCB: Analytic IMA: Corporate Finance AICPA: Measurement
8.31
Zippy Corporation just sold $30 million of convertible bonds with a conversion ratio of 40. Each $1,000 bond is convertible into 25 shares of Zippy’s stock.
a.
What is the conversion price of Zippy’s stock?
SM 19-318
b.
If the current price of Zippy’s stock is $15 and the company’s annual stock return is normally distributed with a standard deviation of $5, what is the probability that investors will find it attractive to convert the bond into Zippy stock in the next year?
Solution: a.
The conversion price is $1,000/40 = $25.
b.
The stock price would have to increase by approximately two standard deviations (2 $5 = $10) for the price to increase to $25 and for conversion to become attractive to the investors. From Chapter 7we know that 95% of possible outcomes fall within two standard deviations of the mean (average) value in a normal distribution. This means that there is a 5 percent chance that the stock price will move up or down by $10 or more. Since the normal distribution is symmetric, this means that there is only a 2.5 percent chance that Zippy’s stock price will increase enough for it to become attractive for the investors to exercise the conversion option in the next year.
LO 1 Bloomcode: Application AASCB: Analytic IMA: Corporate Finance AICPA: Measurement
SM 19-319
Sample Test Problems 8.1
Seven years ago Eastern Corporation issued 20-year bonds that had a $1000 face value, paid interest annually, and that had a coupon rate of 7 percent. If the market rate of interest is 5.5 percent today, what is the current market price of an Eastern Corporation bond? Are these bonds selling at a premium or discount? (LO 2)
Solution: Years to maturity = n = 13 Coupon rate = C = 7% Annual coupon payment = $1,000 × 0.07 = $70 Current market rate = i = 5.5% Present value of bond = PB 0 1 2 3 4 ├───┼────┼───┼───┼─── $70 $70 $70 $70
13 ─────┤ $70 $1,000
1 1 1 1 n 13 (1 i) (1.055) $1,000 F $70 PB C n i (1 i) 0.055 (1.055)13 $498.56 $638.19 $1,136.75 These bonds are selling at a premium. Bloomcode: Application AASCB: Analytic IMA: Corporate Finance AICPA: Measurement
SM 19-320
8.2
You are considering investing in a 10-year zero coupon bond that compounds interest semiannually. If the current market rate is 5.65 percent, what is the maximum price you should have pay for this bond? (LO 2) Solution: Years to maturity = n = 10 Coupon rate = C = 0% Current market rate = i = 5.65% Frequency of payments = m = 2
0 1 2 3 4 5 6 ├───┼────┼───┼───┼───┼────┼── $0 $0 $0 $0 $0 $0 PB
Fmn 1 i
m
mn
$1, 000 2 x10
1 0.0565 / 2
20 ─────┤ $0 $1,000
$572.83
Bloomcode: Application AASCB: Analytic IMA: Corporate Finance AICPA: Measurement
8.3
Bigbox, Inc. has bonds outstanding that will mature in eight years. These bonds pay interest semiannually and have a coupon rate of 4.6 percent. If the bonds are currently selling at $888.92, what is the yield to maturity that an investor who buys them today can expect to earn? What is the effective annual yield? (LO 3) Solution: Years to maturity = n = 8 Coupon rate = C = 4.6% Current market rate = i Semiannual coupon payment = $1,000 × (0.046/2) = 23 Present value of bond = PB = $888.92 0 1 2 3 ├───────┼────────┼────────┼──
16 ─────────┤ SM 19-321
(888.92)
$23
$23
$23
$23 $1,000 To solve for the YTM, a trial-and-error approach has to be used. Since this bond is selling at a discount, we know that the market rate is higher than 4.6 percent.
SM 19-322
Try i = 6% or i/2 = 3%. 1 n 1 (1 i ) 2 FV P C B i i n (1 2) 2 1 1 16 $1,000 (1.03) $23 0.03 (1.03)16 $288.91 $623.16 $912.17 $888.92 Try a higher rate, i = 7%, i/2 = 3.5%. 1 n 1 (1 i ) FV 2 P C B i i n (1 2) 2 1 1 16 $23 (1.035) $1,000 0.035 (1.035)16 $278.17 $576.71 $854.57 $888.92 Try a lower rate, i = 6.4%, i/2 = 3.2%. 1 1 n (1 i) FV PB C (1 i)n i 1 1 (1.032)16 $1, 000 $882.92 $23 0.032 (1.032)16 $284.54 $604.12 $888.66 $888.92 The YTM is approximately 6.4 percent.
SM 19-323
SM 19-324
Using a financial calculator provides an exact YTM of 6.395 percent Enter
16 N
Answer
i% 6.395%
$23 PMT
-$888.92 PV
$1,000 FV
The effective annual yield can be computed as: EAY=(1 Quoted rate m)m 1 1 0.06395 2 1 2
(1.03198)2 1 0.064972 6.497% Bloomcode: Application AASCB: Analytic IMA: Corporate Finance AICPA: Measurement
8.4
Given a change in market interest rates, which will change more; the market price of a bond with 20 years until maturity or the market price of a bond with 5 years until maturity? Assume all the characteristics of these bonds are identical except the maturity dates. (LO 4) Solution:
The price of the bond with 20 years to maturity will change more. The cash flows from the longer term bond are paid further in the future. As a result, these cash flows are more heavily discounted and therefore more sensitive to interest rate changes. Bloomcode: Analysis AASCB: Analytic IMA: Corporate Finance AICPA: Measurement
8.5
Which of the following classes of securities is likely to have the lowest corporate borrowing cost? (LO 5) a. AAA rated bonds. b. A rated bonds. c. BB rated bonds. d. C rated bonds. e. All of the above will have the same corporate borrowing cost. SM 19-325
Solution: a. The issuers of AAA rated bonds should have the lowest borrowing cost because their default risk premium is lower than issuers with weaker credit ratings. Bloomcode: Comprehension AASCB: Analytic IMA: Corporate Finance AICPA: Measurement
Chapter 9 Stock Valuation Before You Go On Questions and Answers
Section 9.1
1. What is NASDAQ?
The National Association of Securities Automatic Quotation (NASDAQ) system is one of the largest electronic stock markets in the world, listing over three thousand companies. It was created in 1971 by the National Assoc. of Securities Dealers
2. How do dealers differ from brokers?
A dealer differs from a broker in that a dealer takes ownership of assets and is exposed to inventory risk, while a broker only facilitates a transaction on behalf of a client. Unlike SM 19-326
brokers, dealers are subject to capital risk, because they must finance their inventories of securities.
3. List the major stock market indexes, and explain what they tell us.
The major U.S. Stock market indexes are the Dow Jones Industrial Average, New York Stock Exchange, Standard & Poor’s 500 Index, and NASDAQ Composite Index. A stock market index is essentially a listing of stocks that bear some commonality, such as being traded on the same market exchange. They are used to measure the stock market performance and many are used to benchmark the performance of portfolios, such as mutual funds.
4. What does the price-earnings ratio tell us? The price-earnings ratio is the firm’s current price divided by the current earnings. When valuing a company’s stock, it is useful to look at a company’s ratio and compare it to that of similar firms. In general, a high P/E ratio means high projected earnings in the future. In addition, it also tells us how much investors are willing to pay per dollar of earnings.
5. Why do some people view preferred stock as a special type of a bond rather than a stock?
Just like debt, preferred stock is often callable and may be convertible into common stock. Also, like debt, it has ―credit‖ ratings that are similar to those issued to bonds. But most important, preferred stock has no voting rights and just like bonds pays fixed dividends. For these reasons, many analysts treat preferred stock as a special kind of debt rather than as an equity. Section 9.2 1. What is the general formula used to calculate the price of a share of a stock? What does it mean?
SM 19-327
The general formula developed to value a share of stock is as follows: P0
D1 D3 D5 D2 D4 3 2 4 (1 R) (1 R) (1 R) (1 R)5 (1 R)
It says that the price of a share of stock is the present value of all expected future dividends, or: Stock price = PV (all future cash dividends).
2. What are growth stocks, and why do they typically pay little on dividends?
Growth stocks are defined as equity in any company whose earnings are growing faster than the average firm and the higher growth rate is expected to continue for some time. Instead of paying dividends, these firms reinvest the earnings back into the firm to pursue other highreturn investment opportunities. Section 9.3 1. What three different models are used to value stocks based on different dividend patterns?
Based on dividend patterns, we can use the following three models to value stock: (1) zerogrowth dividend model, (2) constant-growth dividend model, or (3) supernormal dividend growth model.
2. Explain why the growth rate g must always be less than the rate of return R. For the valuation equation to have any meaning, the firm’s dividend growth rate g must be less than the rate of return R. It also makes intuitive sense, since no firm can in the long run grow faster than the rate of economy. If that was the case, this one firm would become 100 percent of the economy and that makes no sense.
Section 9.4 1. Why can skipping payment of a preferred dividend be a bad signal?
A preferred dividend is treated like an interest payment on debt by financial markets and investors. Although the firm will not be in default, any delay or failure to pay the dividends SM 19-328
by the firm will be treated seriously by the financial markets and make them think that the firm is in financial difficulty.
2. How is a preferred stock with a fixed maturity valued? Any preferred stock with a defined maturity date is similar to a bond with a fixed maturity date. This similarity allows for the preferred stock to be valued in a similar fashion after making adjustments for differences in the preferred stock relative to a bond. Using Equation 7.2, developed to value bonds, the coupons are replaced by the preferred dividends and the number of payments is adjusted to four a year to accommodate the fact that firms pay the dividends every quarter and not semiannually. This results in the model represented by Equation 8.7.
Self- Study Problems
9.1
Ted McKay has just bought the common stock of Ryland Corp. The company expects to grow at the following rates for the next three years: 30 percent, 25 percent, and 15 percent. Last year the company paid a dividend of $2.50. Assume a required rate of return of 10 percent. Compute the expected dividends for the next three years and also the present value of these dividends.
Solution: Expected dividends for Ryland Corp and their present value: 0
10%
1
2
3
├─────────┼─────────┼─────────┼─────────┼───────── ─› D0 = $2.50
D1
g1 = 30%
D2
D3
g2 = 25%
g3 = 15%
R = 10% SM 19-329
D1 = D0 × (1 + g1) = $2.50(1 + 0.30) = $3.25 D2 = D1 × (1 + g2) = $3.25(1 + 0.25) = $4.06 D3 = D2 × (1 + g3) = $4.06(1 + 0.15) = $4.67 Present value of the dividends = PV(D1) + PV(D2) + PV(D3) = $2.95 + $3.36 + $3.51 = $9.82
9.2
Merriweather Manufacturing Company has been growing at a rate of 6 percent for the past two years, and the CEO expects the company to continue to grow at this rate for the next several years. The company paid a dividend of $1.20 last year. If your required rate of return is 14 percent, what is the maximum price that you would be willing to pay for this company’s stock?
Solution: Present value of Merriweather stock: 0
14%
1
2
3
├─────────┼─────────┼─────────┼─────────┼───────── ─› D0 = $1.20
D1
D2
D3
g = 6% R = 14% D1 = D0 × (1 + g) = $1.20(1 + 0.06) = $1.27 D1 $1.27 (R g) (0.14 0.06) $15.88
P0
The maximum price you should be willing to pay for this stock is $15.90.
SM 19-330
9.3
Clarion Corp. has been selling electrical supplies for the past 20 years. The company’s product line has changed very little in the past five years, and the company’s management does not expect to add any new items for the foreseeable future. Last year, the company paid a dividend of $4.45 to its common stockholders. The company is not expected to increase its dividends for the next several years. If your required rate of return for such firms is 13 percent, what is the current value of this company’s stock?
Solution: Present value of Clarion Corp. stock:
0
13%
1
2
3
Year
├─────────┼─────────┼─────────┼─────────┼───────── ─› D0 = $4.45
D1
D2
D3
g = 0% R = 13% Since the company’s dividends are not expected to grow, D0 = D1 =D2= ......... D∞ = $4.45 = D Current value of the stock
9.4
=
D/R
=
$4.45/0.13
=
$34.23
Barrymore Infotech is a fast -growing communications company. The company did not pay a dividend last year and is not expected to do so for the next two years. Last year the company’s growth accelerated, and management expects to grow the business at a rate of 35 percent for the next five years before growth slows to a more stable rate of 7 percent. In the third year, management has forecasted a dividend payment of $1.10. Dividends will grow with the company thereafter. Calculate the value of the company’s stock at the end of its rapid growth period (i.e., at the end of five years). The required rate of return for such stocks is 17 percent. What is the current value of this stock? SM 19-331
Solution: Present value of Barrymore Infotech stock: 0 17%
1
2
3
4
5
6
7
8
├────┼────┼────┼────┼────┼────┼────┼────┤ D0
D1
D2
g1 – g5 = 35%
D3
D4
D5
D6
g6 and beyond = 7%
R = 17% D0 = D1 =D2 = 0 D3 = $1.10 D4 = D3 × (1 + g4) = $1.10(1 + 0.35) =$1.485 D5 = D4 × (1 + g5) = $1.485(1 + 0.35) = $2.005 D6 = D5 × (1 + g6) = $2.005(1 + 0.07) = $2.145
Value of stock at t = 5 P5:
D6 $2.145 (R g) (0.17 0.07) $21.45
P5
Present value of the dividends in years 1 to 5 = PV(D1) + PV(D2) + PV(D3) + PV(D4) + PV(D5) $0 $0 $1.10 $1.485 $2.005 (1.17)3 (1.17)4 (1.17)5 $0 $0 $0.69 $0.79 $0.91 $2.39
Current value of stock = PV(Dividends) + PV(P5) = $2.39 + [$21.45/(1.17)5] = $2.39 + $9.78 = $ 12.18
9.5
You are interested in buying the preferred stock of a bank that pays a dividend of $1.80 every quarter. If you discount such cash flows at 8 percent, what is the value of this stock? SM 19-332
Solution: Present value of preferred bank stock: Quarterly dividend on preferred stock = D = $1.80 Required rate of return = 8% Current value of stock: = P o
D R
$1.80 4 0.08
$90.00
Discussion Questions 9.1
Why can the market price of a stock differ from its true (intrinsic) value?
Let us start by first defining the market price of a stock as the price that equates the demand for a stock with the supply of the stock. The role of the stock markets is to bring buyers and sellers together in the most efficient way such that stocks are bought and sold at the market price. In reality, however, barriers of various kinds including the geographic separation of the two parties, strong demand that leads to overvaluation and vice versa, make the market price of a security slightly different than its true (intrinsic) value. The more efficient the market place, the smaller the deviation between the two. LO: 1 Level: Basic Bloomcode: Comprehension AASCB: Analytic IMA: Corporate Finance AICPA: Measurement 9.2
Why are investors and managers concerned about stock market efficiency? SM 19-333
The role of secondary markets is to bring buyers and sellers together. Ideally, we would like stock markets to be as efficient as possible. Markets are efficient when current market prices of securities traded reflect all available information relevant to the security. If this is the case, security prices will be near or at their true value. The more efficient the market, the more likely this is to happen. This makes it easier for managers to price the stocks close to their intrinsic value. What investors are most concerned about is having complete information regarding a stock’s current price and where that price information can be obtained. Efficient markets allow them to trade at prices that are closer to the true value than otherwise possible. Thus, both investors who provide funds and managers (firms) who raise money are concerned when high transaction costs lead to inefficient markets. LO: 1 Level: Basic Bloomcode: Comprehension AASCB: Analytic IMA: Corporate Finance AICPA: Measurement
9.3
Why are common stockholders considered to be more at risk than the holders of other types of securities?
In the hierarchy of lenders of funds to a firm, common stockholders have the most to lose. In the event of a firm becoming bankrupt, the law requires that creditors of different types, including bondholders, be paid off first. Next, preferred stockholders are paid off. Finally, common stockholders receive their investment if any funds are still available. Thus, common stockholders receive their money back last and are placed at most risk. This feature of common equity is referred to as residual claim. LO: 1 Level: Basic Bloomcode: Comprehension SM 19-334
AASCB: Analytic IMA: Corporate Finance AICPA: Measurement
9.4
Under what conditions does it make sense to use the constant-growth dividend model to value a stock?
It only makes sense to use the constant-growth dividend model to value the stock of a company that is already paying a dividend and when that dividend can reasonably be expected to grow at a constant rate foreverin other words, the stock of a mature company. As discussed in the chapter, the growth rate of such a company will be less than the long term rate of inflation and the real growth rate of the economy. It must also be less than the required rate of return on the stock. LO: 4 Level: Basic Bloomcode: Comprehension AASCB: Analytic IMA: Corporate Finance AICPA: Measurement
9.5
What does it mean when a company has a very high P/E ratio? Give examples of industries in which you believe high P/E ratios are justified. A high P/E ratio implies that investors believe that the firm has good prospects for earnings growth in the future. In fact, they believe that the firm will have higher growth potential than firms with lower P/E ratios. Companies in industries that are fast growing like biotech or any hi-tech industry have high P/E ratios. In the past, firms like Cisco and Intel had very high P/E ratios. As these firms matured and settled to annual growth rates of 15 percent or less, their P/E ratios have declined.
LO: 1 Level: Basic Bloomcode: Comprehension AASCB: Analytic IMA: Corporate Finance AICPA: Measurement
SM 19-335
9.6
Explain why preferred stock is considered to be a hybrid of equity and debt securities.
The law considers preferred stock as equity. Thus, holders are treated as the firm’s owners. Also, like common stockholders, preferred stockholders have to pay taxes on their dividend income. However, preferred stockholders do not have any voting rights. In addition, they receive only a fixed dividend just like bondholders. If a firm is liquidated, then they receive a stated value (par value) similar to bondholders. Preferred stock is rated by credit rating agencies just like bonds. Some preferred issues are convertible to the firm’s common stock just as convertible bonds. Some preferred issues are not perpetual and have a fixed maturity just like bonds. Thus preferred stock is a hybrid security—like equity in some ways and like debt security in others. LO: 2 Level: Basic Bloomcode: Comprehension AASCB: Analytic IMA: Corporate Finance AICPA: Measurement
9.7
Why is stock valuation more difficult than bond valuation?
Despite the availability of mathematical models to value stocks, it is more difficult to apply valuation techniques to stocks than to bonds. First, unlike bonds, firms are not in default if dividends are not declared. This makes it difficult to determine the size and timing of the cash flows. Second, common stock, unlike bonds, does not have a fixed maturity, and hence, it is difficult to determine a terminal value unlike bonds, which have a maturity value. Next, it is easier to calculate the present value of a bond because the required rate of return is observable. In the case of stocks, it is rather difficult to estimate a required rate of return for many stocks and classify them into different risk groups. LO: 2 SM 19-336
Level: Intermediate Bloomcode: Comprehension AASCB: Analytic IMA: Corporate Finance AICPA: Measurement
9.8
You are currently thinking about investing in a stock valued at $25.00 per share. The stock recently paid a dividend of $2.25 and its dividend is expected to grow at a rate of 5 percent for the foreseeable future. You normally require a return of 14 percent on stocks of similar risk. Is the stock overpriced, underpriced, or correctly priced?
D0 = $2.25 Required rate of return (R) = 14% Growth rate (g) = 5% Using Constant-growth model, the price of stock is: P0
D1 D (1 g) 0 Rg Rg
$2.25(1.05) $26.25 0.14 0.05
This stock is underpriced at $25. Using the constant-growth model, we arrived at a price of $26.25 for this stock. This makes the stock underpriced, and it should be considered a good buy. LO: 4 Level: Basic Bloomcode: Analysis AASCB: Analytic IMA: Corporate Finance AICPA: Measurement
9.9
Stock A and Stock B are both priced at $50 per share. Stock A has a P/E ratio of 17, while Stock B has a P/E ratio of 24. Which is the more attractive investment, considering everything else to be the same, and why?
SM 19-337
Stock A is the more attractive investment because it has a lower P/E ratio. The lower the P/E ratio, the larger the amount of earnings supporting the stock price. This makes Stock A a more attractive investment than Stock B. LO: 1 Level: Basic Bloomcode: Analysis AASCB: Analytic IMA: Corporate Finance AICPA: Measurement
9.10
Facebook does not pay dividends. How can it have a positive stock price?
Investors expect that Facebook will eventually start paying dividends and the value of its shares reflects the present value of these expected dividends. The fact that a company is not currently paying dividends does not mean that its stock is worthless. It is not uncommon for high growth firms not to pay dividends, since they often have to reinvest all of their earnings to maintain their high growth rate. However, if they are successful, as their growth rate slows over time they will begin to pay dividends. Examples of such companies that do not pay dividends include Apple, Microsoft, Dell, among many others. LO: 3, 4 Level: Intermediate Bloomcode: Comprehension AASCB: Analytic IMA: Corporate Finance AICPA: Measurement
Questions and Problems
BASIC SM 19-338
9.1
Stock Market Index: What is a stock market index?
Solution: A stock market index is used to measure the performance of the stock market. These indexes reflect the value of the stocks in a particular market, such as the NYSE or the NASDAQ, or across markets, and increase and decrease as the values of the stocks go up and down. Examples of stock market indexes include the Dow Jones Industrial Average, the New York Stock Exchange Index, the Standard & Poor’s 500 Index, and the NASDAQ Composite Index. LO 1 Bloomcode: Knowledge AASCB: Analytic IMA: Corporate Finance AICPA: Measurement
9.2
Stock Market Index: What is the Dow Jones Industrial Average?
Solution: The Dow is the most widely published stock index and reflects the value of the stocks of 30 large companies. The value of the shares of these companies represents about 20 percent of the market value of all U.S. stocks. LO 1 Bloomcode: Knowledge AASCB: Analytic IMA: Corporate Finance AICPA: Industry/Sector Perspective
9.3
Stock Market Index: What does NASDAQ stand for? What is NASDAQ?
Solution: NASDAQ stands for National Association of Securities Dealers Automated Quotation system. NASDAQ is one of the largest electronic stock markets in the world, listing over three thousand companies. It is an OTC market and does not have any physical location.
SM 19-339
Companies listed on NASDAQ had market capitalization of $6.2 trillion as of December 2013.
LO 1 Bloomcode: Knowledge AASCB: Analytic IMA: Corporate Finance AICPA: Industry/Sector Perspective
9.4
Dividend yield: What is a dividend yield? What does it tell us?
Solution: A dividend yield is the ratio of the annual dividend payout by the current market price of a stock. It tells us what percentage return we would earn from dividends alone if we purchased the stock at its current price. Alternatively, it tells us what percentage of the firm’s stock value is being distributed to stockholders each year. LO 1 Bloomcode: Knowledge AASCB: Analytic IMA: Corporate Finance AICPA: Measurement 9.5
Present value of dividends: Fresno Corp. is a fast-growing company whose management expects it to grow at a rate of 30 percent over the next two years and then slow down to a growth rate of 18 percent for the following three years. If the last dividend paid by the company was $2.15, estimate the dividends for the next five years. Compute the present value of these dividends if the required rate of return is 14 percent.
Solution: 0
1
2
3
4
5
├───────┼────────┼───────┼────────┼───────┤ D0 = $2.15
g1-2 = 30%;
g3-5 = 18%;
R = 14%
D1 = D0 × (1 + g1) = $2.15(1.30) = $2.795 SM 19-340
D2 = D1 × (1 + g2) = $2.795(1.30) = $3.634 D3 = D2 × (1 + g3) = $3.634(1.18) = $4.288 D4 = D3 × (1 + g4) = $4.288(1.18) = $5.06
D5 = D4 × (1 + g5) = $5.06(1.18) = $5.97
$2.795 $3.634 $4.288 $5.06 $5.97 1 2 3 4 (1.14) (1.14) (1.14) (1.14) (1.14)5 $2.45 $2.80 $2.89 $3.00 $3.10
PV(Dividends )
$14.24
LO 4 Bloomcode: Application AASCB: Analytic IMA: Corporate Finance AICPA: Measurement
9.6
Zero growth: Nynet, Inc., paid a dividend of $4.18 last year. The company’s management does not expect to increase its dividend in the foreseeable future. If the required rate of return is 18.5 percent, what is the current value of the stock?
Solution: D0 = $4.18; g = 0; P
D
0
R
$4.18
R = 18.5%
$22.59
0.185
LO 4 Bloomcode: Application AASCB: Analytic IMA: Corporate Finance AICPA: Measurement
9.7
Zero growth: Knight Supply Corp. has not grown for the past several years and management expects this lack of growth to continue. The firm last paid a dividend of $3.56. If you require a rate of return of 13 percent, what is the current value of this stock to you? SM 19-341
Solution: D0 = $3.56; g = 0; P 0
D R
$3.56
R = 13%
$27.38
0.13
LO 4 Bloomcode: Application AASCB: Analytic IMA: Corporate Finance AICPA: Measurement
9.8
Zero growth: Ron Santana is interested in buying the stock of First National Bank. While the bank’s management expects no growth in the near future, Ron is attracted by the dividend income. Last year the bank paid a dividend of $5.65. If Ron requires a return of 14 percent on such stocks, what is the maximum price he should be willing to pay for a share of the bank’s stock?
Solution: D0 = $5.65; g = 0; R = 14% D $5.65 P $40.36 0 R 0.14 LO 4 Bloomcode: Application AASCB: Analytic IMA: Corporate Finance AICPA: Measurement
9.9
Zero growth: The current stock price of Largent, Inc., is $44.72. If the required rate of return is 19 percent, what is the dividend paid by this firm if the dividend is not expected to grow in the future?
Solution:
SM 19-342
P0 = $44.72; R = 19%; D P 0 R D $44.72 0.19 D $44.72 0.19 $8.50
D = ?;
LO 4 Bloomcode: Application AASCB: Analytic IMA: Corporate Finance AICPA: Measurement
9.10
Constant growth: Moriband Corp. paid a dividend of $2.15 yesterday. The company’s
dividend is expected to grow at a steady rate of 5 percent for the foreseeable future. If investors in stocks of companies like Moriband require a rate of return of 15 percent, what should be the market price of Moriband stock?
Solution: D0 = $2.15;
R = 15%
D1 D (1 g) 0 Rg Rg
P0
g = 5%;
$2.15(1.05) $22.58 0.15 0.05
LO 4 Bloomcode: Application AASCB: Analytic IMA: Corporate Finance AICPA: Measurement
9.11
Constant growth: Nyeil, Inc., is a consumer products firm that is growing at a constant rate of 6.5 percent. The firm’s last dividend was $3.36. If the required rate of return is 18
SM 19-343
percent, what is the market value of this stock if dividends grow at the same rate as the firm?
Solution: D0 = $3.36; P0
g = 6.5%;
R = 18%
D1 $3.36(1.065) R g 0.18 0.065
$31.12 LO 4 Bloomcode: Application AASCB: Analytic IMA: Corporate Finance AICPA: Measurement
9.12
Constant growth: Reco Corp. is expected to pay a dividend of $2.25 next year. The forecast for the stock price a year from now is $37.50. If the required rate of return is 14 percent, what is the current stock price? Assume constant growth.
Solution: D1 = $2.25;
P1 = $37.50;
R = 14%
SM 19-344
D2 D (1 g) 1 Rg Rg
P1
$2.25(1 g) 0.14 g
$37.50
$37.50(0.14 g) $2.25 2.25g $5.25 37.5g 2.25 2.25g $5.25 $2.25 37.5g 2.25g $3.00 7.55% $39.75
g P0
D1 Rg $2.25 $34.87 0.14 0.0755
LO 4 Bloomcode: Application AASCB: Analytic IMA: Corporate Finance AICPA: Measurement
9.13
Constant growth: Proxicam, Inc., is expected to grow at a constant rate of 7 percent. If the company’s next dividend, which will be paid in a year, is $1.15 and its current stock price is $22.35, what is the required rate of return on this stock?
Solution: D1 = $1.15;
P0 = $22.35;
g = 7%
SM 19-345
P0
D1 Rg
R
D1
g
P0
$1.15 0.07 $22.35
0.0515 0.07 12.15% LO 4 Bloomcode: Application AASCB: Analytic IMA: Corporate Finance AICPA: Measurement
9.14
Preferred stock valuation: X-Centric Energy Company has issued perpetual preferred stock with a stated (par) value of $100 and a dividend of 4.5 percent. If the required rate of return is 8.25 percent, what is the stock’s current market price?
Solution: D = 4.5% ($100) = $4.50; D P $4.50 $54.55 0 R 0.0825
R = 8.25%
LO 6 Bloomcode: Application AASCB: Analytic IMA: Corporate Finance AICPA: Measurement
9.15
Preferred stock valuation: The First Bank of Flagstaff has issued perpetual preferred stock with a $100 par value. The bank pays a quarterly dividend of $1.65 on this stock. What is the current price of this preferred stock given a required rate of return of 11.6 percent?
SM 19-346
Solution: Quarterly dividend = $1.65 Required rate of return = R = 11.6% P
(1.65 4)
0
$56.90
0.116
LO 6 Bloomcode: Application AASCB: Analytic IMA: Corporate Finance AICPA: Measurement
9.16
Preferred stock valuation: The preferred stock of Axim Corp. is currently selling at $47.13. If the required rate of return is 12.2 percent, what is the dividend paid by this stock?
Solution: P0 = $47.13;
R = 12.2%
D 0.122 D $47.13 0.122 $5.75
P0 $47.13
LO 6 Bloomcode: Application AASCB: Analytic IMA: Corporate Finance AICPA: Measurement
9.17
Preferred stock valuation: Each quarter, Sirkota, Inc., pays a dividend on its perpetual preferred stock. Today the stock is selling at $63.37. If the required rate of return for such stocks is 15.5 percent, what is the quarterly dividend paid by Sirkota?
Solution: SM 19-347
P0 = $63.37;
R = 15.5%
D 0.155 D $63.37 0.155 $9.82
P0 $63.37
Annual dividend = $9.82 Quarterly dividend = $9.82 /4 = $2.46 LO 6 Bloomcode: Application AASCB: Analytic IMA: Corporate Finance AICPA: Measurement
INTERMEDIATE 9.18
Constant growth: Kay Williams is interested in purchasing the common stock of Reckers, Inc., which is currently priced at $37.45. The company is expected to pay a dividend of $2.58 next year and to increase its dividend at a constant rate of 7 percent. a.
What should the market value of the stock be if the required rate of return is 14 percent?
b. Is this a good buy? Why or why not?
Solution: a.
P0
b.
D1 $2.58 $36.86 R g 0.14 0.07
The stock is overpriced and not a good buy.
LO 4 Bloomcode: Analysis AASCB: Analytic IMA: Corporate Finance AICPA: Measurement
SM 19-348
9.19
Constant growth: The required rate of return is 23 percent. Ninex Corp. has just paid a dividend of $3.12 and is expected to increase its dividend at a constant rate of 5 percent. What is the expected price of the stock three years from now?
Solution: R = 23%; D0 = $3.12; D D (1 g)4 4 P3 0 Rg Rg
g = 5%
3.12(1.05)4 $21.07 0.23 0.05
LO 4 Bloomcode: Application AASCB: Analytic IMA: Corporate Finance AICPA: Measurement
9.20
Constant growth: Jenny Banks is interested in buying the stock of Fervan, Inc., which is increasing its dividends at a constant rate of 6 percent. Last year the firm paid a dividend of $2.65. The required rate of return is 16 percent. What is the current value of this stock? What should be the price of the stock in year 5?
Solution: g = 6%,
D0 = $2.65,
R = 16%
P0
D1 $2.65(1.06) $28.09 R g 0.16 0.06
P
D6
5
Rg
$2.65(1.06)6 0.16 0.06
$37.59
LO 4 Bloomcode: Application AASCB: Analytic IMA: Corporate Finance AICPA: Measurement SM 19-349
9.21
Constant growth: You own shares of Old World DVD Company and are interested in selling them. With so many people downloading music these days, sales, profits, and dividends at Old World have been declining 6 percent per year. The firm just paid a dividend of $1.15 per share. The required rate of return for a stock this risky is 15 percent. If dividends are expected to decline at 6 percent per year, what is a share of the stock worth today?
Solution: R = 15%; P0
D0 = $1.15;
g = -6%
D1 D (1 g) 0 Rg Rg $1.15 (1 0.06) $5.15 0.15 (0.06)
LO 4 Bloomcode: Application AASCB: Analytic IMA: Corporate Finance AICPA: Measurement
9.22
Nonconstant growth: You own a company that competes with Old World DVD Company (in the previous problem). Instead of selling DVDs, however, your company sells music downloads from a web site. Things are going well now, but you know that it is only a matter of time before someone comes up with a better way to distribute music. Your company just paid a $1.50 per share dividend, and you expect to increase the dividend 10 percent next year. However, you then expect your dividend growth rate to begin going down – to 5 percent the following year, 2 percent the next year, and to -3 percent per year thereafter. Based upon these estimates, what is the value of a share of your company’s stock? Assume that the required rate of return is 12 percent.
Solution: g1 = 10%, g2 = 5%, g3 =2%,
g = -3%,
D0 = $1.50
R = 12%
SM 19-350
D1 = $1.50 × 1.1 = $1.65, $1.7672,
D2 = $1.65 × 1.05 = $1.7325, D3 = $1.7325 ×1.02 =
D4 = $1.7672 × 0.97 = $1.7141.
D4 $1.7141 $11.43 R g 0.12 (0.03) D1 D2 D3 P3 P 0 2 3 1 R (1 R) (1 R) (1 R)3
P3
P0
$1.65 $1.7325 $1.7672 $11.43 1.12 (1.12)2 (1.12)3 (1.12)3
P0 $1.47 $1.38 $1.26 $8.14 P0 $12.25 LO 5 Bloomcode: Application AASCB: Analytic IMA: Corporate Finance AICPA: Measurement
9.23
Nonconstant growth: Tre-Bien, Inc., is a fast-growing technology company. Management projects rapid growth of 30 percent for the next two years, then a growth rate of 17 percent for the following two years. After that, a constant-growth rate of 8 percent is expected. The firm expects to pay its first dividend of $2.45 a year from now. If dividends will grow at the same rate as the firm and the required rate of return on stocks with similar risk is 22 percent, what is the current price of the stock?
Solution: g1 = g2 = 30%,
g3 = g4 = 17%,
D1 = $2.45,
D2 = $2.45 × 1.30 = $3.19,
D4 = $3.73 × 1.17 = $4.36,
g = 8%,
D1 = $2.45,
R = 22%
D3 = $3.19 × 1.17 = $3.73
D5 = 4.36 × 1.08 = $4.71
SM 19-351
P4
D5 $4.71 $33.63 R g 0.22 0.08
P
D1
0
P0
1 R
D2 (1 R)
2
D3 D4 P4 3 4 (1 R) (1 R) (1 R)4
$2.45 $3.19 $3.73 ($4.36 33.64) 1.22 (1.22)2 (1.22)3 (1.22)4
P0 $2.01 $2.14 $2.05 17.15 P0 $23.35 LO 5 Bloomcode: Application AASCB: Analytic IMA: Corporate Finance AICPA: Measurement
9.24
Nonconstant growth: Management of ProCor, a biotech firm, forecasted the following growth rates for the next three years: 35 percent, 28 percent, and 22 percent. Management then expects the company to grow at a constant rate of 9 percent forever. The company paid a dividend of $1.75 last week. If the required rate of return is 20 percent, what is the value of this stock?
Solution: g1 = 35%; g2 = 28%; g3 = 22%; g4 = 9%; D0 = $1.75; R = 20%
SM 19-352
D1 D0 (1 g 1 ) $1.75 1.35 $2.36 D2 D1 (1 g2 ) $2.36 1.28 $3.02 D3 D2 (1 g3 ) $3.02 1.22 $3.69 D4 D3 (1 g) $3.69 1.09 $4.02
P3
D4 $4.02 $36.57 R g 0.20 0.09
P
D1
0
P0
1 R
D2 (1 R)
2
D3 P3 3 (1 R) (1 R)3
$2.36 $3.02 $3.69 $36.57 1.20 (1.20)2 (1.20)3 (1.20)3
P0 $1.97 $2.10 $2.14 $21.16 P0 $27.37 LO 5 Bloomcode: Application AASCB: Analytic IMA: Corporate Finance AICPA: Measurement
9.25
Nonconstant growth: Revarop, Inc., is a fast- growth company that is expected to grow at a rate of 23 percent for the next four years. It is then expected to grow at a constant rate of 6 percent. Revarop’s first dividend, of $4.25, will be paid in year 3. If the required rate of return is 17 percent, what is the current value of the stock if dividends are expected to grow at the same rate as the company?
Solution: g1-4 = 23%; g = 6%;
D3 = $4.25;
R = 17%
D4 = D3 × 1.23 = $4.25 × 1.23 = $5.23
SM 19-353
P4
D5 $5.23 (1.06) $50.37 0.17 0.06 Rg
P
D1
0
D2
1 R
P0 0 0
(1 R)
2
D3 3
(1 R)
D4 P4 (1 R)4
$4.25 ($5.23 $50.37) (1.17)4 (1.17)3
P0 $2.65 $29.69 $32.33
LO 5 Bloomcode: Application AASCB: Analytic IMA: Corporate Finance AICPA: Measurement
9.26
Nonconstant growth: Quansi, Inc., management expects to pay no dividends for the next six years. It has projected a growth rate of 25 percent for the next seven years. After seven years, the firm will grow at a constant rate of 5 percent. Its first dividend, to be paid in year 7, will be $3.25. If the required rate of return is 24 percent, what is the stock worth today?
Solution: g = 5%; R = 24%; D7 = $3.25; D1 – D6 = 0 P7
$3.25 (1.05) D8 $17.96 0.24 0.05 Rg
P0
D7 P7 $3.25 $17.96 $4.71 (1 R)7 (1.24)7
LO 5 Bloomcode: Application AASCB: Analytic IMA: Corporate Finance AICPA: Measurement SM 19-354
9.27
Nonconstant growth: Staggert Corp. will pay dividends of $5.00, $6.25, $4.75, and $3.00 in the next four years. Thereafter, management expects the dividend growth rate to be constant at 6 percent. If the required rate of return is 18.5 percent, what is the current value of the stock?
Solution: D1 = $5; D2 = $6.25; D3 = 4.75; D4 = $3; g = 6%; R = 18.5%; D D2 D3 D4 P4 P 1 0 2 3 4 1 R (1 R) (1 R) (1 R) (1 R)4
P4
P0
D5 $3 (1.06) $25.44 R g 0.185 0.06
$5 $6.25 $4.75 ($3 $25.44) 2 3 1.185 (1.185) (1.185) (1.185)4
P0 $4.22 $4.45 $2.86 $14.42 $25.95 LO 5 Bloomcode: Application AASCB: Analytic IMA: Corporate Finance AICPA: Measurement
9.28
Nonconstant growth: Diaz Corp. is expected to grow rapidly at a rate of 35 percent for the next seven years. The company’s first dividend, to be paid three years from now, will be $5. After seven years, the company (and the dividends it pays) will grow at a rate of 8.5 percent. What is the value of Diaz stock with a required rate of return of 14 percent?
Solution: g1-7 = 35%; D3 = $5.00; g = 8.5%; R = 14% D1 = D2 = 0;
D3 = $5
D4 = $5 × 1.35 = $6.75 SM 19-355
D5 = $6.75 × 1.35 = $9.11 D6 =$9.11 × 1.35 = $12.30 D7 = $12.30 × 1.35 = $16.61 D8 = $16.61 × 1.085 = $18.02 P7
D8 $18.02 $327.64 R g 0.14 0.085
P
D1
0
1 R
D2
(1 R)
2
D3 D4 D5 D6 D P7 7 3 4 5 6 (1 R) (1 R) (1 R) (1 R) (1 R)7
0 0 $5 $6.75 $9.11 $12.30 ($16.61 $327.64) (1.14)3 (1.14)4 (1.14)5 (1.14)6 (1.14)7 $3.38 $4.00 $4.73 $5.60 $137.58 $155.29 LO 5 Bloomcode: Application AASCB: Analytic IMA: Corporate Finance AICPA: Measurement
9.29
Nonconstant growth: Tin-Tin Waste Management, Inc., is growing rapidly. Dividends are expected to grow at rates of 30 percent, 35 percent, 25 percent, and 18 percent over the next four years. Thereafter, management expects dividends to grow at a constant rate of 7 percent. The stock is currently selling at $47.85, and the required rate of return is 16 percent. Compute the dividend for the current year (D0).
Solution: g1 = 30%; g2 = 35%; g3 = 25%; g4 = 18%; g = 7%; R = 16%; P0 = $47.85
SM 19-356
D1
P0
(1 R )
D2 (1 R )2
1
$47.85
2
D3 D4 P4 (1 R )3 (1 R )4 (1 R )4 3
4
4
D0 (1.30) D0 (1.30) (1.35) D0 (1.30) (1.35) (1.25) 1.16 (1.16)2 (1.16)3 D0 (2.5886) (1.07)
D0 (1.30) (1.35) (1.25) (1.18) (1.16)4
0.16 0.07 (1.16)4
D0 [1.12 1.30 1.411.43 17.00] D 0
$47.85
$2.15
$22.26
LO 5 Bloomcode: Application AASCB: Analytic IMA: Corporate Finance AICPA: Measurement
ADVANCED 9.30
Equation 9.4 shows the relation between a stock’s value and the dividend that is expected next year if dividends grow at a constant rate forever. If a firm pays all of its earnings as dividends, show how Equation 9.4 can be rearranged to calculate that firm’s P/E ratio. What does this tell us about the factors that determine a firm’s P/E ratio?
Solution:
P0
D1 E1 , where, E1 is the earnings per share next year. Rg Rg
Rearranging the formula: P0 1 E1 R g This formula tells us that the P/E ratio is determined by both the risk of the dividend cash flows and the rate at which they are expected to grow. LO 4 Bloomcode: Analysis AASCB: Analytic SM 19-357
IMA: Corporate Finance AICPA: Measurement
9.31
Riker Departmental Stores management has forecasted a growth rate of 40 percent for the next two years, followed by growth rates of 25 percent and 20 percent for the following two years. It then expects growth to stabilize at a constant rate of 7.5 percent forever. The firm paid a dividend of $3.50 recently. If the required rate of return is 18 percent, what is the current value of Riker’s stock?
Solution: g1-2 = 40%; g3 = 25%; g4 = 20%; g = 7.5%; D0 = $3.50; R = 18% D1 D0 (1 g1 ) $3.50 1.40 $4.90 D2 D1 (1 g2 ) $4.90 1.40 $6.86 D3 D2 (1 g3 ) $6.86 1.25 $8.575 D4 D3 (1 g4 ) $8.575 1.20 $10.29 D5 D4 (1 g) $10.29 1.075 $11.06 P4
D5 $11.06 $105.35 R g 0.18 0.075
P
D1
0
P0
1 R
D2 (1 R)
2
D3 D4 P4 3 4 (1 R) (1 R) (1 R)4
$4.90 $6.86 $8.575 $10.29 $105.35 1.18 (1.18)2 (1.18)3 (1.18)4 (1.18)4
P0 $4.15 $4.93 $5.22 $5.31 $54.34 P0 $73.94 LO 5 Bloomcode: Application AASCB: Analytic IMA: Corporate Finance AICPA: Measurement
SM 19-358
9.32
Courtesy Bancorp issued perpetual preferred stock a few years ago. The bank pays an annual dividend of $4.27 and your required rate of return is 12.2 percent. a.
What is the value of the stock given your required rate of return?
b.
Should you buy this stock if its current market price is $34.41? Explain.
Solution: a.
D = $4.27; R = 12.2% D $4.27 P $35.00 0 R 0.122
b.
Since the stock is worth $35.00 but can be purchased for $34.41, you should buy this stock.
LO 4 Bloomcode: Analysis AASCB: Analytic IMA: Corporate Finance AICPA: Measurement
9.33
Rhea Kirby owns shares in Ryoko Corp. Currently, the market price of the stock is $36.34. Management expects dividends to grow at a constant rate of 6 percent for the foreseeable future. Its last dividend was $3.25. Rhea’s required rate of return for such stocks is 16 percent. She wants to find out whether she should sell her shares or add to her holdings. a.
What is the value of this stock?
b.
Based on your answer to part a, should Rhea buy additional shares in Ryoko Corp? Why or why not?
Solution:
a.
P0
b.
No, she should not buy more shares. This stock is overpriced with the stock
D1 $3.25 (1.06) $34.45 0.16 0.06 Rg
selling at a higher price than what it is worth. She should sell her shares.
SM 19-359
LO 4 Bloomcode: Analysis AASCB: Analytic IMA: Corporate Finance AICPA: Measurement
9.34
Perry, Inc., paid a dividend of $2.50 yesterday. You are interested in investing in this company, which has forecasted a constant-growth rate of 7 percent for its dividends, forever. The required rate of return is 18 percent. a.
Compute the expected dividends D1, D2, D3, and D4.
b.
Compute the present value of these four dividends.
c.
What is the expected value of the stock four years from now (P4)?
d.
What is the value of the stock today based on the answers to parts b and c?
e.
Use the equation for constant growth (Equation 9.4) to compute the value of the stock today.
Solution: a.
D0 = $2.50 g = 7%
R = 18%
D1 $2.50 (1.07) $2.675 D 2 $2.50 (1.07)2 $2.86 D 3 $2.50 (1.07)3 $3.063 D 4 $2.50 (1.07)4 $3.277
PV (Dividends)
$2.675
$2.86 $3.063 $3.277 (1.18) (1.18)2 (1.18)3 (1.18)4 1
b.
$2.27 2.05 $1.86 $1.69 $7.88
SM 19-360
c.
D5 D4 (1 g) $3.277 (1.07) $3.506 D5 $3.506 P $31.88 4 R g 0.18 0.07
PV (P )
$31.88
4
$16.44
4
(1.18) P0 PV (Dividends) PV (P4 ) $7.87 $16.44
d.
$24.32 e.
For a constant-growth stock: P0
D1 $2.675 $24.32 R g 0.18 0.07
LO 4 Bloomcode: Application AASCB: Analytic IMA: Corporate Finance AICPA: Measurement
9.35
Zweite Pharma is a fast -growing drug company. Management forecasts that in the next three years, the company’s dividend growth rates will be 30 percent, 28 percent, and 24 percent, respectively. Last week it paid a dividend of $1.67. After three years, management expects dividend growth to stabilize at a rate of 8 percent. The required rate of return is 14 percent. a.
Compute the dividends for each of the next three years, and calculate their present value.
b.
Calculate the price of the stock at the end of year 3, when the firm settles to a constant-growth rate.
c.
What is the current price of the stock?
Solution:
SM 19-361
g1 = 30%; g2 = 28%; g3 = 24%; g = 8%; D0 = $1.67; R = 14% D1 D0 (1 g 1 ) $1.67 (1.30) $2.171 D2 D1 (1 g2 ) $2.171 (1.28) $2.779 D3 D2 (1 g3 ) $2.779 (1.24) $3.446
a. PV(Dividends) D1 D2 D3 1 R (1 R)2 (1 R)3
$2.171 $2.779 $3.446 1.14 (1.14)2 (1.14)3
$1.90 $2.14 $2.33 $6.37
b. D4 D3 (1 g) $3.446 (1.08) $3.722 P3
D4 $3.722 $62.02 R g 0.14 0.08
PV (P ) 3
c.
$62.03
$41.87
3
(1.14) P0 PV (Dividends) PV (P3 ) $6.37 $41.87 $48.23
LO 5 Bloomcode: Application AASCB: Analytic IMA: Corporate Finance AICPA: Measurement
SM 19-362
9.36
Triton Inc., is expected to grow at a rate of 22 percent for the next five years and then settle to a constant growth rate of 6 percent. The company recently paid a dividend of $2.35. The required rate of return is 15 percent. a.
Find the present value of the dividends during the rapid-growth period if dividends grow at the same rate as the company.
b.
What is the value of the stock at the end of year 5?
c.
What is the value of the stock today?
Solution: g1-5 = 22%; g = 6%; D0 = $2.35; R = 15% a. D1 D0 (1 g 1 ) $2.35 (1.22) $2.867 D2 D1 (1 g2 ) $2.867 (1.22) $3.498 D3 D2 (1 g3 ) $3.498 (1.22) $4.267 D4 D3 (1 g4 ) $4.267 (1.22) $5.206 D5 D4 (1 g) $5.206 (1.22) $6.351
PV(Dividends) D1 D2 D3 D4 D5 1 R (1 R)2 (1 R)3 (1 R)4 (1 R)5
$2.867 $3.498 $4.267 $5.206 $6.351 1.15 (1.15)2 (1.15)3 (1.15)4 (1.15)5
$2.49 $2.65 $2.81 $2.98 $3.16 $14.09
b.
D6 D5 (1 g) $6.351 (1.06) $6.732 D6 $6.732 P $74.80 5 R g 0.15 0.06
SM 19-363
PV(P ) 5
$74.80
$37.19
(1.15)5
P0 PV(Dividends) PV(P5 )
c.
$14.09 $37.19 $51.28
LO 5 Bloomcode: Application AASCB: Analytic IMA: Corporate Finance AICPA: Measurement
9.37
Ceebros Builders is expanding very fast and is expected to grow at a rate of 25 percent for the next four years. The company recently paid a dividend of $3.60 but is not expected to pay any dividends for the next three years. In year 4, management expects to pay a $5 dividend and thereafter to increase the dividend at a constant rate of 6 percent. The required rate of return on such stocks is 20 percent. a.
Calculate the present value of the dividends during the fast-growth period.
b.
What is the value of the stock at the end of the fast-growth period (P4)?
c.
What is the value of the stock today?
d.
Would today’s stock value be affected by the length of time you intend to hold the stock?
Solution: a.
g1-4 = 25% g = 6% D0 = $3.60 D4 = $5.00 R = 20% PV(Dividends ) 0 0 0
$5 $2.41 (1.20)4
b. c.
P4
$5.00 (1.06) D5 $37.86 0.20 0.06 Rg P0 $2.41
$37.86
$20.67
(1.20)4
SM 19-364
d.
No, the length of the holding period has no bearing on today’s stock price.
LO 5 Bloomcode: Application AASCB: Analytic IMA: Corporate Finance AICPA: Measurement
Sample Test Problems
9.1
Which type of secondary market provides the most efficient market for financial securities? (LO 1) Solution:
An auction market is the most efficient type of secondary market because the buyers and sellers in an auction market interact directly with each other and bargain over price. Bloomcode: Comprehension AASCB: Analytic IMA: Corporate Finance AICPA: Industry/Sector Perspective
9.2
Is preferred stock a debt or an equity security? (LO 2) Solution:
Preferred stock represents an ownership interest in a corporation and is legally a form of equity. However, it does have characteristics that are similar to those of debt, such as no voting rights, fixed payments (dividends), and credit ratings similar to those issued to bonds. Bloomcode: Knowledge AASCB: Analytic IMA: Corporate Finance AICPA: Measurement
9.3
Burnes, Inc. is a mature firm that is growing at a constant rate of 5.5 percent per year. The last dividend that the firm paid was $1.50 per share. If dividends are expected to
SM 19-365
grow at the same rate as the firm and the required rate of return on Burnes’s stock is 12 percent, what is the market value of the company’s stock? (LO 4) Solution: D0 = $1.50; P0
g = 5.5%;
R = 12%
D1 $1.50 1.055 $24.35 R g 0.12 0.055
Bloomcode: Application AASCB: Analytic IMA: Corporate Finance AICPA: Measurement
9.4
Abacus Corporation will pay dividends of $2.25, $2.95, and $3.15 in the next three years. After three years, the dividends are expected to grow at a constant rate of 4 percent per year. If the required rate of return is 14.5 percent, what is the current value of the Abacus common stock? (LO 4) Solution: gconstant = 4%; D1 = $2.25 D2 = $2.95 D3 = $3.15; R = 14.5% P3
D4 $3.151.04 $31.20 R g 0.145 .04
$2.25 $2.95 $3.15 $31.20 2 3 (1.145) (1.145) (1.145) (1.145)3 $1.97 $2.25 $2.10 $20.78
P0
$27.10 Bloomcode: Application AASCB: Analytic IMA: Corporate Finance AICPA: Measurement
SM 19-366
9.5
The preferred stock of Wellcare Inc. is currently trading at $137.50 per share. If the required rate of return is 8 percent and this stock has no maturity date, what is the quarterly dividend paid by this stock? What is the quarterly dividend if the stock will mature in one year and it has a par value of $140? (LO 6) Solution: With no maturity date: PS0 = $137.5; R = 8.0% D PS0 $137.50 0.08 D $137.500.08 $11.00 Annual dividend = $11.00 Quarterly dividend = $11.00 /4 = $2.75
SM 19-367
We can use Equation 9.7 to solve for the dividend with a one year maturity date: D/m Pmn PS D/m D/m D/m 0 (1 i / m) (1 i / m)2 (1 i / m)3 (1 i / m)4 D/m D/m D/m D/m $140 $137.50 2 3 (1 0.08 / 4) (1 0.08 / 4) (1 0.08 / 4) (1 0.08 / 4)4 D/m $137.50 D/m D/m D/m $140 1.02 (1.02)2 (1.02)3 (1.02)4
Using a financial calculator to solve for PMT, we find that the quarterly dividend, D/m = $2.14. Bloomcode: Application AASCB: Analytic IMA: Corporate Finance AICPA: Measurement
CHAPTER 10 The Fundamentals of Capital Budgeting
Before You Go On Questions and Answers Section 10.1 1.
Why are capital investments considered the most important decisions made by a firm’s management? Capital investments are the most important decisions made by a firm’s management, because they usually involve large cash outflows and once made are not easily reversed. These are usually long-term projects that will define the firm’s line of business and significantly contribute to the total revenue figure for years to come.
2.
What are the differences between capital projects that are independent, mutually exclusive, and contingent?
SM 19-368
A project is independent if the decision to accept or reject it does not affect the decision to accept or reject another project. On the other hand, projects are mutually exclusive if the acceptance of one implies rejection of the other. Contingent projects are those in which the acceptance of one project is dependent on another project.
Section 10.2 1.
What is the NPV of a project? NPV is simply the difference between the present value of a project’s expected future cash flows and its cost. It is the recommended technique used to value capital investments, as it takes into account both the timing of the cash flows and their risk.
2.
If a firm accepts a project with a $10,000 NPV, what is the effect on the value of the firm?
If a firm accepts a project with a $10,000 NPV, it will increase its value by $10,000.
3.
What are the five steps used in NPV analysis? The five-step process used in the NPV analysis can be listed as follows: (1) Determine the cost of the project. (2) Estimate the project’s future cash flows over its expected life. (3) Determine the riskiness of a project and the appropriate cost of capital. (4) Compute the project’s NPV. (5) Make a decision.
Section 10.3 1.
What is the payback period? The payback period is defined as the number of years it takes to recover the project’s initial investment. All other things being equal, the project with the shortest payback period is usually the optimal investment. SM 19-369
2.
Why does the payback period provide a measure of a project’s liquidity risk?
The payback period determines how quickly you recover your investment in a project. Thus, it serves as a good measure of the project’s liquidity.
3.
What are the main shortcomings of the payback method?
The payback method does not account for time value of money, nor does it distinguish between high- and low-risk projects. In addition, there is no rationale behind choosing the cutoff criteria. For all these reasons, the payback method is not the ideal capital decision rule.
Section 10.4 1.
What are the major shortcomings of using the ARR method as a capital budgeting method?
The biggest shortcoming of using ARR as a capital budgeting tool is that it uses historical, or book value data rather than cash flows and thus disregards the time value of money principle. In addition, as in the payback method, it fails to establish a rationale behind picking the appropriate hurdle rate.
Section 10.5 1.
What is the IRR method?
The IRR, or the internal rate of return, is the discount rate that makes the net present value of the project’s future cash flows zero. The IRR determines whether the project’s return rate is higher or lower than the required rate of return, which is the firm’s cost of capital. As a rule, a project should be accepted if the IRR exceeds the firm’s cost of capital; otherwise the project should be rejected.
SM 19-370
2.
In capital budgeting, what is a conventional cash flow pattern?
A conventional project cash flow in capital budgeting is one in which an initial cash outflow is followed by one or more future cash inflows.
3.
Why should the NPV method be the primary decision tool used in making capital investment decisions?
Given all the different methods to evaluate capital investment decisions, the NPV method is the preferred valuation tool as it accounts for both time value of money and the project’s risk. Furthermore, NPV is not sensitive to nonconventional projects, and therefore it is superior to the IRR technique and it gives a measure of the value increase/decrease to the firm by undertaking the project.
Section 10.6 1.
What decision criteria should managers use in selecting projects when there is not enough money to invest in all available positive-NPV projects?
When a firm does not have enough money to invest in all available positive NPV projects, managers should identify the bundle of positive NPV projects that creates the greatest total value for stockholders.
2.
What might cause a firm to face capital constraints?
A firm might face capital constraints because it can be difficult for outside investors (new creditors, bondholders, or stockholders) to accurately assess the risks and returns associated with the firm’s projects. This might cause the investors to require returns for SM 19-371
their capital that are so high that they make positive-NPV projects unattractive, because those projects cannot produce the high returns required by investors.
3.
How can the PI help in choosing projects when a firm faces capital constraints? What are its limitations?
The basic principle is to select the projects that yield the largest NPV per dollar invested. The PI tells us the NPV per dollar invested for an individual project. In a single period, the PI can be used to identify the bundle of projects that yields the largest NPV per dollar invested. However, as illustrated in Section 12.5, the PI will not necessarily help identify the most valuable bundle of projects if investments are being compared across more than one year and the timing of cash flows from early investments affects the firm’s ability to make subsequent investments.
Section 10.7 1.
What changes have taken place in the capital budgeting techniques used by U.S. companies?
Over the years, there has been a shift from using payback and ARR as the primary capital budgeting tools to using NPV and IRR instead. Managers today understand the importance of the time value of money and discounting and thus regard ARR as an inaccurate and obsolete decision tool.
Self-Study Problems
SM 19-372
10.1
The Management of Premium Manufacturing Company is evaluating two forklift systems to use in its plant that produces the towers for a windmill power farm. The costs and the cash flows from these systems are shown below. If the company uses a 12 percent discount rate for all projects, determine which forklift system should be purchased using the net present value (NPV) approach. Year 0
Year 1
Year 2
Year 3
Otis Forklifts
−$3,123,450
$979,225
$1,358,886
$2,111,497
Craigmore Forklifts
−$4,137,410
$875,236
$1,765,225
$2,865,110
Solution: NPV for Otis Forklifts: NPV n
CFt
t 0
(1 k)t $3,123,450 $979,225 $1,358,886 $2,111,497 (1.12)2 (1.12)3 (1 0.12)1 $3,123,450 $874,308 $1,083,296 $1,502,922 $337,075 NPV for Craigmore Forklifts: NPV n
t 0
CFt
(1 k)t $4,137,410 $875,236 $1,765,225 $2,865,110 (1.12)2 (1.12)3 (1 0.12)1 $4,137,410 $781,461 $1,407,229 $2,039,227 $90,606
Premium should purchase the Otis forklift since it has a larger NPV.
10.2
Perryman Crafts Corp. management is evaluating two independent capital projects that together will cost the company $250,000. The two projects will provide the following cash flows: SM 19-373
Year
Project A
Project B
1
$80,750
$32,450
2
$93,450
$76,125
3
$40,325
$153,250
4
$145,655
$96,110
Which project will be chosen if the company’s payback criterion is three years? What if the company accepts all projects as long as the payback period is less than five years?
Solution: Payback periods for Perryman projects A and B: Project A Cumulative Year
Cash Flow
Cash Flows
0
$(250,000)
$(250,000)
1
80,750
(169,250)
2
93,450
(75,800)
3
40,235
(35,565)
4
145,655
110,090
Project B Cumulative Year
Cash Flow
Cash Flows
0
$(250,000)
$(250,000)
1
32,450
(217,550)
2
76,125
(141,425)
3
153,250
11,825
4
96,110
107,935
Payback period for Project A: SM 19-374
Remaining cost to recover
Payback period = Years before cost recovery +
Cash flow during the year $35,565 $145, 655 per year 3.24 years = 3
Payback period for Project B: Remaining cost to recover
Payback period = Years before cost recovery +
Cash flow during the year $141, 425 $153, 250 per year 2.92 years = 2
If the payback period is three years, only project B will be chosen. If the payback criterion is five years, both A and B will be chosen.
10.3
Terrell Corp. management is considering purchasing a machine that will cost $117,250 and will be depreciated on a straight-line basis over a five-year period. The sales and expenses (excluding depreciation) for the next five years are shown in the following table. The company’s tax rate is 34 percent.
Year 1 Year 2 Year 3 Year 4 Year 5 TShaelecsompany w$il1l2a3c,c4e5p0tall pro$j1e7c6ts,8th7a5t pro$v2i4d2e,4an55acc ou$n2ti5n5g,4ra4t0e of re$tu26rn7,(1A2R 5 R) of at least 45 percent. S Expenses
$137,410
$126,488
$141,289
$143,112
$133,556
Terrell will accept all projects that provide an accounting rate of return (ARR) of at least 45 percent. Should the company accept the project?
Solution: Year 1 Sales
Year 2
$123,450 $176,875
Year 3
Year 4
Year 5
$242,455
$255,440
$267,125 SM 19-375
Expenses
137,410
126,488
141,289
143,112
133,556
Depreciation
23,450
23,450
23,450
23,450
23,450
$ (37,410) $ 26,937
$ 77,716
$ 88,878
$110,119
9,159
26,423
30,219
37,440
$ (24,691) $ 17,778
$ 51,293
$ 58,659
$ 72,679
EBIT Taxes (34%)
12,719
Net Income Beginning Book Value
117,250
93,800
70,350
46,900
23,450
Less: Depreciation
(23,450)
(23,450)
(23,450)
(23,450)
(23,450)
Ending Book Value
$ 93,800 $ 70,350
$ 46,900
$ 23,450
$
0
Average net income
= (–$24,691 + $17,778 + $51,293 + $58,659 + $72,679) / 5 = $35,143.60
Average book value
= (1$117,250 + $93,800 + $70,350 + $46,900 + $23,450 + $02)/6 = $58,625
Accounting rate of return
= $35,143.6 / $58,625 = 0.599 or 59.9%
The company should accept the project.
10.4
Refer to Problem 9.1. Compute the IRR for each of the two systems. Is the investment decision different from the one determined by NPV?
Solution: IRR for two forklift systems: Otis Forklifts: First compute the IRR by the trial-and-error approach: NPV (Otis) = $337,075 > 0 Use a higher discount rate to get NPV = 0! At k = 15%, NPV
Otis
$3,123, 450
$979, 225 $1, 358,886 $2,111, 497 (1 0.15)1 (1.15)2 (1.15)3
$3,123, 450 $851,500 $1, 027,513 $1, 388, 344 $143, 907.
SM 19-376
Try a higher rate. At k = 17%, NPVOtis $3,123, 450 $836, 944 $992, 685 $1, 318, 357 $24,536. Try a higher rate. At k = 17.5%, NPVOtis $3,123, 450 $833, 383 $984, 254 $1, 301,598 $4, 215 Thus the IRR for Otis is less than 17.5 percent. Using a financial calculator, you can find that the exact rate to be 17.43 percent.
Craigmore Forklifts: First compute the IRR by the trial-and-error approach: NPV (Craigmore) = $90,606 > 0 Use a higher discount rate to get NPV = 0! At k = 15%, NPV Craigmore
$875, 236 $1, 765, 225 $2,865,110 (1.15)1 (1.12)2 (1.12)3 $4,137, 410 $761, 075 $1, 334, 764 $1,883,856 $157, 715 $4,137, 410
Try a lower rate. At k = 13%, NPVCraigmore $4,137, 410 $774,545 $1, 382, 430 $1, 985, 665 $5, 230 Try a higher rate. At k = 13.1%, NPVCraigmore $4,137, 410 $773,860 $1, 379, 987 $1, 980, 403 $3,161 Thus the IRR for Craigmore is less than 13.1 percent. The exact rate is 13.06 percent. Based on the IRR, we would still pick Otis over Craigmore forklift systems. The decision is the same as that indicated by NPV
SM 19-377
10.5 You are considering a project that has an initial outlay of $1 million. The profitability index of the project is 2.24. What is the NPV of the project? Solution: You can use Equation 9.6 to solve for the NPV: NPV Initial investment Initial investment NPV $1, 000, 000 2.24 $1, 000, 000 PI
Therefore: NPV = $2,240,000
Discussion Questions 10.1
Explain why the cost of capital is referred to as the ―hurdle‖ rate in capital budgeting.
Solution The cost of capital is the minimum required return on any new investment that allows a firm to break even. Since we are using the cost of capital as a benchmark or ―hurdle‖ to compare the return earned by any project, it is sometimes referred to as the hurdle rate. LO: 1 Level: Basic Bloomcode: Comprehension SM 19-378
AASCB: Analytic IMA: Investment Decisions AICPA: Industry/Sector Perspective 10.2
a. Sykes, Inc. management is considering two projects: a plant expansion and a new computer system for the firm’s production department. Classify these projects as independent, mutually exclusive, or contingent projects and explain your reasoning.
b. A company is building a new plant on the outskirts of Smallesville. The town has offered to donate the land, and as part of the agreement, the company will have to build an access road from the main highway to the plant. How will the project of building the road be classified in the capital budgeting analysis?
c. Management of your firm is currently considering the upgrading of the operating systems of all the firm’s computers. One alternative is to choose the Linux operating system that a local computer services firm has offered to install and maintain. Microsoft has also put in a bid to install the new Windows operating system for businesses. How would these projects be classified?
Solution a. These two projects are independent projects. Accepting or rejecting one will not influence the decision on the other project. The cash flows of the two projects are unrelated.
b. This is a contingent project. Acceptance of the road-building project is contingent on the new plant being a financially viable project. If the new plant will not have a positive value, then the firm will not even consider this project. However, this project’s cost will have to be considered along with the cost of building the new plant in the capital budgeting analysis. SM 19-379
c. These are two mutually exclusive projects. The company’s computers need only one operating system. Either the Linux or the Windows operating system needs to be installed, not both. Hence, the selection of one will eliminate the other from consideration. LO: 1 Level: Intermediate Bloomcode: Analysis AASCB: Analytic IMA: Investment Decisions AICPA: Industry/Sector Perspective 10.3
In the context of capital budgeting, what is ―capital rationing‖?
Solution Capital rationing implies that a firm does not have the resources necessary to fund all of the available projects. In other words, funding needs exceed funding resources. Thus, the available capital will be allocated to the projects that will benefit the firm and its shareholders the most. Projects that create the largest increase in shareholder wealth will be accepted until all the available resources have been allocated. LO: 1 Level: Basic Bloomcode: Knowledge AASCB: Analytic IMA: Investment Decisions AICPA: Industry/Sector Perspective
10.4
Provide two conditions under which a set of projects might be characterized as mutually exclusive.
Solution When projects are mutually exclusive, acceptance of one project precludes the acceptance of others. Typically, mutually exclusive projects perform the same function and so only one of them needs to be accepted. A funding or resource constraint can also cause projects to be mutually exclusive. SM 19-380
LO: 1 Level: Basic Bloomcode: Comprehension AASCB: Analytic IMA: Investment Decisions AICPA: Industry/Sector Perspective
10.5
a. A firm invests in a project that is expected to earn a return of 12 percent. If the appropriate cost of capital is also 12 percent, did the firm make the right decision. Explain.
b. What is the impact on the firm if it accepts a project with a negative NPV?
Solution a. We would normally argue that a firm should only accept projects in which the project’s return exceeds the cost of capital. In other words, only if the net present value exceeds zero should a project be accepted. But in reality, projects with a zero NPV should also be accepted because the project earns a return that equals the cost of capital. For some firms like the one above, this could be the situation because they may not have projects that provide a return greater than the cost of capital for the firm. b. When a firm takes on positive NPV projects, the value of the firm increases. By the same token, when a project undertaken has a negative NPV, the value of the firm will decrease by the amount of the net present value. LO: 2 Level: Basic Bloomcode: Analysis AASCB: Analytic IMA: Investment Decisions AICPA: Industry/Sector Perspective 10.6
Identify the weaknesses of the payback period method.
SM 19-381
Solution There are several critical weaknesses in the payback period approach of evaluating capital projects.
The payback period ignores the time value of money by not discounting future cash flows.
When comparing projects, it ignores risk differences between the projects.
A firm may establish payback criteria with no economic basis for that decision and thereby run the risk of losing out on good projects.
The method ignores cash flows beyond the payback period, thus leading to nonselection of projects that may produce cash flows well beyond the payback period or more cash flows than accepted projects. This leads to a bias against longer-term projects.
LO: 3 Level: Basic Bloomcode: Comprehension AASCB: Analytic IMA: Investment Decisions AICPA: Industry/Sector Perspective
10.7
What are the strengths and weaknesses of the accounting rate of return approach?
Solution The biggest advantage of Accounting Rate of Return (ARR) approach is that it is easy to compute since accounting data is readily available, whereas estimating cash flows is more difficult. However, the disadvantages outweigh this specific advantage. Similar to the payback, it does not discount cash flows, but merely averages net income over time. No economic rationale is used in establishing an ARR cutoff rate. Finally, the ARR uses net income to evaluate the project and not cash flows or market data. This is a serious flaw in this approach. LO: 4 Level: Basic Bloomcode: Comprehension SM 19-382
AASCB: Analytic IMA: Investment Decisions AICPA: Industry/Sector Perspective
10.8
Under what circumstances might the IRR and NPV approaches produce conflicting
results?
Solution IRR and the NPV methods of evaluating capital investment projects might produce dissimilar results under two circumstances. First, if the project’s cash flows are not conventional—that is, if the sign of the cash flow changes more than once during the life of a project—then multiple IRRs can be obtained as solutions. We would be unable to identify the correct IRR for decision making. (See Learning by Doing Application 10.3.) The second situation occurs when two or more projects are mutually exclusive. The project with the highest IRR may not necessarily be the one with the highest NPV and thereby be the right choice. There is an important reason for this. IRR assumes that all cash flows received during the life of a project are reinvested at the IRR, whereas the NPV method assumes that they are reinvested at the cost of capital. Since the cost of capital is the better proxy for opportunity cost, NPV uses the better proxy, while the IRR may use an unrealistically higher rate as proxy. LO: 5 Level: Intermediate Bloomcode: Comprehension AASCB: Analytic IMA: Investment Decisions AICPA: Industry/Sector Perspective
10.9
The modified IRR (MIRR) alleviates two concerns with using the IRR method for evaluating capital investments. What are they?
Solution SM 19-383
IRR assumes that the cash flows from a project are reinvested at the project’s IRR, while the NPV assumes that they are invested at the firm’s cost of capital. The NPV assumption is correct more often than not. The MIRR assumes that each operating cash flow is reinvested at the firm’s cost of capital.
The second appeal of MIRR is that under this method all of the compounded operating cash flow values are summed up to get the project’s terminal value. Since most projects generate positive total net operating cash flows, MIRR does not suffer from issues associated with unconventional cash flows. LO: 5 Level: Basic Bloomcode: Comprehension AASCB: Analytic IMA: Investment Decisions AICPA: Industry/Sector Perspective
10.10 Elkridge Construction Company has an overall (composite) cost of capital of 12 percent. This cost of capital reflects the cost of capital for an Elkridge Construction project with average risk. However, the firm takes on projects of various risk levels. The company’s experience suggests that low-risk projects have a cost of capital of 10 percent and high-risk projects have a cost of capital of 15 percent. Which of the following projects should the company not select to maximize shareholder wealth?
Project
Expected Return
Risk
1. Single-family homes
13%
Low
2. Multifamily residential
12
Average
3. Commercial
18
High
4. Single-family homes
9
Low
5. Commercial
13
High
Solution Project
Risk
Require Expected Return
Decision SM 19-384
d Return 1. Single-family homes 2. Multifamily residential
Low Averag
10%
13%
12
Accept Accept /
12
Indiffere
e
nt
3. Commercial
High
15
18
Accept
4. Single-family homes
Low
10
9
Reject
5. Commercial
High
15
13
Reject
LO: 2 Level: Basic Bloomcode: Application AASCB: Analytic IMA: Investment Decisions AICPA: Industry/Sector Perspective
10.11
High Tech Monopoly Co. has plenty of cash to fund
any conceivable positive NPV project. Can you describe a situation in which capital rationing could still occur?
Solution: Financial capital is not the only constrainable item within the firm. This might occur, for example, when human capital is in short supply, as is the case with most high-technology firms. SM 19-385
Even if every positive NPV project could be funded, the firm might not have enough employees to manage the projects. Therefore, even with ample financial capital, firms will more than likely still be rationing projects. LO: 6 Level: Basic Bloomcode: Comprehension AASCB: Analytic IMA: Investment Decisions AICPA: Industry/Sector Perspective
10.12
The profitability index is a tool for measuring a
project’s benefits relative to its costs. How might this help to eliminate bias in project selection?
Solution: Since the profitability index is a modified pure-return-type measure, it offers a method to maximize the use of capital that is employed by the firm. This could help eliminate some types of SM 19-386
bias if the measure were to be employed universally. However, it does not necessarily maximize the use of capital that is not employed, which could in some circumstances be problematic. LO: 6 Level: Basic Bloomcode: Comprehension AASCB: Analytic IMA: Investment Decisions AICPA: Industry/Sector Perspective
Questions and Problems BASIC 10.1
Net present value: Riggs Corp. management is planning to spend $650,000 on a newmarketing campaign. They believe that this action will result in additional cash flows of $325,000 over the next three years. If the discount rate is 17.5 percent, what is the NPV on this project?
Solution: Initial investment = $650,000 Annual cash flows = $325,000 Length of project = n = 3 years Required rate of return = k = 17.5% Net present value = NPV
SM 19-387
NPV n NCFt $650,000 $325,000 $325,000 $325,000 (1.175)1 (1.175)2 (1.175)3 t 0 (1 k)t $650,000 276,596 $235,401 $200,341 $62,337 LO 2 Bloomcode: Application AASCB: Analytic IMA: Investment Decisions AICPA: Industry/Sector Perspective
10.2
Net present value: Kingston, Inc. management is considering purchasing a new machine at a cost of $4,133,250. They expect this equipment to produce cash flows of $814,322, $863,275, $937,250, $1,017,112, $1,212,960, and $1,225,000 over the next six years. If the appropriate discount rate is 15 percent, what is the NPV of this investment?
Solution: Cost of new machine = $4,133,250 Length of project = n = 6 years Required rate of return = k = 15% NPV NCFtt t0 (1 k) n
$814,322 $863,275 $937,250 $1,017,112 $1,212,960 $1,225,000 (1.15)1 (1.15)2 (1.15)3 (1.15)4 (1.15)5 (1.15)6 $4,133,250 $708,106 $652,760 $616257 $581,537 $603,055 $529,601 $4,133,250
$441,933 LO 2 Bloomcode: Application AASCB: Analytic IMA: Investment Decisions AICPA: Industry/Sector Perspective
10.3
Net present value: Crescent Industries management is planning to replace some existing SM 19-388
machinery in its plant. The cost of the new equipment and the resulting cash flows are shown in the accompanying table. If the firm uses an 18 percent discount rate for projects like this, should management go ahead with the project? Year
Cash Flow
0
−$3,300,000
1
875,123
2
966,222
3
1,145,000
4
1,250,399
5
1,504,445
Solution: Initial investment = $3,300,000 Length of project = n = 5 years Required rate of return = k = 18% NPV n NCFt
(1 k)t $875,123 $966,222 $1,145,000 $1,250,399 $1,504,455 $3,300,000 (1.18)1 (1.18)2 (1.18)3 (1.18)4 (1.18)5 $3,300,000 $741,630 $693,926 $696,882 $644,942 $657,607 t0
$134,986
Since the NPV is positive, the firm should accept the project. LO 2 Bloomcode: Application AASCB: Analytic IMA: Investment Decisions AICPA: Industry/Sector Perspective
10.4
Net present value: Management of Franklin Mints, a confectioner, is considering purchasing a new jelly bean-making machine at a cost of $312,500. They project that the
SM 19-389
cash flows from this investment will be $121,450 for the next seven years. If the appropriate discount rate is 14 percent, what is the NPV for the project?
Solution: Initial investment = $312,500 Annual cash flows = $121,450 Length of project = n = 7 years Required rate of return = k = 14% NPV n NCFt
(1 k)t $121,450 $121,450 $121,450 $121,450 $121,450 $312,500 (1.14)1 (1.14)2 (1.14)3 (1.14)4 (1.14)5 $121,450 $121,450 (1.14)6 (1.14)7 $312,500 $106,535 $93,452 $81,975 $71,908 $63,077 $55,331 $48,536 t 0
$208,315 LO 2 Bloomcode: Application AASCB: Analytic IMA: Investment Decisions AICPA: Industry/Sector Perspective
10.5
Net present value: Blanda Incorporated management is considering investing in two alternative production systems. The systems are mutually exclusive, and the cost of the new equipment and the resulting cash flows are shown in the accompanying table. If the firm uses a 9 percent discount rate for production system projects, in which system should the firm invest? Year 0 1 2 3
System 1 -15,000 15,000 15,000 15,000
System 2 -45,000 32,000 32,000 32,000
Solution SM 19-390
The NPV of System 1 is $22,969.42 and the NPV of System 2 is $36,001.43. Since the NPV of the System 2 is larger than the NPV for System 1, and the investments are mutually exclusive, the firm should take System 2. LO 2 Bloomcode: Analysis AASCB: Analytic IMA: Investment Decisions AICPA: Industry/Sector Perspective
10.6
Payback: Refer to problem 10.5. What are the payback periods for Production Systems 1 and 2? If the systems are mutually exclusive and the firm always chooses projects with the lowest payback period, in which system should the firm invest?
Solution System 1 has a payback of exactly one year. System 2 has a payback of 1.41 years. Given the shorter payback period for system 1, the investment should be made in System 1 based on the payback criteria. LO 3 Bloomcode: Analysis AASCB: Analytic IMA: Investment Decisions AICPA: Industry/Sector Perspective
10.7
Payback: Quebec, Inc., is purchasing machinery at a cost of $3,768,966. The company’s management expects the machinery to produce cash flows of $979,225, $1,158,886, and $1,881,497 over the next three years, respectively. What is the payback period?
Solution: Cumulative Year
CF
Cash Flow
0
$(3,768,966)
$(3,768,966)
1
979,225
(2,789,741) SM 19-391
2
1,158,886
(1,630,855)
3
1,881,497
250,642
PB = Years before cost recovery + (Remaining cost to recover/ Cash flow during the year) = 2 + ($1,630,855 / $1,881,497) = 2.87 years LO 3 Bloomcode: Application AASCB: Analytic IMA: Investment Decisions AICPA: Industry/Sector Perspective
10.8
Payback: Northern Specialties just purchased inventory-management computer software at a cost of $1,645,276. Cost savings from the investment over the next six years will produce the following cash flow stream: $212,455, $292,333, $387,479, $516,345, $645,766, and $618,325. What is the payback period on this investment?
Solution: Cumulative Year
CF
Cash Flow
0
$(1,645,276)
$(1,645,276)
1
212,455
(1,432,821)
2
292,333
(1,140,488)
3
387,479
(753,009)
4
516,345
(236,664)
5
645,766
409,102
6
618,325
1,027,427
PB = Years before cost recovery + (Remaining cost to recover/ Cash flow during the year) = 4 + ($236,664 / $645,766) = 4.37 years LO 3 Bloomcode: Application SM 19-392
AASCB: Analytic IMA: Investment Decisions AICPA: Industry/Sector Perspective
10.9
Payback: Nakamichi Bancorp has made an investment in banking software at a cost of $1,875,000. Management expects productivity gains and cost savings over the next several years. If, as a result of this investment, te firm is expected to generate additional cash flows of $586,212, $713,277, $431,199, and $318,697 over the next four years, what is the investment’s payback period?
Solution: Cumulative Year
CF
Cash Flow
0
$(1,875,000)
$(1,875,000)
1
586,212
(1,288,788)
2
713,277
(575,511)
3
431,199
(144,312)
4
318,697
174,385
PB = Years before cost recovery + (Remaining cost to recover/ Cash flow during the year) = 3 + ($144,312 / $318,697) = 3.45 years LO 3 Bloomcode: Application AASCB: Analytic IMA: Investment Decisions AICPA: Industry/Sector Perspective 10.10 Average accounting rate of return (ARR): Capitol Corp. management is expecting a project to generate after-tax income of $63,435 in each of the next three years. The average book value of the project’s equipment over that period will be $212,500. If the firm’s investment decision on any project is based on an ARR of 37.5 percent, should this project be accepted? SM 19-393
Solution: Annual after-tax income = $63,435 Average after-tax income = ($63,435 +$63,435 + $63,435) / 3 = $63,435 Average book value of equipment = $212,500 Accounting rate of return
Average after - tax income
Average book value $63,435 29.9% $212,500 Since the project’s ARR is below the acceptance rate of 37.5 percent, the project should be rejected. LO 4 Bloomcode: Analysis AASCB: Analytic IMA: Investment Decisions AICPA: Industry/Sector Perspective
10.11 Internal rate of return: Refer to Problem 10.4. What is the IRR that Franklin Mints management can expect on this project?
Solution: Initial investment = $312,500 Annual cash flows = $121,450 Length of project = n = 7 years Required rate of return = k = 14% To determine the IRR, a trial-and-error approach can be used. Set NPV = 0. Since the project had a positive NPV of $208,315, try IRR > k. Try IRR = 25%.
SM 19-394
NPV 0 n
NCFt t t0 (1 IRR)
1 1 7 0 $312,500 $121,450 (1.25) 0.25 $312,500 $383,920 $71,420
Try a higher rate, IRR = 34%.
NPV 0 n
NCFt t t0 (1 IRR)
1 1 7 0 $312,500 $121,450 (1.34) 0.34 $312,500 $311,161 $1,339
Try a lower rate, IRR = 33.8%. NPV 0 n
NCFt t t0 (1 IRR)
1 1 (1.338)7 0 $312,500 $121,450 0.338 $312,500 $312,515 $15 0
The IRR of the project is 33.8 percent. Using a financial calculator, we find that the IRR is 33.8 percent. LO 5 Bloomcode: Application AASCB: Analytic IMA: Investment Decisions AICPA: Industry/Sector Perspective
SM 19-395
10.12 Internal rate of return: Hathaway, Inc., a resort management company, is refurbishing one of its hotels at a cost of $7.8 million. Management expects that this will lead to additional cash flows of $1.8 million for the next six years. What is the IRR of this project? If the appropriate cost of capital is 12 percent, should Hathaway go ahead with this project?
Solution: Initial investment = $7,800,000 Annual cash flows = $1,800,000 Length of project = n = 6 years Required rate of return = k = 12% To determine the IRR, a trial-and-error approach can be used. Set NPV = 0. Try IRR = 12%. NPV 0 n
NCFt t t0 (1 IRR)
1 1 6 0 $7,800,000 $1,800,000 (1.12) 0.12 $7,800,000 $7,400,533 $399,467
Since NPV < 0, try a lower rate, IRR = 10%. NPV 0 n
NCFt t t0 (1 IRR)
1 1 6 0 $7,800,000 $1,800,000 (1.10) 0.10 $7,800,000 $7,839,469 $39,469
Try IRR = 10.2%.
SM 19-396
NPV 0 n
NCFt t t0 (1 IRR)
1 1 6 0 $7,800,000 $1,800,000 (1.102) 0.102 $7,800,000 $7,793,735 $6,265
Try IRR = 10.15%. NPV 0 n
NCFt t t0 (1 IRR)
1 1 6 0 $7,800,000 $1,800,000 (1.1015) 0.1015
$7,800,000 $7,805,129 $5,129 The IRR of the project is between 10.15 percent and 10.2 percent. Using a financial calculator, we find that the IRR is 10.1725 percent. Since IRR < k, reject the project. LO 5 Bloomcode: Analysis AASCB: Analytic IMA: Investment Decisions AICPA: Industry/Sector Perspective
10.13
Profitability index: What is the profitability index,
and why is it helpful in the capital rationing process?
Solution:
SM 19-397
The profitability index is computed as the ratio of NPV plus initial investment divided by initial investment. In the capital rationing process, we can calculate the profitability index for each potential investment and choose the projects with the largest indexes until we run out of capital. This follows the basic principle that we need to choose the set of projects that creates the greatest value given the limited capital available. LO 6 Bloomcode: Application AASCB: Analytic IMA: Investment Decisions AICPA: Industry/Sector Perspective
INTERMEDIATE 10.14 Net present value: Champlain Corp. management is investigating two computer systems. The Alpha 8300 costs $3,122,300 and will generate cost savings of $1,345,500 in each of the next five years. The Beta 2100 system costs $3,750,000 and will produce cost savings of $1,125,000 in the first three years and then $2 million for the next two years. If the company’s discount rate for similar projects is 14 percent, what is the NPV for the two systems? Which one should be chosen based on the NPV?
Solution: Cost of Alpha 8300 = $3,122,300 SM 19-398
Annual cost savings = $1,345,500 Length of project = n = 5 years Required rate of return = k = 14% 1 1 (1.14)5 NPV $3,122,300 $1,345,500 t t0 (1 k) 0.14 n
NCFt
$3,122,300 $4,619,210 $1,496,910
Cost of Beta 2100 = $3,750,000 Length of project = n = 5 years Required rate of return = k = 14% 1 1 3 (1.14) $2,000,000 $2,000,000 NPV $3,750,000 $1,125,500 t 4 (1.14) (1.14)5 t0 (1 k) 0.14 $3,750,000 $2,611,836 $1,184,161 1,038,737 n
NCFt
$1,084,734 Based on the NPV, the Alpha 8300 system should be chosen. LO 2 Bloomcode: Analysis AASCB: Analytic IMA: Investment Decisions AICPA: Industry/Sector Perspective
10.15 Net present value: Briarcrest Condiments is a spice-making firm. Recently, it developed a new process for producing spices. The process requires new machinery that would cost $1,968,450, have a life of five years, and would produce the cash flows shown in the following table. What is the NPV if the discount rate is 15.9 percent? Year
Cash Flow
1
$512,496
2
(242,637)
SM 19-399
3
814,558
4
887,225
5
712,642
Solution: Cost of equipment = $1,968,450 Length of project = n = 5 years Required rate of return = k = 15.9% NPV NCFtt t0 (1 k) n
$512,496 $242,637 $814,558 $887,225 $712,642 (1.159)1 (1.159)2 (1.159)3 (1.159)4 (1.159)5 $1,968,450 $442,188 $180,630 $523,205 491,700 340,764 $1,968,450
$351,223 LO 2 Bloomcode: Application AASCB: Analytic IMA: Investment Decisions AICPA: Industry/Sector Perspective
10.16 Net present value: Cranjet Industries is expanding its product line and its production capacity. The costs and expected cash flows of the two independent projects are given in the following table. The firm uses a discount rate of 16.4 percent for such projects. a.
What are the NPVs of the two projects?
b
Should both projects be accepted? Or either? Or neither? Explain your reasoning.
Product Line
Production Capacity
Year
Expansion
Expansion
0
$(2,575,000)
$(8,137,250)
1
600,000
2,500,000
SM 19-400
2
875,000
2,500,000
3
875,000
2,500,000
4
875,000
3,250,000
5
875,000
3,250,000
Solution: a.
Required rate of return = k =16.4% Product Line Expansion: Cost of product line expansion = $2,575,000 NPV NCFtt t0 (1 k) n
$600,000 $875,000 $875,000 $875,000 $875,000 (1.164)1 (1.164)2 (1.164)3 (1.164)4 (1.164)5 $2,575,000 $515,464 $645,806 $554,816 476,646 409,490 $2,575,000
$27,222
Production Capacity Expansion: Cost of production capacity expansion = $8,137,250 NPV NCFtt t0 (1 k) n
1 1 3 (1.164) $3,250,000 $3,250,000 $8,137,250 $2,500,000 4 (1.164)5 0.164 (1.164) $8,137,250 $5,578,116 $1,770,400 $1,520,962
$732,228 b.
Since they are independent, and both have NPV > 0, both projects should be accepted.
LO 2 Bloomcode: Application AASCB: Analytic IMA: Investment Decisions AICPA: Industry/Sector Perspective
SM 19-401
10.17 Net present value: Emporia Mills management is evaluating two alternative heating systems. Costs and projected energy savings are given in the following table. The firm uses 11.5 percent to discount such project cash flows. Which system should be chosen?
Year
System 100
System 200
0
$(1,750,000)
$(1,735,000)
1
275,223
750,000
2
512,445
612,500
3
648,997
550,112
4
875,000
384,226
Solution: Required rate of return = k = 11.5% System 100: Cost of System 100 = $1,750,000 n NPV NCFt t $1,750,000 $275,2231 $512,4452 $648,9973 $875,0004 (1.115) (1.115) (1.115) (1.115) t0 (1 k) $1,750,000 $246,837 $412,190 $468,186 566,120
$56,667
System 200: Cost of System 200 = $1,735,000 n NPV NCFt t $1,735,000 $750,0001 $612,5002 $550,1123 $384,2264 (1.115) (1.115) (1.115) (1.115) t0 (1 k) $1,735,000 $672,646 $492,670 $396,850 248,592
$75,758 Since System 200 has a positive NPV, select that system. Reject System 100 as it has negative NPV. LO 2 Bloomcode: Analysis AASCB: Analytic SM 19-402
IMA: Investment Decisions AICPA: Industry/Sector Perspective
10.18 Payback: Creative Solutions, Inc., has just invested $4,615,300 in new equipment. The firm uses a payback period criteria of not accepting any project that takes more than four years to recover its costs. Management anticipates cash flows of $644,386, $812,178, $943,279, $1,364,997, $2,616,300, and $2,225,375 over the next six years. Does this investment meet the firm’s payback criteria?
Solution: Cumulative Year
CF
Cash Flow
0
$(4,615,300)
$(4,615,300)
1
644,386
(3,970,914)
2
812,178
(3,158,736)
3
943,279
(2,215,457)
4
1,364,997
(850,460)
5
2,616,300
1,765,840
6
2,225,375
3,991,215
Payback period = Years before cost recovery +
Remaining cost to recover Cash flow during the year
$850, 460 = 4 $2, 616, 300 4.33 years Since the project payback period exceeds the firm’s target of four years, it should not have been accepted. LO 3 Bloomcode: Analysis AASCB: Analytic IMA: Investment Decisions SM 19-403
AICPA: Industry/Sector Perspective
10.19 Discounted payback: Timeline Manufacturing Co. management is evaluating two projects. The company uses payback criteria of three years or less. Project A has a cost of $912,855, and project B’s cost is $1,175,000. Cash flows from both projects are given in the following table. What are their discounted payback periods, and which will be accepted with a discount rate of 8 percent? Year
Project A
Project B
1
$ 86,212
$586,212
2
313,562
413,277
3
427,594
231,199
4
285,552
Solution: Project A Cumulative Year
CF
CF
0
$(912,855)
1
Cumulative PVCF
PVCF
$(912,855)
$(912,855)
$(912,855)
86,212
(826,643)
79,826
(833,029)
2
313,562
(513,081)
268,829
(564,200)
3
427,594
(85,487)
339,438
(224,762)
4
285,552
200,065
209,889
(14,873)
The payback period of Project A exceeds three years. Project B Cumulative Year 0
CF
CF
$(1,175,000) $(1,175,000)
Cumulative PVCF
PVCF
$(1,175,000)
$(1,175,000)
1
586,212
(588,788)
542,789
(632,211)
2
413,277
(175,511)
354,318
(277,893)
3
231,199
55,688
183,533
(94,359) SM 19-404
The payback period of Project A exceeds three years. Since the firm’s acceptance criteria is three years, neither project will be accepted. LO 3 Bloomcode: Analysis AASCB: Analytic IMA: Investment Decisions AICPA: Industry/Sector Perspective
10.20 Payback: Regent Corp. management is evaluating three competing types of equipment. Costs and cash flow projections for all three are given in the following table. Which would be the best choice based on payback period? Year
Type 1
Type 2
Type 3
0
$(1,311,450)
$(1,415,888)
$(1,612,856)
1
212,566
586,212
786,212
2
269,825
413,277
175,000
3
455,112
331,199
175,000
4
285,552
141,442
175,000
5
121,396
175,000
6
175,000
Solution: Type 1
Type 2
Cumulative
Type 3
Cumulative
Cumulative
Year
CF
CF
CF
CF
CF
CF
0
$(1,311,450)
$(1,311,450)
$(1,415,888)
$(1,415,888)
$(1,612,856)
($1,612,856)
1
212,566
(1,098,884)
586,212
(829,676)
786,212
(826,644)
2
269,825
(829,059)
413,277
(416,399)
175,000
(651,644)
3
455,112
(373,947)
331,199
(85,200)
175,000
(476,644)
4
285,552
(88,395)
141,442
56,242
175,000
(301,644)
SM 19-405
5
121,396
33,001
6
175,000
(126,644)
175,000
48,356
Type 1: Payback period = Years before cost recovery +
Remaining cost to recover Cash flow during the year
$88, 395 = 4 $121, 396 4.73 years
Type 2: Payback period = Years before cost recovery +
Remaining cost to recover Cash flow during the year
$85, 200 = 3 $141, 442 3.6 years Type 3: Payback period = Years before cost recovery +
Remaining cost to recover Cash flow during the year
$126, 644 $175, 000 5.72 years
=5
Select Type 2 because it has the lowest payback period. LO 3 Bloomcode: Analysis AASCB: Analytic IMA: Investment Decisions AICPA: Industry/Sector Perspective
10.21 Discounted payback: Nugent Communication Corp. is investing $9,365,000 in new technologies. The company’s management expects significant benefits in the first three years after installation (as can be seen by the following cash flows), and smaller constant benefits in each of the next four years. What is the discounted payback period for the SM 19-406
project assuming a discount rate of 10 percent? Years
Cash flows
1
2
3
4–7
$2,265,433
$4,558,721
$3,378,911
$1,250,000
Solution: Discount rate = k = 10% Cumulative Year 0
CF
CF
$(9,365,000) $(9,365,000)
Cumulative PVCF
PVCF
$(9,365,000)
$(9,365,000)
1
2,265,433
(7,099,567)
2,059,485
(7,305,515)
2
4,558,721
(2,540,846)
3,767,538
(3,537,977)
3
3,378,911
838,065
2,538,626
(999,352)
4
1,250,000
2,088,065
853,767
(145,585)
5
1,250,000
3,338,065
776,152
630,567
6
1,250,000
4,588,065
705,592
1,336,159
7
1,250,000
5,838,065
641,448
1,977,607
Discounted PB = Years before cost recovery + (Remaining cost to recover/ Cash flow during the year) = 4 + ($145,585 / $776,152) = 4.19 years LO 3 Bloomcode: Application AASCB: Analytic IMA: Investment Decisions AICPA: Industry/Sector Perspective
10.22 Modified internal rate of return (MIRR): Morningside Bakeries recently purchased equipment at a cost of $650,000. Management expects the equipment to generate cash flows of $275,000 in each of the next four years. The cost of capital is 14 percent. What is the SM 19-407
MIRR for this project?
Solution: PV of costs = $650,000 Length of project = n = 4 years Cost of capital = k = 14% Annual cash flows = CFt = $275,000 TV CF1(1 k )n1 CF2 (1 k ) n2 CF (1n k )nn $275,000(1.14)3 $275,000(1.14)2 $275,000(1.14)1 $275,000(1.14)0 $407,425 $357,390 $313,500 $275,000 $1,353,315 Now we can solve for the MIRR using Equation 9.5. TV (1 MIRR)t $1,353,315 $650,000 (1 MIRR)4 $1,353,315 (1 MIRR)4 2.0820 $650,000 PVCosts
(1 MIRR) (2.0820) 4 1.2012 1
MIRR 0.2012 20.1%
LO 5 Bloomcode: Application AASCB: Analytic IMA: Investment Decisions AICPA: Industry/Sector Perspective
10.23 Modified internal rate of return (MIRR): Management of Sycamore Home Furnishings is considering acquiring a new machine that can create customized window treatments. The equipment will cost $263,400 and will generate cash flows of $85,000 over each of the next six years. If the cost of capital is 12 percent, what is the MIRR on this project?
Solution: PV of costs = $263,400 SM 19-408
Length of project = n = 6 years Cost of capital = k = 12% Annual cash flows = CFt = $85,000 TV CF1(1 k )n1 CF2 (1 k )n2 CF (1n k )nn $85,000(1.12)5 $85,000(1.12)4 $85,000(1.12)3 $85,000(1.12)2 $85,000(1.12)1 $85,000(1.12)0 $149,799 $133,749 $119,419 $106,624 $95,200 $85,000 $689,791 Now we can solve for the MIRR using Equation 9.5. TV (1 MIRR)t $689, 791 $263, 400 (1 MIRR)6 $689, 791 (1 MIRR)6 2.6188 $263, 400 PVCosts
(1 MIRR) (2.6188) 6 1.1740 1
MIRR 0.1740 17.4% LO 5 Bloomcode: Application AASCB: Analytic IMA: Investment Decisions AICPA: Industry/Sector Perspective
10.24 Internal rate of return: Management of Great Flights, Inc., an aviation firm, is considering purchasing three aircraft for a total cost of $161 million. The company would lease the aircraft to an airline. Cash flows from the proposed leases are shown in the following table. What is the IRR of this project? Years
Cash Flow
1–4
$23,500,000
5–7
$72,000,000
8–10
$80,000,000
Solution:
SM 19-409
Years
Cash Flow
Initial investment =
1–4
$23,500,000
$161,000,000
Length of project =
5–7
$72,000,000
n = 10 years
Required rate of
8–10
$80,000,000
return = k =
To determine the IRR, the trial-and-error approach can be used. Set NPV=0. Try a higher rate than k = 15%; try IRR = 22%. NPV 0 n
NCFt t t0 (1 IRR)
1 1 1 1 (1.22)4 (1.22)3 1 $72,000,000 0 $161,000,000 $23,500,000 4 0.22 0.22 (1.22) 1 1 3 $80,000,000 (1.22) 1 7 0.22 (1.22)
$161,000,000 $58,600,552 $66,374,346 $40,614,233 $4,589,131 Try IRR = 23%.
NPV 0 NCFt t t0 (1 IRR) n
1 1 1 1 (1.23)4 (1.23)3 1 $72,000,000 0 $161,000,000 $23,500,000 4 0.23 0.23 (1.23) 1 1 (1.23)3 1 $80,000,000 7 0.23 (1.23)
$161,000,000 $57,534,386 $63,271,035 $37,778,708 $2,415,871 Try IRR = 22.7%. SM 19-410
NPV 0 n
NCFt t t0 (1 IRR)
1 1 1 1 (1.227)4 (1.227)3 1 $72,000,000 0 $161,000,000 $23,500,000 4 0.227 0.227 (1.227) 1 1 3 (1.227) 1 $80,000,000 7 0.227 (1.227)
$161,000,000 $57,850,786 $64,183,552 $38,605,355 $360,307 The IRR of the project is between 22 and 23 percent. Using a financial calculator, we find that the IRR is 22.65 percent. LO 5 Bloomcode: Application AASCB: Analytic IMA: Investment Decisions AICPA: Industry/Sector Perspective
10.25 Internal rate of return: Refer to problem 10.5. Compute the IRR for both production System 1 and production System 2. Which has the higher IRR? Which production system has the higher NVP? Explain why the IRR and NPV rankings of Systems 1 and 2 are different?
Solution The IRR of system 1 is 83.93 percent and the IRR of system 2 is 50.07 percent. The NPV of system 1 is $22,969.42 and the NPV of system 2 is $36,001.43. System 1 delivers a higher IRR because it requires a lower initial investment and the cost is recovered the first year. Thus, even with lower cash inflows in the years after startup, system 1is able to deliver a higher return on the initial investment. System 2 has a higher initial investment but delivers a higher net cash flow for the firm. LO 5 SM 19-411
Bloomcode: Analysis AASCB: Analytic IMA: Investment Decisions AICPA: Industry/Sector Perspective
10.26 Internal rate of return: Ancala Corporation management is considering investments in two new golf apparel lines for next season: golf hats and belts. Due to a funding constraint, these lines are mutually exclusive. A summary of each project’s estimated cash flows over its three -year life, as well as the IRR and NPV of each are outlined below. The CFO of the firm has decided to manufacture the belts; however, the CEO is questioning this decision given that the IRR is higher for manufacturing hats. Explain to the CEO why the IRRs and NPVs of the belt and hat projects disagree? Is the CFO’s decision the correct? Year 0 1 2 3
Golf Belts -$1,000 1,000 500 500
Golf Hats -$500 500 300 300
NPV IRR
$697.97 54%
$427.87 61%
Solution The IRRs and NPVs of the belt and hat lines disagree because of the differences in the scale of the project. Golf hats deliver a higher IRR because they require a lower initial investment. Thus, even with lower cash inflows in the years after startup, the golf hat project is able to deliver a higher return on the initial investment. While the golf belts project does cost more, it delivers a higher net cash flow for Ancala’s investors. This NPV factors in the initial cost of the project, and reflects the total net cash flow for the firm’s shareholders. The CFO’s decision to choose the golf belts project is the right choice because it yields the higher net cash flows for Ancala’s investors. LO 5 SM 19-412
Bloomcode: Analysis AASCB: Analytic IMA: Investment Decisions AICPA: Industry/Sector Perspective
10.27 Internal rate of return: Compute the IRR on the following cash flow streams: a.
An initial investment of $25,000 followed by a single cash flow of $37,450 in year 6.
b.
An initial investment of $1 million followed by a single cash flow of $1,650,000 in year 4.
c.
An initial investment of $2 million followed by cash flows of $1,650,000 and $1,250,000 in years 2 and 4, respectively.
LO 5 Bloomcode: Application AASCB: Analytic IMA: Investment Decisions AICPA: Industry/Sector Perspective
Solution: a.
Try IRR = 7%. NPV 0 n
NCFt
t0 (1 IRR)
0 $25,000
t
$37,450
(1.07)6 $25,000 $24,955 $45
Try IRR = 6.97%. NPV 0 n
NCFt t t0 (1 IRR) $37,450 0 $25,000 (1.0697)6 $25,000 $24,997 $3
The IRR of the project is approximately 6.97 percent. Using a financial calculator, we find that the IRR is 6.968 percent.
SM 19-413
b.
Try IRR = 12%. NPV 0 n
NCFt
t t0 (1 IRR)
0 $1,000,000
$1,650,000
(1.12)4 $1,000,000 $1,048,605 $48,605
Try IRR = 13%. NPV 0 n
NCFt t t0 (1 IRR) $1,650,000 0 $1,000,000 (1.13)4 $1,000,000 $1,011,976 $11,976
Try IRR = 13.3%. NPV 0 n
NCFt t t0 (1 IRR) $1,650,000 0 $1,000,000 (1.133)4 $1,000,000 $1,001,300 $1,300
The IRR of the project is approximately 13.3 percent. Using a financial calculator, we find that the IRR is 13.337 percent.
c.
Try IRR = 15%. NPV 0 n
NCFt
t0 (1 IRR)
0 $2,000,000
t
$1,650,000 $1,250,000 (1.15)2 (1.15)4
$2,000,000 $1,247,637 $714,692 $37,671
Try IRR = 14%. NPV 0 n
NCFt t t0 (1 IRR) $1,650,000 $1,250,000 0 $2,000,000 (1.14)2 (1.14)4 $2,000,000 $1,269,621 $740,100 $9,721 SM 19-414
The IRR of the project is between 14 and 15 percent. Using a financial calculator, we find that the IRR is 14.202 percent.
10.28 Internal rate of return: Compute the IRR for the following project cash flows: a.
An initial outlay of $3,125,000 followed by annual cash flows of $565,325 for the next eight years.
b.
An initial investment of $33,750 followed by annual cash flows of $9,430 for the next five years.
c.
An initial outlay of $10,000 followed by annual cash flows of $2,500 for the next seven years.
Solution: a.
Initial investment = $3,125,000 Annual cash flows = $565,325 Length of investment = n = 8 years
Try IRR = 8%. 1 1 8 (1.08) NPV $3,125,000 $565,325 t (1 k) 0.08 t0 $3,125,000 $3,248,719 n
NCFt
$123,719 Try a higher rate, IRR = 9%.
1 1 (1.09)8 NCFt NPV $3,125,000 $565,325 t 0.09 t0 (1 k) n
$3,125,000 $3,128,972 $3,972
SM 19-415
The IRR of the project is approximately 9 percent. Using a financial calculator, we find that the IRR is 9.034 percent.
b.
Initial investment = $33,750 Annual cash flows = $9,430 Length of investment = n = 5 years Try IRR = 12%. 1 1 (1.12)5 t NPV $33,750 $9,430 t 0.12 t0 (1 k) $33,750 $33,993 n
NCF
$243
Try IRR = 12.3%.
1 n 1 (1.123)5 NCFt NPV $33,750 $9,430 t t0 (1 k) 0.123
$33,750 $33742 $8 0 The IRR of the project is approximately 12.3 percent. Using a financial calculator, we find that the IRR is 12.29 percent.
c.
Initial investment = $10,000 Annual cash flows = $2,500 Length of investment = n = 7 years Try IRR = 16%.
SM 19-416
1 7 1 (1.16) NPV $10,000 $2,500 t 0.16 t0 (1 k) n
NCF
t
$10,000 $10,096 $96 Try IRR = 16.3%.
1 1 (1.163)7 NCFt NPV $10,000 $2,500 t (1 k) 0.163 t0 $10,000 $10,008 n
$8 0 The IRR of the project is approximately 16.3 percent. Using a financial calculator, we find that the IRR is 16.327 percent. LO 5 Bloomcode: Application AASCB: Analytic IMA: Investment Decisions AICPA: Industry/Sector Perspective
9.29
Profitability index: Suppose that you could invest in the following projects but have only $30,000 to invest. How would you make your decision and in which projects would you invest? Project
Cost
NPV
A
$ 8,000
$4,000
B
11,000
7,000
C
9,000
5,000
D
7,000
4,000
SM 19-417
Solution: One would compute the Profitability index for each of the projects as follows: PI
NPV+Initial investment Initial investment
The profitability indexes of the projects are: PIA
$4, 000 $8, 000 1.50 $8, 000
PIB
$7,000 $11,000 1.64 11,000
PIC
$5,000 $9,000 1.56 $9,000
PID
$4,000 $7,000 1.57 $7,000
With $30,000, you should invest in B, D, and C. The total cost is $27,000, and the total NPV is $16,000. LO 6 Bloomcode: Analysis AASCB: Analytic IMA: Investment Decisions
AICPA: Industry/Sector Perspective SM 19-418
10.30 Profitability index: Suppose that you could invest in the same projects as in the previous problem, but have only $25,000 to invest. Which projects would you chose?
Solution: The profitability indexes of the projects are: A: 1.50; B: 1.64; C: 1.56; D: 1.57 With $25,000, you cannot invest in all of B, D, and C, since the total cost is $27,000. You may think that you should then invest in only B and D, since they have the highest profitability indexes. This will yield a total NPV of $11,000, and you are left with $7,000 of idle capital. If you give up project B, however, which has the highest profitability index and highest cost, and invest instead in A, C, and D, which require less capital, you will get a total NPV of $13,000, and you are left with less idle capital ($1,000). From
SM 19-419
this example you can see that capital rationing with indivisible projects are sometimes complicated and require a careful thought of all possibilities (or linear/integer programming). LO 6 Bloomcode: Analysis AASCB: Analytic IMA: Investment Decisions
AICPA: Industry/Sector Perspective
ADVANCED 10.31 Management of Draconian Measures, Inc., is evaluating two independent projects. The company uses a 13.8 percent discount rate for such projects. The costs and cash flows for the projects are shown in the following table. What are their NPVs? Year
Project 1
Project 2
0
$(8,425,375)
$(11,368,000)
1
3,225,997
2,112,589
2
1,775,882
3,787,552
3
1,375,112
3,125,650
4
1,176,558
4,115,899
5
1,212,645
4,556,424
6
1,582,156
7
1,365,882
Solution: Project 1: SM 19-420
Cost of Project 1 = $8,425,375 Length of project = n = 7 years Required rate of return = k = 13.8% NPV NCFtt t0 (1 k ) n
$3,225,997 $1,775,882 $1,375,112 $1,176,558 $1,212,645 (1.138)1 (1.138)2 (1.138)3 (1.138)4 (1.138)5 $1,582,156 $1,365,882 (1.138)6 (1.138)7 $8,425,375 $2,834,795 $1,371,291 $933,064 701,527 $635,364 $728,443 $8,425,375
$552,608 $668,283 Since Project 1 NPV is negative, we reject this project.
Project 2: Cost of Project 2 = $11,368,000 Length of project = n = 5 years Required rate of return = k = 13.8% NPV NCFtt t0 (1 k) n
$2,112,589 $3,787,552 $3,125,650 $4,115,899 $4,556,424 (1.138)1 (1.138)2 (1.138)3 (1.138)4 (1.138)5 $11,368,000 $1,856,405 $2,924,651 $2,120,868 $2,454,119 $2,387,332 $11,368,000
$375,375 Since Project 2 NPV is positive, we accept this project. LO 2 Bloomcode: Analysis AASCB: Analytic IMA: Investment Decisions AICPA: Industry/Sector Perspective
10.32 Refer to Problem 10.31.
SM 19-421
a.
What are the IRRs for the projects?
b.
Does the IRR criterion suggest a different decision than the NPV criterion?
c. Explain how you would expect the management of Draconian Measures to decide which project(s) to invest in.
Solution: a.
Project 1: At the required rate of return of 13.8 percent, Project 1 has a NPV of $(668,283). To find the IRR, try lower rates. Try IRR = 11%. NPV NCFtt t0 (1 k) n
$3,225,997 $1,775,882 $1,375,112 $1,176,558 $1,212,645 (1.11)1 (1.11)2 (1.11)3 (1.11)4 (1.11)5 $1,582,156 $1,365,882 (1.11)6 (1.11)7 $8,425,375 $2,906,304 $1,441,346 $1,005,470 $775,035 $719,646 $845,885 $657,889 $8,425,375
$73,801 Try a lower rate, IRR=10.7%. NPV NCFtt t0 (1 k) n
$3,225,997 $1,775,882 $1,375,112 $1,176,558 $1,212,645 (1.107)1 (1.107)2 (1.107)3 (1.107)4 (1.107)5 $1,582,156 $1,365,882 (1.107)6 (1.107)7 $8,425,375 $2,914,180 $1,449,168 $1,013,667 $783,471 $729,450 $859,733 $670,471 $8,425,375
$5,235 The IRR of the project is approximately 10.7 percent. Using a financial calculator, we find that the IRR is 10.677 percent.
SM 19-422
Project 2: At the required rate of return of 13.8 percent, Project 2 has a NPV of $ 375,375. To find the IRR, try higher rates. Try IRR = 15%. NPV 0 n
NCFt t t0 (1 IRR) $2,112,589 $3,787,552 $3,125,650 $4,115,899 $4,556,424 0 $11,368,000 (1.15)1 (1.15)2 (1.15)3 (1.15)4 (1.15)5 $11,368,000 $1,837,034 $2,863,933 $2,055,166 $2,353,279 $2,265,348 $6,760
Try IRR = 15.1%. NPV 0 n
NCFt t t0 (1 IRR) $2,112,589 $3,787,552 $3,125,650 $4,115,899 $4,556,424 0 $11,368,000 (1.151)1 (1.151)2 (1.151)3 (1.151)4 (1.151)5 $11,368,000 $1,835,438 $2,858,959 $2,049,814 $2,345,111 $2,255,524 $23,154
The IRR of the project is between 15 and 15.1 percent. Using a financial calculator, we find that the IRR is 15.023 percent.
b.
Based on the IRR, Project 1 will be rejected and Project 2 will be accepted. These decisions are identical to those based on NPV. (Given that Project 1 had a negative NPV, the IRR will always be less than the required rate of return 13.8%.)
c.
Management would use the decision spelled out by NPV, although in this case the IRR has come up with the same decision.
LO 5 Bloomcode: Application AASCB: Analytic IMA: Investment Decisions AICPA: Industry/Sector Perspective
SM 19-423
10.33 Management of Dravid, Inc., is currently evaluating three projects that are independent. The cost of funds can be either 13.6 percent or 14.8 percent depending on their financing plan. All three projects cost the same at $500,000. Expected cash flow streams are shown in the following table. Which projects would be accepted at a discount rate of 14.8 percent? What if the discount rate was 13.6 percent? Year
Project 1
Project 2
Project 3
1
$0
$0
$245,125
2
125,000
0
212,336
3
150,000
500,000
112,500
4
375,000
500,000
74,000
Solution: Cost of projects = $500,000 Length of project = n = 4 years Required rate of return = k = 14.8%
Project 1:
NPV NCFt t $500,000 $0 1 $125,0002 $150,0003 $375,0004 (1.148) (1.148) (1.148) (1.148) t0 (1 k) $500,000 $0 $94,848 $99,144 $215,906 n
$90,103
Project 2:
NPV NCFt t $500,000 $0 1 $0 2 $500,0003 $500,0004 (1.148) (1.148) (1.148) (1.148) t0 (1 k) n
$500,000 $0 $0 $330,479 $287,874 $118,353
Project 3:
SM 19-424
n NPV NCFt t $500,000 $245,1251 $212,3362 $112,5003 $74,0004 (1.148) (1.148) (1.148) (1.148) t0 (1 k) $500,000 $213,524 $161,116 $74,358 $42,605
$8,397
At a discount rate of 14.8 percent, only project 2 will be accepted. At a discount rate of 13.6 percent, the NPVs of the three projects are -$75,645, $141,295, and $1,491 respectively. Both projects 2 and 3 have positive NPVs and will be accepted.
Year
Project 1
PVCF
Project 2
PVCF
Project 3
PVCF
0
$(500,000)
$(500,000)
$(500,000)
$(500,000)
$(500,000)
$(500,000)
1
245,125
215,779
2
125,000
96,862
212,336
164,538
3
150,000
102,319
500,000
341,063
112,500
76,739
4
375,000
225,175
500,000
300,232
74,000
44,434
NPV
(75,645)
141,295
1,491
LO 2 Bloomcode: Analysis AASCB: Analytic IMA: Investment Decisions AICPA: Industry/Sector Perspective
10.34 Management of Intrepid, Inc., is considering investing in three independent projects. The costs and the cash flows are given in the following table. The appropriate cost of capital is 14.5 percent. Compute the project IRRs and identify the projects that should be accepted.
Year
Project 1
Project 2
Project 3
0
$(275,000)
$(312,500)
$(500,000)
1
63,000
153,250
212,000
2
85,000
167,500
212,000
SM 19-425
3
85,000
4
100,000
112,000
212,000 212,000
Solution: Project 1: Cost of Project 1 = $275,000 Length of project = n = 4 years Required rate of return = k = 14.5% NPV NCFtt t0 (1 k) n
$63,000 $85,000 $85,000 $100,000 (1.145)1 (1.145)2 (1.145)3 (1.145)4 $275,000 55,022 $64,835 $56,624 $58,181 $275,000
$40,338 At the required rate of return of 14.5 percent, Project 1 has a NPV of $(40,338). To find the IRR, try lower rates.
Try IRR = 7.6%. NPV 0 n
NCFt
t t0 (1 IRR)
$63,000 $85,000 $85,000 $100,000 (1.076)1 (1.076)2 (1.076)3 (1.076)4 $275,000 58,550 $73,417 $68,231 $74,602
0 $275,000
$200 0 The IRR of the project is approximately 7.6 percent. Using a financial calculator, we find that the IRR is 7.57 percent.
Project 2: Cost of Project 2 = $312,500 Length of project = n = 3 years Required rate of return = k = 14.5%
SM 19-426
n NPV NCFt t $312,500 $153,2501 $167,5002 $112,0003 (1.145) (1.145) (1.145) t0 (1 k) $312,500 $133,843 $127,763 $74,611
$23,717 At the required rate of return of 14.5 percent, Project 2 has a NPV of $23,717. To find the IRR, try higher rates. Try IRR =19%. NPV 0 n
NCFt t t0 (1 IRR) $153,250 $167,500 $112,000 0 $312,500 (1.19)1 (1.19)2 (1.19)3 $312,500 $128,782 $118,283 $66,463 $1,027
Try IRR=19.2%. NPV 0 n
NCFt t t0 (1 IRR) $153,250 $167,500 $112,000 0 $312,500 (1.192)1 (1.192)2 (1.192)3 $312,500 $128,565 $117,886 $66,129 $80 0
The IRR of the project is approximately 19.2 percent. Using a financial calculator, we find that the IRR is 19.22 percent.
Project 3: Cost of Project 3 = $500,000 Length of project = n = 4 years Required rate of return = k = 14.5% 1 1 (1.145)4 NPV $500,000 $212,000 t 0.145 t0 (1 k) n
NCFt
$500,000 $611,429 $111,429
SM 19-427
At the required rate of return of 14.5 percent, Project 3 has a NPV of $111,429. To find the IRR, try higher rates. Try IRR =25%. NPV 0 n
NCFt t t0 (1 IRR)
1 1 4 0 $500,000 $212,000 (1.25) 0.25 $500,000 $500,659
$659 Try IRR=25.1%. NPV 0 n
NCFt t t0 (1 IRR)
1 1 (1.251)4 0 $500,000 $212,000 0.251 $500,000 $500,659
$231 0 The IRR of the project is approximately 25.1 percent. Using a financial calculator, we find that the IRR is 25.07 percent. Only Projects 2 and 3 will be accepted as the IRRs exceed the required rate of return of 14.5 percent. Bloomcode: Application AASCB: Analytic IMA: Investment Decisions AICPA: Industry/Sector Perspective
10.35 Jekyll & Hyde Corp. management is evaluating two mutually exclusive projects. The cost of capital is 15 percent. Costs and cash flows for each project are given in the following table. Which project should be accepted? SM 19-428
Year
Project 1
Project 2
0
$(1,250,000)
$(1,250,000)
1
250,000
350,000
2
350,000
350,000
3
450,000
350,000
4
500,000
350,000
5
750,000
350,000
Solution: Project 1: Cost of project = $1,250,000 Length of project = n = 5 years Required rate of return = k = 15% NPV NCFtt t0 (1 k) n
$250,000 $350,000 $450,000 $500,000 $750,000 (1.15)1 (1.15)2 (1.15)3 (1.15)4 (1.15)5 $1,250,000 $217,391 $264,650 $295,882 $285,877 $372,883 $1,250,000
$186,683 At the required rate of return of 15 percent, Project 1 has an NPV of $186,683.Based on the positive NPV, Project 1 should be accepted.
To find the IRR, try higher rates.
Try IRR = 20%. NPV 0 NCFtt t0 (1 k) n
$250,000 $350,000 $450,000 $500,000 $750,000 (1.20)1 (1.20)2 (1.20)3 (1.20)4 (1.20)5 $1,250,000 $208,333 $243,056 $260,417 $241,127 $301,408
0 $1,250,000
$4,340
SM 19-429
Try IRR = 20.1%. NPV 0 n
NCFt t t0 (1 IRR) $250,000 $350,000 $450,000 $500,000 $750,000 0 $1,250,000 (1.201)1 (1.201)2 (1.201)3 (1.201)4 (1.201)5 $1,250,000 $208,160 $242,651 $259,767 $240,324 $300,155 $1,057 0
The IRR of the project is approximately 20.1 percent. Using a financial calculator, we find that the IRR is 20.132 percent.
Project 2: Cost of project = $1,250,000 Length of project = n = 5 years Required rate of return = k = 15% 1 1 5 (1.15) NPV $1,250,000 $350,000 t 0.15 t0 (1 k) n
NCFt
$1,250,000 $1,173,254 $76,746 At the required rate of return of 15 percent, Project 2 has an NPV of $(76,746). Based on the negative NPV, Project 2 should be rejected.
To find the IRR, try lower rates. Try IRR = 12%. NPV 0 n
NCFt t t0 (1 IRR)
1 1 5 0 $1,250,000 $350,000 (1.12) 0.12
$1,250,000 $1,261,672 $11,672
SM 19-430
Try IRR = 12.4%. NPV 0 n
NCFt t t0 (1 IRR)
1 1 5 0 $1,250,000 $350,000 (1.124) 0.124 $1,250,000 $1,249,269
$731 0
The IRR of the project is approximately 12.4 percent. Using a financial calculator, we find that the IRR is 12.376 percent.
Given a required rate of return of 15 percent, Project 1 will be accepted as the IRR of 20.1 percent exceeds the required rate of return. Project 2 will be rejected. LO 5 Bloomcode: Analysis AASCB: Analytic IMA: Investment Decisions AICPA: Industry/Sector Perspective
10.36 Management of Larsen Automotive, a manufacturer of auto parts, is considering investing in two projects. The company typically compares project returns to a cost of funds of 17 percent. Compute the IRRs based on the cash flows in the following table. Which project(s) will be accepted? Year
Project 1
Project 2
0
$(475,000)
$(500,000)
1
300,000
117,500
2
110,000
181,300
3
125,000
244,112
4
140,000
278,955 SM 19-431
Solution: Project 1: Cost of project = $475,000 Length of project = n = 4 years Required rate of return = k = 17% NPV NCFtt t0 (1 k) n
$300,000 $110,000 $125,000 $140,000 (1.17)1 (1.17)2 (1.17)3 (1.17)4 $475,000 $256,410 $80,356 $78,046 $74,711 $475,000
$14,524 At the required rate of return of 17 percent, Project 1 has an NPV of $14,524. To find the IRR, try higher rates.
Try IRR = 19%. NPV NCFtt t0 (1 k) n
$300,000 $110,000 $125,000 $140,000 (1.19)1 (1.19)2 (1.19)3 (1.19)4 $475,000 $252,101 $77,678 $74,177 $69,814 $475,000
$1,230
Try IRR = 18.8%. NPV NCFtt t0 (1 k) n
$300,000 $110,000 $125,000 $140,000 (1.188)1 (1.188)2 (1.188)3 (1.188)4 $475,000 $252,525 $77,940 $74,552 $70,285 $475,000
$302 0 The IRR of the project is approximately 18.8 percent. Using a financial calculator, we find that the IRR is 18.839 percent.
SM 19-432
Project 2: Cost of project = $500,000 Length of project = n = 4 years Required rate of return = k = 17% NPV NCFtt t0 (1 k) n
$117,500 $181,300 $244,112 $278,955 (1.17)1 (1.17)2 (1.17)3 (1.17)4 $475,000 $100,427 $132,442 $152,416 $148,864 $500,000
$34,150 At the required rate of return of 17 percent, Project 2 has an NPV of $34,150. To find the IRR, try higher rates.
Try IRR = 20%. NPV 0
n
NCFt t t0 (1 IRR) $117,500 $181,300 $244,112 $278,955 0 $500,000 (1.20)1 (1.20)2 (1.20)3 (1.20)4 $475,000 $97,917 $125,903 $141,269 $134,527 $385 0
The IRR of the project is approximately 20 percent. Using a financial calculator, we find that the IRR is 19.965 percent. Both projects can be accepted since their IRRs exceed the cost of capital of 17 percent.
LO 5 Bloomcode: Analysis AASCB: Analytic IMA: Investment Decisions AICPA: Industry/Sector Perspective
10.37 Compute the IRR for each of the following projects: Year
Project 1
Project 2
Project 3
0
$(10,000)
$(10,000)
$(10,000)
SM 19-433
1
4,750
1,650
800
2
3,300
3,890
1,200
3
3,600
5,100
2,875
4
2,100
2,750
3,400
800
6,600
5
Solution: Project 1: Cost of project = $10,000 Length of project = n = 4 years NPV 0 n
NCFt t t0 (1 IRR) $4,750 $3,300 $3,600 $2,100 0 $10,000 (1.16)1 (1.16)2 (1.16)3 (1.16)4 $10,000 $4,095 $2,452 $2,306 $1,160 $13 0
The IRR of the project is approximately 16 percent. Using a financial calculator, we find that the IRR is 16.076 percent.
Project 2: Cost of project = $10,000 Length of project = n = 5 years NPV 0 n
NCFt t t0 (1 IRR) $1,650 $3,890 $5,100 $2,750 $800 0 $10,000 2 3 4 1 (1.137) (1.137) (1.137)5 (1.137) (1.137) $10,000 $1,451 $3,009 $3,470 $1,645 $421 $4 0
The IRR of the project is approximately 13.7 percent. Using a financial calculator, we find that the IRR is 13.685 percent.
SM 19-434
Project 3: Cost of project = $10,000 Length of project = n = 5 years NPV 0 n
NCFt t t0 (1 IRR) $800 $1,200 $2,875 $3,400 $6,600 0 $10,000 2 3 4 1 (1.109) (1.109) (1.109)5 (1.109) (1.109) $10,000 $721 $976 $2,108 $2,248 $3,934 $13 0
The IRR of the project is approximately 10.9 percent. Using a financial calculator, we find that the IRR is 10.862 percent.
LO 5 Bloomcode: Application AASCB: Analytic IMA: Investment Decisions AICPA: Industry/Sector Perspective
9.38
Primus Corp. management is planning to convert an existing warehouse into a new plant that will increase its production capacity by 45 percent. The cost of this project will be $7,125,000. It will result in additional cash flows of $1,875,000 for the next eight years. The discount rate is 12 percent. a.
What is the payback period?
b.
What is the NPV for this project?
c.
What is the IRR?
Solution: a. Year
Project 1
Cumulative CF
0
$(7,125,000)
$(7,125,000)
1
1,875,000
(5,250,000)
2
1,875,000
(3,375,000)
SM 19-435
3
1,875,000
(1,500,000)
4
1,875,000
375,000
5
1,875,000
2,250,000
6
1,875,000
4,125,000
7
1,875,000
6,000,000
8
1,875,000
7,875,000
PB = Years before cost recovery + (Remaining cost to recover/ Cash flow during the year) = 3 + ($1,500,000 / $1,875,000) = 3.80 years
b.
Cost of this project = $7,125,000 Required rate of return = 12% Length of project = n = 8 years NPV NCFtt t0 (1 k) n
1 1 8 $7,125,000 $1,875,000 (1.12) 0.12
$7,125,000 $9,314,325 $2,189,325
c.
To compute the IRR, try rates higher than 12 percent. Try IRR = 20%. NPV 0 NCFt t t0 (1 k ) n
1 1 8 0 $7,125,000 $1,875,000 (1.20) 0.20 $7,125,000 $7,194,675 $69,675
SM 19-436
Try IRR = 20.3%. NPV 0 NCFtt t0 (1 k) n
1 1 (1.203)8 0 $7,125,000 $1,875,000 0.203
$7,125,000 $7,130,832 $5,832 The IRR of the project is approximately 20.3 percent. Using a financial calculator, we find that the IRR is 20.328 percent. LO 2, LO 3, LO 5 Bloomcode: Application AASCB: Analytic IMA: Investment Decisions AICPA: Industry/Sector Perspective
10.39 Quasar Tech Co. management is investing $6 million in new machinery that will produce the next-generation routers. Sales to its customers will amount to $1,750,000 for the next three years and then increase to $2.4 million for three more years. The project is expected to last six years and operating costs, excluding depreciation, will be $898,620 annually. The machinery will be depreciated to a salvage value of $0 over 6 years using the straight-line method. The company’s tax rate is 30 percent, and the cost of capital is 16 percent. a.
What is the payback period?
b.
What is the average accounting return (ARR)?
c.
Calculate the project NPV.
d.
What is the IRR for the project?
Solution: a.
Year
Net Income
Depreciation
Project 1
Cumulative
Cash Flows
CF SM 19-437
0
$(6,000,000)
$(6,000,000)
1
$(104,034)
$1,000,000
895,966
(5,104,034)
2
$(104,034)
$1,000,000
895,966
(4,208,068)
3
$(104,034)
$1,000,000
895,966
(3,312,102)
4
350,966
$1,000,000
1,350,966
(1,961,136)
5
350,966
$1,000,000
1,350,966
(610,170)
6
350,966
$1,000,000
1,350,966
740,796
PB = Years before cost recovery + (Remaining cost to recover/ Cash flow during the year) = 5 + ($610,170 / $1,350,966) = 5.45 years b. Year 1 Sales
Year 2
Year 3
Year 4
Year 5
Year 6
$ 1,750,000 $ 1,750,000 $ 1,750,000 $ 2,400,000 $ 2,400,000 $ 2,400,000
Expenses
898,620
898,620
898,620
898,620
898,620
898,620
Depreciation
1,000,000
1,000,000
1,000,000
1,000,000
1,000,000
1,000,000
EBIT
$(1,48,620)
$(1,48,620)
$(1,48,620)
$ 501,380
$ 501,380
$ 501,380
Taxes (30%)
44,586
44,586
44,586
(150,414)
(150,414)
(150,414)
Net income
$ (104,034)
$ (104,034)
$ (104,034)
$ 350,966
$ 350,966
$ 350,966
Beginning BV
6,000,000
5,000,000
4,000,000
3,000,000
2,000,000
1,000,000
Less:
1,000,000
1,000,000
1,000,000
1,000,000
1,000,000
1,000,000
Depreciation Ending BV
$ 5,000,000 $ 4,000,000 $ 3,000,000 $ 2,000,000 $ 1,000,000
$
Average after-tax income = $123,466 Average book value of equipment = $3,000,000 Accounting rate of return
Average after - tax income
Average book value $123,466 4.1% $3,000,000
SM 19-438
0
c.
Cost of this project = $6,000,000 Required rate of return = k =16% Length of project = n = 6 years NPV NCFtt t0 (1 k) n
1 1 1 1 (1.16)3 (1.16)3 1 $1,350,966 $6,000,000 $895,966 3 0.16 0.16 (1.16) $6,000,000 $2,012,241 $1,943,833
$2,043,927 d.
To compute the IRR, try rates lower than 16 percent. Try IRR = 3%. NPV 0 n
NCFt t t0 (1 IRR)
1 1 1 1 (1.03)3 (1.03)3 1 $1,350,966 0 $6,000,000 $895,966 3 0.03 0.03 (1.03) $6,000,000 $2,534,340 $3,497,084
$31,424 Try IRR = 3.1%.
NPV 0 NCFt t t0 (1 IRR) n
1 1 1 1 (1.031)3 (1.031)3 1 $1,350,966 0 $6,000,000 $895,966 3 0.031 0.031 (1.031) $6,000,000 $2,529,475 $3,480,225
$9,700 The IRR of the project is approximately 3.1 percent. Using a financial calculator, we find that the IRR is 3.145 percent. SM 19-439
LO 2, LO 3, LO 5 Bloomcode: Application AASCB: Analytic IMA: Investment Decisions AICPA: Industry/Sector Perspective
10.40 Management of Skywards, Inc., an airline caterer, is purchasing refrigerated trucks at a total cost of $3.25 million. After-tax net income from this investment is expected to be $750,000 for the next five years. Annual depreciation expense will be $650,000. The cost of capital is 17 percent. a.
What is the discounted payback period?
b.
Compute the ARR.
c.
What is the NPV on this investment?
d.
Calculate the IRR.
Solution: a.
Year 0 1 2 3 4 5
CF $(3,250,000) 1,400,000 1,400,000 1,400,000 1,400,000 1,400,000
Cumulative CF $(3,250,000) (1,850,000) (450,000) 950,000 2,350,000 3,750,000
PVCF $(3,250,000) 1,196,581 1,022,719 874,119 747,110 638,556
Cumulative PVCF $(3,250,000) (2,053,419) (1,030,700) (156,581) 590,529 1,229,085
Discount payback period = Years before recovery + (Remaining cost / Next year’s CF) = 3 + ($156,581 / $747,110) = 3.21 years b. Net income Beginning BV Less:
Year 1 $ 750,000 3,250,000 650,000
Year 2 $ 750,000 2,600,000 650,000
Year 3 $ 750,000 1,950,000 650,000
Year 4 $ 750,000 1,300,000 650,000
Year 5 $ 750,000 650,000 650,000
SM 19-440
Depreciation Ending BV
$2,600,000
$1,950,000
$1,300,000
$ 650,000
$
0
Average after-tax income = $750,000 Average book value of equipment = $1,625,000
Accounting rate of return
Average after - tax income
Average book value $750,000 46.2% $1,625,000 c.
Cost of this project = $3,250,000 Required rate of return = k =17% Length of project = n = 5 years 1 1 5 (1.17) NPV $3,250,000 $1,400,000 t 0.17 t0 (1 k) n
NCFt
$3,250,000 $4,479,085 $1,229,085
d.
To compute the IRR, try rates much higher than 17 percent. Try IRR = 30%. NPV 0 n
NCFt t t0 (1 IRR)
1 1 5 0 $3,250,000 $1,400,000 (1.30) 0.30 $3,250,000 $3,409,798
$159,798
Try IRR = 32.5%.
SM 19-441
NPV 0 n
NCFt t t0 (1 IRR)
1 1 (1.325)5 0 $3,250,000 $1,400,000 0.325
$3,250,000 $3,252,904 $2,904 0
The IRR of the project is approximately 32.5 percent. Using a financial calculator, we find that the IRR is 32.548 percent. LO 2, LO 3, LO 4, LO 5 Bloomcode: Application AASCB: Analytic IMA: Investment Decisions AICPA: Industry/Sector Perspective
10.41 Trident Corp. management is evaluating two independent projects. The costs and expected cash flows are given in the following table. The cost of capital is 10 percent. Year
A
B
0
$(312,500)
$(395,000)
1
121,450
153,552
2
121,450
158,711
3
121,450
166,220
4
121,450
132,000
5
121,450
122,000
a.
Calculate the projects’ NPV.
b.
Calculate the projects’ IRR.
c.
Which project should be chosen based on NPV? Based on IRR? Is there a conflict?
d.
If you are the decision maker for the firm, which project or projects will be accepted? Explain your reasoning. SM 19-442
Solution: a.
Project A: Cost of this project = $312,500 Annual cash flows = $121,450 Required rate of return = k = 10% Length of project = n = 5 years 1 1 5 (1.10) NPV 312,500 $121,450 t (1 k) 0.10 t0 $312,500 460,391 n
NCFt
$147,891
Project B: Cost of this project = $395,000 Required rate of return = k = 10% Length of project = n = 5 years NPV NCFtt t0 (1 k) n
$153,552 $158,711 $166,220 $132,000 $122,000 (1.10)1 (1.10)2 (1.10)3 (1.10)4 (1.10)5 395,000 $139,593 $131,166 $124,884 90,158 75,752 $395,000
$166,553 b.
Project A: Since NPV > 0, to compute the IRR, try rates higher than 10 percent. Try IRR = 27%.
SM 19-443
NPV 0 n
NCFt t t0 (1 IRR)
1 1 0 312,500 $121,450 (1.27)5 0.27 $312,500 313,666 $1,166 Try IRR = 27.2%, NPV 0 n
NCFt t t0 (1 IRR)
1 1 5 0 312,500 $121,450 (1.272) 0.272 $312,500 312,418 $82 0 The IRR of Project A is approximately 27.2 percent. Using a financial calculator, we find that the IRR is 27.187 percent. Project B: Since NPV > 0, to compute the IRR, try rates higher than 10 percent. Try IRR = 26%. NPV 0 n
NCFt t t0 (1 IRR) $153,552 $158,711 $166,220 $132,000 $122,000 0 $395,000 (1.26)1 (1.26)2 (1.26)3 (1.26)4 (1.26)5 $395,000 $121,867 $99,969 $83,094 $52,371 $38,416 $717
Try IRR = 26.1%. NPV 0 n
NCFt t t0 (1 IRR) $153,552 $158,711 $166,220 $132,000 $122,000 0 $395,000 (1.261)1 (1.261)2 (1.261)3 (1.261)4 (1.261)5 $395,000 $121,770 $99,811 $82,897 $52,205 $38,263 $54 0 SM 19-444
The IRR of Project B is approximately 26.1 percent. Using a financial calculator, we find that the IRR is 26.093 percent.
c.
Since both projects have positive NPVs and they are independent projects, both should be accepted under the NPV decision criteria. Under the IRR decision criteria, since both projects have IRRs greater than the cost of capital, both will be accepted. Thus, there is no conflict between the NPV and IRR decisions.
d.
Based on NPV, both projects will be accepted.
LO 2, LO 5 Bloomcode: Analysis AASCB: Analytic IMA: Investment Decisions AICPA: Industry/Sector Perspective
10.42 Management of Tyler, Inc., is considering switching to a new production technology. The cost of the required equipment will be $4 million. The discount rate is 12 percent. The cash flows that management expects the new technology to generate are as follows. Years
CF
1-2
0
3–5
$845,000
6–9
$1,450,000
a.
Compute the payback and discounted payback periods for the project.
b.
What is the NPV for the project? Should the firm go ahead with the project?
c.
What is the IRR, and what would be the decision based on the IRR?
Solution: a. SM 19-445
Cumulative
Cumulative
Year
Cash Flows
PVCF
CF
PVCF
0
$(4,000,000)
$(4,000,000)
$(4,000,000)
$(4,000,000)
1
--
--
(4,000,000)
(4,000,000)
2
--
--
(4,000,000)
(4,000,000)
3
845,000
601,454
(3,155,000)
(3,398,546)
4
845,000
537,013
(2,310,000)
(2,861,533)
5
845,000
479,476
(1,465,000)
(2,382,057)
6
1,450,000
734,615
(15,000)
(1,647,442)
7
1,450,000
655,906
1,435,000
(991,536)
8
1,450,000
585,631
2,885,000
(405,905)
9
1,450,000
522,885
4,335,000
116,979
Payback period = Years before cost recovery +
Remaining cost to recover Cash flow during the year
$15, 000 $1, 450, 000 6.01 years =6
Discounted Payback period = Years before cost recovery +
Remaining cost to recover Cash flow during the year
$405, 905 $522,885 8.8 years
=8
b.
Cost of this project = $4,000,000 Required rate of return = k = 12% Length of project = n = 9 years
SM 19-446
1 3 (1.12) 1 1 NPV $4,000,000 0 0 $845,000 2 t t0 (1 k) 0.12 (1.12) 1 1 4 $1,450,000 (1.12) 1 5 0.12 (1.12) $4,000,000 0 0 $1,617,943 $2,499,037 n
NCF
t
$116,980 Since NPV > 0, the project should be accepted. c.
Given a positive NPV, to compute the IRR, one should try rates higher than 12 percent.
Try IRR = 12.5%. 1 1 (1.125)3 1 NPV $4,000,000 0 0 $845,000 2 t 0.125 (1.125) t0 (1 k) n
NCFt
1 1 (1.125)4 1 $1,450,000 5 0.125 (1.125)
$4,000,000 0 0 $1,589,915 $2,418,479 $8,394 The IRR is approximately 12.5 percent. Using the financial calculator, we find that the IRR is 12.539 percent. Based on the IRR exceeding the cost of capital of 12 percent, the project should be accepted.
LO 2, LO 3, LO 5 Bloomcode: Application AASCB: Analytic IMA: Investment Decisions AICPA: Industry/Sector Perspective
SM 19-447
10.43
You are analyzing two proposed capital investments
with the following cash flows: Year 0
Project X $(20,000)
Project Y $(20,000)
1
13,000
7,000
2
6,000
7,000
3
6,000
7,000
4
2,000
7,000
The cost of capital for both projects is 10 percent. Calculate the profitability index (PI) for each project. Which project, or projects, should be accepted if you have unlimited funds to invest? Which project should be accepted if they are mutually exclusive?
Solution: The PI calculations are as follows: The NPV is needed first: NPV $20,000 $13,000 $6,000 $6,000 $2,000 $2,650.78 x (1.1)1 (1.1)2 (1.1)3 (1.1)4 NPV $20,000 $7,000 $7,000 $7,000 $7,000 $2,189.06 y (1.1)1 (1.1)2 (1.1)3 (1.1)4 SM 19-448
PIx
NPV + Initial Investment $22, 650.78 1.1325 Initial Investment $20, 000
PIy
NPV + Initial Investment $22,189.06 1.1095 Initial Investment $20, 000
Both methods rank Project X over Project Y. Therefore, both should be accepted if they are independent and sufficient resources are available. If the projects are mutually exclusive, we should choose the project with the higher PI at r = 10%, which in this case is Project X.
LO 6 Bloomcode: Application AASCB: Analytic IMA: Investment Decisions AICPA: Industry/Sector Perspective
CFA Problems 10.44 Given the following cash flows for a capital project, calculate the NPV and IRR. The required rate of return is 8 percent. Year
CASH FLOW
0
1
2
3
4
5
–50,000
15,000
15,000
20,000
10,000
5,000
NPV
IRR
a.
$1,905
10.9%
b.
$1,905
26.0%
c.
$3,379
10.9%
d.
$3,379
26.0%
SOLUTION: SM 19-449
c is correct. NPV 50, 000
15, 000 15, 000 20, 000 10, 000 5, 000 1.08 1.082 1.083 1.084 1.085
NPV = –$50,000 + $13,888.89 + $12,860.08 + $15,876.64 + $7,350.30 + $3,402.92 NPV = –$50,000 + $53,378.83 = $3,378.83 The IRR, found with a financial calculator, is 10.88 percent. Bloomcode: Application AASCB: Analytic IMA: Investment Decisions AICPA: Industry/Sector Perspective
10.45 Given the following cash flows for a capital project, calculate its payback period and discounted payback period. The required rate of return is 8 percent.
Year
CASH FLOW
0
1
2
3
4
5
–50,000
15,000
15,000
20,000
10,000
5,000
The discounted payback period is a.
0.16 year longer than the payback period.
b.
0.80 year longer than the payback period.
c.
1.01 years longer than the payback period.
d.
1.85 years longer than the payback period.
SOLUTION: c is correct.
SM 19-450
YEAR
0
1
2
3
4
5
CASH FLOW
–$50,000
$15,000
$15,000
$20,000
$10,000
$5,000
–50,000
–35,000
–20,000
0
10,000
15,000
–50,000
13,888.89
12,860.08
15,876.64
7,350.30
3,402.92
–
–
–
$36,111.11
$23,251.03
$7,374.38
CUMULATIVE CASH FLOW
DISCOUNTED CASH FLOW
CUMULATIVE DCF
–$50,000
$3,378.8 –$24.09
3
As the table shows, the cumulative cash flow offsets the initial investment in exactly three years. The payback period is 3.00 years. The discounted payback period is between four and five years. The discounted payback period is 4 years plus 24.09/3,402.92 = 0.007 of the fifth year cash flow, or 4.007 = 4.01 years. The discounted payback period is 4.01 – 3.00 = 1.01 years longer than the payback period. LO 3 Bloomcode: Application AASCB: Analytic IMA: Investment Decisions AICPA: Industry/Sector Perspective
10.46 An investment of $100 generates after-tax cash flows of $40 in Year 1, $80 in Year 2, and $120 in Year 3. The required rate of return is 20 percent. The net present value is closest to a.
$42.22
b.
$58.33
c.
$68.52
d.
$98.95
SOLUTION: b is correct. SM 19-451
n NPV NCFt $100 40 80 2 1203 = $58.33 t 1.20 1.20 1.20 t0 (1 k)
LO 2 Bloomcode: Application AASCB: Analytic IMA: Investment Decisions
AICPA: Industry/Sector Perspective
10.47 An investment of $150,000 is expected to generate an after-tax cash flow of $100,000 in one year and another $120,000 in two years. The cost of capital is 10 percent. What is the internal rate of return? a.
28.19 percent
b.
28.39 percent
c.
28.59 percent
d.
28.79 percent
SOLUTION: d is correct. The IRR can be found using a financial calculator or with trial and error. Using trial and error, the total PV is equal to zero if the discount rate is 28.79 percent. YEA
CASH FLOW
R
PRESENT VALUE 28.19%
28.39%
28.59%
28.79%
0
–150,000
–150,000
–150,000
–150,000
–150,000
1
100,000
78,009
77,888
77,767
77,646
2
120,000
73,025
72,798
72,572
72,346
1,034
686
338
–8
Total
A more precise IRR of 28.7855 percent has a total PV closer to zero. LO 5 SM 19-452
Bloomcode: Application AASCB: Analytic IMA: Investment Decisions
AICPA: Industry/Sector Perspective
10.48 An investment requires an outlay of $100 and produces after-tax cash flows of $40 annually for four years. A project enhancement increases the required outlay by $15 and the annual after-tax cash flows by $5. How will the enhancement affect the project’s NPV profile? The vertical intercept of the NPV profile of the project shifts: a.
up and the horizontal intercept shifts left.
b.
up and the horizontal intercept shifts right.
c.
down and the horizontal intercept shifts left.
d.
down and the horizontal intercept shifts right.
SOLUTION: a is correct. The vertical intercept changes from $60 to $65, and the horizontal intercept changes from 21.86 percent to 20.68 percent. LO 2 Bloomcode: Application AASCB: Analytic IMA: Investment Decisions AICPA: Industry/Sector Perspective
Sample Test Problems 10.1
Testco Corporation is considering adding a new product line. The cost of the factory and equipment to produce this product is $1,780,000. Company management expects net cash flows from the sale of this product to be $450,000 in each of the next eight years. If Testco uses a discount rate of 12 percent for projects like this, what is the net present value of this project? What is the internal rate of return? (LO 2 & 5) Solution: Investment = $1,780,000 SM 19-453
Annual net cash flow = NCF = $450,000 Discount rate = k = 12% Length of project = n = 8 years
1 1 8 (1.12) NPV= $1, 780, 000 $450, 000 t (1 k) 0.12 t0 $1, 780, 000 $2, 235, 438 n
NCFt
NPV $455, 437.90
Determine the IRR using trial and error: Since NPV > 0, try IRR > k. Try IRR = 20%
1 8 1 (1.20) NPV 0 $1, 500, 000 $450, 000 t t0 (1 IRR) 0.20 $1, 780, 000 $1, 726, 722 n
NCF
t
$53, 278
Try IRR = 19% n
NCFt NPV 0 $1, 780, 000 $450, 000 t t0 (1 IRR) $1, 780, 000 $1, 779, 465
1
1 (1.19)8 0.19
$535 0 The IRR is approximately 19 percent. Using the financial calculator, we find that the IRR is 18.99 percent. Bloomcode: Application AASCB: Analytic IMA: Investment Decisions AICPA: Industry/Sector Perspective
SM 19-454
10.2
Flowers Unlimited is considering purchasing an additional delivery truck which will have a seven year useful life. The new truck will cost $42,000. Cost savings with this truck are expected to be $12,800 for the first two years, $8,900 for the following two years, and $5,000 for the last 3 years of the truck’s useful life. What is the payback period for this project? What is the discounted payback period for this project with a discount rate of 10 percent? (LO 3) Solution: Discount rate = k = 10% Year 0 1 2 3 4 5 6 7
NCF $(42,000) 12,800 12,800 8,900 8,900 5,000 5,000 5,000
Cumulative NCF $(42,000) (29,200) (16,400) (7,500) 1,400 6,400 11,400 16,400
PV(NCF) $(42,000) 11,636 10,579 6,687 6,079 3,105 2,822 2,566
Cumulative PV(NCF) $(42,000) (30,364) (19,785) (13,098) (7,019) (3,914) (1,092) 1,474
PB = Years before cost recovery + (Remaining cost to recover/ Cash flow during the year) = 3 + ($7,500 / $8,900) = 3.84 years Discounted PB = Years before cost recovery + (Remaining cost to recover/ PV Cash flow during the year) = 6 + ($1,092 / $2,566) = 6.43 years Bloomcode: Application AASCB: Analytic IMA: Investment Decisions AICPA: Industry/Sector Perspective
10.3
What is the average accounting rate of return (ARR) on a piece of equipment that will cost $1.2 million and that will result in pretax cost savings of $380,000 for the first three years and then $280,000 for the following three years? Assume that the machinery will be depreciated to a salvage value of 0 over 6 years using the straight-line method and the company’s tax rate is 32 percent. If the acceptance decision is based on the project exceeding an ARR of 20 percent, should this machinery be purchased? (LO 4)
Solution: Cost savings Depreciation
Year 1 $ 380,000 200,000
Year 2 $ 380,000 200,000
Year 3 $ 380,000 200,000
Year 4 $ 280,000 200,000
Year 5 $ 280,000 200,000
Year 6 $ 280,000 200,000
SM 19-455
EBIT Taxes (32%) Net income
$ 180,000 57,600 $ 122,400
$ 180,000 57,600 $ 122,400
$ 180,000 57,600 $ 122,400
$ 80,000 25,600 $ 54,400
$ 80,000 25,600 $ 54,400
$ 80,000 25,600 $ 54,400
Beg. book value 1,200,000 Less:Depreciation 200,000 End. book value $ 1,000,000
5,000,000 200,000 $ 800,000
4,000,000 200,000 $ 600,000
3,000,000 200,000 $ 400,000
2,000,000 200,000 $ 200,000
1,000,000 200,000 $ 0
Average after-tax income (years 1 through 9) = $88,400 Average book value of equipment (years 0 through 6) = $600,000 Average after-tax income Average book value $88, 400 14.7% $600, 000 Since the project’s ARR is below the acceptance rate of 20 percent, the machinery should not be purchased. Accounting rate of return
Bloomcode: Analysis AASCB: Analytic IMA: Investment Decisions AICPA: Industry/Sector Perspective
10.4
What do we know about that project’s IRR if we know that it has a positive NPV? (LO 2 & 5) Solution: If a project has a positive NPV, the IRR of that project is greater than the required rate of return. Since the IRR is the discount rate that makes the NPV equal zero, a positive NPV results from the project’s IRR being greater than the required rate of return.
Bloomcode: Comprehension AASCB: Analytic IMA: Investment Decisions AICPA: Industry/Sector Perspective
10.5
West Street Automotive is considering adding state safety inspections to their service offerings. The equipment necessary to perform these inspections will cost $557,000 and will generate cash flows of $195,000 over each of the next five years. If the cost of capital is 14 SM 19-456
percent, what is the MIRR on this project? (LO 5) Solution: Investment (cost) = $557,000 Annual net cash flow = NCFt = $195,000 Length of project = n = 5 years Cost of capital = k = 14% TV = NCF (1 k)n1 NCF (1 k)n2 1
NCF (1 k)nn
2
n
$195, 000(1.14) $195, 000 (1.14) $195, 000 (1.14)2 $195, 0001.14 $195, 000 $329, 347 $288, 901 $253, 422 $222, 300 $195, 000 $1, 288, 970 4
3
Solve for the MIRR: TV (1 MIRR)n $1, 288, 970 $557, 000 (1 MIRR)5 $1, 288, 970 (1 MIRR) 5 2.3141 $557, 000 (1 MIRR) (2.3141)1/5 1.1827 PVCost
MIRR 0.1827 18.3% Bloomcode: Application AASCB: Analytic IMA: Investment Decisions AICPA: Industry/Sector Perspective
10.6
You are chairperson of the investment committee at your fi rm. Five projects have been submitted to your committee for approval this month. The investment required and the project profitability index for each of these projects are presented in the following table: (LO 6)
SM 19-457
Project
Investment
PI
A B C D E
$20,000 50,000 70,000 10,000 80,000
2.500 2.000 1.750 1.000 0.800
If you have $500,000 available for investments, which of these projects would you approve? Assume that you do not have to worry about having enough resources for future investments when making this decision.
Solution: Definitely accept projects A, B, and C. They all have a positive NPV. Project D just returns the opportunity cost of capital, so you would be indifferent with regards to accepting this project. Do not accept project E; it has a negative NPV. Bloomcode: Analysis AASCB: Analytic IMA: Investment Decisions AICPA: Industry/Sector Perspective
Chapter 11 Cash Flows and Capital Budgeting
Before You Go On Questions and Answers
Section 11.1 1. Why do we care about incremental cash flows at the firm level when we evaluate a project?
SM 19-458
We care about incremental cash flows at the firm level because they reflect the impact of the project on the total cash flows that the firm produces. This is what the stockholders care about. The difference between the present value of the expected cash flows from the firm with the project and the present value of the expected cash flows from the firm without the project is precisely what the NPV of a project is. Our NPV estimate will be incorrect if we do not account for all of the incremental cash flows at the firm level.
2. Why is D&A first subtracted and then added back in FCF calculations?
By subtracting D&A, calculating the tax obligation, and then adding back D&A, we are accounting for the fact that D&A is a noncash charge that reduces the firm’s tax obligation by the product of D&A and the tax rate (D&A x t). If we did not do this, we would overstate the tax obligation and understate FCF.
3. What types of investments should be included in FCF calculations?
All investments directly associated with the project should be included in FCF calculations. These can include both investments in tangible and intangible assets. They can also include investments in additions to working capital, such as for the credit a firm extends to its customers and inventories.
Section 11.2 1. What are the five general rules for calculating FCF?
SM 19-459
(1) Include cash flows and only cash flows in your calculations. (2) Include the impact of the project on cash flows from other product lines. (3) Include all opportunity costs. (4) Forget sunk costs. (5) Include only after-tax cash flows in the cash flow calculations.
2. What is the difference between nominal and real dollars? Why is it important not to mix them in an NPV analysis?
When most people talk about dollar amounts, they are referring to nominal dollars. Nominal dollars do not take into account changes in purchasing power. Real dollars are dollar amounts that are adjusted for changes in purchasing power. For example, 100 real dollars have the same purchasing power whether they are received today or at some future date. It is important not to mix nominal and real dollars in an NPV analysis because the discount rate is either a nominal rate, which is used to discount nominal dollars, or a real rate, which is used to discount real dollars. Since the discount rate must be either a nominal rate or a real rate, if real and nominal dollars are mixed in an NPV analysis, the NPV will be calculated incorrectly.
3. What is a progressive tax system? What is the difference between a firm’s marginal and average tax rates?
SM 19-460
A progressive tax system is one in which the marginal tax rate at low levels of income is lower than the marginal tax rate at high levels of income. A firm’s marginal tax rate is the rate that it pays on the last dollar earned while the average tax rate is the average rate paid on the firm’s total earnings (tax paid divided by taxable income).
4. How can FCF in the terminal year of a project’s life differ from FCF in the other years?
FCF in the terminal year can differ from FCF in other years in several ways. The terminal year cash flows can include cash flows from the asset sales, including the actual proceeds from the sales themselves and taxes due or received if there is a gain or loss on the sale. Terminal year cash flows can also include cash flows associated with recovery of working capital. 5. Why is it important to understand that cash flow forecasts in an NPV analysis are expected values?
It is important to recognize that we are forecasting expected cash flows in an NPV analysis because uncertainties regarding project cash flows that are unique to the project should be reflected in the cash flow forecasts.
Section 11.3 1. What is the difference between variable and fixed costs, and what are examples of each? Variable costs vary directly with unit sales. Fixed costs do not vary with unit sales. For an example of each, see the video game player scenario on page 379. Variable costs are those
SM 19-461
associated with purchasing the components for the player, the labor required, and sales and marketing. These costs will vary according to the number of units produced. Fixed costs are those associated with assembly space, and administrative expenses.
2. How are working capital items forecast? Why are accounts receivable typically forecast as a percentage of revenue and accounts payable, and inventories as percentages of the cost of good sold? Working capital items are forecast using 1) cash and cash equivalents, 2) accounts receivable, 3) inventories, and 4) accounts payable. Inventories are forecast as a percentage of the cost of goods sold because the COGS represent a measure of the amount of money invested in inventories. Accounts payable are forecast this way because the COGS is a measure of the amount of money actually owed to suppliers.
Section 11.4 1. When can we not simply compare the NPVs of two mutually exclusive projects?
If we expect to replace at least one of the projects at the end of its life, we cannot simply compare the NPVs. Doing so would ignore the subsequent investment(s). You can only directly compare the NPVs of mutually exclusive projects under one condition—that is, if you expect to terminate the project that is chosen (e.g., sell the lawn mower) on or before the end of the life of the shorter-lived project.
2. How do we decide when to harvest an asset?
SM 19-462
We choose the harvest date that maximizes the NPV of the asset. To identify this date, we compare the NPVs expected from harvesting the asset for each of the feasible harvest dates. The best date to harvest the asset is the date that produces the largest NPV, once the NPVs for all of the alternative harvest dates have been discounted to the same point in time.
3. Under what circumstance would you replace an old machine that is still operating with a new one?
You should replace the old machine when the EAC of the new machine is lower than the EAC of the old machine (if revenues are the same for both machines) or when the annualized cash inflow from the replacement is greater.
SM 19-463
Self-Study Problems
11.1
Explain why the announcement of a new investment is usually accompanied by a change in the firm’s stock price.
Solution:
A firm’s investments cause changes in its future after-tax cash flows and stockholders are the residual claimants (owners) of those cash flows. Therefore, the stock price should increase when stockholders expect an investment to have a positive NPV, and decrease when it is expected to have a negative NPV.
11.2
In calculating the NPV of a project, should we use all of the after-tax cash flows associated with the project, or incremental after-tax cash flows from the project? Why?
Solution:
SM 19-464
We should use incremental cash flows of the project. Incremental cash flows reflect the amount by which the firm’s total cash flows will change if the project is adopted. In other words, incremental cash flows represent the net difference in cash revenues, costs, and investment outlays (in net working capital and capital expenditures) at the firm level with and without the project, which is precisely what the stockholders care about.
11.3
You are considering opening another restaurant in the TexasBurgers chain. The new restaurant will have annual revenue of $300,000 and operating expenses of $150,000. The annual depreciation and amortization for the assets used in the restaurant will equal $50,000. An annual capital expenditure of $10,000 will be required to offset wear-andtear on the assets used in the restaurant, but no additions to working capital will be required. The marginal tax rate will be 40 percent. Calculate the incremental annual free cash flow for the project
Solution:
The incremental annual free cash flow is calculated as: SM 19-465
FCF ($300, 000 $150, 000 $50, 000) (1 0.4) $50, 000) $10, 000 $100, 000
11.4 Sunglass Heaven, Inc., is launching a new store in a shopping mall in Houston. The annual revenue of the store depends on the weather conditions in the summer in Houston. The annual revenue will be $240,000 in a sizzling summer, with probability of 0.3; $80,000 in a cool summer, with probability of 0.2; and $150,000 in a normal summer, with probability of 0.5. What is the expected annual revenue of the store?
Solution: The expected annual revenue is: (0.3 × $240,000) + (0.2 × $80,000) + (0.5 × $150,000) = $163,000
11.5
Sprigg Lane Manufacturing, Inc., needs to purchase a new central air-conditioning system for a plant. There are two choices. The first system costs $50,000 and is expected to last 10 years, and the second system costs $72,000 and is expected to last 15 years. Assume
SM 19-466
that the opportunity cost of capital is 10 percent. Which air-conditioning system should Sprigg Lane purchase?
Solution:
The equivalent annual cost for each system is: EAC1 (0.1)($50, 000)
EAC2 (0.1)($72, 000)
(1.1)10 (1.1)10 1 (1.1)15 (1.1)15 1
$8,137.27
$9, 466.11
Therefore Sprigg Lane should purchase the first one.
Discussion Questions
11.1 Do you agree or disagree with the following statement given the discussion in this chapter? We can calculate future cash flows precisely and obtain an exact value for the NPV of an investment.
SM 19-467
The statement is not true. The nature of the real business world includes those risks which cannot be forecasted or foreseen such as changes in nation’s economic policy, fluctuation in inflation and interest rates and likes. Given this, it is almost certain that the cash flows generated by a project will differ from the forecasts used to decide whether to proceed with the project. However, techniques discussed in this chapter provide an important and useful framework that helps minimize errors and ensures that forecasts are internally consistent. LO: 1 Level: Basic Bloomcode: Comprehension AASCB: Analytic IMA: Investment Decisions AICPA: Industry/Sector Perspective
SM 19-468
11.2 What are the differences between cash flows used in capital budgeting calculations and past accounting earnings?
Cash flows used in capital budgeting calculations are forward looking; they are incremental after-tax cash flows based on forecast. Accounting earnings are backward looking; they represent a record of past performance and may not accurately reflect cash flows. LO: 1 Level: Basic Bloomcode: Comprehension AASCB: Analytic IMA: Investment Decisions AICPA: Industry/Sector Perspective
11.3 Suppose that FRA Corporation already has divisions in both Dallas and Houston. FRA is now considering setting up a third division in Austin. This expansion will SM 19-469
require that one senior manager from Dallas and one from Houston relocate to Austin. Ignore relocation expenses. Is their annual compensation relevant to the decision to expand?
The annual compensations of existing senior managers are not incremental to the new investment and therefore are not relevant for capital budgeting analysis. This is consistent with our Rule 1 for incremental cash flow calculations: Include cash flows and only cash flows; do not include allocated costs unless they reflect cash flows. LO: 1, 2 Level: Basic Bloomcode: Comprehension AASCB: Analytic IMA: Investment Decisions AICPA: Industry/Sector Perspective
SM 19-470
11.4 MusicHeaven,Inc., is a producer of media players which currently have either 20 gigabytes or 30 gigabytes of storage. Now the company is considering launching a new production line making mini media players with 5 gigabytes of storage. Analysts forecast that MusicHeaven will be able to sell 1 million such mini media players if the investment is made. In making the investment decision, discuss what the company should consider other than the sales of the mini media players.
The company’s launch of the new mini media players may reduce its current sales of media players of bigger storage. This impact has to be considered. This is consistent with our Rule 2 for incremental cash flow calculations: Include the impact of the project on cash flows from other product lines. SM 19-471
LO: 1, 2 Level: Basic Bloomcode: Application AASCB: Analytic IMA: Investment Decisions AICPA: Industry/Sector Perspective
11.5 QualityLiving Trust is a real estate investment company that builds and remodels apartment buildings in northern California. It is currently considering remodeling a few idle buildings that it owns in San Jose into luxury apartment buildings. The company bought those buildings eight months ago. How should the market value of the buildings be treated in evaluating this project?
Although the buildings are not currently in use, the company can sell them at their market value rather than remodel them into apartments. Therefore, the market value SM 19-472
of the buildings is the opportunity cost of the project and should be considered as cash outflow in the investment decision. This is consistent with our Rule 3 for incremental cash flow calculations: Include all opportunity costs. LO: 1, 2 Level: Basic Bloomcode: Comprehension AASCB: Analytic IMA: Investment Decisions AICPA: Industry/Sector Perspective
11.6 High-End Fashions, Inc., bought a production line of ankle-length skirts last year at a cost of $500,000. This year, however, miniskirts are in and ankle-length skirts are completely out of fashion. High-End has the option to rebuild the production line and use it to produce miniskirts with an annual operating cost of $300,000 and expected revenue of $700,000. How should the
SM 19-473
company treat the $500,000 cost of the old production line in evaluating the rebuilding plan?
The cost of the old production line occurred in the past. It cannot be changed whether or not the company rebuilds it into the miniskirt production line. Therefore, High-End should not consider the cost of $500,000. This is consistent with our Rule 4 for incremental cash flow calculations: Forget sunk costs. LO: 1, 2 Level: Basic Bloomcode: Application AASCB: Analytic IMA: Investment Decisions AICPA: Industry/Sector Perspective
11.7 How is the MACRS depreciation method under IRS rules different from the straight-line depreciation allowed under GAAP rules? What is the implication on SM 19-474
incremental after-tax free cash flows from firms’ investments?
GAAP allows the straight-line depreciation method. In contrast, an ―accelerated‖ method of depreciation, Modified Accelerated Cost Recovery System (MACRS), has been used for U.S. federal tax calculations. The advantage of MACRS, relative to straight-line depreciation, is that it enables a firm to deduct depreciation changes sooner, thereby realizing the tax saving sooner and increasing the present value of the tax savings. LO: 2 Level: Basic Bloomcode: Comprehension AASCB: Analytic IMA: Investment Decisions
AICPA: Industry/Sector Perspective
11.8
Explain the difference between marginal and average tax rates, and identify which of these rates is used in capital budgeting and why? SM 19-475
The marginal tax rate is the rate paid on the next dollar earned. The average tax rate is the dollar value of total taxes paid divided by total income. The marginal tax rate is the appropriate rate to use in capital budgeting analysis because this is the tax rate that will be paid on the incremental income earned by the project. LO: 2 Level: Basic Bloomcode: Analysis AASCB: Analytic IMA: Investment Decisions AICPA: Industry/Sector Perspective
11.9
Under what circumstances will the sale of an asset result in a taxable gain? How do you estimate the taxes or tax benefits associated with the sale of an asset? The sale of an asset results in a taxable gain when the selling price of the asset exceeds its book value. Tax on the sale of an asset = (Selling price of asset – Book value of asset) × Tax rate (t) LO: 2 Level: Basic Bloomcode: Comprehension AASCB: Analytic IMA: Investment Decisions AICPA: Industry/Sector Perspective
11.10
When two mutually exclusive projects have
different lives, how can an analyst determine which is
SM 19-476
better? What is the underlying assumption in this method?
When we choose from mutually exclusive projects with different lives, instead of electing the project with higher NPV or lower net present value of costs, we should choose the project with higher Equivalent Annual Revenue or lower Equivalent Annual Cost. The underlying assumption is that we will continue to operate with the same equivalent annual revenue or equivalent annual cost in the future. LO: 4 Level: Basic Bloomcode: Comprehension AASCB: Analytic IMA: Investment Decisions AICPA: Industry/Sector Perspective
SM 19-477
11.11
What is the opportunity cost of using an existing
asset? Give an example of the opportunity cost of using the excess capacity of a machine.
The opportunity cost of using an existing asset in a project is the present value of the change in the firm’s cash flows that is attributed to the fact that this asset is being used in the project. For example, by using the excess capacity of a machine, you may accelerate the wear-and-tear of the machine and hence will need to replace it sooner. The present value of the added annualized costs is the opportunity cost of using the excess capacity. LO: 4 Level: Basic Bloomcode: Comprehension AASCB: Analytic IMA: Investment Decisions AICPA: Industry/Sector Perspective
SM 19-478
11.12
You are providing financial advice to a shrimp
farmer who will be harvesting his last crop of farmraised shrimp. His current shrimp crop is very young and will, therefore, grow and become more valuable as their weight increases. Describe how you would determine the appropriate time to harvest the entire crop of shrimp.
Assuming that the price of shrimp is directly (and linearly) related to the weight of the shrimp, then the optimal point in time to harvest the shrimp would be where the rate of weight increase is no longer greater than the opportunity cost of capital for the shrimp farmer. Alternatively, the appropriate time is when the value increase of the shrimp is no longer greater than the opportunity cost of capital. LO: 5 SM 19-479
Level: Basic Bloomcode: Application AASCB: Analytic IMA: Investment Decisions AICPA: Industry/Sector Perspective
Questions and Problems
BASIC
11.1
Calculating project cash flows: Why do we use
forecasted incremental after-tax free cash flows instead of forecasted accounting earnings in estimating the NPV of a project?
Solution:
SM 19-480
Accounting earnings can differ from cash flows for a number of reasons, making accounting earnings an unreliable measure of the costs and benefits of a project. For example, ease of manipulating earnings components such as accounts receivable and depreciation may result in distorted estimation of capital budgeting; using forecasted cash flows eliminates such possibilities. In addition, because there is time value of money, cash flows better reflect the actual available funds to be distributed to shareholders at each point in time. LO 1 Bloomcode: Comprehension AASCB: Analytic IMA: Investment Decisions
AICPA: Industry/Sector Perspective
11.2
The FCF calculation: How do we calculate
incremental after-tax free cash flows from forecasted SM 19-481
earnings of a project? What are the common adjustment items?
Solution: We need to adjust for the depreciation and amortization tax shield, capital expenditures, and changes in working capital (including receivables and payables). LO 2 Bloomcode: Comprehension AASCB: Analytic IMA: Investment Decisions
AICPA: Industry/Sector Perspective
11.3
The FCF calculation: How do we adjust for
depreciation when we calculate incremental after-tax free cash flow from EBITDA? What is the intuition for the adjustment?
SM 19-482
Solution: There are two ways to adjust for depreciation: (1) subtract depreciation from EBITDA, multiply it by (1 – tax rate), and then add depreciation back; (2) add the tax shield from depreciation (depreciation multiplied by tax rate) to revenue. These two methods yield the same results. The intuition is that although depreciation itself is not a cash inflow or outflow, increase in depreciation will result in a decrease in taxable income. This saving on tax is treated as cash inflow in calculating incremental after-tax free cash flows. LO 2 Bloomcode: Comprehension AASCB: Analytic IMA: Investment Decisions
AICPA: Industry/Sector Perspective
SM 19-483
11.4 Nominal versus real cash flows: What is the difference between nominal and real cash flows? Which rate of return should we use to discount each type of cash flow?
Solution: Nominal cash flows are cash flows stated as we usually think of them. They represent the actual cash flow that we expect a project to generate in the future, without any adjustment, for changes in purchasing power over time. Real cash flows are cash flows stated so that their purchasing power remains constant. We should use nominal rate of return to discount future nominal cash flows and real rate of return to discount future real cash flows. By doing this, we will get meaningful present values of cash flows and purchasing power. LO 2 SM 19-484
Bloomcode: Analysis AASCB: Analytic IMA: Investment Decisions
AICPA: Industry/Sector Perspective
11.5
Taxes and depreciation: What is the difference between average tax rate and the marginal tax rate? Which one should we use in calculating incremental after-tax cash flows?
Solution: In a progressive tax system, the marginal tax rate is different from the average tax rate. The average tax rate is the total amount of tax divided by total amount of money earned, while the marginal tax rate is the rate paid on the last dollar earned. Since a firm already pays taxes, the appropriate tax rate used for the firm’s new project is the tax rate that the firm will pay on any additional profits that SM 19-485
are earned because the project is adopted. Therefore, we use the marginal tax rate in calculating incremental aftertax cash flows. LO 2 Bloomcode: Analysis AASCB: Analytic IMA: Investment Decisions
AICPA: Industry/Sector Perspective
11.6 Computing terminal-year FCF: Healthy Potions, Inc., a pharmaceutical company, bought a machine at a cost of $2 million five years ago that produces pain-reliever medicine. The machine has been depreciated over the past five years, and the current book value is $800,000. The company decides to sell the machine now at its market price of $1 million. The marginal tax rate is 30 percent. What are the relevant cash flows? How do they change if the market price of the machine is $600,000 instead? SM 19-486
Solution: The relevant cash flows include the sale price of the machine, as well as the tax on the capital gain: $1,000,000 – 0.3 × ($1,000,000 – $800,000) = $940,000 When the market price of the machine is changed to $600,000, the relevant cash flows include the sale price and tax saving on capital loss: $600,000 + 0.3 × ($800,000 – $600,000) = $660,000 LO 2 Bloomcode: Application AASCB: Analytic IMA: Investment Decisions
AICPA: Industry/Sector Perspective
11.7 Cash flows from operations: What are variable costs and fixed costs? What are some examples of each? How are these costs estimated in forecasting operating expenses? SM 19-487
Solution: Variable costs vary directly with unit sales, while fixed costs do not. Variable costs are those associated with purchasing the components for the product, the labor required, and sales and marketing. These costs will vary according to the number of units produced. Fixed costs are those associated with assembly space, and administrative expenses.
LO 3 Bloomcode: Analysis AASCB: Analytic IMA: Business Economics
AICPA: Industry/Sector Perspective
11.8
Cash flows from operations: When forecasting operating expenses, explain the difference between a fixed cost and a variable cost. Solution: While forecasting operating expenses, Fixed costs are based on cost of manufacturing space, administrative expenses required for administration of the project, and so forth. While variable costs vary directly with the number of units sold, they can be forecasted by multiplying the variable cost per unit (that has been estimated to be produced) by the number of units expected to be sold each year. LO 3 Bloomcode: Analysis AASCB: Analytic IMA: Investment Decisions AICPA: Industry/Sector Perspective
11.9
Investment cash flows: Zippy Corporation just purchased computing equipment for $20,000. The equipment will be depreciated using a five-year MACRS depreciation schedule. If the equipment is sold at the end of its fourth year for $12,000, what are the after-tax proceeds from the sale, assuming the marginal tax rate is 35 percent.
SM 19-488
Solution: The 5-year depreciation schedule allows us to depreciate 20 percent of the value of the equipment in year 1, 32 percent in year two, 19.20 percent in year 3, and 11.52 percent in year four after the purchase. The associated depreciation charges in years 1 through 4 in order are $4,000, $6,400, $ 3,840, and $2,304 respectively. Total depreciation at the end of year 4 $16,544, so the book value of the equipment when sold is $3,456. Since the equipment is sold for $12,000 the tax on the sale of the asset is equal to ($12,000-$3,456) × 0.35 = $2,990.40. The total after tax proceeds are $12,000 - $2,990.40 = $9,009.60 LO 2 Bloomcode: Application AASCB: Analytic IMA: Investment Decisions AICPA: Industry/Sector Perspective
11.10
Investment cash flows: Six Twelve, Inc., is
considering opening up a new convenience store in downtown New York City. The expected annual revenue at the new store is $800,000. To estimate the increase in working capital, analysts estimate the ratio of cash and cash-equivalents to revenue to be 0.03 and the ratios of receivables, inventories, and payables to revenue to be 0.05,
SM 19-489
0.10, and 0.04, respectively, in the same industry. What is the expected incremental cash flow related to working capital when the store is opened?
Solution: Cash flow related to working capital in year0 = $800,000 × (0.03 + 0.05 + 0.10 - 0.04) = $112,000 LO 2 Bloomcode: Application AASCB: Analytic IMA: Investment Decisions
AICPA: Industry/Sector Perspective
11.11
Investment cash flows: Keswick Supply Company
wants to set up a division that provides copy and fax services to businesses. Customers will be given 20 days to pay for such services. The annual revenue of the division is estimated to be SM 19-490
$25,000. Assuming that the customers take the full 20 days to pay, what is the incremental cash flow associated with accounts receivable?
Solution: The average accounts receivable balance will be (20days/365days/year) ×100% ×25,000 = 5.48% × 25,000 = $1,370.
Alternatively, the average daily credit sale = $25,000 / 365 = $68.49, and it takes 20 days, on average, to collect the sale.
Therefore, the incremental cash flow related to working capital when the store is opened: 20 × $68.49 = $1,369.86, or about $1,370. SM 19-491
LO 2 Bloomcode: Application AASCB: Analytic IMA: Investment Decisions
AICPA: Industry/Sector Perspective
11.12
Expected cash flows: Define expected cash flows, and
explain why this concept is important in evaluating projects.
Solution: Expected cash flows are probability-weighted averages of the future cash flows generated by a project under alternative scenarios. In the real business world there are a lot of uncertainties. Future cash flows may vary across different states of the world. It is not possible to estimate a unique number of cash flow for all states. We can estimate the expected cash flows across different states and use that SM 19-492
as an estimation of future cash flows. The cash flows that are discounted in an NPV analysis are the expected incremental cash flows the project will produce. LO 2 Bloomcode: Knowledge AASCB: Analytic IMA: Investment Decisions
AICPA: Industry/Sector Perspective
11.13
Projects with different lives: Explain the concept of
equivalent annual cost and how it is used to compare projects with different lives.
Solution: The equivalent annual cost (EAC) is the annualized cost of an investment stated in nominal dollars. In other words, it is the annual payment from an annuity with a life equal to that of a project that has the same NPV as the project. SM 19-493
Since it is a measure of the annual cost or cash inflow from a project, the EAC for one project can be compared directly with the EAC from another project, regardless of the lives of those two projects. LO 4 Bloomcode: Comprehension AASCB: Analytic IMA: Investment Decisions
AICPA: Industry/Sector Perspective
11.14.
Replace an existing asset: Explain how we determine the
optimal time to replace an existing asset with a new one.
Solution: The optimal time to replace an existing asset with a new one is if the benefits of replacing the machine exceed the costs. LO 4 SM 19-494
Bloomcode: Comprehension AASCB: Analytic IMA: Investment Decisions
AICPA: Industry/Sector Perspective
11.15. Projects with different lives: If you had to choose between one project with an expected life of five years and a second project with an expected life of six years, how could you do this without using the equivalent annual cost concept?
Solution: When we choose from mutually exclusive project with different lives, instead of electing the project with higher NPV or lower net present value of costs, we should choose the project with higher Equivalent Annual Revenue or lower Equivalent Annual Cost. The underlying assumption is that we will continue to operate with the same equivalent annual revenue or equivalent annual cost in the future.
LO 4 Bloomcode: Comprehension AASCB: Analytic IMA: Investment Decisions
AICPA: Industry/Sector Perspective
INTERMEDIATE SM 19-495
11.16
Nominal versus real cash flows: You are buying a
sofa. You will pay $200 today and make three consecutive annual payments of $300 in the future. The real rate of return is 10 percent, and the expected inflation rate is 4 percent. What is the actual price of the sofa?
Solution: We can calculate it in two different ways: (1) Use nominal dollars and nominal rate of return: Nominal rate of return = (1 + 10%) × (1 + 4%) - 1 = 14.4% Price = 200 + 300 / (1 + 14.4%) + 300/(1 + 14.4%)2 + 300 / (1 + 14.4%)3 = $891.84
(2) Use real dollars and a real rate of return: SM 19-496
Real annual payments are: 300 / (1 + 4%) = 288.46, 300 / (1 + 4%)2 = 277.37, and 300 / (1 + 4%)3 = 266.70 Price = 200 + 288.46/(1 + 10%) + 277.37/(1 + 10%)2+266.7 / (1+10%)3 =891.84 Note that we get identical results as long as we are consistent in using nominal or real cash flows and corresponding discount rates. LO 2 Bloomcode: Application AASCB: Analytic IMA: Investment Decisions
AICPA: Industry/Sector Perspective
11.17
Nominal versus real cash flows: You are graduating
in two years. You want to invest your current savings of $5,000 in bonds and use the proceeds to purchase a new car when you graduate and start to work. You can invest the SM 19-497
money in either Bond A, a two-year bond with a 3 percent annual interest rate, or Bond B, an inflation-indexed twoyear bond paying 1 percent real interest above the inflation rate (assume this bond makes annual interest payments). The inflation rate over the next two years is expected to be 1.5 percent. Assume that both bonds are default free and have the same market price. Which bond should you invest in?
Solution: The nominal interest rate is 3 percent for bond A, and (1 + 1%) × (1 +1.5%) – 1 = 2.52% for the inflation-indexed bond B. You should invest in bond A. LO 2 Bloomcode: Application AASCB: Analytic IMA: Investment Decisions
AICPA: Industry/Sector Perspective SM 19-498
11.18
Marginal and average tax rates: Given the U.S.
Corporate Tax Rate Schedule in Exhibit 11.6, what was the marginal tax rate and average tax rate of a corporation that had a taxable income of $12 million in 2013? Solution: The marginal tax rate is 35 percent. The total tax payable is 3,400,000 + (12,000,00010,000,000) ×35% = $4,100,000 Therefore the average tax rate = 4,100,000 / 12,000,000 = 34.2% LO 2 Bloomcode: Application AASCB: Analytic IMA: Investment Decisions
AICPA: Industry/Sector Perspective
11.19
Investment cash flows: Healthy Potions, Inc., is
considering investing in a new production line for eye drops. SM 19-499
Other than investing in the equipment, the company needs to increase its cash and cash equivalents by $10,000, increase the level of inventory by $30,000, increase accounts receivable by $25,000, and increase accounts payable by $5,000 at the beginning of the project. Healthy Potions will recover these changes in working capital at the end of the project 10 years later. Assume the appropriate discount rate is 12 percent. What are the present values of the relevant investment cash flows?
Solution: The relevant cash flow related to working capital at the beginning of the project is: $10,000 +$30,000+$25,000 - $5,000 = $60,000 The present value of relevant cash flow related to working capital at the end of the project is: SM 19-500
$60,000 / (1 + 0.12)10 = $19,318.39 LO 1 Bloomcode: Application AASCB: Analytic IMA: Investment Decisions
AICPA: Industry/Sector Perspective
11.20
Cash flows from operations: Given the soaring price
of gasoline, Ford is considering introducing a new production line of gas-electric hybrid sedans. The expected annual unit sales of the hybrid cars is 30,000; the price is $22,000 per car. Variable costs of production are $10,000 per car. The fixed overhead including salary of top executives is $80 million per year. However, the introduction of the hybrid sedan will decrease Ford’s sales of regular sedans by 10,000 cars per year; the regular sedans have a unit price of $20,000, a unit variable cost of SM 19-501
$12,000, and fixed costs of $250,000 per year. Depreciation costs of the production plant are $50,000 per year. The marginal tax rate is 40 percent. What is the incremental annual cash flow from operations?
Solution: Step One: Revenue: $22,000 × 30,000 cars =$660,000,000 Step Two: Op Exp: $10,000 × 30,000 cars = $300,000,000, plus lost net revenue from regular sedans = ($20,000 – $12,000) × 10,000 cars = $80,000,000; total Op Exp = $380,000,000 Step Three: D&A: $50,000 Step Four: Plug information into the text book template as below.
SM 19-502
= = x = + = =
ΔNR ΔOpEx ΔEBITDA ΔD&A ΔEBIT (1-t) ΔNOPAT ΔD&A ΔCFO ΔCapEx ΔAWC ΔFCF
660,000,000 -380,000,000 280,000,000 -50,000 279,950,000 0.60 167,970,000 50,000 168,020,000 0 0 168,020,000
Alternatively, the incremental annual cash flow from operations is: (($22,000-$10,000) ×30,000-($20,000-$12,000) ×10,000) ×(1-0.4) + $50,000 ×0.4 = $168,020,000 Note that the fixed costs are not included in the incremental cash flows calculations, since they exist regardless of the hybrid sedan investment. LO 3 Bloomcode: Application AASCB: Analytic IMA: Investment Decisions
AICPA: Industry/Sector Perspective
SM 19-503
11.21
FCF and NPV for a project: Archer Daniels Midland
Company is considering buying a new farm that it plans to operate for 10 years. The farm will require an initial investment of $12 million. This investment will consist of $2 million for land and $10 million for trucks and other equipment. The land, all trucks, and all other equipment are expected to be sold at the end of 10 years for a price of $5 million, which is $2 million above book value. The farm is expected to produce revenue of $2 million each year, and annual cash flow from operations equals $1.8 million. The marginal tax rate is 35 percent, and the appropriate discount rate is 10 percent. Calculate the NPV of this investment.
Solution: Cash flow of investment in year 0 is: $(12,000,000) Annual cash flows from operations = $1,800,000 SM 19-504
Present value of free cash flows: PV(FCF110 ) Annual CF PV Annuity factor 1- 1 =$1,800,000 (1.10)10 0.10 $11, 060220.79
Book value of asset = $3,000,000 Sale price of asset = $5,000,000 Tax on sale of an asset = (Selling price of asset - Book value of asset) × t = $2,000,000 × 0.35 = $700,000 PV of after-tax salvage value in year 10 is: ($5,000,000 $700,000)
1 $1,657,836.15 (1.10)10
NPV $12, 000, 000 $11, 060, 220.79 1, 657,836.15 $718, 056.94
Since NPV > 0, project should be accepted. LO 1 Bloomcode: Application AASCB: Analytic IMA: Investment Decisions SM 19-505
AICPA: Industry/Sector Perspective
11.22 Projects with different lives: You are trying to choose between purchasing one of two machines for a factory. Machine A costs $15,000 to purchase and has a three-year life. Machine B costs $17,700 to purchase but has a four year life. Regardless of which machine you purchase, it will have to be replaced at the end of its operating life. Which machine should you choose? Assume a marginal tax rate of 35percent and a discount rate of 15percent Solution Since the machines have difference purchase costs and different operating lives, you should choose the machine that has the lowest equivalent annual cost (EAC). (1 k)t EACA kNPVA t (1 k) 1 (1.15)3 (0.15)($15, 000) $6, 569.65 (1.15)3 1 (1.15)4 $6,199.70 EACB (0.15)( $17, 700) (1.15)4 1 You should choose machine B because it has the lower equivalent annual cost. LO 4 Bloomcode: Analysis AASCB: Analytic IMA: Investment Decisions AICPA: Industry/Sector Perspective
11.23
Projects with different lives: You are starting a
family pizza parlor and need to buy a motorcycle for delivery orders. You have two models in mind. Model A costs $9,000 and is expected to run for 6 years; Model B is more expensive, with a price of $14,000, and has an
SM 19-506
expected life of 10 years. The annual maintenance costs are $800 for Model A and $700 for Model B. Assume that the opportunity cost of capital is 10 percent. Which one should you buy? Solution: You need to first calculate the NPV of costs for each of the motorcycles: 1 1 (1.10)6 $12, 484.21 A $9, 000 ($800) 0.10
NPV
1 1 10 (1.10) $18, 301.20 B $14, 000 ($700) 0.10
NPV
Then you need to calculate the EAC of each model: (1 k)t EACA kNPVA t (1 k) 1 (0.10)($12, 484.21)
EACB (0.10)( $18, 301.20)
(1.10)6 (1.10)6 1
$2,866.47
(1.10)10 (1.10)10 1
$2, 978.44
SM 19-507
Since EAC is lower for Model A, you should buy Model A. LO 4 Bloomcode: Analysis AASCB: Analytic IMA: Investment Decisions
AICPA: Industry/Sector Perspective
11.24
When to harvest an asset: Predator LLC, a
leveraged-buyout specialist, recently bought a company and wants to determine the optimal time to sell it. The partner in charge of this investment has estimated the aftertax cash flows from a sale at different times to be as follows: $700,000 if sold one year later; $1,000,000 if sold two years later; $1,200,000 if sold three years later; and $1,300,000 if sold four years later. The opportunity cost of capital is 12 percent. When should Predator sell the company? Why?
SM 19-508
Solution: The NPV of each choice is: NPV1 = $700,000 / (1.12)1 = $625,000 NPV2 = $1,000,000 / (1.12)2 = $797,194 NPV3 = $1,200,000 / (1.12)3 = $854,136 NPV4 = $1,300,000 / (1.12)4 = $826,174 Selling the company in 3 years provides the highest NPV. LO 5 Bloomcode: Application AASCB: Analytic IMA: Investment Decisions
AICPA: Industry/Sector Perspective
11.25
Replace an existing asset: Bell Mountain Vineyards
is considering updating its current manual accounting system with a high-end electronic system. While the new accounting system would save the company money, the cost of the system continues to decline. The Bell SM 19-509
Mountain’s opportunity cost of capital is 10 percent, and the costs and values of investments made at different times in the future are as follows: Year
Cost
Value of Future Savings (at time of purchase)
0
$5,000
$7,000
1
4,500
7,000
2
4,000
7,000
3
3,600
7,000
4
3,300
7,000
5
3,100
7,000
When should Bell Mountain buy the new accounting system?
Solution: The NPV of each choice is: NPV0 = Future savings – Cost = $7,000 - $5,000 = $2,000 NPV1 = $2,500 / (1.1)1 = $2,275 NPV2 = $3,000 / (1.1)2 =$ 2,479
SM 19-510
NPV3 = $3,400 /(1.1)3 =$2,554 NPV4 = $3,700 / (1.1)4 = $2,527 NPV5 = $3,900 / (1.1)5 = $2,422 Therefore the company should purchase the system in year 3. LO 4 Bloomcode: AAnalysis AASCB: Analytic IMA: Investment Decisions
AICPA: Industry/Sector Perspective
11.26
Replace an existing asset: You have a 2000 Nissan
that is expected to run for another three years, but you are considering buying a new Hyundai before the Nissan wears out. You will donate the Nissan to Goodwill when you buy the new car. The annual maintenance cost is $1,500 per year for the Nissan and $200 for the Hyundai. The price of your favorite Hyundai model is $18,000, and it is expected SM 19-511
to run for 15 years. Your opportunity cost of capital is 3 percent. Ignore taxes. When should you buy the new Hyundai?
Solution: NPV of cost of the new car is: 1 1 (1.03)15 $20, 387.59 $18, 000 ($200) Hyundai 0.03
NPV
EAC of the new car is: (1 k )t EACHyundai kNPVt t (1 k ) 1 (0.03)($20, 387.59)
(1.03)15 (1.03)15 1
$1, 707.80
Since the EAC of the new car is $1,707.8 and exceeds that of the Nissan( $1,500), you should drive the 2000 Nissan for three more years and then buy a new Hyundai. LO 4
SM 19-512
Bloomcode: Analysis AASCB: Analytic IMA: Investment Decisions
AICPA: Industry/Sector Perspective
11.27
Replace an existing asset: Assume that you are
considering replacing your old Nissan with a new Hyundai, as in the previous problem. However, the annual maintenance cost of the old Nissan increases as time goes by. It is $1,200 in the first year, $1,500 in the second year, and $1,800 in the third year. When should you replace it with the new Hyundai in this case?
Solution: (1 k )t EACHyundai kNPVt t (1 k ) 1 (0.03)($20, 387.59)
(1.03)15 (1.03)15 1
$1, 707.80
SM 19-513
The EAC of the Hyundai remains at $1,707.80, as calculated above. Comparing this amount with the annual maintenance costs of the Nissan and you will see that in year 2 it is cheaper to drive the Nissan, but in year 3 it is cheaper to drive the Hyundai. Therefore, the optimal time to replace the old car is at the end of year 2. LO 4 Bloomcode: Application AASCB: Analytic IMA: Investment Decisions
AICPA: Industry/Sector Perspective
11.28
When to harvest an existing asset: Anaconda
Manufacturing Company currently own a mine that is known to contain a certain amount of gold. Since Anaconda does not have any gold-mining expertise, the company plans to sell the entire mine and base the selling price on a SM 19-514
fixed multiple of the spot price for gold at the time of the sale. Analysts at Anaconda have forecast the spot price for gold and have determined that the price will increase by 14 percent, 12 percent, 9 percent, and 6 percent during the next one, two, three, and four years, respectively. If Anaconda’s opportunity cost of capital is 10 percent, what is the optimal time for Anaconda to sell the mine?
Solution: The rate of gold price appreciation is greater than the opportunity cost of capital for the next two years and then it drops below the opportunity cost of capital. Therefore, Anaconda should sell the gold at the beginning of the third year (or at the end of the second year). LO 5 Bloomcode: Analysis SM 19-515
AASCB: Analytic IMA: Investment Decisions
AICPA: Industry/Sector Perspective
11.29 Replace an existing asset: You are thinking about delivering pizzas in your spare time. Since you must use your own car to deliver the pizzas, you will wear out your current car one year earlier, which is one year from today, than if you did not take on the delivery job. You estimate that when you purchase a new car, regardless of when that occurs, you will pay $20,000 for the car and it will last you five years. If your opportunity cost of capital is 7 percent, what is the opportunity cost of using your car to deliver pizzas?
Solution: (1 0.07)5 EACNew Car 0.07 $20, 000 $4,877.81 5 (1 0.07) 1
Therefore, the opportunity cost of wearing out your car a year earlier is NPVUsin g your car $4,877.81 / (1.07)1 $4,558, 70
LO 4 Bloomcode: Application AASCB: Analytic IMA: Investment Decisions
AICPA: Industry/Sector Perspective SM 19-516
ADVANCED
11.30 You are the CFO of SlimBody, Inc., a retailer of the exercise machine Slimbody6 and related accessories. Your firm is considering opening up a new store in Los Angeles. The store will have a life of 20 years. It will generate annual sales of 5,000 exercise machines, and the price of each machine is $2,500. The annual sales of accessories will be $600,000, and the operating expenses of running the store, including labor and rent, will amount to 50 percent of the revenues from the exercise machines. The initial investment in the store will equal $30 million and will be fully depreciated on a straight-line basis over the 20-year life of the store. Your firm will need to invest $2 million in additional working capital immediately, and recover it at the end of the investment. Your firm’s marginal tax rate is 30 percent. The opportunity cost of opening up the store is 10 percent. What are the incremental free cash flows from this project at the beginning of the project as well as in years 1-19 and 20? Should you approve it?
Solution:
Step One: Initial outlay = $30,000,000 + $2,000,000 (WC requirement) = $32,000,000 Step Two: ΔNR for years 1- 20: $2,500 × 5,000 machines = $12,500,000 plus $600,000 SM 19-517
= $13,100,000 Step Three: ΔOpExp for years 1- 20: $1,250 × 5,000 machines = $6,250,000 Step Four: ΔD&A for years 1- 20: $30,000,000 / 20 years = $1,500,000 / year Step Five: Plug information into the text book template as below. Step Six: Yr 20 recapture of WC requirements that were funded in year 0. Yrs 1-19 ΔNR Δ
O
Δ
E
Δ
D
&
Δ
E
B
(
1
p
B
-
t
1
E
I
3
x
T
D
A
6
A
I
,
-
,
-
T
5
1
6
8
1
,
0
2
5
5
3
0
5
0
0
0
0
0
0
,
0
0
0
0
0
0
,
0
0
0
0
0
0
5
,
0
0
Yr 20
)
0
Δ
N
O
Δ
D
&
Δ
C
F
Δ
C
a
p
Δ
A
W
Δ
F
C
P
A
3
,
7
A
1
,
5
O
5
,
2
E
T
x
-
3
C
F
-
-
3
4
.
1
3
,
-
6
,
-
5
1
6
8
1
,
0
2
5
5
3
0
5
0
0
5
,
0
0
0
0
0
0
,
0
0
0
0
0
0
,
0
0
0
0
0
7
0
0
.
7
5
,
0
0
0
3
,
7
4
5
,
0
0
0
0
0
,
0
0
0
1
,
5
0
0
,
0
0
0
4
5
,
0
0
0
5
,
2
4
5
,
0
0
0
0
,
0
0
0
,
0
0
0
0
2
,
0
0
0
,
0
0
0
0
2
,
0
0
0
,
0
0
0
2
,
0
0
0
,
0
0
0
0
7
,
2
4
5
,
0
0
0
5
,
2
4
5
,
0
0
0
Therefore the NPV of the project is:
1 1 19 1 (1.10) NPV $32, 000, 000 $5, 245, 000 7, 245, 000 (1.10)20 0.10 $12, 950, 928.97
You should approve the project since it has a positive NPV. Alternative Solution: Incremental cash flows in year 0 is: 0 = -$30,000,000-$2,000,000= -$32,000,000 SM 19-518
Annual incremental cash flows through the life of the investment are: t = ($2,500 ×2,500+$600,000) × (1-0.3)+0.3 ×$1,500,000 = $5,245,000 Additional incremental cash flows at the end of the project are: $2,000,000 Therefore the NPV of the project is: 1 1 (1.10)19 1 NPV $32, 000, 000 $5, 245, 000 7, 245, 000 (1.10)20 0.10 $12, 950, 928.97
You should approve the project since it has a positive NPV. LO 1, 3, 4 Bloomcode: Analysis AASCB: Analytic IMA: Investment Decisions
AICPA: Industry/Sector Perspective 11.31 Merton Shovel Corporation has decided to bid for a contract to supply shovels to the Honduran Army. The Honduran Army intends to buy 1,000 shovels per year for the next three years. To supply these shovels, Merton will have to acquire manufacturing equipment at a cost of $150,000. This equipment will be depreciated on a straight-line basis over its five-year lifetime. At the end of the third year, Merton can sell the equipment for exactly its book value ($60,000). Additional fixed costs will be $36,000 per year, and variable costs will be $3.00 per shovel. An additional investment of $25,000 in net working capital will be required when the project is initiated. This investment will be recovered at the end of the third year. Merton Shovel has a 35 percent marginal tax rate and a 17 percent required rate of return on the project. What is the lowest possible per shove price that Merton can offer for the contract and still create value for its stockholders? Solution Year 0 cash flow = Cap Exp + Add WC = -$150,000 – $25,000 = -$175,000 Year 3: Cap Exp + Add WC = $60,000 + $25,000 = $85,000 SM 19-519
We can’t directly solve for CF Opns since revenue depends on what we are going to charge per shovel. Since CF Opns must be the same in years 1, 2, and 3 of the project this is a three year annuity and we can solve for the total value of CF Opns as: Present value of CF Opns = CF Opns × [(1-1/(1.17)3)/0.17] NPV of the shovel contract = -$175,000 + $85,000/(1.17)3 + OCF [(1-1/(1.17)3)/0.17] Set NPV = 0 and solve for CF Opns: CF Opns = $55,181 Now solve CF Opns formula for the price of each shovel (P): CF Opns = [Revenue – OpEx – D&A) × (1 – t)] + D&A $55,181 = [(P × 1000) – ($3.00 × 1000) – $36,000 - $30,000] × (1-0.35) + $30,000 P = $107.74 This is the minimum bid price for a shovel. If Merton Shovel charges less than this price per shovel, the NPV of the contract will be less than zero. LO 1,3 Bloomcode: Application AASCB: Analytic IMA: Investment Decisions AICPA: Industry/Sector Perspective
11.32 Rocky Mountain Lumber, Inc., is considering purchasing a new wood saw that costs $50,000. The saw will generate revenues of $100,000 per year for five years. The cost of materials and labor needed to generate these revenues will total $60,000 per year, and other cash expenses will be $10,000 per year. The machine is expected to sell for $1,000 at the end of its five-year life and will be depreciated on a straight-line basis over five years to zero. Rocky Mountain’s tax rate is 34 percent, and its opportunity cost of capital is 10 percent. Should the company purchase the saw? Explain why or why not?
Solution:
Step One: Initial outlay = $50,000 Step Two: ΔNR for years 1- 5: $100,000 Step Three: ΔOpExp for years 1- 5: $60,000 + $10,000 = $70,000
SM 19-520
Step Four: ΔD&A for years 1- 5: $50,000 / 5 years = $10,000 / year Step Five: Plug into the text book template as below. Step Six: Yr 5: Capital recovery = $1,000 – (0.34 × $1,000 gain on sale) = $660. Yrs 1-4 ΔNR Δ
O
Δ
E
B
Δ
D
&
Δ
E
B
(
1
p
-
t
1
E
I
x
T
D
A
0
7
3
A
I
0
-
-
T
,
0
0
1
2
Yr 5
,
0
0
)
,
0
0
0
0
0
0
-
0
0
0
3
0
0
0
-
0
0
0
2
6
6
0
Δ
N
O
Δ
D
&
Δ
C
F
O
Δ
C
a
p
Δ
A
W
Δ
F
C
P
A
T
A
E
x
-
5
0
,
0
0
C
F
-
5
0
,
0
0
.
0
0
7
,
0
0
1
,
0
0
,
0
0
0
0
0
0
0
0
0
0
0
0
0
0
0
0
.
6
6
1
3
,
2
0
0
1
3
,
2
0
0
1
0
,
0
0
0
1
0
,
0
0
0
2
3
,
2
0
0
2
3
,
2
0
0
0
0
0
1
2
3
,
2
0
0
0
6
0
6
0
0
2
3
,
8
6
0
Therefore, NPV of investment is: $23, 200 $23, 200 $23, 200 $23, 200 $23,860 NPV $50, 000 (1.10) (1.10)2 (1.10)3 (1.10)4 (1.10)5 $38, 356 Since NPV > 0, the company should buy the machine. Alternatively: The annual operating cash flows from year 1 to 5 are: ($100,000-$60,000-$10,000) ×(1-0.34) + 0.34 ×10,000=$23,200 The after-tax terminal value in year 5 is: $1,000 - (0.34)($1,000-$0) = $660 Therefore, NPV of investment is: $23, 200 $23, 200 $23, 200 $23, 200 $23,860 NPV $50, 000 (1.10) (1.10)2 (1.10)3 (1.10)4 (1.10)5 $38, 356 Therefore the company should buy the machine. LO 1,3,4 Bloomcode: Analysis AASCB: Analytic IMA: Investment Decisions AICPA: Industry/Sector Perspective
SM 19-521
11.33 A beauty product company is developing a new fragrance named Happy Forever. There is a probability of 0.5 that consumers will love Happy Forever, and in this case, annual sales will be 1 million bottles; a probability of 0.4 that consumers will find the smell acceptable and annual sales will be 200,000 bottles; and a probability of 0.1 that consumers will find the smell unpleasant and annual sales will be only 50,000 bottles. The selling price is $38, and the variable cost is $8 per bottle. Fixed production costs will be $1 million per year, and depreciation will be $1.2 million. Assume that the marginal tax rate is 40 percent. What are the expected annual incremental after-tax free cash flows from the new fragrance?
Solution:
Step One: Expected sales units: (0.5) ×1,000,000 + (0.4) ×200,000 + (0.1) ×50,000 = 585,000 units Step Two: ΔNR: 585,000 units × $38 = $22,230,000 Step Three: ΔOpExp: 585,000 units x $8 + $1,000,000 = $5,680,000 Step Four: ΔD&A: $1,200,000 Step Five: Plug into the text book template as below.
= = x = + = =
ΔNR ΔOpEx ΔEBITDA ΔD&A ΔEBIT (1-t) ΔNOPAT ΔD&A ΔCFO ΔCapEx ΔAWC ΔFCF
22,230,000 -5,680,000 16,550,000 -1,200,000 15,350,000 0.60 9,210,000 1,200,000 10,410,000 0 0 10,410,000 SM 19-522
Alternatively, the expected annual incremental cash flows are: (((0.5 ×1,000,000+0.4 ×200,000+0.1 ×50,000) × ($38$8)) -1,000,000) × (1 - 0.4) +1,200,000 ×0.4 = $10,410,000 LO 1,3,4 Bloomcode: Application AASCB: Analytic IMA: Investment Decisions
AICPA: Industry/Sector Perspective 11.34 Great Fit, Inc., is a company that manufactures clothing. The company has a production line that produces women’s tops of regular sizes. The same machine could be used to produce petite sizes as well. However, the remaining life of the machines will be reduced from four years to two years if the petite size production is added. The cost of identical machines with a life of eight years is $2 million. Assume the opportunity cost of capital is 8 percent. What is the opportunity cost of adding petite sizes?
Solution:
The opportunity cost is the incremental costs of the machine in year 3 and year 4 if petite sizes are in production. The EAC of the machine is:
SM 19-523
(1 k)t EACA kNPVA t (1 k) 1 (0.08)($2, 000, 000)
(1.08)8 (1.08)8 1
$348, 029.52
The present value of such cost in year 3 and year 4 is: $348, 029.52 $348, 029.52 (1.08)3 (1.08)4 $532, 089.14
NPV
LO 4 Bloomcode: Application AASCB: Analytic IMA: Investment Decisions
AICPA: Industry/Sector Perspective
11.35Biotech Partners LLC has been farming a new strain of radioactive-material-eating bacteria that the electrical utility industry can use to help dispose of its nuclear waste. Two opposing factors affect Biotech’s decision of when to harvest the bacteria. The bacteria are currently growing at a 22 percent annual rate, but due to known competition from other top firms, Biotech analysts estimate that the price for the bacteria will decline according to the schedule below. If the opportunity cost of capital is 10 percent, then when should Biotech harvest the entire bacteria colony at one time? Year
Change in Price Due to Competition (5)
1
5%
2
-2
SM 19-524
3
-8
4
-10
5
-15
6
-25
Solution: Change in revenue: Yr 1
(1.05)(1.22) = 1.2810 or 28.1%
Yr 2
(0.98)(1.22) = 1.1956 or 19.56%
Yr 3
(0.92)(1.22) = 1.1224 or 12.24%
Yr 4
(0.9)(1.22) = 1.0980 or 9.80%
Yr 5
(0.85)(1.22) = 1.037 or 3.70%
Yr 6
(0.75)(1.22) =- 0.9150 or -8.50%
Since the change in revenue is higher than the 10% opportunity cost of capital for the first three years, Biotech should sell its bacteria colony at the beginning of the third year or at the end of the second year. LO 5 Bloomcode: Analysis AASCB: Analytic SM 19-525
IMA: Investment Decisions
AICPA: Industry/Sector Perspective
11.36 ACME Manufacturing is considering replacing an existing production line with a new line that has a greater output capacity and operates with less labor than the existing line. The new line would cost $1 million, have a five-year life, and be depreciated using MACRS over three years. At the end of five years, the new line could be sold as scrap for $200,000 (in Year 5 dollars). Because the new line is more automated, it would require fewer operators, resulting in a savings of $40,000 per year before tax and unadjusted for inflation (in today’s dollars). Additional sales with the new machine are expected to result in additional net cash inflows, before tax, of $60,000 per year (in today’s dollars). If ACME invests in the new line, a one-time investment of $10,000 in additional working capital will be required. The tax rate is 35 percent, the opportunity cost of capital is 10 percent, and the annual rate of inflation is 3 percent. What is the NPV of the new production line?
Solution: t=
0.35
rate =
0.1
NPV of the New Line 0
1
2
3
4
Op Exp)(1-t)
$66,950
$68,959
$71,027
$73,158
plus Tax x Deprec
$116,655
$155,575
$51,835
$25,935
5
(Revenue $75,353
SM 19-526
minus Cap Exp
$1,000,000
$(200,000)
$(70,000)
plus tax on Salvage
minus Add WC
$10,000
Net Cash Flows
$(1,010,000)
$183,605
$224,534
$122,862
$99,093
$215,353
PV of Net Cash Flows
$(1,010,000)
$166,914
$185,565
$92,308
$67,682
$133,717
Net Present Value
$(10,000)
$(363,814)
LO 4 Bloomcode: Application AASCB: Analytic IMA: Investment Decisions
AICPA: Industry/Sector Perspective
11.37 The alternative to investing in the new production line in Problem 11.36 is to overhaul the existing line, which currently has both a book value and a salvage value of $0. It would cost $300,000 to overhaul the existing line, but this expenditure would extend its useful life to five years. The line would have a $0 salvage value at the end of five years. The overhaul outlay would be capitalized and depreciated using MACRS over three years. Should ACME replace or renovate the existing line?
SM 19-527
t=
0.35
rate =
0.1
Renovate Old Line 0
1
2
3
4
$34,997
$46,673
$15,551
$7,781
5
(Revenue Op Ex)(1-t)
plus Tax × Deprec
minus Cap Exp
$300,000
plus tax on Salvage
minus Add WC
Net Cash Flows
$(300,000)
$34,997
$46,673
$15,551
$7,781
$0
PV of Net Cash Flows
$(300,000)
$31,815
$38,572
$11,683
$5,314
$0
Net Present Value
$(212,615)
NPVnew - NVPold
$(151,199)
Renovating the old line is less costly. LO 4 SM 19-528
Bloomcode: Analysis AASCB: Analytic IMA: Investment Decisions
AICPA: Industry/Sector Perspective
CFA Problems 11.38. FITCO is considering the purchase of new equipment. The equipment costs $350,000, and an additional $110,000 is needed to install it. The equipment will be depreciated straight-line to zero over a five-year life. The equipment will generate additional annual revenues of $265,000, and it will have annual cash operating expenses of $83,000. The equipment will be sold for $85,000 after five years. An inventory investment of $73,000 is required during the life of the investment. FITCO is in the 40 percent tax bracket, and its cost of capital is 10 percent. What is the project NPV?
a.
$47,818
b.
$63,658
c.
$80,189
d.
$97,449
Solution:
d is correct. Outlay = FCInv + NWCInv – Sal0 + T × (Sal0 – B0) Outlay = (350,000 + 110,000) + 73,000 – 0 + 0 = $533,000 The installed cost is $350,000 + $110,000 = $460,000, so the annual depreciation is $460,000/5 = $92,000. The annual after-tax operating cash flow for Years 1–5 is
SM 19-529
CF = (S – C – D)(1 – T) + D = (265,000 – 83,000 – 92,000)(1 – 0.40) + 92,000 CF = $146,000
The terminal year after-tax non-operating cash flow in Year 5 is TNOCF = Sal5 + NWCInv – T(Sal5 – B5) = 85,000 + 73,000 – 0.40(85,000 – 0) TNOCF = $124,000 The NPV is 124, 000 NPV 533, 000 146, 000 t 1.10 1.105 = $97,449 t 1 5
LO 4 Bloomcode: Application AASCB: Analytic IMA: Investment Decisions
AICPA: Industry/Sector Perspective
11.39 After estimating a project’s NPV, the analyst is advised that the fixed capital outlay will be revised upward by $100,000. The fixed capital outlay is depreciated straight-line over an eight-year life. The tax rate is 40 percent, and the required rate of return is 10 percent. No changes in cash operating revenues, cash operating expenses, or salvage value are expected. What is the effect on the project NPV?
a.
$100,000 decrease
b.
$73,325 decrease
C.
$59,988 decrease
d.
No change
Solution:
b is correct. The additional annual depreciation is $100,000/8 = $12,500. The SM 19-530
depreciation tax savings is 0.40 ($12,500) = $5,000. The change in project NPV is 100, 000 5, 000t 100, 000 26, 675 $73, 325 t1 (1.10) 8
LO 4 Bloomcode: Application AASCB: Analytic IMA: Investment Decisions
AICPA: Industry/Sector Perspective
11.40. When assembling the cash flows to calculate an NPV or IRR, the project’s after-tax interest expenses should be subtracted from the cash flows for:
a.
The NPV calculation, but not the IRR calculation.
b.
The IRR calculation, but not the NPV calculation.
c.
Both the NPV calculation and the IRR calculation.
d.
Neither the NPV calculation nor the IRR calculation.
Solution: D is correct. Financing costs are not subtracted from the cash flows for either the NPV or the IRR. The effects of financing costs are captured in the discount rate used. LO 1 Bloomcode: Application AASCB: Analytic IMA: Investment Decisions AICPA: Industry/Sector Perspective
Sample Test Problems
SM 19-531
11.1
You purchased 100 shares of stocks of an oil company, Texas Energy, Inc., at $50 per share. The company has 1 million shares outstanding. Ten days later, Texas Energy announced an investment in an oil field in east Texas. The probability of the investment being successful and generating NPV of $10 million is 0.2; the probability of the investment will be a failure and generate a negative NPV of negative $1 million is 0.8. How would you expect the stock price to change upon the company’s announcement of the investment?
Solution: The expected change in the stock price should be equal to the expected NPV of the project divided by the number of shares outstanding.
SM 19-532
The expected NPV of the project is (0.2 × $10,000,000) + (0.8 × –$1,000,000) = $1,200,000 Therefore, the change in Texas Energy’s stock price should equal $1,200,000 / 1,000,000 shares = $1.2 per share when the investment is announced. The total value of your shares should increase by 100 shares × $1.20 per share = $120. Bloomcode: Application AASCB: Analytic IMA: Investment Decisions
AICPA: Industry/Sector Perspective
11.2
A division of Virginia City Highlands Manufacturing is considering purchasing for 1,500,000 a machine that automates the process of inserting electronic components onto computer motherboards. The annual cost of operating the machine will be $50,000, but it will save the company $370,000 in labor costs each year. The machine will have a SM 19-533
useful life of 10 years, and its salvage value in 10 years is estimated to be $300,000. Straight-line depreciation will be used in calculating taxes for this project, and the marginal corporate tax rate is 32 percent. If the appropriate discount rate is 12 percent, what is the NPV of this project?
Solution: The incremental after-tax free cash flow (FCF) in year 0 is: FCF0 = –$1,500,000 The FCF in years 1 through 9 is: Annual Depreciation = ($1,500,000 – $300,000) / 10 = $120,000 per year FCFt = [($370,000 – $50,000) × (1 – 0.32)] + [ 0.32 × $120,000] = $256,000 The FCF in year 10 is:
SM 19-534
FCF10 = $256,000 + $300,000 – [(Salvage Value – Book Value) × 0.032] = $556,000 Therefore, the NPV of the project is: 1 – 1 9 (1.12) 1 $556,000 NPV = –$1,500,000 + $256,000 0.12 10 1.12
= –$1,500,000 + $1,346,032 + $179,017 = $43,049 Virginia City Highlands Manufacturing should approve the project since it has a positive NPV. Bloomcode: Analysis AASCB: Analytic IMA: Investment Decisions
AICPA: Industry/Sector Perspective
11.3
After examining the NPV analysis for a potential project that would increase the firm’s output by 5 percent, an analyst’s manager tells the analyst to increase the initial
SM 19-535
fixed capital outlay in the analysis by $480,000. The initial fixed capital outlay would be fully depreciated on a straight-line basis over a 12-year life, regardless of whether it is increased. If the firm’s average tax rate is 28 percent, its marginal tax rate is 35 percent, and the required rate of return is 10 percent, what is the effect of the adjustment on the project NPV?
Solution: The additional annual depreciation is $480,000 / 12 = $40,000. For a project such as this we would use the marginal tax rate. Therefore, the annual depreciation tax savings is 0.35 × $40,000 = $14,000. The change in the overall NPV of the project is:
SM 19-536
1 12 1 (1.10) Change in NPV = – $480,000 $14,000 0.10
= –$480,000 + $95,392 = –$384,608 The NPV will be reduced by $384,608 Bloomcode: Application AASCB: Analytic IMA: Investment Decisions AICPA: Industry/Sector Perspective
11.4 Which of the following are relevant cash flows in the evaluation of a proposal to produce a new product? a. Decrease in the cash flows of a substitute product. b. Alternative of leasing an existing building that will be used for manufacturing this product. c. The cost of a new machine required to produce this product. d. Salvage value of the new machine at the end of its useful life.
SM 19-537
e. Increase in net working capital at the beginning of the project’s life. f. Cost to develop a product prototype last year.
Solution: a, b, c, d, and e are all relevant cash flows. f is a sunk cost and is therefore not relevant. Bloomcode: Application AASCB: Analytic IMA: Investment Decisions
AICPA: Industry/Sector Perspective
11.5 Managers of Central Embroidery have decided to purchase a new monogram machine and are considering two alternative machines. The first machine costs $100,000 and is expected to last five years. The second machine costs $160,000 and is expected to last eight years. Assume that the opportunity cost of
SM 19-538
capital is 8 percent. Which machine should Central Embroidery purchase?
Solution: The equivalent annual cost for each system is:
EACMachine 1 = 0.08 × $100,000 × (1.08) 5
= $25,045.65 5 (1.08) – 1
(1.08) 8 EACMachine 2 = 0.08 ×$160,000 × = $27,842.37 8 (1.08) – 1
Central Embroidery should purchase the first machine because it has a lower EAC. Bloomcode: Analysis AASCB: Analytic IMA: Investment Decisions
AICPA: Industry/Sector Perspective
11.6 You have inherited an apple orchard and want to sell it in the next four years. An expert in apple orchard valuation has estimated the after-tax cash flow you would receive if you sold SM 19-539
at the end of each of the next four years as follows: $1,000,000 if you sell in one year; $1,300,000 if you sell in two years; $1,500,000 if you sell in three years; and $1,600,000 if you sell in four years. Your opportunity cost of capital is 10 percent. When should you sell the orchard?
Solution: The NPV in today’s dollars for each alternative is: NPV1 = $1,000,000 / (1.10)1 = $909,091 NPV2 = $1,300,000 / (1.10)2 = $1,074,380 NPV3 = $1,500,000 / (1.10)3 = $1,126,972 NPV4 = $1,600,000 / (1.10)4 = $1,092,822 You should sell the orchard in 3 years because the present value of the NPV is highest in year three. After year three, the value of the orchard is expected to increase at a rate that is less than your opportunity cost of capital. Therefore it is better for you to sell the orchard after three years and invest the proceeds elsewhere. Bloomcode: Analysis AASCB: Analytic IMA: Investment Decisions AICPA: Industry/Sector Perspective
SM 19-540
Chapter 12 Evaluating Project Economics
Before You Go On Questions and Answers
Section 12.1 1.
Why do analysts care about how sensitive EBITDA and EBIT are to changes in
revenue?
Comparing the sensitivity of EBITDA to changes in revenue can help you better understand risks and returns associated with alternative options. For example, if we assume that the sensitivity of EBITDA to changes in revenue is higher for one alternative than for the other. This means that EBITDA for the more sensitive alternative will decline more when revenue is lower than expected. A larger decline in EBITDA reduces the value of the project more and has a greater impact on the amount of cash the firm has available to fund other positive NPV projects. Conversely, EBITDA will increase more when revenue is greater than expected if the level of sensitivity is higher.
2.
How is the proportion of fixed costs in a project’s cost structure related to the sensitivity of EBITDA and EBIT to changes in revenue?
SM 19-541
The greater the proportion of fixed costs, the more sensitive EBITDA and EBIT will be to changes in revenue.
Section 12.2 1.
How does operating leverage change when there is an increase in the proportion of a project’s costs that are fixed?
An increase in the proportion of a project’s costs that are fixed increases the operating leverage of the project.
2.
What do the degree of pretax cash flow operating leverage (Cash Flow DOL) and the degree of accounting operating leverage (Accounting DOL) tell us?
Cash Flow DOL provides us with a measure of how sensitive pretax operating cash flows are to changes in revenue. Cash Flow DOL changes with the level of revenue. Accounting DOL is a measure of how sensitive accounting operating profits (EBIT) are to changes in revenue. Accounting DOL focuses on EBIT, whereas Cash Flow DOL focuses on EBITDA.
Section 12.3
1.
How is the per-unit contribution related to the accounting operating profit breakeven point?
SM 19-542
The per unit contribution is how much is left from the sale of a single unit after paying the variable costs associated with that unit. This is the amount that is available to help cover FC and D&A for the project. When we calculate the accounting operating profit break-even point, we divide the sum of FC and D&A by the per unit contribution to determine how many units must be sold to cover FC and D&A.
2.
What is the difference between the pretax operating cash flow break-even point and the accounting operating profit break-even point?
The pretax operating cash flow break-even point is the number of units that must be sold in a particular year for cash inflows to exactly equal cash outflows. The accounting operating profit break-even point is the number of units that must be sold in a particular year for the project to have operating profits of $0—in other words, to break even on an accounting operating profit basis.
Section 12.5 1.
How is a sensitivity analysis used in project analysis?
Sensitivity analysis is used to examine the sensitivity of a project’s NPV to changes in an individual assumption.
2.
How does a scenario analysis differ from a sensitivity analysis?
SM 19-543
Scenario analysis recognizes that variables typically do not change one at a time. A change in economic or market conditions will usually cause several assumptions to change. Scenario analysis recognizes this by examining a project under alternative scenarios in which each assumption can change under each scenario.
3.
What is a simulation analysis, and what can it tell us?
Simulation analysis is like scenario analysis which enables the analyst to evaluate the effects of different scenarios. A key difference is that simulation analysis uses computers to enable the analyst to examine a large number of scenarios in a short period of time.
Self-Study Problems
12.1 The Yellow Shelf Company sells all of its shelves for $100 per shelf, and incurs $50 in variable costs to produce each. If the fixed costs for the firm are $2,000,000 per year, what will the EBIT for the firm be if it produces and sells 45,000 shelves next
SM 19-544
year? Assume that depreciation and amortization is included in the fixed costs.
Solution:
12.2
Revenue $100 per unit × 45,000 units =
$4,500,000
VC
2,250,000
$50 per unit × 45,000 units =
FC + D&A
2,000,000
EBIT
$ 250,000
Hydrogen Batteries sells its specialty automobile
batteries for $85 each, while its current variable cost per unit is $65. Total fixed costs (including depreciation and amortization expense) are $150,000 per year. Management expects to sell 10,000 batteries next year, but is concerned that variable cost will increase next year due to material cost increases. What is
SM 19-545
the maximum variable cost per unit increase that will keep the EBIT from becoming negative?
Solution: The forecasted EBIT for Hydrogen Batteries is: Revenue $85 per unit × 10,000 units = $850,000 VC
$65 per unit × 10,000 units = 650,000
FC + D&A
150,000
EBIT
$ 50,000
Therefore, total variable cost may increase by $50,000, which means that if the firm produces and sells 10,000 batteries, then the variable cost per unit may increase by $5 ($50,000 / 10,000 units = $5 per unit).
SM 19-546
12.3 The Vinyl CD Co. is going to take on a project that is expected to increase its EBIT by $90,000, its fixed cost cash expenditures by $100,000, and its depreciation and amortization by $80,000 next year. If the project yields an additional 10 percent in revenue, what percentage increase in the project’s EBIT will result from the additional revenue?
Solution: FC D & A EBIT $100, 000 $80, 000 1 $90, 000 3
Accounting DOL 1+
Therefore, a 10 percent additional increase in revenue should result in approximately a 30 percent increase in EBIT.
SM 19-547
12.4 You are considering investing in a business that has monthly fixed costs of $5,500 and sells a single product that costs $35 per unit to make. This product sells for $90 per unit. What is the annual pretax operating cash flow break-even point for this business?
Solution: You can solve for the monthly pretax operating cash flow break-even point using Equation 12.4: EBITDA Break-Even =
FC $5, 500 100 units per month Price Unit VC $90 $35
Therefore, the annual EBITDA break-even point is 100 12 = 1,200 units. 12.5 Belt Bottoms, Inc. is considering a five-year project with an initial investment of $20,000. What annual free cash flow (FCF)
SM 19-548
would be required for this project to have an NPV of $0 if the opportunity cost capital is 11 percent? Solution: If the FCF is equal in each of the five years, then we can solve for FCF using the present value of an ordinary annuity formula: $20,000 = 𝐹𝐶𝐹𝑡 × (1 − .11
1 (1.11)5
) → FCF = $5,411.41
Discussion Questions
12.1 You are involved in the planning process for a firm that is expected to have a large increase in sales next year. Which type of firm would benefit the most from that sales increase: a firm with low fixed costs and high variable costs or a firm with high fixed costs and low variable costs?
SM 19-549
Solution: Under the circumstances described, the firm with the high fixed costs would incur lower total future costs associated with the increased sales than the firm with the low fixed costs due to the higher variable cost per unit of sales. Therefore, the firm with the high fixed costs structure would benefit the most. LO: 1 Level: Basic Bloomcode: Comprehension AASCB: Analytic IMA: Business Economics
AICPA: Industry/Sector Perspective
12.2 You own a firm with a single new product that is about to be introduced to the public for the first time. Your marketing analysis suggests that the annual demand for this product could be anywhere between 500,000 units and 5,000,000 units. Given
SM 19-550
such a wide range, discuss the safest cost structure alternative for your firm.
Solution: Since there is a great deal of variability concerning the demand for the product, then the safest alternative would be to create a cost structure that limits the variability of the firm’s EBIT. This means that you would create a cost structure that is composed of high unit variable costs with low fixed costs. Although this would not enable the firm to maximize earnings if the 5,000,000 unit forecast occurs, it limits the downside profitability for the firm in the event that the 500,000 unit forecast occurs. LO: 1 Level: Intermediate Bloomcode: Application AASCB: Analytic IMA: Business Economics AICPA: Industry/Sector Perspective SM 19-551
12.3
Discuss the interpretation of the degree of accounting
operating leverage and degree of pretax cash flow operating leverage.
Solution: While the degree of accounting operating leverage is defined as: Accounting DOL = 1 + (FC + D&A) / (EBIT) it is used to interpret the percentage change in EBIT that will be driven by a given percentage change in net revenue. Similarly, the degree of pretax cash flow operating leverage is defined as: Cash Flow DOL = 1 + FC / (EBIT + D&A)
SM 19-552
but it is used to interpret the percentage change in EBITDA (or pretax operating cash flow) that will be driven by a given percentage change in net revenue. LO: 2 Level: Basic Bloomcode: Comprehension AASCB: Analytic IMA: Business Economics AICPA: Industry/Sector Perspective
12.4
Explain how EBITDA differs from incremental aftertax free cash flows (FCF) and discuss the types of businesses for which this difference would be especially small or large.
Solution: SM 19-553
Depreciation and amortization, taxes, capital expenditures, and working capital are not reflected in EBITDA. If any of these is not equal to zero, then EBITDA is likely to differ from FCF, which is equal to EBIT(1-T) + depreciation—increases in working capital—capital expenditures. The type of businesses that require large capital expenditures (and therefore have large depreciation expenses per year) such as heavy manufacturing, are likely to have substantial differences between EBITDA and FCF. Conversely, smaller firms that have smaller capital expenditures, such as firms in retail sales, will likely have small differences between FCF and EBITDA for. Setting aside depreciation and special tax subsidies, taxes will always create a difference between EBITDA and FCF for a profitable firm. LO: 2 Level: Intermediate Bloomcode: Analysis SM 19-554
AASCB: Analytic IMA: Business Economics AICPA: Industry/Sector Perspective
12.5 Describe how the pretax operating cash flow break-even point is related to the economic break-even point.
Solution: The pre-tax operating cash flow break-even point establishes the number of units that must be sold in a given year to break even for a particular year. The economic break-even calculation looks at cash flows over the course of an entire project and tells us how many units must be sold to achieve an NPV of $0. The NPV calculation is useful when deciding whether to undertake a project in an economic sense. The cash flow break-even
SM 19-555
calculation is useful when considering whether to abandon a project or to make changes to a project’s cost structure. LO: 4 Level: Basic Bloomcode: Comprehension AASCB: Analytic IMA: Business Economics AICPA: Industry/Sector Perspective
12.6
Is it possible to have a crossover point where the
accounting break-even point is the same for two alternatives that is, above the break-even point for a low-fixed-cost alternative but below the break-even point for a high-fixed-cost alternative? Explain.
Solution: No. Above the low fixed cost break-even sales level implies that income is positive. Below the high fixed cost break-even sales SM 19-556
level implies that income is negative. However, the cross-over point is defined as the sales level at which the income level for both alternatives is the same. Therefore, it cannot occur. LO: 3 Level: Basic Bloomcode: Comrpehension AASCB: Analytic IMA: Business Economics AICPA: Industry/Sector Perspective
12.7
What is the fundamental difference between a sensitivity analysis and a scenario
analysis?
Solution: A sensitivity analysis is a form of ―what if‖ analysis that is very useful for identifying key individual assumptions in models used for financial analysis such as an NPV calculation. Scenario analysis identifies relationships across several key individual assumptions, and is therefore a good way of estimating how project values might vary under different economic scenarios. LO: 5 Level: Basic SM 19-557
Bloomcode: Comprehension AASCB: Analytic IMA: Business Economics AICPA: Industry/Sector Perspective
12.8 The economics break-even calculation assumes that the number of units sold is the same each year during the life of the project. It is possible for the NPV of a project to be negative if unit sakes are not the same each year and the average unit sales are higher than that estimated using the economic break-even calculation.
Solution: Yes. This is possible, if, for example, actual unit sales are lower than the average in the early years of a project’s life and higher than the average in the latter years of a project’s life. This is due to the time value of money. Unit sales in early years have a greater present value than unit sales in later years. Bloomcode: Comprehension AASCB: Analytic IMA: Business Economics AICPA: Industry/Sector Perspective
SM 19-558
12.9 How does the pretax operating cash flow for a project differ from the economic profit for that project?
Solution: The pretax operating cash flow is a measure of the extent to which revenues exceed cash operating expenses. It does not account for taxes or the opportunity cost of the capital that is invested in the project. Economic profit is a measure of the extent to which revenues exceed cash operating expenses, taxes, and the opportunity cost of the capital invested in a project. Economic profit accounts for all of the costs associated with producing a project’s revenue. Bloomcode: Analysis AASCB: Analytic IMA: Business Economics AICPA: Industry/Sector Perspective
SM 19-559
12.10
What is the advantage of using a simulation
analysis instead of a scenario analysis to assess the risk of a project? Solution: Simulation analysis is like a scenario analysis except that in simulation analysis an analyst uses a computer to examine a large number of scenarios in a short period of time. This description from the text suggests that simulation analysis allows for more efficient use of the analyst’s time. In addition, since an analyst can examine a very large number of scenarios (typically in the thousands) in a simulation analysis, this analytical tool can enable the analyst to get a more complete picture of the risks associated with a project. Bloomcode: Comprehension AASCB: Analytic IMA: Business Economics AICPA: Industry/Sector Perspective SM 19-560
Questions and Problems
BASIC
12.1 Fixed and variable costs: Define variable costs and fixed costs, and give an example of each.
Solution: Fixed costs are costs that in the short term cannot be changed regardless of how much output the project produces. One example is the in-home technical computer support business (discussed in Learning by Doing Application 12.1). Regardless of the number of house calls the technical support firm makes, it will incur the full cost of advertising. Variable costs are costs that depend on the number of units of output produced by the SM 19-561
project. An example is the gas that the technical support firm uses to make house calls. The cost to keep the vehicles gassed up is directly related to the number of service calls the firm makes. LO 1 Bloomcode: Knowledge AASCB: Analytic IMA: Business Economics AICPA: Industry/Sector Perspective
12.2 EBIT: Describe the role that the mix of variable versus fixed costs has in the variation of earnings before interest and taxes (EBIT) for the firm.
Solution: By definition, variable costs do not occur unless matching sales or matching revenues also occur, whereas fixed costs are not a
SM 19-562
function of the level of sales. Therefore, a large mix of fixed costs within a firm’s cost structure will make the firm’s EBIT very reactive to a change in the level of sales for the firm. The greater the proportion of fixed costs, (compared to variable costs), the greater the variability in EBIT for the firm. LO 1 Bloomcode: Comprehension AASCB: Analytic IMA: Business Economics AICPA: Industry/Sector Perspective
12.3 EBIT: The Generic Publications Textbook Company sells all of its books for $100 per book, and it currently costs $50 in variable costs to produce each text. The fixed costs, which include depreciation and amortization for the firm, are currently $2 million per year. The firm is considering changing its production technology, which will increase the fixed costs for SM 19-563
the firm by 50 percent but decrease the variable costs per unit by 50 percent. If the firm expects to sell 45,000 books next year, should the firm switch technologies?
Solution: The current EBIT for the firm is: Revenue $100 per book × 45,000 books
=
$4,500,000 VC
$50 per book × 45,000 books
=
2,250,000 FC + D&A
2,000,000
EBIT
$250,000
If the fixed costs increase by 50 percent, then they will be $2,000,000 × 1.5 = $3,000,000, while the unit variable costs would be 0.5 × $50 = $25. The new EBIT for the firm would then be: SM 19-564
Revenue $100 per book × 45,000 books
=
$4,500,000 VC
$25 per book × 45,000 books
=
1,125,000 FC + D&A EBIT
3,000,000 $ 375,000
Since the EBIT after the technology change is $125,000 higher, the firm should adopt the new production technology. LO 1 Bloomcode: Analysis AASCB: Analytic IMA: Business Economics AICPA: Industry/Sector Perspective
12.4 EBIT: WalkAbout Kangaroo Shoe Stores forecasts that it will sell 9,500 pairs of shoes next year. The firm buys its shoes for $50 per pair from the wholesaler and sells them for $75 per pair. If the firm will incur fixed costs plus depreciation and amortization of $100,000, then what is the SM 19-565
percent increase in EBIT if the actual sales next year equal 11,500 pairs of shoes instead of 9,500?
Solution: The forecasted EBIT for the firm is: Revenue
$75 per pair × 9,500 pairs
=
$712,500
VC
$50 per pair × 9,500 pairs
=
475,000
FC + D&A
100,000
EBIT
$137,500
The actual EBIT for the firm is: Revenue
$75 per pair × 11,500 pairs
=
$862,500
VC
$50 per pair × 11,500 pairs
=
575,000
FC + D&A
100,000
EBIT
$187,500
Therefore, the percent increase in EBIT would be ($187,500 – $137,500) / $137,500 = 0.3636 = 36.36%. SM 19-566
LO 1 Bloomcode: Application AASCB: Analytic IMA: Business Economics AICPA: Industry/Sector Perspective
12.5 Cash Flow DOL: The law firm of Dewey, Cheatem, and Howe has monthly fixed costs of $100,000, EBIT of $250,000, and depreciation charges on its office furniture and computers of $5,000. Calculate the Cash Flow DOL for this firm.
Solution:
Cash Flow DOL = 1 +
FC EBITDA
=1+
$100,000 = 1.392 $250,000 +$5,000
LO 2
SM 19-567
12.6. Cash Flow DOL: The degree of pretax cash flow operating leverage at Rackit Corporation is 2.7 when it sells 100,000 units of its new tennis racket and its EBITDA is $95,000. Ignoring the effects of taxes, what are the fixed costs for Rackit Corporation?
Solution: Cash Flow DOL = 1 + FC/EBITDA 2.7 = 1 + FC / ($95,000) FC = $161,500 LO 2 Bloomcode: Application AASCB: Analytic IMA: Business Economics AICPA: Industry/Sector Perspective
SM 19-568
12.7 Accounting DOL: Explain how the value of accounting operating leverage can be used.
Solution: Accounting operating leverage gives us the ratio by which the firm can convert revenues into EBIT. That is, if the firm’s operating leverage is 3, then a 15 percent increase will convert to a 45 percent (15% x 3) increase in EBIT for the firm. LO 2 Bloomcode: Comprehension AASCB: Analytic IMA: Business Economics AICPA: Industry/Sector Perspective
12.8
Accounting DOL: Caterpillar, Inc. is a manufacturer of large earth-moving and mining
equipment. This firm, and other heavy equipment manufacturers, have accounting degrees of operating leverage that are relatively high. Explain why.
SM 19-569
Solution: Caterpillar and other heavy equipment manufacturers are heavily dependent on assets in place, like manufacturing equipment and facilities, for production. These investments result in relatively high fixed costs compared to variable costs of production, leading to a high degree of operating leverage. To illustrate, in 2009, CAT saw a 37% decrease in revenue and an 88% drop in operating income relative to their 2008 results. However, for the third quarter of 2010, CAT saw a 53% increase in sales and a 329% jump in operating profit. LO 2 Bloomcode: Comprehension AASCB: Analytic IMA: Business Economics AICPA: Industry/Sector Perspective
12.9 Break-even analysis: Why is the per-unit contribution important in a break-even analysis?
Solution: Per-unit contribution is critical to break-even analysis in order for a firm to determine how many units are required to SM 19-570
be sold to cover the firm’s fixed costs. The underlying known variable is the dollar amount of the contribution margin the firm will generate from each unit sold in order to make the above calculation. EBITDA Break-even = FC / (Price – Unit VC) and EBIT break-even = (FC + D&A) / (Price – Unit VC) demonstrate the calculation for EBITDA and EBIT break-even points respectively. The term in the denominator (Price-Unit VC) represents the per-unit cash flow contribution. LO 3 Bloomcode: Comprehension AASCB: Analytic IMA: Business Economics AICPA: Industry/Sector Perspective
12.10 Break Even: Calculate the accounting operating profit break-even point and pretax operating cash flow break-even point for each of the three production choices outlined below. SM 19-571
Choice
Price
Unit VC
FC
D&A
A B C
$250 $55 $10
$160 $10 $1.50
$15,000 $1,100 $100
$3,000 $200 $100
Solution:
Choice
Unit price
Unit VC
Fixed costs
D&A
A B C
$250 $55 $10
$160 $10 $1.50
$15,000 $1,100 $100
$3,000 $200 $100
EBIT breakeven* 200 29 24
Pretax operating cash flow breakeven** 167 24 12
* EBIT breakeven = (Fixed Costs + D&A) / (Price – Unit VC) ** Pretax operating cash flow breakeven = Fixed Costs / (Price – Unit VC) LO 3 Bloomcode: Application AASCB: Analytic IMA: Business Economics AICPA: Industry/Sector Perspective
12.11 Economic break-even point: The accounting operating profit break-even point tells us the number of units that must be sold for a firm to break-even in a given year from an accounting operating profit perspective. What measure tells us the number of units that must be sold each year during the life of a project in order for the project to breakeven with regards to its opportunity cost of capital? Solution: The economic break-even point tells us the number of units that must be sold each year during the life of a project in order for the project to break even with regards to its opportunity cost of capital. Because the economic break-even point calculation accounts for all cash flows over the life of a project, it enables us to compute the annual level of unit sales that will result in an NPV of $0. This is, by definition, the break-even point SM 19-572
with regards to a project’s opportunity cost of capital. Bloomcode: Comprehension AASCB: Analytic IMA: Business Economics AICPA: Industry/Sector Perspective
12.12
Simulation analysis: What is simulation analysis, and
how is it used?
Solution: Simulation analysis is like scenario analysis except that in simulation analysis an analyst typically uses a computer to examine a large number of scenarios in a short period of time. Rather than selecting individual values for each of the assumptions—such as unit sales, unit price, and unit variable costs—the analyst assumes that those assumptions can be represented by statistical distributions. The computer then draws upon the distribution of each variable in order to generate an SM 19-573
observation for a single scenario. After repeating the number of computer-generated scenarios, a distribution of cash flow outcomes will be generated, thereby offering the analyst the ability to perform a probability-based analysis on the cash flow distribution. LO 4 Bloomcode: Knowledge AASCB: Analytic IMA: Business Economics AICPA: Industry/Sector Perspective
INTERMEDIATE
12.13
EBIT: If a manufacturing firm and a service firm have
identical cash fixed costs, but the manufacturing firm has much higher depreciation and amortization, then which
SM 19-574
firm is more likely to have a large discrepancy between its FCF and its EBIT?
Solution: Since depreciation and amortization are noncash items, the manufacturing firm would have the greatest discrepancy between FCF and EBIT. LO 1 Bloomcode: Analysis AASCB: Analytic IMA: Business Economics AICPA: Industry/Sector Perspective
12.14
EBIT: Duplicate Footballs, Inc., expects to sell
15,000 balls this year. The balls sell for $110 each and have a variable cost per unit of $80. Fixed costs, including depreciation and amortization, are currently $220,000 per SM 19-575
year. How much can either the fixed costs or the variable cost per unit increase in order to keep the company from having a negative EBIT. Solution: The forecasted EBIT for the firm is: Revenue $110 per ball × 15,000 balls
=
$1,650,000 VC
$80 per ball × 15,000 balls
=
1,200,000 FC + D&A EBIT
220,000 $ 230,000
Therefore, the fixed costs could increase by $230,000 and still keep the EBIT from being negative. If we focus on the variable costs, we know that total variable costs could increase by $230,000. If that cost is spread over 15,000 units, then the variable cost per unit could increase by SM 19-576
($230,000 / 15,000) = $15.33 and still keep the EBIT from being negative. Note that the analysis assumes that increases in either the fixed cost or the variable cost per unit will not change the other. This is probably not a realistic assumption. LO 1 Bloomcode: Analysis AASCB: Analytic IMA: Business Economics AICPA: Industry/Sector Perspective
12.15
EBIT: Specialty Light Bulbs anticipates selling 3,000
light bulbs this year at a price of $15 per bulb. It costs Specialty $10 in variable costs to produce each light bulb, and the fixed costs for the firm are $10,000. Specialty has an opportunity to sell an additional 1,000 bulbs next year at the same price and variable cost, but by doing so the firm
SM 19-577
will incur an additional fixed cost of $4,000. Should Specialty produce and sell the additional bulbs?
Solution: The forecasted EBIT (Additional production) for the firm is: Revenue
$15 per bulb × 1,000 bulbs
=
$15,000
VC
$10 per bulb × 1,000 bulbs
=
10,000
FC
4,000
EBIT
$ 1,000
Since the EBIT is positive, then Specialty should produce and sell the additional bulbs. LO: 1 Bloomcode: Analysis AASCB: Analytic IMA: Business Economics AICPA: Industry/Sector Perspective
SM 19-578
12.16. Cash Flow DOL: The pretax operating cash flow of Memphis Motors declined so much during the recession of 2008 and 2009 that the company almost defaulted on its debt. The owner of the company wants to change the cost structure of his business so that this does not happen again. He has been able to reduce fixed costs from $500,000 to $300,000 and, in doing so, reduce the Cash Flow DOL for Memphis Motors from 3.0 to 2.2 with sales of $1,000,000 and pretax operating cash flow of $250,000. If sales declined by 20 percent from this level, how much more pretax operating cash flow would Memphis Motors have with the new cost structure than under the old?
Solution: With the old cost structure, pretax operating cash flow would decline by 3.0 × 20% = 60% to $100,000 ($250,000 × (1 – 0.6)). With the new cost structure, pretax operating cash flow would decline by 2.2 × 20% = 44% to $140,000 ($250,000 × (1 – 0.44)).
Memphis Motors would have $40,000 more pretax operating cash flow under the cost new structure.
LO: 2 Bloomcode: Application AASCB: Analytic IMA: Business Economics AICPA: Industry/Sector Perspective SM 19-579
12.17
Cash Flow DOL: For the Vinyl CD Co. in Self-Study
Problem 12.3, what percentage increase in pretax operating cash flow will be driven by the additional revenue? Solution: Cash flow DOL = 1 + (FC) / (EBIT + D&A) =1 + ($100,000) / ($90,000 + $80,000) = 1.59 Therefore, a 10 percent additional increase in revenue should drive a 15.9 percent increase in pretax operating cash flow. LO 2 Bloomcode: Application AASCB: Analytic IMA: Business Economics AICPA: Industry/Sector Perspective
Use the following information for Problems 12.18, 12.19, and 12.20:
SM 19-580
Dandle’s Candles will be producing a new line of dripless candles in the coming years and has the choice of producing the candles in a large factory with a small number of workers or a small factory with a large number of workers. Each candle will be sold for $10. If the large factory is chosen, the cost per unit to produce each candle will be $2.50. The cost per unit will be $7.50 in the small factory. The large factory would have fixed cash costs of $2 million and a depreciation expense of $300,000 per year, while those expenses would be $500,000 and $100,000, respectively, in the small factory.
12.18
Accounting operating profit break-even: Calculate
the accounting operating profit break-even point for both factory choices for Dandle’s Candles.
SM 19-581
Solution: The formula for the accounting operating profit break-even is: EBIT break-even = (FC + D&A) / (Price – Unit VC) For the large factory: EBIT break-even = ($2,000,000
/ ($10
– $2.50) = 306,667 units For the small factory: EBIT break-
/ ($10 –
$7.50) = 240,000 units LO 3 Bloomcode: Application AASCB: Analytic IMA: Business Economics AICPA: Industry/Sector Perspective
12.19.
Crossover level of unit sales: Calculate the number
of candles for which the accounting operating profit at SM 19-582
Dandle's Candles is the same regardless of the factory choice.
Solution: The formula for the crossover level of units sales (CO) is: FC D & A COEBIT
COEBIT
FC D & A Alternative1
Alternative2
Unit Contribution Alternative1 Unit Contribution Alternative2 $2,300, 000 $600, 000 340, 000 units ($10 $2.5) ($10 $7.5)
LO 3 Bloomcode: Application AASCB: Analytic IMA: Business Economics AICPA: Industry/Sector Perspective
12.20
Pretax operating cash flow break-even: Calculate
the pretax operating cash flow break-even point for both factory choices for Dandle’s Candles.
SM 19-583
Solution: The formula for the pretax operating cash flow breakeven is: EBITDA Break-even = FC / (Price – Unit VC) and so the EBITDA break-even for the large factory is: EBITDA Break-
/ ($10 –
$2.5) = 266,667 units and the EBITDA break-even for the small factory is: EBITDA Break-even
/ ($10 – $7.5) =
200,000 units LO 3 Bloomcode: Application AASCB: Analytic IMA: Business Economics AICPA: Industry/Sector Perspective
SM 19-584
12.21
Accounting and cash flow break-even: Your
analysis tells you that at a projected level of sales, a project your firm is considering will be below accounting breakeven but above cash flow break-even. Explain why this might still be a viable project for your firm.
Solution: While the business may show an accounting loss, our focus should be on the cash flow gain or loss. The reason that the project will produce an accounting loss but cash flow income is that the depreciation and amortization charges do not apply to the cash flow calculations as they are noncash expenses that help to reduce the tax liability. Therefore, the project is viable if it does not show a cash flow loss.
LO: 3
Bloomcode: Analysis AASCB: Analytic IMA: Business Economics AICPA: Industry/Sector Perspective SM 19-585
12.22
Economic break-even point: Management of March
and Dine Inc. has estimated that the firm’s new TV dinner project must generate $10,200 in FCF during each of the next six years to have an NPV of $0. Management anticipates that depreciation and amortization charges will equal $3,000, capital expenditures will equal $2,000, and additions to working capital will equal $500 during each of those years. What level of EBIT corresponds to an annual FCF of $10,200 if the firm is subject to the 30 percent marginal tax rate?
Solution: From step 2 of the economic break-even point calculations described in the text, we can calculate the EBIT as follows: EBIT = (FCF – D&A + Cap Exp + Add WC)/(1 – t)
SM 19-586
EBIT = ($10,200 - $3,000 + $2,000 + $500)/(1 – 0.3) = $13,857.14
Bloomcode: Application AASCB: Analytic IMA: Business Economics AICPA: Industry/Sector Perspective
12.23
Economic break-even point: RoseWeiser Company
Management is considering a project that will require an initial investment of $50,000 and will last for 10 years. No other capital expenditures or increases in working capital are anticipated during the life of the project. What is the annual EBIT that will make project economically viable if the cost of capital for the project is 9 percent and the firm will depreciate the investment using straight-line depreciation and a salvage value of $0? Assume that the marginal tax rate is 40 percent.
SM 19-587
Solution: If the FCF are expected to be equal in each of the ten years, we can solve for FCF using the present value of an ordinary annuity formula: $50,000 = FCF ×
1
× (1 −
0.09
1 (1.09)10
) → FCF = $7,791.00
EBIT = (FCF – D&A + Cap Exp + Add WC)/(1 – t) EBIT = ($7,791 - $5,000 + $0 + $0)/(1 – 0.4) = $4,651.67
Bloomcode: Application AASCB: Analytic IMA: Business Economics AICPA: Industry/Sector Perspective
12.24
Economic break-even point: The BowGus Archery
Company management estimates that its new Galactically Flexible Bow project will have to generate EBIT of $20,000 each year to be viable. The project’s fixed cash expenses are SM 19-588
expected to equal $8,000 and its depreciation and amortization expenses are expected to be $5,000 each year. If the Galactically Flexible bows are expected to sell for $150 each and the variable cost to produce each bow is expected to be $100, then how many of these bows must the firm produce and sell each year to generate annual EBIT of $20,000?
Solution
Using step 3 of the economic break-even point calculations described in the text, we can compute what the total contribution for the project must be:
Revenue – VC = EBIT + D&A + FC
= $20,000 + $5,000 + $8,000 = $33,000.
SM 19-589
From step 4, we know that we can divide this value by the contribution margin per unit to obtain the required number of units:
$33,000/($150 - $100) = 660 units.
Bloomcode: Application AASCB: Analytic IMA: Business Economics AICPA: Industry/Sector Perspective
12.25
Sensitivity and scenario analyses: Sensitivity
analysis and scenario analysis are somewhat similar. Describe which is a more realistic method of analyzing the impact of different scenarios on a project. Solution: Sensitivity analysis captures the effect of a change in a single item such as unit selling price or a change in the number of units sold on a specific item such as EBIT. However, it is unlikely SM 19-590
that a change in the selling price of an item will not affect the demand, and consequently the number of units sold, for the product in question. Scenario analysis analyzes the multiple effects of a scenario on an item such as EBIT by changing a number of interrelated variables at the same time to measure the effect of an entire scenario change. Therefore, scenario analysis is a much more practical tool for stress-testing a project. LO 5 Bloomcode: Comprehension AASCB: Analytic IMA: Business Economics AICPA: Industry/Sector Perspective
12.26
Sensitivity analysis: Describe the circumstances
under which sensitivity analysis might be a reasonable basis for determining changes to a firm’s EBIT or FCF.
SM 19-591
Solution: Since sensitivity analysis assumes independence among variables (otherwise the analysis is too superficial), then that is the time when the analysis can yield the most meaningful results. One time when that might occur is if the sales level and product price are completely unaffected by movements in the other as with an industry monopoly. In a competitive market, such an assumption could yield disastrous results if they are followed without caution. LO 5 Bloomcode: Comprehension AASCB: Analytic IMA: Business Economics AICPA: Industry/Sector Perspective
SM 19-592
12.27
Scenario analysis: Chip’s Home Brew Whiskey
management forecasts that if the firm sells each bottle of Snake-Bite for $20, then the demand for the product will be 15,000 bottles per year, whereas sales will be 90 percent as high if the price is raised 10 percent. Chip’s variable cost per bottle is $10, and the total fixed cash cost for the year is $100,000. Depreciation and amortization charges are $20,000, and the firm has a 30 percent marginal tax rate. Management anticipates an increased working capital need of $3,000 for the year. What will be the effect of the price increase on the firm’s FCF for the year?
Solution: If the firm increases its price to $22 per bottle, then it will sell 0.9 × 15,000 = 13,500 units next year. We can now find the effect of the change in price. SM 19-593
Normal
Price Hike
Revenue $300,000
$297,000
($22
per
135,000
($10
per
bottle × 13,500) Less: VC
150,000
bottle × 13,500) Less: FC
100,000
100,000
EBITDA Less: D&A EBIT
$ 50,000 $ 62,000 20,000
20,000
$ 30,000
$ 42,000
Less: Tax (30%) NOPAT
9,000
12,600
$ 21,000
$ 29,400
Add: D&A 20,000
20,000
CF Operations
$ 41,000
$
49,400 Less: Add WC FCF
3,000
$ 38,000
3,000 $ 46,400 SM 19-594
By increasing the price of a bottle by 10 percent, the FCF increases by $8,400 from $38,000 to $46,400. LO 5 Bloomcode: Application AASCB: Analytic IMA: Business Economics AICPA: Industry/Sector Perspective
12.28
Sensitivity, scenario, and simulation analysis: If
you were interested in calculating the probability that your project will have a positive FCF, what type of risk analysis tool will you most likely use?
Solution: Sensitivity analysis can only manage a single movement in a modeled variable and can therefore only show the net impact of that movement. Scenario analysis is much more flexible and can quantify the impact of moving many interdependent variables at SM 19-595
once, but it cannot produce confidence intervals for a given level of FCF. Simulation analysis begins with a distribution for the range of possible values for each variable. All of these modeled variables are then ―freed up‖ to randomly move, all at the same time, within the modeled range in order to produce an individual observation. If this process is repeated a large number of times, then a distribution of observations is generated for the FCF value (or EBIT or a whole host of other calculations) in order to be able to make statistical inferences about the probability of achieving a given level of FCF.
LO 5 Bloomcode: Application AASCB: Analytic IMA: Business Economics AICPA: Industry/Sector Perspective
ADVANCED SM 19-596
12.29
Mick’s Soft Lemonade is starting to develop a new
product for which the cash fixed costs are expected to be $80,000. The projected EBIT is $100,000, and the Accounting DOL is expected to be 2.0. What is the Cash Flow DOL for the firm?
Solution: Accounting DOL
= 1 + (FC + D&A) / (EBIT)
= 1 + ($80,000 + D&A) / $100,000 = 2 => D&A = $20,000 Cash flow DOL
= 1 + (FC) / (EBIT + D&A)
= 1 + ($80,000) / ($100,000 + $20,000) = 1.67 LO 2 Bloomcode: Application AASCB: Analytic IMA: Business Economics AICPA: Industry/Sector Perspective
SM 19-597
12.30
If a firm has a fixed asset base, meaning that its
depreciation and amortization for any year is positive, discuss the relation between its Accounting DOL and its Cash flow DOL.
Solution: By comparing the equations for the Accounting DOL and Cash Flow DOL: Accounting DOL = 1 + (FC + D&A) / (EBIT) Cash flow DOL = 1 + (FC) / (EBIT + D&A) We find that the denominator of the Cash Flow DOL will always be greater than the denominator of the Accounting DOL if depreciation and amortization is greater than zero. In addition, the numerator of the Cash Flow DOL will always be less than the denominator of the Accounting DOL if depreciation and amortization is greater than zero. Therefore, if depreciation and SM 19-598
amortization is positive, then Cash Flow DOL must be less than Accounting DOL. LO 2 Bloomcode: Comprehension AASCB: Analytic IMA: Business Economics AICPA: Industry/Sector Perspective
12.31
Silver Polygon, Inc., has determined that if its
revenues were to increase by 10 percent, then EBIT would increase by 25 percent to $100,000. The fixed costs (cash only) for the firm are $100,000. Given the same 10 percent increase in revenues, what would be the corresponding change in EBITDA?
Solution:
SM 19-599
Since a 10 percent increase in revenue will drive a 25 percent corresponding increase in EBIT, then we know that Accounting DOL = 2.5. The new EBIT would be $100,000, after the 25 percent increase, so the original EBIT was$ 100,000 / (1 + 0.25) = $80,000. Therefore, Accounting DOL
= 1 + (FC + D&A) / (EBIT) = 2.5
= 1 + ($100,000 + D&A) / ($80,000) = 2.5 ==> D&A = $20,000 Cash Flow DOL
= 1 + (FC) / (EBIT + D&A) =
= 1 + ($100,000) / ($80,000 + $20,000) = 2.0 A 10 percent increase in revenue will drive a 20 percent increase in EBITDA. LO 2 Bloomcode: Application AASCB: Analytic IMA: Business Economics AICPA: Industry/Sector Perspective
SM 19-600
If a firm’s costs (both variable as well as fixed) are
12.32
known with certainty, then what are the only two sources of volatility for the firm’s operating profits or its operating cash flows?
Solution: If the cost structure is known, then costs will only vary according to the firm’s unit sales. Therefore, one source of volatility would be net revenue uncertainty. The second source of volatility is based on the mix of variable and fixed costs within the firm’s cost structure. A higher mix of fixed costs would increase the operating leverage for a firm (both accounting and cash flow) and therefore increase the accounting profit and cash flow volatility for the firm. LO 1 Bloomcode: Comprehension SM 19-601
AASCB: Analytic IMA: Business Economics AICPA: Industry/Sector Perspective
12.33
In most circumstances, given the choice between a
higher fixed cost structure and a lower fixed cost structure, which of the two would generate a larger contribution margin?
Solution: The firm with the higher fixed cost should have a lower variable cost per unit, assuming that there is a trade-off. A lower variable cost per unit would then create a higher contribution margin for that firm. LO 1 Bloomcode: Comprehension AASCB: Analytic IMA: Business Economics AICPA: Industry/Sector Perspective
SM 19-602
12.34
Using the same logic as with the accounting break-
even calculation in Problem 12.19, adapt the formula for crossover level of unit sales to find the number of units sold where the pretax operating cash flow is the same whether the firm chooses the large or small factory.
Solution:
The formula for the cross-over level of unit sales, based on accounting EBIT, is as follows: FC+D&A COEBIT=
Alternative1
-FC+D&A Alternative2
Unit ContributionAlternative1 - Unit ContributionAlternative2
.
Eliminating D&A, since D&A are non-cash charges, gives us SM 19-603
FC COEBITDA =
- FC
Alternative1
Alternative2
Unit ContributionAlternative1 - Unit ContributionAlternative2
COEBITDA = $2,000,000 -$500,000 $1, 500, 000 300, 000 ($10 -$2.5)-($10 $7.5) $7.5 $2.5
LO 3 Bloomcode: Application AASCB: Analytic IMA: Business Economics AICPA: Industry/Sector Perspective
12.35 You are the project manager for Eagle Golf Corporation. You are considering manufacturing a new golf wedge with a unique groove design. You have put together the estimates in the following table about the potential demand for the new club, and the associated selling and manufacturing prices. You expect to sell the club for five years. The equipment required for the manufacturing process can be depreciated using straight line depreciation over five years and will have a zero salvage value at the end of the project’s life. No additional capital expenditures are required. No new working capital is needed for the project. The required return for projects of this type is 12 percent and the company has a 35 percent marginal tax rate. You estimate that there is a 50 percent chance the project will achieve the expected sales and a 25 percent chance of achieving either the weak or strong sales outcomes. Should you recommend the project?
Units sold
Strong Sales 15,000
Expected Sales 10,000
Weak Sales 7,000 SM 19-604
Selling price per unit Variable costs per unit Fixed Costs Initial Investment
$130 $70 $1,290,000 $1,400,000
$120 $65 $1,290,000 $1,400,000
$110 $60 $1,290,000 $1,400,000
Solution (Excel Model with Solution in Separate File): Expected unit sales = (0.25 × 15,000) + (0.50 × 10,000) + (0.25 × 7,000) = 10,500 units Expected unit price = (0.25 × $130) + (0.50 × $120) + (0.25 × $110) = $120 Expected unit variable cost = (0.25 × $70) + (0.50 × $65) + (0.25 × $60) = $65 Based on the expect values outlined below, the NPV of the project is -$279,365.20 so it should not be accepted.
Opportunity cost of capital
12%
Expected unti sales
10,500
Expected unit price
$120
Expected unti variable cost
$65
Initial investment
$1,400,000
D&A
$280,000
Fixed costs
$250,000
0
1
Revenue
2
3
4
5
$1,260,000
$1,260,000
$1,260,000
$1,260,000
$1,260,000
- VC
(682,500)
($682,500)
($682,500)
($682,500)
($682,500)
- FC
($258,000)
($258,000)
($258,000)
($258,000)
($258,000)
EBITDA
$319,500
$319,500
$319,500
$319,500
$319,500
- D&A
($280,000)
($280,000)
($280,000)
($280,000)
($280,000)
EBIT - Taxes (35%) NOPAT + D&A
$
CF Opns
$39,500
$39,500
$39,500
$39,500
$39,500
($13,825)
($13,825)
($13,825)
($13,825)
($13,825)
$25,675
$25,675
$25,675
$25,675
$25,675
280,000 $
280,000 $
280,000 $
280,000 $
280,000
$305,675
$305,675
$305,675
$305,675
$305,675 $0
- Cap Exp.
($1,400,000)
$0
$0
$0
$0
- Add. WC
$0
$0
$0
$0
$0
$0
FCF
($1,400,000)
$305,675
$305,675
$305,675
$305,675
$305,675
NPV @ k = 12%
($298,110.03)
LO 5 Bloomcode: Analysis AASCB: Analytic IMA: Business Economics AICPA: Industry/Sector Perspective SM 19-605
12.36 Economic break-even point: Commodore Motors management is considering a project to produce toy cars. The project would require an initial outlay of $100,000 and have an expected life of 10 years. Management estimates that each year during the life of the project depreciation and amortization would be $8,000, capital expenditures would be $4,000, additions to working capital would be $2,000, and fixed costs would be $3,000. Also, each toy car would sell for $15 and cost $7 to produce. Finally, the cost of capital for the project would be 12 percent, cash flows from the project would be taxed at a 25 percent rate, and the assets would be depreciated to a salvage value of $0. How many units must be sold each year in order for this project to break-even from an economic standpoint?
Solution: Step one: The required annual FCF is: $100,000 = FCF ×
1 0.12
× (1 −
1 (1.12)10
) → FCF = $17,698.42
Step two: The required EBIT is: EBIT = (FCF – D&A + Cap Exp + Add WC)/(1 – t) EBIT = ($17,698.42 - $8,000 + $4,000 + $2,000)/(1 – 0.25) = $20,931.23 Step three: The required contribution is: Revenue – VC = EBIT + D&A + FC = $20,931.22 + $8,000 + $3,000 = $31,931.23 Step four: Economic break-even level of unit sales is: = $31,931.22/($15 – 7) = 3,991.4, or 3,992 cars Bloomcode: Analysis AASCB: Analytic IMA: Business Economics AICPA: Industry/Sector Perspective
CFA PROBLEMS SM 19-606
12.37. Operating leverage is a measure of the: a.
Sensitivity of net earnings to changes in operating earnings.
b.
Sensitivity of net earnings to changes in sales.
c.
Sensitivity of fixed operating costs to changes in variable costs.
d.
Sensitivity of earnings before interest and taxes to changes in the number of units produced and sold.
Solution: d is correct. Operating leverage is the sensitivity of earnings before interest and taxes to changes in the number of units produced and sold. The degree of operating leverage is the elasticity of operating earnings with respect to the number of units produced and sold. LO 2 Bloomcode: Knowledge AASCB: Analytic IMA: Business Economics AICPA: Industry/Sector Perspective
12.38. The Fulcrum Company produces decorative swivel platforms for home televisions. If Fulcrum produces 40 million units, it estimates that it can sell them for $100 each. The variable production costs are $65 per unit, whereas the fixed production costs are $1.05 billion. Which of the following statements is true? a.
The Fulcrum Company produces a positive operating income if it produces and sells more than 25 million swivel platforms.
b.
The Fulcrum Company’s degree of operating leverage is 1.333. SM 19-607
c.
If the Fulcrum Company increases production and sales by 5 percent, its operating earnings are expected to increase by 20 percent.
d.
Increasing the fixed production costs by 10 percent will result in a lower sensitivity of operating earnings to changes in units produced and sold.
Solution:
c is correct. 40 million($100 $65) 40 million($100 $65) $1.05 billion $1.400 billion = $1.400 billion $1.05 billion $1.4 = $0.35 =4
DOL =
Operating breakeven =
$1.05 billion
= 30 million units
$35
Fulcrum produces positive operating income if it produces more than 30 million units. If it produces and sells fewer than 30 million, it will generate a loss. The DOL is 4. If unit sales increase by 5 percent, Fulcrum’s operating earnings are expected to increase by 4 × 5% = 20%. Increasing fixed production costs will increase the sensitivity of Fulcrum’s operating earnings to changes in sales. LO 1 Bloomcode: Application AASCB: Analytic IMA: Business Economics
SM 19-608
AICPA: Industry/Sector Perspective
Sample Test Problems Sample Test Problems
12.1. Retro Inc. sells vintage football jerseys for $72 each. Variable costs are $58 per unit and total fixed costs (including depreciation and amortization expense) are $84,000 per year. If sales for next year are expected to equal 8,000 jerseys, how much can variable costs per unit increase without EBIT becoming negative? (LO 1) Solution: The values in the problem statement indicate that the expected EBIT for Retro Inc. next year is: Revenue$72 × 8,000 = $576,000 VC $58 × 8,000 = 464,000 FC + D&A 84,000 EBIT $28,000 Therefore, total variable cost could increase by $28,000 without EBIT becoming negative. If the firm sells 8,000 jerseys, then the variable cost per unit could increase by as much as $3.50 ($28,000/8,000 units = $3.50 per unit). Bloomcode: Application AASCB: Analytic IMA: Business Economics AICPA: Industry/Sector Perspective
SM 19-609
12.2 How would a capital intensive company fare during good and poor economic times as compared with other companies? Explain. (LO 1) Solution: A capital intensive company has a higher proportion of fixed costs and therefore higher operating leverage than less capital intensive companies. High operating leverage causes the pretax operating cash flows and accounting profits to be more sensitive to changes in revenues. As a result, during good economic times these cash flows and profits will increase more and during poor economic times periods they will fall more than the corresponding measures at companies that have less operating leverage. Bloomcode: Comprehension AASCB: Analytic IMA: Business Economics AICPA: Industry/Sector Perspective
SM 19-610
12.3 The manager of Roy’s Restaurant has determined that if revenues were to increase by 20 percent, then EBIT would increase by 45 percent to $87,000. What would be the corresponding change in EBITDA if revenues increased 20 percent and cash fixed costs are $35,000? (LO 2) Solution: If a 20 percent increase in revenue causes a 45 percent increase in EBIT, Accounting DOL must equal 2.25. Also, since EBIT would be $87,000 after the 45 percent increase, the original EBIT was $87,000/(1 + 0.45) = $60,000. Therefore: Accounting DOL = 2.25 = 1 + (FC + D&A)/EBIT = 2.25 = 1 + ($35,000 + D&A)/$60,000 => D&A = $40,000 Cash Flow DOL = 1 + FC/(EBIT + D&A) = 1 + $35,000/($60,000 + $40,000) = 1.35 Which tells us that a 20 percent increase in revenues will result in a 27 percent (20 percent × 1.35 = 27 percent) increase in EBITDA. Bloomcode: Application AASCB: Analytic IMA: Business Economics AICPA: Industry/Sector Perspective
12.4 Luminosity Inc. produces modern light fixtures that sell for $150 per unit. The firm’s management is considering purchasing a high-capacity manufacturing machine. If the highcapacity machine is purchased, then the firm’s annual cash fixed costs will be $60,000 per year, variable costs will be $55 per unit, and annual depreciation and amortization expenses will equal SM 19-611
$30,000. If the machine is not purchased, annual cash fixed costs will be $25,000, variable costs will be $105 per unit, and annual depreciation and amortization expenses will equal $10,000. What is the minimum level of unit sales necessary in order for EBIT with the high-capacity machine will be higher than EBIT without that machine? (LO 3)
Solution: This question is asking for the crossover level of unit sales for EBIT, COEBIT. Using Equation 12.7 we find that: FC+ D&A COEBIT =
COEBIT =
- FC+D&A High capacity
Nohighcapacity
Unit contributionHighcapacity - Unit contribution Nohighcapacity
, and so
$90,000 -$35,000 =1,100 units ($150 -$55)-($150 -$105)
Bloomcode: Application AASCB: Analytic IMA: Business Economics AICPA: Industry/Sector Perspective
SM 19-612
12.5 Paper Christmas Trees Inc. is considering introducing a new line of inexpensive Christmas trees. The initial outlay for the project is $175,000, and the company will have to invest $5,000 in working capital and $10,000 in fixed assets each year during the six year life of the project. The initial outlay will be depreciated assuming a salvage value of $0. Annual depreciation and amortization charges for the project will be $15,000 and cash-related fixed costs will be $6,000 per year. The firm will sell each tree for $75, and the variable cost to produce each tree will be $40. Calculate the number of trees that the firm must produce and sell in order to break even economically. Assume that the appropriate cost of capital for the project is 15 percent and that the marginal tax rate for the firm is 40 percent. (LO 4)
Solution: Step one: The required annual FCF is: 1 1 FCF = $46,241.46 0.15 1.156 1
$175,000 = FCF
Step two: The corresponding EBIT is: EBIT = (FCF – D&A + Cap Exp + Add WC)/(1 - t) EBIT = ($46,241.46 – $15,000 + $10,000 + $5,000)/(1 - 0.4) = $77,069.10 Step three: Therefore, the total contribution is: Revenue - VC = EBIT + D&A + FC
=
$77,069.10
+
$5,000
+
$6,000
=
$88,069.10 Step four: Finally, the economic break-even level of sales is: $88,069.10 / ($75 – $40) = 2,516.26 or 2,516 trees.
Bloomcode: Application AASCB: Analytic IMA: Business Economics AICPA: Industry/Sector Perspective
Chapter 13 The Cost of Capital
SM 19-613
Before You Go On Questions and Answers Section 13.1 4. Why does the market value of the claims on the assets of a firm equal the market value of the assets? The investors who own the debt and equity claims on the assets of a firm have the right to receive all of the after-tax cash flows that the assets of the firm produce. Since the market value of the assets equals the present value of the cash flows the assets produce, the market value of the assets must equal the value of the claims on those assets. 5. How is the WACC for a firm calculated? The WACC is calculated as the weighted average of the different types of claims on the firm’s assets. The weights in this calculation are the fractions of the total value of the financing that is represented by each individual type of financing. Equation 11.2 is the general form of the WACC calculation. n
k firm xiki x1k1 x2k2 x3k3 ... xnkn i1
6. What does the WACC for a firm tell us? The WACC tells us the average cost of the money that has been used to finance the firm.
Section 13.2 5. Why do analysts care about the current cost of long-term debt when estimating a firm’s cost of capital? Managers care about the current cost of long-term debt because the opportunity cost of capital that is relevant when discounting future cash flows is the opportunity cost of capital as of today. Managers focus on long-term debt because firms generally use it to finance their long-term assets, and it is the long-term assets that they are concerned about when they think about the value of a firm’s assets. 6. How do you estimate the cost of debt for a firm with more than one type of debt? When a firm has more than one type of debt, its overall cost of debt is estimated as a weighted average of the costs of each type of debt. The weights in this calculation are the fractions of the total value of the debt represented by each individual type of debt. SM 19-614
7. How do taxes affect the cost of debt? In the United States, the ability of firms to deduct interest payments when they compute their taxes actually reduces the cost of using debt. The after-tax cost of debt equals the pretax cost of debt times one minus the firm’s marginal tax rate. Section 13.3 1. What information is needed to use the CAPM to estimate kcs or kps? In order to use the CAPM to estimate kcs or kps, you need to know the risk-free rate, the market risk premium, and the beta for the stock. 2. Under what circumstances can you use the constant-growth dividend formula to estimate kcs? The constant-growth dividend formula can be used to estimate kcs if you can observe the current market price of the common stock and you can estimate the dividend that stockholders will receive next period, D1, and the rate at which the market expects dividends to grow over the long run, g. Of course, it only makes sense to use this model if dividends are expected to grow at a constant rate for the foreseeable future and this growth rate is not greater than the long-term growth rate of the economy. 3. What is the advantage of using a multistage-growth dividend model, rather than the constantgrowth dividend model, to estimate kcs? A multistage model allows dividends to grow at different rates over time, while the constantgrowth model allows for only a single growth rate in perpetuity. Section 13.4 4. Do analysts use book values or market values to calculate the weights when they use Equation 11.7? Why? Analysts use market values because this is what the theory underlying the calculation says they should use. Book values are relevant only if they just happen to equal the market values. 5. What kinds of errors can be made when the WACC for a firm is used as the discount rate for evaluating all projects in the firm? Using the WACC to discount cash flows for projects that are less risky than the firm can result in managers rejecting positive-NPV projects. Using the WACC to discount cash flows SM 19-615
for projects that are more risky than the firm can result in managers accepting negative-NPV projects. 6. Under what conditions is the WACC the appropriate discount rate for a project? The WACC is the appropriate discount rate for a project when the project has the same level of systematic risk as the firm and when the project will be financed with the same proportion of debt, preferred shares, and common shares that have been used to finance the assets of the firm.
Self-Study Problems 13.1 The market value of a firm’s assets is $3 billion. If the market value of the firm’s liabilities is $2 billion, what is the market value of the stockholders’ investment and why? Solution: Since the identity that Assets = Liabilities + Equity holds for market values as well as book values, we know that the market value of the firm’s equity is $3 billion—$2 billion, or $1 billion.
13.2 Berron Comics, Inc., has borrowed $100 million and is required to pay its lenders $8 million in interest this year. If Berron is in the 35 percent marginal tax bracket, then what is the after-tax cost of debt (in dollars as well as in annual interest) to Berron. Solution: Because Berron enjoys a tax deduction for its interest charges, the after-tax interest expense for Berron is $8 million × (1 – 0.35) = $5.2 million, which translates into an annual interest expense of $5.2/$100 = 0.052, or 5.2 percent.
13.3 Explain why the after-tax cost of equity (common or preferred) does not have to be adjusted by the marginal income tax rate for the firm. Solution: The U.S. tax code allows a deduction for interest expense incurred on borrowing. Preferred and common shares are not considered debt and, thus, do not benefit from an interest deduction. As a result, there is no distinction between the before-tax and after-tax cost of equity capital. SM 19-616
13.4 Mike’s T-Shirts, Inc., has debt claims of $400 (market value) and equity claims of $600 (market value). If the after-tax cost of debt financing is 11 percent and the cost of equity is 17 percent, then what is Mike’s weighted average cost of capital? Solution: Mike’s T-Shirts’ total firm value = $400 + $600 = $1,000. Therefore, Debt = 40 percent of financing Equity = 60 percent of financing WACC = xDebtkDebt(1-t) + xpskps + xcskcs WACC = (0.4 × 0.11) + (0.6 × 0.17) = 0.146, or 14.6%
13.5 You are analyzing a firm that is financed with 60 percent debt and 40 percent equity. The current cost of debt financing is 10 percent, but due to a recent downgrade by the rating agencies, the firm’s cost of debt is expected to increase to 12 percent immediately. How will this change the firm’s weighted average cost of capital if you ignore taxes? Solution: The pretax debt contribution to the cost of capital is xDebt × kDebt, and since the firm’s pretax cost of debt is expected to increase by 2 percent, we know that the effect on WACC (pretax) will be 0.6 × 0.02 = 0.012, or 1.2 percent. Incidentally, if we assume that the firm is subject to the 40 percent marginal tax rate, then the after-tax contribution to the cost of capital for the firm would be 0.012 × (1 – 0.4) = 0.0072, or 0.72 percent.
Discussion Questions 13.1
Explain why the required rate of return on a firm’s assets must be equal to the weighted average cost of capital associated with its liabilities and equity. Solution: In order to conceptualize the answer to this question, it helps to think of the case in which the firm has raised all of its capital needs from a single source who owns all of the liability and all of the equity claims on the firm. Assume that this source has no other investments. If we were to measure the rate of return on the combined portfolio of investments for this source, we would find that it is exactly equal to the return on the total assets of the firm since that is the ultimate source of the returns. The weighted return of SM 19-617
that portfolio, which is the weighted average cost of capital for the firm (if for the time being we abstract away tax effects), is the return on the assets of the firm. LO: 1 Level: Basic Bloomcode: Comprehension AASCP: Analytic IMA: Corporate Finance AICPA: Industry/Sector Perspective
13.2
Which is easier to calculate directly, the expected rate of return on the assets of a firm or the expected rate of return on the firm’s debt and equity? Assume that you are an outsider to the firm. Solution: As an outsider to the firm, you will not be privy to the complete information about the projected cash flows of each of the firm’s assets, and so that is a somewhat difficult proposition. However, the collective market has made an inference concerning the expected cash flows of each of the financing claims of the firm, and by pricing those cash flows has given us an expected return for each of those claims. Therefore, finding the expected return on the debt and equity claims of the firm is much easier than finding the expected return on the assets of the firm, although that return can then be calculated from the expected return on the financing claims of the firm.
LO: 1 Level: Basic Bloomcode: Analysis AASCP: Analytic IMA: Corporate Finance AICPA: Industry/Sector Perspective
13.3
With respect to the level of risk and the required return for a firm’s portfolio of projects, discuss how the market and a firm’s management can have inconsistent information and expectations. Solution: Firm management will be fully informed concerning the firm’s project risks, but their ability to accurately predict the required return for the firm’s projects depends on the market’s assessment of those project risks. Alternatively, the collective market is not fully informed (as outsiders) concerning the firm’s project risks and yet uses its incomplete information set to dictate a required return for the firm’s projects. This suggests that if the firm were able to better inform the market, and thereby reduce the market’s perceived risk SM 19-618
on the firm’s projects, then the firm might be able to reduce the required rate of return on the firm’s projects. LO: 1 Level: Basic Bloomcode: Comprehension AASCP: Analytic IMA: Corporate Finance AICPA: Industry/Sector Perspective
13.4
Your friend has recently told you that the federal government effectively subsidizes the use of debt financing (vs. equity financing) for corporations. Do you agree with that statement? Explain. Solution: Your friend is correct. Because interest expense on debt is tax deductible, whereas dividend payments on equity are not, the firm effectively gets a rebate on interest paid through a lowered tax bill. Two firms with identical EBIT amounts with different interest expenses will have different cash flow available to its collective set of investors. The firm with greater interest expense (assuming it is less than the EBIT amount) will have greater cash flow available to all of its investors.
LO: 2 Level: Basic Bloomcode: Comprehension AASCP: Analytic IMA: Corporate Finance AICPA: Industry/Sector Perspective
13.5
Your firm will have a fixed interest expense for the next 10 years. You recently found out that the marginal income tax rate for the firm will change from 30 percent to 40 percent next year. Describe how the change will affect the cash flow available to investors. Solution: Let’s compare our firm to the firm with no interest expense. In order to make a concrete example, assume that our firm has interest expense of $100 per year. Since the after-tax cost of debt for the firm is equal to kDebt pretax × (1 – t), then we can calculate the tax benefit to using debt to be kDebt pretax × t. In order to calculate that benefit in dollar terms, we would just multiply (interest expense) × t. Therefore, the current dollar benefit to the interest expense is $100 × 0.3 = $30. Next year, the dollar benefit is $100 × 0.4 = $40. The net benefit of interest expense from the increased marginal corporate tax rate is $10, and that is a positive benefit. Note that the analysis isolates the effect on debt and does SM 19-619
not consider the lower operating earnings figure caused by the increased tax rate. Overall, the increase in tax rate will result in less cash flows available to investors, but for the leveraged (debt holding) firm the reduction in cash flow is mitigated by the benefit from being able to deduct interest expenses. LO: 2 Level: Intermediate Bloomcode: Application AASCP: Analytic IMA: Corporate Finance AICPA: Industry/Sector Perspective
13.6
Describe why it is not usually appropriate to use the coupon rate on a firm’s bonds to estimate the pretax cost of debt for the firm. Solution: The pretax cost of debt for the firm is the current annual economic cost of borrowing for the firm (before any tax effects). That cost is better measured by the current yield to maturity on the firm’s debt than by the coupon rate that is currently paid on that debt. Since most firms try to issue new bonds very close to par, the coupon rate on a bond is an indication of the yield to maturity on the bond issue at the time of issue. Unless the market-determined borrowing rate for the firm is the same as when the bond was issued, then the current yield to maturity of a bond will not be equal to the current coupon rate on the bond.
LO: 2 Level: Basic Bloomcode: Comprehension AASCP: Analytic IMA: Corporate Finance AICPA: Industry/Sector Perspective
13.7
Maltese Falcone, Inc., has not checked its weighted average cost of capital for four years. Firm management claims that since Maltese has not had to raise capital for new projects in four years, they should not have to worry about their current weighted average cost of capital. They argue that they have essentially locked in their cost of capital. Critique management’s statements. Solution: That is a false statement. Maltese is assuming that since it does not have to raise capital for new projects, then it has essentially locked in its cost of capital. However, in a liquid capital market every firm competes for capital every day since the firm’s investors have SM 19-620
the opportunity to sell their investments to other investors. If a firm does not provide investors with an ample return, then the investors will sell their investments in the firm, which, in aggregate, will have the effect of actually raising the cost of capital for the firm (since the current price of the securities will move down). Therefore, a firm that ignores its current cost of capital by thinking that it has locked in a cost of capital might even be raising its cost of capital by making that incorrect assumption. LO: 1 Level: Intermediate Bloomcode: Application AASCP: Analytic IMA: Corporate Finance AICPA: Industry/Sector Perspective
13.8
Ten years ago, the Edson Water Company issued preferred stock at a price equal to the par value of $100. If the dividend yield on that issue was 12 percent, explain why the firm’s current cost of preferred capital is not likely to equal 12 percent. Solution: Since the price of the preferred shares at issue was $100 and the dividend yield was 12 percent, then we know that the annual dividend on the shares is $12. We also then know that the required rate of return at the time of issue was 12 percent. If during the last 10 years, the required rate of return on Edson’s preferred shares has changed at all, the current required rate of return will not be 12 percent. This will, in turn, change the price of the shares to some amount other than $100.
LO: 3 Level: Intermediate Bloomcode: Appllication AASCP: Analytic IMA: Corporate Finance AICPA: Industry/Sector Perspective
13.9
Discuss under what circumstances you might be able to use a model that assumes constant growth in dividends to calculate the current cost of equity capital for a firm. Solution: In order to be completely correct, a firm must grow its dividends at a constant rate into the indefinite future. If one expects the growth in dividends to change in the future, then using a constant-growth dividend assumption is incorrect and only an estimation.
LO: 3 Level: Basic SM 19-621
Bloomcode: Comprehension AASCP: Analytic IMA: Corporate Finance AICPA: Industry/Sector Perspective
13.10 Your boss just finished computing your firm’s weighted average cost of capital. He is relieved because he says that he can now use that cost of capital to evaluate all projects that the firm is considering for the next four years. Evaluate that statement. Solution: Your boss is incorrect. A firm is always subject to revisions to its cost of capital due to current market and firm conditions. In addition, the firm could also be making an error by using the same cost of capital for all of its future projects. For that particular error to not be made, two conditions must be met. That is, future projects must be financed with the same mix of capital (debt, preferred shares, and common shares) with which the entire firm is currently financed. In addition, the future projects must contain the same level of systematic risk as that of the average project that the firm is currently operating. LO: 4 Level: Basic Bloomcode: Analysis AASCP: Analytic IMA: Corporate Finance AICPA: Industry/Sector Perspective
Questions and Problems BASIC 13.1
Finance balance sheet: KneeMan Markup Company has total debt obligations with book and market values equal to $30 million and $28 million, respectively. It also has total equity with book and market values equal to $20 million and $70 million, respectively. If you were going to buy all of the assets of KneeMan Markup today, how much should you be willing to pay? Solution: The price you should be willing to pay for all of the assets of the firm is the market value of those assets. Using the market price version of the balance sheet identity, we can add the market price of the debt obligations and the equity to find the market price of the assets. That is, $28 million + $70 million = $98 million. SM 19-622
LO 1 Bloomcode: Application AASCP: Analytic IMA: Corporate Finance AICPA: Industry/Sector Perspective
13.2
WACC: What is the weighted average cost of capital for a firm?
Solution: The weighted average cost of capital (WACC) is the weighted average of the costs to the different sources of capital used to fund a firm that is the cost of each type of capital is weighted by the proportion of the total capital that it represents. The WACC is often used as an estimate of the cost of financing a new project given the firm’s current mix of debt and equity. LO 1 Bloomcode: Knowledge AASCP: Analytic IMA: Corporate Finance AICPA: Industry/Sector Perspective
13.3
Taxes and the cost of debt: How are taxes accounted for when we calculate the cost of debt? Solution: When we calculate the cost of debt for a U.S. firm, we must take into account the tax subsidy given in the United States for interest payments on debt. For every dollar the firm pays in interest, the firm’s tax bill will decline by ($1 * t), where t is the firm’s marginal tax rate. We adjust for this tax benefit by multiplying the pretax cost of debt by (1 - t). This calculation gives us the after-tax cost of debt. We use the after-tax cost of debt for cost of capital calculations such as when we calculate the WACC.
LO 2 Bloomcode: Comprehension AASCP: Analytic IMA: Corporate Finance AICPA: Industry/Sector Perspective
13.4
Cost of common stock: List and describe each of the three methods used to calculate the cost of common stock. SM 19-623
Solution: 1) The Capital Asset Pricing Model (CAPM) formula for the cost of common stock, given in Equation 13.4 (kcs = Rrf + (βcs × Market risk premium), can be used to calculate the return investors will demand on investment in the company’s common stock. Read Chapter 7 for further discussion of the CAPM. 2) The constant-growth dividend model can be used to calculate the cost of equity implied by the firm’s current stock price. In an efficient market, the current price of the company’s stock should reflect the cash flows (dividends) that investors will receive in the future from holding equity in the firm, discounted by an appropriate rate (the cost of equity). By knowing the current dividend paid by the firm and the expected growth rate of dividends, we can use Equation 13.5 [(kcs = D1 / P0) + g] to compute the cost of capital that is implied in the firm’s current stock price. The constant-growth dividend model is only appropriate when there is a reasonable expectation that the firm’s dividend will continue growing at approximately the same rate forever. For example, it might be used to calculate the cost of equity for a mature company whose growth rate is similar to that of the economy. 3) The multistage-growth dividend model is very similar to the constant-growth dividend model, but the multistage-growth dividend model can be applied in situations when the growth rate is expected to change in the near term followed by a constant long term growth rate—for example, a small, fast growing company whose growth will certainly slow as the company becomes larger. See Section 13.3 for discussion of the calculation of cost of equity using the multistage-growth dividend model. LO 3 Bloomcode: Knowledge AASCP: Analytic IMA: Corporate Finance AICPA: Industry/Sector Perspective
13.5
Cost of common stock: Whitewall Tire Co. just paid an annual dividend of $1.60 on its common shares. If Whitewall is expected to increase its annual dividend by 2 percent per year into the foreseeable future and the current price of Whitewall’s common shares is $11.66, what is the cost of common stock for Whitewall? Solution: The cost of common equity for Whitewall can be found using the constant-growth assumption equation:
SM 19-624
P =
D1
cs
=
D0 ×(1+ g)
=
$1.60×(1+ 0.02) = $11.66 kcs - 0.02
kcs - g kcs - g Solving for kcs, we find it is equal to 0.16 or 16 percent. LO 3 Bloomcode: Application AASCP: Analytic IMA: Corporate Finance AICPA: Industry/Sector Perspective
13.6
Cost of common stock: Seerex Wok Co. is expected to pay a dividend of $1.10 one year from today on its common shares. That dividend is expected to increase by 5 percent every year thereafter. If the price of Seerex common stock is $13.75, what is the cost of its common equity capital? Solution: We can use the formula to find the cost of common equity assuming constant growth. D k 1 g $1.10 0.05 0.13, or 13% cs
Pcs
$13.75
LO 3 Bloomcode: Application AASCP: Analytic IMA: Corporate Finance AICPA: Industry/Sector Perspective
13.7
Cost of common stock: Two-Stage Rocket paid an annual dividend of $1.25 yesterday, and it is commonly known that the firm’s management expects to increase its dividend by 8 percent for the next two years and by 2 percent thereafter. If the current price of TwoStage’s common stock is $17.80, what is the cost of common equity capital for the firm? Solution: D0(1 g ) D0(1 g )2 D0(1 g )2 (1 g ) Pcs 1 k 1 1 k 12 k g 11 k 22 cs
cs
cs
2
cs
$1.25 (1 0.08) $1.25 (1 0.08)2 $1.25 (1 0.08)2 (1 0.02) , 2 2 $17.80 1 k 1 k k 0.02 1 k cs
cs
cs
cs
Using a spreadsheet to solve for the value of kcs, we find that the cost of common equity capital is 10 percent. LO 3 Bloomcode: Application SM 19-625
AASCP: Analytic IMA: Corporate Finance AICPA: Industry/Sector Perspective
13.8
Cost of preferred stock: Fjord Luxury Liners has preferred shares outstanding that pay an annual dividend equal to $15 per year. If the current price of Fjord preferred shares is $107.14, what is the after-tax cost of preferred stock for Fjord? Solution: Using the equation for finding the cost of preferred equity, we have
kps
Dps $15 0.14, or 14% Pps $107.14
LO 3 Bloomcode: Application AASCP: Analytic IMA: Corporate Finance AICPA: Industry/Sector Perspective
13.9
Cost of preferred stock: Kresler Autos has preferred shares outstanding that pay annual dividends of $12, and the current price of the shares is $80. What is the after-tax cost of new preferred shares for Kresler if the flotation (issuance) costs for preferred shares are 5 percent? Solution: Kresler will only receive 95 percent of the proceeds, so we know that we can use the equation to solve for the cost of preferred equity by adjusting the denominator for the reduced proceeds from the sale of new equity. We then have: k ps
Dps
Pps (1- F)
$12 $80(1 0.05)
$12
0.158, or 15.8%
$76
LO 3 Bloomcode: Application AASCP: Analytic IMA: Corporate Finance AICPA: Industry/Sector Perspective
SM 19-626
13.10 WACC: Describe the alternatives to using a firm’s WACC as a discount rate when evaluating a project. Solution: There are two major reasons why WACC may not be used to discount new projects: 1. It is not appropriate to use a firm’s WACC to discount a project’s free cash flows if the systematic risk of the project is very different from the systematic risk of the firm. To account for this potential problem, some firms estimate discount rates that directly reflect the risk involved in the project’s cash flows. For example, a risky project might be assigned a discount rate that is significantly higher than the firm’s WACC. 2. It is not appropriate to use a firm’s WACC when a project that has the same systematic risk as the firm is not being financed using the same mix of debt and equity as the firm—for example, if a project will be financed entirely with equity. The project’s cash flows should be discounted using the cost of equity rather than the firm’s WACC. These two rates will be the same only if the firm has no debt. LO 4 Bloomcode: Comprehension AASCP: Analytic IMA: Corporate Finance AICPA: Industry/Sector Perspective
13.11 WACC: Capital Co. has a capital structure, based on current market values, that consists of 50 percent debt, 10 percent preferred stock, and 40 percent common stock. If the returns required by investors are 8 percent, 10 percent, and 15 percent for the debt, preferred equity, and common stock, respectively, what is Capital’s after-tax WACC? Assume that the firm’s marginal tax rate is 40 percent. Solution: WACC xdebtkdebt (1 t) x ps k ps xcs kcs = WACC= 0.5×0.08×(1- 0.4) + 0.1×0.10 + 0.4×0.15= 0.094, or 9.4%
LO 4 Bloomcode: Application AASCP: Analytic IMA: Corporate Finance AICPA: Industry/Sector Perspective
13.12 WACC: What are direct out-of-pocket costs? Solution: SM 19-627
Direct out-of-pocket costs are the actual out-of-pocket costs that a firm incurs when it raises capital. They include such things as fees paid to investment bankers and legal and accounting expenses. LO 4 Bloomcode: Knowledge AASCP: Analytic IMA: Corporate Finance AICPA: Industry/Sector Perspective
INTERMEDIATE 13.13 Finance balance sheet: Explain why the total value of all of the securities used to finance a firm must be equal to the value of the firm. Solution: The value of the firm’s assets is equal to the present value of the future cash flows expected to be generated by those assets. The cash flow claim on those assets is prioritized by the financing of those assets. Therefore, the financing claims on the assets of the firm fully account for the entire value of the assets, and the value of the financing claims must equal the value of the assets that are carved up by those claims. LO 1 Bloomcode: Comprehension AASCP: Analytic IMA: Corporate Finance AICPA: Industry/Sector Perspective
13.14 Finance balance sheet: Explain why the cost of capital for a firm is equal to the expected rate of return to the investors in the firm. Solution: If we view the firm as a conduit for the cash flows provided by the assets of the firm, then it is easy to see that the cash flows provided by the assets of the firm must equal the cash flows provided to the aggregate investor group of the firm. We also know that the capital invested in the firm must equal the capital invested by the firm. Therefore, we then know that the rate of return for the investors of the firm must equal the cost of capital provided to the firm. The expected return to investors will also equal the expected cost of capital for the firm. LO 1 Bloomcode: Comprehension SM 19-628
AASCP: Analytic IMA: Corporate Finance AICPA: Industry/Sector Perspective
13.15 Current cost of a bond: You know that the after-tax cost of debt capital for Bubbles Champagne Company is 7 percent. If the firm has only one issue of five-year bonds outstanding, what is the current price of the bonds if the coupon rate on those bonds is 10 percent? Assume the bonds make semiannual coupon payments and the marginal tax rate is 30 percent. Solution: We know the after-tax cost of debt, and from that we can find the pretax cost of debt by dividing by 1 minus the tax rate. This becomes 0.07 / (1 – 0.3) = 0.10 or 10%. Since the YTM on the bonds is equal to the coupon rate, then we know the bonds are priced at par, or $1,000. LO 2 Bloomcode: Application AASCP: Analytic IMA: Corporate Finance AICPA: Industry/Sector Perspective
13.16 Current cost of a bond: Perpetual Ltd. has issued bonds that never require the principal amount to be repaid to investors. Correspondingly, Perpetual must make interest payments into the infinite future. If the bondholders receive annual payments of $75 and the current price of the bonds is $882.35, what is the after-tax cost of this debt for Perpetual if the firm is subject to a 40 percent marginal tax rate? Solution: Since the bonds represent a perpetuity, we know that the pretax cost of debt can be solved using the following: Coupon Payment $75 = 0.085 kdebt = = Bond Price $882.35 And the after-tax cost is 0.085 × (1 - 0.4) = 0.051, or 5.1% LO 2 Bloomcode: Application AASCP: Analytic IMA: Corporate Finance AICPA: Industry/Sector Perspective
SM 19-629
13.17 Current cost of a bond: You are analyzing the cost of debt for a firm. You know that the firm’s 14-year maturity, 8.5 percent coupon bonds are selling at a price of $823.48. The bonds pay interest semiannually. If these bonds are the only debt outstanding, what is the after-tax cost of debt for this firm if it is subject to a 30 percent marginal and average tax rate? Solution: The current YTM for the bonds can be calculated as follows. $823.48 = $42.50 × PVIFA(28, YTM/2) + $1,000 × PVIF(28, YTM/2) Solving, we find that YTM = 0.11, and therefore the after-tax cost of debt is equal to: 0.11 × (1 – 0.3) = 0.077, or 7.7% LO 2 Bloomcode: Application AASCP: Analytic IMA: Corporate Finance AICPA: Industry/Sector Perspective
13.18 Taxes and the cost of debt: Holding all other things constant, does a decrease in the marginal tax rate for a firm provide incentive for the managers of a firm to increase or decrease its use of debt? Solution: The after-tax cost of debt for the firm is equal kDebt pretax x (1 – t). We can then calculate the tax benefit of using debt to be kDebt pretax x t. Therefore, the value of the tax benefit to debt increases with the marginal tax rate. If the marginal tax rate decreases, then the tax benefit to debt decreases as well. Therefore, the incentive to borrow actually decreases with a decrease in the marginal tax rate. LO 2 Bloomcode: Comprehension AASCP: Analytic IMA: Corporate Finance AICPA: Industry/Sector Perspective
13.19 Cost of debt for a firm: You are analyzing the after-tax cost of debt for a firm. You know that the firm’s 12-year maturity, 9.5 percent semiannual coupon bonds are selling at a price of $1,200. If these bonds are the only debt outstanding for the firm, what is the
SM 19-630
after-tax cost of debt for this firm if it has a marginal tax rate of 34 percent? What if the bonds are selling at par? Solution: The current YTM for the bonds can be calculated as follows. 1 1 n (1 i) Fn P C B n i (1 i) 1 1 (1 i)24 $1, 000 $1, 200 $47.50 24 i (1 i) Solving, we find that YTM = 0.07008 or 7.008% and therefore the after-tax cost of debt is equal to 0.07008 × (1 – 0.34) = 0.046253, or 4.63% If the bonds are priced at par, then the YTM on the bonds is 9.5 percent and then the after-tax cost of debt would be 6.27%. LO 2 Bloomcode: Application AASCP: Analytic IMA: Corporate Finance AICPA: Industry/Sector Perspective
13.20 Cost of common stock: Underestimated Inc.’s common shares currently sell for $36 each. The firm’s management believes that its shares should really sell for $54 each. If the firm just paid an annual dividend of $2 per share and management expects those dividends to increase by 8 percent per year forever (and this is common knowledge to the market), what is the current cost of common equity for the firm and what does management believe is the correct cost of common equity for the firm? Solution: The current cost of equity for the firm is D $2.00 1.08 0.08 0.14, or 14% kcs 1 g Pcs $36.00 But the firm believes that its cost of capital is more appropriately D $2.00 1.08 0.08 0.12, or 12% kcs 1 g Pcs $54.00
SM 19-631
LO 3 Bloomcode: Analysis AASCP: Analytic IMA: Corporate Finance AICPA: Industry/Sector Perspective
13.21 Cost of common stock: Write out the general equation for the price of the stock for a firm that will grow dividends very rapidly at a constant rate for the four years after the next dividend is paid and will grow dividends thereafter at a constant, but lower rate. Discuss the problems in estimating the cost of equity capital for such a stock. Solution: D D (1 g ) D (1 g )2 D (1 g )3 D (1 g )4 D (1 g )4(1 g ) 1 1 1 25 Pcs 11 k 21 11 k 31 11 k 41 1 k1 g 1 k 1 k (1 k )5 cs
cs
cs
cs
cs
cs
2
cs
It is easy to see that in order to solve for a cost of capital, kcs, you must have a good idea of what g1 and g2 are. If those growth rates are poor estimates, then the calculation for kcs, will also be a poor estimate. LO 3 Bloomcode: Comprehension AASCP: Analytic IMA: Corporate Finance AICPA: Industry/Sector Perspective
13.22 Cost of common stock: You have calculated the cost of common stock using all three methods described in this chapter. Unfortunately, all three methods have yielded different answers. Describe which answer (if any) is most appropriate. Solution: Two of the methods involve an estimate of the growth rate in dividends for the firm. If you are confident in your estimate of the growth rate, then those methods might be most appropriate. Otherwise, utilizing the CAPM, which does not involve any dividend growth rate estimates, would probably be the best. You may choose to average the results of all three methods. LO 3 Bloomcode: Analysis AASCP: Analytic IMA: Corporate Finance AICPA: Industry/Sector Perspective
SM 19-632
13.23 WACC: The managers of a firm financed entirely with common stock are evaluating two distinct projects. The first project has a large amount of unsystematic risk and a small amount of systematic risk. The second project has a small amount of unsystematic risk and a large amount of systematic risk. Which project, if taken, is more likely to increase the firm’s cost of capital? Solution: Markets adjust the cost of capital according to the level of systematic risk in a project. Therefore, the project with the greatest level of systematic risk will have the greatest positive impact on the cost of capital for the firm, even if it has the lowest level of unsystematic risk. LO 4 Bloomcode: Analysis AASCP: Analytic IMA: Corporate Finance AICPA: Industry/Sector Perspective
13.24 WACC: The Imaginary Products Co. currently has debt with a market value of $300 million outstanding. The debt consists of 9 percent coupon bonds (semiannual coupon payments) which have a maturity of 15 years and are currently priced at $1,440.03 per bond. The firm also has an issue of 2 million preferred shares outstanding with a market price of $12.00 per share. The preferred shares pay an annual dividend of $1.20. Imaginary also has 14 million shares of common stock outstanding with a price of $20.00 per share. The firm is expected to pay a $2.20 common dividend one year from today, and that dividend is expected to increase by 5 percent per year forever. If Imaginary is subject to a 40 percent marginal tax rate, then what is the firm’s weighted average cost of capital? Solution: Step 1: Total amount of debt, common equity, and preferred equity: Debt = $300,000,000 (given) Preferred equity = $12 × 2,000,000 = $24,000,000 Common equity = $20 × 14,000,000 = $280,000,000 Total capital = $604,000,000 xDebt = 300 million/604 million = 0.4967 xps = 24 million/604 million = 0.0397 xcs = 280 million/604 million= 0.4636 Step 2: Cost of capital components: Cost of debt: $1,440.03 = $45 × PVIFA (30, YTM / 2) + $1,000 × PVIF (30, YTM / 2) SM 19-633
Solving, we find that YTM = 0.0484 (this is a pretax number). Cost of preferred equity: D k $1.20 0.10 ps
Pps $12.00 Cost of common equity: D k 1 g $2.20 0.05 0.16 cs
Pcs $20.00 Step 3: Combine using the WACC formula. WACC xdebtkdebt (1 t) x ps k ps xcs kcs = WACC 0.4967 0.0484 (1 0.4)0.0397 0.100.4636 0.16 0.0926, or 9.26% LO 4 Bloomcode: Application AASCP: Analytic IMA: Corporate Finance AICPA: Industry/Sector Perspective
13.25 Choosing a discount rate: For the Imaginary Products firm in Problem 13.24, calculate the appropriate cost of capital for a new project that is financed with the same proportion of debt, preferred shares, and common shares as the firm’s current capital structure. Also assume that the project has the same degree of systematic risk as the average project that the firm is currently undertaking (the project is also in the same general industry as the firm’s current line of business). Solution: Since Imaginary will be financing the project with the same mix of capital that the firm is currently utilizing for its projects, we will have met the first restriction concerning financing mix. In addition, the new project will have the same degree of systematic risk (in addition to being in the same general line of business). Therefore, Imaginary can use the 9.26 percent cost of capital to evaluate its project. LO 1 Bloomcode: Analysis AASCP: Analytic IMA: Corporate Finance AICPA: Industry/Sector Perspective
13.26 Choosing a discount rate: If a firm’s management anticipates financing a project with a capital mix that is different from its current capital structure, describe how the firm is
SM 19-634
subjecting itself to a calculation error if its historical WACC is used to evaluate the project. Solution: Since the firm is financing the project with a different capital mix than it has historically used, we know that the weights and rates for debt, preferred, and common shares in the WACC formula will be different. We know that the cost of capital for each component is a function of the individual weights and rates, and that the WACC will be different for the overall firm versus that of the individual project. Therefore, using its historical WACC can result in an error in the NPV estimate for the project. LO 1 Bloomcode: Comprehension AASCP: Analytic IMA: Corporate Finance AICPA: Industry/Sector Perspective
ADVANCED 13.27 You are analyzing the cost of capital for MacroSwift Corporation, which develops software operating systems for computers. The firm’s dividend growth rate has been a very constant 3 percent per year for the past 15 years. Competition for the firm’s current products is expected to develop in the next year, and MacroSwift is currently expanding its revenue stream into the multimedia industry. Evaluate the appropriateness of continuing to use a 3 percent growth rate in dividends for MacroSwift in your cost of capital model. Solution: While the growth in dividends has been extremely constant for Macroswift over the last 15 years, it is appropriate to assume a constant-growth rate only if that same rate will continue in the future. Two factors will act to alter that growth in the future. MacroSwift will have competition for its current product list in the near future, and that could alter the firm’s growth rate. In addition, the firm is expanding its product line into an area that will probably not yield the same level of growth. It is therefore, unlikely that MacroSwift’s dividend growth rate will continue at a 3 percent annual rate. This suggests that we should consider something other than constant growth in our modeling. LO 3 Bloomcode: Analysis AASCP: Analytic IMA: Corporate Finance SM 19-635
AICPA: Industry/Sector Perspective
13.28 You are an external financial analyst evaluating the merits of a stock. Since you are using a dividend discount model approach to evaluate a cost of equity capital, you need to estimate the dividend growth rate for the firm in the future. Describe how you might go about doing this. Solution: One source for this data would be to measure the firm’s dividend growth rate in recent history. If we could assume that such a growth in dividends will continue into the future, then our measure would be reasonable. One additional source would be to read a financial analyst’s report in which the author of the report may have a better estimate of the firm’s future prospects. LO 3 Bloomcode: Application AASCP: Analytic IMA: Corporate Finance AICPA: Industry/Sector Perspective
13.29 You know that the return of Momentum Cyclicals common shares is 1.6 times as sensitive to macroeconomic information as the return of the market. If the risk-free rate of return is 3.72 percent and the market risk premium is 5.71 percent, what is Momentum Cyclicals’ cost of common equity capital? Solution: We know that the beta for Momentum Cyclicals is 1.6, and we can use the remaining information in the CAPM as follows: 𝐸(𝑅𝐶𝑆 ) = 𝑅𝑟ƒ + 𝛽(𝐸(𝑅𝑚 ) − 𝑅𝑟ƒ ) = 0.0372 + 1.6(0.0.0571) = 0.12856 = 12.86% LO 3 Bloomcode: Application AASCP: Analytic IMA: Quantitative Methods AICPA: Industry/Sector Perspective
13.30 In your analysis of the cost of capital for a common stock, you calculate a cost of capital using a dividend discount model that is much lower than the calculation for the cost of capital using the CAPM model. Explain a possible source for the discrepancy. SM 19-636
Solution: Comparing the two formulas for the two methods, we have:
E(Rcs ) = Rrf + β E(R ) - R m rf
and k DP g 1
cs
cs
Given these two sources of information, we see that the only variable that we are not able to get directly from the market is the growth rate in dividends (note that future dividends are also a function of this growth rate), which is an estimate. Since our dividend discount method provided a lower cost of capital than the CAPM, it seems likely that we estimated the growth rate lower than what the aggregate market has assumed. Of course, this assumes that the market is efficiently pricing the stock. If the market price of the stock is mispriced, then this might lead to a difference. LO 3 Bloomcode: Analysis AASCP: Analytic IMA: Quantitative Methods AICPA: Industry/Sector Perspective
13.31 RetRyder Hand Trucks has a preferred share issue outstanding that pays a dividend of $1.30 per year. The current cost of preferred equity for RetRyder is 9 percent. If RetRyder issues additional preferred shares that pay exactly the same dividend and the investment banker retains 8 percent of the sale price, what is the cost of new preferred shares for RetRyder? Solution: The current cost of preferred shares for RetRyder is D $1.30 $14.44 Pps kps 0.09 and then RetRyder would receive 92 percent of the proceeds. We could then adapt the cost of preferred equity to the following: D $1.30 =0.0978, or 9.78% $1.30 k = = ps Pps (1- F) $14.44(1-0.08) $13.29 LO 3 Bloomcode: Application AASCP: Analytic IMA: Corporate Finance AICPA: Industry/Sector Perspective
SM 19-637
13.32 Enigma Corporation’s management believes that the firm’s cost of capital (WACC) is too high because the firm has been too secretive with the market concerning its operations. Evaluate that statement. Solution: The WACC is a function of the perceived risk involved in the cash flows of the projects that the firm is currently operating. If the market perceives that risk to be higher than the actual risk due to a lack of information concerning those projects, then the firm might be able to lower that perceived risk by sharing more information with the market. That could have the effect of lowering the firm’s WACC. LO 4 Bloomcode: Analysis AASCP: Analytic IMA: Corporate Finance AICPA: Industry/Sector Perspective
13.33 Discuss what valuable information would be lost if you decided to use book values in order to calculate the cost of each of the capital components within a firm’s capital structure. Solution: Market returns are impounded in market prices. If those prices are ignored, then the efficiency of the market’s information process is essentially thrown away. Since the market adjusts securities prices according to the expected return for investing in a security, then ignoring that information is the same as ignoring what the market deems to be an appropriate cost of capital for the firm. LO 4 Bloomcode: Analysis AASCP: Analytic IMA: Corporate Finance AICPA: Industry/Sector Perspective
13.34 Hurricane Corporation is financed with debt, preferred equity, and common equity with market values of $20 million, $10 million, and $30 million, respectively. The betas for the debt, preferred stock, and common stock are 0.2, 0.5, and 1.1, respectively. If the risk-free rate is 3.72 percent, the market risk premium is 5.71 percent, and Hurricane’s average and marginal tax rates are both 30 percent, what is the company’s weighted average cost of capital? Solution: SM 19-638
The fractions of the total financing represented by debt, common equity, and preferred equity are: Debt = $20 million Preferred equity = $10 million Common equity = $30 million Total capital = $60 million xdebt = $20/$60 = 0.3333 xps = $10/$60 = 0.1667 xcs = $30/$60 = 0.5000 The costs of debt, common equity, and preferred equity are: E(RDebt) = Returnrf + βDebt (E(ReturnM) – Returnrf) = 0.0372 + (0.2 × 0.0571) = 0.04862, or 4.862% E(Rps) = Returnrf + βps (E(ReturnM) – Returnrf) = 0.0372+ (0.5 × 0.0571) = 0.06575, or 6.575% E(Rcs) = Returnrf + βcs (E(ReturnM) – Returnrf) = 0.0372+ (1.1 × 0.0571) = 0.10001 or 10.001% WACC = (0.3333 x 0.04862 x (1-0.3)) + (0.1667 x0.06575) + (0.5 x 0.10001) = 0.072308, or 7.2308% LO 4 Bloomcode: Application AASCP: Analytic IMA: Quantitative Methods AICPA: Industry/Sector Perspective
13.35 You are working as an intern at Coral Gables Products, a privately owned manufacturing company. Shortly after you read Chapter 13 in this book, you got into a discussion with the Chief Financial Officer (CFO) at Coral Gables about weighted average cost of capital calculations. She pointed out that, just as the beta of the assets of a firm equals a weighted average of the betas for the individual assets, as shown in Equation 7.11: n
n Asset portfolio xii x11 x2 2 x33 ... xnn i1
the beta of the assets of a firm also equals a weighted average of the betas for the debt, preferred stock, and common stock of a firm: n
n Asset portfolio xii xDebt Debt xps ps xcs cs i 1
Why must this be true? Solution: Since, collectively, the debt and equity holders are entitled to receive all of the cash flows that the assets of the firm are expected to produce, the systematic risk of the cash flows
SM 19-639
that they are entitled to receive must be the same as the systematic risk of the cash flows the assets are expected to produce. LO 1 Bloomcode: Analysis AASCP: Analytic IMA: Quantitative Methods AICPA: Industry/Sector Perspective 13.36 The CFO described in Problem 13.35 asks you to estimate the beta for Coral Gables’s common stock. Since the common stock is not publicly traded, you do not have the data necessary to estimate the beta using regression analysis. However, you have found a company with publicly traded stock that has operations exactly like those at Coral Gables. Using stock returns for this pure-play comparable firm, you estimate the beta for the comparable company’s stock to be 1.06. The market value of that company’s common equity is $45 million, and it has one debt issue outstanding with a market value of $15 million and an annual pretax cost of 4.85 percent. The comparable company has no preferred stock. a. If the risk-free rate is 3.72 percent and the market risk premium is 5.71 percent, what is the beta of the assets of the comparable company? b. If the total market value of Coral Gables’s financing consists of 35 percent debt and 65 percent equity (this is what the CFO estimates the market values to be) and the pretax cost of its debt is 5.45 percent, what is the beta for Coral Gables’s common stock? Solution: You can solve this problem using the equation: n
n Asset portfolio xii xDebt Debt xps ps xcs cs i 1
Since neither company has preferred stock, the equation in this case has only two terms: n Asset portfolio xDebt Debt xcs cs a. To calculate the beta of the assets of the comparable company, you must first estimate the beta of the comparable firm’s debt and the fractions of the total market value of the comparable firm’s financing which is represented by the debt and equity. The beta of the comparable firm’s debt can be estimated using the CAPM formula (Equation 7.10): E(Ri) = Rrf + βi[E(Rm) – Rrf] Since this formula can be used to calculate the expected return on debt, it can be written as: SM 19-640
E(RDebt) = Rrf + βDebt[E(Rm) – Rrf] If we assume that the market is pricing the debt correctly, the pretax cost of debt equals the expected return on debt in this equation. Therefore, substituting the information from the problem statement into this equation yields: 4.85% = 3.72 % + βDebt[5.71 %] and the beta of the debt equals: βDebt = (4.85% - 3.72 %) / 5.71% = 0.198 The fractions of the total financing represented by debt and common equity are: Debt = $15 million Common equity = $45 million Total capital = $60 million xDebt = $15/$60 = 0.25 xcs = $45/$60 = 0.75 Finally, the beta of the assets of the comparable company is: βnAsset portfolio = xDebtβDebt + xcsβcs = (0.25 x 0.198) + (0.75 x 1.06) = 0.8445 b. Since the comparable company is in exactly like Coral Gables, this is a good estimate of asset beta for Coral Gables. To calculate the beta for Coral Gables’s common equity, you use the following equation with the beta of the assets for the comparable company and the beta of debt and financing fractions for Coral Gables.
n Asset portfolio xDebt Debt xcs cs The beta of the Coral Gables debt is: E(RDebt) = Rrf + βDebt[E(Rm) – Rrf] 5.45% = 3.72% + βDebt[5.71%] βDebt = (5.45% - 3.72%) / 5.71% = 0.3030 and the beta of the firm’s equity is: βn Asset portfolio = xDebtβDebt + xcsβcs 0.8445 = (0.35 x 0.3030) + (0.65 x βCS)
SM 19-641
βcs =
0.8445 − (0.35 𝑥 0.3030) 0.7384 = 0.65 = 1.136 0.65
LO 1, LO 3 Bloomcode: Application AASCP: Analytic IMA: Quantitative Methods AICPA: Industry/Sector Perspective
13.37 Estimate the weighted average cost of capital for Coral Gables using your estimated beta and the information in the problem statement in Problem 13.36? Assume that the average and marginal tax rates for Coral Gables are both 25 percent. Solution: The cost of equity is: E(Rcs) = Rrf + βcs[E(Rm) – Rrf] E(Rcs) = 3.72% + (1.136 × 5.71%) = 10.21% The weighted average cost of capital is: WACC = (0.35 x 5.45% x (1-0.25)) + (0.65 x 10.21%) = 0.080651 = 8.07% LO 3, LO 4 Bloomcode: Application AASCP: Analytic IMA: Quantitative Methods AICPA: Industry/Sector Perspective
CFA Problems
a. b. c. d.
13.38 The cost of equity is equal to the: Expected market return. Rate of return required by stockholders. Cost of retained earnings plus dividends. Risk the company incurs when financing.
Solution: b is correct. The cost of equity is defined as the rate of return required by stockholders. LO 1 Bloomcode: Knowledge AASCP: Analytic IMA: Corporate Finance AICPA: Industry/Sector Perspective
SM 19-642
13.39 Dot.Com has determined that it could issue $1,000 face value bonds with an 8 percent coupon paid semiannually and a five-year maturity at $900 per bond. If Dot.Com’s marginal tax rate is 38 percent, its after-tax cost of debt is closest to: a. 6.2 percent. b. 6.4 percent. c. 6.6 percent. d. 6.8 percent. Solution: c is correct. FV = $1,000; PMT = $40; N = 10; PV = $900 Solve for i. The six-month yield, i, is 5.3149% YTM = 5.3149% 2 = 10.6298% kDebtpretax (1 – t)= 10.6298% × ( 1 – 0.38) = 6.5905% LO 2 Bloomcode: Application AASCP: Analytic IMA: Corporate Finance AICPA: Industry/Sector Perspective
13.40 Morgan Insurance Ltd. issued a fixed-rate perpetual preferred stock three years ago and placed it privately with institutional investors. The stock was issued at $25.00 per share with a $1.75 dividend. If the company were to issue preferred stock today, the yield would be 6.5 percent. The stock’s current value is: a. $25.00. b. $26.92. c. $37.31. d. $40.18.
Solution: b is correct. The company can issue preferred stock at 6.5%. Pps= $1.75/0.065 = $26.92 Note: Dividends are not tax deductible so there is no adjustment for taxes. LO 3 Bloomcode: Application AASCP: Analytic IMA: Corporate Finance SM 19-643
AICPA: Industry/Sector Perspective 13.41 The Gearing Company has an after-tax cost of debt capital of 4 percent, a cost of preferred stock of 8 percent, a cost of equity capital of 10 percent, and a weighted average cost of capital of 7 percent. Gearing intends to maintain its current capital structure as it raises additional capital. In making its capital-budgeting decisions for the average-risk project, the relevant cost of capital is: a. 4 percent. b. 7 percent. c. 8 percent. d. 10 percent. Solution: b is correct. The weighted average cost of capital, using weights derived from the current capital structure, is the best estimate of the cost of capital for the average-risk project of a company. LO 4 Bloomcode: Application AASCP: Analytic IMA: Corporate Finance AICPA: Industry/Sector Perspective
13.42 Suppose the cost of capital of the Gadget Company is 10 percent. If Gadget has a capital structure that is 50 percent debt and 50 percent equity, its before-tax cost of debt is 5 percent, and its marginal tax rate is 20 percent, then its cost of equity capital is closest to: a. 10 percent. b. 12 percent. c. 14 percent. d. 16 percent. LO 1 Bloomcode: Application AASCP: Analytic IMA: Corporate Finance AICPA: Industry/Sector Perspective Solution: c is correct. re = ra + (ra – rd) (D/E) x (1-t) Note: If D/(D + E) = 0.50, then D/E = 1.0 SM 19-644
re = 0.10 + (0.10 – 0.05)(1.0)(1 – 0.2) re = 0.10 + [0.05(0.80)] = 0.10 + 0.04 = 0.14, or 14%
Sample Test Problems 13.1
Howard Power and Telecommunications Corporation has three divisions. The names of these divisions, along with the after-tax cost of capital for each division and the market value of the assets in each division are as follows: (LO 1) Division Name Infrastructure development Power Telecommunications
Cost of Capital 8.75 7.50 8.25
MV of Assets $250,000,000 $325,000,000 $675,000,000
What is the overall after-tax cost of capital for Howard Power and Telecommunications? (LO 1)
Solution: The overall after-tax cost of capital for a firm is a value-weighted average of the after-tax cost of capital for the projects (assets) in that firm. We can calculate the overall after-tax cost of capital using Equation 13.2 as follows: n
kFirm xiki x1k1 x2k2 x3k3 ... xnkn . i1
$250,000,000 + $325,000,000 + $675,000,000 = $1,250,000 x1 = $250,000 / $1,250,000 = 0.20 x2 = $325,000 / $1,250,000 = 0.26 x1 = $675,000 / $1,250,000 = 0.54 (0.20 × 0.0875) + (0.26 × 0.0750) + (0.54 × 0.0825) = 0.0816, or 8.16% Bloomcode: Application AASCP: Analytic IMA: Quantitative Methods AICPA: Industry/Sector Perspective
13.2
Quarri Industries has eight percent coupon bonds outstanding. These bonds have a market price of $954.41, pay interest semiannually, and will mature in 6 years. If the tax rate is 35 percent, what are the pre-tax cost and after-tax cost of this debt? (LO 2)
SM 19-645
Solution: $40 $1,000 1 $954.41= × 1 + kDebt pretax 12 1+ k Debt pretax 1+ k Debt pretax 12 kDebt pretax = 0.045, or 4.5% The annualized pretax cost is: kDebt pretax = 1 0.045 -1= 0.0920, or 9.20% 2
and the after-tax cost is: kDebt after-tax = 0.0920×(1- 0.035) = 0.0598, or 5.98% Bloomcode: Application AASCP: Analytic IMA: Quantitative Methods AICPA: Industry/Sector Perspective
13.3
Quarri Industries has a beta of 1.6. If the market risk free rate is 4 percent and the expected return on the market 9 percent, what is Babbitt’s cost of common stock? (LO 3) Solution:
kcs = Rrf + βcs (E(RM) – Rrf) kcs = 0.04 + 1.6 × (0.09 - 0.04) = 0.12, or 12% Bloomcode: Application AASCP: Analytic IMA: Quantitative Methods AICPA: Industry/Sector Perspective 13.4 Miron’s Copper Corp. management expects its common stock dividends to grow 1.5 percent per year for the indefinite future. The firm’s shares are currently selling for $18.45, and the firm just paid a dividend of $3.00 yesterday. What is the cost of common stock for this firm? (LO 3) D0 1+ g D D g P0 1 kcs 1 g kcs g P0 P0 $3.001 0.015 0.015 0.18, or18% kcs $18.45 Bloomcode: Application AASCP: Analytic IMA: Corporate Finance SM 19-646
AICPA: Industry/Sector Perspective
13.5
Use the information in questions 13.2 and 13.3 as well as the following information to compute the WACC for Quarri Industries. In addition to common stock, Quarri has 500,000 preferred shares outstanding that pay a quarterly dividend of $0.50 per share and are currently trading for $20.00 a share. The company’s outstanding bonds have a face value of $209,553,546. There are 2 million shares of common stock outstanding with a current market price of $98.00 per share. (LO 3 and LO 4)
SM 19-647
Solution: The total value of the outstanding debt, preferred stock and common stock are as follows: Debt = $209,553,546 × 0.95441 = $200,000,000 Preferred stock = $20 × 500,000 = $10,000,000 Common stock = $98 × 2,000,000 = $196,000,000 Total capital = $406,000,000 xDebt = $200/$406 = 0.4926 xps = $10/$406 = 0.0246 xcs = $196/$406 = 0.4828 The costs for the individual types of capital are: After-tax cost of debt (from problem 13.2): kDebt after-tax = 5.98% Cost of preferred stock: D 4$0.50 k = = = 0.10, or 10% ps $20 Pps Cost of common equity (from problem 13.3): kcs = 12% WACC calculation: WACC= xDebt kDebtafter-tax xpskps xcskcs WACC = (0.4926 × 0.0598) + (0.0246 × 0.10) + (0.4828 × 0.12) WACC = 0.0899, or 8.99% Bloomcode: Application AASCP: Analytic IMA: Corporate Finance AICPA: Industry/Sector Perspective
13.6
Staunton Energy Corporation managers are considering a capital budgeting project to replace some machinery used in one of the company’s oil refineries. Is the company’s WACC the appropriate discount rate to use in the NPV analysis of this project? (LO 4) Solution:
If the systematic risk of the individual project is the same as the systematic risk of the portfolio of projects that comprise the company as a whole, and the project is likely to be financed with the same financing mix as the company, then company’s WACC is the appropriate discount rate to use. However, since a project involving replacement of machinery is probably less risky than the overall company, the company’s WACC might be too high of a discount rate. If so, using the WACC as the discount rate could cause the company managers to underestimate the NPV of this project. They should probably use a lower discount rate to more accurately reflect the systematic risk of the project. Bloomcode: Analysis SM 19-648
AASCP: Analytic IMA: Corporate Finance AICPA: Industry/Sector Perspective
CHAPTER 14 Working Capital Management Before You Go On Questions and Answers Section 14.1 1.
How do you calculate net working capital, and why is it important?
Net working capital is calculated as the difference between the current assets and current liabilities. It is important for a firm to keep a positive net working capital balance, as these funds are used to cover the day-to-day expenses and short-term liabilities as they come due. 2.
What are some of the trade-offs required in the management of working capital accounts?
When managing working capital accounts, a financial manager is looking to delay paying accounts payable as long as possible without suffering any penalties, maintain minimal finished goods inventories without losing sales, and collect cash payments on accounts receivable as fast as possible to close the loop, among other things.
Section 14.2 1.
What is the operating cycle, and how is it related to the cash conversion cycle? The operating cycle starts with the receipt of raw materials and ends with the collection of cash from customers for the sale of finished goods. The operating cycle can be described in terms of two components: days’ sales in inventory and days’ sales outstanding. The cash conversion cycle is the length of time between the actual cash outflow for materials and the actual cash inflow from sales. To calculate it, we need all of the information used to calculate the operating cycle plus one additional measure: days’ payables outstanding.
Section 14.3 1.
What are the two general current asset management strategies discussed in this section, and how do they differ? Typically, the two main current asset investment strategies are flexible and restrictive strategies. The flexible strategy prompts management to keep large balances of cash, marketable securities, SM 19-649
and inventory. This strategy is perceived to be a relatively low-risk and low-return course of action for management to follow. The restrictive strategy, on the other hand, prompts management to keep the usage of current assets to a minimum and is perceived to be high risk and high return. 2.
What are the types of costs associated with each of these strategies? The flexible strategy is associated with a high level of carrying costs because of a firm’s high levels of inventory and providing liberal credit for customers. The restrictive strategy is associated with shortage costs, which can be either financial or operating in nature.
Section 14.4 1.
What does ―4/15, net 30‖ mean? If a company declares a ―4/15, net 30‖ means of sale, it signifies that it will grant the customer a 4 percent discount if the customer pays the full amount within 15 days of the invoice date. Otherwise the customer has 30 days from the date of the delivery to pay the full amount.
2.
What is an aging schedule, and what is its purpose? The aging schedule for a firm lists the accounts receivable broken down by the number of days until they are due or past due. Firms use aging schedules to keep track of their accounts receivables and to assess how effective they are collecting on these accounts.
Section 14.5 1.
What is the economic order quantity model? The economic order quantity model is an inventory management tool that mathematically determines the minimum total inventory cost, taking into account reorder costs and inventorycarrying costs. The main objective of the model is to find the trade-off between the two costs.
2.
Why can cash investment in inventory be costly? Investment in inventory is considered costly, because inventory must be stored, which results in rental and maintenance costs. Furthermore, inventory on hand is subject to loss and theft, and faces the possibility of becoming obsolete. Finally, investment in inventory provides no return unlike an investment in financial or real assets would.
Section 14.6 1.
What is float? SM 19-650
The collection time, which is the time between when a customer makes a payment and the time the cash becomes available is to the firm, is also referred to as float. 2.
Explain how lockboxes are used.
Lockboxes are post office boxes set up by the firm for its customers to deliver their payments to these boxes instead of the firm’s business address. The post office then collects these payments and delivers them to the bank. The main purpose of lockboxes is to minimize collection time for firms through cutting down on postal time and through processing the payments directly at the bank.
Section 14.7 1.
List and briefly describe the three main short-term financing strategies.
The three main short-term financing strategies are (1) self-liquidating strategy, in which the maturity of the liabilities is matched with that of assets; (2) conservative strategy, which relies more heavily on long-term financing; and (3) restrictive strategy, which relies primarily on shortterm financing. 2.
What are the advantages and disadvantages of short-term financing?
Short-term financing offers companies greater flexibility and usually a lower cost of capital. On the other hand, short-term financing often comes with some illiquidity problems as well as uncertainty due to increased exposure to interest rate fluctuations. 3.
Give some examples of sources of short-term financing.
Examples of short-term financing include accounts payable, short-term bank loans, informal lines of credit, formal lines of credit, or commercial paper. The two main current asset investment strategies are (1) flexible strategy, which encourages management to keep large balances of current assets, and (2) restrictive strategy, which keeps the usage of current assets to a minimum.
SM 19-651
Self-Study Problems 14.1
You are provided the following working capital information for the Blue Ridge Company: Account Beginning Balance Ending Balance Inventory $ 2,600 $2,890 Accounts receivable 3,222 2,800 Accounts payable 2,500 2,670 Net sales 24,589 Cost of goods sold 19,630 If all sales are made on credit, what are the firm’s operating and cash conversion cycles? Solution: We calculate the operating and cash conversion cycles for Blue Ridge as follows: Inventory = $2,890 Accounts receivable = $2,800 Accounts payable = $2,670 Net sales = $24,589 Cost of goods sold = $19,630 DSI Inventory $2,890 53.7 days COGS 365 $19,630 365 Accounts receivable $2,800 = 41.6 days DSO = = Credit sales 365 $24,589 365 Accounts payable $2,670 = 49.6 days = COGS 365 $19,630 365 Operating cycle = DSI + DSO = 53.7 days + 41.6 days = 95.3 days Cash conversion cycle = DSI + DSO – DPO = 53.7 days + 41.6 days – 49.6 days = 45.7 days DPO =
14.2
Merrifield Cosmetics calculates that its operating cycle for last year was 76 days. The company had $230,000 in its accounts receivable account and had sales of $1.92 million. Approximately how many days does it take from the time the raw materials are received at Merrifield until the finished products they are used to produce are sold?
SM 19-652
Solution: Merrifield’s days sales in inventory is calculated as follows: Operating cycle = 76 days Accounts receivables = $230,000 Net sales = $1,920,000 Accounts receivable s $230,000 DSO 43.7 days Credit sales 365 $1,920,000 365 Operating cycle = DSI + DSO 76 days = DSI + 43.7 days DSI = 32.3 days Merrifield Cosmetics takes 32.3 days to turn its inventory into sales.
14.3
Below is a partial aging of accounts receivable for Bitar Roofing Services. Fill in the rest of the information and determine Bitar’s days’ sales outstanding. How does it compare to the industry average of 40 days? Age of Accounts (in days) Value of Account ($) % of Total Account 0-10 $211,000 11-30 120,360 31-45 103,220 46-60 72,800 Over 60 23,740 Total $531,120 Solution: The missing information for Bitar Roofing and its days’ sales outstanding are as follows: Bitar Roofing Age of Accounts (in days) Value of Account ($) % of Total Account 0-10 $211,000 39.7% 11-30 120,360 22.7 31-45 103,220 19.4 46-60 72,800 13.7 Over 60 23,740 4.5 Total $531,120 100%
SM 19-653
Effective DSO (0.397 10 days) (0.227 30 days) (0.194 45 days) (0.137 60 days) (0.045 365 day 3.97 days 6.81 days 8.73 days 8.22 days 16.43 days 44.2 days Bitar takes about 4 days more than the industry average of 40 to collect on its receivables. The firm should focus collection efforts on all credit sales that take 60 days or more to collect.
14.4
By obtaining a lockbox, Nizam’s Manufacturing was able to reduce its total cash collection time by two days. The firm has annual sales of $570,000 and can earn 4.75 percent annual interest. Assuming that the lockbox costs $50 per year, calculate the savings that can be attributed to the lockbox. Solution: The savings that can bed attributed to Nizam’s lockbox are: Annual sales = $570,000 Annual interest rate = 4.75% Annual cost of lockbox = $50 Collection time saved = 2 days $570,000 Average daily sales $1,561.64 365 Savings $1,561.64 0.0475 2 $50 $98.36 The firm can save $98.36 each year by using the lockbox.
14.5 Rockville Corporation is going to borrow $250,000 from its bank at an APR of 8.5 percent. The bank requires its customers to maintain a 10 percent compensating balance. What is the effective interest rate on this bank loan? Solution: The effective rate on Rockville Corporation’s loan is calculated as follows: Rockville Corporation’s loan information is as follows: Amount to be borrowed = $250,000 Stated annual interest rate = 8.5% Compensating balance = 10% Amount deposited as compensating balance = $250,000 × 0.10 = $25,000 Effective borrowing amount equal to $250,000 − $25,000 = $225,000 Interest expense = $250,000 × 0.085 = $21,250 Interest expense $21,250 Effective interest rate 9.44% Effective borrowing amount $225,000 SM 19-654
By setting aside a compensating balance of 10 percent or $25,000 on the loan, the firm increases its interest rate effectively to 9.44 percent.
Discussion Questions 14.1
What factors must a financial manager consider when making decisions about accounts receivable?
When dealing with accounts receivable, important decisions for the financial manager include the amount of credit offered to various customers and the term of the credit. Financial managers should keep close track of both the aging schedule and the effective DSO. If either or both show consistent deterioration, it may be time to reconsider the firm’s credit policy or the characteristics of its customers. In addition, in some industries, sales vary by season. Managers must be aware of seasonal patterns and make the necessary adjustments before drawing any conclusions about firm’s receivables. LO: 4 Level: Basic Bloomcode: Comprehension AASCP: Analytic IMA: Decision Analysis AICPA: Industry/Sector Perspective
14.2
List some of the working capital management practices you would expect to see in a manufacturing company following just-in-time inventory practices. Manufacturing companies like Dell are likely to do an exceptional job of managing their inventory and collecting on their receivables. Dell employs a strategy similar to just-in-time management where they maintain just sufficient inventory to meet the needs for a very short time. This saves the firm a huge investment in inventory. Thus their days’ sales in inventory (DSI) will be very low compared to other industries. Similar to Dell, firms will have a short collection period, and their operating cycle will be much lower than firms in other industries. If other computer manufacturing firms follow the Dell operating philosophy, they will extend their days’ payable outstanding (DPO) to the point that their cash conversion cycle is negative. In other words, instead of having to invest in its working capital, these firms will end up taking more time to pay their suppliers than the time taken to produce, sell, and collect on the receivables. LO: 5
SM 19-655
Level: Basic Bloomcode: Knowledge AASCP: Analytic IMA: Strategic Planning AICPA: Industry/Sector Perspective
14.3
What costs would a firm following a flexible current asset management strategy consider, and why? The strategy’s downsides can include the high carrying costs associated with owning a high level of inventory and providing liberal credit terms to customers. By investing in current assets, management foregoes the higher rate of return it could have earned by investing in long-term assets. Therefore, there is an opportunity cost involved when investing in current assets. Furthermore, large investments in some types of inventory can require significant storage, tax, and insurance costs, which can be expensive. LO: 3 Level: Intermediate Bloomcode: Comprehension AASCP: Analytic IMA: Strategic Planning AICPA: Industry/Sector Perspective
14.4
How are customers and suppliers affected by a firm’s working capital management decisions? Customers want firms to maintain large finished goods inventories because when they go to make a purchase, the item they want will likely be in stock. In general, large inventory helps stimulate sales and increase customer satisfaction, but that can be a costly item on a firm’s balance sheet. Management’s decisions on the firm’s receivables policy is driven by the industry type. Companies selling perishable products, such as food companies, might ask for payment in full in less than 10 days. On the other hand, if the firm is selling durable goods, the terms of credit are likely to be more generous. The terms of sale are also affected by the creditworthiness of the customer. If the firm is confident that it will be paid, it is far more likely to extend credit than if there was some doubt about payment. If the customer is a particularly large firm or if there is a likelihood of repeat business, then extending credit may be part of the marketing effect to secure the order. Thus, when the financial manager makes a decision to increase working capital, good things are likely to happen to the firm—sales should increase, relationships with vendors and suppliers should improve, and work or manufacturing stoppages should be less likely. SM 19-656
LO: 4 Level: Intermediate Bloomcode: Comprehension AASCP: Analytic IMA: Strategic Planning AICPA: Industry/Sector Perspective
14.5
A beverage bottling company in Vermont has days’ sales outstanding of 23.7 days. Is this good? Explain. In general, a lower DSO reflects the fact that the firm is managing its receivables very well. However, it is not possible to decide whether a DSO of 23.7 days is good or bad unless you have a basis for comparison. That basis of comparison could be a peer group, historical data for the firm itself, or targets set by the management. LO: 2 Level: Basic Bloomcode: Analysis AASCP: Analytic IMA: Strategic Planning AICPA: Industry/Sector Perspective
14.6
How do the following circumstances affect the cash conversion cycle: (a) favorable credit terms allow the firm to pay its accounts payable slower, (b) inventory turnover increases, and (c) accounts receivable turnover decreases? (a) (b) (c)
Favorable credit terms from suppliers allow the firm to use the suppliers’ funds to finance their working capital. It also reduces the firm’s cash conversion cycle. An increase in the inventory turnover results into decrease in DSI, which reduces both the firm’s operating cycle and the cash conversion cycle. As the accounts receivable turnover or Day Sales Outstanding (DSO) decreases, the firm improves its receivables management and reduces its operating cycle and hence, its cash conversion cycle.
LO: 2 Level: Basic Bloomcode: Comprehension AASCP: Analytic IMA: Strategic Planning AICPA: Industry/Sector Perspective
SM 19-657
14.7
What are some industries in which the use of lockboxes would especially benefit companies? Explain. Lockboxes are a useful tool to speed up collection of receivables when the customer base is dispersed across a large geographical area. Normally, this would mean customer payments would have to be mailed in, consolidated, and then deposited at the firm’s bank. The alternative of setting up a lockbox system allows the firm to redirect customer payments to regional locations for quicker consolidation and cashing of payments. This is typical in the retail industry where each store of a chain is located in a different city or state. LO: 6 Level: Basic Bloomcode: Comprehension AASCP: Analytic IMA: Strategic Planning AICPA: Industry/Sector Perspective
14.8
Suppose you are a financial manager at a big firm and you expect interest rates to decline in the near future. What current asset investment strategy would you recommend that the company pursue? At a big firm, management would have access to the commercial paper market, which can provide cheaper funding than short-term bank financing. To borrow in the commercial paper market, a firm has to be financially strong. Thus, all or a portion of the working capital needs can be funded through short-term funds that can be rolled over as long as interest rates are declining. LO: 7 Level: Basic Bloomcode: Analysis AASCP: Analytic IMA: Decision Analysis AICPA: Industry/Sector Perspective
14.9
Why is the commercial paper market available only to the most creditworthy companies? Commercial paper is available only to the companies that are financially strong for two reasons. First, there is no secondary market for investors to liquidate prior to maturity. Consequently, investors must hold it to maturity and have the confidence that the issuer would pay them back at that time. Second, this type of debt is not secured by any real assets of the issuing firm. Thus, the companies that are the most creditworthy are able to raise funds in this market at costs that are lower than bank loans. SM 19-658
LO: 7 Level: Basic Bloomcode: Comprehension AASCP: Analytic IMA: Decision Analysis AICPA: Industry/Sector Perspective
14.10 Explain what a negative cash conversion cycle means. Recognizing the cash conversion cycle is a function of a firm’s receivables turnover, inventory turnover, and payables turnover. Firms that are highly efficient in managing their inventory and receivables will have a short operating cycle and do not need a large investment in working capital. A large payables turnover implies that the firm is making use of their suppliers’ funds to fund their working capital needs. The difference between the operating cycle and the payables turnover is the cash conversion cycle. A negative cash conversion cycle means that the time taken by the firm to meet its payables exceeds its operating cycle. In other words, the amount of time to manage their inventory and the time taken to collect its receivables is less than the time taken to pay its suppliers. LO: 2 Level: Basic Bloomcode: Comprehension AASCP: Analytic IMA: Strategic Planning AICPA: Industry/Sector Perspective
Questions and Problems BASIC 14.1 Cash conversion cycle: Wolfgang’s Masonry management estimates that it takes the company 27 days on average to pay its suppliers. Management also knows that the company has days’ sales in inventory of 64 days and days’ sales outstanding of 32 days. How does Wolfgang’s cash conversion cycle compare with the industry average of 75 days? Solution: DPO = 27 days SM 19-659
DSI = 64 days DSO = 32 days Industry average for cash conversion cycle = 75 days. Wolfgang’s cash conversion cycle: Cash conversion cycle DSO DSI DPO 32 days 64 days 27 days 69 days Since the firm’s Cash conversion cycle is less than the industry average of 75 days, the firm is more efficient than other firms in the industry in managing its working capital. LO 2 Bloomcode: Analysis AASCP: Analytic IMA: Strategic Planning AICPA: Industry/Sector Perspective
14.2 Cash conversion cycle: Northern Manufacturing Company management found that during the last year it took an average of 47 days to pay its suppliers, whereas it took 63 days to collect its receivables. The company’s days’ sales in inventory was 49 days. What was Northern’s cash conversion cycle? Solution: DPO = 47 days DSI = 49 days DSO = 63 days Northern's Cash conversion cycle DSO DSI DPO 63 days 49 days 47 days 65 days LO 2 Bloomcode: Application AASCP: Analytic IMA: Strategic Planning AICPA: Industry/Sector Perspective
14.3
Cash conversion cycle: Devon Automotive management estimates that it takes the company 62 SM 19-660
days to collect cash from customers on finished goods from the day it receives raw materials, and it takes 65 days to pay its suppliers. What is the company’s cash conversion cycle? Interpret your answer. Solution: DPO = 65 days Operating cycle = 62 days Devon’s cash conversion cycle = Operating cycle – DPO = 62 days – 65 days = -3 days This firm has a negative cash conversion cycle. The amount of time to manage its inventory and the time taken to collect its receivables is less than the time taken to pay its suppliers. LO 2 Bloomcode: Application AASCP: Analytic IMA: Strategic Planning AICPA: Industry/Sector Perspective
14.4
Operating cycle: Lilly Bakery distributes its products to more than 75 restaurants and delis. The company’s average collection period is 27 days, and it keeps its inventory for an average of four days. What is Lilly’s operating cycle? Solution: DSI = 4 days DSO = 27 days Lilly's Operating cycle DSO DSI 27 days 4 days 31days LO 2 Bloomcode: Application AASCP: Analytic IMA: Strategic Planning AICPA: Industry/Sector Perspective
14.5
Operating cycle: NetSpeed Technologies is a telecom component manufacturer. The firm SM 19-661
typically has a collection period of 44 days and days’ sales in inventory of 29 days. What is the operating cycle for NetSpeed? Solution: DSI = 29 days DSO = 44 days NetSpeed's Operating cycle DSO DSI 44 days 29 days 73 days LO 2 Bloomcode: Application AASCP: Analytic IMA: Strategic Planning AICPA: Industry/Sector Perspective
14.6
Current asset management strategy: Describe the risks that are associated with a restrictive current asset management strategy. Solution: The risk of a restrictive current asset management strategy comes from an exposure to shortage costs, which can be either financial or operating costs. Financial costs are associated with a sudden and unforeseen loss of liquidity that makes it difficult for a firm to repay its current liabilities. In this condition, firms may be forced to rely on costly external emergency borrowing, or if such funds can’t be secured, firms may have to default on their loan obligations. Operating shortage costs result from lost production or sales that occur if a firm does not have sufficient raw material or finished goods. Operating shortage costs may also occur if a firm’s credit policies are too restrictive and deter a significant number of sales to potential customers. LO 3 Bloomcode: Cmoprehension AASCP: Analytic IMA: Strategic Planning AICPA: Industry/Sector Perspective
14.7
Cost of trade credit: Sybex Corp. sells its goods with terms of 2/10 EOM, net 30. What is the implicit cost of the trade credit? SM 19-662
Solution: Credit terms = 2/10 EOM, net 30 365/ days credit Discount Effective annual rate of accounts receivable =1 1 Discounted price = (1 + 2/98)365/20 – 1 = (1.0204)18.2500 – 1 = 1.4459 – 1 = 0.4459, or 44.59 percent
LO 4 Bloomcode: Application AASCP: Analytic IMA: Strategic Planning AICPA: Industry/Sector Perspective 14.8
Cost of trade credit: Mill Street Corporation sells its goods with terms of 4/10 EOM, net 60. What is the implicit cost of the trade credit? Solution: Effective annual rate
365/ days credit Discount 1 of accounts receivable = 1 Discounted price = (1 + 4/96)365/50 – 1 = (1.0417)7.3 – 1 = 1.3472 – 1 = 0.3472, or 34.72%
LO 4 Bloomcode: Application AASCP: Analytic IMA: Strategic Planning AICPA: Industry/Sector Perspective
14.9
Lockbox: Rosenthal Design has daily sales of $59,000. The financial management team has determined that a lockbox would reduce the collection time by 1.6 days. Assuming the company can earn 5.2 percent interest per year, what are the savings from the lockbox? Solution: All sales are assumed to be credit sales. SM 19-663
Annual interest rate = 5.2% Collection time saved = 1.6 days Average daily sales $59,000 Savings $59,000 0.052 1.6 $4,908.80 The firm can save $4,908.80 with the use of lockbox. LO 6 Bloomcode: Application AASCP: Analytic IMA: Strategic Planning AICPA: Industry/Sector Perspective
14.10 Lockbox: Pacific Traders has annual sales of $1,895,000. The firm’s financial manager has determined that using a lockbox will reduce collection time by 2.3 days. If the firm’s opportunity cost on savings is 5.25 percent, what are the savings from using the lockbox? Solution: Annual sales = $1,895,000 Annual interest rate = 5.25% Collection time saved = 2.3 days $1,895,000 $5,191.78 Average daily sales 365 Savings $5,191.78 0.0525 2.3 days $626.91 The firm can save $626.91 with the use of lockbox. LO 6 Bloomcode: Application AASCP: Analytic IMA: Strategic Planning AICPA: Industry/Sector Perspective
14.11 Effective interest rate: The Kellogg Bank requires borrowers to keep an 8 percent compensating balance. Gorman Jewels borrows $340,000 at a 7 percent stated APR. What is the effective interest rate on the loan? Solution: Amount borrowed = $340,000 SM 19-664
Stated annual interest rate = 7% Compensating balance = 8% Amount deposited as compensating balance = $340,000 × 0.08 = $27,200 Effective borrowing amount = $340,000 − $27,200 = $312,800 Interest expense = $340,000 × 0.07 = $23,800 Interest expense $23,800 Effective interest rate 7.61% Effective borrowing amount $312,800 By setting aside a compensating balance of 8 percent or $27,200 on the loan, the firm increases its interest rate effectively to 7.61 percent. LO 7 Bloomcode: Application AASCP: Analytic IMA: Corporate Finance AICPA: Industry/Sector Perspective
14.12 Effective interest rate: Morgan Contractors borrowed $1.75 million at an APR of 10.2 percent. The loan called for a compensating balance of 12 percent. What is the effective interest rate on the loan? Solution: Amount to be borrowed = $1,750,000 Stated annual interest rate = 10.2% Compensating balance = 12% Amount deposited as compensating balance = $1,750,000 × 0.12 = $210,000 Effective borrowing amount equal to $1,750,000 − $210,000 = $1,540,000 Interest expense = $1,750,000 × 0.102 = $178,500 Interest expense $178,500 Effective interest rate 11.6% Effective borrowing amount $1,540,000 By setting aside a compensating balance of 12 percent or $210,000 on the loan, the firm increases its interest rate effectively to 11.6 percent. LO 7 Bloomcode: Application AASCP: Analytic IMA: Corporate Finance AICPA: Industry/Sector Perspective
SM 19-665
14.13 Factoring: Maltz Landscaping has an average collection period of 38 days for its accounts receivable. Currently, Maltz factors all of its receivables at a 2 percent discount. What is the effective annual interest rate on the financing from the factor? Solution: The terms of this factor indicate that for every dollar of receivables sold to the factor today, the firm receives 98-cents today and 38-days later the factor receives the dollar in receivables. The cost to Maltz in percentage terms is 2.04 percent over a 38 day period. There are 9.61 periods (m) during a 365 day year. Effective annual rate = (1+0.0204)365/38 – 1 = .2142 or 21.42 percent. LO 7 Bloomcode: Application AASCP: Analytic IMA: Corporate Finance AICPA: Industry/Sector Perspective
14.14 Formal line of credit: Winegartner Cosmetics management is setting up a line of credit at the company’s bank for $5 million for up to two years. The interest rate is 5.875 percent and the loan agreement calls for an annual fee of 40 basis points on any unused balance for the year. If the firm borrows $2 million on the day the loan agreement is signed, what is the effective rate for the line of credit? Solution: Line of credit limit = $5,000,000 Loan rate = 5.875% Annual fee on used balance = 0.4% Amount borrowed = $2,000,000 Unused balance = $3,000,000 Annual fee = $3,000,000 × 0.004 = $12,000 Interest expense = $2,000,000 × 0.05875 = $117,500 Interest expense Annual fee Effective interest rate Borrowed amount ($117,500 $12,000) 6.48% $2,000,000 The effective borrowing rate for the firm is 6.48% with the annual fee of 40 basis points. LO 7 Bloomcode: Application SM 19-666
AASCP: Analytic IMA: Corporate Finance AICPA: Industry/Sector Perspective
INTERMEDIATE 14.15 Cash conversion cycle: Your boss asks you to compute your company’s cash conversion cycle. Looking at the financial statements, you see that the average inventory for the year was $26,300, accounts receivable averaged $17,900, and accounts payable averaged $15,100. You also see that the company had sales of $154,000 and that cost of goods sold was $122,000. Calculate and interpret your firm’s cash conversion cycle. Solution: All sales are assumed to be credit sales. Accounts receivable = $17,900 Accounts payable = $15,100 Sales = $154,000 Average inventory = $26,300 Cost of goods sold = $122,000 Accounts receivable $17,900 = 42.4 days DSO = = Credit sales 365 $154,000 365 DSI
Inventory $26,300 78.7 days COGS 365 $122,000 365
Accounts payable $15,100 = 45.2 days = COGS 365 $122,000 365 Cash conversion cycle DSO DSI DPO DPO =
42.4 days 78.7 days 45.2 days 75.9 days The firm takes nearly 76 days from the time it pays for its raw materials to the time it realizes cash from its credit sales. By taking a couple of more days to pay it suppliers relative to the time it takes to collect on its receivables, it reduces the cash conversion cycle. LO 2 Bloomcode: Application AASCP: Analytic IMA: Strategic Planning AICPA: Industry/Sector Perspective
SM 19-667
14.16 Cash conversion cycle: Blackwell Automotive, Inc., reported the following financial information for the last fiscal year. Blackwell Automotive, Inc. Assets Liabilities and Equity Cash and marketable securities $ 23,015 Accounts payable and $163,257 accruals Accounts receivable $141,258 Notes payable $21,115 Inventories $212,444 Total current liabilities $184,372 Other current assets $ 11,223 Sales and Costs Total current assets $387,940 Net sales $912,332 Cost of goods sold $547,400 Calculate the firm’s cash conversion cycle and operating cycle. Solution: All sales are assumed to be credit sales. Accounts receivable = $141,258 Accounts payable = $163,257 Net sales = $912,332 Inventory = $212,444 Cost of goods sold = $547,400 Accounts receivable $141,258 = 56.5 days DSO = = Credit sales 365 $912,332 365 DSI Inventory $212,444 141.7 days COGS 365 $547,400 365 Accounts payable $163,257 = 108.9 days = COGS 365 $547,400 365 Operating cycle DSO DSI DPO =
56.5 days 141.7 days 198.2 days Cash conversion cycle = DSO + DSI – DPO = 56.5 days + 141.7 days – 108.9 days = 89.3 days LO 2 Bloomcode: Application AASCP: Analytic IMA: Strategic Planning AICPA: Industry/Sector Perspective
SM 19-668
14.17 Cash conversion cycle: Elsee, Inc., has net sales of $13 million, and 75 percent of these are credit sales. Its cost of goods sold is 65 percent of annual net sales. The firm’s cash conversion cycle is 41.3 days. The inventory balance at the firm is $1,817,344, while its accounts payable balance is $2,171,690. What is the firm’s accounts receivable balance? Solution: Net sales = $13,000,000 Credit sales = (0.75 × Sales) = (0.75 × $13,000,000) = $9,750,000 Accounts payable balance = $2,171,690 Inventory balance = $1,817,344 Cost of goods sold = (0.65 × Sales) = (0.65 × $13,000,000) = $8,450,000 Cash conversion cycle = 41.3 days Accounts payable $2,171,690 = 93.8 days DPO = = COGS 365 $8,450,000 365 DSI = Inventory = $1,817,344 = 78.5 days COGS 365 $8,450,000 365 Cash conversion cycle DSO DSI DPO 41.3 days DSO 78.5 days 93.8 days DSO 56.6 days Using the DSO equation, we can solve for the accounts receivable. Accounts receivable AR DSO 56.6 days $9,750,000 365 Credit sales 365 Accounts receivable 56.6$26,712.33 $1, 511, 917.81 The firm has accounts receivable of $1,511,918. LO 2 Bloomcode: Application AASCP: Analytic IMA: Strategic Planning AICPA: Industry/Sector Perspective 14.18 Cash conversion cycle: Joanna Handicrafts, Inc., has net sales of $4.23 million with 50 percent being credit sales. Its cost of goods sold is $2.54 million. The firm’s cash conversion cycle is 47.9 days and its operating cycle is 86.3 days. What is the firm’s accounts payable? Solution: Net sales = $4,230,000 SM 19-669
Credit sales = (0.5 × $4,230,000) = $2,115,000 Cash conversion cycle = 47.9 days Operating cycle = 86.3 days Cost of goods sold = $2,540,000 Cash conversion cycle (DSO DSI) DPO 47.9 days 86.3 days DPO DPO 38.4 days DPO =
Accounts payable Accounts payable = = 38.4 days COGS 365 $2,540,000 365
Accounts payable = 38.4 ×$6,958.90 = $267, 221.92 The firm has accounts payable of $267,222. LO 2 Bloomcode: Application AASCP: Analytic IMA: Strategic Planning AICPA: Industry/Sector Perspective
14.19 Operating cycle: Aviva Technology’s operating cycle is 81 days. Its inventory was $134,000 at the end of last year, and the company had cost of goods sold of $1.1 million. How long does it take Aviva to collect its receivables on average? Solution: Operating cycle = 81 days Inventory = $134,000 Cost of goods sold = $1,100,000 DSI Inventory $134,000 44.5 days COGS 365 $1,100,000 365 Operating cycle DSO DSI 81 days DSO 44.5 days DSO 36.5 days It takes Aviva 36.5 days to collect its receivables. LO 2 Bloomcode: Application AASCP: Analytic IMA: Strategic Planning SM 19-670
AICPA: Industry/Sector Perspective
14.20 Operating cycle: Premier Corp. has net sales of $812,344, and cost of goods sold equal to 70 percent of net sales. Assume all sales are credit sales. If the firm’s accounts receivable total $113,902 and its operating cycle is 81.6 days, how much inventory does the firm have? Solution: All sales are assumed to be credit sales. Credit sales = $812,344 Operating cycle = 81.6 days Cost of goods sold = (0.7 × $812,344) = $568,641 Accounts receivable = $113,902 Accounts receivable $113,902 = 51.2 days DSO = = Credit sales 365 $812,344 365 Operating cycle DSO DSI 81.6 days 51.2 days DSI DSI 30.4 days Inventory Inventory 30.4 days COGS 365 $568,641 365 Inventory $47,360.78 DSI
The firm has inventory of $47,361. LO 2 Bloomcode: Application AASCP: Analytic IMA: Strategic Planning AICPA: Industry/Sector Perspective
14.21 Operating cycle: Telecraft Enterprises carries 45 days of inventory in its stores. Last year Telecraft reported net sales of $1,400,000 and the company had receivables of $325,000 at the end of the year. What is the operating cycle at Telecraft? Solution: All sales are assumed to be credit sales. Accounts receivable $325,000 = 84.7 days DSO = = Credit sales 365 $1, 400,000 365 DSI = 45 days SM 19-671
Operating cycle DSO DSI 84.7 days 45 days 129.7 days LO 2 Bloomcode: Application AASCP: Analytic IMA: Strategic Planning AICPA: Industry/Sector Perspective 14.22 Operating Cycle: Given the data for Telecraft Enterprises in problem 14.21, re-estimate the firm’s operating cycle if days’ sales outstanding decreased to 75 days. For the same level of net sales, what is the implied dollar value of receivables with 75 days’ sales outstanding? Solution: Operating cycle = DSO + DSI = 75 days + 45 days = 120 days If the DSO is 75 days: DSO = 365 days / (credit sales / accounts receivable) 75 days = 365 days / ($1,400,000/accounts receivable) 75 days × $1,400,000 = 365 days × accounts receivable Accounts receivables = $1,400,000 × 75 days / 365 days Accounts receivables = $287,671.23 The value of receivables with 75 days’ sales outstanding is $287,671. LO 2 Bloomcode: Application AASCP: Analytic IMA: Strategic Planning AICPA: Industry/Sector Perspective 14.23 Economic order quantity: Longhorn Traders is one of the largest RV dealers in Austin, Texas, and sells about 2,800 recreational vehicles a year. The cost of placing an order with Longhorn’s supplier is $800, and the inventory carrying costs are $150 for each RV. Management likes to maintain safety stock of 12 RVs. Most of its sales are made in either the spring or the fall. How many orders should the firm place this year? Solution: Annual sales = 2,800 units Cost of placing an order = $800 Inventory carrying cost per RV = $150 Safety stock = 12 RVs
SM 19-672
EOQ
2 Reorder costsSales per period Carrying cos ts
2$800 2,800 172.8 $150 Economic order quantity = 173 RVs Number of orders the firm needs to place = 2,800 / 173 = 16 LO 5 Bloomcode: Application AASCP: Analytic IMA: Strategic Planning AICPA: Industry/Sector Perspective
14.24 Effective interest rate: The Clarkson Designer Company Management wants to borrow $750,000.The bank will provide the loan at an APR of 6.875 percent. Since the loan calls for a compensating balance, the effective interest rate is actually 9.25 percent. What is the compensating balance on this loan?
Solution: Amount to be borrowed = $750,000 Stated annual interest rate = 6.875% Compensating balance = ? Effective interest rate = 9.25% Interest expense = $750,000 × 0.06875 = $51,562.50 Interest expense Effective interest rate Effective borrowing amount $51,562.50 9.25% Effective borrowing amount $51,562.50 Effective borrowing amount $557,432.43 0.0925 Effective borrowing amount = $750,000 − compensating balance deposit = $557,432.43 Compensating balance deposit = $750,000 - $557,432.43 = $192,567.57 Compensating balance = $192,567.57 / $750,000 = 25.6% By setting aside a compensating balance of 25.6% on the loan, the firm increases its interest rate effectively from 6.875 to 9.25 percent. LO 7 Bloomcode: Application AASCP: Analytic IMA: Corporate Finance AICPA: Industry/Sector Perspective SM 19-673
14.25 Effective interest rate: The Colonial Window Treatments Company is borrowing $1.5 million. The loan requires a 10 percent compensating balance, and the effective interest rate on the loan is 9.75 percent. What is the stated APR on this loan?
Solution: Amount to be borrowed = $1,500,000 Stated annual interest rate = ? Compensating balance = 10% Effective interest rate = 9.75% Compensating balance = (0.10 × $1,500,000) = $150,000 Effective borrowing amount = $1,500,000 − $150,000 = $1,350,000 Interest expense Effective interest rate Effective borrowing amount Interest expense 9.75% Effective borrowing amount Interest expense 0.0975 $1,350,000 $131,625 Stated interest rate = $131,625 / $1,500,000 = 8.775% LO 7 Bloomcode: Application AASCP: Analytic IMA: Corporate Finance AICPA: Industry/Sector Perspective
14.26 Formal line of credit: Gruppa, Inc., has just set up a formal line of credit of $10 million with First Community Commercial Bank. The line of credit is good for up to five years. The bank will charge Gruppa an interest rate of 6.25 percent on any amount borrowed, and the firm will pay an annual fee of 60 basis points on the unused balance. The firm borrowed $7.5 million on the first day the credit line became available. What is the effective interest rate on this line of credit? Solution: Line of credit limit = $10,000,000 Interest rate = 6.25% Annual fee on used balance = 0.6% Amount borrowed = $7,500,000 Unused balance = $10,000,000 – $7,500,000 = $2,500,000 Annual fee = $2,500,000 × 0.006 = $15,000 Interest expense = $7,500,000 × 0.0625 = $468,750 SM 19-674
Interest expense Annual fee Borrowed amount ($468,750 $15,000) 6.45% $7,500,000 The effective interest rate on the line of credit is 6.45% Effective interest rate
LO 7 Bloomcode: Application AASCP: Analytic IMA: Corporate Finance AICPA: Industry/Sector Perspective
14.27 Formal line of credit: Lansdowne Electronics has a formal line of credit of $1 million for up to three years with HND Bank. The interest rate on the loan is 5.3 percent, and under the agreement, Lansdowne has to pay an annual fee of 50 basis points on the unused amount. Suppose the firm borrows $675,000 the first day of the agreement. What is the fee the company must pay on the unused balance? What is the effective interest rate? Solution: Line of credit limit = $1,000,000 Interest rate = 5.3% Annual fee on used balance = 0.5% Amount borrowed = $675,000 Unused balance = $1,000,000 - $675,000 = $325,000 Annual fee = $325,000 × 0.005 = $1,625 Interest expense = $675,000 × 0.053 = $35,775 Interest expense Annual fee Effective interest rate Borrowed amount ($35,775 $1,625) 5.54% $675,000 The effective interest rate is 5.54% LO 7 Bloomcode: Application AASCP: Analytic IMA: Corporate Finance AICPA: Industry/Sector Perspective
14.28 Lockbox: Jennifer Electrical is evaluating whether a lockbox it is currently using is worth keeping. Management estimates that the lockbox reduces the mail float by 1.8 days and the SM 19-675
processing by half a day. The remittances average $50,000 a day for Jennifer Electrical, with the average check being for $500. The bank charges $0.34 per processed check. Assume that there are 270 business days in a year and that the firm’s opportunity cost for these funds is 6 percent. What will the firm’s savings be from using the lockbox? Solution: Average daily sales = $50,000 No. of business days = 270 Opportunity cost of funds = 6% Average check amount = $500 No. of checks processed per day = $50,000 / $500 = 100 Collection time saved = 1.8 days Processing time saved = 0.5 days Per check processing fee = $0.34 The annual cost of a lockbox = 100 checks $0.34 per check 270 days = $9,180 Savings from mail float = 1.8 days $50,000 = $90,000 Savings from processing float = 0.5 days $50,000 = $25,000 Total savings = (Savings from mail float + Savings from processing float) = $90,000 + $25,000 = $115,000 Savings from lockbox = $115,000 × 0.06= $6,900 Since the savings from the lockbox is less than the cost of the lockbox, it is not worth keeping the lockbox for Jennifer Electricals. LO 6 Bloomcode: Application AASCP: Analytic IMA: Strategic Planning AICPA: Industry/Sector Perspective
14.29 Lockbox: Hazel Corp. has just signed up for a lockbox. Management expects the lockbox to reduce the mail float by 2.1 days. Hazel Corp.’s remittances average $37,000 a day, and the average check is $125. The bank charges $0.37 per processed check. Assume that there are 270 business days in a year. What will the firm’s savings be from using the lockbox if the opportunity cost for these funds is 12 percent? Solution: Average daily sales = $37,000 No. of business days = 270 Average check amount = $125 No. of checks processed per day = $37,000 / $125 = 296 Collection time saved = 2.1 days SM 19-676
Per check processing fee = $0.37 The annual cost of a lockbox = 296 checks $0.37 per check 270 days = $29,570.40 Savings from mail float = 2.1 days $37,000 = $77,700 Opportunity cost of funds = 12% Savings from lockbox = $77,700 × 0.12= $9,324 Since the savings from the lockbox is less than the cost of the lockbox, it is not worth keeping the lockbox for Hazel Corp. LO 6 Bloomcode: Application AASCP: Analytic IMA: Strategic Planning AICPA: Industry/Sector Perspective
14.30 Aging schedule: Ginseng Company collects 50 percent of its receivables in 10 days or fewer, 31 percent in 11 to 30 days, 7 percent in 31 to 45 days, 7 percent in 46 to 60 days, and 5 percent in more than 60 days. The company has $1,213,000 in accounts receivable. Prepare an aging schedule for Ginseng Company. Solution: Accounts receivables = $1,213,000 Age of Account (days) 0-10 11-30 31-45 46-60 Over 60 Total
Value of Account $ 606,500 376,030 84,910 84,910 60,650 $1,213,000
% of Total Value 50% 31 7 7 5 100%
LO 4 Bloomcode: Application AASCP: Analytic IMA: Strategic Planning AICPA: Industry/Sector Perspective 14.31 Aging schedule: A partial aging of accounts receivable for Lincoln Cleaning Services is given in the accompanying table. What percent of receivables are in the 45-day range? Determine the firm’s effective days’ sales outstanding. How does it compare with the industry average of 35 days? Age of Account (days) Value of Account % of Total Value SM 19-677
10 30 45 60 75 Total
$271,000 145,220 53,980 31,245 $589,218
100.0%
Solution: Accounts receivable = $589,218 Age of Account Value of Account (days) 10 $271,000 30 145,220 45 87,773 60 53,980 75 31,245 $589,218 Total
% of Total Value 46.0% 24.6 14.9 9.2 5.3 100.0%
= Age of the account category Percent of AR for the account category = (10 × 0.46 + 30 × 0.246 + 45 × 0.149 + 60 × 0.092 + 75 × 0.053) = 28.2 days The firm is more efficient than other firms in the industry as its effective DSO is lower. Effective DSO
LO 4 Bloomcode: Application AASCP: Analytic IMA: Strategic Planning AICPA: Industry/Sector Perspective 14.32 Aging schedule: Keswick Fencing Company collects 45 percent of its receivables in 10 days or fewer, 34 percent in 11 to 30 days, 12 percent in 31 to 45 days, 5 percent in 46 to 60 days, and 4 percent in more than 60 days. The company has $937,000 in accounts receivable. Prepare an aging schedule for Keswick Fencing. Solution: Accounts receivable = $937,000 Age of Account Value of Account (days) 0-10 $421,650 11-30 318,580 31-45 112,440
% of Total Value 45% 34 12 SM 19-678
46-60 Over 60 Total
46,850 37,480 $937,000
5 4 100%
LO 4 Bloomcode: Application AASCP: Analytic IMA: Strategic Planning AICPA: Industry/Sector Perspective 14.34 Factoring: Zenex, Inc., sells $250,000 of its accounts receivable to factors at a 3 percent discount. The firm’s average collection period is 90 days. What is the dollar cost of the factoring service? What is the simple annual interest cost of the factors loan? Solution: Accounts receivable sold = $250,000 Factor discount = 3% Average collection period = 90 days Dollar cost of factoring per month = $250,000 × 0.03 = $7,500 Dollar cost over 90 days = $7,500 × 90/30 = $22,500 Simple monthly interest cost of factoring = 3/97 or 0.03 / (1-0.03) = 0.03093 Simple annual interest cost of factoring = 0.03093 × 12 months = 37.1% LO 4 Bloomcode: Application AASCP: Analytic IMA: Strategic Planning AICPA: Industry/Sector Perspective
14.34 Factoring: A firm sells $100,000 of its accounts receivable to factors at a 2 percent discount. The firm’s average collection period is one month. What is the dollar cost of the factoring service? Solution: Accounts receivable sold = $100,000 Factor discount = 2% Average collection period = 30 days Dollar cost of factoring service per month = $100,000 × 0.02 = $2,000 LO 7 Bloomcode: Application AASCP: Analytic SM 19-679
IMA: Strategic Planning AICPA: Industry/Sector Perspective
ADVANCED 14.35 What impact would the following actions have on the operating and cash conversion cycles? Would the cycles increase, decrease, or remain unchanged? a. More raw material than usual is purchased. b. The company enters into an off season, and finished goods inventory builds up. c. Better terms of payment are negotiated with suppliers. d. The cash discounts offered to customers are decreased. e. All else remaining the same, an improvement in manufacturing technique decreases the cost of goods sold. Solution: Situation a. More raw material than usual is purchased. b. The company enters into an off season, and finished goods inventory builds up. c. Better terms of payment are negotiated with suppliers. d. The cash discounts offered to customers are decreased. e. All else remaining the same, an improvement in manufacturing technique decreases the cost of goods sold. LO 2 Bloomcode: Comprehension AASCP: Analytic IMA: Strategic Planning AICPA: Industry/Sector Perspective
Operating cycle
Cash conversion cycle
Increase
Increase
Increase
Increase
No change
Decrease
Increase
Increase
Increase
Unchanged
14.36 What impact would the following actions have on the operating and cash conversion cycles? Would the cycles increase, decrease, or remain unchanged? a. Less raw material than usual is purchased. b. The company encounters unseasonable demand, and inventory declines rapidly. c. Tighter terms of payment are demanded by suppliers. SM 19-680
d. The cash discounts offered to customers are increased. e. All else remaining the same, due to labor turnover and poor efficiency, the cost of goods sold increases. Solution: Situation
Operating cycle
a. Less raw material than usual is purchased. b. The company encounters unseasonable demand, and inventory declines rapidly. c. Tighter terms of payment are demanded by suppliers. d. The cash discounts offered to customers are increased. e. All else remaining the same, due to labor turnover and poor efficiency, the cost of goods sold increases.
Decrease
Cash conversion cycle Decrease
Decrease
Decrease
No change
Increase
Decrease
Decrease
Decrease
Unchanged
LO 2 Bloomcode: Comprehension AASCP: Analytic IMA: Strategic Planning AICPA: Industry/Sector Perspective
14.37 Morgan Sports Company just reported the following financial information. Morgan Sports Equipment Company Assets Liabilities and Equity Cash $ 677,423 Accounts payable $1,721,669 Accounts receivable 1,845,113 Notes payable 2,113,345 Inventories 1,312,478 Total current liabilities $3,835,014 Total current assets $3,835,014 Sales and Costs
a. b. c. d.
Net sales $9,912,332 Cost of goods sold $5,947,399 Calculate the firm’s days’ sales outstanding. What is the firm’s days’ sales in inventory? What is the firm’s days’ payables outstanding? What is the firm’s operating cycle? How does it compare with the industry average of 72 days? SM 19-681
What is the firm’s cash conversion cycle? How does it compare with the industry average of 42 days?
e.
Solution: All sales are assumed to be credit sales. a.
DSO =
b.
DSI
c.
DPO =
Accounts receivable $1,845,113 = = 67.9 days Credit sales 365 $9,912,332 365
Inventory $1,312,478 80.6days COGS 365 $5,947,399 365 Accounts Payable $1,721,669 = 105.7 days COGS 365 $5,947,399 365
Operating cycle DSO DSI d.
67.9 days 80.6 days 148.5 days The firm is very inefficient in managing its receivable and inventory as its operating cycle exceeds the industry average of 72 days by about 77 days. Cash conversion cycle DSO DSI DPO
e.
(67.9 days 80.6 days) 105.7 days 42.8 days The firm’s cash conversion cycle is on a par with the industry average of 42 days thanks to its suppliers’ very generous credit policy.
LO 2 Bloomcode: Application AASCP: Analytic IMA: Strategic Planning AICPA: Industry/Sector Perspective
14.38 Jackson Electrical, one of the largest generator dealers in Phoenix, sells about 2,000 generators a year. The cost of placing an order with its supplier is $750, and the inventory carrying costs are $170 for each generator. Jackson likes to maintain safety stock of 15 generators at all times. a. What is the firm’s EOQ? b. How many orders will the firm need to place this year? SM 19-682
c.
What is the average inventory for the season?
Solution: Annual sales = 2,000 generators Cost of placing an order = $750 Inventory carrying cost per generator = $170 Safety stock = 15 generators EOQ a.
2 Reorder costsSales per period Carrying costs
2 $750 2,000 132.8 $170 Economic order quantity = 133 generators No. of orders the firm needs to place = 2,000 / 133 = 15 orders Average inventory = ((133 – 0)/2) + 15 = 82 generators
b. c.
LO 5 Bloomcode: Application AASCP: Analytic IMA: Strategic Planning AICPA: Industry/Sector Perspective 12.39 Tanzaniqe, Inc., sells $200,000 of its accounts receivable to factors at a 5 percent discount. The firm’s average collection period is 90 days. a. What is the dollar cost of the factoring service? b. What is the simple annual interest cost of the loan? c. What is the effective annual interest cost of the loan? Solution: Accounts receivable sold = $200,000 Factor discount = 5% Average collection period = 90 days a. Dollar cost of factoring per month = $200,000 × 0.05 = $10,000 Dollar cost over 90 days = $10,000 × 3 or $10,000 × 90 / 30 = $30,000 b.
Simple monthly interest cost of factoring = 5/95 OR 0.05 / (1-0.05) = 0.0526 Simple annual interest cost of factoring = 0.0526 × 12 = 63.2%
SM 19-683
m
Quoted rate EAR 1 1 m c.
(1 0.0526)12 1 0.8506 85.06%
LO 7 Bloomcode: Application AASCP: Analytic IMA: Strategic Planning AICPA: Industry/Sector Perspective
CFA PROBLEMS 12.40 A company increasing its credit terms for customers from 1/10, net 30 to 1/10, net 60 will likely experience: a. An increase in cash on hand. b. An increase in the average collection period. c. Higher net income. d. A higher level of uncollectible accounts. LO 6 Bloomcode: Application AASCP: Analytic IMA: Decision Analysis
AICPA: Industry/Sector Perspective Solution: b is correct.
14.41 Suppose a company uses trade credit with the terms of 2/10, net 50. If the company pays its account on the 50th day, the effective borrowing cost of skipping the discount on Day 10 is closest to a. 14.6percent. b. 14.9percent. c. 15.0percent. d. 20.2percent. LO 4 Bloomcode: Application AASCP: Analytic IMA: Decision Analysis AICPA: Industry/Sector Perspective Solution: d is correct. SM 19-684
365/ 40
0.02 Cost = 1+ 0.98
1 20.24 percent
The following information relates to Problems 14.42 through 14.44. Mary Gonzales is evaluating companies in the office supply industry and has compiled the following information: 2013 2014 Average Average Company Credit Sales Receivables Credit Sales Receivables Balance Balance $5.0 million $1.0 million $6.0 million $1.2 million A $3.0 million $1.2 million $4.0 million $1.5 million B $2.5 million $0.8 million $3.0 million $1.0 million C $0.5 million $0.1 million $0.6 million $0.2 million D Industry $25.0 million $5.0 million $28.0 million $5.4 million
12.42 Which of the companies has the lowest accounts receivable turnover in 2014? a. Company A. b. Company B. c. Company C. d. Company D. LO 4 Bloomcode: Analysis AASCP: Analytic IMA: Strategic Planning AICPA: Industry/Sector Perspective
Solution: b is correct. Company A: $6.0 million/$1.2 million = 5.00 Company B: $4.0 million/$1.5 million = 2.67 Company C: $3.0 million/$1.0 million = 3.00 Company D: $0.6 million/$0.2 million = 3.00
14.43 The industry average receivables collection period: a. Increased from 2013 to 2014. b. Decreased from 2013 to 2014. c. Did not change from 2013 to 2014. SM 19-685
d.
Increased along with the increase in the industry accounts receivable turnover. LO 4 Bloomcode: Analysis AASCP: Analytic IMA: Strategic Planning AICPA: Industry/Sector Perspective Solution: b is correct.
2013: 73 days 2014: 70.393 days Note: If the number of days decreased from 2013 to 2014, the receivable turnover has increased.
14.44 Which of the companies reduced the average time it took to collect on accounts receivable from 2013 to 2014? a. Company A. b. Company B. c. Company C. d. Company D. LO 4 Bloomcode: Analysis AASCP: Analytic IMA: Strategic Planning AICPA: Industry/Sector Perspective Solution: b is correct.
Company B increased its accounts receivable (A/R) turnover and reduced its number of days of receivables between 2013 and 2014. 2013 Company
A/R turnover
A B C D
5.000 2.500 3.125 5.000
2014 Number of days of receivable 73.000 146.000 116.800 73.000
AR turnover 5.000 2.667 3.000 3.000
Number of days of receivable 73.000 136.875 121.667 121.667
SM 19-686
Sample Test Problems 14.1
The Whole Foods Market, Inc. balance sheet for the fiscal year ending September 29, 2013 included the following: total current assets of $1,980 million, total assets of $5,538 million, total current liabilities of $1,088 million, and total liabilities of $1,660 million. What was the company’s net working capital on September 29, 2013? What does this tell us? (LO 1) Solution: Net Working Capital = Total Current assets – Total Current Liabilities = $1,980,000,000 - $1,088,000,000 = $892,000,000 This tells us that Whole Foods could pay off all its current liabilities (those that will come due in the next year) with the cash and other assets that could reasonably be expected to be converted to cash within the next year and still have $892 million left over. Bloomcode: Analysis AASCP: Analytic IMA: Strategic Planning AICPA: Industry/Sector Perspective
14.2
Last year Perpetual Plastics Company took an average of 46 days to pay suppliers and 38 days to collect its receivables. The company’s average days’ sales in inventory was 52 days. What was Perpetual’s operating cycle and cash conversion cycle last year? (LO 2) Solution: DPO = 46 days DSO = 38 days DSI = 52 days Perpetual’s operating cycle = DSO + DSI = 38 days + 52 days = 90 days Perpetual’s cash conversion cycle = DSO + DSI – DPO = 38 days + 52 days – 46 days = 44 days Bloomcode: Application AASCP: Analytic IMA: Strategic Planning AICPA: Industry/Sector Perspective
SM 19-687
14.3
Montrose, Inc. sells its products with terms of 3/15 EOM, net 30. What is the cost of the trade credit it provides its customers? (LO 4)
Solution: Credit terms = 3/15 EOM, net 30
365/days credit Discount -1 Effective annual rate (EAR) = 1+ Discounted price
= (1 + 3/97)365/15 – 1 = (1.030928)24.333333 – 1 = 2.0984 – 1 = 1.0984, or 109.84% Bloomcode: Application AASCP: Analytic IMA: Strategic Planning AICPA: Industry/Sector Perspective
14.4
FRA Manufacturing Company purchases 9,000 units of Part 3BX each year. The cost of placing an order is $5 and the cost of carrying one part in inventory for a year is $1. What is the Economic Order Quantity (EOQ) for part 3BX if the company carries a safety stock of 200 units? How many orders will the company need to place this year? (LO 5)
Solution: Annual purchases = 9,000 units Cost of placing an order = $5 Inventory carrying cost per part = $1 Safety stock = 200 parts EOQ = =
2×Reorder costs×Sales(or purchases)per period Carrying costs 2×$5×9,000 = 300parts $1
The number of orders that the firm should to place each year is 300/9,000 = 30 SM 19-688
Bloomcode: Analysis AASCP: Analytic IMA: Strategic Planning AICPA: Industry/Sector Perspective
SM 19-689
14.5
Rosemary Corporation has daily sales of $139,000. The financial manager at the firm has determined that a lockbox would reduce collection time by 2.2 days. Assuming the company can earn 5.5 percent interest per year, what are the potential annual savings from the lockbox? (LO 6) Solution: Average daily sales = $139,000 Collection time saved = 2.2 days Savings from mail float = 2.2 days × $139,000 = $305,800 Annual savings if invested = $305,800 × 0.055 = $16,819
Bloomcode: Application AASCP: Analytic IMA: Strategic Planning AICPA: Industry/Sector Perspective
14.6
Sunny Way Landscaping has a formal line of credit of $500,000 with First Commerce Bank. The interest rate on the loan is 6 percent, and under the agreement, Sunny Skies must pay an annual fee of 75 basis points on the unused amount. The amount currently outstanding on the loan is $325,000. What is the annual fee the company must pay on the current unused balance? What is the effective interest rate? (LO 7)
Solution: Line of credit limit = $500,000 Interest rate = 6% Annual fee on unused balance = 0.75% Amount borrowed = $325,000 Unused balance = $175,000 Annual fee = $175,000 × 0.0075 = $1,312.50 Interest expense for amount borrowed = $325,000 × 0.06 = $19,500.00 Interest expense + Annualfee Effective interest rate = Borrowed amount ($19,500.00 +$1,312.50) = = 0.0640, or 6.40% $325,000.00 Bloomcode: Application AASCP: Analytic IMA: Strategic Planning AICPA: Industry/Sector Perspective
SM 19-690
CHAPTER 15 How Firms Raise Capital Before You Go On Questions and Answers Section 15.1 1.
Explain bootstrapping, and list the most common sources of seed money. Bootstrapping refers to the securing of initial funding by entrepreneurs to start a new business. It alludes to the idea of an individual or small group of entrepreneurs who keep their business idea barely alive by securing money to keep it going. The initial ―seed‖ money usually comes from the entrepreneur or other founders. Other cash may come from personal savings, the sale of assets such as cars and boats, loans from family members and friends, and loans secured from credit cards.
Section 15.2 1.
Who are venture capitalists, and what do they do? Venture capitalists are individuals or groups of people that help new businesses to get started and provide much of their early financing. They pool money from various sources, such as wealthy individuals, insurance companies, pension funds, or large corporations and invest in start-up ventures. The primary types of businesses seeking the services of venture capitalists are high-tech firms.
2.
How do venture capitalists reduce the risk of their investments? Venture capitalists reduce the risk of their investments in two ways: (1) stage funding, which gives the investors a chance to periodically reassess the project, the management team, and the firm’s financial performance, and to make necessary corrections throughout the duration of the project, and (2) personal investment by the entrepreneurs. Requiring the entrepreneurs to make a substantial personal investment in the business is to make sure that they are highly motivated to succeed.
3.
Explain the venture capital funding cycle. The venture capital funding cycle starts with the development of a detailed business plan by the start-up company. Once a venture capitalist (VC) that will fund the project is selected, they state their terms of funding—how much money will they supply in what stages, criteria of success, and so on. Finally, exit strategy is developed. Since VCs are not long-term investors, it is important that a clear understanding of how the VC can exit the deal is part of the initial agreement. This can involve taking the firm public or sometimes selling the new firm to another business.
SM 19-691
Section 15.3 1.
What is a seasoned offering, and why are seasoned securities valued more highly than securities sold in an IPO? A seasoned offering is a sale of securities by a firm that already has publicly traded securities outstanding. These are usually more expensive as they are viewed to be less risky than IPO securities; since they are sold by firms that already established a reputation for quality with the investing public, many investors prefer them to new issues.
2.
Explain the two ways a security issue can be underwritten. A security can be underwritten in two ways: (1) firm-commitment basis—in this case the banker guarantees the issuer a fixed amount of money from the security sale, and (2) best-effort basis—whereby the banker makes no guarantee to sell the securities at a particular price, but instead promises to sell as much of the issue as possible above a certain price. The main difference between these two types of underwriting is who bears the risk. With the firm-commitment basis, it is the banking firm that buys the securities first before selling them to public. On the other hand, in a best-effort type of deal, the issuer is the risk bearer.
3.
List the steps in the IPO process. The IPO process involves the following steps: (1) selection of the investment bank, (2) preparation of the registration paperwork and SEC process, (3) determination of the structural features of the offering, (4) valuation, (5) marketing of the IPO, and (6) sale of the securities.
Section 15.4 1.
What is underpricing, and why is it a cost to the stockholders? Underpricing is defined as the offering of new securities for sale at an offer price below the true value of the security. It is considered to be a cost to a firm, because if the stock sells at a low price, the issuer is essentially losing money, which goes in the pockets of investors that bought the issues for less than they are worth.
2.
What are the components of the cost associated with an IPO? The cost of issuing an IPO includes the following: (1) underwriting spread, (2) out-of-pocket expenses, and (3) underpricing.
Section 15.5 1.
Explain why firms generally sell their equity and complicated debt issues through negotiated sales. Negotiated sales allow the investment bankers to form a closer relationship with the issuer and develop a better understanding of the firm. This enables them to form more effective selling teams. As a result, this should allow the bankers to reduce uncertainty surrounding the issue, resulting in a lower issue cost.
2.
Explain the importance of shelf registration.
SM 19-692
Shelf registration allows firms to register the inventory of securities for a two-year period. During this time the firm can take the securities and sell them as needed. This process simplifies the sale, as there are no flotation costs involved after the initial registration statement. The firm can also sell as many or as few securities at a time as it needs to, taking advantage of favorable market conditions. There is no penalty if authorized securities are not issued.
Section 15.6 1.
What are the disadvantages of a private placement sale compared with a public sale? Private placement has three main disadvantages compared to public sale. First, because of the lack of liquidity, nominal yields are higher. Second, since private placement securities are not registered, the covenants are stricter. And finally, compared to public securities, private placement securities are more difficult to sell in the secondary market.
2.
Why do companies engage in PIPE transactions? Small and midsize companies can find it difficult or costly to raise money in the public markets. In these circumstances, it can be more efficient or cost effective to sell stock privately, even if the company’s stock is already publicly traded. PIPE transactions bring several advantages to the issuing firm. They give the firm faster access to capital and a lower funding cost than a registered public offering. A PIPE transaction can be completed in as short a time as two weeks, whereas a typical public offering underwritten by an investment bank takes several months. PIPE transactions involving a healthy firm can also be executed without the use of an investment bank, resulting in a cost saving of 7 to 8 percent of the proceeds. A PIPE transaction can be the only way for a small financially distressed company to raise equity capital.
Self-Study Problems 15.1
Management of Oakley, Inc., is planning to raise $1 million in new equity through a private placement. If the sale price is $18 per share, how many shares does the company have to issue?
Solution: To raise $1million, Oakley has to issue 55,556 shares: ($1,000,000 / $18 per share = 55,556 shares).
SM 19-693
15.2
Suppose a firm is doing an IPO and the investment bank offers to buy the securities for $39 per share with an offering price of $42. What is the underwriter’s spread? Assume that the underwriter’s cost of bringing the security to the market is $5 per share. What is the net profit?
Solution: Underwriter’s spread: $42 – $39 = $3, or 7.1% ($3 / $42 = 0.071, or 7.1%) Net profit per share : $3 – $1 = $2
15.3
Management of The Stride Rite Corporation, designer and marketer of athletic apparel, is planning an expansion into foreign markets and needs to raise $10 million to finance this move. Management anticipates raisig the money through a general cash offering for $13 a share. If the underwriters charge a 5 percent spread, how many shares will the company have to sell to achieve its goal?
Solution: Underwriter’s spread = 5% Price per share for the firm = [$13 × (1-0.05)] = $12.35 To raise $10 million, the company needs to issue: ($10,000,000 / $12.35) = 809,717 new shares
15.4
Dean Foods Co. needs to borrow $23 million for a factory equipment upgrade. Management decides to sell 10-year bonds. They determine that the 3-month Treasury bill yields 4.32 percent, the firm’s credit rating is AA, and the yield on 10-year Treasury bonds is 1.06 percent higher than for 3-month bills. Right now, AA bonds are selling for 1.35 percent above the 10-year Treasury bond rate. What is the borrowing cost for this transaction?
Solution: The borrowing cost for Dean Foods can be calculated as follows: kl = 4.32% + 1.35% + 1.06% = 6.73% The approach used here is similar to that used in the bank loan pricing model.
15.6
You are considering starting a new online dating service, but you lack the initial capital. What are your options for financing?
Solution: SM 19-694
Possible sources of capital include your own savings, family and friends, wealthy individuals, venture capital firms, and financial institutions such as banks.
Discussion Questions 15.1
Assume you work for a venture capital firm and have been approached by a couple of recent college graduates with a request to fund their new business. If you are interested in the idea, what process will you follow?
After setting up the initial meeting, we should carefully review their business plan and make sure that it contains all the important information a business plan should, such as industry and market analysis, vision and mission statements, management team description, and cost analysis along with the proposed budget. Providing we like the plan and decide to fund the project, we will try to minimize our risk of investing and only fund the project in stages. In addition, we will require some personal investment from the candidates to ensure their serious interest in the project. Finally, we will draft an exit strategy, outlining the timing and calculation of the exit price. LO: 2 Level: Basic Bloomcode: Comprehension AASCP: Analytic IMA: Corporate Finance AICPA: Industry/Sector Perspective
15.2
Identify the three basic services investment bankers provide to help firms bring new security issues to the market. During which stage of the typical IPO does the investment banker take on the risk of the offering? Is there an alternative in which the risk remains with the company going public? The three services provided by an investment banker in an initial public offering are origination, underwriting, and distribution. The underwriter takes on the risk during the second stage of the IPO process that is underwriting, providing a firm-commitment underwriting deal is in place. Under this scenario, the banker actually buys the securities from the firm at a fixed price and resells them to the public. Hence, the underwriter bears the risk that the securities may be sold for less than the underwriter paid for it. An alternative arrangement is the best-effort underwriting, where the investment banking firm makes no guarantee to sell the securities at a particular price, but instead promises
SM 19-695
only to make its ―best effort‖ to sell as much of the issue as possible at a certain price. In this scenario, the risk stays with the offering firm. LO: 3 Level: Basic Bloomcode: Comprehension AASCP: Analytic IMA: Corporate Finance AICPA: Industry/Sector Perspective
15.3
Define underpricing, and explain why the majority of IPOs are underpriced. What role do investment banks play in the price-setting process?
Underpricing is defined as offering new securities for sale at a price below their true value. It is the loss that the issuer incurs from selling below its true market value. Asymmetric information plays a big role in an IPO underpricing, since both the bankers and the public know less about private companies going public than they do about already publicly traded firms. The lower offering price is to compensate the investors for taking on the risk of buying into a company for which not a lot of information is available. One way for companies to reduce underpricing is to choose a well-known investment bank to take them public. This will send a signal to the potential investors that the bank has done its due diligence and considers the firm to be creditworthy. LO: 4 Level: Intermediate Bloomcode: Knowledge AASCP: Analytic IMA: Corporate Finance AICPA: Industry/Sector Perspective
15.4
Explain why the owners of a company might choose to keep it private.
By remaining private, a company effectively avoids being subjected to SEC regulations. Many of the large private companies are family owned with a long history and prefer not to be subjected to public scrutiny. In addition, the management of the company is not pressured by Wall Street to report short-term earnings, but can concentrate their efforts on long-term growth and strategy realization. LO: 6 Level: Basic Bloomcode: Comprehension AASCP: Analytic SM 19-696
IMA: Corporate Finance AICPA: Industry/Sector Perspective
15.5
Identify the three cost components that make up the total cost of issuing securities for a company. Briefly describe each.
The total cost for a firm to issue securities is equal to the sum of the underwriting spread, out-of-pocket expenses, and underpricing. The underwriting spread is the difference between the proceeds the issuer receives and the total amount raised in the offering. Outof-pocket expenses are usually administrative costs such as investment banking fees, legal fees, accounting expenses, SEC filing fees, and so on. Finally, the total amount of underpricing is determined as the difference between the offering price and the closing price on the first day of trading in the public market. LO: 3 Level: Basic Bloomcode: Comprehension AASCP: Analytic IMA: Corporate Finance AICPA: Industry/Sector Perspective
15.6
What are the characteristics of a public bond? (Think in terms of comparing it to private placement and bank term loans.)
A public bond is usually the most suitable securities for a large firm looking for a large loan. It tends to have the lowest interest costs as well as the lowest default risk, but usually has a long maturity. While it has very few restrictive covenants, it is also very difficult to renegotiate. LO: 3 Level: Basic Bloomcode: Comprehension AASCP: Analytic IMA: Corporate Finance AICPA: Industry/Sector Perspective
15.7
Discuss the advantages of shelf registration. What kinds of securities are most likely to be registered this way?
SM 19-697
Shelf registration allows the company to register an inventory of securities for a two year period, and thus the company can sell the securities when market conditions are most favorable. The company can take the securities ―off the shelf‖ and sell them within minutes. They can also sell securities on a periodic basis, whenever cash is needed. Plain vanilla bonds are most likely to be sold by shelf registration through competitive bidding. LO: 5 Level: Basic Bloomcode: Comprehension AASCP: Analytic IMA: Corporate Finance AICPA: Industry/Sector Perspective
15.8
Identify whether each of the following factors implies a lower or higher price for a bond. a. Low marketability of the security. b. Short term to maturity. c Low credit rating of the issuer. d. No call Bond yield Bond Price provision.Factor a. Low marketability of Increase Decrease the security b. Short term to maturity Decrease Increase c. Low credit rating of the Increase Decrease issuer c. No call provision Decrease Increase LO: 6 Level: Intermediate Bloomcode: Comprehension AASCP: Analytic IMA: Corporate Finance AICPA: Industry/Sector Perspective
15.9
Explain why time might play a significant role during low-interest periods in a decision of whether to choose a private placement or public sale. Solution:
SM 19-698
Private placement deals tend to be much quicker than public sales. Since there is uncertainty about the future behavior of interest rates in the market, a firm might decide to go with private deal in order to lock in the favorable interest rates. LO: 6 Level: Basic Bloomcode: Comprehension AASCP: Analytic IMA: Corporate Finance AICPA: Industry/Sector Perspective
15.10 Managers at a large firm are looking for a medium-size loan with a long term to maturity and low liquidity. Which of the following types of debt would be the most appropriate? a. Public bond. b. Private placement. c. Bank term loan. A large firm can have its pick of the three alternatives. A public offering of debt would be helpful to raise large amounts, but the terms of the bond contract may not be easily negotiated. Private placements have a number of advantages, relative to public offerings, for certain issuers. These include lower cost of funds, ability to negotiate changes to a bond contract, speed of private placement deals, and flexibility in issue size. Bank term loans are preferable when the loan term is short to medium term. A large firm may choose a bank loan only if it cannot raise funds publicly or through private placement. Bank loans offer some similar advantages as private placements. LO: 7 Level: Intermediate Bloomcode: Comprehension AASCP: Analytic IMA: Corporate Finance AICPA: Industry/Sector Perspective
Questions and Problems BASIC
SM 19-699
15.1
Venture capital: What items in a business plan does a venture capitalist look for in deciding whether to provide initial financing?
Solution: Every business plan should contain the following information:
A description of the business and industry trends. Vision and key strategies for the business. Principal products or services and any innovative features or patents. The management team and their experience. Market analysis and sales forecast. How the products will be marketed and sold. Production costs such as materials and labor. Facilities needed and estimated costs. Capital required and the use of the proceeds. Detailed budget with six years of projected financial statements.
LO 2 Bloomcode: Comprehension AASCP: Analytic IMA: Corporate Finance AICPA: Industry/Sector Perspective
15.2
Venture capital: You finally decide to act on your brilliant idea and start an online textbook rental company. You develop a detailed business plan and calculate that you will need about $350,000 of initial funding to get the business going. Luckily for you, you have lined up two venture capital firms offering to supply the funding. What criteria should guide your decision to select one firm over the other?
Solution: Criteria include in-depth knowledge of the business, industry and technology, references, chemistry, access to additional capital, networking, exit strategy, and proximity. LO 2 Bloomcode: Application AASCP: Analytic IMA: Corporate Finance AICPA: Industry/Sector Perspective 15.3
Venture capital: What are some viable exit strategies for a start-up company?
Solution: SM 19-700
A firm can choose to either sell the business at some period, take it public, or in some instances remain a private company with few shareholders. LO 2 Bloomcode: Comprehension AASCP: Analytic IMA: Corporate Finance AICPA: Industry/Sector Perspective
13.4
IPO: Briefly describe the IPO process.
Solution: The IPO process has three phases: origination, which is financial advising and getting the issue ready to sell; underwriting, which involves the investment banker buying the securities from the firm; and distribution, which is when the investment banker resells the securities to the public. LO 3 Bloomcode: Comprehension AASCP: Analytic IMA: Corporate Finance AICPA: Industry/Sector Perspective 15.5
IPO: Based on your knowledge from this and previous chapters, what are some methods an investment banker uses to determine an IPO price? What factors will play a significant role in the calculation?
Solution: In order to determine the price of a new security, the investment banker can look at comparable companies and see what they are trading for. Another approach is to do a discounted cash flow analysis, which is just like determining the price of a stock. You forecast future cash flows of the company, usually for a time horizon of 5 to 10 years, and discount them back to the present in order to get the firm value. The discount rate will be a factor of both the cost of debt and cost of equity (WACC). LO 3 Bloomcode: Comprehension AASCP: Analytic IMA: Corporate Finance AICPA: Industry/Sector Perspective
SM 19-701
15.6
IPO: A majority of firms choose a firm-commitment underwriting arrangement rather than a best-effort arrangement for their IPO. Explain why.
Solution: With a firm-commitment arrangement, the issuing firm is guaranteed to receive a specific amount of money from proceeds of the IPO. The underwriter bears the price risk of the issue. Price risk is the risk that the resale price to the market might be lower than that of agreed price to the issuing firm. LO 3 Bloomcode: Comprehension AASCP: Analytic IMA: Corporate Finance AICPA: Industry/Sector Perspective
15.7
Competitive versus negotiated sale: Why might a negotiated sale be the lowest cost means of issuing a complex debt security?
Solution: Debt issues that are complex in nature and/or issued at times of uncertainty often tend to be brought to the market by negotiated sales. This approach allows the underwriter to better control the uncertain situation and better explains the firm to potential investors, thereby leading to lower issue costs. LO 5 Bloomcode: Comprehension AASCP: Analytic IMA: Corporate Finance AICPA: Industry/Sector Perspective
15.8
IPO pricing: Trajax, Inc., a high-technology firm in Portland, raised a total of $90 million in an IPO. The company received $27 of the $30 per share offering price. The firm’s legal fees, SEC registration fees, and other out-of-pocket costs were $450,000. The firm’s stock price increased 17 percent on the first day of trading. What was the total cost to the firm of issuing the securities?
Solution: The total costs to issue the securities are as follows: 1. Underwriting Spread: Underwriter’s spread ($30 per share - $27 per share) =$3 per share. SM 19-702
Number of shares outstanding = ($90 million/$30 per share) = 3,000,000 shares. Underwriting cost = ($3 per share × 3,000,000 shares) = $9,000,000. 2. Out-of-Pocket Expenses: Out-of-pocket expenses are $450,000 . 3. Underpricing: Stock price at end of first day = $30 per share (1.17) = $35.10 per share. First-day underpricing = ($35.10 per share - $30.00 per share) = $5.10 per share. Total underpricing = ($5.10 per share × 3,000,000 shares) = $15,300,000. Total cost to the firm of selling the IPO = $9,000,000 + $450,000 + $15,300,000 = $24,750,000
LO 4 Bloomcode: Application AASCP: Analytic IMA: Corporate Finance AICPA: Industry/Sector Perspective
15.9
IPO pricing: Myriad Biotech management plans a $114 million IPO in which the offering price to the public will be $51 per share. The company will receive $47.50 per share. The firm’s legal fees, SEC registration fees, and other out-of-pocket costs will total $525,000. If the stock price increases 14 percent on the first day of trading, what will be the total cost of issuing the securities?
Solution: The total costs to issue the securities are as follows: 1. Underwriting Spread: Underwriter’s gross spread ($51per share - $47.50 per share) =$3.50 per share. Number of shares outstanding = ($114 million/$51 per share) = 2,235,294 shares. Underwriting cost = ($3.50 per share × 2,235,294 shares) = $7,823,529. 2. Out-of-Pocket Expenses: Out-of-pocket expenses are $525,000. 3. Underpricing: Stock price at end of first day = ($51per share × 1.14) = $58.14 First-day underpricing = ($58.14 per share - $51per share) = $7.14 per share. Total underpricing = ($7.14 per share × 2,235,294 shares) = $15,959,999. Total cost to the firm of selling the IPO = $7,823,529 + $525,000 + $15,959,999 = $24,308,528
LO 4 Bloomcode: Comprehension AASCP: Analytic IMA: Corporate Finance AICPA: Industry/Sector Perspective
SM 19-703
15.10 Shelf registration: Are the following statements true or false? a. Shelf registration allows firms to register an inventory of securities for an unlimited time. b. The securities can be taken off the shelf at any time within 2 years of such registration and sold to the public. c. Shelf registration reduces flotation and other expenses associated with registration. d. There is a large penalty if the authorized securities are not issued. e. A shelf registration can cover multiple securities. Solution: a. False b. True c. True d. False e. True LO 5 Bloomcode: Knowledge AASCP: Analytic IMA: Corporate Finance AICPA: Industry/Sector Perspective
15.11 General cash offer: What are the steps in a general cash offering? Explain each of them. Solution: (1) Type of security and amount to be raised. (2) Approvals. (3) Registration statement. (4) Offer price and (5) Closing. LO 5 Bloomcode: Knowledge AASCP: Analytic IMA: Corporate Finance AICPA: Industry/Sector Perspective 15.12 General Cash Offer: Explain the difference between a competitive and negotiated cash sale. Which method of sale is likely to yield the lowest funding cost for firms selling plain vanilla bonds in stable markets? Solution: In a competitive offer, an investment bank does the origination work for the securities to be issued and then invites underwriters to competitively bid for the issue. The investment banking firm that pays the highest price for the securities wins the bid. The winning underwriter then pays for the securities and makes them available to individual investors at the offer price. In a negotiated offer, an underwriter works with the issuing firm to both originate and sell the firm’s securities to individual investors. SM 19-704
It is generally assumed that competitive offers yield the lowest cost of funding for firms issuing plain vanilla bonds. This is because the terms of the loans are standardized and well-known to the market participants. LO 5 Bloomcode: Analysis AASCP: Analytic IMA: Corporate Finance AICPA: Industry/Sector Perspective
15.13 Issuing securities: Explain what is meant by economies of scale in issuing securities. Solution: The economies of scale in issuing securities means that as the size of the issue increases, the total issue cost, as a percentage of the amount raised, declines. High fixed costs get spread out over a larger sum. LO 5 Bloomcode: Comprehension AASCP: Analytic IMA: Corporate Finance AICPA: Industry/Sector Perspective
15.14 Bank term lending: Explain how term to maturity affects the price of a bank loan. Solution: Term to maturity (MAT) will essentially increase the borrowing rate. It is defined as the difference between the yield on a Treasury security with the same maturity as the term loan and the yield on 3-month Treasury bill rate. Thus, the longer the loan, the higher the borrowing rate will be. LO 7 Bloomcode: Comprehension AASCP: Analytic IMA: Corporate Finance AICPA: Industry/Sector Perspective
15.15 Private placement versus public debt offering: Nalco Holding is an international company that operates in 130 countries, has a market capitalization (market value of equity) of $2.3 billion, and reported net income of $45 million on $3.3 billion in revenues last year. The company needs to raise $200 million in debt, and management is deciding SM 19-705
between private placement and public offering. What are the advantages and disadvantages of the two alternatives, and which is likely to be the best choice? Solution: Public market allows for higher security since SEC is involved, it offers more liquidity, which might be important for a large issue, and most of the times to earn higher yield. Private placement, on the other hand, offers faster speed to market, lower issuance cost, and ease of restructuring. Given the size of the Nalco Holdings, the company should is better off choosing to raise debt in public market. LO 6 Bloomcode: Analysis AASCP: Analytic IMA: Corporate Finance AICPA: Industry/Sector Perspective
15.16 Prime-rate lending: Suppose two firms want to borrow money from a bank for a period of one year. Firm A has excellent credit, whereas Firm B’s credit standing is such that it would pay prime + 2 percent. The current prime rate is 6.75 percent, the 30-year Treasury bond yield is 4.35 percent, the three-month Treasury bill yield is 3.54 percent, and the 10year Treasury note yield is 4.22 percent. What are the appropriate loan rates for each firm? Solution: Firm A = Prime rate = 6.75% Firm B = Prime rate + 2% = 8.75% LO 7 Bloomcode: Application AASCP: Analytic IMA: Corporate Finance AICPA: Industry/Sector Perspective
15.17 Prime-rate lending: Now suppose that Firm B from Problem 15.16 decides to get a term loan for 10 years. How does this affect the company’s borrowing cost? Solution: Maturity risk premium = k10-year – kT-bill = 4.22% - 3.54% = 0.68% Borrowing rate for firm B = k = PR + DRP + MAT = 6.75% + 2% + 0.68% = 9.43% SM 19-706
LO 7 Bloomcode: Application AASCP: Analytic IMA: Corporate Finance AICPA: Industry/Sector Perspective
15.18 Prime-rate lending: Cartco needs to borrow $5 million for an upgrade to its headquarters and manufacturing facility. Management has decided to borrow using a five-year term loan from its existing commercial bank. The prime rate is 3.25 percent, and Cartco’s current rating is prime + 2.48 percent. The yield on a five-year U.S. Treasury note is 2.01 percent, and the three-month U.S. Treasury bill rate is 0.09 percent. What is the estimated loan rate for the five-year bank loan? Solution: kl = PR + DRP + MAT MAT = y5-year - y3-mo = 2.01% – 0.09% = 1.92% PR = 3.25% DRP = 2.48% = 3.25% + 2.48% + 1.92% = 7.65% LO 7 Bloomcode: Application AASCP: Analytic IMA: Corporate Finance AICPA: Industry/Sector Perspective
INTERMEDIATE 15.19 Venture capital: You work for a venture capital firm and are approached to finance a new high-tech start-up. While you believe in the business idea, you also believe it is very risky. What strategies can help to mitigate the risk to your firm? Explain how these measures would work. Solution: You can fund the project in stages. This will allow you to review the project’s profitability before you commit to further financing. You may also require the entrepreneurs to invest some of their own capital in the new start-up, which will tie them to the project and make it harder for them to simply abandon the idea. LO 2 SM 19-707
Bloomcode: Comprehension AASCP: Analytic IMA: Corporate Finance AICPA: Industry/Sector Perspective
15.20 IPO: On August 19, 2004, Google completed its IPO of 19.6 million shares to the initial investors at $85.00 per share. The closing price of the stock that same day was $100.34. What was the dollar value of the underpricing associated with the Google IPO? Solution: With a closing price of $100.34 per share, the market value of Google shares was $1.97 billion after the issue. The shares issued at $85 yielded proceeds of $1.67 billion for Google, therefore total underpricing was $300 million. LO 4 Bloomcode: Application AASCP: Analytic IMA: Corporate Finance AICPA: Industry/Sector Perspective
15.21 IPO: Deere and Bros. is a broker that brings new issues of small firms to the public market. Its most recent deal for Dextra, Inc., had the following characteristics: Number of shares: 1,000,000 Price to public: $15 per share Proceeds to Dextra: $13,500,000 The legal fees were $150,000, printing costs were $56,000, and all the other expenses were $72,000. What is the profit or loss for Deere and Bros.? Solution: Total proceeds from offer = $15 × 1,000,000 shares = $15 million Proceeds to Dextra = $13.5 million Gross underwriting spread to Deere = $15 million - $13.5 million = $1.5 million Underwriting costs = $150,000 + 56,000 + $72,000 = $278,000 Net profit to underwriter = $1,500,000 - $278,000 = $1,222,000
LO 3 Bloomcode: Application AASCP: Analytic IMA: Corporate Finance AICPA: Industry/Sector Perspective SM 19-708
15.22 IPO: When Global Partners went public in September 2013, the offer price was $22.00 per share and the closing price at the end of the first day was $23.90. The firm issued 4.9 million shares. What was the loss to the company due to underpricing? Solution:
Change in price on first day = $23.90 - $22.00 = $1.90 Number of shares outstanding = 4.9 million Loss due to underpricing = $1.90 × 4,900,000 = $9,310,000 LO 4 Bloomcode: Application AASCP: Analytic IMA: Corporate Finance AICPA: Industry/Sector Perspective
15.23 IPO: Bellex Technologies agreed to complete its IPO on a best-effort basis. The company’s investment bank demanded a spread of 17 percent of the offer price, which was set at $30 per share. Three million shares were issued; however, the bank’s management was overly optimistic and eventually was able to sell all of the stock for only $28 per share. What were the proceeds for the issuer and the underwriter? Solution:
Gross proceeds from offer = $30.00 × 3,000,000 = $90,000,000 Underwriting spread = $90,000,000 × 0.17 = $15,300,000 Proceeds to issuer = ($28 × 3,000,000) − $15,300,000 = $68,700,000 LO 3 Bloomcode: Application AASCP: Analytic IMA: Corporate Finance AICPA: Industry/Sector Perspective
15.24 IPO: Suppose a biotech company in Boston, Massachusetts, completes an $85 million IPO priced to the public at $75 per share. The firm receives $72 per share, and the out-ofpocket expenses are $340,000. The stock’s closing price at the end of the first day is $84. What is the total cost to the firm of issuing the securities? SM 19-709
Solution: The total cost to issue the securities is calculated as follows: Underwriter’s spread = $75 per share – $72 per share = $3 per share No. of shares outstanding = $85,000,000 / $75 = 1,133,333 shares Underwriting cost = $3 per share × 1,133,333 = $3,399,999
First-day underpricing per share = $84 – $75 = $9.00 Total underpricing on day 1 = $9.00 × 1,133,333 shares = $10,199,997 Total cost to the firm = $10,199,997 + $3,399,999 + $340,000 = $13,939,996 LO 4 Bloomcode: Application AASCP: Analytic IMA: Corporate Finance AICPA: Industry/Sector Perspective
15.25 IPO: An online medical advice company just completed an IPO with an investment bank on a firm-commitment basis. The firm issued five million shares of common stock, and the underwriting fees were $1.90 per share. The offering price was $26 per share. a. What were the total proceeds from the common-stock sale? b. How much money did the company receive? c. How much money did the investment bank receive in fees? Solution:
a. Total proceeds from issue = $26.00 per share × 5,000,000 = $130,000,000 b. Net proceeds to firm = ($26 per share - $1.90 per share) x 5,000,000 = $120,500,000 c. Underwriting spread = $1.90 per share x 5,000,000 = $9,500,000 LO 3 Bloomcode: Application AASCP: Analytic IMA: Corporate Finance AICPA: Industry/Sector Perspective
15.26
IPO underpricing: Suppose that a biotech firm in Pittsburgh raised $120 million in an IPO. The firm received $23 per share, and the stock sold to the public for $25 per share. The firm’s legal fees, SEC registration fees, and other out-of-pocket costs were $270,000. The firm’s stock price increased 17.5 percent on the first day. What was the total cost to the firm of issuing the securities?
Solution: The total cost to issue the securities is calculated as follows: SM 19-710
a.
b.
Underwriting Cost: Underwriter’s spread = $25per share - $23 per share = $2 per share No. of shares outstanding = $120,000,000 / $25 = 4,800,000 shares Underwriting cost = $2 × 4,800,000 = $9,600,000 Underpricing:
Price of stock at end of first day = $25 per share × (1.175) = $29.375 Underpricing per share = $29.375 - $25 = $4.375 Total underpricing on day 1= $4.375 × 4,800,000 = $21,000,000 Total cost to the firm= $21,000,000 + $9,600,000 + $270,000 = $30,870,000 LO 4 Bloomcode: Application AASCP: Analytic IMA: Corporate Finance AICPA: Industry/Sector Perspective
13.27 Long-term corporate debt: The 20-year Treasury rate is 4.67 percent, and a firm’s credit rating is BB. Suppose management of the firm decides to raise $20 million by selling 20-year bonds. Management determines that since it has plenty of experience, it will not need to hire an investment banker. At present, 20-year BB bonds are selling for 185 basis points above the 20-year Treasury rate, and it is forecast that interest rates will not stay this low for long. What is the cost of borrowing? What role does timing play in this situation? Solution: Cost of borrowing = 4.67% + 1.85% = 6.52% Cost of borrowing is 6.52 percent. Since the economy is supposed to improve, the interest rates are expected to go up in the near future, which could make the cost of borrowing more expensive. Time is of essence in this case. LO 5 Bloomcode: Analysis AASCP: Analytic IMA: Corporate Finance AICPA: Industry/Sector Perspective
Sample Test Problems
SM 19-711
15.1
Why are traditional sources of funding not usually available for new or emerging businesses? (LO 2)
Solution: There are three reasons such businesses often have little or no access to traditional funding: 1) The high degree of risk associated with them. 2) Their primary assets are often intangible assets that are not as well understood by lenders or other potential investors as tangible assets 3) Potential investors often have limited knowledge of highly specialized technologies or new business areas and therefore find it difficult to assess the quality of their management teams or their prospects. Bloomcode: Comprehension AASCP: Analytic IMA: Corporate Finance AICPA: Industry/Sector Perspective
15.2
A firm is making an initial public offering. The investment bankers agree to a firm underwriting commitment for 500,000 shares that would be priced to the public at $36 a share. The underwriter’s spread is 7 percent. What were the proceeds for the issuer and the underwriter? (LO 3) Solution:
Gross proceeds from offer = $36 × 500,000 = $18,000,000 Underwriting spread = Proceeds to Underwriter = $18,000,000 × 0.07 = $1,260,000 Proceeds to issuer = ($36 × 500,000) − $1,260,000 = $16,740,000 Bloomcode: Application AASCP: Analytic IMA: Corporate Finance AICPA: Industry/Sector Perspective
SM 19-712
15.3
Hilton Worldwide Holdings Inc. completed an initial public offering on December 12, 2013. The offer price was $20.00 per share and the closing price at the end of the first day was $21.50. The firm issued
117.6 million shares. What was the loss to Hilton due to underpricing? Who received this value? (LO 4) Solution:
Change in price on first day of trading = $21.50 - $20.00 = $1.50 Number of shares outstanding = 117.6 million Loss due to underpricing = $1.50 × 117.6 million = $176.4 million The initial investor, who purchased the shares allocated by the underwriters of the IPO received this value. Bloomcode: Application AASCP: Analytic IMA: Corporate Finance AICPA: Industry/Sector Perspective
15.4
SMA Inc. is considering issuing the following securities. For which security would a competitive sale be less costly than a negotiated sale under stable market conditions? Why? (LO 5) a) Plain vanilla bonds. b) IPO of common stock. c) Secondary offering of common stock. d) Convertible bonds.
Solution:
a) Plain vanilla bonds. A competitive sale will be the least costly for bonds with standardized features in a stable market. In a stable market investors better understand the risks associated with these securities and are more comfortable purchasing them. The other three securities are more complex and these complexities are better handled through negotiated sales in which the SM 19-713
underwriter can address investor uncertainty concerning the securities and better explain the firm’s situation. This will result in a lower funding cost for complex securities.
Bloomcode: Comprehension AASCP: Analytic IMA: Corporate Finance AICPA: Industry/Sector Perspective
SM 19-714
15.5
Management of Southern Parts Company has decided to sell 10-year bonds to finance expansion into the Pacific Northwest. The loan rate on these bonds is 8 percent and the 3month Treasury bill rate is 2.1 percent. The firm’s credit rating is B, and the yield on 10year Treasury bonds is 2.5 percent higher than that on 3-month Treasury bills. How much of a premium over the 10-year Treasury bond are these Southern Parts bonds selling for? (LO 7) Solution:
kl = PR + DRP + MAT 8.0% = 2.1% + DRP + 2.5% DRP = 3.4% Bloomcode: Application AASCP: Analytic IMA: Corporate Finance AICPA: Industry/Sector Perspective
Chapter 16 Capital Structure Policy Before You Go On Questions and Answers Section 16.1 7.
What is the optimal capital structure for a firm? The optimal capital structure for a firm is the combination of debt and equity financing, which minimizes the overall cost of financing the firm’s real activities (projects). Minimizing the overall cost of financing a firm’s real activities maximizes the value of its cash flows.
8.
What is M&M Proposition 1? M&M Proposition 2? Modigliani-Miller Proposition 1: The capital structure of a firm does not affect the firm’s value if the following three assumptions hold: SM 19-715
a. There are no taxes. b. There are no information or transaction costs. c. A firm’s capital structure does not affect the firm’s real investment policy. Modigliani-Miller Proposition 2: The required rate of return on a firm’s equity (cost of equity) increases as the firm’s debt-to-equity ratio increases. 3.
What is the difference between business risk and financial risk? Business risk reflects the uncertainty associated with the underlying assets of the firm. The level of business risk is reflected in the required rate of return associated with these assets. Financial risk represents the uncertainty that fixed debt payments introduce to the cash flows to the stockholders. The total risk faced by stockholders is affected by both the business risk of the firm and its financial risk.
4.
How can the three conditions specified by M&M help us understand how the capital structure of a firm affects its value? M&M identify the three conditions under which the capital structure of a firm would not affect its value. By examining the three conditions specified by M&M, and how they are violated in the real world, we can better understand why capital structure does affect firm value.
Section 16.2 1.
What are some benefits of using debt financing? The benefits of using debt financing are: The interest tax shield (interest is tax deductable, dividends are not). The cost of issuing debt (underwriting spreads and out-of-pocket costs) are lower than the cost of issuing equity. With more debt, managers have greater incentives to work hard to produce larger cash flows and thus avoid bankruptcy. Increased debt limits the manager’s flexibility to spend money on wasteful, negative NPV projects.
2.
What are bankruptcy costs, and what are the two types of bankruptcy costs? Bankruptcy costs are costs that firms face when they are in financial distress. Direct bankruptcy costs are out-of-pocket costs, such as the payments that are made to lawyers, accountants, and consultants, as well as court costs, when a firm gets into financial distress. Indirect bankruptcy costs result from changes in the behavior of the people that SM 19-716
contract with the firm when they learn that it is in distress. For example, there may be less demand for a distressed firm’s product because of the concern that the firm will not exist to provide customer support in the future or suppliers might demand cash on delivery out of concern that the firm will not be able to pay for supplies. 3.
What are agency costs, and how are they related to the use of debt financing? Agency costs are costs that are incurred when someone delegates decision-making authority to someone who has different objectives. Agency costs might arise in relationships between stockholders and the managers they hire because managers do not have the same incentives as stockholders. For example, when a firm has debt, managers can have incentives to invest in low-risk projects that do not maximize stockholder value. Agency costs also arise between stockholders and debt holders. Stockholders, through the managers they hire, can have incentives to engage in asset substitution, turn down positive-NPV projects (underinvestment), or pay out excess cash in the form of dividends.
Section 16.3 1.
What is the trade-off theory of capital structure? The trade-off theory of capital structure postulates that managers set a specific target for the capital structure of a firm in which they try to achieve the mix of debt and equity that will minimize the cost of financing the firm’s projects and thereby maximize its value.
2.
What is the pecking order theory of capital structure? The pecking order theory says that, instead of trying to achieve a specified target capital structure, firms use the cheapest form of capital available at any given time, until it is used up. According to this theory, the cost of capital from least expensive to most expensive is: 1. Internally generated funds 2. Debt issue 3. Equity issue
3.
What does the empirical evidence tell us about the two theories? The empirical evidence provides some support for both theories. Studies find that industries with significant tangible assets tend to have more debt. This is consistent with the trade-off theory. These companies can generally borrow at a lower rate using their tangible assets as collateral. SM 19-717
There is also evidence that more profitable firms hold less debt. This is consistent with the pecking order theory. These companies are capable of financing more projects with internal funds before taking on debt. The trade-off theory would suggest that more profitable companies should carry more debt, because they have more to benefit from the tax shelter of interest payments.
Section 16.4 1.
Why is financial flexibility important in the choice of a capital structure? The choice of capital structure may limit the ability of managers to take advantage of unexpected opportunities or to deal appropriately with unexpected problems. For example, suppose the firm learns of a positive-NPV project that is available for only a short period of time. If the firm already holds significant amounts of debt, lenders may be unwilling to loan more money to the firm. Raising funds though issuing equity is often a slow process, and the window of opportunity to accept the project may have passed. A firm that holds cash or can quickly issue debt (e.g., through a line of credit) has more flexibility in financing new projects.
2.
How can capital structure decisions affect the risk associated with net income? The more debt a firm carries, the more risk will be associated with the firm’s net income. This occurs because given a certain level of fluctuating operating income, a larger fixed debt payments will magnify the effects of the fluctuations on net income.
3.
How can capital structure decisions affect the control of a firm? Issuing new equity reduces the control of the firm’s existing stockholders (assuming the existing stockholders do not purchase the new issues). In some cases firms may issue debt rather than equity to prevent the control of existing stockholders from being diluted. For example, if a firm is 51 percent owned by the founders, the founders may require the company to issue debt rather than equity to finance a new project. This will prevent the founder’s ownership from falling to less than 50 percent.
Appendix A16 1.
What is a lease? What are the two types of leases? A lease is an alternative way of financing the acquisition of an asset. In a lease agreement the lessee pays for the right to use the asset during the term of the lease while the lessor, who owns the asset, receives the lease payments in return for giving up the right to use the asset during the term of the lease. The lessor retains the right to use the asset after the term of the lease. The two types of SM 19-718
leases that the IRS recognizes are: (1) leases which are truly rentals called operating leases, and (2) leases that have the key elements of an outright sale, known as capital leases. 2. What is the primary motivation for leasing? The primary motivation for leasing an asset is that leasing is a less expensive way of obtaining the use of an asset for a period of time than purchasing the asset. 3. What types of conflicts arise with leases and why? The two conflicts that arise with leases are the (1) intensity of use and (2) maintenance conflicts. These conflicts arise because a lease separates the right to use an asset during the term of a lease from the right to use the asset afterwards. Since two different parties own these rights (the lessee and the lessor) conflicts of interest are bound to arise.
Self Study Problems 16.1
If any of the three assumptions in Modigliani and Miller Proposition 1 are relaxed, which has the most predictably quantifiable impact on the value of the firm?
Solution: The assumption with the most measurable impact is that involving taxes. We can calculate the present value of the tax shield generated by the interest costs of borrowing. The impacts of the other two assumptions, though real, are more difficult to predict.
16.2
If we assume that the cash flows for a firm with financial leverage are equal to the cash flows for the same firm without financial leverage, what can we say about the value of this firm if its cost of capital also does not vary with the degree of leverage utilized?
Solution: If the firm’s cash flows and cost of capital are the same, regardless of the amount of leverage utilized, then we can say that the value of the firm is also unchanged by the amount of financial leverage.
16.3
Are taxes necessary for the cost of debt financing to be less than the cost of equity financing?
Solution: SM 19-719
The deduction for interest expense does make debt borrowing more attractive than it would otherwise be. However, even without the interest deduction benefit, the cost of debt is less than the cost of equity, because equity is a riskier investment than debt. This means that the pretax cost to the firm for debt is still lower than the cost of equity.
16.4
You are offered jobs with identical responsibilities by two different firms in the same industry. One has no debt in its capital structure, and the other has 99 percent debt in its capital structure. Will you require a higher level of compensation from one firm than from the other? If so, which firm will have to pay you more?
Solution: The firm with the large amount of debt financing (the 99 percent debt firm) has a higher probability of becoming financially distressed. Therefore, you should require greater compensation from that firm because your income is less certain and working at that firm poses a greater risk to your career..
16.5
You are valuing two otherwise identical firms in the same industry. One firm has a corporate jet for every executive at the vice president level and above, while the other does not have a single corporate jet. More than likely, which firm has the greatest stockholder-manager agency costs?
Solution: While corporate jets can make economic sense because they enable managers to use their time more efficiently, one jet per vice president is certainly not cost effective. The multijet firm has higher stockholder-manager agency costs than the no-jet fi rm. It is spending too much on jets. The cash that is being spent on excess jets could be invested in positive NPV projects or returned to the firm’s stockholders.
Discussion Critical Thinking Questions 16.1 List and briefly describe the three key assumptions in Modigliani and Miller’s Proposition 1 that are required for total firm value to be independent of capital structure. 1. There are no taxes. This assumption is necessary in order to avoid creating a tax benefit when using debt instead of equity to finance firm assets. 2. There are no information or transactions costs for the firm.
SM 19-720
a. There are no bankruptcy costs (financial distress) to the firm associated with increased supplier, employee, and customer expenses, due to the firm having a higher likelihood of not being able to meet its debt obligations. Note that one interpretation of this assumption is that, the firm has a zero probability of not being able to meet its debt obligations. b. There are no agency costs. There are no costs associated with the conflict of interest between managers and stockholders, and there are no costs associated with the conflict between the interest between stockholders and debt holders of the firm. c. There are no market frictions. This means that individuals may borrow and lend at the same rate that the firm can borrow and lend. 3. The real investment policy of the firm is not affected by its capital structure. decisions. Without this, the assumption that an individual investor could replicate the actions of the firm, does not hold because the actual investment policy of the firm would be affected by the capital structure decision. Investors cannot replicate real investments. LO: 1 Level: Basic Bloomcode: Knowledge AASCP: Analytic IMA: Corporate Finance AICPA: Industry/Sector Perspective 16.2 Evaluate the statement that the weighted average cost of capital (WACC) for a firm (assuming that all three assumptions of Modigliani and Miller’s propositions hold) is always less than or equal to the cost of equity for the firm. If we look at the following WACC formula for a firm (without taxes): WACC = [Vcs/(VDebt+Vcs)] × kcs + [VDebt/(VDebt+Vcs )] × kDebt We see that with 100 percent equity financing, the WACC = kcs. A firm cannot be 100 percent debt-financed, even so as D/V approaches 1, then the WACC approaches kDebt. Since we know that debt is less risky than equity, we know that the WACC in that instance, must be greater than kDebt but less than kcs. Therefore, the WACC must be greater than kDebt but less than or equal to kcs . LO: 1 Level: Basic Bloomcode: Analysis AASCP: Analytic IMA: Corporate Finance AICPA: Industry/Sector Perspective
SM 19-721
16.3
If the value of the firm remains constant as a function of its capital structure and the three Modigliani and Miller assumptions apply, why might the overall cost of capital change or not change as capital structure changes?
If the cash flow produced by a firm is unchanged, and the value of the firm producing that cash flow is unchanged by a firm’s capital structure changes, we can use a perpetual cash flow valuation model as follows: VFirm = CF/i If we know that VFirm and CF are unchanged due to capital structure changes; then kAssets must also remain unchanged. LO: 1 Level: Basic Bloomcode: Analysis AASCP: Analytic IMA: Corporate Finance AICPA: Industry/Sector Perspective 16.4
Consider the WACC for a firm that pays taxes. Explain what a firm’s best course of action would be to minimize its WACC and thereby maximize the firm value. Use the WACC formula for your explanation.
WACC = (1 – t) × kDebt × [VDebt/(VDebt+Vcs)] + kcs × [Vcs/(VDebt+Vcs)] If kcs > kDebt, then it would make sense to increase debt relative to equity, in order to raise as much financing as possible with borrowing, rather than the more expensive equity. Therefore, by blindly following the WACC formula, we are led to believe that more debt will increase the value of the firm from its current valuation. This highlights the effect of taxes without taking into account the other M&M assumptions. LO: 1 Level: Basic Bloomcode: Comprehension AASCP: Analytic IMA: Corporate Finance AICPA: Industry/Sector Perspective 16.5
The Modigliani and Miller propositions, when the no-tax assumption is relaxed, suggest that the firm should finance itself with as much debt as possible. Taking this suggestion to the extreme, is it even possible to finance a firm with 100 percent debt and no equity? Why or why not? Even with the list of assumptions required to get this result, a 100 percent debt firm is not practical. While we might think that there is no residual claimant for the firm’s cash flows, even this is not true. In fact, a 100 percent debt-financed firm is really a 100 SM 19-722
percent equity-financed firm since the debt holders themselves must then be the residual claimants. LO: 1 Level: Basic Bloomcode: Comprehension AASCP: Analytic IMA: Corporate Finance AICPA: Industry/Sector Perspective 16.6
Crossler Automobiles sells autos in a market where the standard auto comes with a 10-year/100,000-mile warranty on all parts and labor. Describe how an increased probability of bankruptcy could affect sales of autos by Crossler. In a market, where a warranty is a significant portion of the cost of an automobile, purchasing a car where the seller might not be able to completely perform on that warranty,due to increased probability of bankruptcy of Crossler, would have negative impact on the firm’s future automobile sales. This would decrease the amount of cash flow available to the investors of the firm, which would lower the value of the firm.
LO: 2 Level: Basic Bloomcode: Application AASCP: Analytic IMA: Corporate Finance AICPA: Industry/Sector Perspective 16.7
Agency problems occur because the nonowner managers and stockholders of a firm have different interests. Propose a capital structure change that might help better align these different interests. The managers of a firm would rather work as little as possible, given a set level of compensation, whereas stockholders would rather compensate the manager for a high level of effort, with a low effort receiving very little compensation. Increasing the proportion of debt in the capital structure would increase the probability of the firm falling into bankruptcy. This increased probability of bankruptcy, and therefore the increased chance that the manager will lose his job, would then help to align the interests of the manager and stockholders by giving the manager an additional incentive to work harder for the stockholders.
LO: 2 Level: Basic Bloomcode: Analysis AASCP: Analytic IMA: Corporate Finance SM 19-723
AICPA: Industry/Sector Perspective 16.8
If a firm increases its debt to a very high level, then the positive effect of debt in aligning the interests of management with those of stockholders tends to become negative. Explain why this occurs. Increasing the debt level for a firm tends to catch management’s attention and force the managers to work harder; therefore, a very high level of debt can be detrimental. That is, at very high levels of debt, risk-averse managers begin to minimize the risks that a firm takes on, for the managers’ own job preservation needs. This risk minimization can deter managers from taking risky, but positive-NPV projects, which are needed to help the firm meet the very debt obligations that are causing the problem. In addition, the financially risky firm may also bear costs not previously borne in relationships with employees, suppliers, and customers.
LO: 2 Level: Basic Bloomcode: Comprehension AASCP: Analytic IMA: Corporate Finance AICPA: Industry/Sector Perspective 16.9 Using the Modigliani and Miller framework but excluding the assumptions that there are no taxes and no information or transaction costs, describe the value of the firm as a function of the proportion of debt in its capital structure. With taxes included, we would predict that the firm would take on greater proportions of debt. Agency costs (part of the information and transactions cost) are actually reduced when adding moderate portions of debt. This would also predict that the firm value increases as we add moderate levels of debt to the firm. However, agency costs once again become positive (in total) as very high levels of debt are incurred. This suggests that at very high levels of debt the value of the firm could be dropping. Bankruptcy costs (which are also part of the information and transactions cost) are probably immaterial at low debt levels, but can become very high at higher levels of debt. Therefore, we should predict that by including dropping the three assumptions in the question, the value of the firm should be increase with additional debt for moderate debt levels while decreasing for high levels of debt. LO: 1 Level: Basic Bloomcode: Comprehension AASCP: Analytic IMA: Corporate Finance AICPA: Industry/Sector Perspective
SM 19-724
16.10 When we observe the capital structure of many firms, we find that they tend to utilize lower levels of debt than that predicted by the trade-off theory. Offer an explanation for this.
This empirical result is consistent with a firm maintaining a reserve level of debt or high cash levels in order to have ample internally generated funds for new projects. One explanation would be that, firms like to have this ―reserve‖ financing available for new projects when they are identified. Another explanation is that, firms do not have to offer an explanation to the new investors for the use of these funds and that makes it less expensive on an information basis to keep this reserve, compared to having to issue new securities for the financing of the new projects. Both of these explanations are in line with the pecking order theory. LO: 2 Level: Basic Bloomcode: Comprehension AASCP: Analytic IMA: Corporate Finance AICPA: Industry/Sector Perspective
Questions and Problems BASIC 16.1
M&M Proposition 1: The Modigliani and Miller theory suggests that the value of the firm’s assets is equal to the value of the claims on those assets and is not dependent on how the asset claims are divided. The common analogy to the theorem is that the total amount of pie available to be eaten (the firm) does not depend on the size of each slice of pie. If we continue with that analogy, then what if we cut up the pie with a very dull knife such that the total amount of pie available to be eaten is less after it is cut than before it was cut. Which of the three Modigliani and Miller assumptions, if relaxed, is analogous to the dull knife? Hint: Think about the process by which investors could undo the effects of a firm’s capital structure decisions.
Solution: The inability of individual investors to borrow and lend and at the same rate as firms, is analogous to the dull knife. This friction is part of the no information or transactions costs assumption. Since the process of rearranging the claims on the firm’s assets is similar to carving up the firm into different pieces, it is easy to see that such a market friction is similar to the dull knife in our pie example. SM 19-725
LO 1 Bloomcode: Comprehension AASCP: Analytic IMA: Corporate Finance AICPA: Industry/Sector Perspective 16.2 M&M Proposition 1: Describe what exactly is meant when someone is describing the value of the firm versus the value of the equity of the firm. Solution: The value of the firm can be described as either the total market value of all the assets owned by the firm, or the total market value of all the claims of all the investors in the firm. In that case, it means that we are valuing all of the equity claims (shares) in the firm in addition to all of the debt claims (bonds, bank borrowing, etc.) of the firm. LO 1 Bloomcode: Comprehension AASCP: Analytic IMA: Corporate Finance AICPA: Industry/Sector Perspective
16.3
M&M Proposition 1: Under Modigliani and Miller’s Proposition 1, where all three of the assumptions remain in effect, explain how the value of the firm changes due to changes in the proportion of debt and equity utilized by the firm.
Solution: Under Modigliani and Miller’s Proposition 1, the value of the firm is independent of the proportion of debt and equity utilized by the firm. LO 1 Bloomcode: Comprehension AASCP: Analytic IMA: Corporate Finance AICPA: Industry/Sector Perspective
16.4
M&M Proposition 1: Cerberus Security Company produces a cash flow of $200 per year and is expected to continue doing so in the infinite future. The cost of equity capital for Cerberus is 20 percent, and the firm is financed entirely with equity. Management would like to repurchase $100 in shares by borrowing $100 at a 10 percent rate (assume that the debt will also be outstanding into the infinite future). Using Modigliani and Miller’s Proposition 1, what is the value of the firm today, and what will be the value of the claims SM 19-726
on the firm’s assets after the stock repurchase? What will be the rate of return on common stock required by investors after the share repurchase? Solution: Using the 20 percent cost of equity capital, the value of the firm today is $200/0.2 = $1,000, keeping in mind that the cash flow is expected to be produced into the infinite future. Note that, using M&M Proposition 1, this must be the value of the firm after the repurchase. The value of the debt claim must be $100 at the time of the borrowing, so we know that the value of the outstanding equity after the repurchase must be $1,000 – $100 = $900. Since $10 in cash flow must be paid to the debt holders each year, $190 will be available to the stockholders on an annual basis. This implies that the new cost of equity capital for the firm must be such that $900 = $190 / kcs, or kcs = 21.11%. LO 1 Bloomcode: Application AASCP: Analytic IMA: Corporate Finance AICPA: Industry/Sector Perspective
16.5
M&M Proposition 1: A firm that is financed completely with equity currently has a cost of capital equal to 15 percent. If Modigliani and Miller’s Proposition 1 holds and the firm’s management is thinking about changing its capital structure to 50 percent debt and 50 percent equity, then what will be the cost of equity after the change if the cost of debt is 10 percent?
Solution: It is easy to see that the current cost of capital (also the cost of equity in this case) is 15 percent. Given the proposition, we know that the cost of capital (or WACC) for the firm must be constant at 15 percent. If we know that the firm can borrow at a 10 percent cost of capital, with a 50–50 percent debt-equity mix, then we can use the WACC formula to solve for the new cost of equity capital as follows: WACC xDebt kDebt xcskcs 0.15 = 0.5 × (0.10) + 0.5 × (kcs) kcs = 0.20, or 20% LO 1 Bloomcode: Application AASCP: Analytic IMA: Corporate Finance AICPA: Industry/Sector Perspective
SM 19-727
16.6
M&M Proposition 1: Swan Specialty Cycles is currently financed with 50 percent debt and 50 percent equity. The firm pays $125 each year to its debt investors (at a 10 percent cost of debt), and the debt has no maturity date. What will be the value of the equity if the firm repurchases all of its debt and raises the funds to do this by issuing equity? Assume that all of the assumptions in Modigliani and Miller’s Proposition 1 hold.
Solution: Since the debt has no maturity date, we can find the value of the current debt claims: Value = $125 / 0.10 = $1,250. Since debt is 50 percent of the capital structure, then the value of the entire firm must be 2 x Value of debt = $2,500, which must be the value of the equity in an all-equity firm. LO 1 Bloomcode: Application AASCP: Analytic IMA: Corporate Finance AICPA: Industry/Sector Perspective
16.7
M&M Proposition 1: The weighted average cost of capital for a firm, assuming all three Modigliani and Miller assumptions hold, is 10 percent. What is the current cost of equity capital for the firm if the cost of debt for the firm is 8 percent, and the firm is 80 percent financed with debt?
Solution: Using the formula given in the text: kcs = kAssets + (VDebt /Vcs) x (kAssets – kDebt) kcs = 0.10 + (0.8 / 0.2) × (0.10 – 0.08) = 0.18, or 18% LO 1 Bloomcode: Application AASCP: Analytic IMA: Corporate Finance AICPA: Industry/Sector Perspective
16.8
Interest tax shield benefit: Legitron Corporation has $350 million of debt outstanding at an interest rate of 9 percent. What is the dollar value of the tax shield on that debt, just for this year, if Legitron is subject to a 35 percent marginal tax rate?
Solution: SM 19-728
Legitron will pay $31,500,000 ($350,000,000 × 0.09) in interest this year, which will shield Legitron from paying a tax amount equal to: VTax-savings debt = D × t = ($31,500,000 × 0.35) = $11,025,000 Therefore, the amount of this year’s tax shield, due to debt issuance, for Legitron is $11,025,000. LO 2 Bloomcode: Application AASCP: Analytic IMA: Corporate Finance AICPA: Industry/Sector Perspective
16.9
Interest tax shield benefit: FAJ, Inc. has $500 million of debt outstanding at an interest rate of 9 percent. What is the present value of the tax shield on that debt if it has no maturity and if FAJ is subject to a 30 percent marginal tax rate?
Solution: The present value of FAJ’s tax shield is: t × D = 0.30 x $500,000,000 = $150,000,000 An alternative calculation would be: (t × D × kDebt ) / kDebt = (0.30 × $500,000,000 × 0.09) / 0.09 = $150,000,000 LO 2 Bloomcode: Application AASCP: Analytic IMA: Corporate Finance AICPA: Industry/Sector Perspective
16.10 Interest tax shield benefit: Springer Corp. has $250 million of debt outstanding at an interest rate of 11 percent. What is the present value of the debt tax shield if the debt has no maturity and if Springer is subject to a 40 percent marginal tax rate? Solution: The present value of Springer’s tax shield is: t × D = 0.40 × $250,000,000 = $100,000,000 LO 2 Bloomcode: Application AASCP: Analytic IMA: Corporate Finance AICPA: Industry/Sector Perspective
SM 19-729
16.11 Interest tax shield benefit: Structural Corp. currently has an equity cost of capital equal to 15 percent. If the Modigliani and Miller Proposition 1 assumptions hold, with the exception of the assumption that there are no taxes, and the firm’s capital structure is made up of 50 percent debt and 50 percent equity, what is the weighted average cost of capital for the firm if the cost of debt is 10 percent and the firm is subject to a 40 percent marginal tax rate? Solution: WACC xDebt kDebt xcskcs = (0.5) × (0.10) × (1-0.4) + 0.5 × (0.15) = 0.105, or 10.5% LO 2 Bloomcode: Application AASCP: Analytic IMA: Corporate Finance AICPA: Industry/Sector Perspective
16.12 Practical considerations in capital structure choice: List and describe three practical considerations that concern managers when they make capital structure decisions. Solution: 1.
2. 3.
Financial flexibility: Managers must minimize the firm’s cost of capital while also ensuring that the firm has the flexibility to raise new capital quickly to deal with unexpected problems or to take advantage of unexpected opportunities. Net income risk: Increasing the leverage of a firm increases the risk associated with firm’s net income, and the risk of default. Earnings impact: When a project is financed with debt, the interest payments reduce the accounting dollar value of net income. However, when debt is used, no new shares of equity are issued, so the company’s earnings per share would be expected to increase (given a positive NPV project). Although financial theory suggests that neither of these effects should matter, managers often take them into account when making financing decisions.
LO 4 Bloomcode: Knowledge AASCP: Analytic IMA: Corporate Finance AICPA: Industry/Sector Perspective
SM 19-730
INTERMEDIATE 16.13 M&M Proposition 1: Keyboard Chiropractic Clinics produces $300,000 of cash flow each year. The firm has no debt outstanding, and its cost of equity capital is 25 percent. The firm’s management would like to repurchase $600,000 of its equity by borrowing a similar amount at a rate of 8 percent per year. If we assume that the debt will be perpetual, find the cost of equity capital for Keyboard after it changes its capital structure. Assume that Modigliani and Miller Proposition 1 holds. Solution: Using the 25 percent cost of equity capital, the value of the firm today is $300,000/0.25 = $1,200,000 This must be the value of the firm after the repurchase. The value of the debt claim must be $600,000 at the time of the borrowing, so the value of the outstanding equity after the repurchase must be $1,200,000 – $600,000 = $600,000 Since $48,000 ($600,000 × 0.08) in cash flow must be paid to the debt holders each year, $252,000 ($300,000 – $48,000) will be available to the stockholders on an annual basis. This implies that the new cost of equity capital for the firm must be such that $600,000 = $252,000 / kcs, or kcs = 0.42, or 42.00% LO 1 Bloomcode: Application AASCP: Analytic IMA: Corporate Finance AICPA: Industry/Sector Perspective
16.14 M&M Proposition 1: Marx and Spender Corp. currently has a WACC of 21 percent. If the cost of debt capital for the firm is 12 percent and the firm is currently financed with 25 percent debt, then what is the current cost of equity capital for the firm? Assume that the assumptions in Modigliani and Miller’s Proposition 1 hold. Solution: Using the WACC formula we can solve for the cost of equity capital for the firm: WACC xDebt kDebt xcskcs 0.21 = 0.25 × (0.12) + 0.75(kcs) kcs = 0.24, or 24% Bloomcode: Application AASCP: Analytic IMA: Corporate Finance AICPA: Industry/Sector Perspective SM 19-731
16.15 M&M Proposition 1: What is the effect on Modigliani and Miller’s Proposition 1 of relaxing the assumption that there are no information or transaction costs? Solution: No information or transaction costs means, for example, that individuals may borrow and lend at the same rate that the firm can borrow and lend. If an investor were unable to sell (or buy) shares by investing (or borrowing) without transaction costs, then some of the value conserved in the Modigliani and Miller arguments through personal trading would be dissipated by transaction costs. This dissipation would then negate the conservation of value proposed by M&M. LO 1 Bloomcode: Comprehension AASCP: Analytic IMA: Corporate Finance AICPA: Industry/Sector Perspective
16.16 M&M Proposition 1: The weighted average cost of capital for a firm (assuming all three Modigliani and Miller assumptions apply) is 15 percent. What is the current cost of equity capital for the firm if its cost of debt is 10 percent and the proportion of debt to total firm value for the firm is 0.5? Solution: Since VDebt /VFirm = 0.5, VDebt /Vcs = 0.5 / 0.5 = 1 Using the formula given in the text, kcs = kAssets + (VDebt /Vcs) × (kAssets – kDebt) = 0.15 + (1) × (0.15 – 0.10) = 0.2, or 20% LO 1 Bloomcode: Application AASCP: Analytic IMA: Corporate Finance AICPA: Industry/Sector Perspective
16.17 M&M Proposition 2: Mikos Processed Foods is currently valued at $500 million. Mikos will be repurchasing $100 million of its equity by issuing a nonmaturing debt issue at a SM 19-732
10 percent annual interest rate. Mikos is subject to a 30 percent marginal tax rate. If all of the Modigliani and Miller assumptions apply, except the assumption that there are no taxes, what will be the value of Mikos after the recapitalization? Solution: Mikos will be worth $500,000,000 plus the present value of the tax shield. The present value of the tax shield is $100,000,000 × 0.3 = $30,000,000. Therefore, Mikos will be worth $530,000,000 after the recapitalization. Bloomcode: Application AASCP: Analytic IMA: Corporate Finance AICPA: Industry/Sector Perspective
16.18 M&M Proposition 2: Backwards Resources Company has a WACC of 12.6 percent, and it is subject to a 40 percent marginal tax rate. Backwards has $250 million of debt outstanding at an interest rate of 9 percent and $750 million of equity (at market value) outstanding. What is the expected return on the equity with this capital structure? Solution: Using the WACC formula when taxes are included: WACC = (1-t) × kDebt × (VDebt/VFirm) + kcs × (Vcs/VFirm) We can solve for the missing variable: 0.126 = (1-0.4) × 0.09 × [250 million / (250 million + 750 million)] + kcs × (750 million/ [250 million + 750 million]) kcs = 0.15, or 15% LO 1 Bloomcode: Application AASCP: Analytic IMA: Corporate Finance AICPA: Industry/Sector Perspective
16.19 The costs of debt: Briefly discuss costs of financial distress to a firm that may arise when employees believe it is highly likely that the firm will declare bankruptcy. Solution: If the employees of a firm understand that the firm has a significant chance of filing for bankruptcy, then costs to the firm could be manifested in a number of ways, including:
SM 19-733
1. Lower productivity due to lower morale and job hunting. This could be as simple as employees spending time gossiping about what is going to happen to them as well as employees actively pursuing other jobs while on the payroll of the troubled firm. 2. Higher recruiting costs. Understanding that working for the firm is a risky venture, new employees will seek compensation for this additional risk. Therefore, recruiting employees will become more expensive due to greater recruiting efforts as well as greater compensation expense when a new employee is finally located and hired. LO 2 Bloomcode: Comprehension AASCP: Analytic IMA: Corporate Finance AICPA: Industry/Sector Perspective
16.20 The costs of debt: Santa’s Shoes is a retailer that has just begun having financial difficulty. Santa’s suppliers are aware of the increased possibility of bankruptcy. What might Santa’s suppliers do based on this information? Solution: Santa’s Shoes is not certain to go into bankruptcy, so its suppliers would still like to do business with Santa’s Shoes as long as it is profitable to do so. Therefore, the suppliers would still make sales to Santa’s Shoes as long as payment for the sales was made at the time of purchase rather than on credit. This would require Santa’s Shoes to maintain a higher cash balance. This requirement to hold additional cash could be viewed as a cost of financial distress to Santa’s Shoes. LO 2 Bloomcode: Analysis AASCP: Analytic IMA: Corporate Finance AICPA: Industry/Sector Perspective
16.21 Stockholder-manager agency costs: Deficit Corp. management has determined that they will be $50 million short of being able to pay the firm’s debt obligations at the end of this year. Management has identified a positive NPV project that will require a great deal of effort on their part. However, this project is expected to generate only $40 million at the end of the year. Assume that all the members of Deficit’s management team will lose their jobs if the firm goes into bankruptcy at the end of the year. How likely is management to take the positive NPV project? If management declines the project, what kind of cost will Deficit’s stockholders incur?
SM 19-734
Solution: Managers expect to lose their jobs in one year whether they work hard or not and take the project or not. Although, there may be a slim chance that the firm will not declare bankruptcy, management has no incentive to take on the difficult project. This makes the shortage to the debt holders, as well as the stockholders, greater than it would be if the firm followed the rule of always accepting positive NPV projects. This is another example of agency costs that can arise from financial distress. LO 2 Bloomcode: Analysis AASCP: Analytic IMA: Corporate Finance AICPA: Industry/Sector Perspective
16.22 Two theories of capital structure: Use the information in the following table to make a suggestion concerning the proportion of debt that the firm should utilize in its capital structure. Benefit or (Cost) No Debt Tax shield $0 Agency cost −$10 Financial distress cost −$ 1
25% Debt $10 −$ 5 −$ 3
50% Debt $20 −$ 5 −$10
75% Debt $30 −$20 −$10
Solution: By totaling the cost and benefits for each proportion of debt we find: Benefit or (Cost) Total cost/benefit
No Debt
25% Debt -$11
50% Debt 75% Debt $2 $5 $0
Therefore, this firm can maximize firm value by choosing a 50 percent debt capital structure. LO 3 Bloomcode: Comprehension AASCP: Analytic IMA: Corporate Finance AICPA: Industry/Sector Perspective
16.23 Two theories of capital structure: Problem 16.22 introduces taxes and information and transaction costs to the simplified Modigliani and Miller model. If the marginal tax rate for SM 19-735
the firm were to suddenly increase by a material amount, would the capital structure that maximizes the firm’s value include less or more debt? Solution: If we hold all others things equal, then the value of the tax shield would become more valuable in the scenarios with positive debt amounts. While we cannot say for certain, given the information in the question, an increase in the tax rate will increase the value of the tax shield and should increase the amount of debt in the optimal capital structure. Therefore, it would appear that an increase in the tax rate should motivate firms to increase their debt levels. LO 3 Bloomcode: Analysis AASCP: Analytic IMA: Corporate Finance AICPA: Industry/Sector Perspective
16.24 Two theories of capital structure: Describe the order of financial sources for managers who subscribe to the pecking order theory of financing. Evaluate that order by observing the costs of each source relative to the costs of other sources. Solution: According to the pecking order theory, the costs, from lowest to highest, are: 1. Internally generated funds (this is essentially retained earnings)—This will actually be the cheapest source in this list. 2. New issue debt—This will be the costlier than internally generated funds cheapest, but relatively inexpensive but relatively inexpensive costlier than internally generated funds, but relatively inexpensive source in this list. 3. New issue equity—This will be the most expensive source in this list. It appears that the managers who subscribe to the pecking order theory, choose the cheapest sources of financing before moving on to more expensive sources. Bloomcode: Comprehension AASCP: Analytic IMA: Corporate Finance AICPA: Industry/Sector Perspective
SM 19-736
16.25 Two theories of capital structure: The pecking order theory suggests that managers prefer to first use internally generated equity to finance new projects. Does this preference mean that these funds represent an even cheaper source of funds than debt? Justify your answer. Solution: The internally generated equity is utilized first as a source of financing, but it does not mean that the internally generated funds are cheaper than debt. Internally generated funds belong to stockholders and, therefore, are really equity financing, which is more expensive than debt. However, using internally generated funds enables the firm to avoid the costs associated with borrowing or selling stock (including the costs associated with the signals that financing announcements send investors), which, in turn, can make internal funds most attractive. LO 3 Bloomcode: Comprehension AASCP: Analytic IMA: Corporate Finance AICPA: Industry/Sector Perspective
16.26 The costs of debt: Discuss how the legal costs of financial distress may increase with the probability that a firm will formally declare bankruptcy, even if the firm has not reached that point yet. Solution: If a firm is anticipating bankruptcy to a greater extent, then it will increase its legal efforts to protect the firm from creditors or if the firm reaches that point. Therefore, the legal costs of bankruptcy will increase with financial distress even if the firm has not yet declared bankruptcy. LO 2 Bloomcode: Comprehension AASCP: Analytic IMA: Corporate Finance AICPA: Industry/Sector Perspective
ADVANCED 16.27 Operating a firm without debt is generally considered to be a conservative practice. Discuss how such a conservative approach to a firm’s capital structure is good or bad for SM 19-737
the value of the firm in the absence of information or transaction costs and any effect of debt on the real investment policy of the firm. Solution: In the absence of information or transaction costs and any effect of debt on the real investment policy of the firm, value of the firm is increasing in proportion to debt in the firm’s capital structure, due to the present value of the tax shield on the debt. Therefore, although operating without debt may be a safer play for investors, it does not maximize stockholder value, which should be the goal of managers. LO 2 Bloomcode: Analysis AASCP: Analytic IMA: Corporate Finance AICPA: Industry/Sector Perspective
16.28 Finite Corp. has $250 million of debt outstanding at an interest rate of 11 percent. What is the present value of the debt tax shield if the debt will mature in five years (and no new debt will replace the old debt), assuming that Finite is subject to a 40 percent marginal tax rate? Solution: Finite will pay $27,500,000 ($250,000,000 × 0.11) in interest each year, which will shield Finite from paying a tax amount equal to $11,000,000 ($27,500,000 × 0.4). The tax shield will last for five years, so the present value of receiving this amount for the next five years is: $11,000,000 × PVIFA (11%, 5) = $11,000,000 × 3.695897 = $40,654,867.19 LO 2 Bloomcode: Application AASCP: Analytic IMA: Corporate Finance AICPA: Industry/Sector Perspective
16.29 The Boring Corporation is currently valued at $900 million, but management wants to completely pay off its perpetual debt of $300 million. Boring is subject to a 30 percent marginal tax rate. If Boring pays off its debt, what will be the total value of its equity? Solution: SM 19-738
Boring will be worth $900 million less the present value of the tax shield on its current debt. The present value of the tax shield is $300,000,000 × 0.3 = $90,000,000 Therefore, Boring will be worth $810 million after the recapitalization, and since it will be an all-equity firm, that will be the value of the equity. LO 2 Bloomcode: Application AASCP: Analytic IMA: Corporate Finance AICPA: Industry/Sector Perspective
16.30 If we drop the assumption that there are no information or transaction costs, in addition to dropping the no-tax assumption, then will the Modigliani and Miller model still suggest that the firm should take on greater proportions of debt in its capital structure? Explain. Solution: If we only drop the no-tax assumption, then it is evident that the firm should always increase its use of debt within a firm’s capital structure. However, if we drop the no information or transaction costs assumption, then we are effectively introducing the possibility that the firm will default on its debt obligations. The higher the level of debt, the greater the possibility of defaulting and then the greater the interest costs on the debt. Given this increasing cost of debt, relative to the increased use of debt within a firm’s capital structure, then we are introducing the possibility that the firm’s overall cost of capital will increase at some high-debt level capital structure, thereby reducing the value of the firm. Therefore, by dropping the no information or transaction costs assumption, the firm will not always increase its use of debt. LO 1 Bloomcode: Comprehension AASCP: Analytic IMA: Corporate Finance AICPA: Industry/Sector Perspective
16.31 PolyAna Corporation has an abundant cash flow. It is so high that the managers take Fridays off for a weekly luncheon in Cancun using the corporate jet. Describe how altering the capital structure of the firm might make the management of this firm stay in the office on Fridays in order to work on new positive NPV projects. SM 19-739
Solution: The root of the problem is that the firm’s management is too comfortable, because their weekly trip to Cancun is not costly enough to the managers of the firm. PolyAna could drastically increase the proportion of debt in the firm’s capital structure. This would decrease the amount of ―free‖ cash that PolyAna’s management could spend on their weekly outings. If enough debt is placed on this firm, then a cash shortage, or lack of a large cash surplus, would necessitate that the managers of the firm work on new positive NPV projects rather than spend their Fridays in Cancun. LO 2 Bloomcode: Comprehension AASCP: Analytic IMA: Corporate Finance AICPA: Industry/Sector Perspective
CFA PROBLEMS 16.32 Consider two companies that operate in the same line of business and have the same degree of operating leverage: the Basic Company and the Grundlegend Company. The Basic Company has no debt in its capital structure, but the Grundlegend Company has a capital structure that consists of 50 percent debt. Which of the following statements is true? a. The Grundlegend Company has a degree of total leverage that exceeds that of the Basic Company by 50 percent. b. The Grundlegend Company has the same sensitivity of net earnings to changes in earnings before interest and taxes as the Basic Company. c. The Grundlegend Company has the same sensitivity of earnings before interest and taxes to changes in sales as the Basic Company. d. The Grundlegend Company has the same sensitivity of net earnings to changes in sales as the Basic Company. Solution: c is correct. The degree of total leverage of the Grundlegend Company exceeds that of the Basic Company, but the extent of the difference depends on the amount of interest expense, not the amount of debt. In the case of financial leverage, it is the interest that acts as a fulcrum. LO 2 Bloomcode: Analysis AASCP: Analytic IMA: Corporate Finance SM 19-740
AICPA: Industry/Sector Perspective
16.33 According to the pecking order theory: a. New debt is preferable to new equity. b. New equity is preferable to internally generated funds. c. New debt is preferable to internally generated funds. d. New equity is always preferable to other sources of capital. Solution:
a is correct. According to pecking order theory, internally generated funds are preferable to both new equity and new debt. But new debt is preferable to new equity as it has lower cost. LO 3 Bloomcode: Comprehension AASCP: Analytic IMA: Corporate Finance AICPA: Industry/Sector Perspective
16.34 According to the trade-off theory: a. The amount of debt a company has is irrelevant. b. Debt should be used only as a last resort. c. Debt will not be used if a company’s tax rate is high. d. Companies have an optimal level of debt. Solution: d is correct. The static trade-off theory indicates that there is a trade-off between the tax shield from interest on debt and the costs of financial distress, leading to an optimal range of debt for a company. LO 3 Bloomcode: Comprehension AASCP: Analytic IMA: Corporate Finance AICPA: Industry/Sector Perspective
SM 19-741
Sample Test Problems 16.1
Central Grocers Inc. produces annual cash flows of $175,000, which are expected to continue indefinitely. The company is financed entirely with equity capital at an annual cost of 12 percent. Management is considering borrowing $400,000 at an annual interest rate of 6 percent to repurchase $400,000 of the company’s outstanding stock (You can assume that the debt will be outstanding into the indefinite future.). What is the total value of Central Grocers’ stock before the stock repurchase? Under Modigliani and Miller’s Proposition 1, what would be the value of the total claims on the company’s assets after the stock repurchase? What will be the rate of return on common stock required by investors after the repurchase? (LO 1) Solution: Using the perpetuity formula, we can compute the value of Central Grocers’ stock before the repurchase to be $175,000/0.12 = $1,458,333 if the cash flow is expected to be produced into the infinite future. Under M&M Proposition 1, the value of the firm will be the same before and after the repurchase. Therefore, since the value of the debt claim will be $400,000 at the time of the transaction, the value of the outstanding equity after the repurchase must be $1,458,333 – $400,000 = $1,058,333.
Since interest of $400,000 × 0.06 = $24,000 must be paid to the debt holders each year, the amount that will be available for the stockholders each year will equal $175,000 $24,000 = $151,000. This implies that the new cost of equity capital for the firm will be $1,058,333 = $151,000/kcs, or kcs = 0.1427, or 14.27%. Bloomcode: Application AASCP: Analytic IMA: Corporate Finance AICPA: Industry/Sector Perspective
16.2 The required rate of return on the assets of a firm is 12 percent, the firm has a debt-tocommon-stock ratio of 40 percent, and the cost of debt is 6 percent. If the firm has no preferred stock and the three conditions specified by M&M hold, what is the expected rate of return on the firm's common stock? (LO 1) Solution: Using the equation for M&M Proposition 2, Equation 16.3, we get: kcs = kAssets + (VDebt /Vcs) × (kAssets – kDebt) = 0.12 + [(0.4) × (0.12 – 0.06)] = 0.144, or 14.4% Bloomcode: Application AASCP: Analytic SM 19-742
IMA: Corporate Finance AICPA: Industry/Sector Perspective
16.3
Your boss at Box and Freight Company asks you how much additional debt the company would have to add through a capital restructuring in order to create $9 million in present value from the resulting interest tax shields. What would you tell him if the debt will have no maturity and if Box and Freight is subject to a 32 percent marginal tax rate? (LO 2) Solution: The present value of Box & Freight’s additional tax shield will be: tc × D = 0.32 × D = $9,000,000 Therefore:
D = $28,125,000 Bloomcode: Application AASCP: Analytic IMA: Corporate Finance AICPA: Industry/Sector Perspective
16.4
Southwest Airlines has substantial cash reserves and an investment-grade bond rating. How would the trade-off theory predict that managers of Southwest would raise capital and choose the company’s capital structure if they were planning an expansion into Mexico? What would the pecking order theory suggest? (LO 3) Solution:
The trade-off theory would predict that Southwest would raise additional funds by first issuing more debt. The additional debt would provide tax savings, and as long as the incremental tax savings exceeded any added financial distress or agency costs, Southwest’s managers would increase the debt in the company’s capital structure. The pecking order theory would suggest that internally generated funds, specifically, cash on hand, would be the cheapest source of capital, and therefore used first to fund the expansion. Only after exhausting the internal funds would Southwest managers seek to add more debt to the company’s capital structure. Bloomcode: Analysis AASCP: Analytic IMA: Corporate Finance AICPA: Industry/Sector Perspective SM 19-743
16.5
What control implications do a firm’s capital structure decisions have? (LO 4) Solution:
Issuing debt does not change the ownership and therefore control of the firm, unless it is so much debt as to cause financial distress and the stockholders lose control to creditors. Issuing more common stock could dilute existing shareholders’ ownership if they do not purchase a proportionate amount of the new shares, thereby reducing their control. Bloomcode: Comprehension AASCP: Analytic IMA: Corporate Finance AICPA: Industry/Sector Perspective
Chapter 17 Dividends, Stock Repurchases, and Payout Policy Before You Go On Questions and Answers Section 17.1 9.
How does a dividend affect the size of a stockholder’s investment in a firm?
A dividend reduces the size of the stockholder’s investment in a firm. After a dividend is issued, the shareholder owns the same percentage of the company, which is worth less than it did before the dividend. The shareholder can use the dividend as he or she sees fit (after paying taxes). 10.
List and define four types of dividends. a. Regular Cash Dividend — Paid out regularly, usually quarterly. This dividend is usually set at a level that the board expects to be sustainable in the long run. b. Extra Dividend — Usually paid at the same time as a regular dividend. Often used to pay out extra earnings that will not be maintainable over the long run and/or to ensure that a certain percentage of earnings are returned to shareholders as dividends.
SM 19-744
c. Special Dividend — One-time dividend usually resulting from a special event such as the sale of an asset or a large cash balance. For example, Microsoft’s special dividend of $3 per share at the beginning of the chapter. d. Liquidating Dividend — Dividend resulting from the sale of assets of a company being liquidated (dissolved). This is the value left over for shareholders after the claims of other investors, such as bondholders, have been paid. 3.
What are the key events and dates in the dividend payment process?
Board Vote — The firm’s board votes to issue a dividend, specifying the amount and key dates of the issue. Public Announcement Date — The firm releases the information regarding the dividend payment. Often the stock price will move on the dividend announcement date because investors use this dividend information as a signal regarding the future prospects of the firm. Ex-Dividend Date — This is the first date on which purchasing the stock will not result in receiving a dividend. Record Date — This is the date on which one must be a stockholder of record to receive a dividend. This date is set by the board and is used by the exchange to set the ex-dividend day. The difference between the ex-dividend date and the record date reflects the time needed to compile and update the records of stock ownership. Payable Date — This is the date the dividend will actually be paid.
Section 17.2 3.
What is a stock repurchase? A stock repurchase takes place when a company purchases some of its own stock from shareholders. Shareholders who choose to participate receive payment in exchange for their shares.
4.
How do stock repurchases differ from dividends? There are four major ways in which stock repurchases differ from dividends. First, the payments are not made on a pro-rata basis. Some stockholders choose to take the repurchase offer, whereas others choose to hold on to their shares. Second, stock repurchases reduce the number of outstanding shares. This can change the liquidity of the shares and ownership control of the firm. Third, stock repurchases are taxed differently, often resulting in a smaller tax burden for investors. Finally, accounting for stock repurchases and dividends is different. When a dividend is issued, the retained earnings account decreases on the liability and stockholder’s equity side of the balance sheet. For a SM 19-745
stock repurchase, the treasury stock account on the right side of the balance sheet becomes more negative. Both stock repurchases and dividends reduce the cash account on the asset side of the balance sheet. 3.
In what ways can a company repurchase its stock? Open-Market Repurchase — The company purchases shares at the exchange in the same manner as normal trades. This is convenient for small ongoing repurchases. Regulations meant to prevent price manipulation limit the amount that a company can purchase at the exchange in a day. These regulations make an open-market repurchase cumbersome for large share repurchases. Tender Offer Repurchases — The company makes an open offer to buy shares. There are two types of tender offers. In a fixed-price offer, the company offers a fixed price to investors who agree to sell their shares. In a Dutch auction offer, the company seeks bids for the number of shares investors would sell at a series of prices. The company may then choose the price in the series that will result in the desired number of shares being repurchased. Targeted Stock Repurchase — A company repurchases shares directly from one or more large stockholders. This is sometimes used to negotiate with a large stockholder who is attempting to gain control of the company.
Section 17.3 3.
What are the benefits and costs associated with dividends?
The benefits associated with dividends are as follows: 1. Dividends may attract investors who prefer to receive income directly from their investments. However, the tax costs of dividends may drive away other investors. 2. Dividends can function as a signal to investors that the company is performing well and has higher then expected cash flows. 3. Dividends can help align manager and stockholder incentives. By issuing dividends and raising capital through equity issues (rather than internal funds), managers are subject to more scrutiny. This increases the incentives for managers to perform well. 4. Dividends reduce equity claims on the company; this can help managers achieve the target capital structure suggested by the trade-off theory.
a.
Some costs associated with dividends include: Taxes. Dividends have historically taxed at a higher rate than other forms of income.
SM 19-746
b. c.
4.
Reinvestment costs. Investors who don’t intend to spend the cash must pay the transactions costs associated with reinvesting (brokerage fees, etc.). Increased cost of debt. By reducing the amount of equity through a dividend issue, the firm becomes more leveraged. If the increase is significant, this could increase the risk associated with the company and increase the cost of debt should the company desire to borrow.
How do stock prices react to dividend announcements? Stock prices generally react favorably to announcements of higher than expected dividends and negatively to announcements of lower than expected dividends. This is consistent with the theory that dividends act as signals that convey to investors positive or negative changes to the firm’s fundamentals.
3.
Why might stock repurchases be preferred to dividends? Stock repurchases might be preferred to dividends because they result in lower taxes for investors and allow management more flexibility in distributing value. If the company has excess cash flow that is uncertain to continue in the future, it may be beneficial for management to initiate a stock repurchase, which can be quietly halted without sending a negative signal to investors.
Section 17.4 1.
What is a stock dividend? A stock dividend is a distribution of shares of stock to existing stockholders in proportion to the fraction of shares they own.
2.
How does a stock dividend differ from a stock split? A stock dividend usually occurs on a regular basis, and the number of new shares is usually small compared to the number of existing shares. Stock splits are infrequent and involve a large number of new shares. For example, in a 2-for-1 split the number of newly created shares is equal to the number of existing shares, and the total number of shares is doubled.
3.
How does a stock dividend differ from other types of dividends?
SM 19-747
Unlike other dividends, a stock dividend does not distribute anything of value, as the stockholders’ claim on the assets of the firm is unchanged. A stockholder may own more shares, but each share is worth less. The effects must exactly offset each other.
Section 17.5 1.
How are dividend payouts affected by expected earnings? Managers tend to set dividend policies that are achievable with expected long-run sustainable earnings. Because of the negative effect of a dividend reduction, managers are hesitant to increase dividends if they are concerned that the increase may have to be undone in the future.
2.
What did the 2005 study conclude about how managers view stock repurchases? The study concluded that managers tend to view repurchases as the preferred option for returning extra cash left over after investment spending. The study also found that managers prefer the flexibility of share repurchase plans, repurchasing shares when they believe the companies’ stock is undervalued. Finally, managers believe that the decision between share repurchases and dividend payments has little effect on what type of investor is likely to own the company’s stock.
3.
List three practical considerations that managers should take into account when setting a dividend policy?
The level of earnings (cash flows from operations) in excess of a company’s investment requirements over the long run and how certain this level is. Whether the firm has sufficient financial reserves to maintain the dividend payout during periods when earnings are down or investment opportunities are up. Whether the firm has sufficient financial flexibility to maintain dividends if unforeseen circumstances wipe out its financial reserves when earnings are down. Whether the firm is able to quickly raise enough capital if necessary. The control (voting) implications if a firm chooses to finance dividends by selling equity.
Self-Study Problems 17.1 You would like to own a common stock that has a record date of Friday, September 6, 2013. What is the last date that you can purchase the stock and still receive the dividend? SM 19-748
Solution: The ex-dividend date is the first day that the stock will be trading without the rights to the dividend, and that occurs two days before the record date, or on September 4, 2013. Therefore, the last day that you can purchase the stock and still receive the dividend will be the day before the ex-dividend date, or Thursday, September 3, 2013.
17.2 You believe that the average investor is subject to a 20 percent tax rate on dividend payments. If a firm is going to pay a $0.30 dividend, by what amount would you expect the stock price to drop on the ex-dividend date? Solution: If the tax rate of the average investor is reflected in the stock price change, we would expect investors to receive 80 percent (1.0 − 0.20 = 0.80 or 80 percent) of the dividend after paying taxes. This would necessitate a $0.24 (0.80 × $0.30 = $.24 drop in the stock price of the firm on the ex-dividend date.
17.3 The Veil Acts Company just announced that instead of a regular dividend this quarter, it will be repurchasing shares using the same amount of cash that would have been paid in the suspended dividend. Should this be a positive or negative signal from the firm? Solution: Veiled Acts has replaced a committed cash flow with one that is stated but does not have to be acted on. Therefore, the firm’s actions should be greeted with suspicion, and the signal is not a positive one.
17.4 The Bernie Rubbel Company has just declared a three-for-one stock split. If you own 12,000 shares before the split, how many shares do you own after the split? What if it were aone-for-three reverse stock split? Solution: You will own three shares of Bernie Rubbel for every one share that you currently own. Therefore, you will own 3 × 12,000 = 36,000 shares of the company. In the case of the reverse split, you will own 1/3 × 12,000 = 4,000 shares of the company.
SM 19-749
1717.5 Two publicly traded companies in the same industry are similar in all respects except one. Where Publicks has issued debt in the public markets (bonds), Privicks has never borrowed from any public source. In fact, it always uses private bank debt for its borrowing. Which firm might be marginally more inclined to have a more aggressive regular dividend payout than the other? Explain. Solution: If all other things are the same about the two companies, then Publicks could be expected to have a more aggressive dividend payout. Since Publicks has issued debt in the past, while Privicks has not, Publicks is likely to have greater access to the capital markets than Privicks. Firms with greater access to capital markets can be more aggressive in their dividend payouts to the extent that they can raise capital more easily (cheaply) if necessary.
Discussion Questions 17.1
Suppose that you live in a country where it takes 10 days to settle a stock purchase. By how many days will the ex-dividend date precede the record date?
The ex-dividend date is the first day that a stock will trade without the right to a dividend. In the United States, where it takes three days for a stock purchase to settle, the exdividend date is two days before the record date. Therefore, if it takes ten days for a stock purchase to settle, then the ex-dividend date will be nine days before the record date. LO 1 Level: Basic Bloomcode: Comprehension AASCP: Analytic IMA: Corporate Finance AICPA: Industry/Sector Perspective
17.2
The price of a share of stock is $15.00 on Tuesday, November 11, 2014. The record date for a $0.50 dividend is Friday, November 14, 2014. If there are no taxes on dividends, what would you expect the price of a share to be on each day from November 11 through 14 if no other information relevant to the price of the shares becomes public? Since we are excluding all non- dividend related information, the price of the shares will remain constant until the ex-dividend date, when the price will drop by $0.50 per share. SM 19-750
November 11—$15.00 November 12—$15.00 November 13—$14.50 November 14—$14.50 LO 1 Level: Basic Bloomcode: Application AASCP: Analytic IMA: Corporate Finance AICPA: Industry/Sector Perspective
17.3
You find that you are the only investor in a particular stock who is subject to a 15 percent tax rate on dividends (all other investors are subject to a 5 percent tax rate on dividends). Is there greater value to you in holding the stock beyond the exdividend date or selling the stock and then repurchasing it on or after the exdividend date? Assume that the stock is currently selling for $10.00 per share and the dividend will be $0.25 per share.
All other investors (who we can assume will be dictating the market equilibrium price) expect to pay 5 percent tax rate on their dividend receipt. Therefore, they expect to keep $0.25 × (1 – 0.05) = $0.2375 of the dividend after taxes are paid. Then, you would expect the price of the shares to be $9.7625 ($10.00 – $0.2375) on the ex-dividend date. However, you believe the correct price (based on your tax situation) to be $10.00 – (0.25 × (1 – 0.15)) = $9.7875. Therefore, by holding on to the stock, you would find that the share price dropped by $0.025 more than what you expected. Therefore, if we ignore transactions costs, it would be beneficial for you to sell your shares before the exdividend date and then repurchase them on the ex-dividend date. This argument, of course, ignores any taxes that might have to be paid by selling the shares at $10.00. LO 1 Level: Intermediate Bloomcode: Analysis AASCP: Analytic IMA: Corporate Finance AICPA: Industry/Sector Perspective
17.4
Discuss why the dividend payment process is so much simpler for private companies than for public companies.
SM 19-751
Since private companies have greater access to their shareholders than their public counterparts, the process of paying a dividend is not complicated by having to constantly monitor who owns the companies’ shares at any given time. In fact, the shares are bought and sold less frequently in private companies. This makes the dividend process much easier for private companies than for the public ones. LO 1 Level: Basic Bloomcode: Comprehension AASCP: Analytic IMA: Corporate Finance AICPA: Industry/Sector Perspective
17.5
You are the CEO of a firm that appears to be the target of a hostile takeover attempt. Thibeaux Piques has been accumulating the shares of your stock and now holds a substantial percentage of the outstanding shares. You would like to purchase the shares that he owns. What method of stock repurchase will you opt for?
Since Mr. Piques is hostile to your firm, it will probably not help you to use an openmarket purchase. You could announce a tender offer, but unless he is willing to sell his shares for purely economic gain (and ignore the benefits of control of your firm), then a tender offer would probably not work very well either. You could negotiate directly with Mr. Piques for his shares and therefore isolate the shares that you need to purchase using a targeted share repurchase. However, this direct negotiation might not be advantageous to your remaining shareholders, and it may require a premium price to convince Mr. Piques to sell his shares. LO 2 Level: Basic Bloomcode: Analysis AASCP: Analytic IMA: Corporate Finance AICPA: Industry/Sector Perspective
17.6
You have accumulated stock in a firm that does not pay cash dividends. You have read that, according to Modigliani and Miller, you can create a ―homemade‖ dividend should you require cash. Discuss why this choice may not be very good for the value of your position. You could sell a portion of your shares to generate a cash dividend for yourself. However, if no other investors take that course of action, then you would be the only investor SM 19-752
liquidating a portion of your holdings. If 20 percent of the voting interest in this firm helps you maintain a certain level of control of the firm, then dropping below that threshold might be enough for your diluted voting interest to lose whatever control of the firm you had maintained. LO 3 Level: Basic Bloomcode: Comprehension AASCP: Analytic IMA: Corporate Finance AICPA: Industry/Sector Perspective
17.7
You have just read a press release in which a firm claims that it will be able to generate a higher level of cash flows for its investors going forward. Justify the choice of a dividend payout that could credibly convey that information to the market.
Either an extra or a special dividend is signaling a higher level of cash flow but without sustainability. The best way to convey to the market that this new level of cash flow is permanent is to increase (or commence) a regular cash dividend since a regular dividend comes with the expectation that it will not be reduced without unforeseen events occurring in the future. LO 5 Level: Basic Bloomcode: Analysis AASCP: Analytic IMA: Corporate Finance AICPA: Industry/Sector Perspective
17.8
Some people argue that a high tax rate on dividends creates incentives for managers to go about their business without credibly convincing investors that the firm is doing well, even when it is. Discuss why this may be true. A credible signal is usually interpreted as a signal that is costly. Paying a regular dividend is costly, and we know that it credibly signals that the firm expects to maintain the current level of cash flow. If dividends are taxed at a rate high enough to discourage firms from paying them (discouraged because a very small portion of the cash sent to investors reaches them after the effects of taxes), the government is discouraging a credible signal that the firm’s prospects are still good. Without that costly signal, managers may continue
SM 19-753
about their normal routine without finding other ways to convey a firm’s prospects to investors. LO 3 Level: Basic Bloomcode: Comprehension AASCP: Analytic IMA: Corporate Finance AICPA: Industry/Sector Perspective
17.9
Fled Flightstone Mining’s management does not like to pay cash dividends due to the volatility of the company’s cash flows. Fled management has found, however, that when it does not pay dividends, its stock price becomes too high for individual investors to afford round lots. What course of action could Fled take to get its stock price down without dissipating firm value for stockholders?
Fled Flightstone Mining can double, triple, quadruple, etc., the number of shares outstanding without taking any meaningful economic steps. That is, it can take a 2-for-1, 3-for-1, or 4-for-1 stock split. This would increase the number of shares outstanding and decrease the value of each share outstanding proportionately. Since each shareholder would continue owning his or her prior pro-rata share of the firm, shareholders would not see any of their value dissipated by the firm’s actions. LO 5 Level: Basic Bloomcode: Analysis AASCP: Analytic IMA: Corporate Finance AICPA: Industry/Sector Perspective
17.10 Lintner found that firms are reluctant to make dividend changes that might have to be reversed. Discuss the rationale for that behavior. The simplest reason is that the markets look to dividends for the information that the dividends convey as well as for their economic benefit. That is, an increase in the dividend rate tells the market that the cash flow that the firm produces, above and beyond that needed for projects, is anticipated to remain high. If the firm were to quickly reverse the increased dividend rate, then that might convey to the firm that the firm’s fortunes either were not that good to begin with or have changed drastically. Such a reversal could convey that the firm is riskier than what investors anticipated before the series of events
SM 19-754
described above. Therefore, firm management would naturally want to keep from having to quickly reverse its dividend decisions. LO 5 Level: Basic Bloomcode: Comprehension AASCP: Analytic IMA: Corporate Finance AICPA: Industry/Sector Perspective
Questions and Problems BASIC 17.1
Dividends: The Poseidon Shipping Company has paid a $0.25 dividend per quarter for the past three years. Poseidon just lowered its declared dividend to $0.20 for the next dividend payment. Discuss what this new information might convey concerning Poseidon management’s belief about the future of the company.
Solution: Since dividends convey information concerning the future prospects of the firm, any change in dividend levels is also believed to convey a change in management’s forecast of the firm’s prospects. That is, lowering the dividend from $0.25 to $0.20 suggests that the firm’s future cash flow may be reduced. This reduction could be because of a general reduced level of profitability, because the firm’s projects are winding down, or even because of an increased need to invest in new positive NPV projects for the future. LO 1 Bloomcode: Application AASCP: Analytic IMA: Corporate Finance AICPA: Industry/Sector Perspective
17.2
Dividends: Marx Political Consultants has decided to discontinue all of its business operations. The firm has total debt of $7 million, and the liquidation value of its assets is $10 million. If the book value of the firm’s equity is $5 million, then what will be the amount of the liquidating dividend when the firm liquidates all of its assets?
Solution:
SM 19-755
The liquidating value of the firm’s assets is $10 million while the firm owes $7 million. Therefore, $3 million will remain after complete liquidation for the shareholders. The firm will be able to pay a $3 million dividend to its shareholders. LO 1 Bloomcode: Application AASCP: Analytic IMA: Corporate Finance AICPA: Industry/Sector Perspective 17.3
Dividends: Place the following in the proper chronological order, and describe the purpose of each: ex-dividend date, record date, payment date, and declaration date.
Solution: 1. Declaration date: the day the dividend payment was announced. 2. Ex-dividend date: the first day you can buy shares and not receive the dividend. 3. Record date: the day shareholders of record receive the dividend when it is paid. 4. Payment date: the date when the dividend is actually paid. LO 1 Bloomcode: Knowledge AASCP: Analytic IMA: Corporate Finance AICPA: Industry/Sector Perspective 17.4 Dividends and firm value: Explain how the issuance of new securities by a firm can produce useful information about the issuing firm. How can this information make the shares of the firm more valuable, even if it only confirms existing information about the firm? Solution: When issuing new securities, the issuing firm must submit to a process that amounts to a special audit by outsiders such as investment bankers and other experts. This additional production of information increases the level of monitoring concerning the firm’s actual financial status. In a sense, it reduces the variability in the information that the firm may already have released. If the production of this information reduces the level of risk borne by investors, then the issue of new securities could actually increase the value of the securities issued by the firm and, in turn, the total value of the firm. LO 1 Bloomcode: Comprehension AASCP: Analytic IMA: Corporate Finance AICPA: Industry/Sector Perspective
SM 19-756
17.5
Dividends: Explain why holders of a firm’s debt should insist on a covenant that restricts the amount of cash dividends the firm pays.
Solution: We have to remember that any cash paid to shareholders reduces the amount that is available to bondholders in the event of bankruptcy. Because bondholders are aware of this potential problem, they should then restrict the amount of cash that can be paid to shareholders to at least a level where bondholders will still be able to generate their expected rate of return. LO 1 Bloomcode: Comprehension AASCP: Analytic IMA: Corporate Finance AICPA: Industry/Sector Perspective
17.6
Stock splits and stock dividends: Explain why managers of firms might prefer that their firms’ shares trade in a moderate per-share price range rather than in a high per-share price range. How do managers of firms keep their shares trading in a moderate price range?
Solution: Historically, the transactions and liquidity costs to trade 100 shares were lower than the cost to trade a smaller number of shares. Therefore, if small investors could not afford to trade 100 shares, then they might refrain from purchasing the shares. In that event, the high per-share price of the shares might eliminate potential investors for those shares. Consequently, firms preferred that their shares trade in an affordable range rather than at an expensive price per share. Note that no conclusive empirical evidence supports that notion. LO 4 Bloomcode: Comprehension AASCP: Analytic IMA: Corporate Finance AICPA: Industry/Sector Perspective
17.7
Dividends: Scintilla, Inc., stock is trading for $10.00 per share on the day before the exdividend date. If the dividend is $0.25 and there are no taxes, what should the price of the shares be on the ex-dividend date?
SM 19-757
Solution: Since there are no taxes, the value of the shares should drop by the amount of the dividend. Therefore, the shares should trade for $9.75 on the ex-dividend date. LO 1 Bloomcode: Application AASCP: Analytic IMA: Corporate Finance AICPA: Industry/Sector Perspective 17.8
Dividends: A company’s management announces a $1.00 per share dividend payment. Assuming all investors are subject to a 15 percent tax rate on dividends, how much should the company’s share price drop on the ex-dividend date?
Solution: Investors will be able to capture 85 percent of the amount of any dividend paid. Therefore, the price on the ex-dividend date should go down by $0.85 [$1.00 x (1 – 15%)] per share. LO 1 Bloomcode: Comprehension AASCP: Analytic IMA: Corporate Finance AICPA: Industry/Sector Perspective
INTERMEDIATE 17.9
Dividends and firm value: Explain how a stock repurchase is different from a dividend payment.
Solution: A share repurchase, if followed through by management, will place the same amount of cash in the hands of its shareholders. However, since shareholders have the option of selling their shares or holding on to their shares, the repurchase leaves it up to the individual shareholders whether or not they would like to receive the cash. The dividend payment method will effectively force the shareholders to receive the cash. LO 2 Bloomcode: Analysis AASCP: Analytic IMA: Corporate Finance AICPA: Industry/Sector Perspective
SM 19-758
17.10 Dividends and firm value: You have just encountered two identical firms with identical investment opportunities, as well as the ability to fund these opportunities. One of the firms has just announced that it will pay a dividend, while the other has continued to pay no dividend. Which of the two firms is worth more? Explain. Solution: If we begin with a world described by the Modigliani and Miller paper in 1961, which assumes that (1) investors incur no taxes, (2) there are no information or transactions costs, and (3) the dividend payout rates have no effect on the firm’s real investment policy, then both firms will be worth exactly the same. That is because investors who want dividends but own a no-dividend stock can liquidate their appreciated value shares to create a homemade dividend, and shareholders who do not want dividends but own a dividend-paying stock can use their ―unwanted‖ dividends to purchase additional shares of that stock. LO 3 Bloomcode: Analysis AASCP: Analytic IMA: Corporate Finance AICPA: Industry/Sector Perspective
17.11 Dividends and firm value: Explain what the introduction of transaction costs does to the Modigliani and Miller assumption that dividends are irrelevant. Start with a firm that pays dividends to investors that do not want to receive dividend payments. Do not consider taxes. Solution: The no-transaction costs assumption is required for investors to create their own homemade dividends or for investors to ―undo‖ their unwanted dividend payments by purchasing additional shares of the company stock. Therefore, by relaxing the no transaction cost assumption, we would increase the cost of producing the homemade dividend or the cost of undoing the unwanted dividends, which would then make dividend policy a relevant factor when valuing shares. For instance, receiving an unwanted dividend would now make it more costly to convert that dividend into new shares, in as much as part of that dividend would be dissipated to transaction costs. In that situation, the investor would value a non-dividendpaying share at a higher value than a dividend-paying share. LO 3 Bloomcode: Comprehension AASCP: Analytic SM 19-759
IMA: Corporate Finance AICPA: Industry/Sector Perspective 17.12 Dividends and firm value: CashCo increased its cash dividend each quarter for the past eight quarters. While this may signal that the firm is financially very healthy, what else could we conclude from these actions? Solution: If we rule out the possibility that the firm is just producing a high level of cash, then we must conclude that the firm has more cash than it has investment opportunities to utilize that cash. In short, we might conclude that the firm’s growth rate will be slowing down in the future. LO 3 Bloomcode: Analysis AASCP: Analytic IMA: Corporate Finance AICPA: Industry/Sector Perspective
17.13 Dividends and firm value: At the end of 2012 the maximum tax rate on dividends increased from 15 percent to 23.8 percent. How would you expect this increase to affect the prices of dividend-paying stocks versus those of nondividend-paying stocks? Solution: The value of a dividend received, after taxes are included, will be less than before. Therefore, we would expect the value of dividend-paying stocks to decrease relative to non-dividendpaying stocks. LO 2 Bloomcode: Analysis AASCP: Analytic IMA: Corporate Finance AICPA: Industry/Sector Perspective 17.14 Dividends: Undecided Corp. has excess cash on hand right now, although management is not sure about the level of cash flows going forward. If management would like to put cash in stockholders’ hands, what kind of dividend should the firm pay, and why? Solution: Since this is a one-time increase in cash flow, the firm would not want to commit to an ongoing higher dividend rate. If the firm went that route but then later had to reduce the dividend back to current dividend levels, then the market could interpret that action as SM 19-760
indecision on the part of management, or worse. Therefore, an increase in the regular dividend would not be appropriate. A more appropriate choice would be to declare an extra dividend. Note that it would be called a special dividend if the extra cash flow came from something other than the firm’s regular operations or if the cash were an unusually large amount. LO 3 Bloomcode: Application AASCP: Analytic IMA: Corporate Finance AICPA: Industry/Sector Perspective 17.15 Dividends and firm value: A firm can deliver a negative signal to stockholders by increasing the level of dividends or by reducing the level of dividends. Explain why this is true. Solution: When a firm reduces its dividend, the firm is telling the market that it does not have sufficient cash, which is of course a bad or negative signal. However, by increasing the dividend, the firm is telling its investors that it has greater cash than it has investment uses for that cash. If the firm is currently viewed as a growth firm, then the market could interpret an increase in the dividend as a slowdown in the growth rate of the firm precipitated by the firm’s lower investment rate. LO 3 Bloomcode: Comprehension AASCP: Analytic IMA: Corporate Finance AICPA: Industry/Sector Perspective 17.16 Dividends and firm value: A commentator on a financial talk show on TV says that ―On average, firms pay out too little to stockholders. This is why stock prices go up with dividend increases and down with dividend decreases.‖ Is the commentator right? Solution: No. It is true that stock prices do tend to increase with dividend increases and to decrease with dividend decreases. However, this does not necessarily mean that firms are paying out too little to stockholders on average. The price of a company’s stock might increase when a firm increases its dividend because the increase signals to investors that the company’s management is optimistic about future cash flows or is willing to distribute excess cash rather than keep it and invest in negative NPV projects. The stock price might decrease when a firm reduces its dividend because investors might be concerned that a firm’s cash flows will decrease in the future or that management will invest excess cash that is not being distributed in negative NPV projects. These SM 19-761
arguments do not require that the firm be paying out too little at that time that the dividend is increased or decreased. The stock price reaction to the announcement of a dividend change reflects a change in investor expectations about what the announcement tells them about the cash flows they can expect to receive in the future. As the cash flow identity suggests, there are a number of reasons besides those above that might cause stock prices to increase with dividend increases and to decrease with dividend decreases. LO 3 Bloomcode: Analysis AASCP: Analytic IMA: Corporate Finance AICPA: Industry/Sector Perspective
17.17 Dividends and firm value: You own shares in a firm that has extra cash on hand to distribute to stockholders. You do not want the cash. What course of action would you prefer the firm take? Solution: You would prefer that the firm initiate a share repurchase. You can opt not so sell your shares to the firm but still participate in the increased value of the firm’s shares since your pro-rata share of the expected future cash flows generated by the firm will increase. You would not like a dividend payment since you would then be required to receive the cash if you owned the shares at the time of the record date. LO 4 Bloomcode: Application AASCP: Analytic IMA: Corporate Finance AICPA: Industry/Sector Perspective
17.18 Dividends and firm value: Stock repurchases, once announced, do not actually have to occur in total or in part. From a signaling perspective, why would a special dividend be better than a stock repurchase? Solution: If we ignore the preferences of individual shareholders, then a special dividend is preferred to a share repurchase. Although dividends are binding, once declared by the firm’s board of directors, a share repurchase is not binding. Therefore, a special dividend is considered a stronger signal than a share repurchase. LO 4 Bloomcode: Comprehension SM 19-762
AASCP: Analytic IMA: Corporate Finance AICPA: Industry/Sector Perspective
17.19 Dividends and firm value: Consider a firm that repurchases shares from its stockholders in the open market, and explain why this action might be detrimental to the stockholders from whom the firm buys shares. Solution: To understand this argument, we have to consider two points. First, the firm should be managed for the benefit of its shareholders. Second, the firm is the ultimate insider concerning the value of its shares. Given the above points, we must realize that any time the firm is purchasing its shares; it must be doing so because the firm’s management believes that the firm’s shares are undervalued. Therefore, by purchasing its shares, the firm is utilizing its inside information to purchase shares and ultimately to take advantage of the current owner of those shares in order to benefit the remaining shareholders of the firm. It is then not doing something in the interest of all of its shareholders since those who sell will be selling at a price lower than what they could have realized had they held their shares until the repurchase was complete. LO 3 Bloomcode: Comprehension AASCP: Analytic IMA: Corporate Finance AICPA: Industry/Sector Perspective
17.20 Dividends and firm value: You read that a number of public companies have been financing their dividend payments in recent years entirely through equity issues. A colleague of yours argues that this only increase taxes paid by individual stockholders and boosts underwriting and other transactions costs for the company. He says that such a policy cannot make sense. What do you say? Solution: Such a policy can make sense. While there are costs associated with doing this, financing dividends with equity issues can help control agency conflicts by forcing managers to periodically submit to the scrutiny associated with the equity issuance process. It can ultimately lead to better company performance and the willingness of investors to pay a higher price for the company’s stock. LO 3 SM 19-763
Bloomcode: Analysis AASCP: Analytic IMA: Corporate Finance AICPA: Industry/Sector Perspective
17.21 Stock repurchases: Briefly discuss the methods available for a firm to repurchase its shares and explain why you might expect the stock price reaction to the announcement of each of these methods to differ. Solution: 1. Open-market purchase — the firm simply purchases the shares in the market. 2. Tender offer — the firm makes an offer through a general announcement, offering to buy up to a certain number of shares from anyone who wishes to sell. Note that either the fixed-price or Dutch auction method can be used for a tender offer. 3. Targeted share repurchases — the firm directly negotiates with an individual shareholder to buy shares from that individual. LO 2 Bloomcode: Knowledge AASCP: Analytic IMA: Corporate Finance AICPA: Industry/Sector Perspective
17.22 Stock repurchases: What is the advantage of a Dutch auction over a fixed-price tender offer? Solution: If the firm that is trying to purchase its own shares believes that there may be some variability in the supply curve for those shares, then a Dutch auction would essentially provide the firm with the necessary information to make the tender offer successful. In those same circumstances, a fixed-price tender offer might leave the firm paying too much for the shares or unable to purchase the desired number of shares given the fixedprice offer. LO 2 Bloomcode: Analysis AASCP: Analytic IMA: Corporate Finance AICPA: Industry/Sector Perspective
SM 19-764
ADVANCED 17.23 In the early 1990s, the amount of time that elapsed between purchasing a stock and actually obtaining that stock was five business days. This period was known as the settlement period. The settlement period for stock purchases is now two business days. Describe what should have happened to the number of days between the ex-dividend date and the record date at the time of this change. Solution: The purpose of setting the number of days between the ex-dividend date and the record date is to allow for a sale of securities to clear in order to determine which ―owner‖ is entitled to the dividend. Since the settlement period was reduced from five days to two days, we should have seen the number of days between the ex-dividend date and the record date to be reduced from four days to two. LO 1 Bloomcode: Analysis AASCP: Analytic IMA: Corporate Finance AICPA: Industry/Sector Perspective
17.24 Dividend reinvestment programs (DRIPs) sometimes sell shares at a discount to stockholders who reinvest their dividends through such plans. Your boss tells you that such plans are just a scheme to transfer wealth from nonparticipating to participating stockholders and that they should be stopped. Do you agree? Why or why not? Solution: Disagree. It is true that the cost of a DRIPs (including any discount) is borne by all stockholders. However, it is not necessarily true that DRIPs are bad for non-participating stockholders. DRIPs help firms attract investors who might not otherwise invest in a dividend-paying stock. Being able to attract these investors can help improve the liquidity of the shares and benefit all stockholders. Even without this benefit, it can make sense to sell shares at a discount. If DRIPs reduces the amount of capital that the firm has to raise in the future, it also reduces the issuance costs that the firm incurs in raising that capital. These cost savings can more than offset the cost of a discount offered to participating stockholders. LO 3 SM 19-765
Bloomcode: Analysis AASCP: Analytic IMA: Corporate Finance AICPA: Industry/Sector Perspective
17.25 WeAreProfits, Inc., has not issued any new debt securities in 10 years. It will begin paying cash dividends to its stockholders for the first time next year. Explain how a dividend might help the firm get closer to its optimal capital structure of 50 percent debt and 50 percent equity. Solution: If the firm has been successful over the last 10 years, then the value of its equity has increased but the total value of its debt has not increased accordingly. Therefore, the debtto-equity ratio of the firm has been dropping. By paying a dividend, the value of the equity, after the dividend is paid, will drop relative to that of the debt. This will help the firm to balance out its debt-equity ratio and get closer to a 50 percent-50 percent mix (assume that the debt was below the 50 percent level to begin with). LO 3 Bloomcode: Application AASCP: Analytic IMA: Corporate Finance AICPA: Industry/Sector Perspective
17.26 Shadows, Inc., had shares outstanding that were valued at $120 per share before a twofor-one stock split. After the stock split, the shares were valued at $62 per share. If we accept that the firm’s financial maneuver did not create any new value, then why might the market be increasing the total value of the firm’s equity? Solution: We know that the stock split did not create value for investors by itself. Therefore, there must be information in the split that accounts for the $4 increase in value to shareholders. Generally speaking, firms have a tendency to increase their dividend rate after a stock split. Therefore, since a stock split is generally followed by dividend rate increases, the increase tends to generate information for investors that there is a potential increase in cash flow to investors of the firm’s equity. LO 4 Bloomcode: Analysis AASCP: Analytic IMA: Corporate Finance AICPA: Industry/Sector Perspective SM 19-766
17.27 Saguaro Company currently has 30,000 shares outstanding. Each share has a market value of $20. If the firm pays $5 per share in dividends, what will each share be worth after the dividend payment? Ignore taxes. Solution: The current value of all of the shares is 30,000 × $20 = $600,000. If the firm pays $5 per share, then the total cash paid out will be 30,000 × $5 = $150,000. Therefore, the value of all of the shares will be worth $150,000 less or $450,000, and the price of each share will be $450,000 / 30,000 = $15 per share. However, it seems logical that if shares were worth $20 before the dividend payout, then they should be worth $5 less after the dividend payout or $15 just as we have calculated. LO 1 Bloomcode: Application AASCP: Analytic IMA: Corporate Finance AICPA: Industry/Sector Perspective
17.28 Cholla Company currently has 30,000 shares outstanding. Each share has a market value of $20. If the firm repurchases $150,000 worth of shares, then what will be the value of each share outstanding after the repurchase? Ignore taxes. Solution: The current value of all of the shares is 30,000 × $20 = $600,000. If the firm repurchases $150,000 worth of shares, then it will repurchase $150,000 / $20 = 7,500 shares. Therefore, the value of all of the shares not purchased will be worth $150,000 less, or $450,000, and the price of each share will be $450,000 / (30,000 – 7,500) = $20 per share. It seems logical, however, that if shares were worth $20 before the repurchase, then they should be worth $20 after the repurchase since investors should be indifferent between selling their shares to the firm and retaining shares. LO 3 Bloomcode: Application AASCP: Analytic IMA: Corporate Finance AICPA: Industry/Sector Perspective
17.29 You purchased 1,000 shares of Koogal stock five years ago for $30 per share. Today Koogal is repurchasing your shares through a fixed-price tender offer for $80 per share. SM 19-767
What are the after-tax proceeds to you if only your capital gain is taxed at a 15 percent rate? Solution: The tax on the stock would be 1,000 × ($80 – $30) × 0.15 = $7,500. Therefore, the aftertax proceeds would equal (1,000 × $80) – $7,500 = $72,500. LO 1 Bloomcode: Analysis AASCP: Analytic IMA: Corporate Finance AICPA: Industry/Sector Perspective
17.30 You purchased 1,000 shares of Zebulon Copper Co. five years ago for $50 per share. Today Zebulon management is trying to decide whether to repurchase shares for $70 per share through a fixed-price tender offer or pay a $70 cash dividend per share. If capital gains are taxed at a 15 percent rate, then at what rate must dividends be taxed for you to be indifferent between receiving the dividend and selling your shares back to Zebulon? Solution: The tax on the stock would be 1,000 × ($70 – $50) × 0.15 = $3,000. Therefore, the tax on the dividend must be equal to $3,000 for you to be indifferent between the two choices. Therefore, $3,000 = 1,000 × $70 × Dividend Tax rate Dividend Tax rate = 4.29% If dividends are taxed at any rate higher than 4.29 percent, then you would prefer the share repurchase. Note that the point of indifference is partially a function of the ratio of capital gains to the cash dividend. LO 2 Bloomcode: Analysis AASCP: Analytic IMA: Corporate Finance AICPA: Industry/Sector Perspective
17.31 Llama Wool Company management is doing some financial planning for the coming year. Llama plans to raise $10,000 in new equity this year and wants to pay a dividend to stockholders of $30,000. The firm must pay $20,000 of interest during the year and will also pay down principal on its debt obligations by $10,000.Its capital budgeting plan calls for $100,000 of capital expenditures during the year. Given the above information, how much cash must be provided from operations for the firm to meet its plan? Solution: Using the equation given in the text: SM 19-768
CFOAt + Equityt + Debtt = Divt + Repurchasest + Interestt + Principalt + Invt CFOAt + $10,000 = $30,000 + $20,000 + $10,000 + $100,000 CFOAt = $150,000 LO 3 Bloomcode: Application AASCP: Analytic IMA: Corporate Finance AICPA: Industry/Sector Perspective
17.32 You are the Chief Financial Officer (CFO) of a large publicly traded company. You would like to convey positive information about the firm to the market. If you agree with the conclusions from the Lintner study, will you keep paying your currently high dividend or raise that dividend by a small amount? Explain. Solution: Although the current high level of dividends certainly suggests that the firm has the ability to sustain a high payout to investors, that high payout does not convey any positive information to the market. However, by increasing the dividend, the CFO would be telling the market that the firm will be able to support an even higher level of cash payout in the future. Therefore, it would be better to increase the dividend by a small amount. LO 3 Bloomcode: Analysis AASCP: Analytic IMA: Corporate Finance AICPA: Industry/Sector Perspective
17.33 You are the CFO of a public company that advises distressed companies about how to manage their businesses. Your company has been performing extremely well. In fact, it has earned so much money that the increase in its retained earnings has resulted in a decline in the firm’s debt to total capital ratio from 30 percent to 15 percent. Much of the retained earnings is sitting in a cash account because your firm does not need the money to fund investments. You would like to increase the debt-to-total capital ratio to 30 percent, which you view as optimal for your firm. How would you recommend doing this if you want to complete the adjustment as soon as possible? Solution: You can do this by paying out some of the cash that the firm has accumulated using either a special dividend or a stock repurchase. A special dividend can be completed quickly, but would SM 19-769
subject your stockholders to taxes if they are not tax-exempt organizations or stockholders who wanted to retain the same dollar investment in your firm. LO 3 Bloomcode: Analysis AASCP: Analytic IMA: Corporate Finance AICPA: Industry/Sector Perspective
Sample Test Problems 17.1
Shares of Convoy West, Inc. are trading for $55.45 on the day before the ex-dividend date. If the quarterly dividend is $0.16 per share and there are no taxes, how will the share price change on the ex-dividend date?
Solution: Since there are no taxes, the value of the shares drop by the $0.16, the amount of the dividend. Therefore, the shares should trade for $55.29 ($55.45 – $0.16 = $55.29) on the ex-dividend date if no other information relevant to the price of the stock becomes public. Bloomcode: Analysis AASCP: Analytic IMA: Corporate Finance AICPA: Industry/Sector Perspective
17.2
Three years ago, you purchased 4,000 shares of Metwa Inc. for $17 per share. Today Metwa is repurchasing its shares through a fixed-price tender offer at a price of $45 per share. What are the after-tax proceeds that you will receive if capital gains are taxed at a rate of 20 percent?
Solution: The tax on the stock would be 4,000 × ($45 – $17) × 0.20 = $22,400. Your after-tax proceeds would equal (4,000 × $45) – $22,400 = $157,600. Bloomcode: Application AASCP: Analytic IMA: Corporate Finance AICPA: Industry/Sector Perspective
17.3
Why does an ongoing stock repurchase program offer management greater flexibility in distributing value to stockholders than a regular cash dividend?
Solution: SM 19-770
An ongoing stock repurchase program, such as that announced by DuPont in the chapter opener, is less perceived in the stock market as being less of a commitment than a regular cash dividend. The actual amount of cash being distributed through such a program is also less visible than the amount of money being distributed through dividends. If management decides to decrease the amount of cash being distributed to stockholders through a repurchase program, it can cut back or even terminate the program without causing the negative stock price reaction that typically accompanies reductions in regular cash dividends. Bloomcode: Analysis AASCP: Analytic IMA: Corporate Finance AICPA: Industry/Sector Perspective
17.4
Why would management of a company undertake a reverse stock split?
Solution: Often, a reverse stock split is undertaken to satisfy exchange requirements. A reverse stock split can be used to increase the share price to a level above the required threshold at an exchange such as the NYSE or NASDAQ. Bloomcode: Comprehension AASCP: Analytic IMA: Corporate Finance AICPA: Industry/Sector Perspective
17.5
Mastercard, Inc. completed a 10-for-1 stock split on January 22, 2014. Immediately before the stock split there were 120.38 million shares outstanding at a price of $826.00 per share. After the split how many shares were outstanding, and at what price would you expect them trade? Did the stock split cause any substantial change for Mastercard or the investors? Why would Mastercard management choose to split the company’s stock?
Solution: Bloomcode: Analysis AASCP: Analytic IMA: Corporate Finance AICPA: Industry/Sector Perspective
CHAPTER 18 Business Formation, Growth, and Valuation SM 19-771
Before You Go On
Section 18.1 11. What are three general reasons that new businesses fail? Businesses fail for a variety of reasons. In some cases, a firm’s product(s) may find no demand among consumers. For others, the founding management may lack a clearly thought out strategy or lack the skills to move the firm forward. In addition, a lot of firms fail because they run out of money to operate the business and are unable to raise more.
12. How do financing considerations affect the choice of organizational form?
Of all the different forms of organizations, the sole proprietor takes the least amount of money to set up and is only dependent on the founder’s money. Forming a partnership is more involved and costs more. Setting up as a corporation is the most expensive and calls for a more active role by attorneys to set up the organization.
3.
How does a cash budget help an entrepreneur?
The cash budget is a very useful planning tool for entrepreneurs. It summarizes the cash flows into and out of a firm over a period of time. Cash budgets often present the inflows and outflows on a monthly basis but can be prepared for any period, including daily or weekly. Preparing a cash budget helps an entrepreneur better understand where money is SM 19-772
coming from, where it is going, and how much external financing is likely to be needed and when. Understanding where the money is coming from and where it is going helps an entrepreneur maintain control of the company’s finances. Knowing how much external financing is likely to be needed and when helps the entrepreneur plan fund-raising efforts before it is too late.
Section 18.2 5. Why is a business plan important in raising capital for a young company?
A well-prepared business plan makes it easier for an entrepreneur to communicate to potential investors precisely what he or she expects the business to look like in the future, how he or she expects to get it to that point, and what returns an investor might expect to receive. This allows potential investors to get a feel for the strategic plan of the firm and also allows them to gauge the management ability of the entrepreneur.
2.
What else can a business plan be used for?
A business plan is a tool that
can help raise capital
can help an entrepreneur set the goals and objectives for the company
serve as a benchmark for evaluating and controlling the company’s performance,
SM 19-773
communicate the entrepreneur’s ideas to managers, outside directors, customers, suppliers, and others.
3.
Why is it important to discuss the qualifications of the management team in a
business plan?
The discussion of the qualification of the management team is especially important when it comes to raising capital. Investors in young businesses invest in the key people as much as in the business idea itself.
Section 18.3 5. Why is it important to specify a valuation date when you value a business?
The value of a business changes over time. This can be caused by changes in general economic conditions, industry conditions, and decisions that are made by the managers— all affect the value of the cash flows that a business is expected to generate in the future. Actions by competitors also affect the value of a business. The investment, operating, and financing decisions made by managers also affect the value of a business. Given this, it is important to specify a valuation date while valuing a business. This is the date as of which the value estimate applies.
6. What is the difference between investment value and fair market value?
SM 19-774
A strategic investor is one who has an interest in acquiring the business. The investment value of the firm to a strategic investor will take into consideration the benefits that can accrue from the acquisition and hence is likely to carry a higher value. In contrast, the fair market value is the valuation arrived by an investor who is only interested in the financial performance of the firm and not in acquiring the business. The fair market value can differ considerably from the investment value of the business.
3.
What are the two market approaches that can be used to value a business,
and how do they differ?
The two market approaches that are commonly used to value a business are multiples analysis and transactions analysis. Multiples analysis uses stock price or other value multiples that are observed for public companies to estimate the value of a company’s stock or an entire business. In transaction analysis, analysts use the information from transactions involving a merger or an acquisition of similar companies to estimate the value of a company or its stock.
4.
What is a nonoperating asset, and how are such assets accounted for in
business valuation?
Nonoperating assets (NOA) are assets like cash or other assets that are not essential to support the operations of a business. As Equation 18.3 shows, when valuing a business, the
SM 19-775
net operating assets are included in the cash flows that are discounted to arrive at the present value of a firm.
5.
What are three income approaches used to value a business?
The three income approaches to valuing a business are free cash flow from the firm (FCFF), free cash flow to equity (FCFE), and dividend discount model (DDM). In the free cash flow from the firm (FCFF) approach, an analyst values the discounted value of free cash flows that the assets of the firm are expected to produce in the future to estimate the value of the enterprise. The free cash flow to equity (FCFE) approach uses the discounted value of the portion of the cash flows that are available for distribution to stockholders to estimate the value of the enterprise. The dividend discount model (DDM) approach estimates the value of an enterprise using the discounted value of the dividends to be distributed to stockholders in the future.
6.
What is the difference between FCFE and dividends?
The FCFE is an estimate of all cash flows available for distribution to shareholders of the firm. Recognize that not all of this will be distributed to stockholders as dividends. Thus, FCFE includes both dividends and retained earnings, the portion not distributed to shareholders but reinvested in the firm. Section 18.4 3. How might financial statements for private companies differ from those for public companies? SM 19-776
Some private companies have complete, audited financial statements, while others have incomplete financial statements that are not prepared in accordance with the GAAP. All public companies are required to file audited financial statements with the Securities and Exchange Commission (SEC).
In contrast to the financial statements of publicly held company, private company financials often include personal expenses of the owner and excess compensation expenses.
4. Why is marketability an important issue in business valuation?
The shareholders of a publicly held company have an easier time of liquidating their holdings if they so desire. Since a market and the value for their holdings are already established, a simple call to a broker will take care of it. In contrast, the shareholders of a privately held firm will have to spend a lot more time and money to liquidate their shares. This additional time and effort is a reflection of a lower level of marketability of such firms and will result in a discount in the value of the firm.
3.
What is a key person?
If the cash flows that a business is expected to generate depend heavily on the retention of a particular individual or group of individuals, then that individual or group can be
SM 19-777
referred to as key people. The survival of the firm is dependent on the continued employment of this individual or group.
Self-Study Problems 18.1
Your sister wants to open a store that sells antique-style jewelry and accessories. She has $15,000 of savings to invest, but opening the store will require an initial investment of $20,000. Net cash inflows will be −$2,000, −$1,000, and $0 in the first three months. As the store becomes better known, net cash inflows will become +$500 in the fourth month and grow at a constant rate of 5 percent in the following months. You want to help your sister by providing the additional money that she needs. How much money do you have to invest each month to start and to keep the store operating with a minimum cash balance of $1,000?
Solution: You will have to invest in $5,000 initially for her business to start (the difference between $20,000 and $15,000). You will then need to invest an additional $3,000 during the first month to cover the cash flow of -$2,000 and to establish a cash balance of $1,000. Another $1,000 will be required in the second month to cover the negative cash flow during that month. Since cash flows will be positive beginning in the third month, you will not have to invest any additional funds after the second month.
18.2
You have the following information for a company you are valuing and for a comparable company:
SM 19-778
Comparable company:
Company you are valuing:
Stock price = $23.45
Value of debt = $3.68 million
Number of shares outstanding = 6.23 million
Est. EBITDA next year = $4.4 million
Value of debt = $18.45 million
Est. income next year = $1.5 million
Est. EBITDA next year = $17.0 million Est. income next year = $5.3 million Estimate the enterprise value of the company you are evaluating using the P/E and enterprise value/EBITDA multiples.
Solution: The P/E and enterprise value/EBITDA multiples for the comparable company are:
P Stock price E Comparable Earnings per share Comparable
$23.45 per share $5.3 million 6.23 million shares
27.6 Enterprise value EBITDA
VD VE EBITDA Comparable Comparable
$18.45 million ($23.45 per share 6.23 million shares) $17.0 million
9.68 Using the P/E multiple, we can calculate the value of the equity as:
SM 19-779
V E
P Net income company being valued E Comparable
27.6 $1.5 million $41.4 million which suggests an enterprise value of: VF VE VD $41.4 million $3.68 million $45.08 million Using the enterprise/EBITDA multiple, we obtain: Enterprise value EBITDA VF EBITDA Company being valued Comparable
9.68 $4.4 million $42.59 million
18.3
How do the cash flows that are discounted when the WACC approach (FCFF approach) is used to value a business differ from those that are discounted when the free cash flow to equity (FCFE) approach is used to value the equity in a business?
Solution: The cash flows that are discounted when the WACC approach is used to value a business are calculated in the same way that the cash flows are calculated for a project analysis. These cash flows represent the total cash flows that the business is expected to generate from operations. The cash flows that are discounted when the FCFE approach is used are the portion of the total cash flows from the business that are available for distribution to the stockholders. In other words, they equal the total cash flows that the business is expected to generate less the net cash flows to the debt holders. The net cash flows to the SM 19-780
debt holders is equal to the interest and principal payments that the firm makes less any proceeds for the sale of new debt. 18.4
You are valuing a company using the WACC approach and have estimated that the free cash flows from the firm (FCFF) in the next five years will be $36.7, $42.6, $45.1, $46.3, and $46.6 million, respectively. Beginning in year 6, you expect the cash flows to decrease at a rate of 3 percent per year for the indefinite future. You estimate that the appropriate WACC to use in discounting these cash flows is 10 percent. What is the value of this company?
Solution: The present value of the cash flows expected over the next five years is:
PV(FCFF5) =
$36.7 million $42.6 million $45.1 million $46.3 million $46.6 million + + + + 2 3 4 5 1+ 0.1 1+ 0.1 1+ 0.1 1+ 0.1 1+ 0.1
= $163.01million.
The terminal value is:
TV5 =
FCFF5 ×1+ g $46.6 million ×1- 0.03 = = $347.71million WACC- g 0.1+ 0.03
and the present value of the terminal value is:
PV(TV5 ) =
$347.71 TV5 = 5 = $215.90 million. 5 1+ WACC 1+ 0.1
Therefore, if there are no nonoperating assets, the value of the firm is: VF = $163.01 million + $215.90 million = $378.91 million SM 19-781
18.5
You want to estimate the value of a local advertising firm. The earnings of the
firm are expected to be $2 million next year. Based on expected earnings next year, the average price-to-earnings ratio of similar firms in the same industry is 48. Therefore, you estimate the value of the firm you are valuing to be $96 million. Further investigation shows that a large portion of the firm’s business is obtained through connections that John Smith, a senior partner of the firm, has with various advertising executives at customer firms. Mr. Smith only recently started working with his junior partners to establish similar relationships with these customers. Mr. Smith is approaching 65 years of age and might announce his retirement at the next board meeting. If he does retire, revenues will drop significantly and earnings are estimated to shrink by 30 percent. You estimate that the probability that Mr. Smith will retire this year is 50 percent. If he does not retire this year, you expect that Mr. Smith will have sufficient time to work with his junior partners so his departure will not affect earnings when he departs. How does this information affect your estimate of the value of the firm?
Solution: Mr. Smith is a key person in this firm. An adjustment should be made to the valuation to account for his potential departure this year. Taking the possibility that Mr. Smith will retire into account, the expected earnings next year will be: (2,000,000 × 0.5) + [$2,000,000 × (1 – 0.30) × 0.5] = $1,700,000
SM 19-782
Therefore, the adjusted value for the firm is: $1.7 million × 48 = $81.6 million. We can see that this implies a 15 percent key person discount from the original estimate of $96 million [($81.6 million ─ $96.0 million)/$96.0 million = ─ 0.15, or ─15 percent].
Discussion Questions 18.1
Given that many new businesses fail in the first few years after they are
established, how should an entrepreneur think about the risk of failure associated with a new business? From what you have learned in this chapter, what can an entrepreneur do to increase the chance of success?
First of all, it is important for an entrepreneur to understand why many new businesses fail. Common reasons include poor products, poor business strategy, poor management skills, and wrong financing decisions. However, the risk of failure should not stop the start of a new business if the entrepreneur believes that he or she has a great business idea and is capable of implementing the idea. The entrepreneur should be careful and realistic in assessing the opportunities and taking risks into consideration. From what we have learned in this chapter, the entrepreneur should consult an attorney and choose the right organizational form according to the business’s operating and financing needs. In addition, he or she should carefully prepare a business plan that incorporates the main aspects of the business and facilitates the raising of external capital. The entrepreneur can use several tools to assess the financing needs of the new business: cash flow break-even analysis and cash budget. To value the new business, the
SM 19-783
entrepreneur can use one or more of the following methods: cost approach, market approach, or income approach. LO 1 Level: Basic Bloomcode: Comprehension AASCB: Analytic IMA: Corporate Finance AICPA: Industry/Sector Perspective
18.2
Explain how the taxation of a C-corporation differs from the taxation of the
other forms of business organization discussed in this chapter. With the exception of C-corporations, all profits flow through to the owners in proportion to their ownership interests. These owners pay taxes on the business profits only when they file their personal tax returns. In contrast, profits earned in C-corporations are taxed at the corporate tax rate, and the after-tax profits are taxed a second time when they are distributed to stockholders in the form of dividends. On the bright side, because profits are taxed in the corporation, certain benefits like health insurances that are paid to stockholders who work in a C-corporation are tax deductible. These benefits are not generally deductible with the other forms of organization. LO 1 Level: Basic Bloomcode: Analysis AASCB: Analytic IMA: Corporate Finance AICPA: Industry/Sector Perspective
18.3 What is a business plan? Explain how a business plan can help an entrepreneur succeed in building a business.
SM 19-784
A business plan is a document that describes the details of how a business will be developed over time. A well-prepared business plan helps an entrepreneur raise external capital. It makes it easier for an entrepreneur to communicate to potential investors precisely what he or she expects the business to look like in the future, how he or she expects to get it to that point, and what returns an investor might expect to receive. In addition, a business plan is a tool that can help an entrepreneur set the goals and objectives for the company, serve as a benchmark for evaluating and controlling the company's performance, and communicate the entrepreneur’s ideas to managers and stakeholders. LO 2 Level: Basic Bloomcode: Knowledge AASCB: Analytic IMA: Corporate Finance AICPA: Industry/Sector Perspective
18.4 You are entering negotiations to purchase a business and are trying to formulate a negotiating strategy. You want to determine the minimum price you should offer and the maximum you should be willing to pay. Explain how the concepts of fair market value and investment value can help you do this.
An estimate of the fair market value of a business would provide you with an idea of the price that the seller could probably get from someone else. Since this is an indication of the alternatives available to the seller, it is a useful benchmark for determining the minimum price you should offer. The investment value of the business to you represents the maximum price that you should pay. If you pay more than the investment value, the acquisition will be a negative NPV project. LO 3 Level: Intermediate Bloomcode: Application AASCB: Analytic IMA: Corporate Finance AICPA: Industry/Sector Perspective SM 19-785
18.5 You have just received a business valuation report that is dated six months ago. Describe the factors that might have changed during the past six months and, therefore, caused the value of the business today to be different from the value six months ago. Which of these changes affect the expected cash flows, and which affect the discount rate that you would use in a discounted cash flow valuation of this company?
A lot might have changed in the last six months. For example, there might have been changes in general economic conditions, capital market conditions, the competitive environment in the industries in which the company competes, and in company characteristics such as the products it sells, the geographic markets it competes in, and management quality. Changes in all of these areas can affect expected cash flows through their impact on demand for the company’s products, the prices of these products, production costs, and investment requirements. Changes in capital market conditions can affect the company’s cost of capital and therefore the discount rate. LO 3 Level: Basic Bloomcode: Analysis AASCB: Analytic IMA: Corporate Finance AICPA: Industry/Sector Perspective
18.6 Is the replacement cost of a business generally related to the value of the cash flows that the business is expected to produce in the future? Why or why not? Illustrate your answer with an example.
The replacement cost of a business is an estimate of what it would cost to replace the tangible and intangible assets of an on-going business rather than a reflection of the value of the expected future cash flows of the business. Let us say that you are planning to start up a new coffee shop. You have a choice of either buying an existing coffee shop or starting up one entirely from scratch. The replacement cost of the coffee shop business SM 19-786
should include the cost of all tangible assets, such as property, plant, and equipment, and all intangible assets, such as brand names, customer lists, and the cost of hiring the people necessary to run the business and the time that it would take to build the business. This provides you a fair value of what you would pay for an existing business. If you can start up a business from start at a lower cost than that estimated value, then you should not buy up the existing business. It does not reflect the cash flows that the existing coffee shop will produce in the future, but only what it is worth now. LO 3 Level: Intermediate Bloomcode: Application AASCB: Analytic IMA: Corporate Finance AICPA: Industry/Sector Perspective
18.7 You want to estimate the value of a company that has three very different lines of business. It manufactures aircraft, is in the data-processing business, and manufactures automobiles. How could you use an income approach to value a company such as this—one with three very distinct businesses that will have different revenue growth rates, profit margins, investment requirements, discount rates, and so forth?
In using an income approach like the free cash flow from the firm (FCFF) to value an entire firm, one would first estimate the expected future cash flows from each of the three distinct lines of business. If all three lines of businesses are financed the same way, then one would add up the cash flows across all the different lines of businesses and discount them using the firm’s WACC to estimate the value of the firm. If, however, the three distinct businesses have been financed differently, then each of the business’s value should be estimated independently and then finally added up to determine the value of the firm. LO 3 Level: Basic Bloomcode: Application SM 19-787
AASCB: Analytic IMA: Corporate Finance AICPA: Industry/Sector Perspective
18.8 Your boss has asked you to estimate the intrinsic value of the equity for Google, which does not currently pay any dividends. You are going to use an income approach and are trying to choose between the free cash flow to equity (FCFE) approach and the dividend discount model (DDM) approach. Which would be more appropriate in this instance? Why? What concerns would you have in applying either of these valuation approaches to a company such as this?
Google is a fast growing, young firm that is unlikely to pay dividends in the near future. The dividend discount model (DDM) approach typically uses the constant-growth model to determine the intrinsic value of the firm’s equity. This model will not be applicable to firms like Google until it has settled down to a steady growth pattern at some point in the future. In contrast, the FCFE approach values cash flows that are available for distribution to stockholders. It allows for the firm to not distribute all available cash flows as dividends in any particular year. Hence, it would be more appropriate to use the FCFE approach to determine the intrinsic value of Google’s equity. LO 3 Level: Intermediate Bloomcode: Analysis AASCB: Analytic IMA: Corporate Finance AICPA: Industry/Sector Perspective
18.9 Explain how the financial statements of a private company might differ from those of a public company. What does this imply for valuing a private company? Financial statements of many private companies are not as complete and reliable as those of public companies. One needs to take caution in valuing a private business based on its financial statements. For example, private company financials often include personal SM 19-788
expenses of the owner and extremely high compensation expenses. Excess compensations for the owners benefit them by voiding double taxation. To value a private firm more precisely, adjustments for excess expenses and compensations should be made. LO 4 Level: Basic Bloomcode: Analysis AASCB: Analytic IMA: Corporate Finance AICPA: Industry/Sector Perspective
18.10 Explain why it is difficult to value a young, rapidly growing company.
It is difficult to value a young, rapidly growing company for several reasons. First, such a company has less reliable historical information available; second, the future of such a company is often less certain than a matured company; third, such a company is usually not yet profitable, and therefore it is difficult to apply earnings multiples; and finally, such a company usually invests. LO 4 Level: Basic Bloomcode: Comprehension AASCB: Analytic IMA: Corporate Finance AICPA: Industry/Sector Perspective
Questions and Problems BASIC 18.1 Organizational form: List some common forms of business organization, and discuss how access to capital differs across these forms of organization.
SM 19-789
Solution: The forms of organizations discussed in this chapter include: sole proprietorship, partnership (general partnership and limited partnership), limited liability company (LLC), and corporations (S-corporation and C-corporation). The closer the type of organization is to the end of the list above, the better is its access to capital. Sole proprietorships must rely on equity contribution from the proprietor and debt or lease financing. In contrast, partnerships can turn to all of the partners for additional capital, and corporations can sell shares to both insiders and outsiders. Limited partnerships and limited liability companies (LLCs) are less constrained than general partnerships because they can raise money from limited partners or ―members,‖ as outside investors in LLCs are called, who are not directly involved in running the business. C-corporations can have virtually an unlimited number of stockholders. LO 1 Bloomcode: Knowledge AASCB: Analytic IMA: Corporate Finance AICPA: Industry/Sector Perspective
18.2 Starting a business: What are some of the things that the founder of a company must do to launch a new business?
Solution: The founder of a company needs to decide on its products, market, and business strategy. Then, he or she needs to estimate the company’s financing needs and raise the money to develop the products, acquire assets, and hire employees. As the business is being built, the founder must also manage the day-to-day operations. LO 1 Bloomcode: Knowledge AASCB: Analytic IMA: Corporate Finance AICPA: Industry/Sector Perspective
SM 19-790
18.3 Organizational form: Explain how financial liabilities differ among different forms of business organization.
Solution: Sole proprietorships, general partnerships, and limited partnerships all are at a disadvantage. Sole proprietors and general partnerships face the possibility that their personal assets can be taken from them to satisfy claims on their businesses. In contrast, the liabilities of investors in limited partnerships, LLCs, and corporations are limited to the money that they have invested in the business. LO 1 Bloomcode: Comprehension AASCB: Analytic IMA: Corporate Finance AICPA: Industry/Sector Perspective
18.4 Cash requirements: List two useful tools to help an entrepreneur to understand the cash requirements of a business and to estimate the financing needs of his or her business.
Solution: An entrepreneur can use cash flow break-even analysis and cash budget to estimate the cash requirement and financing needs of his or her business. The cash flow break-even analysis is used to compute the level of unit sales that is necessary to break even on operations from a pretax operating cash flow perspective. The cash budget presents the cash inflows and outflows as well as additional financing needs, usually on a monthly basis but can be prepared for any period, including daily or weekly. LO 1 Bloomcode: Knowledge AASCB: Analytic IMA: Corporate Finance AICPA: Industry/Sector Perspective
SM 19-791
18.5 Cash requirements: You believe you have a great business idea and want to start your own company. However, you do not have enough savings to finance it. Where can you get the additional funds you need?
Solution: You can try to raise additional equity financing from your friends and family, from venture capitalists, or from other potential investors that you know. Debt financing might be obtained through bank loans, cash advances on credit cards, or loans from other local individual investors or other businesses. Of course, if a company is large enough, equity and debt financing can be obtained in the public markets. LO 1 Bloomcode: Knowledge AASCB: Analytic IMA: Corporate Finance AICPA: Industry/Sector Perspective
18.6 Raising capital: Why is it especially difficult for an entrepreneur with a new business to raise capital? What tool can help him or her to raise external capital?
Solution: To raise external capital, an entrepreneur must convince potential investors that purchasing debt or equity from the firm will yield attractive returns, given the risks they will bear. It is especially difficult for an entrepreneur with a new business to do this, because the business does not have a well-established record and its future is typically very uncertain. A well-written business plan can be very helpful in convincing potential investors to put their money in the business. LO 2 Bloomcode: Comprehension AASCB: Analytic IMA: Corporate Finance AICPA: Industry/Sector Perspective SM 19-792
18.7
Replacement cost: What is the replacement cost of a business?
Solution: The replacement cost is the cost of duplicating the assets of a business in their present form, consisting of tangible and intangible assets as of the valuation date. LO 3 Bloomcode: Knowledge AASCB: Analytic IMA: Corporate Finance AICPA: Industry/Sector Perspective
18.8 Multiples analysis: It is April 4, 2015, and your company is considering the possibility of purchasing the Chrysler automobile manufacturing business. Managers of Fiat, the Italian automobile manufacturer that owns Chrysler, have hinted that they might be interested in selling the firm. Since Chrysler does not have publicly traded shares of its own, you have decided to use Ford Motor Company as a comparable company to help you determine the market value of Chrysler. This morning Ford’s common stock was trading at $16.69 per share, and the company had 3.47 billion shares outstanding. You estimated that the market value of all of the company’s other outstanding securities (excluding the common stock but including special shares owned by the Ford family) is $100 billion and that its revenues from auto sales were $133.4 billion last year. Chrysler’s revenue in 2014 was $50.0 billion. Based on the enterprise value/revenue ratio, what is the total value of Chrysler that is implied by the Ford market values?
Solution: The enterprise value of Ford is:
VF = VE + VOther outstanding securities = ($16.69 per share × 3.47 billion shares) + $100 billion = $157.91 billion SM 19-793
and the enterprise value/revenue ratio is:
$157.91 billion/$133.4 billion = 1.18
This suggests that the value of the Chrysler business is 1.18 × $50.0 billion = $59.0 billion. (Note that in this example we used the enterprise value/revenue ratio because EDITDA was unclear for Chrysler.) LO 3 Bloomcode: Application AASCB: Analytic IMA: Corporate Finance AICPA: Industry/Sector Perspective
18.9 Nonoperating assets: Why is excess cash a nonoperating asset (NOA)? Why does it make sense to add the value of excess cash to the value of the discounted cash flows when we use the WACC or FCFE approach to value a business?
Solution: Excess cash is a nonoperating asset because, by definition, this cash can be distributed to stockholders without affecting the operations of the business and therefore the value of the expected future cash flows. It makes sense to add back the value of excess cash because it represents value over and above that which the business is expected to produce. LO 3 Bloomcode: Comprehension AASCB: Analytic IMA: Corporate Finance AICPA: Industry/Sector Perspective
18.10 Dividend discount approach: You want to estimate the total intrinsic value of a large gas and electric utility company. This company has publicly traded stock and has been paying a regular dividend for many years. You decide that, due to the predictability SM 19-794
of the dividend that this company pays, you can use the dividend discount valuation approach. The company is expected to pay a dividend of $1.25 per share next year, and the dividend is expected to grow at a rate of 3 percent per year thereafter. You estimate that the appropriate rate for discounting future dividends is 12 percent. In addition, you know that the company has 46 million shares outstanding and that the market value of its debt is $350 million. What is the total enterprise value of the company?
Solution: The value of a share of stock can be calculated using the constant-growth dividend model as: P0 =
D1 $1.25 $13.89. kcs - g 0.12 0.03
Multiplying the price per share by the number of shares outstanding gives us the value of the equity: VE = $13.89 × 46 million = $638.94 million Therefore, the total value of the firm is: VF = VE + VD = $638.94 million + $350 million = $988.94 million
LO 3 Bloomcode: Application AASCB: Analytic IMA: Corporate Finance AICPA: Industry/Sector Perspective
18.11 Public versus private company valuation: You are considering investing in a private company that is owned by a friend of yours. You have read through the company’s financial statements and believe that they are reliable. Multiples of similar publicly traded companies in the same industry suggest that the value of a share of stock in your friend’s company is $12. Should you be willing to pay $12 per share?
Solution: Shareholders in a private company may have to spend considerable resources in money and time to sell their shares. This additional transaction cost will make value derived from similar publicly traded company an overestimated figure. Therefore, you will most
SM 19-795
likely not be willing to pay for $12 a share. You can apply an appropriate marketability discount to this price as a fair price for this private equity. LO 4 Bloomcode: Analysis AASCB: Analytic IMA: Corporate Finance AICPA: Industry/Sector Perspective
18.12 Control: Does the expected rate of return that is calculated using CAPM, with a beta estimated from stock returns in the public market, reflect a minority or a controlling ownership position? How is it likely to differ between a minority and a controlling position?
Solution: The estimates we obtain using public market data with CAPM are based on small stock transactions. To the extent that having control would enable an investor to better manage the systematic risk associated with a business, a discount rate for a controlling transaction is likely to be lower than that implied by CAPM (this generates a higher value and therefore a premium price for control). LO 4 Bloomcode: Comprehension AASCB: Analytic IMA: Corporate Finance AICPA: Industry/Sector Perspective
INTERMEDIATE
18.13 Organizational form: Compare the characteristics of an LLC with those of a partnership and a C-corporation.
Solution:
SM 19-796
An LLC is a hybrid of a limited partnership and a corporation. Like a corporation, an LLC provides limited liability for the people who make the business decisions in the firm while enabling all investors to retain the tax advantages of a limited partnership. LO 1 Bloomcode: Analysis AASCB: Analytic IMA: Corporate Finance AICPA: Industry/Sector Perspective
18.14 Organizational form: Discuss the pros and cons of an S-corporation compared with a C-corporation.
Solution: An S-corporation is a variation of the C-corporation. The advantage of an S-corporation compared to a C-corporation is that all of its profits are passed directly to the stockholders, and therefore double taxation is avoided while it still offers the protection of limited liability for shareholders. The disadvantage of an S-corporation is the limitation on stock ownership. Currently, an S-corporation can have no more than 100 stockholders and have only one class of common stock, and all stockholders must be individuals who are U.S. citizens or residents. As a result, access to capital is less limited, and cost to transfer ownership is much higher for an S-corporation than for a Ccorporation. LO 1 Bloomcode: Analysis AASCB: Analytic IMA: Corporate Finance AICPA: Industry/Sector Perspective
18.15 Break-even: You have started a business that sells a home gardening system that allows people to grow vegetables on the countertop in their kitchens. You are considering two options for marketing your product. The first is to advertise on local TV. The second is to distribute flyers in the local community. The TV option, which costs $50,000 annually, will promote the product more effectively and create a SM 19-797
demand for 1,200 units per year. The flyer advertisement option costs only $6,000 annually but will create a demand for only 250 units per year. The price per unit of the indoor gardening system is $100, and the variable cost is $60 per unit. Assume that the production capacity is not limited and that the marketing cost is the only fixed cost involved in your business. What are the break-even points for both marketing options? Which one should you choose?
Solution: The break-even point for TV advertisement = $50,000/$40 = 1,250 units. However, the demand is only 1,200 units with TV advertisement. Therefore you cannot break even. The break-even point for flyers = $6,000/$40 = 150 units. With a market demand of 250, you will make a profit. Therefore you should choose the flyer option. LO 1 Bloomcode: Application AASCB: Analytic IMA: Corporate Finance AICPA: Industry/Sector Perspective
18.16 Going-concern value: Aggie Motors is a chain of used car dealerships that has publicly traded stock. Using the adjusted book value approach, you have estimated the value of Aggie Motors to be $45,646,000. The company has $40.5 million of debt outstanding. Its stock price is $5.5 per share, and there are 1,378,000 shares outstanding. What is the going concern value of Aggie Motors?
Solution: The going-concern value equals the difference between the market value of the business (the present value of the expected cash flows) and its adjusted book value. The market value of the business can be calculated as: VF = VD + VE = $40,500,000 + ($5.5 × 1,378,000) = $48,079,000 Therefore, the going concern value is $48,079,000 – $45,646,000 = $2,433,000 LO 3 Bloomcode: Application AASCB: Analytic IMA: Corporate Finance SM 19-798
AICPA: Industry/Sector Perspective
Use the following information concerning Johnson Machine Tool Company in Problems 18.17, 18.18, and 18.19. Johnson’s income statement from the fiscal year that ended this past December is: Revenue
$995
Cost of goods sold
652
Gross profit
$343
Selling, general, & administrative expenses
135
Operating profit (EBIT)
$208
Interest expense Earnings before taxes Taxes Net income
48 $160 64 $ 96
All dollar values are in millions. Depreciation and amortization expenses last year were $42 million, and the company has $533 million of debt outstanding.
18.17 Multiples analysis: You are an analyst at a private equity firm that buys private companies, improves their operating performance, and sells them for a profit. Your boss has asked you to estimate the fair market value of the Johnson Machine Tool Company. Billy’s Tools is a public company with business operations that are virtually identical to those at Johnson. The most recent income statement for Billy’s Tools is as follows:
Revenue
$1,764
Cost of goods sold
1,168
Gross profit
$ 596
Selling, general, & administrative expenses Operating profit (EBIT) Interest expense Earnings before taxes
211 $ 385 12 $ 373 SM 19-799
Taxes
147
Net income
$ 226
All dollar values are in millions. Billy’s had depreciation and amortization expenses of $71 million last year and had 200 million shares and $600 million of debt outstanding as of the end of the year. Its stock is currently trading at $12.25 per share. Using the P/E multiple, what is the per share value of Johnson’s stock? What is the total value of Johnson Machine Tool Company?
Solution: The P/E multiple for Billy’s Tools is: P / E ($12.25 200 million shares) / $226 10.84
which implies a value of 10.84 × $96 = $1,040.64 million for the equity of Johnson Machine Tools Company. Therefore, the implied total value of Johnson is: VF = VE + VD = $1,040.64 + $533.00 = $1,573.64 million LO 3 Bloomcode: Application AASCB: Analytic IMA: Corporate Finance AICPA: Industry/Sector Perspective
18.18 Multiples analysis: Using the enterprise value/EBITDA multiple, what is the total value of Johnson Machine Tool Company? What is the per share value of Johnson’s stock?
Solution: The enterprise value/EBITDA multiple for Billy’s is: EV/EBITDA = (($12.25 × 200 million shares) + $600 million debt) / ($385 million EBIT+ $71 million D&A) = 6.69 Therefore, the implied total value of Johnson is: VF = 6.69 × ($208 million EBIT + $42 million D&A) = $1,672.50 SM 19-800
And the value of its stock is: VE = VF – VD = $1,672.50 million - $533.00 million = $1,139.50 million LO 3 Bloomcode: Application AASCB: Analytic IMA: Corporate Finance AICPA: Industry/Sector Perspective
18.19 Multiples analysis: Which of the above multiples analyses do you believe is more appropriate?
Solution: While the value estimates in the previous questions are reasonably similar, the enterprise value/EBITDA multiple is more appropriate for this analysis. The reason is that the capital structures of Johnson and Billy’s differ considerably and the enterprise value/EBITDA multiple is less sensitive to differences in leverage. The debt/total capital ratio for Billy’s is $600 million /(($12.25 × 200 million shares) + $600 million debt) = 0.196 The debt/total capital ratio for Johnson is: $533 million / $1,672.50 million = 0.319 LO 3 Bloomcode: Analysis AASCB: Analytic IMA: Corporate Finance AICPA: Industry/Sector Perspective
18.20 Income approaches: You are using the FCFF approach to value a business. You have estimated that the FCFF for next year will be $123.65 million and that it will increase at a rate of 8 percent for each of the following four years. After that point, the FCFF will increase at a rate of 3 percent forever. If the WACC for this firm is 10 percent and it has no NOA, what is it worth?
Solution: SM 19-801
You can value the FCFF for the first five years using the growing annuity formula from Chapter 6 (Equation 6.5). 1 + g n $123.65 1.08 5 PVAn = ×1- ×1- = (WACC - g) 1 + WACC 0.10 - 0.08 1.1 CF
1
=$541.98 million The present value of the terminal value is 𝐹𝐶𝐹𝐹1 × (1 + 𝑔1)4 × (1 + 𝑔2) $123.65 × (1 + 0.08)4 × (1 + 0.03) ] [ [ W𝐴𝐶𝐶 − 𝑔2 ] 0.1 − 0.03 = 𝑃𝑉(𝑇𝑉5) = (1 + W𝐴𝐶𝐶)5 (1 + 0.1)5 = $1,536.97 million and the value of the firm is therefore: VF = $541.98 million + $1,536.97 million = $2,078.95 million LO 3 Bloomcode: Application AASCB: Analytic IMA: Corporate Finance AICPA: Industry/Sector Perspective
18.21 Valuing a private business: You want to estimate the value of a privately owned restaurant that is financed entirely with equity. Its most recent income statement is as follows:
Revenue Cost of goods sold
$3,000,000 600,000
Gross profit
$2,400,000
Salaries and wages
1,400,000
Selling expenses
100,000
Operating profit (EBIT) Taxes Net income
$ 900,000 315,000 $ 585,000
You note that the profitability of this restaurant is significantly lower than that of comparable restaurants, primarily due to high salary and wage expenses. Further investigation reveals that the annual salaries for the SM 19-802
owner and his wife, the firm’s accountant, are $900,000 and $300,000, respectively. These salaries are much higher than the industry median salaries for these two positions of $100,000 and $50,000, respectively. Compensation for other employees ($200,000 in total) appears to be consistent with the market rates. The median P/E ratio of comparable restaurants with no debt is 10. What is the total value of this restaurant?
Solution: The owner is paying himself and his wife salaries that are above normal for this industry, probably to avoid double taxation (assuming that the restaurant is organized as a C corporation). You should adjust the income statement to reflect the market rates of compensation for these positions. The adjusted income statement is as follows:
Revenue Cost of goods sold Gross profit
$3,000,000 600,000 $2,400,000
Salaries and wages
350,000
Selling expenses
100,000
Operating profit (EBIT) Taxes Net income
$1,950,000 682,500 $1,267,500
Therefore you estimate the company’s value to be: $1,267,500 × 10 = $12,675,000. The value of the company will be underestimated if no adjustment for excess compensation is made. LO 3 Bloomcode: Application AASCB: Analytic IMA: Corporate Finance AICPA: Industry/Sector Perspective
18.22 Valuing a private business: A few years ago, a friend of yours started a small business that develops gaming software. The company is doing well and is valued at $1.5 million based on multiples for comparable public companies after adjustments for their lack of marketability. With 300,000 shares outstanding, each share is estimated to be SM 19-803
worth $5. Your friend, who has been serving as CEO and CTO (chief technology officer), has decided that he lacks sufficient managerial skills to continue to build the company. He wants to sell his 160,000 shares and invest the money in an MBA education. You believe you have the appropriate managerial skills to run the company. Would you pay $5 each for these shares? What are some of the factors you should consider in making this decision?
Solution: You should consider at least two additional factors in valuing these equity shares. First, note that since the company has a total 300,000 shares of equity outstanding, you will be purchasing a majority of the shares and will gain control of the company. Second, while you might be able to use your excellent managerial skills to create new value for the firm, if your friend is a key person, you must consider the impact of his departure on the development and sales of the company’s products. Your valuation of the equity could be higher or lower than $5 per share, based on adjustments for both potential control premium and key person discount. LO 4 Bloomcode: Analysis AASCB: Analytic IMA: Corporate Finance AICPA: Industry/Sector Perspective
ADVANCED 18.23 You plan to start a business that sells waterproof sun block with a unique formula that reduces the damage of UVA radiation 30 percent more effectively than similar products on the market. You expect to invest $50,000 in plant and equipment to begin the business. The targeted price of the sun block is $15 per bottle. You forecast that unit sales will total 1,500 bottles in the first month and will increase by 20 percent in each of the following months during the first year. You expect the cost of raw materials to be $3 per bottle. In addition, monthly gross wages and payroll are expected to be $13,000, rent is expected to be $3,000, and other expenses are expected to total $1,000. Advertising costs are SM 19-804
estimated to be $35,000 in the first month, but to remain constant at $5,000 per month during the following eleven months. You have decided to finance the entire business at one time using your own savings. Is an initial investment of $75,000 adequate to avoid a negative cash balance in any given month? If not, how much more do you need to invest up front? How much do you need to invest up front to keep a minimum cash balance of $5,000? What is the break-even point for the business?
Solution: The monthly cash budget is as follows: Monthly Cash Budget Month
1
2
3
4
Beginning cash balance
$ 75,000
$ (9,000)
$ (9,400)
$ (5,480)
22,500
27,000
32,400
38,880
$ 97,500
$ 18,000
$ 23,000
$ 33,400
Raw material
4,500
5,400
6,480
7,776
Gross wages and payroll
13,000
13,000
13,000
13,000
Advertising
35,000
5,000
5,000
5,000
Rent
3,000
3,000
3,000
3,000
Cash receipts Cash sales Total cash available Operations
Other expenses
1,000
1,000
1,000
1,000
Operations total
$ 56,500
$ 27,400
$ 28,480
$ 29,776
---
---
---
Financing and investments Capital expenditures
50,000
Total cash payments
$106,500
$ 27,400
$ 28,480
$ 29,776
Ending cash balance
$ (9,000)
$ (9,400)
$ (5,480)
$ 3,624
With $75,000 of capital invested in the initial period, the company will have negative cash balances during each of the first three months and positive cash balances in the following months. Additional capital investment totaling $9,400 will be needed to SM 19-805
avoid negative cash balances in month 2. In other words, you will have to invest $84,400 of capital in total at the beginning of the business just to avoid negative cash balances. To maintain a minimum cash balance of $5,000, you will need to make an initial capital investment of $89,400 ($84,000 + $5,000). Break-even: Fixed costs in the initial month will equal: $13,000 + $35,000 + $3,000 + $1,000 = $52,000 Fixed monthly costs in the following months will be: $13,000 + $5,000 + $3,000 + $1,000 = $22,000. Since variable costs per bottle are $3, the monthly break-even points for the firm are: $52,000 / ($15 – $3) = 4,333.3 bottles in the initial month and $22,000 / ($15 – $3) =1,833.3 bottles in the following months. The sales of the company are 1,500, 1,800, 2,160, and 2,592 bottles in the first four months. Therefore, the firm will start to make a profit in the third month. This can be seen from the cash budget analysis above. Month 3 is the first month the firm’s cash balance is increasing (becoming less negative). LO 1 Bloomcode: Application AASCB: Analytic IMA: Corporate Finance AICPA: Industry/Sector Perspective
18.24 For the previous question, assume that you do not have sufficient savings to cover the entire amount required to start your sun-block business. You are going to have to get external financing. A local banker whom you know has offered you a six-month loan of $20,000 at an APR of 12 percent. You will pay interest each month and repay the entire principal at the end of six months. Assume that instead of making a single up-front investment, you are going to finance the business by making monthly investments as cash is needed in the business. If the proceeds from the loan go directly into the business on the first day and are therefore available to pay for some of the capital expenditures, how much money do you need to pull out of your savings account every month to run the business and keep the cash balances positive? SM 19-806
Solution: The monthly payment of the loan is $20,000 ×1% = $200. The cash budget without your investment is as follows: Monthly Cash Budget Month
1
2
3
4
5
6
Beginning cash balance
$ 20,000
$(64,200) $(64,800) $(61,080) $(52,176) $(37,051)
Cash receipts Investments by owner Cash sales
22,500
27,000
32,400
38,880
46,656
55,987
$ 42,500
$(37,200) $(32,400) $(22,200) $(5,520)
$ 18,936
4,500
5,400
6,480
7,776
9,331
11,197
and 13,000
13,000
13,000
13,000
13,000
13,000
Advertising
35,000
5,000
5,000
5,000
5,000
5,000
Rent
3,000
3,000
3,000
3,000
3,000
3,000
Total cash available Cash payments Operations Raw material Gross
wages
payroll
Other expenses
1,000
1,000
1,000
1,000
1,000
1,000
Operations total
$ 56,500
$ 27,400
$ 28,480
$ 29,776
$ 31,331
$ 33,197
Capital expenditures
50,000
---
---
---
---
---
Debt/interest payment
200
200
200
200
200
20, 200
Total cash payments
$106,700
$ 27,600
$28,680
$ 29,976
$ 31,531
$ 53,397
Ending cash balance
$(64,200) $(64,800) $(61,080) $(52,176) $(37,051) $(34,461)
Financing
and
investments
Therefore, you need to put in $64,200 initially, and $600 ($68,400–64,200) in the next month. You don’t need to invest in more money afterward, because the cash balance is increasing afterward. LO 1 SM 19-807
Bloomcode: Application AASCB: Analytic IMA: Corporate Finance AICPA: Industry/Sector Perspective
18.25 Your friend is starting a new company. He wants to write a business plan to clarify the company’s business outlook and raise venture capital. Knowing that you have taken this course, he has asked you, as a favor, to help him prepare a template for a business plan. Prepare a template that includes the key elements of a business plan.
Solution: Business varies and so does a business plan. Here is a template for an intermediate-level business plan, which could be tailored to specific industry and company situations: 1. Executive Summary 1.1 Objectives 1.2 Mission 1.3 Keys to Success 2. Company Overview 5. Product Description 6.
Market Analysis 2.1 Market Segmentation 2.2 Target Market Segment Strategy 2.3 Market Needs 2.4 Competitions and Buying Patterns
5. Marketing and Sales 5.1 Competitive Edge 5.2 Sales Strategy 6. Operations 6.1 Production 6.2 Distribution 6.3 Supply 7. Management and Ownership SM 19-808
7.1 Organizational Structure 7.2 Management Team 7.3 Ownership 8. Financial Plan 8.1 Break-even Analysis 8.2 Projected Profit and Loss 8.3 Projected Cash Flow 9. Appendices: Tables and charts LO 2 Bloomcode: Application AASCB: Analytic IMA: Corporate Finance AICPA: Industry/Sector Perspective
18.26 A friend of yours is trying to value the equity of a company and, knowing that you have read this book, has asked for your help. So far she has tried to use the FCFE approach. She estimated the cash flows to equity to be as follows:
Sales
$800.0
− CGS
−450.0
− Depreciation
−80.0
− Interest
−24.0
Earnings before taxes (EBT)
$246.0
− Taxes (0.35 × EBT)
−86.1
= Cash flow to equity
$159.9
She also computed the cost of equity using CAPM as follows: kE = kF
E(Risk premium) = 0.06 + (1.25 × 0.084) = 0.165 or 16.5%
where the beta is estimated for a comparable publicly traded company. Using this cost of equity, she estimates the discount rate as WACC = xDebtkDebt pretax(1 − t) + xcs kcs SM 19-809
=[ 0.20 × 0.06 × (1 − 0.35)] + (0.80 × 0.165) = 0.14, or 14% Based on this analysis, she concludes that the value of equity is $159.9 million/0.14 = $1,142 million. Assuming that the numbers used in this analysis are all correct, what advice would you give your friend regarding her analysis?
Solution: There are a number of potential problems with your friend’s analysis. First, she has calculated FCFF incorrectly. She is assuming that net income equals FCFE. Once she obtains net income she should add back depreciation, subtract capital expenditures and additions to working capital, and subtract any net repayment of debt principal. Second, she is using the incorrect discount rate. She should be using the cost of equity to discount the FCFE. Finally, since your friend is using the perpetuity formula, she is assuming that the expected future FCFE will be constant forever. You should inquire whether this is really what she expects. LO 3 Bloomcode: Analysis AASCB: Analytic IMA: Corporate Finance AICPA: Industry/Sector Perspective
18.27 Forever Youth Technology is a biochemical company that is two years old. Its main product, an antioxidant drink that is supposed to energize the consumer and delay aging, is still under development. The company’s equity consists of $5 million invested by its founders and $5 million from a venture capitalist. The company has spent $3 million in each of the past two years, mostly on lab equipment and R&D costs. The company has had no sales so far. What are the challenges associated with valuing such a young and uncertain company?
Solution: As discussed in the chapter, it is very difficult to value such a young firm with many uncertainties. The company has a short history, high investments, no sales, and highly SM 19-810
uncertain cash flows in the future. The cost approach is not valid for such a young biochemical company. It is hard to value the company using multiples analysis because of the lack of sales and negativity of earnings, and because of lack of comparable companies of the same business. The transaction analysis approach is also hard to apply. Despite the many uncertainties, we should try to estimate the future cash flows and risks associated with these cash flows. First of all, we need to estimate the probability and time length to achieve success in developing the product. In addition, we need to find out what happens in case the firm’s capital is used up before the product is developed. We need to decide whether, based on the company’s future prospects, it is possible to raise additional external capital at that point of time, and if not, we need to decide whether the business has any liquidation value. Then, we need to estimate the market demand and shares as well as target price of the product, in case it is developed successfully. We also need to estimate the short-term and long-term growth rate of the company based on market, industry, and firm-specific conditions. LO 4 Bloomcode: Analysis AASCB: Analytic IMA: Corporate Finance AICPA: Industry/Sector Perspective
18.28 Mad Rock Inc. is a company that sells mp3 music online. It is expected to generate earnings of $1 per share this year after its Web site is upgraded and online marketing is stepped up. Given the popularity of the iPod and iPad devices, the stock price of Mad Rock has rocketed from $8 to $95 per share in the past 12 months. The cost of capital for the company is 18 percent. Of course, the future of a young Internet company such as Mad Rock is highly uncertain. Nevertheless, using the very limited information provided in this problem, do you think $95 per share could be a fair price for its stock? Support your argument with a simple analysis.
Solution:
SM 19-811
Assume that the earnings of $1 will be realized. To simplify the problem, assume that all earnings will be paid out as dividends. Using the dividend growth model, $95 = $1/(0.18 – g), which implies that g = 16.9%. We know that it is not possible for a firm to grow at this rate forever because G is a growth rate that must be sustainable in perpetuity. No firm can grow indefinitely at such a high rate, which is many times the average growth rate of the world economy. Therefore, this analysis suggests that Mad Rock’s stock is overpriced. On the other hand, it is possible that Mad Rock could grow very rapidly for a few years and then grow at a sustainable rate that justifies the $95 price. For example, if we assume that a sustainable rate is 6 percent, we can solve for the earnings per share that would be required to justify a $95 stock price. This value is $95 = D/(0.18 – 0.06) which implies D = $11.4. Therefore, if the earnings per share increased rapidly to a value above $11.4, it is possible that a price of $95 could be justified. How much above $11.4 it would have to be, would depend on how quickly it increased. One other consideration here is that, as the business matures, its cost of
capital is likely to decrease because its risk will decline. This will tend to increase the value of the equity. LO 4 Bloomcode: Analysis AASCB: Analytic IMA: Corporate Finance AICPA: Industry/Sector Perspective
18.29 At the end of 2014 the value of the S&P 500 Index divided by the estimated 2014 earnings for S&P 500 firms (the S&P 500 P/E multiple) was 18.66. Assume that the long term Treasury bond yield was 3.72 percent, the market risk premium was 5.17 percent, and firms in the S&P 500 were expected to pay out an average of 37.6 percent of their earnings as dividends in the future. At what rate were dividends paid by S&P 500 firms expected to grow in the future? SM 19-812
Solution: The equation of Price/earnings multiple based on constant-growth model can be used to answer this question.
P0 b = E1 kcs g
Recognizing that E1 = E0 × (1 + g) we can rewrite this equation as:
P0 E0
=
(1+ g)b kcs g
Since we know that P0/E0 = 18.66, and b = 37.6%, all that we have to do is calculate kcs in order to use the above equation to solve for g.
We can use CAPM to estimate kcs: kcs = Rrf + (βcs × Market risk premium)
From the problem statement we know that Rrf = 3.72 percent and that the market risk premium equals 5.17 percent. Therefore, we only need a value for βcs in order to calculate kcs. Since The S&P 500 represents more than 70 percent of the market capitalization of all stocks traded in the U.S. (see Chapter 9), we know that the beta for the S&P will be relatively close to 1. Using this value in CAPM we estimate that kcs equals: kcs = 0.0372 + (1 × 0.0517) = 0.0889. We can now write the formula for the P/E as follows: SM 19-813
P0 (1 g) 0.376 =18.66 = E0 0.0889 g
Solving for g yields g = 0.0674, or 6.74 percent. The S&P 500 index value at the end of 2014 implied that the market expected the dividends paid by firms in the S&P 500 to increase at an average annual rate of 6.74 percent forever. LO 3 Bloomcode: Application AASCB: Analytic IMA: Corporate Finance AICPA: Industry/Sector Perspective
18.30 The S&P 500 P/E multiple of 18.66 at the end of 2014 was higher than its historical average of approximately 15. Some financial commentators argued that this meant that the firms in the S&P 500 were, on average, overvalued at the end of 2014. Based on your analysis in problem 18.29 and the concepts covered in this book, do you think that these commentators are right or wrong? Why or why not?
Solution: There is no way to know for sure whether the commentators are right or wrong. If the market is semi-strong form efficient, as this term is defined in Chapter 2, then all public information should be reflected in the prices of S&P 500 shares. This implies that there is no way for an outsider to know for sure whether the firms in the S&P 500 were, on average, over- or undervalued.
However, we can make some observations. First, with regards to the analysis in problem 18.29, if the constant growth perpetuity model (which was used to derive Equation 18.1) is an appropriate model for the dividends paid by S&P 500 firms, then the implied growth rate of 7.9 percent suggests that the market might be over-valuing the shares of S&P 500 firms. Recall from Chapter 9 that g cannot be greater than the sum of the expected real
SM 19-814
growth rate in the economy and the expected rate of inflation. This sum has historically been less than 7 percent. If g turns out to be less that 7 percent in the future, then the value of the S&P 500 index will declinemeaning that the firms in the S&P 500 would have been overvalued on average at the end of 2014.
On the other hand, if the mixed (supernormal) dividend growth model discussed in Chapter 9 is a more appropriate model for dividend growth at S&P 500 firms, then it is possible that the firms in the S&P 500 were not, on average, overvalued at the end of 2014. For example, since the U.S. economy was just emerging from a severe recession in 2010 it might have been perfectly reasonable to expect relatively large increases in 2011 and 2012 earnings as the economy recovered. As can be seen from the following table, which shows estimated earnings for S&P 500 firms for the 2004 to 2010 period, the 2010 earnings estimate was still considerably below the 2007 earnings.
S&P 500 Year
Earnings
2010
67.46
2009
12.65
2008
66.60
2007
88.70
2006
78.16
2005
67.86
2004
58.87
A multiple of 18.66 would be justified if the earnings of S&P 500 firms were expected to return to the 2007 level in two years (e.g in 2012) and then grow at 4.2 percent per year afterwards. LO 3 Bloomcode: Analysis AASCB: Analytic IMA: Corporate Finance AICPA: Industry/Sector Perspective SM 19-815
18.31 You own a company that produces and distributes course packets for classes at local universities via the Internet. You have asked a friend to invest $35,000 in the business. Your friend wants to know what the business is worth so that he can determine how much of the equity (e.g., what percentage) he should expect to receive for his investment. You offer to help him value the business.
The business is expected to generate revenue of $110,000 and incur cash operating expenses of $70,000 next year. Over the following three years, revenue and cash operating expenses are expected to increase 15 percent, 10 percent, and 7 percent. After year 4 they are expected to grow 2 percent per year forever. Depreciation and amortization, capital expenditures, and additions to working capital are expected to equal 5 percent, 6 percent, and 1 percent of revenue, respectively, in the future. You have determined that a target capital structure of 10 percent is reasonable for this business. With this capital structure, the pretax cost of debt will be 6 percent and the beta for the equity will be 1.30. The average tax rate for the business is 10 percent, and the marginal rate is 20 percent. The risk-free rate is 4.25 percent and the market risk premium is 6.01 percent. What is a 100 percent equity interest in the business worth? What percentage of the equity should your friend get for his investment?
Solution: We can value the equity of this business using the FCFF Income approach. We will first estimate the WACC using Equation 13.7. To do this we calculate kcs using Equation 13.4: kcs = Rrf + (βcs × Market risk premium) = 0.0425 + (1.30)(0.0601) = 0.1206, or 12.06% and substitute into Equation 13.7:
WACC xDebtkDebt pretax (1-t) xpskps xcskcs = (0.10)(0.06)(1-.10) +(0)(0) +(0.9)(0.1206) = 0.1140 or11.40% SM 19-816
Note that we use the average tax rate in this calculation because we are assuming that the company will continue to operate on a stand-alone basis and that its average tax rate will continue to be 10 percent.
We next forecast the FCFF and discount them to the present using the WACC to obtain the enterprise value. There is no mention of NOA in the problem statement and so we assume that there are none. Since we are assuming that the firm will have 10 percent debt, the value of the equity equals 90 percent of the enterprise value.
The spreadsheet below shows the forecasted FCFF for the next five years and the calculation of the equity value.
The trickiest part of the calculations in the spreadsheet is in estimating the additions to working capital. Unlike depreciation and amortization and capital expenditures, we do not simply multiply the 1percent value given in the problem statement by revenue in the same year to obtain additions to working capital. The 1percent value is for the total working capital that the business has. Since it already has working capital in the previous year, we multiply the change in revenue in the upcoming year by 1 percent. This tells us how much must be invested in working capital at the beginning of that year in order to have enough working capital to support the forecasted revenue. For example, the additions to working capital for Year 1 (remember that this is also the beginning of Year 2) equals:
$110,000 Year 1 revenue × 15% Year 2 revenue growth × 1% = $165
YEARS Revenues Operating costs Depreciation and amortization
1
2
3
4
5
$110,000 70,000 5,500
$126,500 $80,500 6,325
$139,150 $88,550 6,958
$148,891 $94,749 7,445
$151,868 $96,643 7,593
SM 19-817
Operating profit Taxes (10%)
34,500 3,450
39,675 3,968
43,643 4,364
46,697 4,670
47,631 4,763
NOPAT Depreciation and amortization Capital expenditures Additions to working capital
31,050 5,500 6,600 165
35,708 6,325 7,590 127
39,278 6,958 8,349 97
42,028 7,445 8,933 30
42,868 7,593 9,112 30
FCFF Terminal Value with 2% growth FCFF + TV PV(FCFF) @ 11.40%
$29,785
$34,316
$37,789
$41,319
$29,785 $26,738
$34,316 $27,654
$37,789 $27,337
$40,509 $439,721 $480,230 $311,860
Enterprise Value Less: 10% debt Equity value
$393,588 $39,359 $354,230
The equity value that we have calculated does not take into account any adjustments for control, lack of marketability (remember that it is a private firm), or key people. Since there is no information in the problem statement we do not consider them here. With this caveat, we can calculate the percentage of the equity that your friend should receive as:
$35,000/$354,230 = 0.988, or 9.88% LO 3 Bloomcode: Analysis AASCB: Analytic IMA: Corporate Finance AICPA: Industry/Sector Perspective
Sample Test Problems
18.1 You plan to start a business to produce and sell custom kitchen cabinets. The targeted price for each order of cabinets is $10,000. You estimate that you will receive orders for cabinets for eight kitchens in each of the first two months, nine kitchens in the third month, and ten kitchens in the fourth month. The cost of the equipment necessary to produce the cabinets is $105,000. You expect the cost of raw materials to be $3,000 per SM 19-818
order. In addition, you expect monthly gross wages and payroll to be $27,000, rent to be $8,000, and other expenses to total $4,000. You also expect advertising costs to be $10,000 in the first month, but to remain constant at $1,000 per month during the following three months. How much will you have to initially invest ensure that you have a cash balance of $10,000 at the beginning of the second month? If you invest this amount, what will be your cash balance at the end of the fourth month?
Solution: Ending cash balance of Month 1 = Beginning cash balance of Month 2 = $10,000 Ending cash balance of Month 1 = Beginning cash balance of Month 1 + Cash receipt – Cash payment $10,000 = Beginning cash balance of Month 1 + $80,000 – $178,000 Beginning cash balance of Month 1 = $108,000
Cash Budget for 4 months:
Beginning cash balance Cash receipts: Cash sales Total cash receipts Total cash available Cash payments: Equipment
Month 1 $108,000
Month 2 $10,000
Month 3 $ 26,000
Month 4 $ 49,000
80,000 $ 80,000 $188,000
80,000 $80,000 $90,000
90,000 $ 90,000 $116,000
100,000 $100,000 $149,000
$105,000
$
$
$
0
0
0 SM 19-819
Raw material cost Gross wages Rent Other expenses Advertising cost Total cash payments Ending cash balance
24,000 27,000 8,000 4,000 10,000 $178,000 $ 10,000
24,000 27,000 8,000 4,000 1,000 $64,000 $26,000
27,000 27,000 8,000 4,000 1,000 $ 67,000 $ 49,000
30,000 27,000 8,000 4,000 1,000 $ 70,000 $ 79,000
We have to initially invest $108,000 to ensure that we have a cash balance of $10,000 at the beginning of the second month. If we invest $108,000, our cash balance at the end of the fourth month will be $79,000. Bloomcode: Application AASCB: Analytic IMA: Corporate Finance AICPA: Industry/Sector Perspective
18.2 Which of the following is/are usually included in an entrepreneur’s business plan? a. Detailed description of the company’s products and services. b. Discussion of the management team, including organizational structure. c. A listing of the types of securities that have been issued and who owns them. d. A market analysis. e. All of the above are typically included in a business plan.
Solution: Option ―e‖ is the correct answer because all of the above are usually included in a business plan. Bloomcode: Knowledge SM 19-820
AASCB: Analytic IMA: Corporate Finance AICPA: Industry/Sector Perspective
18.3 Sessler Corporation is a private company that had EBIT of $186 million and depreciation and amortization of $22 million in the most recent fiscal year. At the end of that year, a similar, public firm has an Enterprise Value/EBITDA multiple of 4.3. What is the implied enterprise value of Sessler?
Solution: We can estimate the enterprise value for the Sessler Corporation as:
Enterprise value EBITDA Companybeing valued Comparable F EBITDA EBITDA = EBIT + DA = $186 million + $22 million = $208 million V =
VF = 4.3 × $208 million VF = $894.40 million Bloomcode: Application AASCB: Analytic IMA: Corporate Finance AICPA: Industry/Sector Perspective
18.4 Winters Inc. management estimates that the company will generate after-tax free cash flows from the firm (FCFF) of $12.5 million, $16.8 million and $19.7 million, respectively, over the next three years. After that, FCFF are expected to grow at a constant five percent per year forever. The company has $5 million in non-operational assets. If the appropriate WACC is 8 percent, what is the enterprise value of this business?
Solution: We can calculate the enterprise value of Winters Inc. using following equation: VF = PV(FCFT) + PV(TVT) + NOA The present value of the free cash flows is: SM 19-821
PV(FCFF3) =
$19.7 million $12.5 million $16.8 million (1 0.08) 2 (1 0.08) 3 $41.62 million 1 0.08
We can then estimate the cash flows for the remainder of the business’s life (the terminal value) using two simple steps: (1) calculating the present value of all cash flows after the final (third) year of the detailed forecast using the formula for a growing perpetuity (2) discounting this value to the present TV3
FCFF3 (1 g) $19.7 million (1 0.05) $689.50 million WACC – g 0.08 – 0.05
PV(TV3 )
TV3 $689.50 million $547.35 million 3 (1 WACC) (1 0.08)3
The total value of Winters Inc.: VF = PV(FCFT) + PV(TVT) + NOA VF = $41.62 million + $547.35 + $5 million VF = $593.97 million Enterprise value of Winters Inc. is $593.97 million. Bloomcode: Application AASCB: Analytic IMA: Corporate Finance AICPA: Industry/Sector Perspective
18.5 Do private companies have audited financial statements prepared in accordance with GAAP?
Solution: There is no requirement that the financial statements of private companies be audited. Some private companies have complete, audited financial statements, whereas others have incomplete financial statements that are not prepared in accordance with the generally accepted accounting principles (GAAP). Bloomcode: Comprehension AASCB: Analytic IMA: Corporate Finance SM 19-822
AICPA: Industry/Sector Perspective
Chapter 19 Financial Planning and Forecasting Before You Go On Questions and Answers Section 19.1 1.
What are the four planning documents on which the financial plan is based? The four important planning documents are: (1) the strategic plan, which describes where the firm is headed and articulates the strategies that will be used to get it there; (2) the investment plan, which identifies the capital assets needed to execute the strategies; (3) the financing plan, which explains how the firm will raise the money to buy the assets; and (4) the cash budget, which determines whether the firm will have sufficient cash to pay its bills. These four planning documents provide the foundation for the firm’s financial plan, which consolidates the documents into a single scheme.
2.
What is the strategic plan? The strategic plan is developed to communicate the firm’s long term goals from a very high conceptual level. It is extremely important as the strategy of the firm ultimately drives all other decisions at the firm.
3.
How are the investment decision and financing decision related? When a firm makes a decision to invest in some asset, it must also identify a source of funding to pay for it. Thus, these two decisions are intertwined and must be made together.
Section 19.2 1.
Why is the sales forecast the key component of a financial model? Sales forecast is the key component in a financial model as the majority of data used are calculated as a percentage of sales. Hence, it is important to get as accurate a sales forecast as possible in order to build a meaningful model.
SM 19-823
2.
What are pro forma financial statements, and why are they an important part of the financial planning process? Pro forma financial statements are hypothetical statements that are projected into the future. They are then evaluated to determine which investment and under what conditions will be the most beneficial for the firm.
3.
What is the plug factor in a financial model? The plug factor in a financial model is the source of external financing needed to bring the balance sheet into balance.
Section 19.3 1.
How are historical financial data used to determine the forecasted values of balance sheet accounts? To forecast balance sheet values, the financial manager prepares a table that shows several years of historical accounting data as a percentage of sales. Then some simple trend lines are fitted to the data to see if there are any trends. This allows the forecaster to decide which financial accounts can safely be estimated as a percent of sales and which must be forecasted using other information.
2.
Why might you expect accounts receivable to vary with sales? Accounts receivable can be expected to vary with sales since higher sales mean more money coming in, and hence increased level of receivables.
Section 19.4 1.
Why is it that some working capital accounts may not vary proportionately with sales? Working capital takes into account current assets and current liabilities, and in many instances inventories and cash balances do not proportionately change with sales. The working capital ratio often increases at a decreasing rate as sales increase.
2.
What are lumpy assets, and how do these assets vary with sales? In most instances, fixed assets do not vary directly with sales, but are rather periodically added in large increments or lumps, hence the name ―lumpy assets‖.
SM 19-824
Section 19.5 1.
What two factors determine the amount of EFN? The amount of EFN is determined by the difference between the total amount of new assets the firm needs to finance less the amount of internal financing available.
2.
What is IGR, and why is it of interest to management? IGR stands for internal growth rate and is defined as the maximum growth rate a firm can achieve without external financing. IGR is of management interest, as firms that can generate a high volume of retained earnings and/or use fewer assets can sustain a higher growth rate without raising more capital
3.
If a firm continually exceeds its SGR, what problems may it face in the future? If the firm’s actual growth rate consistently exceeds its sustainable growth, the firm will have a cash shortage problem if it is unwilling to change its targeted capital structure or sell equity. If the firm has a high credit rating and its use of leverage is not excessive, the firm may have no problem going to the market and securing additional funds.
Self-Study Problems 19.1
The Starlight, Inc. financial statements for the fiscal year ended June 30, 2013, are presented below. The firm’s sales are projected to grow at a rate of 20 percent next year, and all financial statement accounts will vary directly with sales. Based on that projection, develop a pro forma balance sheet and income statement for the fiscal year ending June 30, 2014. Starlight, Inc., Balance Sheet as of June 30, 2013
Assets: Cash Accounts receivables Inventories Total current assets Net fixed assets Other assets
$ 25,135 43,758 167,112 $236,005 325,422 13,125
Liabilities and Stockholders’ Equity: Accounts payables $ 67,855 Notes payables 36,454 Total current liabilities
$104,309
Long-term debt Common stock Retained earnings
223,125 150,000 97,118 SM 19-825
Total assets
$574,552
Total liabilities and equity
$574,552
Starlight, Inc. Income Statement for the Fiscal Year Ended June 30, 2013 Net Sales Costs EBITDA Depreciation EBIT Interest EBT Taxes (35%) Net income
$1,450,000 812,500 $ 637,500 175,000 $ 462,500 89,575 $ 372,925 130,524 $ 242,401
Solution: The pro forma statements for Starlight are as follows: Starlight, Inc. Balance Sheet as of June 30, 2014 Assets: Liabilities and Stockholders’ Equity: Cash $ 30,162 Accounts payables Accounts receivables 52,510 Notes payables Inventories 200,534 Total current assets $283,206 Total current liabilities Net fixed assets Other assets Total assets
390,506 Long-term debt 15,750 Common stock Retained earnings $689,462 Total liabilities and equity
$ 81,426 43,745 $125,171 267,750 180,000 116,542 $689,462
Starlight, Inc. Income Statement for the Fiscal Year Ended June 30, 2014
Net Sales Costs EBITDA Depreciation EBIT Interest EBT Taxes (35%) Net income
$1,740,000 975,000 $ 765,000 210,000 $ 555,000 107,490 $ 447,510 156,629 $ 290,882
SM 19-826
19.2
Use the financial information for Starlight from Problem 16.1. Assume now that equity accounts do not vary directly with sales but change when retained earning change or new equity is issued. The company pays 45 percent of its income as dividends every year. In addition, the company plans to expand production capacity by building a new facility that will cost $225,000. The firm has no plans to issue new equity this year. Any funds that need to be raised will be in the form of long-term debt. Prepare a pro forma balance sheet using this information.
Solution: The pro forma income statement is the same as that shown in the solution to Problem 16.1. We now have to accommodate payment of dividends. Since the company pays 45 percent of its net income as dividend, the amount of retained earnings is calculated as follows: Retained earnings from 2014 income statement = $290,882 × (1 - 0.45) = $159,985 This is the amount by which retained earnings will increase by in 2014, from $97,118 to $257,103. No new equity is added. The increase in assets is financed externally through the sale of long-term debt. The pro forma balance sheet is as follows: Starlight, Inc. Balance Sheet as of June 30, 2014 Assets: Liabilities and Stockholders’ Equity: Cash $ 30,162 Accounts payables $ 81,426 Accounts receivables 52,510 Notes payables 43,745 Inventories 200,534 Total current assets $283,206 Total current liabilities $125,171 Net fixed assets Addition to fixed assets Other assets Total assets
19.3
390,506 Long-term debt 225,000 Common stock 15,750 Retained earnings $914,462 Total liabilities and equity
382,188 150,000 257,103 $914,462
Use the financial statements from Problem 16.1 and the information from Problem 16.2 to calculate the company’s retention(plowback) ratio, external funds needed (EFN), internal growth rate (IGR) and sustainable growth rate (SGR).
. Solution: The retention (plowback) ratio, external funds needed, internal growth rate, and sustainable growth rate are calculated as follows:
SM 19-827
Retention (plowback) ratio
Addition to retained earnings Net income $159, 985 $290,882 0.55, or 55%
= (Growth rate × Initial assets) – Addition to retained earnings = (0.20 × $574,552) ─ $159,985 = ─ $45,075 Thus, without considering the investment of $225,000 for the new facility, the firm will not need any external financing. However, if you add that in, then, EFN
= New investments – Addition to retained earnings = (0.20 × $574,552) + $225,000 ─ $159,985 = $179,925 Additions to retained earnings IGR = Initial assets $159, 985 = $574,552 0.278 or 27.8% SGR = Plowback ratio× ROE Addition to retained earnings Net income = × Net income Total equity = 0.55×0.715 = 0.393 or 39.3%
EFN
19.4
Northwood Corp. has a dividend payout ratio of 60 percent, return on equity of 14.5 percent, total assets of $11,500,450, and equity of $4,652,125. Calculate the firm’s internal rate of growth (IGR).
Solution: We calculate Northwood’s internal growth rate as follows: IGR = Plowback ratio×ROE×Measure of leverage $4, 652,125 = 0.40×0.145× $11,500, 450 = 0.235 or 2.35%
SM 19-828
19.5
Renewal Company has net income of $1.25 million and a dividend payout ratio of 35 percent. It currently has equity of $2,875,223. What is the firm’s sustainable growth rate?
Solution: Renewal’s sustainable growth rate is: SGR = Plowback ratio× ROE $1, 250, 000 = 0.65× $2,875, 223 = 0.283 or 28.3%
Discussion Questions 19.1
What is financial planning? What four types of plans/budgets are involved in financial planning? Financial planning is the process by which a company’s management identifies the best investment opportunities from a strategic perspective and determines how to finance these investments. The end result is referred to as a financial plan. The four plans in financial planning are the strategic plan, the investment plan or capital budget, the financing plan, and the cash budget.
LO: 1 Level: Basic Bloomcode: Knowledge AASCB: Analytic IMA: Corporate Finance AICPA: Industry/Sector Perspective
19.2
Why is the capital budget an important part of a firm’s financial planning? The investment plan, also known as the capital budget, lays out the firm’s proposed spending on capital assets for the year. The capital budget describes in detail all the firm’s planned spending on productive assets like plant and equipment, a new production line, etc.
LO: 1 Level: Basic Bloomcode: Comprehension AASCB: Analytic IMA: Investment Decisions SM 19-829
AICPA: Industry/Sector Perspective
19.3
Why do financing and investment decisions have to be made concurrently? Any investment decision has to be linked to a source of funds to pay for the investment or any capital expenditures, and hence the investment decision and the financing decision have to be made concurrently.
LO: 1 Level: Basic Bloomcode: Comprehension AASCB: Analytic IMA: Investment Decisions AICPA: Industry/Sector Perspective
19.4
Explain how sales can be used to develop pro forma financial statements. For most financial planning models, the principal input variable is a forecast of the firm’s sales or sales growth rate. The forecasted sales can be the basis for a financial planning model because so many items on the income statement and balance sheet vary with changes in the level of sales.. Sales forecasts are given for some time period, such as a quarter or a year, and are often expressed as percent change in sales: % S = (St+1 –St)/ St
LO: 2 Level: Basic Bloomcode: Comprehension AASCB: Analytic IMA: Budget Preparation AICPA: Industry/Sector Perspective
19.5
Why is sales not always a good measure to use in forecasting fixed assets? When a percent of sales method is used to prepare pro forma financial statements, the issue of whether or not a firm is operating at full capacity is ignored. Only if the firm is
SM 19-830
operating at full capacity does fixed assets vary directly with sales. If a firm has excess capacity, then sales can increase without a need to invest in additional fixed assets. . LO: 3 Level: Basic Bloomcode: Comprehension AASCB: Analytic IMA: Budget Preparation AICPA: Industry/Sector Perspective
19.6
List all the accounts that can be affected by the ―plug‖ value. How does this value help managers? The plug value relates to the amount of external funds needed based on a given level of sales. Thus, all the various sources of funds for a firm like short-term and long-term debt, common equity, and retained earnings are affected by the plug value.
LO: 2 Level: Basic Bloomcode: Knowledge AASCB: Analytic IMA: Investment Decisions AICPA: Industry/Sector Perspective
19.7
Explain why the fixed asset account may or may not vary with sales. Generally, companies invest in fixed assets in large discrete amounts rather than in small, continuous amounts. Thus, each investment in fixed assets can accommodate a further growth in sales without calling for additional investment in plant and equipment. Only after each level of sales is accomplished at full production capacity would management have to consider another increase in fixed assets.
LO: 3 Level: Basic Bloomcode: Comprehension AASCB: Analytic IMA: Investment Decisions AICPA: Industry/Sector Perspective
SM 19-831
19.8
How does the dividend payout ratio affect the amount of funds needed to finance growth? Any change in the dividend payout ratio directly affects the amount of funds retained by the company. The higher the firm’s ratio for a given level of net income, the lower the amount of retained earnings, and hence the larger will be the amount of external debt financing needed by the firm by holding the growth rate constant.
LO: 5 Level: Basic Bloomcode: Comprehension AASCB: Analytic IMA: FSA AICPA: Industry/Sector Perspective
19.9
Define internal growth rate (IGR). Identify the characteristics of a high-growth firm that has no external funds needed. Internal growth rate (IGR) is the maximum growth rate a firm can sustain without needing any external financing. Such firms have a high enough retention ratio to fund growth, generate a high level of net income for every dollar of equity than other firms, and use low amounts of leverage or debt financing.
LO: 5 Level: Basic Bloomcode: Knowledge AASCB: Analytic IMA: FSA AICPA: Industry/Sector Perspective
19.10 What is the sustainable growth rate? Why is it important? Sustainable growth rate (SGR) is the maximum growth rate a firm can sustain without needing any external equity financing, holding leverage constant. Holding the firm to a sustainable growth rate allows the firm to avoid costly equity issues, thus reducing the dilution of earnings. Whenever firms grow faster than the SGR, the firm is going to need external financing and may have to resort to debt financing, which increases the company’s risk. LO: 5 SM 19-832
Level: Basic Bloomcode: Comprehension AASCB: Analytic IMA: FSA AICPA: Industry/Sector Perspective
Questions and Problems BASIC 19.1
Strategic plan: Explain the importance of the strategic plan.
Solution: The strategic plan is the blueprint for a company’s management that determines the firm’s long-term goals, identifies the strategies to achieve these goals, and determines the company’s ability to be competitive. It drives all decision making within the firm and covers all areas of a firm’s operations. LO 1 Bloomcode: Comprehension AASCB: Analytic IMA: Strategic Planning AICPA: Industry/Sector Perspective
19.2
Capital budget: What are the various steps in preparing a capital budget?
Solution: First, management identifies all strategically acceptable investment opportunities and ranks them based on their contribution to shareholder wealth. Next, the senior management reviews the capital budget plan and may revise it, if necessary, based on need and the amount they can finance. LO 1 Bloomcode: Knowledge AASCB: Analytic IMA: Investment Decisions AICPA: Industry/Sector Perspective
SM 19-833
19.3
Financing plan: What are the elements of a financing plan?
Solution: First, the financing plan identifies the amount of external financing needed and where the funds could be obtained from. Second, the plan identifies the firm’s target capital structure. Last, the plan identifies the firm’s dividend policy, which directly affects the amount of funds available for new investment projects, the more funds the firm payout as cash dividends, the more the external funding needed for its investment. LO 1 Bloomcode: Comprehension AASCB: Analytic IMA: Corporate Finance AICPA: Industry/Sector Perspective
19.4
Financial planning: Identify the steps in the financial planning process.
Solution: o o o o o
Establish a set of financial goals. Identify the investment opportunities and financing alternatives. Employ a financial planning model. Evaluate the investment and financing choices and decide on the ones that are the best fit for the firm. Prepare contingency plans for future use.
LO 1 Bloomcode: Knowledge AASCB: Analytic IMA: Corporate Finance AICPA: Industry/Sector Perspective
19.5
Financial modeling: List the various elements of financial modeling.
Solution: The important elements of financial modeling are the sales forecast, economic assumptions, pro forma statements, investment decisions, financing requirements, and the ―plug factor‖ that determines the source of the external funds needed. LO 2 Bloomcode: Comprehension AASCB: Analytic
SM 19-834
IMA: Corporate Finance AICPA: Industry/Sector Perspective
19.6
Payout ratio: Define the retention (plowback) ratio and the dividend payout ratio.
Solution: The dividend payout ratio tells us how much of a firm’s earnings are paid to the shareholders as dividends. The retention ratio is complementary to the payout ratio and tells us how much of the firm’s earnings are retained in the firm. Thus, the retention ratio equals: Retention ratio (1 Dividend payout ratio) LO 3 Bloomcode: Knowledge AASCB: Analytic IMA: FSA AICPA: Industry/Sector Perspective
19.7
Addition to retained earnings: Northwood, Inc., has revenue of $455,316, costs of $316,487, and a tax rate of 31 percent. If the firm pays out 45 percent of its earnings as dividends every year, how much earnings are retained and what is the firm’s retention ratio?
Solution: Revenue = $455,316 Costs = $316,487 Tax rate = 31% Payout ratio = 45% Net income ($455,316 - $316,487) (1 - 0.31) $95,792 Retained earnings = 95,792 × (1 ─ 0.45) = $52,686 Retention ratio (1 - Dividend payout ratio) Retention ratio = (1 0.45) 55% LO 3 Bloomcode: Application AASCB: Analytic IMA: FSA AICPA: Industry/Sector Perspective
SM 19-835
19.8
Payout and retention ratio: Goodwin Corp. has revenues of $12,112,659, costs of $9,080,545, interest payments of $412,375, and a tax rate of 34 percent. It paid dividends of $1,025,000 to its stockholders. What are the firm’s dividend payout ratio and retention ratio?
Solution: Goodwin Corp.
Revenues Costs EBIT Interest EBT Taxes (34%) Net income
$12,112,659 9,080,545 $ 3,032,114 412,375 $ 2,619,739 890,711 $1,729,028
Dividends paid = $1,025,000 Dividend payout ratio Dividends $1, 025, 000 59.28% Net income $1, 729, 028 Retention ratio (1 - Dividend Payout ratio) = (1 – 0.5928) = 40.72% LO 3 Bloomcode: Application AASCB: Analytic IMA: FSA AICPA: Industry/Sector Perspective
19.9
Percent of sales: Cattail Corporation’s financial statements for the fiscal year just ended are shown below: Cattail Corporation Financial Statements for Fiscal Year Just Ended ($ thousands) Income Statement
Net sales $600
Balance Sheet
$1,500
Assets
$700
Debt
SM 19-836
350
Costs 100 Net income $700
$1,150
Equity
Total
$700
Total
Cattail management expects sales to increase by 14 percent next year. Assume that the financial statement accounts vary directly with changes in sales and that management has no financing plan at this time. Given this information, develop a pro forma income statement for Cattail for the next fiscal year.
Solution: Projected sales = $1,500,000 × 1.14 = $1,710,000 Projected costs = $350,000 × 1.14 = $399,000
Cattail Corporation Pro Forma Income Statement ($ thousands) Net sales Costs Net income
$1,710 399 $1,311
LO 3 Bloomcode: Application AASCB: Analytic IMA: FSA AICPA: Industry/Sector Perspective
SM 19-837
19.10 Percent of sales: Given the data for Cattail Corporation in Problem 19.9, if you assume that all balance sheet items also vary with the change in sales, develop a pro forma balance sheet for Cattail for the next fiscal year. Assuming that the firm did not sell or repurchase stock, what is the cash dividend implied by the pro forma income statement and balance sheet?
Solution: Projected assets = $700,000 × 1.14 = $798,000 Projected debt = $600,000 × 1.14 = $684,000 Projected equity = $100,000 × 1.14 = $114,000
Cattail Corporation Pro Forma Income Statement ($ thousands) Balance Sheet Assets
$798
Debt Equity
Total
$798
Total
$684 $114 $798
Pro forma net income is $1,311,000 and the change in the equity account is $14,000, therefore the difference ($1,311,000 - $14,000 = $1,297,000) is the amount of the cash dividends that must have been paid. LO 3 Bloomcode: Application AASCB: Analytic IMA: FSA AICPA: Industry/Sector Perspective
SM 19-838
19.11 Capital intensity ratio: Define capital intensity ratio, and explain its significance. Solution: The capital intensity ratio measures the amount of assets needed to generate one dollar in sales. The higher the ratio, the more assets are needed to generate every dollar of sales, and hence, the more capital intense is the firm and vice versa. LO 3 Bloomcode: Knowledge AASCB: Analytic IMA: FSA AICPA: Industry/Sector Perspective
19.12 Capital intensity ratio: Tantrix Confectioners has total assets of $3,257,845 and net sales of $5,123,951. What is the firm’s capital intensity ratio? Solution: Total assets = $3,257,845 Sales = $5,123,951 Total assets Capital intensity ratio Net sales $3,257,845 63.6% $5,123,951 LO 3 Bloomcode: Application AASCB: Analytic IMA: FSA AICPA: Industry/Sector Perspective
19.13 Capital intensity ratio: McDonald Metal Works has been able to generate net sales of $13,445,196 on assets of $9,145,633. What is the firm’s capital intensity ratio? Solution: Sales = $13,445,196 Total assets = $9,145,633 Total assets Net sales $9,145, 633 0.6802 or 68.02% $13, 445,196
Capital intensity ratio
SM 19-839
LO 3 Bloomcode: Application AASCB: Analytic IMA: FSA AICPA: Industry/Sector Perspective
19.14 Capital intensity ratio: For McDonald Metal Works in Problem 19.13, how much must net sales grow if the capital intensity ratio has to drop to 60 percent? State your answer as both a percent of sales and a dollar sales increase. Solution: Target capital intensity ratio = 60% Total assets Capital intensity ratio Net sales $9,145,633 0.60 Net sales $9,145,633 Net sales $15,242,721.67 0.60 Growth in net sales needed = New sales – Old sales = $15,242,722 – $13,445,196 = $1,797,526 Percentage change in net sales needed = $1,797,526 / $13,445,196 = 13.37% LO 3 Bloomcode: Application AASCB: Analytic IMA: FSA AICPA: Industry/Sector Perspective
19.15 Internal growth rate: Swan Supply Company has net income of $1,212,335, assets of $12,522,788 and retains 70 percent of its income every year. What is the company’s internal growth rate? Solution: Net income = $1,212,335 Total assets = $12,522,788 Plowback ratio = 70%
SM 19-840
Internal growth rate (IGR) Plowback ratio ROE Measure of Leverage Net income Plowback ratio Equity Total assets Equity Net income Plowback ratio Total assets $1,212,335 0.70 $12,522,788 6.8% IGR = Addition to retained earnings/Initial total assets Addition to retained earnings= 70% × $1,212,335=$ 848,634.5 So, IGR = $ 848,634.5/$12,522,788 = 6.8% LO 5 Bloomcode: Application AASCB: Analytic IMA: FSA AICPA: Industry/Sector Perspective
19.16 Sustainable growth rate: If Newell Corp. has a ROE of 13.7 percent and a dividend payout ratio of 32 percent, what is its sustainable growth rate?
Solution: ROE = 13.7% Payout ratio = 32% Plowback ratio = (1 Payout ratio) = (1 − 0.32) = 68% SGR Plowback ratio ROE 0.68 0.137 9.3% LO 5 Bloomcode: Application AASCB: Analytic IMA: FSA AICPA: Industry/Sector Perspective
19.17 EFN and growth: Refer to Exhibits 19.10 and 19.11 in the text. The EFN for several growth rates for Empire Enterprises are as follows: SM 19-841
Growth _Rate_ (%) 0% 5 9.6 10 15 20
EFN_ ($ millions)_ -$4.8 -2.3 0.0 0.2 2.7 5.2
Check the calculations and plot the line to replicate the graph in Exhibit 19.11. Solution: Exhibit 19.11 gives you the plot. LO 5 Bloomcode: Application AASCB: Analytic IMA: FSA AICPA: Industry/Sector Perspective
INTERMEDIATE 19.18 Retention ratio: Refer to Problem 19.7. Northwood expects to increase its sales by 15 percent next year. All costs vary directly with sales. If Northwood wants to retain $65,000 of earnings next year, will it have to change its dividend payout ratio? If so, what will be the new dividend payout and retention ratios for the firm? Solution: Expected increase in sales = 15% All costs vary directly with sales Revenues next year $455,316(1.15) $523,613 Costs $316, 487 (1.15) $363, 960 EBT $159, 653 Net income $159, 653 (1- 0.31) $110,161 Targeted retained earnings = $65,000 Retention ratio = $65,000/$110,161 = 59.0% Dividend payout ratio = (1 - 0.59) = 41% So, the dividend payout ratio declines from 45 percent to 41 percent. LO 3 Bloomcode: Application SM 19-842
AASCB: Analytic IMA: FSA AICPA: Industry/Sector Perspective
19.19 Capital intensity: Identify two industries (other than airlines) that are capital intensive. Using online or other data sources, compute the capital intensity ratio for the largest firm in each of the chosen industries. Solution: Two industries that may be considered to be capital intensive are the electric utilities industry and the aluminum processing industry. LO 3 Bloomcode: Application AASCB: Analytic IMA: FSA AICPA: Industry/Sector Perspective
19.20 Percent of sales: Tomey Supply Company’s financial statements for the most recent fiscal year are shown below. The company management projects that sales will increase by 20 percent next year. Assume that all costs and assets increase directly with sales. The company has a constant 33 percent dividend payout ratio and has no plans to issue new equity. Any financing needed will be raised through the sale of long-term debt. Prepare pro forma financial statements for the coming year based on this information, and calculate the EFN for Tomey. Tomey Supply Company Income Statement and Balance Sheet Income Statement Balance Sheet Net sales $1,768,121 Assets: Costs 1,116,487 Current assets $280,754 EBT $651,634 Net fixed assets 713,655 Taxes (35%) 228,072 Total assets $994,409 Net income $ 423,562 Liabilities and Equity: Current liabilities $167,326 Long-term debt 319,456 Common stock 200,000 Retained earnings 307,627 Total liabilities and equity $994,409
SM 19-843
Solution: Tomey Supply Company Net sales Costs EBT Taxes (35%) Net income Dividends paid Addition to retained earnings
Sales Change $1,768,121 1.2 1,116,487 1.2 $ 651,634 228,072 $ 423,562
Pro Forma $2,121,745 1,339,784 $ 781,961 273,686 $ 508,275 167,731 $340,544
Assets: Current assets Net fixed assets Total assets
$ 280,754 713,655 $ 994,409
1.2 1.2
$ 336,905 856,386 $1,193,291
Liabilities and Equity: Current liabilities Long-term debt Common stock Retained earnings Total liabilities & equity
$ 167,326 319,456 200,000 307,627 $ 994,409
1.2
$ 200,791 144,329 200,000 648,171 $1,193,291
EFN (Initial assets Growth rate) Addition to retained earnings = ($994, 409 0.20) - $340,544 $141, 662 Addition to current liabilities = $33,465 So, actual EFN = -$141,662 - $33,465 = -$175,127 LO 3 Bloomcode: Application AASCB: Analytic IMA: Budget Preparation AICPA: Industry/Sector Perspective
19.21 Internal growth rate: Using the pro forma financial statements for Tomey Supply Company developed in Problem 19.20, find the internal growth rate for Tomey. Solution:
SM 19-844
IGR Addition to retained earnings / Initial assets = $340, 543 / $994, 409 0.3425 or 34.25% LO 5 Bloomcode: Application AASCB: Analytic IMA: FSA AICPA: Industry/Sector Perspective
19.22 Sustainable growth rate: Use the following pro forma information for Tomey Supply Company for next year: net income = $508,275; addition to retained earnings = $340,544; common equity = $848,171; net sales = $2,121,745. Assume that management does not want the ratio of long-term debt to equity to exceed the current long-term debtto-equity ratio of 63 percent and also does not want to issue new equity. What level of sales growth can Tomey Supply Company sustain? Calculate the new sales level. Solution: Current debt to equity ratio = 63% Sustainable growth rate (SGR) = Plowback ratio ROE Addition to retained earnings Net income SGR = × Net income Total equity $340, 544 $508, 275 = × $508, 275 $848,171 40.2% New sales level Old sales (1 SGR) $2,121,745 (1.402) $2, 974, 686 LO 5 Bloomcode: Application AASCB: Analytic IMA: FSA AICPA: Industry/Sector Perspective 19.23 Sustainable growth rate: Rowan Company has a net profit margin of 8.3 percent, debt ratio of 45 percent, total assets of $4,157,550, and sales of $6,852,654. If the company has a dividend payout ratio of 67 percent, what is its sustainable growth rate? Solution: Net profit margin = 8.3% SM 19-845
Debt ratio = 45% Total assets = $4,157,550 Sales = $6,852,654 Dividend payout ratio = 67% Plowback ratio = (1 - 0.67) = 33% Net income Net profit margin = Sales Net income = Sales× Net profit margin = $6,852,654 × 0.083 = $568,770 Measure of Leverage = 1- Debt ratio = (1- 0.45) = 55% Total equity = 0.55$4,157,550 = $2, 286, 653 Net income ×Plowback ratio Total equity $568, 770 = × 0.33 $2, 286, 653 = 8.2%
SGR =
LO 5 Bloomcode: Application AASCB: Analytic IMA: FSA AICPA: Industry/Sector Perspective
19.24 Sustainable growth rate: Refer to the information for Rowan Company in Problem 19.23. The firm’s management desires a sustainable growth rate (SGR) of 10 percent but does not wish to change the company’s level of debt or its payout ratio. What will the firm’s new profit margin have to be in order to achieve the desired growth rate? Solution: Target SGR = 10% Plowback ratio = 33% Debt ratio = 45% Equity multiplier = EM =
1 Equity
1
=
= 1.82
(1- 0.45)
Total assets turnover ratio = TATO =
$6,852,654 Sales = == 1.65 Times Total assets $4,157,550 SM 19-846
ROE =
SGR 0.10 = = 30.3% Plowback ratio 0.33 ROE = PM ×TATO×EM
Net profit margin = PM =
ROE 0.303 = 10.1% = TATO×EM (1.65×1.82)
LO 5 Bloomcode: Application AASCB: Analytic IMA: FSA AICPA: Industry/Sector Perspective
19.25 Sustainable growth rate: Rocky Sales, Inc., has current sales of $1,215,326 and net income of $211,253. It also has a debt ratio of 25 percent and a dividend payout ratio of 75 percent. The company’s total assets are $712,455. What is its sustainable growth rate? Solution: Sales = $1,215,326 Net income = $211,253 Debt ratio = 25% Dividend payout ratio = 75% Total assets = $712,455 Plowback ratio (1- 0.75) 25% Equity ratio or Measure of leverage (1- 0.25) 75% Total equity = 0.75 Total assets = 0.75 $712, 455 = $534, 341 Net income $211, 253 39.5% Total equity $534, 341 SGR Plowback ratio ROE ROE
0.25 0.395 9.9% LO 5 Bloomcode: Application AASCB: Analytic IMA: FSA AICPA: Industry/Sector Perspective
SM 19-847
19.26 Sustainable growth rate: Ellicott Textile Mills management has reported the following financial information for the year ended September 30, 2014. The company generated a net income of $915, 366 on a net profit margin of 6.4 percent. It has a dividend payout ratio of 50 percent, a capital intensity ratio of 62 percent, and a debt ratio of 45 percent. What is the company’s sustainable growth rate? Solution: Net income = $915,366 Net profit margin = 6.4% Dividend payout ratio = 50% Capital intensity ratio = 62% Debt ratio = 45% Net income $915,366 Sales $14,302,594 0.064 Profit margin Total assets Capital intensity ratio = Sales Total Assets = Capital intensity ratio×sales = 0.62 ×$14,302,594 = $8,867,608 Equity = (1- Debt ratio) ×Total assets (1- 0.45) ×$8,867,608 $4, 877,184 Net income $915,366 18.8% ROE Total equity $4,877,184 SGR Plowback ratio ROE 0.50 0.188 9.4% LO 3, LO 5 Bloomcode: Application AASCB: Analytic IMA: FSA AICPA: Industry/Sector Perspective
19.27 Internal growth rate: Given the information in Problem 19.26, what is the internal growth rate of Ellicott Textile Mills? Solution:
SM 19-848
IGR Plowback ratio ROA 0.50
$915,366 5.2% $8,867,608
LO 5 Bloomcode: Application AASCB: Analytic IMA: FSA AICPA: Industry/Sector Perspective
19.28 Internal growth rate: Fantasy Travel Company has a return on equity of 17.5 percent, a total equity/total assets ratio of 65 percent, and a dividend payout ratio of 75 percent. What is the company’s internal growth rate? Solution: ROE = 17.5% Equity ratio = 65% Dividend payout ratio = 75% Plowback ratio = (1 - 0.75) = 25% IGR Plowback ratio ROE Measure of Leverage 0.25 0.175 0.65 2.8% LO 5 Bloomcode: Application AASCB: Analytic IMA: FSA AICPA: Industry/Sector Perspective
19.29 EFN: Maryland Micro Brewers generated revenues of $12,125,800 with a 72 percent capital intensity ratio during the year ended September 30, 2014. Its net income was $873,058. With the introduction of a half dozen new specialty beers, management expects to grow sales by 15 percent next year. Assume that all costs vary directly with sales and that the firm maintains a dividend payout ratio of 70 percent. What will be the EFN needed by this firm? If the company wants to raise no more than $750,000 externally and is not averse to adjusting its payout policy, what will be the new dividend payout ratio? Solution: Total sales = $12,125,800 Capital intensity ratio = 72% SM 19-849
Total assets = Capacity intensity ratio×Sales = 0.72×$12,125,800 = $8, 730,576 Net income = $873,058 Projected net income =$873,058 x 1.15 =$1,004,017 Dividend payout ratio = 70% Addition to retained earnings = $1,004,017(1-0.7) = $301,205
EFN = (Growth rate×Total assets) - Addition to retained earnings = (0.15×$8, 730, 576) $301, 205 = $1, 008, 381 Target EFN = $750,000 Addition to retained earnings (Growth rate Total assets)- EFN (0.15 $8,730,576) $750,000 $559,586
Plowback ratio =
$559, 586 = 55.73% $1,004,017
Dividend payout ratio = (1- Plowback ratio) = (1- 0.5573) = 44.27% LO 5 Bloomcode: Application AASCB: Analytic IMA: FSA AICPA: Industry/Sector Perspective
19.30 EFN: Ritchie Marble Company has total assets of $12,899,450, sales of $18,174,652, and net income of $4,589,774. Management expects sales to grow by 25 percent next year. All assets and costs (including taxes) vary directly with sales, and management expects to maintain a payout ratio of 65 percent. Calculate Ritchie’s EFN. Solution: Total assets = $12,899,450 Sales = $18,174,652 Net income = $4,589,774 SM 19-850
Expected growth rate = 25% Projected net income = $4,589,774 (1.25) = $5,737,217.50 Payout ratio = 65% Addition to retained earnings = Net income ×(1- Payout ratio) = $5,737,217.50 × 0.35 = $2,008,026.13 EFN = (Growth rate×Total assets) - Addition to retained earnings = (0.25×$12,899,450) - $2,008,026 $1, 216, 836 LO 5 Bloomcode: Application AASCB: Analytic IMA: FSA AICPA: Industry/Sector Perspective
19.31 EFN: Norton Group, Inc., expects to add $1,213,777 to retained earnings and currently has total assets of $23,159,852. If the company has the ability to borrow up to $1 million, how much growth can the firm support if it is willing to borrow to its maximum capacity? Solution: Addition to retained earnings = $1,213,777 Total Assets = $23,159,852 EFN = $1,000,000 EFN (Growth rate Total assets)- Addition to Retained earnings (EFN Addition to Retained earnings) Growth rate Total assets EFN Addition to retained earnings Total assets $1, 000, 000 $1, 213, 777 9.6% $23,159,852
Growth rate
LO 5 Bloomcode: Application AASCB: Analytic IMA: FSA AICPA: Industry/Sector Perspective
SM 19-851
19.32 EFN: Capstone Marketing Group has total assets of $5,568,000, sales of $3,008,725, and net income of $822,000. The company expects its sales to grow by 12 percent next year. All assets and costs (including taxes) vary directly with sales, and the firm expects to maintain a payout ratio of 55 percent. Calculate Capstone’s EFN.
Solution: With a growth rate in sales of 12 percent, if all costs increase at the same rate, then pro forma net income will be: $822,000 × 1.12 = $920,640. Addition to retained earnings = (1 ─ 0.55) × $920,640 = $414,288 EFN = (0.12 × $5,568,000) – $414,288 = $253,872 LO 5 Bloomcode: Application AASCB: Analytic IMA: FSA AICPA: Industry/Sector Perspective
19.33 Maximum sales growth: Given the data for Capstone Marketing Group in Problem 19.32, what would Capstone’s payout ratio have to be for the firm’s EFN to be zero?
Solution: EFN will be equal to zero if the change in assets is equal to the change in retained earnings. Projected net income = $822,000 × 1.12 = $920,640 Projected change in assets = 0.12 × $5,568,000 = $668,160 $668,160 = $920,640 × (1 – payout ratio) Payout ratio = 0.274, or 27.4% SM 19-852
LO 5 Bloomcode: Application AASCB: Analytic IMA: FSA AICPA: Industry/Sector Perspective
19.34 Maximum sales growth: Rockville Consulting Group expects to add $271,898 to retained earnings this year. The company has total assets of $3,425,693 and wishes to add no new external funds for the coming year. If assets and costs vary directly with sales, how much sales growth can the company support while retaining an EFN of zero? What is the firm’s internal growth rate? Solution: Addition to retained earnings = $271,898 Total assets = $3,425,693 EFN = 0 (EFN + Addition to retained earnings) Maximum growth rate = Total assets (0 + $271,898) = = 7.9% $3, 425, 693) Addition to retained earnings $271,898 = 7.9% = Initial assets $3, 425, 693 With no external financing, the maximum possible growth equals the internal growth rate. IGR =
LO 5 Bloomcode: Application AASCB: Analytic IMA: FSA AICPA: Industry/Sector Perspective
ADVANCED 19.35 The financial statements for the year ended June 30, 2014, are given below for Morgan Construction Company. The firm’s sales are projected to grow at a rate of 25 percent next year, and all financial statement accounts will vary directly with sales. Based on that SM 19-853
projection, develop a pro forma balance sheet and an income statement for the 2015 fiscal year. Morgan Construction Company Balance Sheet as of June 30, 2014 Assets: Liabilities and Stockholders’ Equity: Cash $ 3,349,239 Accounts payable $ 9,041,679 Accounts receivable 5,830,754 Notes payable 4,857,496 Inventories 22,267,674 Total current assets $31,447,667 Total current liabilities $13,899,175 Net fixed assets Other assets Total assets
43,362,482 Long-term debt 1,748,906 Common stock Retained earnings $76,559,055 Total liabilities and equity
29,731,406 19,987,500 12,940,974 $76,559,055
Morgan Construction Company Income Statement for the Fiscal Year Ended June 30, 2014 Net sales $193,212,500 Costs 145,265,625 EBITDA $ 47,946,875 Depreciation 23,318,750 EBIT $ 24,628,125 Interest 11,935,869 EBT $ 12,692,256 Taxes (35%) 4,442,290 Net income $ 8,249,966
SM 19-854
Solution:
Cash Accounts receivable Inventories Total current assets
Morgan Construction Company—Pro Forma Balance Sheet for Year Ended June 30, 2015 2014 2015 2014 $ 3,349,239 1.25 $ 4,186,549 Accounts payable $ 9,041,679 5,830,754 1.25 7,288,443 Notes payable 4,857,496 22,267,674 1.25 27,834,593 $31,447,667 $39,309,584 Total current liabilities $13,899,175
Net fixed assets Other assets
43,362,482 1,748,906
Total assets
$76,559,055
1.25 1.25
54,203,103 2,186,133 $95,698,819
Long-term debt Common stock Retained earnings Total liabilities & equity
Pro Forma Income Statement for Year Ended June 30, 2015 2014 Revenues $193,212,500 1.25 Costs 145,265,625 1.25 EBITDA $ 47,946,875 Depreciation 23,318,750 1.25 EBIT $ 24,628,125 Interest 11,935,869 1.25 EBT $ 12,692,256 Taxes (35%) 4,442,290 1.25 Net income $ 8,249,966 LO 3 Bloomcode: Application AASCB: Analytic IMA: FSA AICPA: Industry/Sector Perspective
Copyright © 2015 John Wiley & Sons, Inc.
29,731,406 19,987,500 12,940,974 $76,559,055
1.25 1.25
2015 $11,302,099 6,071,870 $17,373,969
1.25 1.25 1.25
37,164,258 24,984,375 16,176,218 $95,698,819
2015 $241,515,625 181,582,031 $ 59,933,594 29,148,438 $ 30,785,156 14,919,836 $ 15,865,320 5,552,862 $ 10,312,458
SM 19-855
19.36 Use the financial information for Morgan Construction Company from Problem 19.35. Assume now that equity accounts do not vary directly with sales but change when retained earnings change or new equity is issued. The company pays 75 percent of its income as dividends every year. In addition, the company plans to expand production capacity by expanding the current facility and acquiring additional equipment. This will cost the firm $10 million. The firm has no plans to issue new equity this year. Prepare a pro forma balance sheet using this information. Any funds that need to be raised (in addition to changes in current liabilities) will be in the form of long-term debt. What is the external funding needed in this case? Solution: Retention ratio = 25% Net income in 2015 = $10,312,458 Addition to retained earnings = $2,578,114.50 or $2,578,115
Cash Accounts receivable Inventories Total current assets Net fixed assets Addition to fixed assets Other assets Total assets
Morgan Construction Company—Pro Forma Balance Sheet for Year Ended June 30, 2015 2014 2015 2014 $ 3,349,239 1.25 $ 4,186,549 Accounts payable $ 9,041,679 5,830,754 1.25 7,288,443 Notes payable 4,857,496 22,267,674 1.25 27,834,593 $31,447,667 $ 39,309,584 Total current liabilities $13,899,175 43,362,482
1.25
1,748,906 $76,559,055
1.25
54,203,102 10,000,000 2,186,133 $105,698,819
Long-term debt Common stock Retained earnings Total liabilities & equity
29,731,406 19,987,500 12,940,974 $76,559,055
1.25 1.25
2015 $11,302,099 6,071,870 $ 17,373,969 $52,818,261 19,987,500 15,519,089 $105,698,819
EFN (Growth rateTotal assets)-Addition to Retained earnings (0.25$76, 559, 055) $2, 578,115 $16, 561, 649 Thus, without considering the investment of $10,000,000 for the new facility, the firm will not need any external funding. However, if you add the investment, then, EFN= New investment – Addition to retained earnings (0.25 ×$76,559,055) + $10,000,000 – $2,578,115 = $26,561,649 LO 3, LO 5 Copyright © 2015 John Wiley & Sons, Inc.
SM 19-856
Bloomcode: Application AASCB: Analytic IMA: FSA AICPA: Industry/Sector Perspective
Copyright © 2015 John Wiley & Sons, Inc.
SM 19-857
19.37 Using the information for Morgan Construction Company in the preceding problem, calculate the firm’s internal growth rate and sustainable growth rate. Solution: IGR =
Addition to retained earnings $2,578,115 = 3.37% = Initial assets $76,559, 055
Net Income $ 10,312,458 = = Equity $ 41,160,593 SGR Plowback ratio ROE ROE =
25.05%
0.25 0.2505 6.26% LO 5 Bloomcode: Application AASCB: Analytic IMA: FSA AICPA: Industry/Sector Perspective
19.38 Use the information for Morgan Construction Company from Problems 19.35 and 19.36. Assume that equity accounts do not vary directly with sales, but change when retained earnings change or new equity is issued. The company’s long-term debt-to-equity ratio is approximately 90 percent, and its equity-to-total assets ratio is about 43 percent. The company management wishes to increase its equity-to-total assets ratio to at least 50 percent. Management is willing to reduce the company’s payout ratio, but will retain no more than 40 percent of earnings. The company will raise any additional funds needed, including funds for expansion, by selling new equity. No new long-term debt will be issued. Prepare pro forma statements to reflect this new scenario. a. What is the external funding needed to accommodate the expected growth? b. What is the firm’s internal growth rate? c. What is the firm’s sustainable growth rate? d. How much new equity will the firm have to issue? e. What is the firm’s new equity ratio and debt- to- equity ratio? Solution: a. Retention ratio = 40% EFN = (Growth rate×Total assets) - Addition to retained earnings = (0.25×$76, 559, 055) - (0.40×$10, 312, 458) = $15, 014, 780.55 EFN including expansion = $25,014,781
SM 1-858
b. c. d.
e.
Addition to retained earnings $4,124,983 = 5.39% = Initial assets $76,559, 055 SGR = Plowback ratio×ROE = 0.40× 0.2505 = 10.02% Retained earnings in 2015 = $12,940,974 + $4,124,983 = $17,065,957 Total liabilities and equity = $105,698,819 Current liabilities = $17,373,969 Common equity = $105,698,819 – ($17,373,969+$29,731,406+$17,065,957) = $41,527,487 Common equity in 2014 = $19,987,500 New common equity issued = $41,527,487 - $19,987,500 = $21,539,987 New equity ratio = Equity = ($41, 523, 487 + $17, 069, 957) Total assets $105, 698,818 55.43% Debt - to - equity ratio = Debt = $29, 731, 406 = 50.74% Equity $58,593, 444 IGR =
Cash Accounts receivable Inventories Total current assets
Morgan Construction Company—Pro Forma Balance Sheet for Year Ended June 20, 2015 2014 2015 2014 $ 3,349,239 1.25 $ 4,186,549 Accounts $ 9,041,679 1.25 payable 5,830,754 1.25 7,288,443 Notes 4,857,496 1.25 payable 22,267,674 1.25 27,834,593 $31,447,667 $ 39,309,584 $13,899,175 Total current liabilities
Net fixed assets Addition to fixed assets Other assets
43,362,482
Total assets
$76,559,055
1.25
54,203,103 10,000,000
1,748,906
1.25
2,186,133 $105,698,819
Long-term debt Common stock Retained earnings Total liabilities & equity
29,731,406 19,987,500 12,940,974 $76,559,055
LO 5 Bloomcode: Application AASCB: Analytic IMA: Budget Preparation AICPA: Industry/Sector Perspective
SM 1-859
19.39 Munson Communications Company has just reported earnings for the year ended June 30, 2014. Below are the firm’s income statement and balance sheet. The company had a 55 percent dividend payout ratio for the last 10 years and management does not plan to change this policy. Based on internal forecasts, Management expects sales growth in 2015 to be 20 percent. Assume that equity accounts and long-term debt do not vary directly with sales, but change when retained earnings change or additional capital is issued. Munson Communications Company Balance Sheet as of June 30, 2014 Assets: Cash Accounts receivable Inventories Total current assets
Liabilities and Stockholders’ Equity: $ 1,728,639 Accounts payable 3,009,421 Notes payable 11,492,993 $16,231,053 Total current liabilities
Net fixed assets Other assets
22,380,636 Long-term debt 1,748,906 Common stock Retained earnings $40,360,595 Total liabilities and equity
Total assets
$ 4,666,673 2,507,094 $ 7,173,767 13,345,242 10,165,235 9,676,351 $40,360,595
Munson Communications Company Income Statement for the Fiscal Year Ended June 30, 2014 Net sales Costs EBITDA Depreciation EBIT Interest EBT Taxes (35%) Net income a. b. c. d.
$79,722,581 59,358,499 $20,364,082 7,318,750 $13,045,332 3,658,477 $ 9,386,855 3,285,399 $ 6,101,456
What is the firm’s internal growth rate (IGR)? What is the firm’s sustainable growth rate (SGR)? What is the external funding needed (EFN) to accommodate the expected growth? Construct the firm’s 2015 pro forma financial statements under the assumption that long-term debt will provide all external funding.
SM 1-860
Solution: a. Retention ratio = 45.0% Net income in 2014 = $ 6,101,456 Expected net income in 2015 = $6,101,456 (1.20) = $7,321,747 Addition to retained earnings = $ 3,294,786 ROE = 30.8% Growth rate = 20% IGR =
b. c.
Addition to retained earnings Initial assets
=
$ 3,294,786 $ 40,360,595
=
8.16%
SGR = Plowback ratio×ROE = 0.45× 0.308 = 13.84% EFN = (Growth rate×Total assets) - Addition to retained earnings = (0.20×$40, 360,595) - (0.45×$7, 321, 747) = $4, 777, 333
d.
Cash Accounts receivable Inventories Total current assets
Munson Communications Company—Pro Forma Balance Sheet as of June 20, 2015 2014 2015 2014 $ 1,728,639 1.20 $ 2,074,367 Accounts payable 4,666,673 3,009,421 1.20 3,611,305 Notes payable 2,507,094 11,492,993 1.20 13,791,592 $16,231,053 $19,477,264 Total current liabilities $ 7,173,767
1.20 1.20
Net fixed assets Other assets
22,380,636 1,748,906
1.20 1.20
Total assets
$40,360,595
1.20 1.20
26,856,763 2,098,687 $48,432,714
Long-term debt Common stock Retained earnings Total liabilities & equity
13,345,242 10,165,235 9,676,351 $40,360,595
Pro Forma Income Statement for the Fiscal Year Ended June 30, 2015 2014 2015 Revenues $79,722,581 1.20 $95,667,097 Costs 59,358,499 1.20 71,230,199 EBITDA $20,364,082 $24,436,898 Depreciation 7,318,750 1.20 8,782,500 EBIT $13,045,332 $15,654,398 Interest 3,658,477 1.20 4,390,172 EBT $ 9,386,855 $11,264,226 Taxes (35%) 3,285,399 1.20 3,942,479 Net income $ 6,101,456 $ 7,321,747 LO 5 Bloomcode: Application AASCB: Analytic
SM 1-861
IMA: FSA AICPA: Industry/Sector Perspective
Sample Test Problems 19.1 Mars Company had net sales of $18 million in the year that just ended. Next year, the company’s management expects a 15 percent increase in sales. If cost of goods sold is 60 percent of sales and inventory is 25 percent of sales, what would you estimate sales, inventory, and cost of goods sold to be next year? Solution: Net sales (St) = $18 million % change in net sales (%ΔS) = 15% Expected net sales (St +1) = (0.15 × $18 million) + $18 million = $20.7 million Expected inventory (I t+1) = $20.7 million × 0.25 = $5.175 million Expected cost of goods sold (COGS t+1) = $20.7 million × 0.60 = $12.42 million 19.2 Lavaca Inc. management expects net sales to be $855,000, total costs to be $647,000, and to pay taxes at an average rate of 32 percent this year. If the Lavaca pays out 38 percent of its earnings as dividends, what is its retention ratio? How much will Lavaca’s retained earnings increase? Solution: Lavaca Inc. Pro Forma Income Statement Net sales $855,000 Total costs 647,000 Taxable income $208,000 Taxes (32%) 66,560 Net income $141,440 Dividends (38%) $ 53,747 Addition to retained earnings $ 87,693 Retention ratio (1 Dividend payout ratio)
= (1 – 0.38) 0.62 or 62% Increase in Lavaca’s retained earnings = Net income – Dividends = ($141,440 – $53,747)
SM 1-862
= $87,693
19.3 Spurlock Inc. had net income of $266,778 in its most recent fiscal year and total assets of $1,833,400 at the end of the year. The company’s total debt ratio (total debt to total assets) is 35 percent, and Spurlock retains 60 percent of its income every year. What is Spurlock’s internal growth rate? What is its sustainable growth rate? Solution: Addition to retained earnings Initial total assets As Spurlock retains 60 percent of its income every year, its retention ratio = 0.60, or 60%. Spurlock’s addition to retained earnings = Retention ratio × Net income = 0.60 × $266,778 = $160,066.80 Initial total assets = $1,833,400 Internal Growth Rate (IGR) = $160,066.80 = 0.0873, or 8.73% $1,833,400.00 Sustainable growth rate (SGR) = Retention ratio × ROE Total debt = Total debt ratio × Total assets = 0.35 × $1,833,400 = $641,690 Total equity = Total assets – Total debt = $1,833,400 – $641,690 = $1,191,710 Net income ROE = Total equity $266,778 ROE = = 0.2239, or 22.39% $1,191,710 Sustainable growth rate (SGR) = 0.60 × 0.2239 = 0.1343, or 13.43% Internal Growth Rate (IGR) =
19.4 Using the information in Sample Test Problem 19.3, what is Spurlock’s capital intensity ratio if the company has net sales of $3,557,100? What does this ratio tell us? Solution: Total assets Net sales $1,833,400 = $3,557,100 = 0.5154, or 51.54% Capital intensity ratio tells us something about the amount of assets the firm needs to generate $1 in sales. The higher the ratio, the more capital the firm needs to generate sales— that is, the more capital intensive the firm. Capital intensity ratio =
SM 1-863
Bloomcode: Application AASCB: Analytic IMA: FSA AICPA: Industry/Sector Perspective
19.5 Edgefield Excavation Company has total assets of $4,976,456, sales of $1,225,700, and net income of $587,000. The company’s management expects sales to grow by 9 percent next year. All costs (including taxes) and assets vary directly with sales, and the firm expects to maintain a payout ratio of 35 percent. Calculate the external funds needed (EFN) by Edgefield. What would Edgefield’s payout ratio have to be in order for the company’s EFN to equal zero? Solution: External funds needed (EFN) = New investments – Addition to retained earnings New investments = Growth rate × Initial total assets New investments = 0.09 × $4,976,456 = $447,881.04 Addition to retained earnings = Retention ratio × Net income = (1 – Dividend payout ratio) × Net income = (1 – 0.35) × $587,000 = $381,550 External funds needed (EFN) = $447,881.04 – $381,550 = $66,331.04 So external funds needed (EFN) by Edgefield is $66,331.04. The company’s EFN will be zero if new investments is equal to the addition to retained earnings. Edgefield’s addition to retained earnings = New investments = $447,881.04 Dividend payout ratio = 1 – Retention ratio Addition to retained earnings Retention ratio = Net income $447,881.04 = = 0.763, or 76.3% $587,000 Dividend payout ratio = (1 – 0.763) = 0.237, or 23.7% Therefore, to be in order for Edgefield’s EFN to equal zero, the company’s payout ratio would be 0.237, or 23.7%. Bloomcode: Application AASCB: Analytic IMA: FSA AICPA: Industry/Sector Perspective
Chapter 20 SM 1-864
Options and Corporate Finance Before You Go On Section 20.1 13.
What is a call option, and what do the payoff functions for the owner and seller of a call option look like?
A call option gives the owner the right to buy or ―call‖ the underlying asset at a pre-specified price. The payoff for the owner of a call option is zero when the value of the underlying asset is below the exercise price, and increases dollar for dollar with the price of the underlying asset once the asset price goes above the exercise price. The payoff for the seller of a call option is 0 when the value of the underlying asset is below the exercise price, and decreases dollar for dollar with the price of the underlying asset once the asset price goes above the exercise price.
14.
What is a put option, and what do the payoff functions for the owner and seller of a put option look like?
A put option gives the owner the right to sell the underlying asset at a pre-specified price. The payoff for the owner of a put option is 0 when the value of the underlying asset is above the exercise price, and increases dollar for dollar with decrease of the price of the underlying asset when the asset price drops below the exercise price. The payoff for the seller of a put option is 0 when the value of the underlying asset is above the exercise price, and decreases dollar for dollar with decrease of the price of the underlying asset once the asset price drops below the exercise price.
SM 1-865
3.
Why does the payoff function for an option have a kink in it?
The payoff function for an option has a ―kink‖ at the exercise price. This kink exists because the owner of the option has a right, not an obligation, to buy or sell the underlying asset. If it is not in the owner’s interest to exercise the option, he or she can simply let it expire.
Section 20.2 6.
What are the limits on the value of a call option prior to its expiration date?
The value of a call option can never be less than zero since the owner of the option can always decide not to exercise it if doing so is not beneficial. A second limit on the value of a call option is that it can never be greater than the value of the underlying asset. It would not make sense to pay more for the right to buy an asset than you would pay for the asset itself. The third limit is that the value of a call option prior to expiration will never be less than the value of that option at expiration. Because of the time value of money, the final limit is that the value of a call option prior to expiration will never be less than the difference between the current value of the underlying asset and the present value of the exercise price.
7.
What variables affect the value of a call option?
The following five variables affect the value of a call option prior to expiration:
SM 1-866
(1) Current value of the underlying asset (2) Exercise price (3) Volatility of the value of the underlying asset (4) Time until the expiration of the option (5) Risk-free rate of interest
3.
Why are the variables that affect the value of a put option the same as those that affect the value of a call option?
Given the value of a call option, the value of a put option can be calculated using the putcall parity: P = C + Xe-rt – V. where "P" is the value of the put option, "C" is the value of the call option, "X" is the exercise price, "r" is the risk-free rate, "t" is the amount of time before the option expires, and "V" is the current value of the underlying asset. The term "e–rt" is the exponential function that you can calculate using the ―ex‖ key on your calculator; it is simply a discount factor that assumes continuous compounding. We can see that the formula does not include any variables other than the five factors in valuation of a call option.
Section 20.3
7.
What is a real option?
SM 1-867
Real options are options on real assets. In some cases, the value of real options can be incorporated into an investment analysis by valuing the option separately using valuation methods similar to those used to value financial options, and then adding this value to the value estimated by traditional NPV analysis.
8.
What are four different types of real options commonly found in business?
(1) Options to defer investment. (2) Options to make follow-on investments. (3) Options to change operations. (4) Options to abandon projects.
3.
Is it always possible to estimate the value of a real option? Why or why not?
In some instances, it is not possible to estimate the value of a real option because we do not have enough information or because the necessary analysis is too complex. Although it might not even be possible to directly estimate the value of the real options associated with a project, it is important to recognize that they exist when you perform a project analysis.
Section 20.4 3.
What do the payoff functions for stockholders and lenders look like?
SM 1-868
The payoff function for the stockholders looks exactly like that for the owner of a call option where the exercise price is the amount owed on the loan and the underlying asset is the firm itself. The payoff function for the lenders looks like that for the seller of a put option, where the exercise price is the amount of the loan and the underlying asset is the firm itself.
4.
What does the payoff function for a typical manager look like?
When a company defaults on its debt, the payoff function for the manager will look something like that for the lender—it will slope downward as the value of the firm decreases. On the positive side, a manager’s payoff will increase with the value of the firm when this value is above the amount that the company owes to its lenders.
Section 20.5 7.
What is hedging?
Hedging is a method of reducing financial risks faced by a firm. Options, along with other derivative instruments, such as forwards, futures, and swaps, are commonly used in hedging.
2.
What types of risks can options be used to manage?
SM 1-869
Options can be used to manage risks associated with commodity prices, interest rates, foreign exchange rates, and equity prices.
SM 1-870
Self-Study Problems
20.1
Of the two parties to an option contract, the buyer and the seller, who has a right and who
has an obligation?
Solution: The buyer (owner) of the option has the right to exercise the option but is not required to do so. The seller (or writer) of the option is obligated to take the other side of the transaction if the option owner decides to exercise it.
20.2
The stock of Augusta Light and Power is currently selling at $12 per share. Over the next year the company is undertaking a new electricity production project. If the project is successful, the company’s stock is expected to rise to $24 per share. If the project fails, the stock is expected to fall to $8 per share. The risk free rate is 6 percent. Calculate the value today of a one year call option on one share of Augusta Light and Power with an exercise price of $20.
Solution: First determine the payoffs for the stock, a risk-free loan, and the call option under the two possible outcomes. In one year, the stock price is expected to be either $8 or $24. The loan will be worth $1.06 regardless of whether the project is successful. If the project fails, the stock price will be less than the exercise price of the call option. The option will not be exercised, and will be worth $0. If the project is successful, the stock price will be
SM 1-871
higher than the exercise price of the call option. The option will be exercised and its value will be the difference between the stock price and the exercise price, $4. Stock (x) Today
$12
Expiration
$8
Risk-Free Loan (y) $1
$24
$1.06
Call Option ?
$1.06
$0 $24 – $20 = $4
The stock and loan can be used to create a replicating portfolio which has the same payoff as the call option: ($8 × x) + (1.06 × y) = $0 ($24 × x) + (1.06 × y) = $4 Solving the two equations yields: x = 0.25, y = –1.887 The value of the call option is the same as the current value of this portfolio: ($12 × 0.25) + ($1 × -1.887) = $1.11
20.3
ADCAP International is a U.S.-based company which sells its products primarily in overseas markets. The company’s stock is currently trading at $50 per share. Depending on the outcome of U.S. trade negotiations with the countries to which ADCAP exports its products, the company’s stock price is expected to be either $65 or $30 in six months. The risk free rate is 8 percent per year. What is the value of a put option on ADCAP stock that has an exercise price of $40 per share?
Solution:
SM 1-872
Here we solve directly for the value of the put option. First we determine the payoffs for the stock, a risk free bond, and the put option under the two possible outcomes. To determine payoff of the bond six months from now, we must calculate the six-month risk free interest rate given the one year risk free rate in the problem statement. Six month risk free rate = (1 + 0.08)1/2 – 1 = 1.039, or 3.9% The payoffs are therefore: Stock (x) Today
$50
Expiration
$30
Risk-Free Loan (y)
Put Option
$1
$65
$1.039
?
$1.039
$40 – $30 = $10
$0
Now we can use the stock and bond to create a replicating portfolio, which will give the same payoff as the put option: ($30 × x) + (1.039 × y) = $10 ($65 × x) + (1.039 × y) = $0 Solving the two equations we determine x = –0.286, y = 17.87 The value of the put option is the same as the current value of this portfolio: ($50 × –0.286) + ($1 × 17.87) = $3.58 Alternatively, you could solve this problem by calculating the value of a call option with an exercise price of $40 per share and then using the put-call parity relation. The value of the call option is $15.09 and the value of the associated put option calculated using the
SM 1-873
put-call parity relation is $3.52. The difference ($3.58 vs. $3.52) is due to rounding and the compounding assumption for the discount rate.
20.4
Your company is considering opening a new factory in Europe to serve the growing
demand for your product there. What real options might you want to consider in your capital budgeting analysis of the factory?
Solution: Several significant real options might be associated with the factory. First, by having a factory in Europe, and the employees and management associated with it, your company might be better positioned to introduce products to the European markets. In addition, you will have options to change operations, to sell the factory, or to simply abandon the project.
20.5 Your firm, which uses oil as an input to its production processes, hedges its exposure to changes in the price of oil by buying call options on oil at today’s price. If the price of oil goes down by the time the contract expires, what effect will that have on your company?
Solution: The effect on your company of the decline in the price of oil will be to increase earnings. This is because the oil is an input to your production process, and a drop in prices will reduce your expenses. If the price of oil goes down, you would let the call option expire without exercising it. Of course, the benefit your company receives from the drop in oil prices would be reduced by the amount that you paid to purchase the option.
SM 1-874
.
SM 1-875
Discussion Questions
20.1
Options can be combined to create more complicated payoff structures. Consider the combination of one put option and one call option with the same expiration date and the same strike price. Draw the payoff diagram and describe what the purchaser of such a combination thinks will happen before expiration.
The payoff diagram will be in the shape of a V centered at the strike price:
Buy Put and Call
Value
K
Stock Price
This combination is called a straddle. Its purchaser thinks that the value of the underlying asset will change significantly, but he is unsure in which direction it will move. If he is correct, one of his options will expire worthless, but the other will be exercised (and have value). If he is wrong, the payoff from the option that ends up being exercised will not be enough to cover the expense of buying the options in the first place. LO: 1
SM 1-876
Level: Basic Bloomcode: Application AASCP: Analytic IMA: Corporate Finance AICPA: Industry/Sector Perspective 20.2 A writer (seller) of a call option may or may not actually own the underlying asset. If he or she owns the asset, and therefore will have the asset available to deliver should the option be exercised, he or she is said to be writing a covered call. Otherwise, he or she is writing a naked call and will have to buy the underlying asset on the open market should the option be exercised. Draw the payoff diagram of a covered call (including the value of the owned underlying asset) and compare it with the payoff of other options.
Value
Covered Call
K
Stock Price
A covered call has the same payoff shape as a put option, but it is shifted upward by the value of the strike price. This is equivalent to the combination of a put option and a risk-free bond. LO: 1 Level: Basic
SM 1-877
Bloomcode: Application AASCP: Analytic IMA: Corporate Finance AICPA: Industry/Sector Perspective
20.3
An American option will never be worth less than a European option. Evaluate this
statement.
This statement is true. An American option has all the rights that a European option has. In addition, it can be exercised on any date prior to the exercise date. Since these additional rights can never have a negative value, an American option will always be worth at least as much as a corresponding European option. LO: 1 Level: Basic Bloomcode: Analysis AASCP: Analytic IMA: Corporate Finance AICPA: Industry/Sector Perspective
20.4
Explain why, in the binomial pricing theory, the probabilities of an upward move versus a downward move are not important.
The replicating portfolio calculated to value the option in this model will have the same value as the option whether the stock goes up or down. The probability of an up move could be 99 percent or 1 percent, and that would not change. The replicating portfolio is not determined by the likelihood of the two possibilities—only the option value in those
SM 1-878
two cases. That said, the value of the replicating portfolio (and therefore the option) is affected by the current stock price, which, presumably, would change with the probabilities of up or down stock price moves. LO: 2 Level: Basic Bloomcode: Comprehension AASCP: Analytic IMA: Corporate Finance AICPA: Industry/Sector Perspective
20.5
Like all other models, the binomial pricing model is a simplification of reality. In this model, how do we represent high volatility or low volatility of the value of the underlying asset?
The difference between the high and low possible future prices represents volatility. If these two numbers are relatively close together, that represents low volatility of the value of the underlying asset. The further apart they are, the higher the volatility the model represents. LO: 2 Level: Basic Bloomcode: Comprehension AASCP: Analytic IMA: Corporate Finance AICPA: Industry/Sector Perspective
20.6
What kinds of real options should be considered in the following situations?
SM 1-879
a. Wingnuts R Us is considering two sites for a new factory. One is just large enough for the planned facility, while the other is three times larger. b. Carousel Cruises is purchasing three new cruise ships to be built sequentially. The first ship will commence construction today and will take one year to build. The second will then be started. Carousel can cancel the order for a given cruise ship at any time before construction begins.
a. Wingnuts R Us should consider the option to expand operations. b. Carousel should consider the option to abandon their order and not take delivery of the additional ships. LO: 3 Level: Basic Bloomcode: Analysis AASCP: Analytic IMA: Corporate Finance AICPA: Industry/Sector Perspective
20.7
Future Enterprises is considering building a factory that will include an option to expand operations in three years. If things go well, the anticipated expansion will have a value of $10 million and will cost $2 million to undertake. Otherwise, the anticipated expansion will have a value of only $1 million and will not take place. What information would we need in order to analyze this capital budgeting problem using the traditional NPV approach that we would not need using option valuation techniques?
SM 1-880
In order to use traditional NPV techniques, we need to know the probability of ―things going well‖ so that we can estimate the expected cash flows. We will also need to know how the performance of the overall economy affects these probabilities so that we can estimate an appropriate discount rate for the expected expansion cash flows. LO: 3 Level: Basic Bloomcode: Comprehension AASCP: Analytic IMA: Corporate Finance AICPA: Industry/Sector Perspective
20.8
Corporations frequently include employee stock options as a part of the compensation for their managers and sometimes for all of their employees. These options allow the holder to buy the stock of the company for a preset price like any other option, but they are usually very long lived, with maturities of 10 years. The goal of stock option plans is to align the incentives of employees with those of stockholders. What are the implications of these compensation plans for current stockholders?
Such plans have the effect of diluting the value of any gains in stock price. When the value of the firm goes up, the options will be exercised and the result will be more people with a claim on the same underlying assets. The value of each claim will be reduced. At the same time, the objective of the plans is to mitigate the agency problems associated with managers’ and employees’ incentives. Managers and employees who have been granted stock options have more to gain when the company does well. They are less
SM 1-881
likely to be too conservative when deciding which projects the company will pursue. If the stock option plans achieve their objective, the firm will be worth more than it would have been without the plan, and so there will be more wealth to split up. Finding a good trade-off between these effects is a challenge. LO: 1 Level: Intermediate Bloomcode: Comprehension AASCP: Analytic IMA: Corporate Finance AICPA: Industry/Sector Perspective
20.9
You are a bond holder of ABC Corp. Using option pricing theory, explain what agency concerns you would have if ABC were in danger of bankruptcy.
The equity of a firm can be thought of as a call option on the assets of the firm. If the firm is close to bankruptcy, you as a bondholder should be concerned that the company will take actions to increase the volatility of the assets by investing in highly risky projects, turn down positive-NPV projects, and, possibly, try to increase the dividend payouts. All of these actions can increase the value of the equity at the expense of the value of the debt holders. LO: 4 Level: Basic Bloomcode: Comprehension AASCP: Analytic IMA: Corporate Finance AICPA: Industry/Sector Perspective
SM 1-882
20.10 A bond covenant is a part of a bond contract that restricts the behavior of the firm, barring it from taking certain actions. Using the terminology of options, explain why a bond contract might include a covenant preventing the firm from making large dividend payments to its stockholders.
Cash on hand reduces the total volatility of the value of the firm. If any cash on hand is paid out to stockholders in the form of a large dividend payment, the volatility of the firm will increase and the value of the debt will decrease. In order to prevent this, bonds often feature covenants restricting the firm to dividend payments no larger than some fraction of current earnings. LO: 4 Level: Basic Bloomcode: Comprehension AASCP: Analytic IMA: Corporate Finance AICPA: Industry/Sector Perspective
20.11 How can the insurance policy on a car be viewed as an option?
The insurance policy on a car can be viewed as a put option. In the extreme case, where your car is totaled, you have the right to put the car to the insurance company for its market value prior to the accident.
LO: 5
SM 1-883
Level: Basic Bloomcode: Comprehension AASCP: Analytic IMA: Corporate Finance
AICPA: Industry/Sector Perspective
SM 1-884
Questions and Problems
BASIC 20.1
Option characteristics: What is an option? Solution:
An option is the right to buy or sell an asset at a pre-specified price on or before a pre-specified date. LO: 1 Bloomcode: Knowledge AASCP: Analytic IMA: Corporate Finance AICPA: Industry/Sector Perspective
20.2
Option characteristics: Explain how the payoff functions differ for the owner (buyer) and the seller: (1) of a call option; (2) of a put option.
Solution: (1) Call option: When the value of the underlying asset is below the exercise price, the payoffs for both the buyer and seller of a call option are 0. Once the asset price goes above the exercise price, the payoff for the buyer increases dollar for dollar with the price of the underlying asset, while the payoff for the seller decreases dollar for dollar with the price of the underlying asset. The sum of the payoffs of the buyer and seller of a call option is always 0. (2) Put option:
SM 1-885
When the value of the underlying asset is above the exercise price, the payoffs for both the buyer and seller of a put option are 0. Once the asset price drops below the exercise price, the payoff for the buyer increases dollar for dollar with decrease of the price of the underlying asset, while the payoff for the seller decreases dollar for dollar with decrease of the price of the underlying asset. The sum of the payoffs of the buyer and seller of a put option is always 0. LO: 1 Bloomcode: Comprehension AASCP: Analytic IMA: Corporate Finance AICPA: Industry/Sector Perspective
20.3
Option payoffs: What is the payoff for a call option with a strike price of $50 if the stock price at expiration is $40? What if the stock price is $65?
Solution: If the stock price is $40, then the option will not be exercised and the option is worthless. (Why buy for $50 what you can buy in the market for $40?) If the stock price is $65, then the option is worth the difference between the price and the strike price: $65 – $50 = $15. LO: 1 Bloomcode: Application AASCP: Analytic IMA: Corporate Finance AICPA: Industry/Sector Perspective
20.4
Option payoffs: What is the payoff for a put option with a strike price of $50 if the stock price at expiration is $40? What if the stock price is $65?
SM 1-886
Solution: If the stock price is $40, then the option is worth the difference between the strike price and the stock price: $50 – $40 = $10. If the stock price is $65, then the option is worthless. LO: 1 Bloomcode: Application AASCP: Analytic IMA: Corporate Finance AICPA: Industry/Sector Perspective
20.5
Option valuation: What are the five variables that affect the value of an option, and how do changes in each of these variables affect the value of a call option?
Solution: The value of a call option increases as: (1) Current value of the underlying asset increases; (2) Exercise price decreases; (3) Volatility of the value of the underlying asset increases; (4) Time until the expiration of the option increases; or (5) Risk-free rate of interest increases. LO: 2 Bloomcode: Knowledge AASCP: Analytic IMA: Corporate Finance AICPA: Industry/Sector Perspective
SM 1-887
20.6
Option valuation: Assuming nothing else changes, what happens to the value of an option as time passes and the expiration date gets closer? Solution: The value of an option declines as time passes. LO: 2
Bloomcode: Comprehension AASCP: Analytic IMA: Corporate Finance AICPA: Industry/Sector Perspective
20.7
Option valuation: What does the seller of a put option hope will happen? Solution: He hopes the option will not be exercised. In this case, the option will be exercised if the asset value at expiration is lower than the strike price, so he hopes the asset value will rise (or at least will remain above the strike price). LO: 2
Bloomcode: Comprehension AASCP: Analytic IMA: Corporate Finance AICPA: Industry/Sector Perspective
20.8
Option valuation: What is the value of a call option if the stock price is zero? What if the stock price is extremely high (relative to the strike price)? Solution: If the stock price is zero, then there is no possibility that the stock will have positive value. Thus the option to buy the stock in the future is worthless. If the stock price is
SM 1-888
extremely high compared to the strike price, then there is essentially zero probability that the stock price will be below the strike price at expiration. The option will always be exercised, and the value of the option is the difference between the current stock price and the present value of the strike price. LO: 2 Bloomcode: Comprehension AASCP: Analytic IMA: Corporate Finance AICPA: Industry/Sector Perspective
20.9
Option valuation: Like owners of stock, owners of options can lose no more than the amount they invested. They are far more likely to lose that full amount, but they cannot lose more. Do sellers of options have the same limitation on their losses?
Solution: No. The seller of a call option can lose a theoretically unlimited amount of money because the value of the underlying asset can go arbitrarily high. The seller of a put option is limited to losing the amount of the strike price (since that is how much they would lose if the stock price went to zero). LO: 2 Bloomcode: Comprehension AASCP: Analytic IMA: Corporate Finance AICPA: Industry/Sector Perspective
SM 1-889
20.10 Option valuation: What is the value at expiration of a call option with a strike price of $65 if the stock price is $1? $50? $65? $100? $1,000?
Solution: The value of the option for any stock price less than $65 (including $1 and $50) is zero because the option would not be exercised. When the stock price is equal to the strike price ($65 in this case), the option owner does not care whether or not he exercises the option. He gains (or loses) nothing. Again, the option is worth zero. If the stock price is higher than the strike price, the option is worth the difference. If the stock is worth $100, the option is worth $35. If the stock is worth $1,000, the option is worth $935. LO: 2 Bloomcode: Comprehension AASCP: Analytic IMA: Corporate Finance AICPA: Industry/Sector Perspective
20.11 Option valuation: Suppose you have an option to buy a share of ABC Corp. stock for $100. The option expires tomorrow, and the current price of ABC Corp. is $95. How much is your option worth?
Solution: Your option is worth very slightly more than zero. There is little chance that the stock price will move above $100 by tomorrow, but the chance is not zero, so the option still has some value. LO: 2 Bloomcode: Application
SM 1-890
AASCP: Analytic IMA: Corporate Finance AICPA: Industry/Sector Perspective
20.12 Option valuation: You hold an American option to sell one share of Zyther Co. stock. The option expires tomorrow. The strike price of the option is $50, and the current stock price is $49. What is the value of exercising the option today? If you wanted to sell the option instead, about how much would you expect to receive?
Solution: The value of exercising today is the difference between the strike price and the stock price: $50 – $49 = $1. If you sold the option, you should expect to receive slightly more than that. LO: 2 Bloomcode: Application AASCP: Analytic IMA: Corporate Finance AICPA: Industry/Sector Perspective
20.13 Real options: What is the difference between a financial option and a real option?
Solution: The underlying asset of a financial option is a financial asset, while the underlying asset of a real option is a real asset like investment projects. LO: 3 Bloomcode: Analysis
SM 1-891
AASCP: Analytic IMA: Corporate Finance AICPA: Industry/Sector Perspective
20.14 Real options: List and describe four different types of real options that are associated with investment projects.
Solution: Four types of real options are associated with investment projects: (1)
Options to defer investment: the ability to defer an investment decision
until information on its future cash flows is less uncertain. (2)
Options to make follow-on investments: the possibility to exploit future
business opportunities that will not otherwise be available if the investment is not taken. (3)
Options to change operations: the flexibility in changing operations as
business conditions change if the investment is taken. (4)
Options to abandon projects: the ability to terminate the project at a
smaller loss if things do not go as well as anticipated. LO: 3 Bloomcode: Knowledge AASCP: Analytic IMA: Corporate Finance AICPA: Industry/Sector Perspective
20.15 Agency costs: How are options related to the agency costs of debt and equity?
SM 1-892
Solution: Agency costs arise since the incentives of shareholders are different from those of the debt holders. Equity and debt claims are like different types of options on the firm. The payoff function for the stockholders looks exactly like that for the owner of a call option where the exercise price is the amount owed on the loan and the underlying asset is the firm itself. The payoff function for the lenders looks like that for the seller of a put option, where the exercise price is the amount of the loan and the underlying asset is the firm itself. The different payoff functions create different incentives for shareholders and debt holders. For example, shareholders are likely to pursue more risky projects. The increased volatility of cash flows increases the expected payout for the holder of a call option, the shareholder, and decreases the expected payout for the seller of a put option, the debt holder. LO: 4 Bloomcode: Comprehension AASCP: Analytic IMA: Corporate Finance AICPA: Industry/Sector Perspective
INTERMEDIATE 20.16 Option valuation: Suppose that you own a call option and a put option on the same stock and that these options have the same exercise price. Explain how the relative values of these two options will change as the stock price increases or decreases.
Solution:
SM 1-893
When stock prices increase, the value of the call option increases and the value of the put option decreases; when stock prices decrease, the value of the call option decreases and the value of the put option increases. The further the stock price is from the exercise price, the more valuable the combination of these options becomes. LO: 2 Bloomcode: Comprehension AASCP: Analytic IMA: Corporate Finance AICPA: Industry/Sector Perspective
20.17 Other options: A callable bond is a bond that can be bought back by the bond issuer before maturity for some pre-specified price (normally a small amount above face value) at the discretion of the bond issuer. How would you go about finding the value of such a bond? Would the bond be worth more or less than an equivalent noncallable bond?
Solution: The purchaser of a callable bond is simultaneously selling the issuer a call option on that bond. The value would be equal to the value of the straight bond minus the value of the option. This is clearly less than the value of the (noncallable) bond by itself. LO: 2 Bloomcode: Comprehension AASCP: Analytic IMA: Corporate Finance AICPA: Industry/Sector Perspective
SM 1-894
20.18 Other options: A convertible bond is a bond that can be exchanged for stock at the discretion of the bondholder. How would you go about finding the value of such a bond? Would the bond be worth more or less than an equivalent nonconvertible bond?
Solution: A convertible bond is equivalent to a combination of a nonconvertible bond and a call option on the stock, where the strike price of the option is the value of the bond. Because the value of the bond may change along with the value of the stock, this is not a straightforward problem, but the value would be equal to the value of the straight bond plus the value of the option. This is clearly more than the value of the (nonconvertible) bond by itself. LO: 2 Bloomcode: Comprehension AASCP: Analytic IMA: Corporate Finance AICPA: Industry/Sector Perspective
20.19 Option valuation: The seller of an option can never make any money from a change in the value of the underlying asset; he or she can only hope that the option will not be exercised and that he or she will not lose any money. Given that this is the case, why do people sell options?
Solution: Option writers receive the value of the option up front. The money the seller receives by selling the bond is referred to as the bond premium. This payment compensates them for the obligation they are taking on.
SM 1-895
. LO: 2 Bloomcode: Comprehension AASCP: Analytic IMA: Corporate Finance AICPA: Industry/Sector Perspective
20.20 Option valuation: The stock of Socrates Motors is currently trading for $40 and will either rise to $50 or fall to $35 in one month. The risk-free rate for one month is 1.5 percent. What is the value of a one-month call option with a strike price of $40?
Solution:
Today
Expiration
$35
Stock (X)
Risk-Free (Y)
Option
$40
$1
???
$50
$1.015
$1.015
$0
$10
These two equations define our portfolio: $10 = ($50 × X) + ($1.015 × Y) $0 = ($35 × X) + ($1.015 × Y) and the solution is X = 2/3 and Y = -$22.99. That is, we borrow $22.99 and buy twothirds of a share of Socrates Motors at a cost of $40.00 per share. The total net cost of this portfolio is $40.00 × (2/3) – 22.99 = $3.68 and this is the value of the option. LO: 2 Bloomcode: Application
SM 1-896
AASCP: Analytic IMA: Corporate Finance AICPA: Industry/Sector Perspective
20.21 Option valuation: Again assume that the price of Socrates Motors stock will either rise to $50 or fall to $35 in one month and that the risk-free rate for one month is 1.5 percent. How much is an option with a strike price of $40 worth if the current stock price is $45 instead of $40?
Solution: The replicating portfolio of X = 2/3 and Y = -$22.99 does not depend on the current stock price and so would not change. The value of that portfolio would now be $45 × (2/3) – $22.99 = $7.01 and this is the value of the option. LO: 2 Bloomcode: Application AASCP: Analytic IMA: Corporate Finance AICPA: Industry/Sector Perspective
20.22 Option valuation: Assume that the stock of Socrates Motors is currently trading for $40 and will either rise to $50 or fall to $35 in one month. The risk-free rate for one month is 1.5 percent. What is the value of a one-month call option with a strike price of $25?
SM 1-897
Solution:
Stock (X)
Risk-Free (Y)
Option
$40
$1
???
Today
Expiration
$35
$50
$1.015
$1.015
$10
$25
These two equations define our portfolio: $25 = ($50 × X) + ($1.015 × Y) $10 = ($35 × X) + ($1.015 × Y) and the solution is X = 1 and Y = -$24.63. That is, we borrow $24.63 and buy 1 share of Socrates Motors at a cost of $40. The total net cost of this portfolio is $40 – $24.63 = $15.37, and this is the value of the option. LO: 2 Bloomcode: Application AASCP: Analytic IMA: Corporate Finance AICPA: Industry/Sector Perspective
20.23 Option valuation: You are considering buying a three-month put option on Wing and a Prayer Construction stock. The company’s stock currently trades for $10 per share and its price will either rise to $15 or fall to $7 in three months. The risk-free rate for three months is 2 percent. What is the appropriate price for a put option with a strike price of $9?
SM 1-898
Solution: If the stock rises to $15, then the option will not be exercised and its payoff will be $0. If the stock falls to $7, the option will be exercised and its value will be the difference between the strike price and the stock price, $2.
Today
Expiration
Stock (X)
Risk-Free (Y)
Option
$10
$1
???
$7
$15
$1.02
$1.02
$2
$0
The formulas that define the replicating portfolio are: $2 = ($7 × X) + ($1.02 × Y) $0 = ($15 × X) + ($1.02 × Y) and the solution is X = -$0.25 and Y = $3.68. That is, we will short sell one-fourth of a share of the stock, receiving $2.5, and we will lend $3.68 at the risk-free rate. (Short selling is the process of borrowing an asset that you do not own and selling, with the promise that at some time in the future you will buy it back and return it to its owner.) The net cost is $3.68 – $2.50 = $1.18. This is the value of the option. LO: 2 Bloomcode: Application AASCP: Analytic IMA: Corporate Finance AICPA: Industry/Sector Perspective
SM 1-899
20.24 Option valuation: You hold a European put option on Tubes, Inc., stock, with a strike price of $100. Things haven’t been going too well for Tubes. The current stock price is $2, and you think that it will either rise to $3 or fall to $1.50 at the expiration of your option. The appropriate risk-free rate is 5 percent. What is the value of the option? If this were an American option, would it be worth more?
Solution: Note that this option will be exercised regardless.
Stock (X)
Risk-Free (Y)
Option
$2.0
$1
???
Today
Expiration
$1.5
$3.0
$1.05
$1.05
$98.5
$97
The formulas that define the replicating portfolio are: $98.5 = ($1.5 × X) + ($1.05 × Y) $97 = ($3 × X) + ($1.05 × Y) and the solution is X = –1 and Y = $95.24. That is, we will short sell 1 of a share of the stock, receiving $2, and we will lend $95.24 at the risk-free rate. The net cost is $95.24 – $2.00 = $93.24. This is the value of the (European) option. If this were an American option, we could exercise today if we wanted. We would want to do so if the value of exercising today was greater than the value of the option.
SM 1-900
Exercising today would allow us to sell for $100 something worth $2, a gain of $98. This is more than $93.24. The American option is worth more. LO: 2 Bloomcode: Application AASCP: Analytic IMA: Corporate Finance AICPA: Industry/Sector Perspective
20.25 Other options: A golden parachute is a part of a manager’s compensation package that makes a large lump-sum payment in the event that the manager is fired (or loses his or her job in a merger, for example). Providing such payouts to managers seems ill advised to most people first hearing about it. Explain how a golden parachute can help reduce agency costs between stockholders and managers.
Solution: Managers have incentives to avoid bankruptcy or even underperformance because the personal cost to them is quite high. As a result they may choose to avoid high risk investments even if those investments are highly positive NPV. The costs to the manager if the project goes badly outweigh the benefits if it goes well. However, stockholders want the manager to take positive-NPV projects, even if they are risky. Golden parachutes help to solve this problem by reducing the costs to the manager associated with poor performance or bankruptcy. The manager faces a payoff structure like the seller of an option. Golden parachutes serve to reduce the volatility of that option and therefore to reduce the value of that option.
SM 1-901
LO: 4 Bloomcode: Comprehension AASCP: Analytic IMA: Corporate Finance AICPA: Industry/Sector Perspective
ADVANCED 20.26 Consider the following payoff diagram.
Covered Call
Value
$10
$40
$50
$60
Stock price
Find a combination of calls, puts, risk-free bonds, and stock that has this payoff. (You need not use all of these instruments, and there are many possible solutions.)
Solution: One possible solution would consist of four options: (a) Buy a call with a strike of $50.
SM 1-902
(b) Buy a put with a strike of $50. (c) Sell a put with a strike of $40. (d) Sell a call with a strike of $60. If the stock price at expiration is below $40, then neither call will be exercised, but both puts will be. We will be forced to buy the stock for $40 because of option c, but we will be able to sell it for $50, using option b. Our net gain will be $10. If the stock price is between $40 and $50, the only option to be exercised will be option b, and we will gain the difference between $50 and the stock price. If the stock price is between $50 and $60, the only option to be exercised will be a call option and we will gain the difference between the stock price and $50. If the stock price is above $60, then neither put will be exercised, but both calls will be. We will be able to buy the stock for $50 using option a, but we will be forced to sell it for $60 by the owner of option d. Our net gain will be $10. LO: 1 Bloomcode: Analysis AASCP: Analytic IMA: Corporate Finance AICPA: Industry/Sector Perspective
20.27 Consider the payoff structures of the following two portfolios: a. Buying a one month call option on one share of stock at a strike price of $50 and saving the present value of $50 (so that at expiration it will have grown to $50 with interest). b. Buying a one month put option on one share of stock at a strike price of $50 and buying one share of stock.
SM 1-903
What conclusion can you draw about the relation between call prices and put prices from a comparison of these two portfolios?
Solution: The payoff of these two portfolios is identical. In the first case, if the stock price is below $50 at expiration, you will not exercise and you will be left with $50. If it is above $50, you will exercise and you will have a share of stock (whatever it is worth). In the second case, if the price is below $50, you will exercise your put option and get $50 for your share of stock. If it is above $50, you will not exercise and you will retain your share of stock (whatever it is worth). From this, we can see that the value of the two portfolios today must be the same. That is: P = C + Xe-rt – V where P is the value of the put option, C is the value of the call option, X is the exercise price, r is the risk-free rate, t is the amount of time before the option expires, and V is the current value of the underlying asset. Th e term e2rt is the exponential function that you can calculate using the ―ex‖ key on your calculator; it is simply a discount factor that assumes continuous compounding. If we know the value of a call (and the current stock price and interest rate), we can calculate the value of a put. This relation is call put-call parity. Note that this relation is not dependent on any option pricing model. LO: 2 Bloomcode: Analysis AASCP: Analytic IMA: Corporate Finance AICPA: Industry/Sector Perspective
SM 1-904
20.28 One way to extend the binomial pricing model is by including multiple time periods. Suppose Splittime, Inc., is currently trading for $100 per share. In one month, the price will either increase by $10 (to $110) or decrease by $10 (to $90). The following month will be the same. The price will either increase by $10 or decrease by $10. Notice that in two months, the price could be $120, $100, or $80. The risk-free rate is 1 percent per month. Find the value today of an option to buy one share of Splittime in two months for a strike price of $105. (Hint: To do this, first find the value of the option at each of the two possible one-month prices. Then use those values as the payoffs at one month and find the value today.)
Solution: If the price of Splittime goes from $100 to $90 during the first month, then this option will never be exercised because the highest price to which Splittime could rise by two months is $100, which is lower than the strike price of $105. Therefore, the value of the option in the ―down case‖ is $0. If the price rises to $110 in the first month, then the payoff of the option will be either $15 (if the price continues to rise to $120) or $0 (if the price falls back to $100).
SM 1-905
One month
Expiration
Stock (X)
Risk-Free (Y)
Option
$110
$1
???
$120
$100
$1.01
$1.01
$0
$15
These two equations define our portfolio: $15 = $120 × X + $1.01 × Y $0 = $100 × X + $1.01 × Y The solution is X = 0.75 shares and Y = -$74.26. The value of this portfolio, and therefore the value of the option at this point, is $8.24. (0.75 × $110 – $74.26). Now we are ready to calculate the value of the option today using $0 and $8.24 as the one-month option ―payoffs.‖
Today
One month
$90
Stock (X)
Risk-Free (Y)
Option
$100
$1
???
$110
$1.01
$1.01
$0
$8.24
Now the equations that define our portfolio are $8.24 = $110 × X + $1.01 × Y $0 = $90 × X + $1.01 × Y
SM 1-906
The solution is X = 0.412 and Y = -$36.72. The portfolio value is$4.49, and this is the value of the option. LO: 2 Bloomcode: Application AASCP: Analytic IMA: Corporate Finance AICPA: Industry/Sector Perspective
20.29 SpinTheWheel Co. has assets currently worth $10 million in the form of one-year riskfree bonds that will return 10 percent. The company has debt with a face value of $5.5 million due in one year. (No interest payments will be made.) The stockholders decided to sell $8 million of the risk-free bonds and to invest the money in a very risky venture. This venture consists of giving Mr. William Kid the money now and, in one year, flipping a coin. If it comes up heads, Mr. Kid will pay SpinTheWheel $17.6 million. If it is tails, SpinTheWheel gets nothing. This investment has an NPV of zero. a. What is the value of the debt and equity before the stockholders make this ―investment‖? b. Using the binomial pricing model, with the payoff to the equity holders representing the option and the assets of the company representing the underlying asset, estimate the value of the equity after the stockholders make the investment. c. What is the new value of the debt after the investment?
Solution: a. The firm is certain to have the $5.5 million owed to the debt holders in one year, so the correct discount rate for the debt is the risk-free rate, 10 percent. Thus the value
SM 1-907
today of the debt is the present value of the debt at the risk-free rate: $5.5 million / 1.1 = $5 million. The value of the equity is therefore $10 million – $5 million = $5 million. b. The payoffs of the option (the equity), the underlying (the firm), and the risk-free bond are:
Today
One year
Firm
Risk-Free (Y)
Equity
$10 million
$1
???
$2.2 million $19.8 million $1.1
$1.1
$0
$14.3
Note that the payoff of $14.3 for the option in the up case comes from the $19.8 million less the $5.5 million they will pay the bondholders. In the down case, there is not enough money to pay the bondholders what they are owed, so the bondholders will receive the entire $2.2 million value of the firm and the equity holders get nothing. The equations are:
$14.3 = $19.8 × X + $1.1 × Y $0 = $2.2 × X + $1.1 × Y and the solution is X = 0.8125 and Y = -$1.625. The value of this portfolio is $6.5 million. This is the new value of the equity.
SM 1-908
c. The current value of the firm did not change (this was a zero NPV project) and remains at $10 million. If the equity has increased in value to $6.5 million, the debt has decreased in value to $3.5 million. LO: 4 Bloomcode: Application AASCP: Analytic IMA: Corporate Finance AICPA: Industry/Sector Perspective
20.30 The price of a stock that does not pay dividends is currently $35, and the risk-free rate is 4 percent. A European call option on the stock, with a strike price of $35 and which expires in six months, sells for $3.04. A European put option on the same stock with the same strike price sells for $2.35. Is there an arbitrage opportunity here? If so, what is it? Solution: To check whether there is an arbitrage opportunity, substitute the values into the put-call parity relation and see if it holds. P = C + Xe–rt – V $2.35 = $3.04 + $35e-(0.04)(6/12) - $35 Since the put-call parity relation holds, there is no arbitrage opportunity. LO: 1 Bloomcode: Application AASCP: Analytic IMA: Corporate Finance AICPA: Industry/Sector Perspective
20.31 Two call options have been written on the same underlying stock. Call #1 has a strike price of $42, and call #2 has a strike price of $52. Call #1 is selling for $5.00, and call #2 is selling for $6.00. What arbitrage opportunity do these prices present investors? Show the potential payoffs from this opportunity. Solution: The payoff diagrams for the buyers of the two call options are as follows:
SM 1-909
Option Value at Expiration
$20 $18 $16 $14 $12 $10 $8 $6 $4 $2 $0
$42 Call $52 Call
$28
$32
$36
$40
$44
$48
$52
$56
$60
Stock Price
Notice that call #1 has a lower strike price and costs less. In a situation like this you can earn an arbitrage opportunity by purchasing the less expensive option (#1) and selling the more expensive (#2). The payoffs for this strategy are: Buy $42 Call $0 $0 $0 $0 $0 $0 $0 $0 $2 $4 $6 $8 $10 $12 $14 $16 $18
Sell Total $52 Call Proceeds $6 $6 $6 $6 $6 $6 $6 $6 $6 $6 $6 $6 $6 $6 $6 $6 $6 $8 $6 $10 $6 $12 $6 $14 $6 $16 $4 $16 $2 $16 $0 $16 -$2 $16
$20
Option Value at Expiration
Stock Price $28 $30 $32 $34 $36 $38 $40 $42 $44 $46 $48 $50 $52 $54 $56 $58 $60
$42 Call $52 Call
$15
Total Proceeds $10
$5
$0
$28
$32
$36
$40
$44
$48
$52
$56
-$5 Stock Price
LO: 4 Bloomcode: Application AASCP: Analytic IMA: Corporate Finance AICPA: Industry/Sector Perspective 20.32 Husky Motors has two debt issues outstanding, both of which mature in five years. The senior debt issue, which has a face value of $10 million, must be paid in full before any of the
SM 1-910
$60
principal for the junior debt issue is paid. The junior debt issue also has a face value of $10 million. Draw the payoff diagrams for Husky’s equity and both debt issues as the value of the firm changes. Under what circumstances would you expect to see conflicts between the senior and junior debt holders? Solution: The payoff functions are as follows:
SM 1-911
VD Senior
VF VD Junior
VF VE From these payoff functions you can see that there are likely to be conflicts between the senior isFnear or below $20 million. Below $20 and junior debt hol ders when the value of the firm V million, the payof$f1f0unction$f2o0r the junior debt holder looks like that for the stockholders. This means that the junior debt holders will prefer riskier investments and strategies than the senior debt holders. These types of conflicts between lenders are quite common when a firm gets into financial distress. LO: 4 Bloomcode: Analysis AASCP: Analytic IMA: Corporate Finance AICPA: Industry/Sector Perspective 20.33 The payoff function for the holder of straight debt looks like that for the seller of a put option. Convertible debt is straight debt plus a call option on a firm’s stock. How does the addition of a call option to straight debt affect the concern that lenders have about the asset substitution problem, and why?
Solution: It mitigates this concern because the lenders will benefit through the call option from increased volatility in the value of the firm. How much a conversion option mitigates this concern depends on the specific characteristics of the option. LO: 2 Bloomcode: Comprehension
SM 1-912
AASCP: Analytic IMA: Corporate Finance AICPA: Industry/Sector Perspective
SM 1-913
Sample Test Problems
20.1 You own a call option on Pepsico stock with a strike price of $60 per share that expires in 60 days. The current market price of Pepsico stock is $63.50 per share. What are the limits on the value of the call option you own? Solution: The value of a call option must be equal to or greater than zero. The option also cannot be worth less than the value that you would receive if you exercised it immediately, where the exercise price is computed using the present value of the $60 exercise price, discounted for 60 days at the risk free rate. Bloomcode: Application AASCP: Analytic IMA: Corporate Finance AICPA: Industry/Sector Perspective 20.2 Assume that the current market price of Montrose Industrials stock is $28 per share and will either rise to $38 per share or fall to $21 per share in one month. The risk-free rate for one month is 1 percent. What is the value of a one-month call option with a strike price of $24 per share? Solution: If the stock rises to $38, the option will be exercised and the payoff will be the $38 – $24 = $14. If the stock falls to $21, then the option will not be exercised and its payoff will be $0.
Today
Expiration
$21
Stock (x)
Risk-Free (y)
Option
$28
$1
???
$38
$1.01
$1.01
$0
$14
The two equations define the replicating portfolio are: $14 = ($38 × x) + ($1.01 × y) $0 = ($21 × x) + ($1.01 × y) Solving for x and y yields x = 0.8235 and y = –$17.12. That is, we borrow $17.12 and buy 0.8235 of a share of Montrose Industrial stock at a cost of $28 per share. The total cost of this portfolio is ($28.00 × 0.8235) – $17.12 = $5.94, which is the value of the call option. Bloomcode: Application AASCP: Analytic IMA: Corporate Finance
SM 1-914
AICPA: Industry/Sector Perspective
20.3 The market value of Whole Foods stock is currently $53.73 per share, and the annual risk-free rate is 3 percent. A three-month call option on the stock with a strike price of $55 sells for $2.15. What is the value of a put option on Whole Foods stock that has the same strike price and expiration date if there are no arbitrage opportunities? Solution: P = C + Xe–rt – V = $2.15 + $55e–(0.03)(3/12) – $53.73 = $3.01 Bloomcode: Application AASCP: Analytic IMA: Corporate Finance AICPA: Industry/Sector Perspective
20.4
Why is it hard to account for real options in an NPV analysis
Solution: The riskiness of a project with real options varies with time and the appropriate discount rate varies with risk. To account for real options in an NPV analysis, you must not only project future cash flows, but also the probability that the options will be exercised and the impact of these exercise decisions on the rate used to discount the cash flows. Bloomcode: Comprehension AASCP: Analytic IMA: Corporate Finance AICPA: Industry/Sector Perspective
20.5 Fuel costs are a significant fraction of total costs in the airline industry. How might airline managers use options to manage fuel costs? What is the downside of doing this? Solution: Airline managers can use options to reduce volatility in operating expenses (fuel costs) and therefore profitability. By purchasing call options on fuel, they can lock in fuel prices for at least some of their fuel purchases. This can reduce the impact of increases in fuel prices on profits. The downside of doing this is that if fuel prices go down, the options will expire worthless and yet the airlines will still incur the costs (the call premiums) of buying the options. Bloomcode: Application AASCP: Analytic IMA: Corporate Finance AICPA: Industry/Sector Perspective Chapter 21
SM 1-915
International Financial Management Before You Go On Questions and Answers Section 21.1 1. What is globalization? Globalization refers to the removal of barriers to free trade and the closer integration of national economies. Consumers in many countries buy goods that are purchased from a number of countries, other than just their own. The production of goods and services has also become highly globalized. Like product markets, the financial system has also become highly integrated. 2.
What are multinational corporations? Multinational corporations are business firms that have their operations in at least one other country than their home country where they are headquartered. These firms are headquartered all over the world and are owned by a mixture of domestic and foreign stockholders.
3.
Explain the difference between American and European business executives’ views on wealth maximization. American business executives embrace shareholder wealth maximization as their number one goal, whereas in most parts of Europe, the main goal of business executives is to maximize corporate wealth. This means that shareholders are treated on par with other stakeholders, such as management, employees, creditors, or the government.
Section 21.2 1. What is foreign exchange rate risk? Foreign exchange risk is the risk of a change in investment’s value due to changes in currency exchange rates. This risk usually affects firms that import/export goods as well as investors that make international investments. 2.
How is the equilibrium exchange rate determined? The equilibrium exchange rate is the point of intersection between the currencies’ supply and demand curves. It is the point at which the quantity of the currency demanded equals the quantity of currency supplied.
SM 1-916
3.
What does it mean to hedge a financial transaction? To hedge a financial transaction with foreign exchange means to reduce the potential gains or losses caused by fluctuations in the price of foreign exchange. Many instruments can be used to hedge this risk, such as foreign currency options.
Section 21.3 17. What difficulties do firms face in estimating cash flows from an overseas project? Estimating cash flows from an overseas project is difficult because of the various ways a firm is likely to get cash flows from a subsidiary – dividends, royalty or license payments, debt repayment, management or consulting fees. It is also difficult to precisely time the cash flows or estimate the magnitude because of restrictions and controls a country is likely to put on the transfer of funds from the overseas project back to the home country. 2.
Why is the repatriation of cash flows from an overseas project considered critical? From a parent firm’s point of view, the cash flows expected from the subsidiary are the basis for undertaking a project or rejecting it. Any delays in receiving the cash flows will affect the NPV of the project. Parent companies often rely on the cash flows from foreign operations to be repatriated and to fund projects in other countries. 17.
When do companies have to consider country or political risk? Any company doing business in a foreign country has to consider political or country risks. Especially if the target country has a relatively unstable political environment, financial managers must incorporate the potential risk into the cost of the project.
Section 21.4 17. Which currency is the preferred currency of exchange in global financial markets? Why? The U.S. dollar is the preferred currency of exchange on the global financial markets, as it is perceived to be the most stable currency. This is mainly due to the size and strength of the U.S. economy, the long history of political stability, and the strength of military forces. 2.
What is the difference between foreign bonds and Eurobonds?
SM 1-917
Both foreign bonds and Eurobonds are types of international bonds. However, foreign bonds are long-term debt instruments sold by a foreign firm to investors in another country, and they are denominated in that country’s currency. On the other hand, Eurobonds are longterm debt instruments sold by a firm in one country but are denominated in the currency of another country. Section 21.5 17. Why is credit risk higher in international markets? In international markets, the credit risk of borrowers is higher, as it is more difficult to obtain credit information abroad. It is mainly the issue of familiarity with foreign economic conditions and business practices. Thus, the cost of gathering all this necessary information is higher, which is then being passed onto the customer in terms of higher premium. 2.
List the inputs that are used in calculating a Eurocredit price. The loan pricing for Eurocredits is similar to the loan pricing that U.S. money center banks use for their largest domestic customers. The loan rate (k) is equal to a base rate, such as LIBOR, which represents the bank’s cost of funds, plus a markup, which is the bank’s lending margin: k = Base rate + X where X is the lending margin. The lending margin depends on the borrower’s credit risk; international risk factors, such as foreign exchange risk and country risk; and the bank’s gross profit margin. Equation 21.3 represents the Eurocredit pricing. K = BR + DRP + FXR + CR + GPMAR where k = individual firm’s loan rate BR = Eurocurrency base rate, such as LIBOR DRP = default risk premium FXR = foreign exchange or currency risk premium CR = country risk premium GPMAR = bank’s gross profit margin
Self-Study Problems 21.1
If a Volkswagen Passat costs $26,350 in Baltimore and €21,675 in Frankfurt, what is the exchange rate between the U.S. dollar and the euro?
SM 1-918
Solution: Cost of the car in Baltimore = Cost of the car in Frankfurt = Dollar to euro exchange rate = =
21.2
Calculate the following exchange rates given the following information. Given Compute a. ¥86.3500/$ $/¥ b. $1.8694/£ £/$ c. $0.9981/C$ C$/$
Solution: a. b. c.
21.3
$26,350 €21,675 $26,350 / €21,675 $1.2157/€
1 / ¥86.3500/$ 1 / $1.8694/£ 1 / $0.9981/C$
= = =
$0.01581/¥ £0.5349/$ C$1.00190/$
Digital, Inc., an electronic games manufacturer, is planning to purchase flash memory from one of two sources. Kyoto, Inc., quotes a price of ¥6,800 per gigabyte. The current exchange rate is ¥84.30/$. Another Japanese manufacture offers to supply the same flash memory at a price of €58.46 per gigabyte. The spot rate available is ¥121.57/€. Which is the cheaper source of flash memory for Digital?
Solution: Cost from Vendor 1: Flash memory price quote Spot rate for U.S. dollar Cost to Digital in dollars
Cost from Vendor 2: Flash memory price quote Spot rate for U.S. dollar
= = = =
¥6,800 per gigabyte ¥84.30/$ ¥6,800 / ¥84.30/$ $80.66 per gigabyte
= =
€58.46 per gigabyte ¥121.57/€
To compute the dollar cost, we need to compute the cross rate between the euro and the dollar. ¥121.57 / € = $1.4421 / € ¥84.30 / $
SM 1-919
= €58.46 × $1.4421/€ = $84.31 per gigabyte The first vendor has the cheaper quote for Digital. Cost to Digital in dollars
21.4
Columbia Corp. has just made a sale to a British customer. The sale was for a total value of £135,000 and is to be paid 60 days from now. Columbia is concerned that the British pound will depreciate against the U.S. dollar, and management plans to hedge. The company’s bank informs management that the spot rate is $1.8133/£ and the 60-day forward rate is $1.7864/£. If Columbia sells its pounds receivable at the forward rate, what is the dollar value of its receivables? If it did not enter into a forward contract and the spot rate 60 days later was $1.7635/£, how much did the company lose by not hedging?
Solution: Amount received by Columbia by selling at the forward rate: = £135,000 × $1.7864/£ = $241,164 Amount received by Columbia by selling at the spot rate 60 days later: = £135,000 × $1.7635/£ = $238,072.50 Loss incurred by not hedging = $241,164 - $238,072.50 = $3,091.50
21.5
American Bancorp is planning to make a $3.5 million loan to a French firm. Currently, LIBOR is at 4.5 percent. American considers a default risk premium of 1.15 percent, a foreign exchange risk premium of 0.35 percent, and a country risk premium of 0.13 percent. What is the loan rate charged by American Bancorp?
Solution: The loan rate charged by American Bancorp is calculated as follows: k = BR + DRP + FXR + CR = 4.5% + 1.15% + 0.35% + 0.13% = 6.13%
SM 1-920
Discussion Questions 21.1 Royal Dutch Shell, an oil company, has headquarters in both the Netherlands and the United Kingdom. What type of firm is it?
This company is both a multinational corporation and a transnational company. Since it produces, refines, and sells oil in a variety of different locations around the world, it is an MNC. Since it is owned by investors in more than one country and is managed from a global perspective, it is a transnational firm. LO: 1 Level: Basic Bloomcode: Knowledge AASCB: Diversity IMA: Global Business AICPA: Industry/Global Perspective
21.2
International economic integration and technological changes in the last couple of decades have dramatically increased globalization across many industries. Explain how a biotech firm or a medical firm (for example, a hospital) can take advantage of these changes.
Technological changes have come in a variety of forms—computers, telecommunication, data digitalization, and such. A biotech firm may be able to conduct research and development on new drugs in India, clinical trials in other countries before submitting it for FDA approval. A hospital in the United States may be able to cost efficiently digitalize medical records by transmitting the records overseas. Hospitals and doctors can use these digital records to consult physicians halfway across the world to get a more precise diagnosis. LO: 1 Level: Basic Bloomcode: Comprehension AASCB: Diversity IMA: Global Business AICPA: Industry/Global Perspective
21.3
In the United States, managers are asked to focus on maximizing stockholder value. Is this consistent with the goals of managers in Germany and Japan?
SM 1-921
In countries such as France and Germany, stockholders are treated no differently from other stakeholders of the firm. Thus, they are on a par with employees, suppliers, creditors, customers, and even the government. The firm’s management focus is on maximizing the wealth of the firm such that everyone benefits, not just the shareholders. In Japan, a group of businesses have ownerships in each other. This is known as keiretsu. Japanese managers focus on growing the market share of their firm such that the wealth of their keiretsu, and not the wealth of the shareholders, is maximized. Maximizing shareholder wealth is consistent with these other goals. Management taking this approach will, in the long run, help all the stakeholders of the firms, including the community in which they operate, to benefit. LO: 1 Level: Basic Bloomcode: Comprehension AASCB: Diversity IMA: Global Business AICPA: Industry/Global Perspective
21.4
A Canadian cooperative of wheat farmers sold wheat to a grain company in Russia. Under what circumstances will the Canadian farmers be exposed to foreign exchange risk? When will the Russian importer be facing foreign exchange risk? If the payment to the Canadian exporter is in any currency other than the Canadian dollar, the Canadian exporter faces foreign exchange risk. If the Russian importer is forced to pay the Canadian exporter in Canadian dollars, then the entire foreign exchange risk is borne by the Russian importer.
LO: 1 Level: Basic Bloomcode: Comprehension AASCB: Analytic IMA: Global Business AICPA: Industry/Global Perspective
21.5
Stardust, Inc., is an exporter of plumbing fixtures. About 30 percent of its sales are made in Canada. The sales department just found out that the Canadian dollar is at a premium against the U.S. dollar based on the 90-day forward rate, while the 180-day forward rate indicates that the Canadian dollar is at a forward discount. What is the likely impact of these rates on the company’s sales to Canada?
SM 1-922
The fact that the Canadian dollar is at a forward premium indicates that the Canadian dollar will be able to buy more U.S. dollars. In the short run, this can lead to increased sales for Stardust since Canadians will be able to buy more with their money. Six months from now, this Canadian dollar is at a forward discount. Imports from the United States will be more expensive for Canadians, and this could lead to Stardust seeing a decrease in their exports to Canada at that time. LO: 2 Level: Basic Bloomcode: Analysis AASCB: Analytic IMA: Global Business AICPA: Industry/Global Perspective
21.6
Mello Wines, a California winery, grows its grapes locally, uses local labor, and sells its wines only in the United States. Can this firm be exposed to foreign exchange risk?
Even though Mello is merely a domestic company, it does have foreign exchange risk. Suppose the U.S. dollar appreciates against the euro. This will make imported wine from the euro zone more affordable to Americans and increase import competition to Mello. LO: 2 Level: Basic Bloomcode: Application AASCB: Analytic IMA: Global Business AICPA: Industry/Global Perspective
21.7
A U.S. firm owns a subsidiary in Belgium. What kind of foreign exchange risk does the U.S. firm face?
Since it owns assets and has liabilities in Belgium denominated in euros, the firm faces foreign exchange risk. Converting the euro-denominated assets and liabilities to U.S. dollars for reporting purposes will make the dollar value of the Belgian subsidiary’s assets and liabilities fluctuate. LO: 5 Level: Basic Bloomcode: Application AASCB: Analytic IMA: Global Business
SM 1-923
AICPA: Industry/Global Perspective
21.8
Ray Corp is a U.S. electronics manufacturer with a production plant in Turkey. This morning, the Turkish government introduced a new law prohibiting the repatriation of any funds from the country for two years. What type of risk does Ray Corp. face?
Ray Corp. is facing a form of country or political risk. Country risk can be of many forms, including changes in tax or labor laws, currency or trade controls, remittance controls, or even expropriation of the firm’s assets. LO: 5 Level: Basic Bloomcode: Application AASCB: Analytic IMA: Global Business AICPA: Industry/Global Perspective
21.9
Suppose GE issues bearer bonds in France denominated in British pounds. What type of bonds are these?
This is an example of a Eurobond. GE, a foreign issuer in France is issuing a bearer bond in a currency (British pounds) to investors in a country whose currency is different from that of the bond issue. LO: 4 Level: Basic Bloomcode: Application AASCB: Analytic IMA: Global Business AICPA: Industry/Global Perspective
21.10 Give examples of U.S. banks facing different risks in international lending.
The first risk that a bank will face in any market is credit risk—the risk that the interest and/or principal will not be paid. Suppose Citicorp made a loan to a firm in Honduras. Unlike lending in the domestic market, the bank is likely to be less familiar with the new market, economic conditions, and business practices. That makes it more difficult to
SM 1-924
judge the credit worthiness of a loan customer. The second risk to consider is currency risk. The loan is made in the Honduran currency, and repayment of principal and interest payments are also going to be in it. So, Citicorp needs to protect itself against currency risk. The third risk is potentially the worst. This is political or country risk, which relates the ability of Citicorp making a profitable loan and being able to remit the funds back to the United States due to the political climate in Honduras. The more unstable a government is, the higher the country risk that Citicorp will face. LO: 5 Level: Basic Bloomcode: Comprehension AASCB: Analytic IMA: Global Business AICPA: Industry/Global Perspective
Questions and Problems BASIC 21.1
Spot rate: Ryan wants to buy a pair of leather shoes at Harrods in London that cost £113.60. If the exchange rate is $1.6177/£, what is Ryan’s cost in U.S. dollars?
Solution: Cost of shoes in London = £113.60 Spot rate = $1.6177/£ Dollar cost of shoes = £113.60 × $1.6177/£ = $183.77 LO: 2 Bloomcode: Application AASCB: Analytic IMA: Global Business AICPA: Industry/Global Perspective
21.2
Spot rate: Crescent Corporation’s recent sale to a firm in Mexico produced revenues of 13,144,800 Mexican pesos (MPs). If the firm sold the pesos to its bank and was credited with $1,077,873.60, what was the spot rate at which the pesos were converted? Solution: Revenue from sale to Mexican firm = MP 13,144,800 Dollar revenue received from sale = $1,077,873.60 Spot rate at which the foreign currency revenue was converted:
SM 1-925
$ 1,077,873.60 MP 13,144,800
=
$0.082 /MP
This is the same as MP12.195/$ LO:2 Bloomcode: Application AASCB: Analytic IMA: Global Business AICPA: Industry/Global Perspective
21.3
Spot rate: Given the following direct quotes, calculate the equivalent indirect quotes. a. $0.0844/Mexican pesos b. £0.8513/€ c. Rs 54.64/ C$
Solution: a. b. c.
1 MP11.8483/$ $0.0844 / MP 1 / £0.8513/€ = €1.1747£ 1 / Rs 54.64/ C$ = C$0.0183/Rs.
LO: 2 Bloomcode: Application AASCB: Analytic IMA: Global Business AICPA: Industry/Global Perspective 21.4
Spot rate: Convert the following indirect quotes to the appropriate American quotes. a. £0.6917/$ b. ¥104.28/$ c. SF 1.0769/$
Solution: a. 1 / £0.6917/$ = $1.4457/£ b. 1 / ¥104.28/$ = $0.0096 / ¥ c. 1 / SF 1.0769/$ = $0.9286 / SF LO: 2 Bloomcode: Application AASCB: Analytic IMA: Global Business
SM 1-926
AICPA: Industry/Global Perspective
21.5
Spot rate: Suppose a BMW 528i is priced at $68,750 in New York and € 50,267 in Berlin. In which place is the car more expensive if the spot rate is $1.3677/€?
Solution: Cost of car in New York = $68,750 Cost of car in Berlin = € 50,267 Spot rate = $1.3677/€? Dollar cost in Berlin = € 50,267 × $1.3677/€? = $68,750 The cost of the car is almost the same in both cities based on the spot rate! LO: 2 Bloomcode: Analysis AASCB: Analytic IMA: Global Business AICPA: Industry/Global Perspective
21.6
Forward rate: Explain the relation between each pair of currencies. Spot Rate Forward Rate a. $1.655/ £ $1.6001/£ b. ¥104.45/$ ¥102.33/$ c. C$1.1121/$ C$1.0940/$
Solution: a.
b. c.
Since the forward rate indicates that fewer dollars will be required to buy a pound in the future, we say that the dollar is at a forward premium against the British pound. The yen is at a forward premium against the dollar as the forward quote indicates that the dollar can be purchased with fewer yen. The U.S. dollar is at a forward discount against the Canadian dollar as a dollar is expected to purchase fewer Canadian dollars in the future.
LO: 2 Bloomcode: Analysis AASCB: Analytic IMA: Global Business AICPA: Industry/Global Perspective
SM 1-927
21.7
Forward rate: If the spot rate was $1.0413/C$ and the 90-day forward rate was $1.0507/C$, how much more (in U.S. dollars) would you receive by selling C$ 1,000,000 at the forward rate than at the spot rate?
Solution: If C$1,000,000 was sold at the spot rate, dollar revenue received: C$1,000,000 × $1.0413/C$ = $1,041,300 If C$1,000,000 was sold at the 90 day forward rate, dollar revenue received: C$1,000,000 × $1.0507/C$ = $1,050,700 Additional revenue received by selling forward = $1,050,700 - $1,041,300 = $9,400 LO: 2 Bloomcode: Application AASCB: Analytic IMA: Global Business AICPA: Industry/Global Perspective 21.8
Forward rate: Crane, Inc., sold equipment to an Irish firm and will receive €1,319,405 in 30 days. If the company entered a forward contract to sell at the 30-day forward rate of $1.3012/€, what is the dollar revenue received?
Solution: Expected euro revenue = €1,319,405 30-day forward rate = $1.3012/€ Dollar revenue received = €1,319,405 × $1.3012/€ = $1,716,809.79 LO: 2 Bloomcode: Application AASCB: Analytic IMA: Global Business AICPA: Industry/Global Perspective 21.9
Forward rate: Brilliant Equipment purchased machinery from a Japanese firm and must make a payment of ¥313.25 million in 45 days. The bank quotes a forward rate of ¥103.01/$ to buy the required yen. What is the cost to Brilliant in U.S. dollars?
Solution: Cost of machinery = ¥313.25 million Forward rate = ¥103.01/$ Cost of equipment in dollars = ¥313,250,000 / ¥103.01/$ = $3,040,966.9 LO :2
SM 1-928
Bloomcode: Application AASCB: Analytic IMA: Global Business AICPA: Industry/Global Perspective
21.10 Forward rate: Triumph Autos has contracted with an Indian software firm for design software. The payment of 22,779,750 rupees (₹) is due in 30 days. What is the cost in dollars if the 30-day forward rate is ₹64.39/$. Solution: Cost of software = ₹22,779,750 Forward rate= ₹64.39/$ Cost of software in dollars = ₹22,779,750 / ₹64.39/$= $353,777.76 LO :2 Bloomcode: Application AASCB: Analytic IMA: Global Business AICPA: Industry/Global Perspective
21.11 Forward rate: Use the data in Exhibit 21.5 to answer these questions: a. What is the six-month forward rate (in U.S. dollars) for Swiss francs? Is the Swiss franc selling for a premium or a discount? b. What is the six-month forward rate (in U.S. dollars) for the Japanese yen? Is the Japanese yen selling for a premium or a discount? c. Given the information above, what do you think will happen to the value of the U.S. dollar relative to the Swiss franc and the Japanese yen? Solution: a. The six month forward rate on the Swiss Francs is $1.1201/SF relative to the spot rate of $1.1182/SF. The forward premium (discount) = (1.1201 – 1.1182)/1.1182 = 0.17%. So there is a forward premium on the Swiss franc. b. The six month forward rate on the Japanese yen is $0.00979/¥, while the spot rate is $0.00977/¥. The forward premium (discount) = (0.00979 – 0.00977)/0.00977= 0.20%. The yen is a forward premium against the U. S. dollar. c. Given the data on forward rates in a.) and b.) above, we can expect the Swiss franc and the Japanese yen to appreciate relative to the U.S. dollar. LO: 2 Bloomcode: Analysis
SM 1-929
AASCB: Analytic IMA: Global Business AICPA: Industry/Global Perspective
21.12 Bid-ask spread: Nova Scotia Bank offers quotes on the Canadian dollar as shown below. What is the bid-ask spread based on these quotes? Bid Ask C$ 0.9973/$ C$ 0.9978/$ Solution: Ask rate - Bid rate 0.9978 - 0.9973 = Ask rate 0.9978 = 0.0005 = 0.05%
Bid - ask spread =
LO: 2 Bloomcode: Application AASCB: Analytic IMA: Global Business AICPA: Industry/Global Perspective
21.13 Bid-ask spread: A local community bank has requested foreign exchange quotes for the Swiss Franc from Citibank. Citibank quotes a bid rate of $1.0934/SF and an ask rate of $1.0997/SF. What is the bid-ask spread? Solution:
Ask rate - Bid rate 1.0997 -1.0934 = Ask rate 1.0997 = 0.000573 = 0.573%
Bid - ask spread =
LO: 2 Bloomcode: Application AASCB: Analytic IMA: Global Business AICPA: Industry/Global Perspective 21.14 Bid-ask spread: A foreign exchange dealer is willing to buy the Danish krone (DKr) at $0.1556/DKr and will sell it at a rate of $0.1563/DKr. What is the bid-ask spread on the Danish krone?
SM 1-930
Solution: Ask rate - Bid rate 0.1563 - 0.1556 = Ask rate 0.1563 = 0.00448 = 0.45%
Bid - ask spread =
LO: 2 Bloomcode: Application AASCB: Analytic IMA: Global Business AICPA: Industry/Global Perspective
21.15 Cross rate: Given the following quotes, calculate the €/£ cross rate. Bank of America $1.663/£ JP Morgan Chase $1.3914/€ Solution: To find the €/£ cross rate, divide the Bank of America quote by the JP Morgan Chase quote. Cross rate = $1.663/£ / $1.3914/€ = €1.1952/£ LO: 2 Bloomcode: Application AASCB: Analytic IMA: Global Business AICPA: Industry/Global Perspective
21.16 Cross rate: Barclays Bank of London has offered the following exchange rate quotes: ¥134.64/£ and Korean won 13.8374/¥. What is the cross rate between the Korean won and the British pound? Solution: To find the won/£ cross rate, multiply the Korean won quote by the yen quote. Cross rate = Won 13.8374/¥ × ¥ 134.64/£ = Won 1,863.07 /£ LO: 2 Bloomcode: Application AASCB: Analytic IMA: Global Business AICPA: Industry/Global Perspective
SM 1-931
21.17 Cross rate: Bremer Corporation observes that the Swiss franc (SF) is being quoted at €0.7660/SF, while the Swedish krona (SK) is quoted at €0.1114/SK. What is the SK/SF cross rate? Solution: To find the SK/SF cross rate, divide the Swiss franc quote by the Swedish krona quote. Cross rate = €0.7660/SF / €0.1114/SK = SK6.8761/SF LO: 2 Bloomcode: Application AASCB: Analytic IMA: Global Business AICPA: Industry/Global Perspective 21.18 Country risk: Ford Motor Company maintains production facilities in many different countries including Brazil, Taiwan, and the United States. Given the data in Exhibit 21.6, which production plant is likely to face the greatest country risk? How does country risk affect a firm’s capital budgeting decisions? Solution: Exhibit 21.6 contains composite country risk ratings as per 2014 survey. Using these ratings, Brazil has the greatest country risk while Taiwan has the least. Country risk factors are associated with additional systematic risk for projects located in that country and therefore are associated with a relatively higher discount rate. LO: 3 Bloomcode: Application AASCB: Analytic IMA: Global Business AICPA: Industry/Global Perspective
21.19 Foreign exchange risk: How is transaction exposure different from operating exposure? Solution: Transaction exposure is related to foreign exchange risk faced by firms that are expecting revenues in foreign currency or have expenses in foreign currency that relate to transactions they have already entered into. As the exchange rate changes, the home currency value of these revenues or expenses change, leading to transaction exposure. If exchange rate changes are more permanent in nature and modify the way a firm does its business, then we say that a firm is facing operating exposure.
SM 1-932
LO: 3 Bloomcode: Analysis AASCB: Analytic IMA: Global Business AICPA: Industry/Global Perspective 21.20 International debt: What are Yankee bonds?
Solution: When corporate bonds are issued by a foreign firm in the United States and denominated in U.S. dollars, they are considered to be foreign bonds and referred to as ―Yankee‖ bonds. LO :4 Bloomcode: Knowledge AASCB: Analytic IMA: Global Business AICPA: Industry/Global Perspective INTERMEDIATE 21.21 Forward premium: The spot rate on the London market was £0.5514/$, while the 90day forward rate is £0.5589/$. What is the annualized forward premium or discount on the British pound? Solution: Forward discount (premium)
Forward rate - Spot rate
360
100
Spot rate n 0.5589 0.5514 360 5.44% 0.5514 90 The British pound is at a forward discount against the U.S. dollar as it takes more pounds to buy a dollar at the forward rate. LO: 2 Bloomcode: Application AASCB: Analytic IMA: Global Business AICPA: Industry/Global Perspective
SM 1-933
21.22 Forward premium: Bank of America quoted the 180-day forward rate on the Swiss franc at $1.0407/SF. The spot rate was quoted at $1.0268/SF. What is the forward premium or discount on the Swiss franc? Solution: Forward rate - Spot rate 360 100 Spot rate n $1.0407 $1.0268 360 2.71% $1.0268 180 The Swiss franc is at a 2.71 percent forward premium against the dollar. Forward premium (discount)
LO: 2 Bloomcode: Application AASCB: Analytic IMA: Global Business AICPA: Industry/Global Perspective
21.23 Forward premium: The foreign exchange department at Tokyo’s Daiwa Bank quoted the spot rate on the euro at €0.007269/¥. The 90-day forward rate is quoted at a premium of 5.42 percent on the euro. What is the 90-day forward rate? Solution: Spot rate = €0.007269/¥ 90-day forward premium on the euro = 5.42% It’s mean that 90-day forward discount on the Yen= –5.42% -0.054 = F - 0.00727 x 360 0.007269 90 F 0.007269 = -0.0542 X 0.007269 X 90 360 = €0.007171/¥
The 90-day forward rate is €0.007171/¥.
LO: 2 Bloomcode: Application AASCB: Analytic IMA: Global Business AICPA: Industry/Global Perspective
SM 1-934
21.24 Forward premium: The spot rate of the Australian dollar (A$) is A$1.1667/$. The Australian dollar is quoted at a 30-day forward premium of 4.90 percent against the U.S. dollar. What is the 30-day forward quote? Solution: Spot rate = A$1.1667/$ 30-day forward premium on the Australian dollar = 4.90%Its mean that 90-day forward discount on the dollar= –4.90% Forward rate - Spot rate 360 100 Spot rate n F 1.1667 360 0.049 0 1.1667 30 0.049 1.1667 30 F0 1.1667 360 F0 0.004764 1.1667
Forward premium (discount)
A$1.1619/ $ The 30-day forward rate is A$1.1619/$. LO: 2 Bloomcode: Application AASCB: Analytic IMA: Global Business AICPA: Industry/Global Perspective
21.25 Bid-ask spread: The foreign exchange department of Bank of America has a bid quote on Canadian dollars (C$) of C$1.0800/$. If the bank typically tries to make a bid-ask spread of 0.5 percent on these foreign exchange transactions, what will the ask rate have to be? Solution: Bid-ask spread = Ask rate - Bid rate Ask rate 0.005 = Ask rate -1.08 Ask rate Ask rate - 0.005 x Ask rate = 1.08 Ask rate = 1.08 = 1.09 0.995
SM 1-935
Ask rate - Bid rate Ask rate Ask Rs. 43.21 / $ 0..005 = Ask Ask 43.21 = 0.005 × Ask
Bid - ask spread =
Ask - 0.005 Ask = 43.21 43.21 Ask rate = = Rs.43.43 / $ 0.995 The ask rate will have to be C$1.09/$ to provide a 0.5 percent bid-ask spread. LO: 2 Bloomcode: Application AASCB: Analytic IMA: Global Business AICPA: Industry/Global Perspective
21.26 Bid-ask spread: Banco Santiago wants to make a bid-ask spread of 0.65 percent on its foreign exchange transactions. If the ask rate on the Mexican peso (MP) is MP10.3092/$, what does the bid rate have to be? Solution: Bid -ask spread = Ask rate - Bid rate Ask rate 0.0065 = MP10.3092 - Bid rate MP10.3092 MP10.3092 Bid rate = 0.0065 × MP10.3092 = 0.067 Bid rate = MP10.3092 -0.067 = MP10.2422/$
Ask rate - Bid rate Ask rate 10.3092 - Bid rate 0..0065 = 10.3092 10.3092 - Bid rate = 0.0065 ×10.3092 Bid - ask spread =
= 0.0067 Bid rate = 10.3092 - 0.0067 = MP10.2422 / $ The ask rate will have to be MP10.2422/$ to provide a 0.65 percent bid-ask spread. LO: 2 Bloomcode: Application AASCB: Analytic
SM 1-936
IMA: Global Business AICPA: Industry/Global Perspective
21.27 Cross rate: Alcor Pharma just received revenues of $3,165,300 in Australian dollars (A$). Management has the following exchange rates:A$1.08010/£ and $1.5906/£. What is the U.S. dollar value of the company’s revenues? Solution: Revenues received by Alcor Pharma = A$3,165,300 To determine the A$ /$ cross rate, you divide the A$ to £ quote by the $/£ quote. Cross rate = A$1.08010/£ / $1.5906/£ = A$0.6796/$ U.S. dollar value of their revenue = A$3,165,300 / A$0.6796/$ = $4,657,593 LO: 2 Bloomcode: Application AASCB: Analytic IMA: Global Business AICPA: Industry/Global Perspective
21.28 Cross rate: Flint Corp. recently purchased auto parts worth 17.5 million Mexican pesos (MP) on credit. Management needs to find out the U.S. dollar cost of the purchase. It has access to two quotes for Canadian dollars (C$): C$1.0174/$ and C$0.0820/MP. What did it cost Flint to purchase the auto parts? Solution: Foreign currency payables = MP 17.5 million To find the MP/$ quote, divide the C$/MP quote by the C$/$ quote. Cross rate = C$0.0820/MP / C$1.0174/$ = $0.08060/MP Dollar cost of payables = MP 17,500,000 × $0.08060/MP = $1,410,458 LO: 2 Bloomcode: Application AASCB: Analytic IMA: Global Business AICPA: Industry/Global Perspective
SM 1-937
21.29 Hedging: Tricolor Industries has purchased equipment from a Brazilian firm for a total cost of 272,500 Brazilian reals. The firm has to pay in 30 days. Citibank has given the firm a 30-day forward quote of $0.4723/real. Assume that on the day the payment is due, the spot rate is at $0.4917/real. How much would Tricolor save by hedging with a forward contract? Solution: Foreign currency payables = BR 272,500 Spot rate on payment date = $0.4917/real Dollar cost of payables = BR 272,500 × $0.4917/real = $133,988.25 30-day forward rate = $0.4723/real Cost if hedged at forward rate = BR 272,500 × $0.4723/real = $128,701.75 Cost savings = $133,988.25 − $128,701.75 = $5,286.50 LO :2 Bloomcode: Application AASCB: Analytic IMA: Global Business AICPA: Industry/Global Perspective
21.30 Eurocredit Loan: A Swiss sporting goods company borrows in yen in the Eurocredit market at a rate of 4.35 percent from Bank of America using a three-month rollover loan. Bank of America assigns a default risk premium of 2 percent on the loan, and the country risk is an additional 0.75 percent. The Bank can borrow funds in the Euromarket at the three-month LIBOR rate of 0.40 percent. What is Bank of America’s gross profit margin on this loan? Solution: GPMAR = k - BR – DRP – CR = 4.35% - 0.40% - 2.0% - 0.75% = 1.2% LO: 5 Bloomcode: Application AASCB: Analytic IMA: Global Business AICPA: Industry/Global Perspective
ADVANCED 21.31 Covington Industries just sold equipment to a Mexican firm. Payment of 11,315,000 pesos will be due to Covington in 30 days. Covington has the option of selling the pesos today at a 30-day forward rate of $0.09139/peso. If it waits 30 days to sell the pesos, the
SM 1-938
expected spot rate is $0.0907/peso. In dollars, how much better off is Covington by selling the pesos in the forward market? Solution: Foreign currency receivables = MP 11,315,000 Spot rate on payment date = $0.0907/peso Dollar revenues from receivables = MP 11,315,000 × $0.0907/peso= $1,026,270.50 30-day forward rate = $0.09139/peso Revenues if hedged at forward rate = MP 11,315,000 × $0.09139/peso= $1,034,077.85 Additional revenues received = $1,034,077.85 − $1,026,270.50 = $7,807.35 LO: 2 Bloomcode: Application AASCB: Analytic IMA: Global Business AICPA: Industry/Global Perspective
21.32 Barrington Fertilizers, Inc., exports its specialized lawn care products to Canada. It made a sale worth C$1,150,000, with the payment due in 90 days. Barrington’s banker gave it a forward quote of $0.9021/C$. By using the forward rate, the firm gained an additional $8,433.25 over what it would have gotten if it had sold the Canadian dollars in the spot market 90 days later. What was the spot rate at the time the payment was received? Solution: Foreign currency receivables = C$1,150,000 90-day forward rate = $0.9021/C$ Revenues if hedged at forward rate = C$1,150,000 × $0.9021/C$= $1,037,415 Additional dollar revenues received = $8,433.25 Revenues at spot rate = $1,037,415− $8,433.25 = $1,028,981.75 Spot rate on payment date = $1,028,981.75/ C$1,150,000 = $0.8948/C$ LO: 2 Bloomcode: Application AASCB: Analytic IMA: Global Business AICPA: Industry/Global Perspective
21.33 Moon Rhee Auto Supply, a Korean supplier of parts to Kia Motors, is evaluating an opportunity to set up a plant in Alabama, where Kia Motors has an auto assembly plant for its SUVs. The cost of this plant will be $13.5 million. The current spot rate is
SM 1-939
1,120.318 Korean won per U.S. dollar. The firm is expected to use this plant for the next five years and is expecting to generate the following cash flows. Years Cash flows ($ millions) Expected exchange rate (Korean won/$)
1
2
3
4
5
$2.3
$4.2
$3.6
$5.8
$7.6
1,105.231
1,115.632
1,146.155
1,120.221
1,110.670
The firm uses a discount rate of 9 percent for projects like this in the United States. What is the NPV of this project? Should Moon Rhee Auto Supply take on this project? Solution: Moon Rhee Auto Supply knows what the expected cash flows in U. S. dollars and the expected cash flows to the parent firm in Korean won can be calculated by dividing the dollar cash flows by the appropriate exchange rate. Year Cash flows ($ millions)
1
2
3
4
5
2.3
4.2
3.6
5.8
7.6
Given in the following table are the cash flows that the Korean firm expects to receive from its American subsidiary in U.S. dollars.
Year 0 1 2 3 4 5
Cash Flows (in millions of U.S. $) (13.5) 2.3 4.2 3.6 5.8 7.6
Exchange Rate (won/$) 1,120.318 1,105.231 1,115.632 1,146.155 1,120.221 1,110.670
Cash Flows (in millions of won) (15,124.293) 2,542.031 4,685.654 4,126.158 6,497.282 8,441.092
The firm has determined that the appropriate discount rate is 9 percent for capital budgeting projects in the United States. By discounting the cash flow at the risk-adjusted discount rate of 9 percent, we can compute the NPV for this project. NPV = −15,124.293 + 2,542.031/(1.09)1 + 4,685.654/(1.09)2 + 4,126.158/(1.09)3 +6,497.282/(1.09)4 + 8,441.092/(1.09)5 million won
SM 1-940
=-15124.293 + 2,332.13881+ 3,943.82156 + 3,186.15104 + 4,602.83823 + 5,486.13063 million won= 4,426.787million won Since NPV is positive, the project should be accepted. LO: 3 Bloomcode: Analysis AASCB: Analytic IMA: Global Business AICPA: Industry/Global Perspective
21.34 The Boeing Company has two different debt issues, both maturing four years from now. The domestic bond issue pays semiannual coupons and has a coupon rate of 4.80 percent. The current price on the bond is $962.75. The Eurobond issue is priced at $964.33 and pays an annual coupon of 4.95 percent. What is the yield to maturity for each bond? Solution: For the Euro bond with annual compounding, the coupon payment is $49.50 per year, and the yield calculation is: $49.50 $49.50 $49.50 $1, 049.50 $964.33 (1 i)1 (1 i)2 (1 i)3 (1 i)4 Using our financial calculator, the annual yield of the Eurodollar bond issue is 5.9789 percent. For the domestic bond issue the semiannual coupon payments are $24 ($48/2), and the semiannual bond yield calculation is: $1,024 $24 $24 $962.75 2 1 (1 i)8 (1 i) (1 i) Making the calculation, the bond issue’s semiannual yield is 2.9291 percent. As discussed in Chapter 8, to compare interest rates with different compounding periods, we need to compute the effective annual yield (EAY). Applying the equation EAY: EAY = (1 + quoted rate/m)m – 1 = (1 + 0.029291)2 – 1 = 1.05944 – 1 = 5.94% An investor in Boeing’s Eurobond will earn 5.98 percent, while the domestic bond would provide a yield of 5.94 percent. Thus, the Eurobond is the better investment. LO: 4 Bloomcode: Analysis AASCB: Analytic IMA: Global Business AICPA: Industry/Global Perspective
SM 1-941
21.35 Caterpillar, Inc. management is trying to decide between selling a new bond issue in the U.S. or the Eurodollar bond market. In either market the bonds will be denominated in dollars and will have a three-year maturity. The domestic bond will have a coupon rate of 4.1 percent, paid semiannually and would sell at a market price of $1,034.25. The Eurobonds will have a coupon rate of 4 percent, paid annually, and will sell at $1,029.76. Which bond issue will have the lowest cost to the firm? Solution: For the domestic bond issue the semiannual coupon payments are $20.50 ($41/2), and the semiannual bond yield calculation is: $20.50 $20.50 $1, 020.50 $1, 034.25 1 2 (1 i) (1 i) (1 i)6
Making the calculation, we see that the bond issue’s semiannual yield is 1.4499 percent. As discussed in Chapter 8, to compare interest rates with different compounding periods, we need to compute the effective annual yield (EAY). Applying the equation EAY: EAY = (1 + quoted rate/m)m – 1 = (1 + 0.014499)2 – 1 = 1.02921 – 1 = 2.92% For the Eurodollar bond with annual compounding, the coupon payment is $40 per year, and the yield calculation is: $40 $40 $1, 040 $1, 029.76 2 1 (1 i) (1 i) (1 i)3 Using our financial calculator, the annual yield of the Eurodollar bond issue yield is 2.9489 percent. From Caterpillar’s perspective, the domestic bond issue is the cheaper alternative. LO: 4 Bloomcode: Analysis AASCB: Analytic IMA: Global Business AICPA: Industry/Global Perspective
21.36 IBM’s German unit is looking to borrow €7.5 million from Deutsche Bank. Deutsche Bank quotes a rate of three-month LIBOR plus 0.25 percent for the 90-day loan. Currently, the three-month LIBOR is 3.875 percent. What is IBM’s interest cost on the
SM 1-942
loan in Euros? If the exchange rate on the payoff date is €0.8164/$, what is the dollar cost of the loan? Solution: Amount IBM plans to borrow = €7.5 million Term of loan = 90 days Interest cost = LIBOR + 0.25% = 3.875% + 0.25% = 4.125% Interest cost in euros = €7,500,000 x 0.04125 × (90/360) = €77,343.75 Spot rate on payoff date = €0.8164/$ Dollar interest cost = €77,343.75 / €0.8164/$ = $94,737.57 LO: 5 Bloomcode: Application AASCB: Analytic IMA: Global Business AICPA: Industry/Global Perspective
21.37 Toyota is interested in borrowing $5 million for 90 days. Bank of America has quoted a rate that is 1.125 percent under the prime rate of 6.25 percent. Daiwa Bank is offering Toyota a rate that is 0.75 percent over the three-month LIBOR of 4.2 percent. Which is the better deal for Toyota, and what is the lower interest cost in dollars? Solution: Bank of America Loan Amount Toyota plans to borrow = $5 million Term of loan = 90 days Interest cost = Prime rate − 1.125% = 6.25% − 1.125% = 5.125% Interest cost = $5,000,000 × 0.05125 × (90/360) = $64,062.50 Daiwa Bank Loan Amount Toyota plans to borrow = $5 million Term of loan = 90 days Interest cost = LIBOR + 0.75% = 4.2% + 0.75% = 4.95% Interest cost = $5,000,000 × 0.0495 × (90/360) = $61,875 The Daiwa Bank offers Toyota the lower cost loan with a lower interest cost of $61,875 versus $64,062.50. LO: 5 Bloomcode: Analysis AASCB: Analytic IMA: Global Business AICPA: Industry/Global Perspective
SM 1-943
Sample Test Problems 21.1
What are six factors that cause international transactions to differ from domestic transactions? Solution: Six factors that cause international transactions to differ from domestic transactions are differences in: 1) Currencies 2) Legal systems and tax codes 3) Languages 4) Cultures 5) Economic systems 6) Country risks
Bloomcode: Knowledge AASCB: Diversity IMA: Global Business AICPA: Industry/Global Perspective
21.2
If a Dell Studio laptop sells for $999 in Austin, Texas and £689 in London, what is the implied exchange rate between the U.S. dollar and the euro? Solution: Cost of the computer in Austin
= $999
Cost of the computer in London = £689 Dollar to euro exchange rate
= $999 / £689 = $1.4499/£
Bloomcode: Application AASCB: Analytic IMA: Global Business AICPA: Industry/Global Perspective
21.3
A bank in India has offered a spot rate quote on Indian rupees (Rs) of Rs62.2905/$. The Indian rupee is quoted at a 30-day forward premium of 5.22 percent against the dollar. What is the 30-day forward quote?
SM 1-944
Solution: Spot rate = Rs62.2905/$ 30-day forward premium on the dollar = 5.22% Forward rate Spot rate 360 Forward premium = × × 100 Spot rate n 0.0522 = F0 62.2905 =
F0 Rs62.2905/$ 360 × Rs62.2905/$ 30 0.0522 × Rs62.2905/$ × 30 360
F0 = 0.2710 + Rs62.2905/$ = Rs62.5615/$ The 30-day forward rate is Rs62.5615/$. Bloomcode: Application AASCB: Analytic IMA: Global Business AICPA: Industry/Global Perspective 21.4
Technocorp has purchased industrial parts from a German company for a total cost of €1,225,000. The firm has 30 days to pay. A bank has given Technocorp a 30-day forward quote of $1.355/€. Assume that on the day the payment is due, the spot rate is $1.368/€. How much would Technocorp have saved by hedging with a forward contract? Solution: Foreign currency payable = €1,225,000 Spot exchange rate on payment date = $1.368/€. Dollar cost of payables = €1,225,000 × $1.368/€ = $1,675,800 30-day forward rate = $1.355/€ Cost if hedged at forward rate = €1,225,000 × $1.355/€ = $1,659,875 Cost savings = $1,675,800 − $1,659,875 = $15,925
Bloomcode: Application AASCB: Analytic IMA: Global Business AICPA: Industry/Global Perspective
SM 1-945
21.5
Tass Co., Ltd, a Japanese electrical parts producer, is considering building a plant in the U.S. The cost of this plant will be $20 million and the current spot exchange rate between the yen and the U.S. dollar is ¥101.8/$. Tass management expects to use this plant for the next five years and expects it to generate the following cash flows during this period. Year Cash flows ($ millions) Expected exchange rate (¥/$)
1 $2.0 ¥101.5/$
2 $3.6 ¥100.4/$
3 $5.0 ¥98.6/$
4 $6.8 ¥95.9/$
5 $8.0 ¥92.5/$
If Tass uses a discount rate of 8 percent for projects in the United States, what is the NPV of this project? Should Tass Company take on this project? Solution: Expected cash flows to Tass in yen can be calculated by multiplying the dollar cash flows by the appropriate exchange rate.
Year 0 1 2 3 4 5
Cash Flows (U.S. $ in millions) ($20) $2.0 $3.6 $5.0 $6.8 $8.0
Exchange Rate (¥/$) ¥101.8/$ ¥101.5/$ ¥100.4/$ ¥98.6/$ ¥95.9/$ ¥92.5/$
Cash Flows (in millions of ¥) (¥2,036) ¥203 ¥361.44 ¥493 ¥652.12 ¥740
By discounting the cash flow at the risk-adjusted discount rate of 8 percent, we can compute the NPV for this project. NPV = −2,036 + 203 / 1.08 + 361.44 / (1.08)2 + 493 / (1.08)3 +652.12 / (1.08)4 + 740 / (1.08)5 = −2,036 + 187.96 + 309.88 + 391.36 + 479.33 + 503.63 = −¥163.84 million The NPV is negative, so the project should be rejected. Bloomcode: Analysis AASCB: Analytic IMA: Global Business AICPA: Industry/Global Perspective
SM 1-946